You are on page 1of 413

t.me/Ebooks_Encyclopedia27. t.

me/Magazines4all

visit My Collection -
https://rebrand.ly/DownloadMore
t.me/Ebooks_Encyclopedia27. t.me/Magazines4all

The secret of KOTA


now at your Doorstep

PHYSICS
DPP BOOKLET
60 + 28
Topic-wise Chapter-wise Improves
Tests for Concept your learning
Checking & by at least
Speed Building
20%
Ü Collection of 3100 + MCQs of all variety of questions
Ü Unique & innovative way of learning
Ü Detailed solutions to Topic-wise & Chapter-wise practice sheets
Ü Covers all important concepts of each topic
Ü As per latest pattern & syllabus
t.me/Ebooks_Encyclopedia27. t.me/Magazines4all

EBD_7156
• Corporate Office : 45, 2nd Floor, Maharishi Dayanand Marg, Corner Market,
Malviya Nagar, New Delhi-110017
Tel. : 011-49842349 / 49842350

Typeset by Disha DTP Team

DISHA PUBLICATION
ALL RIGHTS RESERVED

© Copyright Publisher
No part of this publication may be reproduced in any form without prior permission of the publisher. The author and the
publisher do not take any legal responsibility for any errors or misrepresentations that might have crept in. We have tried
and made our best efforts to provide accurate up-to-date information in this book.

For further information about books from DISHA,


Log on to www.dishapublication.com or email to info@dishapublication.com
t.me/Ebooks_Encyclopedia27. t.me/Magazines4all

Daily Practice Problem (DPP) Sheets PHYSICS for


NEET/AIIMS/JIPMER (Kota's formula to Success)
PREPARE ASSESS IMPROVE

Assessment is the most integral part of a student’s preparation but still most of them avoid it. Only assessment can tell
where you stand and how you can improve from that point. So it is very important that you take the right assessment, which
is on the correct pattern, has the same level of difficulty as the actual exam and covers all the important concepts of the
subject.
Disha Publication launches a first of its kind product which changed the way coaching was conducted in KOTA - the hub
of Engineering and Medical Entrance education in India.
The book “Daily Practice Problem (DPP) Sheets for NEET/AIIMS” is precise, apt and tuned to all the requirements of a
NEET/AIIMS aspirant.

KEY DIFFERENTIATING FEATURES OF THE DPP SHEETS

• Part A provides 60 DPP's with division of the complete NEET syllabus of Physics into 60 most important Topics.
Each of the chapter has been broken into 2 or more topics.
• Part B consist of — Chapter-wise tests based on NCERT and NEET syllabus.
• Time Limit and Maximum Marks have been provided for each DPP Sheet/ topic. You must attempt each Sheet in
test like conditions following the time limits. Further to achieve perfect preparation in a topic or chapter one has
to score atleast 135 marks.
• Ultimate tool for Concept Checking & Speed Building.

• Collection of 3100 Standardised MCQ’s of all variety of NEW pattern questions – MCQ only one correct option
and Assertion-Reason.

• Unique & innovative way of learning. Whenever you have prepared a topic(Part A) or a chapter (Part B) just
attempt that worksheet.
• Do not refer the Solution Booklet until and unless you have made all the efforts to solve the DPP Sheets.
• Covers all important Concepts of each Topic in the form of different Questions in the DPP Sheets.

• As per latest pattern & syllabus of NEET/AIIMS JIPMER exam.


• Compliant to all boards of education.

No matter where you PREPARE from – a coaching or NCERT books or any other textbook/ Guide - Daily Practice Problem
Sheets provides you the right ASSESSMENT on each topic. Your performance provides you the right clues to IMPROVE
your concepts so as to perform better in the final examination.
It is to be noted here that these are not tests but act as a checklist of student’s learning and ability to apply concepts to
different problems. Do proper analysis after you attempt each DPP sheet and try to locate your weak areas.
It is our strong belief that if an aspirant works hard on the clues provided through each of the DPP sheets he/ she can
improve his/ her learning and finally the SCORE by at least 20%.

Χisha ∆xperts
t.me/Ebooks_Encyclopedia27. t.me/Magazines4all

EBD_7156
The book comprises of following two parts
Part A : Topic-wise DPP Sheets Page No.
Detailed Index (i) to (iv)
Topic-wise Sheets 1-60 (Each sheet 4 pages) 1 - 4
Solutions of Topic-wise Sheets 1 - 158

Part B : Chapter-wise DPP Sheets


Detailed Index (a) to (c)
Chapter-wise Sheets 1-28 p-1 – p-112
Solutions of Chapter-wise Sheets S-1– S-115
t.me/Ebooks_Encyclopedia27. t.me/Magazines4all

https://rebrand.ly/DownloadMore

TOPIC-WISE
DPP SHEETS
WITH SOLUTIONS

[i]
t.me/Ebooks_Encyclopedia27. t.me/Magazines4all

EBD_7156
INDEX/SYLLABUS
DPP-1Page No.

DPP-1 PHYSICAL WORLD, UNITS & DIMENSIONS P1 - P 4

DPP-2 MEASUREMENTS (ERRORS) P1 - P 4

DPP-3 MOTION IN A STRAIGHT LINE 1 (Distance, Displacement, Uniform & Non-uniform motion) P1 - P 4

DPP-4 MOTION IN A STRAIGHT LINE 2 (Relative Motion & Motion Under Gravity) P1 - P 4

DPP-5 VECTORS P1 - P 4

DPP-6 MOTION IN A PLANE-1 (Projectile Motion) P1 - P 4

DPP-7 MOTION IN A PLANE-2 (Horizontal Circular Motion) P1 - P 4

DPP-8 MOTION IN A PLANE-3 (Vertical Circular Motion, Relative Motion) P1 - P 4

DPP-9 LAWS OF MOTION-1 (Newton's laws, momentum, pseudo force concept) P1 - P 4

DPP-10 LAWS OF MOTION-2 (Blocks in contact, connected by string, pulley arrangement) P1 - P 4

DPP-11 LAWS OF MOTION-3 (Friction) P1 - P 4

DPP-12 WORK, ENERGY AND POWER-1 (Work by constant and variable forces, kinetic and potential energy,
work energy theorem) P1 - P 4

DPP-13 WORK, ENERGY AND POWER-2 (Conservation of momentum and energy, collision, rocket case) P1 - P 4

DPP-14 CENTRE OF MASS AND ITS MOTION P1 - P 4

DPP-15 ROTATIONAL MOTION – 1: Basic concepts of rotational motion, moment of a force, torque, angular
momentum and its conservation with application P1 - P 4

DPP-16 ROTATIONAL MOTION-2 : Moment of inertia, radius of gyration, (values of moments of inertia simple
geometrical objects) P1 - P 4

DPP-17 ROTATIONAL MOTION - 3 : Rolling Motion, Parallel and perpendicular theorems and their
applications, Rigid body rotation, equations of rotational motion P1 - P 4

DPP-18 GRAVITATION - 1 (The Universal law of gravitation, Acceleration due to gravity and its variation with
altitude and depth, Kepler's law of planetary motion) P1 - P 4

DPP-19 GRAVITATION - 2 (Gravitational potential energy, Gravitational potential, Escape velocity & Orbital
velocity of a satellite, Geo-stationary satellites) P1 - P 4

DPP-20 MECHANICAL PROPERTIES OF SOLIDS P1 - P 4

DPP-21 FLUID MECHANICS P1 - P 4

[ii]
t.me/Ebooks_Encyclopedia27. t.me/Magazines4all

DPP-22 THERMAL EXPANSION, CALORIMETRY AND CHANGE OF STATE P1 - P 4

DPP-23 HEAT TRANSFER & NEWTON’S LAW OF COOLING P1 - P 4

DPP-24 THERMODYNAMICS-1 (Thermal equilibrium, zeroth law of thermodynamics, concept of temperature,


Heat, work and internal energy, Different thermodynamic processes) P1 - P 4

DPP-25 THERMODYNAMICS-2 (1st and 2nd laws of thermodynamics, Reversible & irreversible processes,
Carnot engine and its efficiency) P1 - P 4

DPP-26 KINETIC THEORY P1 - P 4

DPP-27 OSCILLATIONS-1 (Periodic motion - period, Frequency, Displacement as a function of time. Periodic
functions, Simple harmonic motion and its equation, Energy in S.H.M. - kinetic and potential energies)
P1 - P 4

DPP-28 OSCILLATIONS-2 (Oscillations of a spring, simple pendulum, free, forced and damped oscillations,
Resonance) P1 - P 4

DPP-29 WAVES-1 (Wave motion, longitudinal and transverse waves, speed of a wave, displacement relation for a
progressive wave, principle of superposition of waves, reflection of waves) P1 - P 4

DPP-30 WAVES-2 (Standing waves in strings and organ pipes, Fundamental mode and harmonics, Beats, Doppler
effect in sound) P1 - P 4

DPP-31 PRACTICAL PHYSICS - 1 P1 - P 4

DPP-32 ELECTROSTATICS-1 (Coulomb's law, electric field, field lines, Gauss's law) P1 - P 4

DPP-33 ELECTROSTATICS-2 (Electric potential and potential difference, equipotential surfaces, electric dipole)
P1 - P 4

DPP-34 ELECTROSTATICS -3 (Electrostatic Potential energy, conductors) P1 - P 4

DPP-35 ELECTROSTATICS-4 (Capacitors, dielectrics) P1 - P 4

DPP-36 CURRENT ELECTRICITY – 1 (Electric Current, drift velocity, Ohm's law, Electrical resistance, Resistances
of different materials, V-I characteristics of Ohm and non-ohmic conductors, electrical energy and power,
Electrical resistivity, Colour code of resistors, Temperature dependance of resistance) P1 - P 4

DPP-37 CURRENT ELECTRICITY – 2 Electrical cell and its internal resistance, Potential difference and E.M.F of a
cell, Combination of cells in series and in parallel, Kirchoff's laws and their applications, RC transient circuit,
Galvanometer, Ammeter, Voltmeter] P1 - P 4

DPP-38 CURRENT ELECTRICITY-3 : Wheatstone bridge, Meter bridge, Potentiometer-principle and its applications.
P1 - P 4

DPP-39 MAGNETIC EFFECTS OF CURRENT-1 (Magnetic field due to current carrying wires, Biot savart law)
P1 - P 4

DPP-40 MAGNETIC EFFECTS OF CURRENT-2 : (Motion of charge particle in a magnetic field, force between
current carrying wires.) P1 - P 4

[iii]
t.me/Ebooks_Encyclopedia27. t.me/Magazines4all

EBD_7156
DPP-41 MAGNETIC EFFECTS OF CURRENT-3 (Magnetic dipole, Current carrying loop in magnetic
field,Galvanometer ) P1 - P 4

DPP-42 MAGNETISM AND MATTER - 1 (Bar magnet as an equivalent solenoid, Magnetic field lines, Earth's
magnetic field and magnetic elements) P1 - P 4

DPP-43 MAGNETISM & MATTER-2 (Para, dia and ferro-magnetic substances, magnetic susceptibility and
permeability, Hysteresis, Electromagnets and permanent magnets.) P1 - P 4

DPP-44 ELECTROMAGNETIC INDUCTION-1 (Magnetic flux, Faraday's law of electromagnetic induction,


Lenz's law, motional e.m.f.) P1 - P 4

DPP-45 ELECTROMAGNETIC INDUCTION - 2 : Self inductance, mutual inductance, Growth and decay of
current in L.R. circuit, Transformer, Electric motor, Generator P1 - P 4

DPP-46 ALTERNATING CURRENT - 1 (Alternating currents, peak and rms value of alternating current/voltage;
reactance and impedance, Pure circuits, LR, CR ac circuits.) P1 - P 4

DPP-47 ALTERNATING CURRENT - 2 (LCR series circuit, resonance, quality factor, power in AC circuits, wattless
and power current) P1 - P 4

DPP-48 EM WAVES P1 - P 4

DPP-49 RAY OPTICS-1 (Reflection on plane mirrors and curved mirrors) P1 - P 4

DPP-50 RAY OPTICS - II (Refraction on plane surface, total internal reflection, prism) P1 - P 4

DPP-51 RAY OPTICS - 3 (Refraction on curved surface lens, Optical instrument) P1 - P 4

DPP-52 WAVE OPTICS - I (Interference of Light) P1 - P 4

DPP-53 WAVE OPTICS - II (Diffraction and polarisation of light) P1 - P 4

DPP-54 DUAL NATURE OF MATTER & RADIATION (Matter Waves, Photon, Photoelectric effect, X-ray) P1 - P 4

DPP-55 ATOMS P1 - P 4

DPP-56 NUCLEI P1 - P 4

DPP-57 SEMICONDUCTOR ELECTRONICS - 1 (Semiconductors, LED, Photodiode, Zener diode) P1 - P 4

DPP-58 SEMICONDUCTOR ELECTRONICS-2 (Junction transistor, transistor action, characteristics of a transistor,


transistor as an amplifier, logic gates) P1 - P 4

DPP-59 COMMUNICATION SYSTEMS, LASER P1 - P 4

DPP-60 PRACTICAL PHYSICS - 2 P1 - P 4

Solutions to Topic-wise DPP Sheets (1-60) 1-158

[iv]
t.me/Ebooks_Encyclopedia27. t.me/Magazines4all

DPP - Daily Practice Problems


Name : Date :

Start Time : End Time :

01
SYLLABUS : Physical World, Units & Dimensions

Max. Marks : 120 Time : 60 min.


GENERAL INSTRUCTIONS
• The Daily Practice Problem Sheet contains 30 MCQ's. For each question only one option is correct. Darken the correct
circle/ bubble in the Response Grid provided on each page.
• You have to evaluate your Response Grids yourself with the help of solution booklet.
• Each correct answer will get you 4 marks and 1 mark shall be deduced for each incorrect answer. No mark will be given/
deducted if no bubble is filled. Keep a timer in front of you and stop immediately at the end of 60 min.
• The sheet follows a particular syllabus. Do not attempt the sheet before you have completed your preparation for that
syllabus. Refer syllabus sheet in the starting of the book for the syllabus of all the DPP sheets.
• After completing the sheet check your answers with the solution booklet and complete the Result Grid. Finally spend time
to analyse your performance and revise the areas which emerge out as weak in your evaluation.

DIRECTIONS (Q.1-Q.21) : There are 21 multiple choice and n2 are number of particles per unit volume in the position
questions. Each question has 4 choices (a), (b), (c) and (d), out x1 and x2. Find dimensions of D called as diffusion constant.
of which ONLY ONE choice is correct. (a) [M 0 L T 2] (b) [M 0 L2 T –4]
0
(c) [M L T ] –3 (d) [M 0 L2 T –1]
Q.1 If L, C and R represent inductance, capacitance and
2
Q.3 X = 3YZ find dimensions of Y in (MKSA) system, if X and
resistance respectively, then which of the following does
not represent dimensions of frequency? Z are the dimensions of capacity and magnetic field
respectively
1 R
(a) (b) (a) [M –3L –2T – 4A –1] (b) [ML– 2]
RC L –3 –2 4 4
(c) [M L T A ] (d) [M –3L– 2T 8A 4]
1 C aZ
(c) (d) a - kq
LC L Q.4 In the relation P = , P is pressure, Z is the distance,
e
Q.2 Number of particles crossing unit area perpendicular to b
k is Boltzmann constant and q is the temperature. The
n2 - n1
X-axis in unit time is given by n = - D , where n1 dimensional formula of b will be
x2 - x1 (a) [M 0L 2T 0] (b) [M 1L 2T 1]
1 0
(c) [M L T ] –1 (d) [M 0L 2T –1]

RESPONSE GRID 1. 2. 3. 4.
Space for Rough Work
t.me/Ebooks_Encyclopedia27. t.me/Magazines4all

EBD_7156
2 DPP/ P 01
1/ 2
Q.12 Wavelength of ray of light is 0.00006 m. It is equal to
P éF ù (a) 6 microns (b) 60 microns
Q.5 The frequency of vibration of string is given by n = ê ú .
2l ë m û (c) 600 microns (d) 0.6 microns
Here P is number of segments in the string and l is the Q.13 SI unit of permittivity is
length. The dimensional formula for m will be (a) C2 m2 N–2 (b) C–1 m 2N –2
(a) [M 0LT –1] (b) [ML 0T –1] (c) C2 m2 N2 (d) C2 m–2 N –1
(c) [ML T ] –1 0 (d) [M 0L 0T 0]
Q.6 What is the relationship between dyne and newton of force? 1
Q.14 The dimensions of e E2 (e0 = permittivity of free space
(a) 1 dyne = 10–5 newton (b) 1 dyne = 10–7 newton 2 0
(c) 1 dyne = 105 newton (d) 1 dyne = 107 newton and E = electric field) are
Q.7 The speed of light (c), gravitational constant (G) and (a) MLT–1 (b) ML2 T–2
(c) ML T –1 –2 (d) ML2 T–1
Planck's constant (h) are taken as the fundamental units in a
system. The dimensions of time in this new system should Q.15 Which of the following pairs is wrong?
be (a) Pressure-Baromter
(a) G1/2 h1/2 c –5/2 (b) G–1/2 h 1/2 c 1/2 (b) Relative density-Pyrometer
(c) G1/2 h1/2 c –3/2 (d) G1/2 h 1/2 c 1/2 (c) Temperature-Thermometer
Q.8 If the constant of gravitation (G), Planck's constant (h) and (d) Earthquake-Seismograph
the velocity of light (c) be chosen as fundamental units. Q.16 A physical quantity x depends on quantities y and z as
The dimensions of the radius of gyration is follows: x = Ay + B tan Cz, where A, B and C are constants.
(a) h1/2 c –3/2G 1/2 (b) h1/2 c 3/2 G 1/2 Which of the following do not have the same dimensions?
(c) h c1/2 –3/2 G –1/2 (d) h–1/2 c –3/2 G 1/2 (a) x and B (b) C and z–1
Q.9 The magnitude of any physical quantity (c) y and B/A (d) x and A
(a) depends on the method of measurement Q.17 If the time period (T) of vibration of a liquid drop depends
(b) does not depend on the method of measurement on surface tension (S), radius (r) of the drop and density (r)
(c) is more in SI system than in CGS system of the liquid, then the expression of T is
(d) directly proportional to the fundamental units of mass,
length and time
(a) T = k rr 3 / S (b) T = k r1/ 2 r 3 / S
Q.10 The unit of Stefan's constant s is (c) T = k rr 3 / S1/ 2 (d) None of these
(a) Wm–2 K–1 (b) Wm2K–4
(c) Wm K –2 –4 (d) Wm–2K4 Q.18 The dimensional formula for Planck’s constant (h) is
Q.11 In S = a + bt + ct2 , S is measured in metres and t in seconds. (a) [ML–2T–3 ] (b) [M0L2T–2 ]
The unit of c is (c) [M0L2T–1 ] (d) [ML–2T–2 ]
(a) ms–2 (b) m Q.19 What are the dimensions of permeability (m0) of vaccum?
(c) ms –1 (d) None (a) MLT–2I2 (b) MLT–2I–2
(c) ML–1T–2I2 (d) ML–1T –2I–2

5. 6. 7. 8. 9.
RESPONSE 10. 11. 12. 13. 14.
GRID
15. 16. 17. 18. 19.

Space for Rough Work


t.me/Ebooks_Encyclopedia27. t.me/Magazines4all

DPP/ P 01 3

Q.20 A small steel ball of radius r is allowed to fall under gravity Q.23 P represents radiation pressure, c represents speed of light and
through a column of a viscous liquid of coefficient of S represents radiation energy striking unit area per sec. The non
viscosity h. After some time the velocity of the ball attains a zero integers x, y, z such that Px Sy cz is dimensionless are
constant value known as terminal velocity vT. The terminal (1) x = 1 (2) y = – 1
velocity depends on (i) the mass of the ball m, (ii) h, (iii) r (3) z = 1 (4) x = – 1
and (iv) acceleration due to gravity g. Which of the following Q.24 Which of the following pairs have same dimensions?
relations is dimensionally correct? (1) Angular momentum and work
mg hr (2) Torque and work
(a) vT µ (b) vT µ
hr mg (3) Energy and Young’s modulus
(4) Light year and wavelength
mgr
(c) vT µ hrmg (d) vT µ DIRECTION (Q.25-Q.27) : Read the passage given below and
h
answer the questions that follows :
Q.21 The equation of state of some gases can be expressed as
Three of the fundamental constants of physics are the universal
æ a ö gravitational constant, G = 6.7 × 10–11m3kg–1s–2, the speed
ç P + 2 ÷ (V - b) = RT . Here P is the pressure, V is the of light, c = 3.0 × 10 8 m/s, and Planck’s constant,
è V ø
h = 6.6 × 10–34 kg m2 s–1.
volume, T is the absolute temperature and a, b and R are
Q.25 Find a combination of these three constants that has the
constants. The dimensions of 'a' are
dimensions of time. This time is called the Planck time and
(a) ML5 T–2 (b) ML–1 T–2 represents the age of the universe before which the laws of
(c) M0L3T0 (d) M0L6T0 physics as presently understood cannot be applied.
DIRECTIONS (Q.22-Q.24) : In the following questions, more
hG hG
than one of the answers given are correct. Select the correct (a) (b)
answers and mark it according to the following codes: c 4
c3
Codes :
hG hG
(a) 1, 2 and 3 are correct (b) 1 and 2 are correct (c) (d)
c c5
(c) 2 and 4 are correct (d) 1 and 3 are correct
Q.26 Find the value of Planck time in seconds
Q.22 The frequency of vibration f of a mass m suspended from
a spring of spring constant k is given by a relation of the (a) 1.3 × 10– 33 s (b) 1.3 × 10– 43 s
type f = c mx ky, where c is a dimensionless constant. The (c) 2.3 × 10– 13 s (d) 0.3 × 10– 23 s
values of x and y are
Q.27 The energy of a photon is given by E = hc .
1 1 l
(1) x= (2) x=-
2 2
If l = 4 ´ 10 -7 m , the energy of photon is
1 1 (a) 3.0 eV (b) 4.5 eV
(3) y=- (4) y=
2 2 (c) 2.10 eV (d) 3.95 eV

RESPONSE 20. 21. 22. 23. 24.


GRID 25. 26. 27.

Space for Rough Work


t.me/Ebooks_Encyclopedia27. t.me/Magazines4all

EBD_7156
4 DPP/ P 01
DIRECTIONS (Q. 28-Q.30) : Each of these questions contains two
æ 1 1 ö
statements: Statement-1 (Assertion) and Statement-2 (Reason). v = Rç - ÷ , where the symbols have their usual
çn 2 2÷
Each of these questions has four alternative choices, only one of è 1 n2 ø
which is the correct answer. You have to select the correct choice.
meaning.
(a) Statement-1 is True, Statement-2 is True; Statement-2 is a Q.29 Statement -1: The time period of a pendulum is given by
correct explanation for Statement-1.
the formula, T = 2p g / l .
(b) Statement-1 is True, Statement-2 is True; Statement-2 is NOT
a correct explanation for Statement-1. Statement -2: According to the principle of homogeneity of
(c) Statement -1 is False, Statement-2 is True. dimensions, only that formula is correct in which the
(d) Statement -1 is True, Statement-2 is False. dimensions of L.H.S. is equal to dimensions of R.H.S.

Q.28 Statement -1 : Unit of Rydberg constant R is m–1 Q.30 Statement -1: L/R and CR both have same dimensions.
Statement -2 : It follows from Bohr’s formula Statement -2: L/R and CR both have dimension of time.

RESPONSE GRID 28. 29. 30.

DAILY PRACTICE PROBLEM SHEET 1 - PHYSICS


Total Questions 30 Total Marks 120
Attempted Correct
Incorrect N et Score
Cut-off Score 30 Qualifying Score 50
Success Gap = Net Score – Qualifying Score
Net Score = (Correct × 4) – (Incorrect × 1)

Space for Rough Work


t.me/Ebooks_Encyclopedia27. t.me/Magazines4all

DPP - Daily Practice Problems


Name : Date :

Start Time : End Time :

02
SYLLABUS : Measurements (Errors)

Max. Marks : 120 Time : 60 min.


GENERAL INSTRUCTIONS
• The Daily Practice Problem Sheet contains 30 MCQ's. For each question only one option is correct. Darken the correct
circle/ bubble in the Response Grid provided on each page.
• You have to evaluate your Response Grids yourself with the help of solution booklet.
• Each correct answer will get you 4 marks and 1 mark shall be deduced for each incorrect answer. No mark will be given/
deducted if no bubble is filled. Keep a timer in front of you and stop immediately at the end of 60 min.
• The sheet follows a particular syllabus. Do not attempt the sheet before you have completed your preparation for that
syllabus. Refer syllabus sheet in the starting of the book for the syllabus of all the DPP sheets.
• After completing the sheet check your answers with the solution booklet and complete the Result Grid. Finally spend time
to analyse your performance and revise the areas which emerge out as weak in your evaluation.

Q.1 A wire has a mass 0.3 ± 0.003g, radius 0.5 ± 0.005 mm and measurement in a, b, c and d are 1%, 3%, 2% and 2%
length 6 ± 0.06 cm. The maximum percentage error in the respectiely. What is the percentage error in the quantity A
measurement of its density is (a) 12% (b) 7%
(a) 1 (b) 2 (c) 3 (d) 4 (c) 5% (d) 14%
Q.2 If 97.52 is divided by 2.54, the correct result in terms of
Q.4 A physical quantity is given by X = M a LbT c . The
significant figures is
percentage error in measurement of M, L and T are a, b and
(a) 38.4 (b) 38.3937
g respectively. Then maximum percentage error in the
(c) 38.394 (d) 38.39
quantity X is
Q.3 A physical quantity A is related to four observable a, b, c
(a) aa+ bb + cg (b) aa+ bb – cg
a 2b3 a b c
and d as follows, A = the percentage errors of (c) + + (d) None of these
c d a b g

RESPONSE GRID 1. 2. 3. 4.
t.me/Ebooks_Encyclopedia27. t.me/Magazines4all

EBD_7156
2 DPP/ P 02
Q.5 If the length of rod A is 3.25 ± 0.01 cm and that of B is Q.11 The mean time period of second's pendulum is 2.00s and
4.19 ± 0.01 cm then the rod B is longer than rod A by mean absolute error in the time period is 0.05s. To express
(a) 0.94 ± 0.00 cm (b) 0.94 ± 0.01 cm maximum estimate of error, the time period should be
(c) 0.94 ± 0.02 cm (d) 0.94 ± 0.005 cm written as
Q.6 If L = 2.331 cm, B = 2.1 cm, then L + B = (a) (2.00 ± 0.01)s (b) (2.00 + 0.025) s
(a) 4.431 cm (b) 4.43 cm (c) (2.00 ± 0.05) s (d) (2.00 ± 0.10) s
(c) 4.4 cm (d) 4 cm Q.12 Error in the measurement of radius of a sphere is 1%. The
Q.7 The number of significant figures in all the given numbers error in the calculated value of its volume is
25.12, 2009, 4.156 and 1.217 × 10–4 is (a) 1% (b) 3%
(a) 1 (b) 2 (c) 3 (d) 4 (c) 5% (d) 7%
Q.8 In an experiment, the following observation's were Q.13 The relative density of material of a body is found by
recorded: L = 2.820 m, M = 3.00 kg, l = 0.087 cm, weighing it first in air and then in water. If the weight in air
is (5.00 ± 0.05) newton and weight in water is (4.00 ± 0.05)
Diameter D = 0.041 cm. Taking g = 9.81 m/s2 using the
newton. Then the relative density along with the maximum
formula, Y = 4 MgL , the maximum percentage error in Y permissible percentage error is
pD 2l (a) 5.0 ± 11% (b) 5.0 ± 1%
is
(a) 7.96% (b) 4.56% (c) 6.50% (d) 8.42% (c) 5.0 ± 6% (d) 1.25 ± 5%
Q.9 A physical parameter a can be determined by measuring V
Q.14 The resistance R = where V = 100 ± 5 volts and
i
b a cb
the parameters b, c, d and e using the relation a = . i = 10 ± 0.2 amperes. What is the total error in R ?
d g ed
If the maximum errors in the measurement of b, c, d and e 5
(a) 5% (b) 7% (c) 5.2% (d) %
are b1%, c1%, d1%, and e1%, then the maximum error in 2
the value of a determined by the experiment is Q.15 The length of a cylinder is measured with a meter rod having
(a) (b1+ c1+ d1+ e1)% (b) (b1+ c1– d1– e1)% least count 0.1 cm. Its diameter is measured with vernier
(c) (ab1+ bc1– gd1– de1)%(d) (ab1+ bc1+ gd1+ de1)% calipers having least count 0.01 cm. Given that length is
5.0 cm. and radius is 2.0 cm. The percentage error in the
Q.10 The period of oscillation of a simple pendulum is given by
calculated value of the volume will be
l (a) 1% (b) 2%
T = 2p where l is about 100 cm and is known to have
g (c) 3% (d) 4%
1mm accuracy. The period is about 2s. The time of 100 Q.16 According to Joule’s law of heating, heat produced H =
oscillations is measured by a stopwatch of least count 0.1s. I2Rt, where I is current, R is resistance and t is time . If the
The percentage error in g is errors in the measurements of I,R. and t are 3%, 4% and
6% respectively then error in the measurement of H is
(a) 0.1% (b) 1%
(a) ± 17% (b) ± 16%
(c) 0.2% (d) 0.8% (c) ± 19% (d) ± 25%

5. 6. 7. 8. 9.
RESPONSE 10. 11. 12. 13. 14.
GRID
15. 16.

Space for Rough Work


t.me/Ebooks_Encyclopedia27. t.me/Magazines4all

DPP/ P 02 3

1 Q.22 In the context of accuracy of measurement and significant


3
A B2 figures in expressing results of experiment, which of the
Q.17 A physical quantity P is given by P = 3
. The quantity following is/are correct?
-4 1. Out of the two measurements 50.14 cm and 0.00025
C D2
ampere, the first one has greater accuracy
which brings in the maximum percentage error in P is 2. Out of the two measurements 50.14 cm and 0.00025
(a) A (b) B ampere, the second has greater accuracy.
(c) C (d) D 3. If one travels 478 km by rail and 397 m by road, the
Q.18 If there is a positive error of 50% in the measurement of total distance travelled is 875 km.
velocity of a body, then the error in the measurement of 4. If one travels 697 m by rail and 478 km by road, the
kinetic energy is total distance is 478 km.
(a) 25% (b) 50% Q.23 A thin copper wire of length l metre increases in length by
(c) 100% (d) 125% 2% when heated through 10°C. Which is not the
Q.19 The random error in the arithmetic mean of 100 percentage increase in area when a square copper sheet of
observations is x; then random error in the arithmetic mean length l metre is heated through 10°C
of 400 observations would be (1) 12% (2) 8% (3) 16% (4) 4%
1 1 Q.24 A body travels uniformly a distance of (13.8 ± 0.2) m in a
(a) 4x (b) x (c) 2x (d) x
4 2 time (4.0 ± 0.3) s.
Q.20 The percentage errors in the measurement of mass and
speed are 2% and 3% respectively. How much will be the 1. Its velocity with error limit is (3.5 ± 0.31) ms–1
maximum error in the estimation of the kinetic energy 2. Its velocity with error limit is (3.5 ± 0.11) ms–1
obtained by measuring mass and speed?
(a) 11% (b) 8% 3. Percentage error in velocity is ± 4%
(c) 5% (d) 1% 4. Percentage error in velocity is ± 9%
Q.21 The unit of percentage error is
(a) Same as that of physical quantity DIRECTION (Q.25-Q.27) : Read the passage given below
(b) Different from that of physical quantity and answer the questions that follows :
(c) Percentage error is unitless
(d) Errors have got their own units which are different The internal radius of a 1m long resonance tube is measured as
from that of physical quantity measured 3 cm. A tuning fork of frequency 2000 Hz is used. The first
resonating length is measured as 4.6 cm and the second
DIRECTIONS (Q.22-Q.24) : In the following questions, resonating length is measured as 14.0 cm.
more than one of the answers given are correct. Select the Q.25 Calculate the maximum percentage error in measurement
correct answers and mark it according to the following of e.
codes: (a) 3.33% (b) 2.23% (c) 4.33% (d) 5.33%
Codes : Q.26 Calculate the speed of sound at the room temperature.
(a) 1, 2 and 3 are correct (b) 1 and 2 are correct (a) 275 m/s (b) 376 m/s
(c) 2 and 4 are correct (d) 1 and 3 are correct (c) 356 m/s (d) 330 m/s

RESPONSE 17. 18. 19. 20. 21.


GRID 22. 23. 24. 25. 26.

Space for Rough Work


t.me/Ebooks_Encyclopedia27. t.me/Magazines4all

EBD_7156
4 DPP/ P 02
Q.27 Calculate the end correction. (c) Statement-1 is False, Statement-2 is True.
(a) 0.2 cm (b) 0.3 cm (d) Statement-1 is True, Statement-2 is False.
(c) 0.1 cm (d) 0.4 cm Q.28 Statement-1: Number of significant figures in 0.005 is
one and that in 0.500 is three.
DIRECTIONS (Q. 28-Q.30) : Each of these questions contains Statement-2 : This is because zero is not significant.
two statements: Statement-1 (Assertion) and Statement-2 Q.29 Statement-1: Out of three measurements l = 0.7 m;
(Reason). Each of these questions has four alternative choices, l = 0.70 m and l = 0.700 m, the last one is most accurate.
only one of which is the correct answer. You have to select the
Statement-2: In every measurement, only the last
correct choice.
significant digit is not accurately known.
(a) Statement-1 is True, Statement-2 is True; Statement-2 is a Q.30 Statement-1: Parallex method cannot be used for
correct explanation for Statement-1. measuring distances of stars more than 100 light years
(b) Statement-1 is True, Statement-2 is True; Statement-2 is away.
NOT a correct explanation for Statement-1. Statement-2: Because parallex angle reduces so much that
it cannot be measured accurately.

RESPONSE GRID 27. 28. 29. 30.

DAILY PRACTICE PROBLEM SHEET 2 - PHYSICS


Total Questions 30 Total Marks 120
Attempted Correct
Incorrect N et Score
Cut-off Score 28 Qualifying Score 46
Success Gap = Net Score – Qualifying Score
Net Score = (Correct × 4) – (Incorrect × 1)

Space for Rough Work


t.me/Ebooks_Encyclopedia27. t.me/Magazines4all

DPP - Daily Practice Problems


Name : Date :

Start Time : End Time :

SYLLABUS : MOTION IN A STRAIGHT LINE 1 (Distance, Displacement, Uniform & Non-uniform motion)
03
Max. Marks : 116 Time : 60 min.
GENERAL INSTRUCTIONS
• The Daily Practice Problem Sheet contains 29 MCQ's. For each question only one option is correct. Darken the correct
circle/ bubble in the Response Grid provided on each page.
• You have to evaluate your Response Grids yourself with the help of solution booklet.
• Each correct answer will get you 4 marks and 1 mark shall be deduced for each incorrect answer. No mark will be given/
deducted if no bubble is filled. Keep a timer in front of you and stop immediately at the end of 60 min.
• The sheet follows a particular syllabus. Do not attempt the sheet before you have completed your preparation for that
syllabus. Refer syllabus sheet in the starting of the book for the syllabus of all the DPP sheets.
• After completing the sheet check your answers with the solution booklet and complete the Result Grid. Finally spend time
to analyse your performance and revise the areas which emerge out as weak in your evaluation.

DIRECTIONS (Q.1-Q.20) : There are 20 multiple choice 1 1 3


(a)v0 t + bt 2 (b) v0 t + bt
questions. Each question has 4 choices (a), (b), (c) and (d), out 3 3
of which ONLY ONE choice is correct. 1 3 1 2
Q.1 A particle moving in a straight line covers half the distance (c) v0 t + bt (d) v0t + bt
6 2
with speed of 3 m/s. The other half of the distance is Q.3 The motion of a body is given by the equation
covered in two equal time intervals with speed of 4.5 m/s
dv(t )
and 7.5 m/s respectively. The average speed of the particle = 6.0 - 3v(t ), where v(t) is speed in m/s and t in sec.
during this motion is dt
(a) 4.0 m/s (b) 5.0 m/s If body was at rest at t = 0
(c) 5.5 m/s (d) 4.8 m/s (a) The terminal speed is 4 m/s
Q.2 The acceleration of a particle is increasing linearly with (b) The speed varies with the time as v(t) = 2(1 – e–5t)m/s
time t as bt. The particle starts from the origin with an initial (c) The speed is 0.1m/s when the acceleration is half the
velocity v0. The distance travelled by the particle in time t initial value
will be (d) The magnitude of the initial acceleration is 6.0 m/s2

RESPONSE GRID 1. 2. 3.
Space for Rough Work
t.me/Ebooks_Encyclopedia27. t.me/Magazines4all

EBD_7156
2 DPP/ P 03
Q.4 A particle of mass m moves on the x-axis as follows: it Q.8 A particle starts moving from the position of rest under a
starts from rest at t = 0 from the point x = 0 and comes to constant acc. If it covers a distance x in t second, what
rest at t = 1 at the point x = 1. No other information is distance will it travel in next t second?
available about its motion at intermediate time (0 < t < 1). (a) x (b) 2 x (c) 3 x (d) 4 x
If a denotes the instantaneous acceleration of the particle, Q.9 What will be the ratio of the distances moved by a freely
then falling body from rest in 4th and 5th seconds of journey?
(a) a cannot remain positive for all t in the interval (a) 4 : 5 (b) 7 : 9 (c) 16 : 25 (d) 1 : 1
0 £ t £1 Q.10 If a ball is thrown vertically upwards with speed u, the
(b) | a | cannot exceed 2 at any point in its path distance covered during the last t seconds of its ascent is
(c) | a | must be > 4 at some point or points in its path 1 2 1
(d) | a | = 2 at any point in its path. (a) (u+gt)t (b) ut (c) gt (d) ut – gt2
2 2
Q.5 A particle starts from rest. Its acceleration (a) versus time Q.11 If the displacement of a particle is (2t2 + t + 5) meter
(t) graph is as shown in the figure. The maximum speed of then, what will be acc. at t = 5 second?
the particle will be (a) 21 m/s2 (b) 20 m/s2
a (c) 4 m/s 2 (d) 10 m/s2
Q.12 A particle moves along x-axis with acceleration a = a0 (1 – t/
10 m/s2
T) where a0 and T are constants if velocity at t = 0 is zero
then find the average velocity from t = 0 to the time when a
= 0.
t (s) a 0T a 0T a 0T a 0T
11 (a) (b) (c) (d)
(a) 110 m/s (b) 55 m/s 3 2 4 5
(c) 550 m/s (d) 660 m/s Q.13 A point moves with uniform acceleration and v1, v2 and v3
denote the average velocities in the three successive
Q.6 A car accelerates from rest at a constant rate a for some
intervals of time t1, t2 and t3 . Which of the following
time, after which it decelerates at a constant rate b and relations is correct ?
comes to rest. If the total time elapsed is t, then the maximum (a) (v1– v2) : (v2– v3) = (t1– t2) : (t2+ t3)
velocity acquired by the car is (b) (v1– v2) : (v2– v3) = (t1+ t2) : (t2+ t3)
æ a 2 + b2 ö æ a 2 - b2 ö (c) (v1– v2) : (v2– v3) = (t1– t2) : (t2 – t3)
(a) çç ÷t
÷ (b) çç ÷t
÷
(d) (v1– v2) : (v2– v3) = (t1– t2) : (t2 – t3)
è ab ø è ab ø Q.14 The position of a particle moving in the xy-plane at any
(a + b)t abt time t is given by x = (3t2 – 6t) metres, y = (t2 – 2t) metres.
(c) (d) Select the correct statement about the moving particle from
ab a +b
Q.7 A small block slides without friction down an inclined plane the following
starting from rest. Let Sn be the distance travelled from (a) The acceleration of the particle is zero at t = 0 second
Sn (b) The velocity of the particle is zero at t = 0 second
time t = n – 1 to t = n. Then is (c) The velocity of the particle is zero at t = 1 second
S n +1
2n - 1 2n + 1 2n - 1 2n (d) The velocity and acceleration of the particle are never
(a) (b) (c) (d) zero
2n 2n - 1 2n + 1 2n + 1

4. 5. 6. 7. 8.
RESPONSE 9. 10. 11. 12. 13.
GRID
14.

Space for Rough Work


t.me/Ebooks_Encyclopedia27. t.me/Magazines4all

DPP/ P 03 3
Q.15 Two cars A and B are travelling in the same direction with Q.19 A particle moves along x-axis as x = 4 (t – 2) + a (t – 2)2
velocities v1 and v2 (v1 > v2). When the car A is at a distance Which of the following is true?
d ahead of the car B, the driver of the car A applied the (a) The initial velocity of particle is 4
brake producing a uniform retardation a. There will be no (b) The acceleration of particle is 2a
collision when (c) The particle is at origin at t = 0
(d) None of these
( v1 - v2 ) 2 v12 - v22 Q.20 The displacement x of a particle varies with time t,
(a) d< (b) d <
2a 2a x = ae –at + bebt, where a, b, a and b are positive constants.
The velocity of the particle will
(v1 - v2 ) 2 v 2 - v22 (a) Go on decreasing with time
(c) d > (d) d > 1
2a 2a (b) Be independent of a and b
Q.16 A body travels for 15 second starting from rest with (c) Drop to zero when a = b
constant acceleration. If it travels distances S1, S2 and S3 (d) Go on increasing with time
in the first five seconds, second five seconds and next five
seconds respectively the relation between S1, S2 and S3 is DIRECTIONS (Q.21-Q.23) : In the following questions,
(a) S1 = S2 = S3 (b) 5S1 = 3S2 = S3 more than one of the answers given are correct. Select
1 1 the correct answers and mark it according to the following
1 1
(c) S1 = S2 = S3 (d) S1 = S2 = S3 codes:
3 5 5 3
Q.17 The position of a particle moving along the x-axis at certain Codes :
times is given below (a) 1, 2 and 3 are correct (b) 1 and 2 are correct
(c) 2 and 4 are correct (d) 1 and 3 are correct
t (s) 0 1 2 3
x ( m) -2 0 6 16
Q.21A particle moves as such acceleration is given by
a = 3 sin 4t, then :
Which of the following describes the motion correctly? (1) the acceleration of the particle becomes zero after
(a) Uniform, accelerated p
each interval of second
(b) Uniform, decelerated 4
(c) Non-uniform, accelerated (2) the initial velocity of the particle must be zero
(d) There is not enough data for generalization (3) the particle comes at its initial position after sometime
Q.18 A body A moves with a uniform acceleration a and zero (4) the particle must move on a circular path
initial velocity. Another body B, starts from the same point
Q.22 A reference frame attached to the earth :
moves in the same direction with a constant velocity v. The
(1) is an inertial frame by definition
two bodies meet after a time t. The value of t is
(2) cannot be an inertial frame because the earth is
2v v revolving round the sun
(a) (b)
a a (3) is an inertial frame because Newton’s laws are
v v applicable in this frame
(c) (d) (4) cannot be an inertial frame because the earth is
2a 2a
rotating about its own axis

RESPONSE 15. 16. 17. 18. 19.


GRID 20. 21. 22.

Space for Rough Work


t.me/Ebooks_Encyclopedia27. t.me/Magazines4all

EBD_7156
4 DPP/ P 03
Q.23 If a particle travels a linear distance at speed v1 and comes 2 2t 2 2t
back along the same track at speed v2. (a) sin pt - (b) - 2 sin pt +
p2 p p p
(1) Its average speed is arithmetic mean (v1 + v2)/2 2t
(2) Its average speed is harmonic mean 2 v1v2/(v1 + v2)/2 (c) (d) None of these
p
(3) Its average speed is geometric mean v1v2 DIRECTIONS (Qs. 27-Q.29) : Each of these questions
(4) Its average velocity is zero contains two statements: Statement-1 (Assertion) and
DIRECTION (Q.24-Q.26) : Read the passage given below Statement-2 (Reason). Each of these questions has four
and answer the questions that follows : alternative choices, only one of which is the correct answer.
You have to select the correct choice.
A particle moves along x-axis and its acceleration at any time t
(a) Statement-1 is True, Statement-2 is True; Statement-2 is a
is a = 2 sin (pt), where t is in seconds and a is in m/s2. The initial
correct explanation for Statement-1.
velocity of particle (at time t = 0) is u = 0.
(b) Statement-1 is True, Statement-2 is True; Statement-2 is
Q.24. The distance travelled (in meters) by the particle from
NOT a correct explanation for Statement-1.
time to t = 0 to t = 1s will be –
(c) Statement -1 is False, Statement-2 is True.
2 1
(a) (b) (d) Statement -1 is True, Statement-2 is False.
p p Q.27 Statement-1 : The position-time graph of a uniform motion
4
(c) (d) None of these in one dimension of a body can have negative slope.
p
Statement-2 : When the speed of body decreases with time,
Q.25 The distance travelled (in meters) by the particle from time
the position-time graph of the moving body has negative slope.
t = 0 to t = t will be –
Q.28 Statement-1 : A body having non-zero acceleration can
2 2t 2 2t
(a) sin pt - (b) - 2 sin pt + have a constant velocity.
p 2 p p p Statement-2 : Acceleration is the rate of change of velocity.
2t Q.29 Statement-1 : Displacement of a body may be zero when
(c) (d) None of these
p distance travelled by it is not zero.
Q.26 The magnitude of displacement (in meters) by the particle Statement-2 : The displacement is the longest distance
from time t = 0 to t = t will be – between initial and final position.

RESPONSE 23. 24. 25. 26. 27.


GRID 28. 29.

DAILY PRACTICE PROBLEM SHEET 3 - PHYSICS


Total Questions 29 Total Marks 116
Attempted Correct
Incorrect N et Score
Cut-off Score 28 Qualifying Score 48
Success Gap = Net Score – Qualifying Score
Net Score = (Correct × 4) – (Incorrect × 1)
Space for Rough Work
t.me/Ebooks_Encyclopedia27. t.me/Magazines4all

DPP - Daily Practice Problems


Name : Date :

Start Time : End Time :

SYLLABUS : MOTION IN A STRAIGHT LINE 2 (Relative Motion & Motion Under Gravity)
04
Max. Marks : 112 Time : 60 min.
GENERAL INSTRUCTIONS
• The Daily Practice Problem Sheet contains 28 MCQ's. For each question only one option is correct. Darken the correct
circle/ bubble in the Response Grid provided on each page.
• You have to evaluate your Response Grids yourself with the help of solution booklet.
• Each correct answer will get you 4 marks and 1 mark shall be deduced for each incorrect answer. No mark will be given/
deducted if no bubble is filled. Keep a timer in front of you and stop immediately at the end of 60 min.
• The sheet follows a particular syllabus. Do not attempt the sheet before you have completed your preparation for that
syllabus. Refer syllabus sheet in the starting of the book for the syllabus of all the DPP sheets.
• After completing the sheet check your answers with the solution booklet and complete the Result Grid. Finally spend time
to analyse your performance and revise the areas which emerge out as weak in your evaluation.

DIRECTIONS (Q.1-Q.19) : There are 19 multiple choice v 2hg vé 2h ù


(a) + (b) 1- 1+ ú
questions. Each question has 4 choices (a), (b), (c) and (d), out g 2 g êë g û
of which ONLY ONE choice is correct.
Q.1 A stone is dropped from a minar of height h and it reaches vé 2 gh ù vé 2 2g ù
(c) ê1 + 1 + 2 ú (d) ê1 + v + ú
after t seconds on earth. From the same minar if two stones gë v û gë h û
are thrown (one upwards and other downwards) with the same
velocity u and they reach the earth surface after t1 and t2 Q.3 A man throws balls with the same speed vertically upwards,
seconds respectively, then one after the other at an interval of 2 seconds. What should
be the speed of the throw so that more than two balls are in
t1 + t2
(a) t = t1 - t2 (b) t = the sky at any time? (Given g = 9.8 m/s2)
2
(a) At least 0.8 m/s
(c) t = t1t2 (d) t = t12 t22
(b) Any speed less than 19.6 m/s
Q.2 A ball is projected upwards from a height h above the surface (c) Only with speed 19.6 m/s
of the earth with velocity v. The time at which the ball strikes
(d) More than 19.6 m/s
the ground is

RESPONSE GRID 1. 2. 3.
Space for Rough Work
t.me/Ebooks_Encyclopedia27. t.me/Magazines4all

EBD_7156
2 DPP/ P 04
Q.4 If a ball is thrown vertically upwards with speed u, the Q.8 A body is projected up with a speed 'u' and the time taken
distance covered during the last t second of its ascent is by it is T to reach the maximum height H. Pick out the
1 2 1 correct statement
(a) gt (b) ut - gt 2 (c) (u – gt)t (d) ut d
2 2 (a) It reaches H/2 in T/2 sec
Q.5 A ball is thrown vertically upwards. Which of the following (b) It acquires velocity u/2 in T/2 sec
graphs represent velocity-time graph of the ball during its (c) Its velocity is u/2 at H/2
flight? (air resistance is neglected) (d) Same velocity at 2T
v v
Q.9 Time taken by an object falling from rest to cover the height
of h1 and h2 is respectively t1 and t2 then the ratio of t1 to t2
is
(a) t (b) t
(a) h1 : h2 (b) h1 : h2 (c) h1 : 2h2 (d) 2h1 : h2
Q.10 Three different objects of masses m1, m2 and m3 are allowed
v v to fall from rest and from the same point 'O' along three
different frictionless paths. The speeds of the three objects,
t t on reaching the ground, will be in the ratio of
(c) (d) (a) m1 : m2 : m3 (b) m1 : 2m2 : 3m3
1 1 1 1
Q.6 A ball is dropped vertically from a height d above the (c) 1 : 1 : 1 (d) : :
m1 m2 m3
ground. It hits the ground and bounces up vertically to a
height d/2. Neglecting subsequent motion and air Q.11 From the top of a tower, a particle is thrown vertically
resistance, its velocity v varies with the height h above the downwards with a velocity of 10 m/s. The ratio of the
ground is distances, covered by it in the 3rd and 2nd seconds of the
v v motion is (Take g = 10m/s2)
(a) 5 : 7 (b) 7 : 5 (c) 3 : 6 (d) 6 : 3
d Q.12 A body falls from a height h = 200 m. The ratio of distance
h h
(a) (b) d travelled in each 2 second during t = 0 to t = 6 second of
the journey is
(a) 1 : 4 : 9 (b) 1 : 2 : 4
v v (c) 1 : 3 : 5 (d) 1 : 2 : 3
Q.13 The effective acceleration of a body, when thrown upwards
d d with acceleration a will be :
h h
(c) (d)
(a) a - g2 (b) a2 + g 2

Q.7 P, Q and R are three balloons ascending with velocities U, (c) (a – g) (d) (a + g)
4U and 8U respectively. If stones of the same mass be Q.14 An aeroplane is moving with a velocity u. It drops a packet
dropped from each, when they are at the same height, then from a height h. The time t taken by the packet in reaching
(a) They reach the ground at the same time the ground will be
(b) Stone from P reaches the ground first æ 2g ö æ 2u ö æ h ö æ 2h ö
(c) Stone from R reaches the ground first (a) ç ÷ (b) ç ÷ (c) ç ÷ (d) ç ÷
è h ø è g ø è 2g ø è g ø
(d) Stone from Q reaches the ground first

4. 5. 6. 7. 8.
RESPONSE 9. 10. 11. 12. 13.
GRID
14.

Space for Rough Work


t.me/Ebooks_Encyclopedia27. t.me/Magazines4all

DPP/ P 04 3
Q.15 Two trains, each 50 m long are travelling in opposite direction Q.20 Two particles move simultaneously from two points A and B,
with velocity 10 m/s and 15 m/s. The time of crossing is 300m apart. The particle at A, starts towards B with a velocity
(a) 2s (b) 4s of 25 m/s and that at B, moves normal to the former with a
velocity of 20 m/s.
(c) 2 3s (d) 4 3s (1) The relative velocity of the particle at A, w.r.t. that at B is
Q.16 A train of 150 metre length is going towards north direction 32.02 m/s
at a speed of 10 m/s. A parrot flies at the speed of 5 m/s (2) The relative velocity of the particle at A, w.r.t. that at B is
towards south direction parallel to the railway track. The 12.04 m/s
time taken by the parrot to cross the train is (3) They are closest to each other after 7.32 sec.
(a) 12 sec (b) 8 sec (4) They are closest to each other after 4.25 sec.
(c) 15 sec (d) 10 sec Q.21 A plane is to fly due north. The speed of the plane relative
Q.17 The distance between two particles is decreasing at the rate to the air is 200 km/h, and the wind is blowing from west
of 6 m/sec. If these particles travel with same speeds and in to east at 90 km/h.
(1) The plane should head in a direction of sin –1 (0.45)
the same direction, then the separation increase at the rate
(2) The plane should head in a direction of sin –1 (0.60)
of 4 m/s. The particles have speeds as
(3) The relative velocity of plane w.r.t. ground is 179 km/h
(a) 5 m/sec; 1 m/sec (b) 4 m/sec; 1 m/sec (4) The relative velocity of plane w.r.t. ground is 149 km/h
(c) 4 m/sec; 2 m/sec (d) 5 m/sec; 2 m/sec Q.22 From the top of a multi-storeyed building 40m tall, a boy
Q.18 A train is moving towards east and a car is along north, both projects a stone vertically upwards with an initial velocity
with same speed. The observed direction of car to the of 10 ms–1 such that it eventually falls to the ground.
passenger in the train is (1) After 4 s the stone will strike the ground
(a) East-north direction (b) West-north direction (2) After 2 s the stone will pass through the point from
(c) South-east direction (d) None of these where it was projected
Q.19 An express train is moving with a velocity v1. Its driver (3) Its velocity when it strikes the ground is 30 m/s
finds another train is moving on the same track in the same (4) Its velocity when it strikes the ground is 40 m/s
direction with velocity v2. To escape collision, driver applies
DIRECTIONS (Q.23-Q.25) : Read the passage given below
a retardation a on the train, the minimum time of escaping
and answer the questions that follows :
collision will be
When an airplane flies, its total velocity with respect to the
v -v v 2 - v22
(a) t= 1 2 (b) t1 = 1 ground is vtotal = vplane + vwind,
a 2
where vplane denotes the plane’s velocity through motionless air,
(c) Both (a) and (b) (d) None of these and vwind denotes the wind’s velocity. Crucially, all the quantities
in this equation are vectors. The magnitude of a velocity vector
DIRECTIONS (Q.20-Q.22) : In the following questions, more
is often called the “speed.”
than one of the answers given are correct. Select the correct
Consider an airplane whose speed through motionless air is 100
answers and mark it according to the following codes: meters per second (m/s). To reach its destination, the plane must
Codes : fly east.
(a) 1, 2 and 3 are correct (b) 1 and 2 are correct The “heading” of a plane is in the direction in which the nose of
(c) 2 and 4 are correct (d) 1 and 3 are correct the plane points. So, it is the direction in which the engines propel
the plane.

RESPONSE 15. 16. 17. 18. 19.


GRID 20. 21. 22.

Space for Rough Work


t.me/Ebooks_Encyclopedia27. t.me/Magazines4all

EBD_7156
4 DPP/ P 04
Q.23 If the plane has an eastward heading, and a 20 m/s wind (a) Statement-1 is True, Statement-2 is True; Statement-2 is a
blows towards the southwest, then the plane’s speed is – correct explanation for Statement-1.
(a) 80 m/s (b) Statement-1 is True, Statement-2 is True; Statement-2 is NOT
(b) more than 80 m/s but less than 100 m/s a correct explanation for Statement-1.
(c) 100 m/s (c) Statement -1 is False, Statement-2 is True.
(d) more than 100 m/s (d) Statement -1 is True, Statement-2 is False.
Q.24 The pilot maintains an eastward heading while a 20 m/s Q.26 Statement-1 : The magnitude of velocity of two boats
wind blows northward. The plane’s velocity is deflected relative to river is same. Both boats start simultaneously
from due east by what angle?
from same point on the bank may reach opposite bank
-1 20 -1 20 simultaneously moving along different paths.
(a) sin (b) cos
100 100
Statement-2 : For boats to cross the river in same time.
-1 20 The component of their velocity relative to river in
(c) tan (d) none
100 direction normal to flow should be same.
Q.25 Let f denote the answer of above question. The plane has Q.27 Statement-1 : The acceleration of a body of mass 2 kg
what speed with respect to the ground ? thrown vertically upwards is always constant.
(a) (100 m/s) sin f (b) (100 m/s) cos f Statement-2 : A body of all mass group travels under
100 m/ s 100 m/ s constant acceleration when only gravity acts on it.
(c) sin f
(d) cosf Q.28 Statement-1 : The velocity of a body A relative to the body
DIRECTIONS (Qs. 26-Q.28) : Each of these questions B is the sum of the velocities of bodies A and B if both
contains two statements: Statement-1 (Assertion) and travel in opposite direction on a straight line.
Statement-2 (Reason). Each of these questions has four Statement-2 : The velocity of a body A relative to the body
alternative choices, only one of which is the correct answer. B is the difference of the velocities of bodies A and B if
You have to select the correct choice. both travel in opposite direction on a straight line.

RESPONSE 23. 24. 25. 26. 27.


GRID 28.

DAILY PRACTICE PROBLEM SHEET 4 - PHYSICS


Total Questions 28 Total Marks 112
Attempted Correct
Incorrect N et Score
Cut-off Score 28 Qualifying Score 44
Success Gap = Net Score – Qualifying Score
Net Score = (Correct × 4) – (Incorrect × 1)

Space for Rough Work


t.me/Ebooks_Encyclopedia27. t.me/Magazines4all

DPP - Daily Practice Problems


Name : Date :

Start Time : End Time :

SYLLABUS : Vectors
05
Max. Marks : 116 Time : 60 min.
GENERAL INSTRUCTIONS
• The Daily Practice Problem Sheet contains 29 MCQ's. For each question only one option is correct. Darken the correct
circle/ bubble in the Response Grid provided on each page.
• You have to evaluate your Response Grids yourself with the help of solution booklet.
• Each correct answer will get you 4 marks and 1 mark shall be deduced for each incorrect answer. No mark will be given/
deducted if no bubble is filled. Keep a timer in front of you and stop immediately at the end of 60 min.
• The sheet follows a particular syllabus. Do not attempt the sheet before you have completed your preparation for that
syllabus. Refer syllabus sheet in the starting of the book for the syllabus of all the DPP sheets.
• After completing the sheet check your answers with the solution booklet and complete the Result Grid. Finally spend time
to analyse your performance and revise the areas which emerge out as weak in your evaluation.

DIRECTIONS (Q.1-Q.21) : There are 21 multiple choice 1


(a) zero (b) m/s 2 N-W
questions. Each question has 4 choices (a), (b), (c) and (d), out 2
of which ONLY ONE choice is correct. 1 1
Q.1 The length of second’s hand in watch is 1 cm. The change
(c) m/s2 N-E (d) m/s 2S-W
2 2
in velocity of its tip in 15 seconds is ur
Q.3 A force F = - K ( yiˆ + xjˆ ) (where K is a positive constant)
p
(a) zero (b) cm/sec acts on a particle moving in the x-y plane. Starting from
30 2 the origin, the particle is taken along the positive x-axis to
p the point (a, 0) and then parallel to the y-axis to the point
p 2 ur
(c) cm/sec (d) cm/sec
30 30 (a, a). The total work done by the forces F on the particle
Q.2 A particle moves towards east with velocity 5 m/s. After is
10 seconds its direction changes towards north with same (a) – 2 Ka2 (b) 2 Ka2
(c) – Ka 2 (d) Ka2
velocity. The average acceleration of the particle is

RESPONSE GRID 1. 2. 3.
Space for Rough Work
t.me/Ebooks_Encyclopedia27. t.me/Magazines4all

EBD_7156
2 DPP/ P 05
Q.4 A metal sphere is hung by a string fixed to a wall. The
A
sphere is pushed away from the wall by a stick. The forces
acting on the sphere are shown in the second diagram. 30°
Which of the following statements is wrong? 30 N
O

W
q

q (a) 30 3,30 (b) 30 3, 60


(c) 60 3,30 (d) None of these
P
W
Q.9 A boat is moving with a velocity 3iˆ + 4 ˆj with respect to
r r r
(a) P = W tan q (b) T + P + W = 0 ground. The water in the river is moving with a velocity
(c) T 2 = P 2 + W 2 (d) T = P + W – 3iˆ - 4 ˆj with respect to ground. The relative velocity of
Q.5 The speed of a boat is 5km/h in still water. It crosses a
the boat with respect to water is
river of width 1 km along the shortest possible path in 15
minutes. The velocity of the river water is (a) 8 ˆj (b) -6iˆ - 8 ˆj (c) 6iˆ + 8 ˆj (d) 5 2 iˆ
(a) 1 km/h (b) 3 km/h Q.10 A person aiming to reach the exactly opposite point on the
(c) 4 km/h (d) 5 km/h bank of a stream is swimming with a speed of 0.5 m/s at an
Q.6 A man crosses a 320 m wide river perpendicular to the angle of 120° with the direction of flow of water. The speed
current in 4 minutes. If in still water he can swim with a of water in the stream is
speed 5/3 times that of the current, then the speed of the (a) 1 m/s (b) 0.5 m/s (c) 0.25 m/s(d) 0.433 m/s
current in m/min is Q.11 A man can swim with velocity v relative to water. He has to
(a) 30 (b) 40 cross a river of width d flowing with a velocity u (u > v).
(c) 50 (d) 60 The distance through which he is carried down stream by
Q.7 P, Q and R are three coplanar forces acting at a point and the river is x. Which of the following statements is correct?
are in equilibrium. Given P = 1.9318 kg wt, sin q1 = 0.9659,
the value of R is (in kg wt) (a) If he crosses the river in minimum time x = du
v
150° du
P Q
(b) x cannot be less than
v
q2 q1
(c) For x to be minimum he has to swim in a direction making
R
ο ævö
an angle of , sin,1 çç ÷÷÷ with the direction of the
2 çè u ø
(a) 0.9659 (b) 2 flow of water.
1 (d) x will be maximum if he swims in a direction making
(c) 1 (d) p
2 ævö
an angle of + sin -1 ç ÷ with direction of the flow of
Q.8 As shown in figure the tension in the horizontal cord is 30 2 èuø
N. The weight W and tension in the string OA in newton are water.

RESPONSE 4. 5. 6. 7. 8.
GRID 9. 10. 11.

Space for Rough Work


t.me/Ebooks_Encyclopedia27. t.me/Magazines4all

DPP/ P 05 3
Q.12 A 120 m long train is moving towards west with a speed of r r
Q.20 Two forces F1 = 5iˆ + 10 ˆj - 20kˆ and F2 = 10iˆ - 5 ˆj - 15kˆ
10 m/s. A bird flying towards east with a speed of 5 m/s
r r
crosses the train. The time taken by the bird to cross the act on a single point. The angle between F1 and F2 is nearly
train will be
(a) 30° (b) 45°
(a) 16 sec (b) 12 sec (c) 10 sec (d) 8 sec
ur (c) 60° (d) 90°
Q.13 What is the value of linear velocity, if w = 3$i - 4 $j + k$ and Q.21 With respect to a rectangular cartesian coordinate system,
r
r = 5i$ - 6 $j + 6k$
three vectors are expressed as
r r
(a) 6$i - 2 $j + 3k$ (b) 6$i - 2 $j + 8k$ a = 4iˆ - ˆj, b = -3iˆ + 2 ˆj , and cr = - kˆ

(c) 4$i - 13 $j + 6k$ (d) -18$i - 13 $j + 2k$ where iˆ, ˆj , kˆ are unit vectors, along the X, Y and Z-axis
ur ur ur ur ur ur
Q.14 If | A ´ B | = 3 A . B, then the value of | A + B | is respectively. The unit vectors r̂ along the direction of sum
1/ 2 of these vector is
æ 2 2 AB ö
(a) çè A + B + ÷ (b) A + B 1 ˆ ˆ ˆ 1 ˆ ˆ ˆ
3ø (a) rˆ = (i + j - k ) (b) rˆ = (i + j - k )
3 2
(c) ( A2 + B 2 + 3 AB )1/ 2 (d) ( A2 + B 2 + AB )1/ 2 1 1 ˆ ˆ ˆ
ur (c) rˆ = (iˆ - ˆj + kˆ) (d) rˆ = (i + j + k )
Q.15 Find the torque of a force F = -3$i + $j + 5k$ acting at a point 3 2
r
r = 7$i + 3 $j + k$ DIRECTIONS (Q.22-Q.24) : In the following questions,
(a) 14$i - 38 $j + 16k$ (b) 4$i + 4 $j + 6k$ more than one of the answers given are correct. Select
the correct answers and mark it according to the following
(c) 21$i + 4 $j + 4k$ (d) -14$i + 34 $j - 16k$ codes:
ur ur ur ur ur ur
Q.16 If | A ´ B | = | A . B |, then angle between A and B will be Codes :
(a) 30° (b) 45° (c) 60° (d) 90° (a) 1, 2 and 3 are correct (b) 1 and 2 are correct
ur ur
Q.17 The vector P = ai + a j + 3k and Q = ai$ - 2 $j - k$ are
$ $ $ (c) 2 and 4 are correct (d) 1 and 3 are correct
Q.22 A boy walks uniformally along the sides of a rectangular
perpendicular to each other. The positive value of a is
(a) 3 (b) 4 park of size 400 m × 300 m, starting from one corner to
(c) 9 (d) 13 the other corner diagonally opposite. Which of the
following statements is correct?
Q.18 A particle moves from position 3$i + 2 $j - 6k$ to (1) He has travelled a distance of 700 m
14$i + 13 $j + 9k$ due to a uniform force of (4i$ + $j + 3k$ ) N . (2) His displacement is 500 m
If the displacement in metres then work done will be (3) His velocity is not uniform throughout the walk
(a) 100 J (b) 200 J (c) 300 J d) 250 J (4) His displacement is 700 m
ur ur r r
Q.19 The three vectors A = 3iˆ - 2 ˆj + kˆ, B = iˆ - 3 ˆj + 5kˆ and Q.23 The three vectors A = 3iˆ - 2jˆ - k, ˆ B = iˆ - 3jˆ + 5kˆ and
ur r
C = 2iˆ + ˆj - 4kˆ form C = 2iˆ – ˆj - 4kˆ does not form
(a) an equilateral triangle (b) isosceles triangle (1) an equilateral triangle (2) isosceles triangle
(c) a right angled triangle (d) no triangle (3) a right angled triangle (4) no triangle

12. 13. 14. 15. 16.


RESPONSE 17. 18. 19. 20. 21.
GRID
22. 23.

Space for Rough Work


t.me/Ebooks_Encyclopedia27. t.me/Magazines4all

EBD_7156
4 DPP/ P 05
r r r r
Q.24 If for two vectors A and B, A ´ B < 0, which of the following DIRECTIONS (Q.28-Q.29) : Each of these questions contains two
is not correct? statements: Statement-1 (Assertion) and Statement-2 (Reason).
(1) They are perpendicular to each other Each of these questions has four alternative choices, only one of
(2) They act at an angle of 60° which is the correct answer. You have to select the correct choice.
(3) They act at an angle of 30°
(a) Statement-1 is True, Statement-2 is True; Statement-2 is a
(4) They are parallel to each other
correct explanation for Statement-1.
DIRECTIONS (Q.25-Q.27) : Read the passage given below (b) Statement-1 is True, Statement-2 is True; Statement-2 is
and answer the questions that follows : NOT a correct explanation for Statement-1.
r r
A = 2iˆ + ˆj + kˆ and B = iˆ + ˆj + kˆ are two vectors. (c) Statement -1 is False, Statement-2 is True.
r (d) Statement -1 is True, Statement-2 is False.
Q.25 The unit vector perpendicular to A is
r r r r r
- ˆj + kˆ - ˆj - kˆ iˆ + kˆ iˆ - kˆ Q.28 Statement-1:If A + B = A - B , then angle between A
(a) (b) (c) (d)
2 2 2 2 r
r and B is 90°
Q.26 The unit vector parallel to A is r r r r
2iˆ - ˆj + 3kˆ 2iˆ + ˆj + kˆ Statement-2 : A + B = B + A
(a) (b) Q.29 Statement-1 : The sum of two vectors can be zero.
2 6
ˆ ˆ
2i - j - k ˆ 2i + ˆj - 2kˆ
ˆ Statement-2 : Two vectors cancel each other, when they
(c) (d) are equal and opposite.
5 6
r
Q.27 The unit vector perpendicular to B is
- ˆj - kˆ - ˆj + kˆ iˆ - kˆ iˆ + kˆ
(a) (b) (c) (d)
3 2 3 2

RESPONSE 24. 25. 26. 27. 28.


GRID 29.

DAILY PRACTICE PROBLEM SHEET 5 - PHYSICS


Total Questions 29 Total Marks 116
Attempted Correct
Incorrect N et Score
Cut-off Score 30 Qualifying Score 44
Success Gap = Net Score – Qualifying Score
Net Score = (Correct × 4) – (Incorrect × 1)

Space for Rough Work


t.me/Ebooks_Encyclopedia27. t.me/Magazines4all

DPP - Daily Practice Problems


Name : Date :

Start Time : End Time :

SYLLABUS : MOTION IN A PLANE-1 (Projectile Motion)


06
Max. Marks : 112 Time : 60 min.
GENERAL INSTRUCTIONS
• The Daily Practice Problem Sheet contains 28 MCQ's. For each question only one option is correct. Darken the correct
circle/ bubble in the Response Grid provided on each page.
• You have to evaluate your Response Grids yourself with the help of solution booklet.
• Each correct answer will get you 4 marks and 1 mark shall be deduced for each incorrect answer. No mark will be given/
deducted if no bubble is filled. Keep a timer in front of you and stop immediately at the end of 60 min.
• The sheet follows a particular syllabus. Do not attempt the sheet before you have completed your preparation for that
syllabus. Refer syllabus sheet in the starting of the book for the syllabus of all the DPP sheets.
• After completing the sheet check your answers with the solution booklet and complete the Result Grid. Finally spend time
to analyse your performance and revise the areas which emerge out as weak in your evaluation.

DIRECTIONS (Q.1-Q.20) : There are 20 multiple choice Q.3 If t1 be the time taken by a body to clear the top of a building
questions. Each question has 4 choices (a), (b), (c) and (d), out and t2 be the time spent in air, then t2 : t1 will be –
of which ONLY ONE choice is correct. (a) 1 : 2 (b) 2 : 1 (c) 1 : 1 (d) 1 : 4
Q.1 The path followed by a body projected along y axis is given Q.4 The co-ordinates of a moving particle at any time t are given
by x = ct2 and y = bt2. The speed of the particle is
by y = 3 x – (1/2) x2. If g = 10 m/s2 , then the initial
(a) 2t (c + b) (b) 2t c2 - b2
velocity of projectile will be – (x and y are in m)
(c) t c 2 + b 2 (d) 2t c 2 + b 2
(a) 3 10 m/s (b) 2 10 m/s
Q.5 The height y and the distance x along the horizontal at plane
(c) 10 3 m/s (d) 10 2 m/s of the projectile on a certain planet (with no surrounding
Q.2 When the angle of elevation of a gun are 60º and 30º atmosphere) are given by y = (8t – 5t2) metre and x = 6t
respectively, the height it shoots are h1 and h2 respectively, metre where t is in second. The velocity with which the
h1/h2 equal to – projectile is projected is
(a) 3/1 (b) 1/3 (c) 1/2 (d) 2/1 (a) 8 m/s (b) 6 m/s
(c) 10 m/s (d) Data is insufficient

RESPONSE GRID 1. 2. 3. 4. 5.
Space for Rough Work
t.me/Ebooks_Encyclopedia27. t.me/Magazines4all

EBD_7156
2 DPP/ P 06
Q.6 A body is thrown at an angle 30º to the horizontal with the to meet the ball at that instant. What must be his speed so
velocity of 30 m/s. After 1 sec, its velocity will be that he could catch the ball before hitting the ground ?
(in m/s) (g = 10 m/s2)
(a) 2.82 m/s (b) 2/ 2 m/s
(a) 10 7 (b) 700 10 (c) 100 7 (d) 10 (c) 39.2 m/s (d) 10 m/s
Q.7 A particle is moving in a plane with a velocity given by, Q.13 A ball is thrown from ground level so as to just clear a wall
r 4 metres high at a distance of 4 metres and falls at a distance
u = u0 î + (wa cos wt) ĵ , where î and ĵ are unit vectors of 14 metres from the wall. The magnitude of velocity of
along x and y-axes respectively. If the particle is at the origin the ball will be
at t = 0, then its distance from the origin at time t = 3p/ (a) 182 m/s (b) 181 m/s
2w will be
(c) 185 m/s (d) 186 m/s
éæ 3pu ö 2 ù éæ 3pu 0 ö 2 ù Q.14 A ball is projeced from O with an initial velocity 700 cm/
(a) êç 0
÷ + a2ú (b) êç ÷+a ú
êëè 2w ø úû ëè 2w ø û s in a direction 37° above the horizontal. A ball B, 500 cm
away from O on the line of the initial velocity of A, is
éæ 3pu ö 2 ù éæ 4pu ö 2 ù released from rest at the instant A is projected. The height
(c) ê ç 0
÷ + a ú (d) êç 0
÷ + a2 ú through which B falls, before it is hit by A and the direction
êëè 2w ø úû êëè 2w ø úû of the velocity A at the time of impact will respectively be
Q.8 A ball thrown by one player reaches the other in 2 sec. The [given g = 10 m/s2, sin 37° = 0.6 and cos 37° = 8.0]
maximum height attained by the ball above the point of (a) 250 cm, 28° 42' (b) 255 cm, 27° 43'
projection will be about- (c) 245 cm, 20° 44' (d) 300 cm, 27° 43'
(a) 2.5 m (b) 5 m (c) 7.5 m (d) 10 m Q.15 A ball is thrown horizontally from a height of 20 m. It hits
Q.9 Rishabh and Bappy are playing with two different balls of the ground with a velocity three times its initial velocity.
masses m and 2m respectively. If Rishabh throws his ball The initial velocity of ball is
vertically up and Bappy at an angle q, both of them stay in (a) 2 m/s (b) 3 m/s (c) 5 m/s (d) 7 m/s
our view for the same period. The height attained by the Q.16 A projectile thrown from a height of 10 m with velocity of
two balls are in the ratio of
(a) 2 : 1 (b) 1 : 1 (c) 1 : cos q (d) 1 : sec q 2 m/s, the projectile will fall, from the foot of
Q.10 A projectile is thrown at an angle q and (900 – q) from the projection, at distance-(g = 10 m/s2)
same point with same velocity 98 m/s. The heights attained (a) 1 m (b) 2 m (c) 3m (d) 2m
by them, if the difference of heights is 50 m will be (in m) Q.17 Savita throws a ball horizontally with a velocity of 8 m/s
(a) 270, 220 (b) 300, 250 from the top of her building. The ball strikes to her brother
(c) 250, 200 (d) 200, 150 Sudhir playing at 12 m away from the building. What is the
Q.11 A particle is projected with a velocity u so that its horizontal height of the building ?
range is twice the maximum height attained. The horizontal (a) 11m (b) 10 m (c) 8 m (d) 7 m
range is Q.18 A body is projected downdwards at an angle of 30º to the
(a) u2/g (b) 2u2 /3g (c) 4u2/5g (d) u2/2g horizontal with a velocity of 9.8 m/s from the top of a tower
Q.12 Mr C.P. Nawani kicked off a football with an initial speed 29.4 m high. How long will it take before striking the
19.6 m/s at a projection angle 45º. A receiver on the goal ground?
line 67.4 m away in the direction of the kick starts running (a) 1s (b) 2s (c) 3s (d) 4s

6. 7. 8. 9. 10.
RESPONSE 11. 12. 13. 14. 15.
GRID
16. 17. 18.

Space for Rough Work


t.me/Ebooks_Encyclopedia27. t.me/Magazines4all

DPP/ P 06 3
Q.19 A ball is thrown from the top of a tower with an initial velocity (3) A hunter aims his gun and fires a bullet directly at a
of 10 m/s at an angle of 30º above the horizontal. It hits the monkey on a tree. At the instant bullet leaves the gun,
ground at a distance of 17.3 m from the base of the tower. monkey drops, the bullet misses to hit the monkey.
The height of the tower (g = 10 m/s2) will be (4) If a baseball player can throw a ball at maximum
(a) 10 m (b) 12 m (c) 110 m (d) 100 m distance = d over a ground, the maximum vertical
height to which he can throw it, will be d
Q.20 A ball 'A' is projected from origin with an initial velocity
(Ball have same initial speed in each case)
v0 = 700 cm/sec in a direction 37º above the horizontal as Q.22 A ball projected with speed 'u' at an angle of projection
shown in fig .Another ball 'B' 300 cm from origin on a line 15º has range R. The other angle of projection at which the
37º above the horizontal is released from rest at the instant range will not be same with same initial speed 'u' is
A starts. How far will B have fallen when it is hit by A ? (1) 45º (2) 35º
Y (3) 90º (4) 75º
B Q.23 A projectile can have the same range R for two angles of
projections. If t1 and t2 be the times of flight in two cases,
cm A
0 then choose the incorrect relations –
30
= (1) t1t2 µ 1/R2 (2) t1t2 µ R2
B y-axis
O
(3) t1t2 µ 1/R (4) t1t2 µ R
O X
(a) 9 cm (b) 90 cm DIRECTIONS (Q.24-Q.26) : Read the passage given below
(c) 0.9 cm (d) 900 cm and answer the questions that follows :

DIRECTIONS (Q.21-Q.23) : In the following questions, Velocity at a general point P(x, y) for a horizontal projectile
more than one of the answers given are correct. Select motion is given by
the correct answers and mark it according to the following u
codes: u
h

v= u 2 + g 2 t 2
Codes : vy gt
v= v 2x + v 2y ; tan a =
(a) 1, 2 and 3 are correct (b) 1 and 2 are correct vx
R
(c) 2 and 4 are correct (d) 1 and 3 are correct
Q.21 Choose the correct options a is angle made by v with horizontal in clockwise direction
(1) A ball is dropped from the window of a moving train Trajectory equation for a horizontal projectile motion is given
on horizontal rails, the path followed by the ball as by x = vxt = ut
vx
observed by the observer on the ground is parabolic a
path.
vy
(2) If T be the total time of flight of a current of water y = – (1/2) gt2
and H be the maximum height attained by it from the v
point of projection, then H/T will be (1/4) u sinq
gx 2
(u = projection velocity and q = projection angle) eliminating t, we get y = – (1/2)
u2

RESPONSE GRID 19. 20. 21. 22. 23.

Space for Rough Work


t.me/Ebooks_Encyclopedia27. t.me/Magazines4all

EBD_7156
4 DPP/ P 06
Q.24 A ball rolls off top of a stair way with a horizontal velocity DIRECTIONS (Q.27-Q.28) : Each of these questions contains
u m/s. If the steps are h m high and b meters wide, the ball two statements: Statement-1 (Assertion) and Statement-2
will just hit the edge of nth step if n equals to (Reason). Each of these questions has four alternative choices,
only one of which is the correct answer. You have to select
hu 2 u 2g the correct choice.
(a) (b) (a) Statement-1 is True, Statement-2 is True; Statement-2 is a
gb 2 gb 2
correct explanation for Statement-1.
(b) Statement-1 is True, Statement-2 is True; Statement-2 is
2hu 2 2u 2g NOT a correct explanation for Statement-1.
(c) (d)
gb 2 hb 2 (c) Statement -1 is False, Statement-2 is True.
(d) Statement -1 is True, Statement-2 is False.
Q.25 An aeroplane is in a level flying at an speed of 144 km/hr Q.27 Statement -1 : Two projectiles are launched from the top
at an altitude of 1000 m. How far horizontally from a given of a cliff with same initial speed with different angles of
target should a bomb be released from it to hit the target ? projection. They reach the ground with the same speed.
(a) 571.43 m (b) 671.43 m Statement -2 : The work done by gravity is same in both the
cases.
(c) 471.34 m (d) 371.34 m
Q.28 Statement-1 : A man projects a stone with speed u at some
Q.26 An aeroplane is flying horizontally with a velocity of 720 angle. He again projects a stone with same speed such that
km/h at an altitude of 490 m. When it is just vertically above time of flight now is different. The horizontal ranges in
the target a bomb is dropped from it. How far horizontally it both the cases may be same. (Neglect air friction)
missed the target? Statement-2 : The horizontal range is same for two
(a) 1000 m (b) 2000 m projectiles projected with same speed if one is projected
at an angle q with the horizontal and other is projected at
(c) 100 m (d) 200 m
an angle (90° – q) with the horizontal. (Neglect air friction)

RESPONSE GRID 24. 25. 26. 27. 28.

DAILY PRACTICE PROBLEM SHEET 6 - PHYSICS


Total Questions 28 Total Marks 112
Attempted Correct
Incorrect N et Score
Cut-off Score 28 Qualifying Score 42
Success Gap = Net Score – Qualifying Score
Net Score = (Correct × 4) – (Incorrect × 1)

Space for Rough Work


t.me/Ebooks_Encyclopedia27. t.me/Magazines4all

DPP - Daily Practice Problems


Name : Date :

Start Time : End Time :

SYLLABUS : MOTION IN A PLANE-2 (Horizontal Circular Motion)


07
Max. Marks : 120 Time : 60 min.
GENERAL INSTRUCTIONS
• The Daily Practice Problem Sheet contains 30 MCQ's. For each question only one option is correct. Darken the correct
circle/ bubble in the Response Grid provided on each page.
• You have to evaluate your Response Grids yourself with the help of solution booklet.
• Each correct answer will get you 4 marks and 1 mark shall be deduced for each incorrect answer. No mark will be given/
deducted if no bubble is filled. Keep a timer in front of you and stop immediately at the end of 60 min.
• The sheet follows a particular syllabus. Do not attempt the sheet before you have completed your preparation for that
syllabus. Refer syllabus sheet in the starting of the book for the syllabus of all the DPP sheets.
• After completing the sheet check your answers with the solution booklet and complete the Result Grid. Finally spend time
to analyse your performance and revise the areas which emerge out as weak in your evaluation.

DIRECTIONS (Q.1-Q.21) : There are 21 multiple choice (a) 1 : 12 (b) 6 : 1 (c) 12 : 1 (d) 1 : 6
questions. Each question has 4 choices (a), (b), (c) and (d), out Q.4 The angular displacement of a particle is given by
of which ONLY ONE choice is correct. 1 2
q = w0t + at , where w0 and a are constant and
Q.1 A particle completes 1.5 revolutions in a circular path of 2
radius 2 cm. The angular displacement of the particle will w0 = 1 rad/sec, a = 1.5 rad/sec2. The angular velocity at
be – (in radian) time, t = 2 sec will be (in rad/sec) -
(a) 6 p (b) 3 p (c) 2 p (d) p (a) 1 (b) 5
Q.2 A particle revolving in a circular path completes first one
third of circumference in 2 sec, while next one third in 1 (c) 3 (d) 4
sec. The average angular velocity of particle will be – (in Q.5 The magnitude of the linear acceleration of the particle
rad/sec) moving in a circle of radius 10 cm with uniform speed
(a) 2p/3 (b) p/3 (c) 4p/3 (d) 5p/3 completing the circle in 4 s, will be -
Q.3 The ratio of angular speeds of minute hand and hour hand (a) 5p cm/s2 (b) 2.5p cm/s2
of a watch is - (c) 5p2 cm/s2 (d) 2.5p2 cm/s2

RESPONSE GRID 1. 2. 3. 4. 5.
Space for Rough Work
t.me/Ebooks_Encyclopedia27. t.me/Magazines4all

EBD_7156
2 DPP/ P 07
Q.6 A cane filled with water is revolved in a vertical circle of Q.13 A particle is moving in a circular path with velocity varying
radius 4 m and water just does not fall down. The time with time as v = 1.5t2 + 2t. If the radius of circular path is
period of revolution will be – 2 cm, the angular acceleration at t = 2 sec will be -
(a) 1 s (b) 10 s (a) 4 rad/sec2 (b) 40 rad/sec2
(c) 400 rad/sec 2 (d) 0.4 rad/sec2
(c) 8 s (d) 4 s
Q.7 The length of second's hand in a watch is 1 cm. The change Q.14 A grind stone starts from rest and has a constant-angular
in velocity of its tip in 15 second is - acceleration of 4.0 rad/sec2.The angular displacement and
p angular velocity, after 4 sec. will respectively be -
(a) 0 (b) cm/s
30 2 (a) 32 rad, 16 rad/s (b) 16 rad, 32 rad/s
p p 2 (c) 64 rad, 32 rad/s (d) 32 rad, 64 rad/s
(c) cm/s (d) cm/s
30 30 Q.15 The shaft of an electric motor starts from rest and on the
Q.8 An electron is moving in a circular orbit of radius 5.3 × 10– application of a torque, it gains an angular acceleration
11 metre around the atomic nucleus at a rate of 6.6 × 1015
given by a = 3t – t2 during the first 2 seconds after it starts
revolutions per second. The centripetal force acting on the after which a = 0. The angular velocity after 6 sec will be -
electron will be - (a) 10/3 rad/sec (b) 3/10 rad/sec
(The mass of the electron is 9.1 × 10–31kg) (c) 30/4 rad/sec (d) 4/30 rad/sec
(a) 8.3 × 10–8N (b) 3.8 × 10–8N Q.16 Using rectangular co-ordinates and the unit vectors i and
(c) 4.15 × 10 N –8 (d) 2.07 × 10–8N j, the vector expression for the acceleration a will be (r is
Q.9 An air craft executes a horizontal loop of radius 1 km with a position vector) -
a steady speed of 900 km/h. The ratio of centripetal (a) wr2 (b) –w2 r/2
acceleration to that gravitational acceleration will be- (c) –2wr 2 (d) –w2r
(a) 1 : 6.38 (b) 6. 38 : 1 Q.17 The vertical section of a road over a canal bridge in the
(c) 2.25 : 9.8 (d) 2.5 : 9.8 direction of its length is in the form of circle of radius 8.9
Q.10 A car driver is negotiating a curve of radius 100 m with a metre. Find the greatest speed at which the car can cross
speed of 18 km/hr. The angle through which he has to lean this bridge without losing contact with the road at its highest
from the vertical will be - point, the center of gravity of the car being at a height h =
1 1 1.1 metre from the ground. (Take g = 10 m/sec2)
(a) tan–1 (b) tan–1
4 40 (a) 5 m/s (b) 7 m/s
æ 1 ö÷ æ 1 ö÷ (c) 10 m/s (d) 13 m/s
(c) tan –1 çççè ø÷÷ (d) tan –1 çççè ø÷÷ Q.18 The maximum speed at which a car can turn round a curve
2 20
of 30 metre radius on a level road if the coefficient of
Q.11 A particle moves in a circle of radius 20cm with a linear
friction between the tyres and the road is 0.4, will be -
speed of 10m/s. The angular velocity will be -
(a) 10.84 m/s (b) 17.84 m/s
(a) 50 rad/s (b) 100 rad/s
(c) 11.76 m/s (d) 9.02 m/s
(c) 25 rad/s (d) 75 rad/s
Q.19 The angular speed with which the earth would have to rotate
Q.12 The angular velocity of a particle is given by w = 1.5 t – 3t2 +
on its axis so that a person on the equator would weigh
2, the time when its angular acceleration decreases to be
(3/5)th as much as present, will be:
zero will be -
(Take the equatorial radius as 6400 km)
(a) 25 sec (b) 0.25 sec
(a) 8.7 × 104 rad/sec (b) 8.7 × 103 rad/sec
(c) 12 sec (d) 1.2 sec 4
(c) 7.8 × 10 rad/sec (d) 7.8 × 103 rad/sec

6. 7. 8. 9. 10.
RESPONSE 11. 12. 13. 14. 15.
GRID
16. 17. 18. 19.

Space for Rough Work


t.me/Ebooks_Encyclopedia27. t.me/Magazines4all

DPP/ P 07 3
Q.20 A smooth table is placed horizontally and a spring of (1) The speed of the particle will be 0.98 m/s
unstreched length l0 and force constant k has one end fixed
rg
to its centre. To the other end of the spring is attached a (2) tan q = (q is semi-apex angle)
mass m which is making n revolution per second around v2
the centre. Tension in the spring will be (3) The speed of the particle will be 98 m/s
(a) 4p2 m k l0 n2/ (k – 4p2 m n2) rg
(b) 4p2 m k l0 n2/ (k + 4p2 m n2) (4) tan q = (q is semiapex angle)
v
(c) 2p2 m k l0 n2/ (k – 4p2 m n2) Q.24 Choose the correct statements
(d) 2p m k l0 n2/ (k – 4p2 m n2) (1) Centripetal force is not a real force. It is only the
Q.21 A motor car is travelling at 30 m/s on a circular road of requirement for circular motion.
radius 500 m. It is increasing its speed at the rate of 2 m/ (2) Work done by centripetal force may or may not be
s2. Its net acceleration is (in m/s2) – zero.
(a) 2 (b) 1. 8 (3) Work done by centripetal force is always zero.
(c) 2.7 (d) 0 (4) Centripetal force is a fundamental force.
DIRECTIONS (Q.22-Q.24) : In the following questions, DIRECTIONS (Q.25-Q.27) : Read the passage given below
more than one of the answers given are correct. Select the and answer the questions that follows :
correct answers and mark it according to the following
codes: The velocity of the particle changes moving on the curved path,
this change in velocity is brought by a force known as centripetal
Codes :
force and the acceleration so produced in the body is known as
(a) 1, 2 and 3 are correct (b) 1 and 2 are correct
centripetal acceleration. The direction of centripetal force or
(c) 2 and 4 are correct (d) 1 and 3 are correct
acceleration is always towards the centre of circular path.
Q.22 Three identical particles are connected by three strings as
Q.25 A ball is fixed to the end of a string and is rotated in a
shown in fig. These particles are revolving in a horizontal
horizontal circle of radius 5 m with a speed of 10 m/sec.
circle. The velocity of outer most particle is v, then choose
The acceleration of the ball will be -
correct relation for T1,T2 and T3
(a) 20 m/s2 (b) 10 m/s2
(where T1 is tension in the outer most string etc.) 2
(c) 30 m/s (d) 40 m/s2
m m Q.26 A body of mass 2 kg lying on a smooth surface is attached
O m
l l l to a string 3 m long and then whirled round in a horizontal
circle making 60 revolution per minute. The centripetal
mvA2
5mv 2A acceleration will be-
(1) T1 = (2) T2 = (a) 118.4 m/s2 (b) 1.18 m/s2
3l 9l 2
(c) 2.368 m/s (d) 23.68 m/s2
6mvA2 5mvA2 Q.27 A body of mass 0.1 kg is moving on circular path of diameter
(3) T3= (4) T3 =
9l 9l 1.0 m at the rate of 10 revolutions per 31.4 seconds. The
Q.23 A particle describes a horizontal circle on the smooth centripetal force acting on the body is -
surface of an inverted cone. The height of the plane of the (a) 0.2 N (b) 0.4 N
circle above the vertex is 9.8 cm, then choose the correct (c) 2 N (d) 4 N
options

RESPONSE 20. 21. 22. 23. 24.


GRID 25. 26. 27.

Space for Rough Work


t.me/Ebooks_Encyclopedia27. t.me/Magazines4all

EBD_7156
4 DPP/ P 07
DIRECTIONS (Q. 28-Q.30) : Each of these questions contains two normal as well as tangential acceleration.
statements: Statement-1 (Assertion) and Statement-2 (Reason). Each Q.29 Statement - 1 : A cyclist is cycling on rough horizontal
of these questions has four alternative choices, only one of which is circular track with increasing speed. Then the frictional force
the correct answer. You have to select the correct choice. on cycle is always directed towards centre of the circular
track.
(a) Statement-1 is True, Statement-2 is True; Statement-2 is a Statement - 2 : For a particle moving in a circle, radial
correct explanation for Statement-1. component of net force should be directed towards centre.
(b) Statement-1 is True, Statement-2 is True; Statement-2 is r
NOT a correct explanation for Statement-1. Q.30 Statement - 1 : If net force F acting on a system is
r
(c) Statement -1 is False, Statement-2 is True. changing in direction only, the linear momentum ( p) of
(d) Statement -1 is True, Statement-2 is False. system changes in direction.
Q.28 Statement - 1 : In non-uniform circular motion, velocity Statement - 2 : In case of uniform circular motion,
vector and acceleration vector are not perpendicular to each magnitude of linear momentum is constant but direction
other. of centripetal force changes at every instant.
Statement - 2 : In non-uniform circular motion, particle has

RESPONSE GRID 28. 29. 30.

DAILY PRACTICE PROBLEM SHEET 7 - PHYSICS


Total Questions 30 Total Marks 120
Attempted Correct
Incorrect N et Score
Cut-off Score 30 Qualifying Score 48
Success Gap = Net Score – Qualifying Score
Net Score = (Correct × 4) – (Incorrect × 1)

Space for Rough Work


t.me/Ebooks_Encyclopedia27. t.me/Magazines4all

DPP - Daily Practice Problems


Name : Date :

Start Time : End Time :

SYLLABUS : MOTION IN A PLANE-3 (Vertical Circular Motion, Relative Motion)


08
Max. Marks : 112 Time : 60 min.
GENERAL INSTRUCTIONS
• The Daily Practice Problem Sheet contains 28 MCQ's. For each question only one option is correct. Darken the correct
circle/ bubble in the Response Grid provided on each page.
• You have to evaluate your Response Grids yourself with the help of solution booklet.
• Each correct answer will get you 4 marks and 1 mark shall be deduced for each incorrect answer. No mark will be given/
deducted if no bubble is filled. Keep a timer in front of you and stop immediately at the end of 60 min.
• The sheet follows a particular syllabus. Do not attempt the sheet before you have completed your preparation for that
syllabus. Refer syllabus sheet in the starting of the book for the syllabus of all the DPP sheets.
• After completing the sheet check your answers with the solution booklet and complete the Result Grid. Finally spend time
to analyse your performance and revise the areas which emerge out as weak in your evaluation.

DIRECTIONS (Q.1-Q.19) : There are 19 multiple choice A


questions. Each question has 4 choices (a), (b), (c) and (d), out D
of which ONLY ONE choice is correct. E r
Q.1 A man whirls a stone round his head on the end of a string r C
r
4.0 metre long. Can the string be in a horizontal, plane? If
the stone has a mass of 0.4 kg and the string will break, if B
the tension in it exceeds 8 N. The smallest angle the string (a) h > 5 r/2 (b) h < 5 r/2
can make with the horizontal and the speed of the stone (c) h < 2r/5 (d) h > 2r/5
will respectively be (Take g = 10 m/sec2)
Q.3 An aircraft loops the loop of radius R = 500 m with a
(a) 30º, 7.7 m/s (b) 60º, 7.7 m/s
constant velocity v = 360 km/h. The weight of the flyer of
(c) 45º, 8.2 m/s (d) 60º, 8.7 m/s
mass m = 70 kg in the lower, upper and middle points of
Q.2 In figure ABCDE is a channel in the vertical plane, part BCDE
the loop will respectively be-
being circular with radius r. A ball is released from A and
slides without friction and without rolling. It will complete (a) 210 N, 700 N, 1400 N (b) 1400 N, 700 N, 2100 N
the loop path when (c) 700 N, 1400 N, 210 N (d) 2100 N, 700 N, 1400 N

RESPONSE GRID 1. 2. 3.
Space for Rough Work
t.me/Ebooks_Encyclopedia27. t.me/Magazines4all

EBD_7156
2 DPP/ P 08
Q.4 A particle of mass 3 kg is moving under the action of a Q.11 The roadway bridge over a canal is the form of an arc of a
central force whose potential energy is given by U(r) = circle of radius 20 m. What is the minimum speed with
10r3 joule. For what energy and angular momentum will which a car can cross the bridge without leaving contact
the orbit be a circle of radius 10 m ? with the ground at the highest point (g = 9.8 m/s2)
(a) 2.5 × 104 J, 3000 kgm2/sec (a) 7 m/s (b) 14 m/s (c) 289 m/s (d) 5 m/s
(b) 3.5 × 104 J, 2000 kgm2/sec Q.12 A cane filled with water is revolved in a vertical circle of
(c) 2.5 × 103 J, 300 kgm2/sec radius 0.5 m and the water does not fall down. The
(d) 3.5 × 103 J, 300 kgm2/sec maximum period of revolution must be -
Q.5 A string of length 1 m is fixed at one end and carries a (a) 1.45 (b) 2.45 (c) 14.15 (d) 4.25
mass of 100 gm at the other end. The string makes 2/p Q.13 A particle of mass m slides down from the vertex of semi-
revolutions per second about a vertical axis through the
hemisphere, without any initial velocity. At what height
fixed end. The angle of inclination of the string with the
from horizontal will the particle leave the sphere-
vertical, and the linear velocity of the mass will respec-
tively be - (in M.K.S. system) 2 3 5 8
(a) 52º14', 3.16 (b) 50º14', 1.6 (a) R (b) R (c) R (d) R
3 2 8 5
(c) 52º14', 1.6 (d) 50º14', 3.16 Q.14 A body of mass m tied at the end of a string of length l is
Q.6 A particle of mass m is moving in a circular path of constant
radius r such that its centripetal acceleration ac is varying projected with velocity 4lg , at what height will it leave
with time t as ac = k2rt2, where k is a constant. The power the circular path -
delivered to the particle by the force acting on it will be -
(a) mk2t2r (b) mk2r2t2 (c) m2k2t2r2 (d) mk2r2t 5 3 1 2
(a) l (b) l (c) l (d) l
Q.7 A car is moving in a circular path of radius 100 m with 3 5 3 3
velocity of 200 m/sec such that in each sec its velocity Q.15 A string of length L is fixed at one end and carries a mass
increases by 100 m/s, the net acceleration of car will be - M at the other end. The string makes 2/p revolutions per
(in m/sec) second around the vertical axis through the fixed end as
(a) 100 17 (b) 10 7 (c) 10 3 (d) 100 3 shown in the figure, then tension in the string is
Q.8 A 4 kg balls is swing in a vertical circle at the end of a cord
1 m long. The maximum speed at which it can swing if the (a) ML S

cord can sustain maximum tension of 163.6 N will be - q


(a) 6 m/s (b) 36 m/s (c) 8 m/s (d) 64 m/s (b) 2 ML L

Q.9 The string of a pendulum is horizontal. The mass of the T


(c) 4 ML
bob is m. Now the string is released. The tension in the M
string in the lowest position is -
(d) 16 ML R
(a) 1 mg (b) 2 mg (c) 3 mg (d) 4 mg
Q.10 A swimmer can swim in still water at a rate 4 km/h. If he Q.16 A train has to negotiate a curve of radius 400 m. By how
swims in a river flowing at 3 km/h and keeps his direction much should the outer rail be raised with respect to inner
(w.r.t. water) perpendicular to the current. Find his velocity rail for a speed of 48 km/hr. The distance between the rails
w.r.t. the ground. is 1 m.
(a) 3 km/hr (b) 5 km/hr (a) 12 m (b) 12 cm
(c) 4 km/hr (d) 7 km/hr (c) 4.5 cm (d) 4.5 m

4. 5. 6. 7. 8.
RESPONSE 9. 10. 11. 12. 13.
GRID
14. 15. 16.

Space for Rough Work


t.me/Ebooks_Encyclopedia27. t.me/Magazines4all

DPP/ P 08 3
Q.17 A ship is steaming towards east at a speed of 12 ms–1. A Q.21 A swimmer who can swim in a river with speed mv (with
woman runs across the deck at a speed of 5 ms–1 in the respect to still water) where v is the velocity of river
direction at right angles to the direction of motion of the current, jumps into the river from one bank to cross the
ship i.e. towards north. What is the velocity of the woman river. Then
relative to sea ? (1) If m < 1 he cannot cross the river
(a) 13 m/s (b) 5 m/s (c) 12 m/s (d) 17 m/s (2) If m £ 1 he cannot reach a point on other bank di-
Q.18 A man is walking on a level road at a speed of 3 km/h. rectly opposite to his starting point.
Raindrops fall vertically with a speed of 4 km/h. Find the (3) If m > 1 he can reach a point on other bank
velocity of raindrops with respect to the men. (4) He can reach the other bank at some point, whatever
(a) 3 km/hr (b) 4 km/hr (c) 5 km/hr (d) 7 km/hr be the value of m.
Q.19 A stone of mass 1 kg tied to a light inextensible string of Q.22 Consider two children riding on the merry-go-round Child
1 sits near the edge, Child 2 sits closer to the centre.
length L = 10 m is whirling in a circular path of radius L Let vl and v2 denote the linear speed of child 1 and child 2,
3 respectively. Which of the following is/are wrong ?
in a vertical plane. If the ratio of the maximum tension in (1) We cannot determine v 1 & v 2 without more
the string to the minimum tension in the string is 4 and if g information
is taken to be 10m / sec2, the speed of the stone at the (2) v1 = v2
highest point of the circle is (3) v1 < v2
(a) 20 m/sec (b) 10 3m/sec (4) v1 > v2
(c) 5 2m/sec (d) 10 m/sec DIRECTIONS (Q.23-Q.25) : Read the passage given below
and answer the questions that follows :
DIRECTIONS (Q.20-Q.22) : In the following questions,
more than one of the answers given are correct. Select the Three of the fundamental constants of physics are the universal
correct answers and mark it according to the following gravitational constant, G = 6.7 × 10–11m3kg–1s–2, the speed of
codes: light, c = 3.0 × 108 m/s, and Planck’s constant, h = 6.6 × 10–34 Js–1.
Two particles A and B are projected in the vertical plane with
Codes :
same initial velocity u0 from part (0, 0) and (l, –h) towards each
(a) 1, 2 and 3 are correct (b) 1 and 2 are correct
other as shown in figure at t = 0.
(c) 2 and 4 are correct (d) 1 and 3 are correct
Q.20 Two bodies P and Q are moving along positive x-axis their y
r
position-time graph is shown below. If VPQ is velocity of u0
r
P w.r.t. Q and VQP is velocity of Q w.r.t P, then g = 10m/s2
r r x
(1) | VPQ | = | VQP | = constant A x
P (0,0)
r l
(2) VPQ towards origin Q
u0 h
r
(3) VQP towards origin
r r
(4) | VPQ | ¹ | VQP | = constant t (l, –h) B

RESPONSE 17. 18. 19. 20. 21.


GRID 22.

Space for Rough Work


t.me/Ebooks_Encyclopedia27. t.me/Magazines4all

EBD_7156
4 DPP/ P 08
Q.23 The path of particle A with respect to particle B will be – (a) Statement-1 is True, Statement-2 is True; Statement-2 is a
(a) parabola correct explanation for Statement-1.
(b) straight line parallel to x-axis (b) Statement-1 is True, Statement-2 is True; Statement-2 is
(c) straight line parallel to y-axis NOT a correct explanation for Statement-1.
(d) None of these (c) Statement-1 is False, Statement-2 is True.
Q.24 Minimum distance between particle A and B during motion (d) Statement-1 is True, Statement-2 is False.
will be –
Q.26 Statement-1 : The relative velocity between any two
(a) l (b) h
bodies moving in opposite direction is equal to sum of the
(c) l2 + h 2 (d) l + h velocities of two bodies.
Q.25 The time when separation between A and B is minimum is Statement-2 : Sometimes relative velocity between two
x 2h bodies is equal to difference in velocities of the two.
(a) u 0 cosq (b) g Q.27 Statement-1: A river is flowing from east to west at a
l 2l
speed of 5m/min. A man on south bank of river, capable of
(c) 2u cos q (d) u cosq swimming 10 m/min in still water, wants to swim across
0 0
the river in shortest time. He should swim due north.
DIRECTIONS (Qs. 26-Q.28) : Each of these questions contains Statement-2 : For the shortest time the man needs to swim
two statements: Statement-1 (Assertion) and Statement-2 perpendicular to the bank.
(Reason). Each of these questions has four alternative choices,
Q.28 Statement-1 : Rain is falling vertically downwards with
only one of which is the correct answer. You have to select the
velocity 6 km/h. A man walks with a velocity of 8 km/h.
correct choice.
Relative velocity of rain w.r.t. the man is 10 km/h.
Statement-2 : Relative velocity is the ratio of two
velocities.

RESPONSE 23. 24. 25. 26. 27.


GRID 28.

DAILY PRACTICE PROBLEM SHEET 8 - PHYSICS


Total Questions 28 Total Marks 112
Attempted Correct
Incorrect N et Score
Cut-off Score 28 Qualifying Score 44
Success Gap = Net Score – Qualifying Score
Net Score = (Correct × 4) – (Incorrect × 1)

Space for Rough Work


t.me/Ebooks_Encyclopedia27. t.me/Magazines4all

DPP - Daily Practice Problems


Name : Date :

Start Time : End Time :

SYLLABUS : LAWS OF MOTION-1 (Newton's laws, momentum, pseudo force concept)


09
Max. Marks : 116 Time : 60 min.
GENERAL INSTRUCTIONS
• The Daily Practice Problem Sheet contains 29 MCQ's. For each question only one option is correct. Darken the correct
circle/ bubble in the Response Grid provided on each page.
• You have to evaluate your Response Grids yourself with the help of solution booklet.
• Each correct answer will get you 4 marks and 1 mark shall be deduced for each incorrect answer. No mark will be given/
deducted if no bubble is filled. Keep a timer in front of you and stop immediately at the end of 60 min.
• The sheet follows a particular syllabus. Do not attempt the sheet before you have completed your preparation for that
syllabus. Refer syllabus sheet in the starting of the book for the syllabus of all the DPP sheets.
• After completing the sheet check your answers with the solution booklet and complete the Result Grid. Finally spend time
to analyse your performance and revise the areas which emerge out as weak in your evaluation.

DIRECTIONS (Q.1-Q.20) : There are 20 multiple choice Q.3 A boat of mass 1000 kg is moving with a velocity of
questions. Each question has 4 choices (a), (b), (c) and (d), out 5 m/s. A person of mass 60 kg jumps into the boat. The
of which ONLY ONE choice is correct. velocity of the boat with the person will be -
(a) 4.71 m/s (b) 4.71 cm/s
Q.1 A boy standing on a weighing machine observes his weight (c) 47.1 m/s (d) 47.1 cm/s
as 200 N. When he suddenly jumpes upwards, his friend Q.4 A disc of mass 10 gm is kept horizontally in air by firing
notices that the reading increased to 400 N. The bullets of mass 5 g each at the rate of 10/s. If the bullets
acceleration by which the boy jumped will be- rebound with same speed. The velocity with which the
(a) 9.8 m/s2 (b) 29.4 m/s2 bullets are fired is -
(c) 4.9 m/s 2 (d) 14.7 m/s2 (a) 49 cm/s (b) 98 cm/s (c) 147 cm/s (d) 196 cm/s
Q.2 A force of (6 î + 8 ĵ ) N acted on a body of mass 10 kg. The Q.5 A fire man has to carry an injured person of mass 40 kg
displacement after 10 sec, if it starts from rest, will be - from the top of a building with the help of the rope which
(a) 50 m along tan –1 4/3 with x axis can withstand a load of 100 kg. The acceleration of the
(b) 70 m along tan –1 3/4 with x axis fireman if his mass is 80 kg, will be-
(c) 10 m along tan–1 4/3 with x axis (a) 8.17 m/s2 (b) 9.8 m/s2
(d) None (c) 1.63 m/s 2 (d) 17.97 m/s2

RESPONSE GRID 1. 2. 3. 4. 5.
Space for Rough Work
t.me/Ebooks_Encyclopedia27. t.me/Magazines4all

EBD_7156
2 DPP/ P 09
Q.6 A body of mass 0.02 kg falls from a height of 5 metre into force of engine is 4500 N. The acceleration of the engine
a pile of sand. The body penetrates the sand a distance of 5 and tension in the coupling will respectively be-
cm before stoping. What force has the sand exerted on the (a) 0.04 m/s2, 2000 N (b) 0.4 m/s2, 200 N
body ? 2
(c) 0.4 m/s , 20 N (d) 4 m/s2, 200 N
(a) 1.96 N (b) –19.6 N Q.11 A body whose mass 6 kg is acted upon by two forces
(c) –0.196 N (d) 0.0196 N
(8iˆ + 10j)
ˆ N and (4iˆ + 8j)
ˆ N. The acceleration produced will
Q.7 The magnitude of the force (in newton) acting on a body
varies with time t (in microsecond) as shown in fig. AB, be (in m/s2) –
BC, and CD are straight line segments. The magnitude of (a) (3iˆ + 2j)
ˆ (b) 12iˆ + 18jˆ
the total impulse of the force on the body from t = 4 ms to
t = 16 ms is 1 ˆ ˆ
800 C (c) (i + j) (d) 2iˆ + 3jˆ
(a) 5 × 10–4 N.s 3
600
Force F (N)

400 Q.12 A car of 1000 kg moving with a velocity of 18 km/hr is


(b) 5 × 10–3 N.s 200
A B
stopped by the brake force of 1000 N. The distance covered
F E
(c) 5 × 10–5 N.s by it before coming to rest is -
0 2 4 6 8 10 12 1416D
Time (m s)____ (a) 1 m (b) 162 m (c) 12.5 m (d) 144 m
(d) 5 × 10–2 N.s Q.13 A block of metal weighing 2 kg is resting on a frictionless
Q.8 The total mass of an elevator with a 80 kg man in it is 1000 plane. It is struck by a jet releasing water at a rate of 1 kg/
kg. This elevator moving upward with a speed of 8 m/sec, s and at a speed of 5 m/s. The initial acceleration of the
is brought to rest over a distance of 16m. The tension T in block will be –
the cables supporting the elevator and the force exerted (a) 2.5 m/s2 (b) 5 m/s2 (c) 0.4 m/s2 (d) 0
on the man by the elevator floor will respectively be- Q.14 A man fires the bullets of mass m each with the velocity v
(a) 7800 N, 624 N (b) 624 N, 7800 N with the help of machine gun, if he fires n bullets every
(c) 78 N, 624 N (d) 624 N, 78 N sec, the reaction force per second on the man will be -
Q.9 In the arrangement shown in fig. the ends P and Q of an
unstretchable string move downwards with a uniform speed m mv vn
(a) n (b) m n v (c) (d)
U. Pulleys A and B are fixed. Mass M moves upwards with v n m
a speed of Q.15 A body of mass 15 kg moving with a velocity of 10 m/s is
A B to be stopped by a resistive force in 15 sec, the force will
be -
q q (a) 10 N (b) 5 N
P Q (c) 100 N (d) 50 N
Q.16 A cricket ball of mass 250 gm moving with a velocity of
24 m/s is hit by a bat so that it acquires a velocity of 28 m/
M s in the opposite direction. The force acting on the ball, if
(a) 2U cos q (b) U cos q the contact time is 1/100 of a second, will be -
(c) 2U/cos q (d) U/cos q (a) 1300 N in the final direction of ball
Q.10 An engine of mass 5 × 104 kg pulls a coach of mass 4 × (b) 13 N in the initial direction of ball
104 kg. Suppose that there is a resistance of 1 N per 100 (c) 130 N in the final direction of ball
kg acting on both coach and engine, and that the driving (d) 1.3 N in the initial direction of ball

6. 7. 8. 9. 10.
RESPONSE 11. 12. 13. 14. 15.
GRID
16.

Space for Rough Work


t.me/Ebooks_Encyclopedia27. t.me/Magazines4all

DPP/ P 09 3
Q.17 A force of 2 N is applied on a particle for 2 sec, the change Q.23 Choose the correct options
in momentum will be - (1) A reference frame in which Newton’s first law is valid
(a) 2 Ns (b) 4 Ns (c) 6 Ns (d) 3 Ns is called an inertial reference frame.
Q.18 A body of mass 2 kg is moving along x-direction with a (2) Frame moving at constant velocity relative to a known
velocity of 2 m/sec. If a force of 4 N is applied on it along inertial frame is also an inertial frame.
y-direction for 1 sec, the final velocity of particle will be - (3) Idealy, no inertial frame exists in the universe for
practical purpose, a frame of reference may be
(a) 2 2 m/s (b) 2 m/s
considered as inertial if its acceleration is negligible
(c) 1/ 2 m/s (d) 1/ 2 2 m/s with respect to the acceleration of the object to be
Q.19 A cricket ball of mass 150 g is moving with a velocity of observed.
12m/sec and is hit by a bat so that the ball is turned back (4) To measure the acceleration of a falling apple, earth
with a velocity of 20 m/sec, the force on the ball acts for cannot be considered as an inertial frame.
0.01 sec, then the average force exerted by the bat on the
ball will be DIRECTIONS (Q.24-Q.26) : Read the passage given below
(a) 48 N (b) 40 N (c) 480 N (d) 400 N and answer the questions that follows :
Q.20 A body of mass 20 kg moving with a velocity of 3 m/s, Pseudo force is an imaginary force which is recognised only by a
rebounds on a wall with same velocity. The impulse on the non-inertial observer to explain the physical situation according
body is - to newton’s laws. Magnitude of pseudo force Fp is equal to the
(a) 60 Ns (b) 120 Ns (c) 30 Ns (d) 180 Ns product of the mass m of the object and the acceleration a of the
DIRECTIONS (Q.21-Q.23) : In the following questions, frame of reference. The direction of the force is opposite to the
more than one of the answers given are correct. Select the direction of acceleration, Fp = –ma
correct answers and mark it according to the following Q.24 A spring weighing machine inside a stationary lift reads
codes: 50 kg when a man stand on it. What would happen to the
scale reading if the lift is moving upward with (i) constant
Codes : velocity (ii) constant acceleratioin ?
(a) 1, 2 and 3 are correct (b) 1 and 2 are correct
æ 50a ö
(c) 2 and 4 are correct (d) 1 and 3 are correct (a) 50 kg wt, 50 + kg wt
çè g ÷ø
Q.21 A mass of 60 kg is on the floor of a lift moving down. The
lift moves at first with an acceleration of 3 m/sec2, then æ 50g ö
(b) 50 kg wt, ç 50 + kg wt
with constant velocity and finally with a retardation of è a ÷ø
3m/sec2 . Choose the correct options related to possible
reactions exerted by the lift on the body in each part of the æ 50a ö
(c) 50 kg wt, ç kg wt
motion – è g ÷ø
(1) 408 N (2) 588 N (3) 768 N (4) 508 N æ 50g ö
Q.22 A mass of 10 kg is hung to a spring balance in lift. If the (d) 50 kg wt, ç ÷ kg wt
è a ø
lift is moving with an acceleration g/3 in upward &
Q.25 A 25 kg lift is supported by a cable. The acceleration of
downward directions, choose the correct options related
the lift when the tension in the cable is 175 N, will be -
to the reading of the spring balance.
(a) 2.8 m/s2 (b) 16.8 m/s2
(1) 13.3 kg (2) 6.67 kg (3) 32.6 kg (4) 0 2
(c) – 9.8 m/s (d) 14 m/s2

RESPONSE 17. 18. 19. 20. 21.


GRID 22. 23. 24. 25.

Space for Rough Work


t.me/Ebooks_Encyclopedia27. t.me/Magazines4all

EBD_7156
4 DPP/ P 09
Q.26 A body is suspended by a string from the ceiling of an (b) Statement-1 is True, Statement-2 is True; Statement-2 is NOT
elevator. It is observed that the tension in the string is a correct explanation for Statement-1.
doubled when the elevator is accelerated. The acceleration (c) Statement-1 is False, Statement-2 is True.
will be - (d) Statement-1 is True, Statement-2 is False.
(a) 4.9 m/s2 (b) 9.8 m/s2
(c) 19.6 m/s 2 (d) 2.45 m/s2 Q.27 Statement-1 : A cloth covers a table. Some dishes are kept
on it. The cloth can be pulled out without dislodging the
DIRECTIONS (Q. 27-Q.29) : Each of these questions contains dishes from the table.
two statements: Statement-1 (Assertion) and Statement-2 Statement-2 : To every action there is an equal and oppo-
(Reason). Each of these questions has four alternative choices, site reaction.
only one of which is the correct answer. You have to select the Q.28 Statement-1 : If the net external force on the body is zero
correct choice. then its acceleration is zero.
(a) Statement-1 is True, Statement-2 is True; Statement-2 is a Statement-2 : Acceleration does not depend on force.
correct explanation for Statement-1. Q.29 Statement-1 : The slope of momentum versus time graph
give us the acceleration.
Statement-2 : Force is given by the rate of change of
momentum.

RESPONSE GRID 26. 27. 28. 29.

DAILY PRACTICE PROBLEM SHEET 9 - PHYSICS


Total Questions 29 Total Marks 116
Attempted Correct
Incorrect N et Score
Cut-off Score 30 Qualifying Score 44
Success Gap = Net Score – Qualifying Score
Net Score = (Correct × 4) – (Incorrect × 1)

Space for Rough Work


t.me/Ebooks_Encyclopedia27. t.me/Magazines4all

DPP - Daily Practice Problems


Name : Date :

Start Time : End Time :

SYLLABUS : LAWS OF MOTION-2 (Blocks in contact, connected by string, pulley arrangement)


10
Max. Marks : 112 Time : 60 min.
GENERAL INSTRUCTIONS
• The Daily Practice Problem Sheet contains 28 MCQ's. For each question only one option is correct. Darken the correct
circle/ bubble in the Response Grid provided on each page.
• You have to evaluate your Response Grids yourself with the help of solution booklet.
• Each correct answer will get you 4 marks and 1 mark shall be deduced for each incorrect answer. No mark will be given/
deducted if no bubble is filled. Keep a timer in front of you and stop immediately at the end of 60 min.
• The sheet follows a particular syllabus. Do not attempt the sheet before you have completed your preparation for that
syllabus. Refer syllabus sheet in the starting of the book for the syllabus of all the DPP sheets.
• After completing the sheet check your answers with the solution booklet and complete the Result Grid. Finally spend time
to analyse your performance and revise the areas which emerge out as weak in your evaluation.

DIRECTIONS (Q.1-Q.20) : There are 20 multiple choice force of 25 N. The tension in the rope at the point 7 m
questions. Each question has 4 choices (a), (b), (c) and (d), out away from the point of application, will be -
of which ONLY ONE choice is correct. (a) 11.67 N (b) 13.33 N
(c) 36.67 N (d) None of these
Q.1 A block of mass M is pulled along a horizontal frictionless
Q.4 A force of 100 N acts in the direction as shown in figure
surface by a rope of mass m. If a force P is applied at the
on a block of mass 10 kg resting on a smooth horizontal
free end of the rope, the force exerted by the rope on the
table. The speed acquired by the block after it has .moved
block will be -
a distance of 10 m, will be -
Pm MP mP (in m/sec) (g = 10 m/sec2)
(a) P (b) (c) (d)
M+m M+m M+m (a) 17 m/sec 100N
Q.2 A body of mass 50 kg is pulled by a rope of length 8 m on (b) 13.17 m/sec 30°
a surface by a force of 108N applied at the other end. The 10kg
(c) 1.3 m/sec
force that is acting on 50 kg mass, if the linear density of (d) 1.7 m/sec
rope is 0.5 kg/m will be - Q.5 In the above example, the velocity after 2 sec will be -
(a) 108 N (b) 100 N (c) 116 N (d) 92 N (in m/sec)
Q.3 A rope of length 15 m and linear density 2 kg/m is lying
length wise on a horizontal smooth table. It is pulled by a (a) 10 3 (b) 5 3 (c) 10 (d) 5

RESPONSE GRID 1. 2. 3. 4. 5.
Space for Rough Work
t.me/Ebooks_Encyclopedia27. t.me/Magazines4all

EBD_7156
2 DPP/ P 10
Q.6 Two blocks of mass m = 1 kg and M = 2 kg are in contact on Q.11 The pulley arrangements of fig (a) and (b) are identical. The
a frictionless table. A horizontal force F(= 3N) is applied to mass of the rope is negligible. In (a) the mass m is lifted up
m. The force of contact between the blocks, will be- by attaching a mass 2 m to the other end of the rope. In (b)
(a) 2 N (b) 1 N (c) 4 N (d) 5 N m is lifted up by pulling the other end of the rope with a
Q.7 A force produces an acceleration of 5 m/s2 in a body and constant downward force F = 2 mg. Which of the following
same force an acceleration of 15 m/s2 in another body. is correct?
The acceleration produced by the same force when applied
to the combination of two bodies will be -
(a) 3.75 m/s2 (b) 20 m/s2
(c) 10 m/s 2 (d) 0.667 m/s2
Q.8 What is the tension in a rod of length L and mass M at a A m m
B 2m F=2mg
distance y from F1 when the rod is acted on by two unequal
forces F1 and F2 (<F1) as shown in fig (a) (b)
y (a) Acceleration in case (b) is 3 times more than that in
C B A
F2
case (a)
T T F1
(b) In case (a) acceleration is g, while in case (b) it is 2g
L (c) In both the cases, acceleration is same
æ yö æyö M æç F1 , F2 ö÷
F1 çç1 , ÷÷ ∗ F2 çç ÷÷
(d) None of the above
(a) (b) yç ÷
èç L ø÷ èç L ø÷ L çè M ø÷ Q.12 Three equal weights of mass m each are hanging on a string
passing over a fixed pulley as shown in fig. The tensions in
æ yö æ yö M æç F1 ∗ F2 ö÷
F1 çç1 ∗ ÷÷ ∗ F2 çç ÷÷ yç ÷ the string connecting weights A to B and B to C will
L çè M ø÷
(c) (d)
çè Lø÷ èç L ø÷ respectively be -
1 1 1 2 2
Q.9 A force produces acceleration 1 , , , , ......... (all in m/ (a) mg, mg
2 3 4 3 3
2 4
s2 ), applied separetly to n bodies. If these bodies are (b) mg , mg T1 T1
combined to form single one, then the acceleration of the 3 3
4 2 A
system will be, if same force is taken into account. (c) mg, mg
3 3 B T2
n 2 n2 n 2 (n + 1) 3
(a) (b) (c) (d) 3 C
2 n(n + 1) 2 2 (d) mg, mg
Q.10 Two blocks of masses 6 kg and 4 kg connected by a rope of 2 4
mass 2 kg are resting on frictionless floor as shown in fig. If a Q.13 In the situation shown in figure, both the pulleys and the
constant force of 60 N is applied to 6 kg block, tension in the strings are light and all the surfaces are frictionless. The
rope at A, B, and C will respectively be - acceleration of mass M, tension in the string PQ and force
exerted by the clamp on the pulley, will respectively be -
6kg
2kg 2kg F=60N
(a) (2/3)g, (1/3)Mg, ( 2 /3)Mg T’ Q T
CBA B 2M
Q P (b) (1/3)g, (1/3)Mg, ( 2 /3)Mg
T T
(c) (1/3)g, (2/3)Mg, 3 Mg 2Mg
(a) 30 N, 25 N, 20 N (b) 25 N, 30 N, 20 N
(d) 2g, (1/2)g, 2 Mg A M
(c) 20 N, 30 N, 25 N (d) 30 N, 20 N, 25 N

6. 7. 8. 9. 10.
RESPONSE 11. 12. 13.
GRID

Space for Rough Work


t.me/Ebooks_Encyclopedia27. t.me/Magazines4all

DPP/ P 10 3
Q.14 A body of mass 50kg resting on a smooth inclined plane is Q.19 Consider th e double Atwood’s
connected by a massless inextensible string passing over a machine as shown in the figure.
smooth pulley, at the top of the inclined plane have another What is acceleration of the masses ?
mass of 40 kg as shown in the figure. The distance through (a) g/3 T
m
T
a
which 50 kg mass fall in 4 sec will be - (b) g/2
(The angle of the inclined plane is 30º) (c) g mg

T (d) g/4 T
(a) 13.04 m T

T
Q.20 In above question, what is the a m m a
(b) 1.63m C tension in each string ? mg mg

(c) 1.304 m B (a) mg/3 (b) 4mg/3 (c) 2mg/3 (d) 5mg/3
mgcosq mgsinq
mg q
(d) 16.3m 40g DIRECTIONS (Q.21-Q.22) : In the following questions,
Q.15 A bob is hanging over a pulley inside a car through a string. more than one of the answers given are correct. Select the
The second end of the string is in the hand of a person correct answers and mark it according to the following codes:
standing in the car. The car is moving with constant
Codes :
acceleration 'a' directed horizontally as shown in figure.
(a) 1, 2 and 3 are correct (b) 1 and 2 are correct
Other end of the string is pulled with constant acceleration
(c) 2 and 4 are correct (d) 1 and 3 are correct
‘a’ vertically. The tension in the string is – Q.21 Choose the correct options –
car (1) Inertia µ mass (2) 1 newton = 105 dyne
(a) m g2 + a 2
a r DM r r
a (3) Thrust on rocket F = v - Mg
2
(b) m g + a - ma2 Dt
(4) Apparent weight of a body in the accelerated lift is
m
W = m (g + a).
(c) m g 2 + a 2 + ma
Q.22 Choose the correct statements –
(d) m (g + a) (1) For equilibrium of a body under the action of
® ® ® ®
Q.16 In the fig shown, the velocity of each concurrent forces F1 + F2 + F3 + ..... Fn = 0
particle at the end of 4 sec will be - (2) If the downward acceleration of the lift is a = g, then
(a) 0.872 m/s the body will experience weightlessness.
(b) 8.72 m/s (3) If the downward acceleration of the body is a > g,
T T
(c) 0.218 m/s a then the body will rise up to the ceiling of lift
(d) 2.18 m/s A a (4) If the downward acceleration of the lift is a > g, then
Q.17 In the above example, the height B the body will experience weightlessness.
ascended or descended, as the case may
11.5g DIRECTIONS (Q.23-Q.25) : Read the passage given below
be, during that time i.e. 4 sec will be -
(a) 1.744 m (b) 17.44 m and answer the questions that follows :
(c) 0.1744 m (d) None of these A bead of mass m is attached to one end of
B
Q.18 In the above question, if at the end of 4 sec, the string be a spring of natural length R and spring
cut, the position of each particle in next 2 seconds will ( 3 + 1) mg
constant K = . The other end of A
30°
respectively be - R
(a) 17.856 m, 21.344 m (b) –21.344 m, 17.856 m the spring is fixed at point A on a smooth
(c) –17.856 m, 21.344 m (d) –17.856 m, –21.344 m ring of radius R as shown in figure. When
bead is released to move then

RESPONSE 14. 15. 16. 17. 18.


GRID 19. 20. 21. 22.

Space for Rough Work


t.me/Ebooks_Encyclopedia27. t.me/Magazines4all

EBD_7156
4 DPP/ P 10
Q.23 Initial elongation in the spring is – Statement– 2 Acceleration of the both blocks will be
(a) R (b) 2R (c) 2R (d) 3R
different.
Q.24 The normal reaction force at B is –
mg 3 3mg F B
(a) (b) 3mg (c) 3 3mg (d) A
2 2
Q.25 Tangential acceleration of bead just after it is released.
Q.27 Statement– 1 Block A is moving on horizontal surface
g 3 g 2 towards right under the action of force F. All surfaces are
(a) (b) g (c) (d) g
2 4 4 3 smooth. At the instant shown the force exerted by block A
DIRECTIONS (Qs. 26-Q.28) : Each of these questions contains on block B is equal to net force on block B.
two statements: Statement-1 (Assertion) and Statement-2 Statement– 2 From Newton’s third law of motion, the
(Reason). Each of these questions has four alternative choices, force exerted by block A on B is equal in magnitude to
only one of which is the correct answer. You have to select the force exerted by block B on A.
correct choice. A
B
(a) Statement-1 is True, Statement-2 is True; Statement-2 is a F
correct explanation for Statement-1. Q.28 Statement– 1 : In the given fig.
(b) Statement-1 is True, Statement-2 is True; Statement-2 is
æ m , m1 ÷ö
NOT a correct explanation for Statement-1. a < ççç 2 ÷g
(c) Statement -1 is False, Statement-2 is True. çè m1 ∗ m 2 ÷÷ø a a
(d) Statement -1 is True, Statement-2 is False. Statement– 2 : In the given fig., T
T
Q.26 Statement–1 In fig the ground is smooth and the masses m ∗ m2
of both the blocks are different. Net force acting on each T< 1 g m1
2m1m 2 m2
of block is not same.

RESPONSE 23. 24. 25. 26. 27.


GRID 28.

DAILY PRA CTICE PROBLEM SHEET 10 - PHYSICS


Total Questions 28 Total Marks 112
Attempted Correct
Incorrect N et Score
Cut-off Score 28 Qualifying Score 42
Success Gap = Net Score – Qualifying Score
Net Score = (Correct × 4) – (Incorrect × 1)
Space for Rough Work
t.me/Ebooks_Encyclopedia27. t.me/Magazines4all

DPP - Daily Practice Problems


Name : Date :

Start Time : End Time :

SYLLABUS : LAWS OF MOTION-3 (Friction)


11
Max. Marks : 112 Time : 60 min.
GENERAL INSTRUCTIONS
• The Daily Practice Problem Sheet contains 28 MCQ's. For each question only one option is correct. Darken the correct
circle/ bubble in the Response Grid provided on each page.
• You have to evaluate your Response Grids yourself with the help of solution booklet.
• Each correct answer will get you 4 marks and 1 mark shall be deduced for each incorrect answer. No mark will be given/
deducted if no bubble is filled. Keep a timer in front of you and stop immediately at the end of 60 min.
• The sheet follows a particular syllabus. Do not attempt the sheet before you have completed your preparation for that
syllabus. Refer syllabus sheet in the starting of the book for the syllabus of all the DPP sheets.
• After completing the sheet check your answers with the solution booklet and complete the Result Grid. Finally spend time
to analyse your performance and revise the areas which emerge out as weak in your evaluation.

DIRECTIONS (Q.1-Q.20) : There are 20 multiple choice (a) tan–1 (0.1) (b) tan –1 (0.2)
–1
(c) tan (0.3) (d) tan –1 (0.4)
questions. Each question has 4 choices (a), (b), (c) and (d), out
of which ONLY ONE choice is correct. Q.4 The coefficient of static friction between the two blocks
shown in figure is m and the table is smooth. What maximum
Q.1 A body of mass 400 g slides on a rough horizontal surface.
horizontal force F can be applied to the block of mass M
If the frictional force is 3.0 N, the angle made by the contact
so that the blocks move together?
force on the body with the vertical will be
(a) 37º (b) 53º (c) 63º (d) 27º
m
Q.2 In the above question, the magnitude of the contact force
is
(g = 10 m/s2) M F
(a) 3.0 N (b) 4.0 N (c) 5.0 N (d) 7.0 N
Q.3 The coefficient of static friction between a block of mass
m and an inclined plane is ms = 0.3. What can be the (a) mg (M + m) (b) mg (M – m)
maximum angle q of the inclined plane with the horizontal (c) 2mg (M + m) (d) mg (M + 2m)
so that the block does not slip on the plane?

RESPONSE GRID 1. 2. 3. 4.
Space for Rough Work
t.me/Ebooks_Encyclopedia27. t.me/Magazines4all

EBD_7156
2 DPP/ P 11
Q.5 Block A weighs 4 N and block B weighs 8 N. The coefficient of é ù é ù
1 1
kinetic friction is 0.25 for all surfaces. Find the force F to slide B at (a) Rêê1 + ú (b) R ê1 - ú
a constant speed when A rests on B and moves with it. ë (m 2 + 1) úû ê (m2 - 1) úû
ë
(a) 2N (b) 3N (c) 1N (d) 5N é
2
ù é ù
1
Q.6 In the above question, find the force F to slide B at a (c) R êê1 - ú (d) R ê1 - ú
ë (m 2 + 1) úû ê (m + 1) úû
2
constant speed when A is held at rest. ë
Q.12 A body of mass m is released from the top of a rough
(a) 2N (b) 3N (c) 1N (d) 4N
inclined plane as shown in figure. If the frictional force be
Q.7 In the above question, find the force F to slide B at a
F, then body will reach the bottom with a velocity
constant speed when A and B are connected by a light cord
passing over a smooth Pulley. m
(a) 2N (b) 3N (c) 1N (d) 5N
Q.8 Find the maximum value of M/m in the situation shown in
L h
figure so that the system remains at rest. Friction
coefficient at both the contacts is m.
m

2 1
M (a) (mgh - FL) (b) (mgh - FL)
m m
q 2
(c) (mgh + FL) (d) None of these
m 2m m
(a) sin q - m cos q
(b) sin q - m cos q Q.13 A block of mass 2 kg is placed on the floor. The coefficient
m m of static friction is 0.4.A force F of 2.5 N is applied on the
(c) sin q + m cos q
(d) cos q - m sin q block, as shown. Calculate the force of friction between
Q.9 A block placed on a horizontal surface is being pushed by the block and the floor. (g = 9.8 ms–2)
a force F making an angle q with the vertical, if the (a) 2.5 N (b) 25 N (c) 7.84 N (d) zero
coefficient of friction is m, how much force is needed to Q.14 A block is kept on a horizontal table. The table is undergoing
get the block just started? simple harmonic motion of frequency 3 Hz in a horizontal
m 2m plane. The coefficient of static friction between the block and
(a) sin q - m cos q
(b) sin q - m cos q the table surface is 0.72. Find the maximum amplitude of the
m m table at which the block does not slip on the surface (g = 10 ms–
(c) sin q + m cos q (d) cos q - m sin q 2)

Q.10 Assuming the length of a chain to be L and coefficient of (a) 0.01 m (b) 0.02 m (c) 0.03 m (d) 0.04 m
static friction m. Compute the maximum length of the chain Q.15 Two cars of unequal masses use similar tyres. If they are
which can be held outside a table without sliding. moving at the same initial speed, the minimum stopping
2m L mL mL 3mL distance -
(a) 1 + m (b) 1 - m (c) 1 + m (d) 1 + m (a) is smaller for the heavier car
Q.11 If the coefficient of friction between an insect and bowl is (b) is smaller for the lighter car
m and the radius of the bowl is r, find the maximum height (c) is same for both cars
to which the insect can crawl in the bowl. (d) depends on the volume of the car

5. 6. 7. 8. 9.
RESPONSE 10. 11. 12. 13. 14.
GRID
15.

Space for Rough Work


t.me/Ebooks_Encyclopedia27. t.me/Magazines4all

DPP/ P 11 3
Q.16 Consider the situation shown in figure. The wall is smooth DIRECTIONS (Q.21-Q.23) : In the following questions, more
but the surfaces of A and B in contact are rough in than one of the answers given are correct. Select the correct
equilibrium the friction on B due to A – answers and mark it according to the following codes:
(a) is upward Codes :
A B F
(b) is downward (a) 1, 2 and 3 are correct (b) 1 and 2 are correct
(c) is zero (c) 2 and 4 are correct (d) 1 and 3 are correct
(d) the system cannot remain in equilibrium Q.21 Choose the correct statements –
Q.17 A block is placed on a rough floor and a horizontal force F (1) Kinetic friction is lesser than limiting friction.
is applied on it. The force of friction f by the floor on the
(2) In rolling the surfaces at contact do not rub each other.
block is measured for different values of F and a graph is
(3) If a body is at rest and no pulling force is acting on it,
plotted between them –
force of friction on it is zero.
(i) The graph is a straight line of slope 45°
(4) Kinetic friction is greater than limiting friction.
(ii) The graph is straight line parallel to the F axis
Q.22 Choose the correct statements –
(iii) The graph is a straight line of slope 45º for small F and
(1) Force of friction is partically independent of
a straight line parallel to the F-axis for large F.
microscopic area of surface in contact and relative
(iv) There is small kink on the graph
velocity between them. (if it is not high)
(a) iii, iv (b) i, iv (c) i, ii (d) i, iii
(2) Normally with increase in smoothness friction
Q.18 The contact force exerted by a body A on another body B decreases. But if the surface area are made too smooth
is equal to the normal force between the bodies. We by polishing and cleaning the bonding force of
conclude that - adhesion will increase and so the friction will increase
(i) the surfaces must be smooth resulting in 'Cold welding'
(ii) force of friction between two bodies may be equal to (3) Friction is a non conservative force, i.e. work done
zero against friction is path dependent.
(iii) magnitude of normal reaction is equal to that of (4) Force of fricton depends on area
friction Q.23 Choose the correct options –
(iv) bodies may be rough (1) Friction always opposes the motion
(a) ii, iv (b) i, ii (c) iii, iv (d) i, iv (2) Friction may opposes the motion
Q.19 It is easier to pull a body than to push, because - (3) If the applied force is increased the force of static
(a) the coefficient of friction is more in pushing than that friction remains constant.
in pulling (4) If the applied force is increased the force of static
(b) the friction force is more in pushing than that in friction also increases upto limiting friction.
pulling
(c) the body does not move forward when pushed DIRECTIONS (Q.24-Q.26) : Read the passage given below and
(d) None of these answer the questions that follows :
Q.20 A block of metal is lying on the floor of a bus. The
maximum acceleration which can be given to the bus so A block of mass 1 kg is placed on a rough horizontal surface. A
that the block may remain at rest, will be - spring is attached to the block whose other end is joined to a rigid
(a) µg (b) µ/g (c) µ2g (d) µg2 wall, as shown in the figure. A horizontal force is applied on the
block so that it remains at rest while the spring is elongated by x.

RESPONSE 16. 17. 18. 19. 20.


GRID 21. 22. 23.

Space for Rough Work


t.me/Ebooks_Encyclopedia27. t.me/Magazines4all

EBD_7156
4 DPP/ P 11
mmg DIRECTIONS (Q. 27-Q.28) : Each of these questions contains
x³ . Let Fmax and Fmin be the maximum and minimum values two statements: Statement-1 (Assertion) and Statement-2
k
of force F for which the block remains a equilibrium. For a particular (Reason). Each of these questions has four alternative choices, only
x, one of which is the correct answer. You have to select the correct
choice.
Fmax – Fmin = 2N.
(a) Statement-1 is True, Statement-2 is True; Statement-2 is a
Also shown is the variation of Fmax + Fmin versus x, the elonga-
correct explanation for Statement-1.
tion of the spring.
(b) Statement-1 is True, Statement-2 is True; Statement-2 is
NOT a correct explanation for Statement-1.
Fmax+Fmin (c) Statement -1 is False, Statement-2 is True.
///////////////

5N
K (d) Statement -1 is True, Statement-2 is False.
1 kg Q.27 A solid sphere and a hollow sphere of same mass M and
////////////////////////////////////////// same radius R are released from the top of a rough
0.1M
x inclined plane. Friction coefficient is same for both the
Q.24 The coefficient of friction between the block and the hori- bodies. If both bodies perform imperfect rolling, then
zontal surface is – Statement - 1 : Work done by friction for the motion of
(a) 0.1 (b) 0.2 bodies from top of incline to the bottom will be same
(c) 0.3 (d) 0.4 for both the bodies.
Q.25 The spring constant of the spring is – Statement - 2 : Force of friction will be same for both
the bodies.
(a) 25 N/m (b) 20 N/m
(c) 2.5 N/m (d) 50 N/m Q.28 Statement - 1 : Maximum value of friction force between
two surfaces is m × normal reaction.
Q.26 The value of Fmin, if x = 3 cm. is –
where m = coefficient of friction between surfaces.
(a) 0 (b) 0.2 N
Statement - 2 : Friction force between surfaces of two
(c) 5N (d) 1N
bodies is always less than or equal to externally applied
force.

RESPONSE GRID 24. 25. 26. 27. 28.

DAILY PRA CTICE PROBLEM SHEET 11 - PHYSICS


Total Questions 28 Total Marks 112
Attempted Correct
Incorrect N et Score
Cut-off Score 26 Qualifying Score 42
Success Gap = Net Score – Qualifying Score
Net Score = (Correct × 4) – (Incorrect × 1)

Space for Rough Work


t.me/Ebooks_Encyclopedia27. t.me/Magazines4all

DPP - Daily Practice Problems


Name : Date :

Start Time : End Time :

SYLLABUS : Work, Energy and Power-1 (Work by constant and variable forces,
12
kinetic and potential energy, work energy theorem)
Max. Marks : 112 Time : 60 min.
GENERAL INSTRUCTIONS
• The Daily Practice Problem Sheet contains 28 MCQ's. For each question only one option is correct. Darken the correct
circle/ bubble in the Response Grid provided on each page.
• You have to evaluate your Response Grids yourself with the help of solution booklet.
• Each correct answer will get you 4 marks and 1 mark shall be deduced for each incorrect answer. No mark will be given/
deducted if no bubble is filled. Keep a timer in front of you and stop immediately at the end of 60 min.
• The sheet follows a particular syllabus. Do not attempt the sheet before you have completed your preparation for that
syllabus. Refer syllabus sheet in the starting of the book for the syllabus of all the DPP sheets.
• After completing the sheet check your answers with the solution booklet and complete the Result Grid. Finally spend time
to analyse your performance and revise the areas which emerge out as weak in your evaluation.
r
DIRECTIONS (Q.1-Q.20) : There are 20 multiple choice Q.3 A force F = (7 – 2x + 3x2) N is applied on a 2 kg mass
questions. Each question has 4 choices (a), (b), (c) and (d), out which displaces it from x = 0 to x = 5 m. Work done in
of which ONLY ONE choice is correct. joule is -
r (a) 70 (b) 270 (c) 35 (d) 135
Q.1 A body is acted upon by a force F = -ˆi + 2jˆ + 3kˆ . The work
Q.4 An automobile of mass m accelerates from rest. If the engine
done by the force in displacing it from (0,0,0) to (0,0,4m) will supplies a constant power P, the velocity at time t is given by -
be -
(a) 12 J (b) 10 J Pt 2Pt Pt 2Pt
(a) v = (b) v = (c) (d)
(c) 8 J (d) 6 J m m m m
Q.2 The work done in pulling a body of mass 5 kg along an inclined Q.5 In the above question, the position (s) at time (t) is given by -
1/2
plane (angle 60º) with coefficient of friction 0.2 through 2 æ 2Ptö æ 8P ö
(a) çè m ÷ø t (b) çè ÷ø t 3/2
m, will be - 9m
(a) 98.08 J (b) 94.08 J æ 9P ö
1/2
æ 8P ö
1/ 2
(c) 90.08 J (d) 91.08 J (c) ç ÷ t1/2 (d) ç ÷ t
è 8m ø è 9m ø

RESPONSE GRID 1. 2. 3. 4. 5.
Space for Rough Work
t.me/Ebooks_Encyclopedia27. t.me/Magazines4all

EBD_7156
2 DPP/ P 12
Q.6 A particle moving in a straight line is acted by a force, which Q.13 The power output of a 92 U235 reactor if it takes 30 days to
works at a constant rate and changes its velocity from u to use up 2 kg of fuel and if each fission gives 185 MeV of
v in passing over a distance x. The time taken will be - energy (Avogadro number = 6 × 1023/mole) will be -
v-u æ v+u ö (a) 58.4 MW (b) 5.84 MW
(a) x = 2 2 (b) x çè 2 2 ø÷ (c) 584 W (d) 5840 MW
v +u v +u
æ v2 - u 2 ö
Q.14 The stopping distance for a vehicle of mass M moving with
3 æ vö a speed v along a level road, will be -
(c) (x) ç 3 3 ÷ (d) x çè u ÷ø
2 è v -u ø (µ is the coefficient of friction between tyres and the road)
Q.7 A chain of linear density 3 kg /m and length 8 m is lying on
the table with 4 m of chain hanging from the edge. The v2 2v2 v2 v
work done in lifting the chain on the table will be - (a) (b) (c) (d)
mg mg 2mg mg
(a) 117.6 J (b) 235.2 J (c) 98 J (d) 196 J
Q.15 The earth circles the sun once a year. How much work
Q.8 The work done in lifting water from a well of depth 6 m
would have to be done on the earth to bring it to rest
using a bucket of mass 0.5 kg and volume 2 litre, will be-
relative to the sun, (ignore the rotation of earth about -
(a) 73.5 J (b) 147 J (c) 117. 6 J (d) 98 J
its own axis) Given that mass of the earth is 6 × 1024 kg
Q.9 An object of mass 5 kg falls from rest through a vertical
and distance between the sun and earth is 1.5 × 108 km-
distance of 20 m and reaches a velocity of 10 m/s. How
(a) 2.7 × 1033 (b) 2.7 × 1024
much work is done by the push of the air on the object ? 23
(c) 1.9 × 10 (d) 1.9 × 1024
(g = 10 m/s2).
Q.16 A particle of mass m is moving in a horizontal circle of
(a) 350 J (b) 750 J (c) 200 J (d) 300 J
radius r, under a centripetal force equal to (–k/r 2), where k
Q.10 A boy pulls a 5 kg block 20 metres along a horizontal sur-
is a constant. The total energy of the particle is -
face at a constant speed with a force directed 45° above
(a) k/2r (b) – k/2r
the horizontal. If the coefficient of kinetic friction is 0.20,
(c) kr (d) –k/r
how much work does the boy do on the block?
Q.17 The work done by a person in carrying a box of mass 10 kg
(a) 163.32 J (b) 11.55 J
through a vertical height of 10 m is 4900 J. The mass of
(c) 150 J (d) 115 J
the person is -
Q.11 A uniform chain is held on a frictionless table with one-
(a) 60 kg (b) 50 kg
fifth of its length hanging over the edge. If the chain has a
(c) 40 kg (d) 130 kg
length l and a mass m, how much work is required to pull
Q.18 A uniform rod of length 4 m and mass 20 kg is lying
the hanging part back on the table ?
horizontal on the ground. The work done in keeping it
(a) mg l / 10 (b) mg l / 5
vertical with one of its ends touching the ground, will be -
(c) mg l / 50 (d) mg l / 2
(a) 784 J (b) 392 J (c) 196 J (d) 98 J
Q.12 A bus of mass 1000 kg has an engine which produces a
Q.19 If g is the acceleration due to gravity on the earth’s surface,
constant power of 50 kW. If the resistance to motion,
the gain in the potential energy of an object of mass m
assumed constant is 1000 N. The maximum speed at which
raised from surface of the earth to a height equal to radius
the bus can travel on level road and the acceleration when
R of the earth is - [M = mass of earth]
it is travelling at 25 m/s, will respectively be -
(a) 50 m/s, 1.0 m/s2 (b) 1.0 m/s, 50 m/s2 GMm GM GMm GM
2 (a) (b) (c) (d)
(c) 5.0 m/s, 10 m/s (d) 10 m/s, 5m/s2 2R R R 2R

6. 7. 8. 9. 10.
RESPONSE 11. 12. 13. 14. 15.
GRID
16. 17. 18. 19.

Space for Rough Work


t.me/Ebooks_Encyclopedia27. t.me/Magazines4all

DPP/ P 12 3
Q.20 The potential energy between two atoms in a molecule is given DIRECTIONS (Q.23-Q.25) : Read the passage given below
a b and answer the questions that follows :
by, U(x) = - , where a and b are positive constant and x
x12 x 6 In the figure shown, the system is released from rest with both
is the distance between the atoms. The atoms is an stable the springs in unstretched positions. Mass of each block is 1 kg
equilibrium, when- and force constant of each spring is 10 N/m.
1/6
æ a ö
(a) x=0 (b) x = ç ÷
è 2b ø
1/6 1/6
æ 2a ö æ 11a ö
(c) x= ç ÷ (d) x = ç
è bø è 5b ÷ø
DIRECTIONS (Q.21-Q.22) : In the following questions,
more than one of the answers given are correct. Select the
correct answers and mark it according to the following
codes:
Codes :
Q.23 Extension of horizontal spring in equilibrium is:
(a) 1, 2 and 3 are correct (a) 0.2 m (b) 0.4 m
(b) 1 and 2 are correct (c) 0.6 m (d) 0.8 m
(c) 2 and 4 are correct Q.24 Extension of vertical spring in equilibrium is
(a) 0.4 m (b) 0.2 m
(d) 1 and 3 are correct
(c) 0.6 m (d) 0.8 m
Q.21 A man pushes a wall and fails to displace it. Choose Q.25 Maximum speed of the block placed horizontally is:
incorrect statements related to his work (a) 3.21 m/s (b) 2.21 m/s
(1) Negative work (c) 1.93 m/s (d) 1.26 m/s
(2) Positive but not maximum work DIRECTIONS (Qs. 26-Q.28) : Each of these questions contains
(3) Maximum work two statements: Statement-1 (Assertion) and Statement-2
(4) No work at all (Reason). Each of these questions has four alternative choices,
only one of which is the correct answer. You have to select the
Q.22 Choose the correct options –
correct choice.
(1) The work done by forces may be equal to change in
kinetic energy (a) Statement-1 is True, Statement-2 is True; Statement-2 is a
(2) The work done by forces may be equal to change in correct explanation for Statement-1.
potential energy (b) Statement-1 is True, Statement-2 is True; Statement-2 is
(3) The work done by forces may be equal to change in NOT a correct explanation for Statement-1.
total energy (c) Statement -1 is False, Statement-2 is True.
(4) The work done by forces must be equal to change in (d) Statement -1 is True, Statement-2 is False.
potential energy.

RESPONSE 20. 21. 22. 23. 24.


GRID 25.

Space for Rough Work


t.me/Ebooks_Encyclopedia27. t.me/Magazines4all

EBD_7156
4 DPP/ P 12
Q.26 As shown in the figure, a uniform sphere is rolling on a Q.27 Statement - 1 : Sum of work done by the Newton’s 3rd law
horizontal surface without slipping, under the action of a pair internal forces, acting between two particles may be
horizontal force F. zero.
Statement - 2 : If two particles undergo same displacement
F
then work done by Newton’s 3rd law pair forces on them is
of opposite sign and equal magnitude.
Q.28 Statement - 1: A particle moves along a straight line with
constant velocity. Now a constant non-zero force is applied
Statement - 1 : Power developed due to friction force is on the particle in direction opposite to its initial velocity.
zero. After the force is applied, the net work done by this force
Statement - 2 : Power developed by gravity force is non- may be zero in certain time intervals.
zero. Statement - 2 : The work done by a force acting on a particle
is zero in any time interval if the force is always
perpendicular to velocity of the particle.

RESPONSE GRID 26. 27. 28.

DAILY PRA CTICE PROBLEM SHEET 12 - PHYSICS


Total Questions 28 Total Marks 112
Attempted Correct
Incorrect N et Score
Cut-off Score 28 Qualifying Score 44
Success Gap = Net Score – Qualifying Score
Net Score = (Correct × 4) – (Incorrect × 1)

Space for Rough Work


t.me/Ebooks_Encyclopedia27. t.me/Magazines4all

DPP - Daily Practice Problems


Name : Date :

Start Time : End Time :

SYLLABUS : Work, Energy and Power-2 (Conservation of momentum and energy, collision, rocket case)
13
Max. Marks : 112 Time : 60 min.
GENERAL INSTRUCTIONS
• The Daily Practice Problem Sheet contains 28 MCQ's. For each question only one option is correct. Darken the correct
circle/ bubble in the Response Grid provided on each page.
• You have to evaluate your Response Grids yourself with the help of solution booklet.
• Each correct answer will get you 4 marks and 1 mark shall be deduced for each incorrect answer. No mark will be given/
deducted if no bubble is filled. Keep a timer in front of you and stop immediately at the end of 60 min.
• The sheet follows a particular syllabus. Do not attempt the sheet before you have completed your preparation for that
syllabus. Refer syllabus sheet in the starting of the book for the syllabus of all the DPP sheets.
• After completing the sheet check your answers with the solution booklet and complete the Result Grid. Finally spend time
to analyse your performance and revise the areas which emerge out as weak in your evaluation.

DIRECTIONS (Q.1-Q.20) : There are 20 multiple choice travelled a distance of 15 m. The coefficient of friction
questions. Each question has 4 choices (a), (b), (c) and (d), out between the block and the floor will be - (Duration of
of which ONLY ONE choice is correct. impact is very short)
1 2 1 3
Q.1 A rifle man, who together with his rifle has a mass of 100 (a) (b) (c) (d)
2 3 3 4
kg, stands on a smooth surface fires 10 shots horizontally. Q.3 A 20 g bullet pierces through a plate of mass m1 = 1 kg and
Each bullet has a mass 10 gm and a muzzle velocity of 800 then comes to rest inside a second plate of mass m 2 = 2.98
m/s. What velocity does rifle man acquire at the end of 10 kg. It is found that the two plates, initially at rest, now move
shots with equal velocities. The percentage loss in the initial
(a) 0.8 m/s (b) 0.5 m/s (c) 0.3 m/s (d) 1.2 m/s velocity of bullet when it is between m1 and m2. (Neglect
Q.2 A bullet of mass 10 g travelling horizontally with a velocity of any loss of material of the bodies, due to action of bullet.)
300 m/s strikes a block of wood of mass 290 g which rests will be -
on a rough horizontal floor. After impact the block and the (a) 20% (b) 25%
bullet move together and come to rest when the block has (c) 30% (d) 45%

RESPONSE GRID 1. 2. 3.
Space for Rough Work
t.me/Ebooks_Encyclopedia27. t.me/Magazines4all

EBD_7156
2 DPP/ P 13
Q.4 A bullet of mass 20 g hits a block of mass 1.98 kg suspended Q.9 The mass of a rocket is 500 kg and the relative velocity of
from a massless string of length 100 cm and sticks to it. the gases ejecting from it is 250 m/s with respect to the
The bullet flies down at an angle of 30º to the horizontal rocket. The rate of burning of the fuel in order to give the
with a velocity of 200 m/s. Through what height the block rocket an initial acceleration 20 m/s2 in the vertically
will rise- upward direction (g = 10 m/s2), will be -
(a) 0.15 m (a) 30 kg/s (b) 60 kg/s
(c) 45 kg/s (d) 10 kg/s
(b) 0.30 m v Q.10 A slow moving electron collides elastically with a hydrogen
(M+m)
(c) 0.45 m atom at rest. The initial and final motions are along the
30° h
same straight line. What fraction of electron's kinetic
(d) 0.75 m M
energy is transferred to the hydrogen atom? The mass of
Q.5 A bullet of mass 0.01 kg travelling at a speed of 500 m/s
hydrogen atom is 1850 times the mass of electron.
strikes a block of mass 2 kg, which is suspended by a string
(a) 0.217 % (b) 2.17 % (c) 0.0217 % (d) 21.7 %
of length 5 m. The centre of gravity of the block is found to
rise a vertical distance of 0.1 m. The speed of the bullet Q.11 A particle of mass 4 m which is at rest explodes into three
after it emerges from the block will be - fragments, two of the fragments each of mass m are found
to move each with a speed v making an angle 90º with each
(a) 1.4 m/s other. The total energy relased in this explosion is -

(b) 110 m/s 1


(a) mv2 (b) mv2
2
(c) 220 m/s v1 3
u1
(c) mv2 (d) 2 mv2
(d) 14 m/s 2
0.1m v2
m1 m
2
Q.12 A body of mass M splits into two parts aM and (1 – a) M
Q.6 The rate of burning of fuel in a rocket is 50 gm/sec. and by an internal explosion, which generates kinetic energy T.
comes out with and velocity 4 × 103 m/s. The force exerted After explosion if the two parts move in the same direction
by gas on rocket will be - as before, their relative speed will be -
(a) 200 N (b) 250 N
T 2T
(c) 2.5 × 106 N (d) 2.5 × 104 N (a) (b)
Q.7 A body of mass 1 kg strikes elastically with another body at (1 - a )M a(1 - a)M
rest and continues to move in the same direction with one T 2T
fourth of its initial velocity. The mass of the other body is - (c) (d)
2(1 - a )M (1 - a )M
(a) 0.6 kg (b) 2.4 kg (c) 3 kg (d) 4 kg
Q.8 A ball moving with a speed of 9 m/s strikes with an identical Q.13 A body of mass 1 kg initially at rest explodes and breaks
stationary ball such that after the collision the direction of into three fragments of masses in the ratio 1 : 1 : 3. The
each ball makes an angle of 30° with the original line of two pieces of equal mass fly off perpendicular to each other
motion. Find the speeds of the two balls after the collision. with a speed of 30 m/sec each. What is the velocity of the
Is the kinetic energy conserved in this collision process ? heavier fragment ?
(a) 3 3 m/s, no (b) 3 3 m/s, no (a) 10 2 m/s (b) 15 2 m/s
(c) 6 3 m/s, yes (d) 0, yes (c) 5 2 m/s (d) 20 2 m/s

RESPONSE 4. 5. 6. 7. 8.
GRID 9. 10. 11. 12. 13.

Space for Rough Work


t.me/Ebooks_Encyclopedia27. t.me/Magazines4all

DPP/ P 13 3
Q.14 A body of mass m moving with a velocity v1 in the X- Q.19 In an inelastic collision-
direction collides with another body of mass M moving in (a) momentum is conserved but kinetic energy is not
Y-direction with a velocity v2. They coalasce into one body conserved
during collision. The magnitude and direction of the (b) momentum is not conserved but kinetic energy is
momentum of the final body, will be- conserved
æ Mv 2 ö
(c) neither momentum nor kinetic energy is conserved
(a) (mv1 ) + (Mv2 ) , tan–1 çè mv ÷ø (d) both the momentum and kinetic energy are conserved
1 Q.20 Inelastic collision is the-
æ Mv1 ö (a) collision of ideal gas molecules with the walls of the
(b) (mv1 ) + (Mv2 ) , tan–1 çè mv ÷ø
2 container
æ Mv 2 ö (b) collision of electron and positron to an inhilate each
(c) (mv1 ) 2 + (Mv 2 ) 2 , tan–1 çè mv ÷ø other.
1
æ Mv1 ö (c) collision of two rigid solid spheres lying on a
(d) (mv1 ) 2 + (Mv 2 ) 2 , tan–1 çè mv ÷ø frictionless table
2 (d) scattering of a-particles with the nucleus of gold atom
Q.15 A ball of mass m hits a wall with a speed v making an angle
q with the normal. If the coefficient of restitution is e, the DIRECTIONS (Q.21-Q.23) : In the following questions,
direction and magnitude of the velocity of ball after more than one of the answers given are correct. Select the
reflection from the wall will respectively be - correct answers and mark it according to the following
æ tan q ö
codes:
(a) tan–1 çè e ÷ø , v sin 2 q + e 2 cos 2 q Codes :
æ e ö 1 (a) 1, 2 and 3 are correct (b) 1 and 2 are correct
(b) tan–1 çè tan q ÷ø , 2 2 2 (c) 2 and 4 are correct (d) 1 and 3 are correct
v e sin q + cos q Q.21 Which of the following statements is false for collisions-
v
(c) tan–1(e tan q), tan q (1) Momentum is conserved in elastic collisions but not
e in inelastic collisions.
(d) tan–1 (e tan q), v sin 2 q + e2 (2) Total-kinetic energy is conserved in elastic collisions
Q.16 A tennis ball dropped from a height of 2 m rebounds only 1.5 but momentum is not conserved.
metre after hitting the ground. What fraction of energy is lost (3) Total kinetic energy and momentum both are conserved
in the impact? in all types of collisions
(a) 1/2 (b) 1/4 (c) 1/8 (d) 1/16 (4) Total kinetic energy is not conserved in inelastic
Q.17 A bullet is fired from the gun. The gun recoils, the kinetic collisions but momentum is conserved
energy of the recoil shall be- Q.22 Which of the following hold when two particles of masses
m1 and m2 undergo elastic collision?
(a) equal to the kinetic energy of the bullet (1) When m1 = m2 and m2 is stationary, there is maximum
(b) less than the kinetic energy of the bullet transfer of kinetic energy in head on collision
(c) greater than the kinetic energy of the bullet (2) When m1 = m2 and m2 is stationary, there is maximum
(d) double that of the kinetic energy of the bullet transfer of momentum in head on collision
Q.18 Conservation of linear momentum is equivalent to- (3) When m1 >> m2 and m2 is stationary, after head on
(a) Newton's second law of motion collision m2 moves with twice the velocity of m1.
(b) Newton's first law of motion (4) When the collision is oblique and m1 = m2 with m2
(c) Newton's third law of motion stationary, after the collision the particle move in
(d) Conservation of angular momentum. opposite directions.

RESPONSE 14. 15. 16. 17. 18.


GRID 19. 20. 21. 22.

Space for Rough Work


t.me/Ebooks_Encyclopedia27. t.me/Magazines4all

EBD_7156
4 DPP/ P 13
Q.23 Two balls at the same temperature collide inelastically. DIRECTIONS (Q. 27-Q.28) : Each of these questions contains
Which of the following is not conserved? two statements: Statement-1 (Assertion) and Statement-2
(1) Kinetic energy (2) Velocity (Reason). Each of these questions has four alternative choices,
(3) Temperature (4) Momentum only one of which is the correct answer. You have to select the
DIRECTIONS (Q.24-Q.26) : Read the passage given below correct choice.
and answer the questions that follows : (a) Statement-1 is True, Statement-2 is True; Statement-2 is a
correct explanation for Statement-1.
A small particle of mass m/10 is moving horizontally at a height (b) Statement-1 is True, Statement-2 is True; Statement-2 is
of 3R/2 from ground with velocity 10 m/s. A perfectly inelastic NOT a correct explanation for Statement-1.
collision occurs at point P of sphere of mass m placed on smooth (c) Statement -1 is False, Statement-2 is True.
horizontal surface. The radius of sphere is R. (m = 10 kg and R = (d) Statement -1 is True, Statement-2 is False.
0.1 m) (Assume all surfaces to be smooth). Q.27 A particle of mass m strikes a wedge of mass M horizontally
as shown in the figure.
m/10 P
m
10 m/s R/2
m M
R
B Statement - 1 : If collision is perfectly inelastic then, it
can be concluded that the particle sticks to the wedge.
Q.24 Speed of particle just after collision is Statement - 2 : In perfectly inelastic collision velocity of
(a) approx 5.0 m/s (b) approx 10 m/s both bodies is same along common normal just after
(c) approx. 15.0 m/s (d) approx 20.0 m/s collision.
Q.25 Speed of sphere just after collision is Q.28 Statement - 1 : In an elastic collision in one dimension
(a) 27/43 m/s (b) 30/43 m/s between two bodies, total momentum remains the same
(c) 35/43 m/s (d) 40/43 m/s before, during and after the collision.
Q.26 Angular speed of sphere just after collision is Statement - 2 : In an elastic collision in one dimension
(a) zero (b) 2 rad/sec between two bodies, total kinetic energy remains the same
(c) 2.5 rad/sec (d) 3 rad/sec before, during and after the collision.
[Assume external forces are absent in both the above
statements].

RESPONSE 23. 24. 25. 26. 27.


GRID 28.

DAILY PRA CTICE PROBLEM SHEET 13 - PHYSICS


Total Questions 28 Total Marks 112
Attempted Correct
Incorrect N et Score
Cut-off Score 28 Qualifying Score 44
Success Gap = Net Score – Qualifying Score
Net Score = (Correct × 4) – (Incorrect × 1)
Space for Rough Work
t.me/Ebooks_Encyclopedia27. t.me/Magazines4all

DPP - Daily Practice Problems


Name : Date :

Start Time : End Time :

SYLLABUS : Centre of mass and its motion


14
Max. Marks : 112 Time : 60 min.
GENERAL INSTRUCTIONS
• The Daily Practice Problem Sheet contains 28 MCQ's. For each question only one option is correct. Darken the correct
circle/ bubble in the Response Grid provided on each page.
• You have to evaluate your Response Grids yourself with the help of solution booklet.
• Each correct answer will get you 4 marks and 1 mark shall be deduced for each incorrect answer. No mark will be given/
deducted if no bubble is filled. Keep a timer in front of you and stop immediately at the end of 60 min.
• The sheet follows a particular syllabus. Do not attempt the sheet before you have completed your preparation for that
syllabus. Refer syllabus sheet in the starting of the book for the syllabus of all the DPP sheets.
• After completing the sheet check your answers with the solution booklet and complete the Result Grid. Finally spend time
to analyse your performance and revise the areas which emerge out as weak in your evaluation.

DIRECTIONS (Q.1-Q.20) : There are 20 multiple choice Q.3 The distance between the carbon atom and the oxygen atom
questions. Each question has 4 choices (a), (b), (c) and (d), out in a carbon monoxide molecule is 1.1 Å. Given, mass of
of which ONLY ONE choice is correct. carbon atom is 12 a.m.u. and mass of oxygen atom is 16
a.m.u., calculate the position of the centre of mass of the
Q.1 In the HCl molecule, the separation between the nuclei of
carbon monoxide molecule
the two atom is about 1.27 A° (1A° = 10 –10 m). The
(a) 6.3 Å from the carbon atom
approximate location of the centre of mass from the
(b) 1 Å from the oxygen atom
hydrogen atom, assuming the chlorine atom to be about
(c) 0.63 Å from the carbon atom
35.5 times massive as hydrogen is
(d) 0.12 Å from the oxygen atom
(a) 1 Å (b) 2.5 Å (c) 1.24 Å (d) 1.5 Å
Q.4 The velocities of three particles of masses 20g, 30g and
Q.2 A 2 kg body and a 3 kg body are moving along the x-axis. At
a particular instant the 2 kg body has a velocity of 3 ms–1 and the 50g are 10iˆ,10 ˆj and10kˆ respectively. The velocity of the
3 kg body has the velocity of 2ms–1. The velocity of the centre of mass of the three particles is
centre of mass at that instant is (a) 2iˆ + 3 ˆj + 5kˆ (b) 10(iˆ + ˆj + kˆ)
(a) 5 ms–1 (b) 1 ms–1
(c) 0 (d) None of these (c) 20iˆ + 30 ˆj +5 kˆ (d) 2iˆ + 30 ˆj +50 kˆ

RESPONSE GRID 1. 2. 3. 4.
Space for Rough Work
t.me/Ebooks_Encyclopedia27. t.me/Magazines4all

EBD_7156
2 DPP/ P 14
Q.5 The centre of mass of a triangle shown in figure has Q.10 Two particles of masses m1 and m2 initially at rest start
coordinates moving towards each other under their mutual force of
h b attraction. The speed of the centre of mass at any time t,
(a) x = , y = y when they are at a distance r apart, is
2 2
b h æ m1m2 1 ö
(b) x= , y=
2 2 (a) zero (b) çè G 2 . m ÷ø t
h r 1
b h
(c) x = , y =
3 3 æ m1m2 1 ö æ m1m2 1 ö
h b x (c) çè G 2 . m ÷ø t (d) çè G 2 . m + m ÷ø t
x= , y= r 2 r 1 2
(d) b
3 3
Q.11 A 'T' shaped object, dimensions shown in the figure, is
Q.6 Two bodies of masses 2 kg and 4 kg are moving with r
velocities 2 m/s and 10 m/s respectively along same lying on a smooth floor. A force ' F ' is applied at the point
direction. Then the velocity of their centre of mass will be P parallel to AB, such that the object has only the
(a) 8.1 m/s (b) 7.3 m/s translational motion without rotation. Find the location of
(c) 6.4 m/s (d) 5.3 m/s P with respect to C
Q.7 Four particles of masses m, 2m, 3m and 4m are arranged at 4 l
the corners of a parallelogram with each side equal to a (a) l
A
3 B
and one of the angle between two adjacent sides is 60°. (b) I P
The parallelogram lies in the x-y plane with mass m at the 2 F 2l
origin and 4m on the x-axis. The centre of mass of the (c) l
3
arrangement will be located at 3 C
(d) l
æ 3 ö æ 3 ö 2
(a) ç 2 a, 0.95a ÷ (b) ç 0.95a , 4 a ÷
è ø è ø
Q.12 Two spheres of masses 2M and M are initially at rest at a
æ 3a a ö æ a 3a ö
(c) çè , ÷ø (d) çè , ÷ø distance R apart. Due to mutual force of attraction, they
4 2 2 4
approach each other. When they are at separation R/2, the
Q.8 Three identical metal balls each of radius r are placed touching acceleration of the centre of mass of spheres would be
each other on a horizontal surface such that an equilateral (a) 0 m/s2 (b) g m/s2
triangle is formed, when centres of three balls are joined. (c) 3 g m/s2 (d) 12 g m/s2
The centre of the mass of system is located at Q.13 Masses 8 kg, 2 kg, 4 kg and 2 kg are placed at the corners
(a) Horizontal surface A, B, C, D respectively of a square ABCD of diagonal 80
(b) Centre of one of the balls cm. The distance of centre of mass from A will be
(c) Line joining centres of any two balls (a) 20 cm (b) 30 cm
(d) Point of intersection of the medians (c) 40 cm (d) 60 cm
Q.9 2 bodies of different masses of 2 kg and 4 kg are moving Q.14 If linear density of a rod of length 3m varies as l = 2 + x,
with velocities 20 m/s and 10 m/s towards each other due them the position of the centre of gravity of the rod is
to mutual gravitational attraction. What is the velocity of 7 12
their centre of mass? (a) m (b) m
3 7
(a) 5 m/s (b) 6 m/s 10 9
(c) 8 m/s (d) Zero (c) m (d) m
7 7

RESPONSE 5. 6. 7. 8. 9.
GRID 10. 11. 12. 13. 14.

Space for Rough Work


t.me/Ebooks_Encyclopedia27. t.me/Magazines4all

DPP/ P 14 3
Q.15 Four bodies of equal mass start moving with same speed Q.19 A cricket bat is cut at the location of its centre of mass as
as shown in the figure. In which of the following shown in the fig. Then
combination the centre of mass will remain at origin?
Y
(a) c and d d c

(b) a and b (a) The two pieces will have the same mass
X' X (b) The bottom piece will have larger mass
(c) a and c (c) The handle piece will have larger mass
a b
(d) b and d (d) Mass of handle piece is double the mass of bottom
Y' piece
Q.16 Three identical spheres, each of mass 1 kg are kept as Q.20 Consider a system of two particles having mass m1 and m2.
shown in figure, touching each other, with their centres on If the particle of mass m1 is pushed towards the centre of
a straight line. If their centres are marked P, Q, R mass of particles through a distance d, by what distance
respectively, the distance of centre of mass of the system would be particle of mass m2 move so as to keep the centre
from P is of mass of particles at the original position?
PQ + PR + QR m1 m m2
(a) y
(a) d (b) 1 d (c) d (d) d
3
m1 + m2 m2 m1
PQ + PR
(b) P Q R
3 x DIRECTIONS (Q.21-Q.22) : In the following questions,
PQ + QR
(c) more than one of the answers given are correct. Select the
3
PR + QR
correct answers and mark it according to the following
(d) codes:
3
Q.17 A ladder is leaned against a smooth wall and it is allowed Codes :
to slip on a frictionless floor. Which figure represents (a) 1, 2 and 3 are correct (b) 1 and 2 are correct
trace of motion of its centre of mass
(c) 2 and 4 are correct (d) 1 and 3 are correct
Q.21 Choose the wrong statements about the centre of mass
(CM) of a system of two particles
(a) (b) (1) The CM lies on the line joining the two particles
midway between them
Time Time
(2) The CM lies on the line joining them at a point whose
distance from each particle is proportional to the
square of the mass of that particle
(c) (d) (3) The CM is on the line joining them at a point whose
Time Time distance from each particle is proportional to the mass
Q.18 The two particles X and Y, initially at rest, start moving of that particle
towards each other under mutual attraction. If at any instant (4) The CM lies on the line joining them at a point whose
the velocity of X is V and that of Y is 2V, the velocity of distance from each particle is inversely proportional
their centre of mass will be to the mass of that particle
(a) 0 (b) V (c) 2V (d) V/2

RESPONSE 15. 16. 17. 18. 19.


GRID 20. 21.

Space for Rough Work


t.me/Ebooks_Encyclopedia27. t.me/Magazines4all

EBD_7156
4 DPP/ P 14
Q.22 Choose the wrong statements about the centre of mass of Q.25 Maximum extension in the spring after system loses
a body contact with wall
(1) It lies always outside the body 3mg 3mg
(2) It lies always inside the body (a) (b)
2k 2k
(3) It lies always on the surface of the body 3mg
(4) It may lie within, outside or on the surface of the body (c) (d) None of these
2k
DIRECTIONS (Q.23-Q.25) : Read the passage given below DIRECTIONS (Q. 26-Q.28) : Each of these questions contains
and answer the questions that follows : two statements: Statement-1 (Assertion) and Statement-2
A system consists of block A and B each of mass m connected (Reason). Each of these questions has four alternative choices,
by a light spring as shown in the figure with block B in contact only one of which is the correct answer. You have to select the
with a wall. The block A compresses the spring by 3mg/k from correct choice.
natural length of spring and then released from rest. Neglect (a) Statement-1 is True, Statement-2 is True; Statement-2 is a
friction anywhere. correct explanation for Statement-1.
(b) Statement-1 is True, Statement-2 is True; Statement-2 is
NOT a correct explanation for Statement-1.
3mg/k (c) Statement -1 is False, Statement-2 is True.
k
B A (d) Statement -1 is True, Statement-2 is False.
Q.26 Statement-1 : The centre of mass of a system of n particles
Q.23 Acceleration of centre of mass of system comprising A is the weighted average of the position vector of the n
and B just after A is released is particles making up the system.
(a) 0 (b) 3g/2 Statement-2 : The position of the centre of mass of a
(c) 3g (d) None of these system is independent of coordinate system.
Q.24 Velocity of centre of mass of system comprising A and B Q.27 Statement-1 : The centre of mass of a proton and an electron,
when block B just loses contact with the wall released from their respective positions remains at rest.
Statement-2 : The centre of mass remains at rest, if no
3g m
(a) 3 g m (b) external force is applied.
k 2 k
Q.28 Statement-1 : Position of centre of mass is independent
m of the reference frame.
(c) 2g (d) None of these
k Statement-2 : Centre of mass is same for all bodies.

RESPONSE 22. 23. 24. 25. 26.


GRID 27. 28.

DAILY PRA CTICE PROBLEM SHEET 14 - PHYSICS


Total Questions 28 Total Marks 112
Attempted Correct
Incorrect N et Score
Cut-off Score 28 Qualifying Score 46
Success Gap = Net Score – Qualifying Score
Net Score = (Correct × 4) – (Incorrect × 1)
Space for Rough Work
t.me/Ebooks_Encyclopedia27. t.me/Magazines4all

DPP - Daily Practice Problems


Name : Date :

Start Time : End Time :

SYLLABUS : Rotational Motion – 1: Basic concepts of rotational motion, moment of a force, torque,
15
angular momentum and its conservation with application
Max. Marks : 112 Time : 60 min.
GENERAL INSTRUCTIONS
• The Daily Practice Problem Sheet contains 28 MCQ's. For each question only one option is correct. Darken the correct
circle/ bubble in the Response Grid provided on each page.
• You have to evaluate your Response Grids yourself with the help of solution booklet.
• Each correct answer will get you 4 marks and 1 mark shall be deduced for each incorrect answer. No mark will be given/
deducted if no bubble is filled. Keep a timer in front of you and stop immediately at the end of 60 min.
• The sheet follows a particular syllabus. Do not attempt the sheet before you have completed your preparation for that
syllabus. Refer syllabus sheet in the starting of the book for the syllabus of all the DPP sheets.
• After completing the sheet check your answers with the solution booklet and complete the Result Grid. Finally spend time
to analyse your performance and revise the areas which emerge out as weak in your evaluation.

DIRECTIONS (Q.1-Q.20) : There are 20 multiple choice (c) When no external impulse acts upon the system
questions. Each question has 4 choices (a), (b), (c) and (d), out (d) When axis of rotation remains same
of which ONLY ONE choice is correct. Q.3 Two rigid bodies A and B rotate with rotational kinetic
Q.1 A thin circular ring of mass M and radius r is rotating energies EA and EB respectively. The moments of inertia
about its axis with a constant angular velocity w. Four of A and B about the axis of rotation are IA and IB respectively.
objects each of mass m, are kept gently to the opposite If IA = IB/4 and EA = 100 EB, the ratio of angular momentum
ends of two perpendicular diameters of the ring. The (LA) of A to the angular momentum (LB) of B is
angular velocity of the ring will be (a) 25 (b) 5/4 (c) 5 (d) 1/4
Q.4 A uniform heavy disc is rotating at constant angular velocity
Mw ( M + 4m) w ( M - 4m) w Mw w about a vertical axis through its centre and perpendicular
(a) (b) (c) (d)
M + 4m M M + 4m 4m to the plane of the disc. Let L be its angular momentum. A
Q..2 The angular momentum of a system of particles is lump of plasticine is dropped vertically on the disc and
conserved sticks to it. Which of the following will be constant?
(a) When no external force acts upon the system (a) w (b) w and L both
(b) When no external torque acts upon the system (c) L only (d) Neither w nor L

RESPONSE GRID 1. 2. 3. 4.
Space for Rough Work
t.me/Ebooks_Encyclopedia27. t.me/Magazines4all

EBD_7156
2 DPP/ P 15
Q.5 Two discs of moment of inertia I1 and I2 and angular speeds Q.11 An automobile engine develops 100 kW when rotating at a
w1 and w2 are rotating along collinear axes passing through speed of 1800 rev/min. What torque does it deliver
their centre of mass and perpendicular to their plane. If (a) 350 N-m (b) 440 N-m
the two are made to rotate combindly along the same axis
(c) 531 N-m (d) 628 N-m
the rotational KE of system will be
Q.12 A constant torque acting on a uniform circular wheel
(a)
I1w1 + I 2 w 2
(b)
( I1 + I2 )( w1 + w 2 )2 changes its angular momentum from A0 to 4A0 in 4 seconds.
2 ( I1 + I 2 ) 2 The magnitude of this torque is
( I1w1 + I2 w 2 ) 2
(c) (d) None of these 3A 0
2 ( I1 + I 2 ) (a) (b) A0
4
Q.6 A particle performs uniform circular motion with an
angular momentum L. If the frequency of a particle's (c) 4A0 (d) 12A0
motion is doubled and its kinetic energy is halved, the Q.13 A wheel having moment of inertia 2 kg -m2 about its
angular momentum becomes. vertical axis, rotates at the rate of 60 rpm about this axis.
(a) 2 L (b) 4 L (c) L/2 (d) L/4 The torque which can stop the wheel's rotation in 1 minute
Q.7 A round disc of moment of inertia I 2 about its axis would be
perpendicular to its plane and passing through its centre is 2p p
placed over another disc of moment of inertia I1 rotating (a) Nm (b) Nm
15 12
with an angular velocity w about the same axis. The final
angular velocity of the combination of discs is p p
(c) Nm (d) Nm
I 2w I1w ( I1 + I2 ) w 15 18
(a) I1 + I 2 (b) w (c) I1 + I 2 (d) I1 r Ù Ù Ù
Q.14 Find the torque of a force F = -3 i + j + 5k acting at the
Q.8 Calculate the angular momentum of a body whose rotational
energy is 10 joule. If the angular momentum vector r Ù Ù Ù
point r = 7 i + 3 j + k
coincides with the axis of rotation and its moment of inertia
about this axis is 8×10-7 kg m2 Ù Ù Ù Ù Ù Ù
(a) 14 i - 38 j + 16 k (b) 4 i + 4 j + 6 k
(a) 4 × 10–3 kg m2 /s (b) 2 × 10–3 kg m2/s
–3
(c) 6 × 10 kg m /s 2 (d) None of these Ù Ù Ù Ù Ù Ù
(c) -14 i + 38 j - 16 k (d) -21 i + 3 j + 5 k
Q.9 If the earth is treated as a sphere of radius R and mass M. Its
angular momentum about the axis of rotation with period T Q.15 A constant torque of 1000 N -m, turns a wheel of moment
is of inertia 200 kg -m2 about an axis passing through the
centre. Angular velocity of the wheel after 3 s will be
pMR 3 MR 2 p 2pMR 2 4pMR 2
(a) (b) (c) (d) (a) 15 rad/s (b) 10 rad/s (c) 5 rad/s (d) 1 rad/s
T T 5T 5T
Q.10 If the earth is a point mass of 6×1024kg revolving around Q.16 A torque of 30 N-m is applied on a 5 kg wheel whose
the sun at a distance of 1.5×108 km and in time T = 3.14 moment of inertia is 2kg-m2 for 10 sec. The angle covered
×107s. then the angular momentum of the earth around by the wheel in 10 sec will be
the sun is (a) 750 rad (b) 1500 rad
(a) 1.2 × 1018 kg m2/s (b) 1.8 × 1029 kg m2/s
(c) 1.5 × 1037 kg m2/s (d) 2.7 × 1040 kg m2/s (c) 3000 rad (d) 6000 rad

5. 6. 7. 8. 9.
RESPONSE
10. 11. 12. 13. 14.
GRID
15. 16.

Space for Rough Work


t.me/Ebooks_Encyclopedia27. t.me/Magazines4all

DPP/ P 15 3
Q.17 A horizontal force F is applied such that the block remains Q.21 A child is standing with folded hands at the centre of a
stationary, then which of the following statement is false platform rotating about its central axis. The kinetic energy
of the system is K. The child now stretches his arms so
a
that the moment of inertia of the system doubles. The
kinetic energy of the system now is
a
(1) less than 2K (2) equal to K/2
F
(3) more thanK/4 (4) equal to 4K
Q.22 Two uniforms discs of equal mass but unequal radii are
mounted on fixed horizontal axiles. Light strings are
wrapped on each of the discs. The strings are pulled by
constant equal forces F for same amount of time as shown
(a) f = mg [where f is the friction force] in the figure.
(b) F = N [ where N is the normal reaction] F
F
(c) F will not produce torque
(d) N will not produce torque
Q.18 In a bicycle, the radius of rear wheel is twice the radius of
front wheel. If r F and rr are the radius, vF and vr are speeds
Disc I
of top most points of wheel, then
Disc II
(a) vr = 2 vF (b) vF = 2 vr Angular momenta of discs are L1 and L2 and their kinetic
(c) vF = vr (d) vF > vr energies are K1 and K2. Which of the following statements
Q.19 The wheel of a car is rotating at the rate of 1200 revolutions true –
per minute. On pressing the accelerator for 10 seconds, it (1) L1 = L2 (2) L1 < L2 (3) K1 > K2 (4) K1 = K2
starts rotating at 4500 revolutions per minute. The angular
acceleration of the wheel is DIRECTIONS (Q.23-Q.25) : Read the passage given below
(a) 30 rad/sec2 (b) 1880 degree/sec2 and answer the questions that follows :
(c) 40 rad/sec 2 (d) 1980 degree/sec2 Consider a cylinder of mass M = 1kg and radius R = 1 m lying on
Q.20 A wheel rotates with a constant acceleration of 2.0 radian/ a rough horizontal plane. It has a plank lying on its top as shown in
sec 2 . It the wheel starts from rest, the number of the figure.
revolutions it makes in the first ten seconds will be
approximately m = 1kg
F
60°
(a) 8 (b) 16 (c) 24 (d) 32
A
DIRECTIONS (Q.21-Q.22) : In the following questions,
more than one of the answers given are correct. Select the M R
correct answers and mark it according to the following
B
codes:
Codes : A force F = 55 N is applied on the plank such that the
(a) 1, 2 and 3 are correct (b) 1 and 2 are correct plank moves and causes the cylinder to roll. The plank
always remains horizontal. There is no slipping at any
(c) 2 and 4 are correct (d) 1 and 3 are correct
point of contact.

RESPONSE 17. 18. 19. 20. 21.


GRID 22.

Space for Rough Work


t.me/Ebooks_Encyclopedia27. t.me/Magazines4all

EBD_7156
4 DPP/ P 15
Q.23 Calculate the acceleration of cylinder. (b) Statement-1 is True, Statement-2 is True; Statement-2 is
(a) 20 m/s2 (b) 10 m/s2 NOT a correct explanation for Statement-1.
(c) 5 m/s 2 (d) None of these (c) Statement -1 is False, Statement-2 is True.
Q.24 Find the value of frictional force at A (d) Statement -1 is True, Statement-2 is False.
(a) 7.5 N (b) 5.0 N Q.26 Statement -1: Torque is equal to rate of change of angular
(c) 2.5 N (d) None of these momentum.
Q.25 Find the value of frictional force at B Statement -2: Angular momentum depends on moment
(a) 7.5 N (b) 5.0 N of inertia and angular velocity.
(c) 2.5 N (d) None of these Q.27 Statement -1: Torque due to force is maximum when angle
r r
DIRECTIONS (Q.26-Q.28) : Each of these questions contains between r and F is 90°.
two statements: Statement-1 (Assertion) and Statement-2 Statement -2: The unit of torque is newton- meter.
(Reason). Each of these questions has four alternative choices, Q.28 Statement -1: It is harder to open and shut the door if we
only one of which is the correct answer. You have to select the apply force near the hinge.
correct choice. Statement -2: Torque is maximum at hinge of the door.
(a) Statement-1 is True, Statement-2 is True; Statement-2 is a
correct explanation for Statement-1.

RESPONSE 23. 24. 25. 26. 27.


GRID 28.

DAILY PRA CTICE PROBLEM SHEET 15 - PHYSICS


Total Questions 28 Total Marks 112
Attempted Correct
Incorrect N et Score
Cut-off Score 28 Qualifying Score 44
Success Gap = Net Score – Qualifying Score
Net Score = (Correct × 4) – (Incorrect × 1)

Space for Rough Work


t.me/Ebooks_Encyclopedia27. t.me/Magazines4all

DPP - Daily Practice Problems


Name : Date :

Start Time : End Time :

SYLLABUS : Rotational Motion-2 : Moment of inertia, radius of gyration,


16
(values of moments of inertia simple geometrical objects)
Max. Marks : 120 Time : 60 min.
GENERAL INSTRUCTIONS
• The Daily Practice Problem Sheet contains 30 MCQ's. For each question only one option is correct. Darken the correct
circle/ bubble in the Response Grid provided on each page.
• You have to evaluate your Response Grids yourself with the help of solution booklet.
• Each correct answer will get you 4 marks and 1 mark shall be deduced for each incorrect answer. No mark will be given/
deducted if no bubble is filled. Keep a timer in front of you and stop immediately at the end of 60 min.
• The sheet follows a particular syllabus. Do not attempt the sheet before you have completed your preparation for that
syllabus. Refer syllabus sheet in the starting of the book for the syllabus of all the DPP sheets.
• After completing the sheet check your answers with the solution booklet and complete the Result Grid. Finally spend time
to analyse your performance and revise the areas which emerge out as weak in your evaluation.

DIRECTIONS (Q.1-Q.21) : There are 21 multiple choice (a) 2.5 × 10–3 kg–m2 (b) 2 × 10–3 kg–m2
–3
(c) 5 × 10 kg–m 2 (d) 3.5 × 10–3 kg–m2
questions. Each question has 4 choices (a), (b), (c) and (d), out
of which ONLY ONE choice is correct. Q.4 A constant torque of 31.4 N–m is exerted on a pivoted
wheel. If angular acceleration of wheel is 4 p rad/sec2, then
Q.1 Five particles of mass 2 kg are attached to the rim of a
the moment of inertia of the wheel is
circular disc of radius 0.1 m & negligible mass. Moment
(a) 2.5 kg–m2 (b) 2.5 kg–m2
of inertia of the system about an axis passing through the 2
(c) 4.5 kg–m (d) 5.5 kg–m2
centre of the disc & perpendicular to its plane is
Q.5 From a uniform wire, two circular loops are made (i) P of
(a) 1 kg-m2 (b) 0.1kg-m2 (c) 2 kg-m2 (d) 0.2 kg-m2
radius r and (ii) Q of radius nr. If the moment of inertia of
Q.2 Two discs of the same material and thickness have radii 0.2
loop Q about an axis passing through its centre and
m and 0.6 m. Their moments of inertia about their axes will
perpendicular to its plane is 8 times that of P about a similar
be in the ratio of
axis, the value of n is (diameter of the wire is very much
(a) 1 : 81 (b) 1: 27 (c) 1 : 9 (d) 1 : 3
smaller than r or nr)
Q.3 A cylinder of 500 g and radius 10 cm has moment of inertia
(a) 8 (b) 6
(about its natural axis)
(c) 4 (d) 2
RESPONSE GRID 1. 2. 3. 4. 5.
Space for Rough Work
t.me/Ebooks_Encyclopedia27. t.me/Magazines4all

EBD_7156
2 DPP/ P 16
Q.6 The moment of inertia of a sphere of mass M and radius R Q.12 Three point masses m1, m2, m3 are located at the vertices
2
MR 2 . The radius
of an equilateral triangle of length 'a'. The moment of
about an axispassing through itscentre is 5 inertia of the system about an axis along the altitude of the
of gyration of the sphere about a parallel axis to the above triangle passing through m1, is
and tangent to the sphere is a2
æ 7 ö÷ æ 3 ÷ö (a) (m2 + m3) (b) (m1 + m2 + m3) a2
7 3 çç ÷ R ççç ÷÷ R
4
(a) R (b) R (c) çç 5 ÷÷ (d) çè 5 ÷ø a2
5 5 è ø (c) (m1 + m2) (d) (m2 + m3) a2
Q.7 Four particles each of mass m are placed at the corners of 4
a square of side length l. The radius of gyration of the Q.13 Three rods each of length L and mass M are placed along
system about an axis perpendicular to the square and X, Y and Z axis in such a way that one end of each of the
passing through its centre is rod is at the origin. The moment of inertia of this system
about Z axis is
l l
(a) (b) (c) l (d) ( 2)l 2ML2 4ML2 5ML2 ML2
2 2 (a) (b) (c) (d)
3 3 3 3
Q.8 The radius of gyration of a disc of mass 50 g and radius 2.5 Q.14 ABC is a triangular plate of uniform
cm, about an axis passing through its centre of gravity and thickness. The sides are in the ratio A
perpendicular to the plane is shown in the figure. IAB, IBC, ICA are
(a) 0.52 cm (b) 1.76 cm (c) 3.54 cm (d) 6.54 cm the moments of inertia of the plate
Q.9 Moment of inertia of a ring of mass m = 3 gm and radius about AB, BC, CA respectively. For 4 5
r = 1 cm about an axis passing through its edge and parallel this arrangement which one of the
to its natural axis is following relation is correct?
(a) 10 gm–cm2 (b) 100 gm–cm2 B 3 C
(c) 6 gm–cm2 (d) 1 gm–cm2
Q.10 A disc is of mass M and radius r. The moment of inertia of (a) ICA is maximum (b) IBC > IAB
it about an axis tangential to its edge and in plane of the (c) IBC > IAB (d) IAB + IBC = ICA
disc or parallel to its diameter is
Q.15 A 1m long rod has a mass of 0.12 kg. The moment of inertia
5 Mr 2 3 Mr 2
(a) Mr 2 (b) (c) Mr 2 (d) about an axis passin through the centre and perpendicular
4 4 2 2
to the length of rod will be
Q.11 Two spheres each of mass M and radius R/2 are connected
(a) 0.01kg-m2 (b) 0.001 kg-m2
with a massless rod of length 2R as shown in the figure. 2
(c) 1 kg-m (d) 10 kg-m2
M M Q.16 Two rings of the same radius and mass are placed such that
The moment of inertia
R/2 R/2 their centres are at a common point and their planes are
of the system about an
perpendicular to each other. The moment of inertia of the
axis passing through
system about an axis passing through the centre and
the centre of one of the 2R perpendicular to the plane of one of the rings is (mass of
spheres and
the ring = m and radius = r)
perpendicular to the
1 2
rod will be (a) mr (b) mr2
2
3 2
21 2 2 5 5 (c) mr (d) 2 mr2
(a) Mr (b) Mr 2 (c) Mr 2 (d) Mr 2 2
5 5 2 21

6. 7. 8. 9. 10.
RESPONSE
11. 12. 13. 14. 15.
GRID
16.

Space for Rough Work


t.me/Ebooks_Encyclopedia27. t.me/Magazines4all

DPP/ P 16 3
Q.17 One quarter sector is cut from a uniform (a) 3 MR2 Y
circular disc of radius R. This sector has
3
mass M. It is made to rotate about a line (b) MR 2
2
perpendicular to its plane and passing
through the centre of the original disc. (c) 5 MR2
90°
Its moment of inertia about the axis of 7
(d) MR 2 Y¢
rotation is 2
1 1 1
(a) MR 2 (b) MR2 (c) MR 2 (d) 2MR 2 DIRECTIONS (Q.22-Q.24) : In the following questions,
2 4 8
Q.18 A thin wire of length L and uniform linear mass density r is more than one of the answers given are correct. Select
bent into a circular loop with centre at O as shown in figure. the correct answers and mark it according to the following
The moment of inertia of the loop about the axis XX¢ is codes:
rL3 Codes :
(a)
8p 2 (a) 1, 2 and 3 are correct (b) 1 and 2 are correct
rL3 X X¢ (c) 2 and 4 are correct (d) 1 and 3 are correct
(b)
16 p 2 Q.22 The density of a rod AB increases linearly from A to B. Its
90°
5rL3 midpoint is O and its centre of mass is at C. Four axes pass
(c) O through A, B, O and C, all perpendicular to the length of
16 p 2
3rL3 the rod. The moments of inertia of the rod about these
(d) axes are I A, IB, IO and IC respectively then:.
8p 2
Q.19 Two disc of same thickness but of different radii are made (1) IA > IB (2) IA < IB
of two different materials such that their masses are same. (3) IO > IC (4) IO < IC
The densities of the materials are in the ratio 1 : 3. The Q.23 The moment of inertia of a thin square plate ABCD of
ratio of the moments of inertia of these discs about the uniform thickness about an axis passing through the centre
respective axes passing through their centres and O and perpendicular to the plane of the plate is
perpendicular to their planes will be in 4
(1) I1 + I2 1
(a) 1 : 3 (b) 3 : 1 (c) 1 : 9 (d) 9 : 1
(2) I3 + I4 A B
R
Q.20 A circular disc of radius R and thickness has moment (3) I1 + I3
6
of inertia I about an axis passing through its centre and (4) I1 + I2 + I3 + I4 3
where I1 , I2 , I3 and I4 are O
perpendicular to its plane. It is melted and recasted into a
solid sphere. The moment of inertia of the sphere about respectively moments of
inertia about axes 1, 2, 3 and D C
one of its diameter as an axis of rotation will be
4 which are in the plane of the 2
2I I I
(a) I (b) (c) (d) plate.
8 5 10
Q.24 Moment of inertia doesn’t depend on
Q.21 Three rings each of mass M and radius R are
(1) distribution of particles
arranged as shown in the figure. The moment
(2) mass
of inertia of the system about YY' will be
(3) position of axis of rotation
(4) None of these

RESPONSE 17. 18. 19. 20. 21.


GRID 22. 23. 24.

Space for Rough Work


t.me/Ebooks_Encyclopedia27. t.me/Magazines4all

EBD_7156
4 DPP/ P 16
DIRECTIONS (Q.25-Q.27) : Read the passage given below (a) be doubled
and answer the questions that follows : (b) increase but be less than double
(c) remain the same
Four identical spheres having mass M and (d) decrease
radius R are fixed tightly within a massless ring
such that the centres of all spheres lie in the DIRECTIONS (Q. 28-Q.30) : Each of these questions contains
plane of ring. The ring is kept on a rough two statements: Statement-1 (Assertion) and Statement-2
(Reason). Each of these questions has four alternative choices,
horizontal table as shown. The string is wrapped
only one of which is the correct answer. You have to select the
around the ring can roll without slipping. correct choice.
The other end of the string is passed over a massless frictionless
(a) Statement-1 is True, Statement-2 is True; Statement-2 is a
pulley to a block of mass M. A force F is applied horizontally on
correct explanation for Statement-1.
the ring, at the same level as the centre, so that the system is in
(b) Statement-1 is True, Statement-2 is True; Statement-2 is
equilibrium. NOT a correct explanation for Statement-1.
Q.25 The moment of inertia of the combined ring system about (c) Statement-1 is False, Statement-2 is True.
the centre of ring will be (d) Statement-1 is True, Statement-2 is False.
(a)
12
MR 2 Q.28 Statement-1 : Radius of gyration of a body is a constant
5 quantity.
48
MR2
F Statement-2 : The radius of gyration of a body about an
(b)
15 axis of rotation may be defined as the root mean square
(c)
24
MR 2
distance of the particles of the body from the axis of rotation.
5 M Q.29 Statement-1 : Moment of inertia of a particle is same,
48
MR2
whatever be the axis of rotation.
(d)
5 Statement-2 : Moment of inertia depends on mass and
Q.26 The magnitude of F is perpendicular distance of the particle from its axis of
(a) Mg (b) 2Mg rotation.
Mg Q.30 Statement-1 : If earth shrink (without change in mass) to
(c) (d) None of these half of its present size, length of the day would become 6
2
Q.27 If the masses of the spheres were doubled keeping their hours.
dimensions same, the force of friction between the ring Statement-2 : When the size of the earth will change, its
and the horizontal surface would moment of inertia will also change.

RESPONSE 25. 26. 27. 28. 29.


GRID 30.

DAILY PRA CTICE PROBLEM SHEET 16 - PHYSICS


Total Questions 30 Total Marks 120
Attempted Correct
Incorrect N et Score
Cut-off Score 32 Qualifying Score 50
Success Gap = Net Score – Qualifying Score
Net Score = (Correct × 4) – (Incorrect × 1)
Space for Rough Work
t.me/Ebooks_Encyclopedia27. t.me/Magazines4all

DPP - Daily Practice Problems


Name : Date :

Start Time : End Time :

SYLLABUS : Rotational Motion - 3 : Rolling Motion, Parallel and perpendicular theorems and their
17
applications, Rigid body rotation, equations of rotational motion
Max. Marks : 116 Time : 60 min.
GENERAL INSTRUCTIONS
• The Daily Practice Problem Sheet contains 30 MCQ's. For each question only one option is correct. Darken the correct
circle/ bubble in the Response Grid provided on each page.
• You have to evaluate your Response Grids yourself with the help of solution booklet.
• Each correct answer will get you 4 marks and 1 mark shall be deduced for each incorrect answer. No mark will be given/
deducted if no bubble is filled. Keep a timer in front of you and stop immediately at the end of 60 min.
• The sheet follows a particular syllabus. Do not attempt the sheet before you have completed your preparation for that
syllabus. Refer syllabus sheet in the starting of the book for the syllabus of all the DPP sheets.
• After completing the sheet check your answers with the solution booklet and complete the Result Grid. Finally spend time
to analyse your performance and revise the areas which emerge out as weak in your evaluation.

DIRECTIONS (Q.1-Q.20) : There are 20 multiple choice Q.2 A uniform rod of length 2L is placed with one end in contact
questions. Each question has 4 choices (a), (b), (c) and (d), out with the horizontal and is then inclined at an angle a to the
of which ONLY ONE choice is correct. horizontal and allowed to fall without slipping at contact
point. When it becomes horizontal, its angular velocity will
Q.1 A disc is rolling (without slipping) on a horizontal surface. C is be
its centre and Q and P are two points equidistant from C . Let vP,
3g sin a
vQ and vC be the magnitude of velocities of points P, Q and (a) w =
C respectively, then 2L
Q
(a) vQ > vC > vP 2L
C (b) w =
(b) vQ < vC < vP 3g sin a
vP P 6 g sin a
(c) vQ = vP , vC = (c) w =
2 L
(d) vQ < vC > vP L
(d) w =
g sin a

RESPONSE GRID 1. 2.
Space for Rough Work
t.me/Ebooks_Encyclopedia27. t.me/Magazines4all

EBD_7156
2 DPP/ P 17
Q.3 According to the theorem of parallel axes I = I cm + Mx 2 , Q.8 A solid sphere is rolling on a frictionless surface, shown
the graph between I and x will be in figure with a transnational velocity v m/s. If sphere climbs
I I up to height h then value of v should be

(a) (b) h
v
O x O x

I I 10
(a) ³ 10 gh (b) ³ 2gh (c) 2gh (d) gh
7 7
Q.9 Moment of inertia of a disc about its own axis is I. Its
(c) (d) moment of inertia about a tangential axis in its plane is
O x O x 5 3
(a) I (b) 3I (c) I (d) 2I
Q.4 A solid cylinder of mass M and radius R rolls without 2 2
slipping down an inclined plane of length L and height h. Q.10 Three rings each of mass M and radius R are arranged as
What is the speed of its centre of mass when the cylinder shown in the figure. The moment of inertia of the system
reaches its bottom about YY' will be Y
3 4
(a) gh (b) gh (c) 4gh (d) 2gh (a) 3MR 2
4 3
Q.5 An inclined plane makes an angle 30° with the horizontal. 3
(b) MR 2
A solid sphere rolling down this inclined plane from rest 2
without slipping has a linear acceleration equal to (c) 5MR 2
g 2g 5g 5g
(a) (b) (c) (d) 7
3 3 7 14 (d) MR 2
Q.6 A cord is wound round the circumference of wheel of radius 2 Y'

r. The axis of the wheel is horizontal and moment of inertia Q.11 One circular ring and one circular disc, both are having the
about it is I. A weight mg is attached to the end of the cord same mass and radius. The ratio of their moments of inertia
and falls from the rest. After falling through a distance h, about the axes passing through their centres and perpendicular
the angular velocity of the wheel will be to their planes, will be
é 2mgh ù
1/ 2 (a) 1 : 1 (b) 2 : 1
2 gh
(a) (b) ê ú (c) 1 : 2 (d) 4 : 1
I + mr ë I + mr 2 û
1/ 2 Q.12 From a disc of radius R, a concentric circular portion of
é 2mgh ù radius r is cut out so as to leave an annular disc of mass M.
(c) ê ú (d) 2gh
ë I + 2 mr 2 û The moment of inertia of this annular disc about the axis
Q.7 A solid sphere, disc and solid cylinder all of the same mass perpendicular to its plane and passing through its centre of
and made up of same material are allowed to roll down gravity is
(from rest) on an inclined plane, then 1
1 M (R2 - r 2 )
(a) Solid sphere reaches the bottom first (a) M (R2 + r 2 ) (b)
2 2
(b) Solid sphere reaches the bottom late 1
(c) Disc will reach the bottom first (c) M (R4 + r 4 ) (d) 1 M ( R 4 - r 4 )
2 2
(d) All of them reach the bottom at the same time

RESPONSE 3. 4. 5. 6. 7.
GRID 8. 9. 10. 11. 12.

Space for Rough Work


t.me/Ebooks_Encyclopedia27. t.me/Magazines4all

DPP/ P 17 3
Q.13 The moment of inertia of a straight thin rod of mass M and (a) 4MR 2
length l about an axis perpendicular to its length and passing
through its one end, is (b) 40
MR 2 2R/3
9 O
2 2 2
Ml Ml Ml
(a) (b) (c) (d) Ml2
12 3 2 (c) 10MR 2 R

Q.14 Four thin rods of same mass M and same length l, form a 37
square as shown in figure. Moment of inertia of this (d) MR 2
9
system about an axis through centre
O and perpendicular to its plane is Q.19 The moment of inertia of a thin rod of mass M and length L
4 2 about an axis perpendicular to the rod at a distance L/4 from
(a) Ml l one end is
3
ML2 ML2 7 ML2 7 ML2
Ml 2 (a) (b) (c) (d)
(b) 6 12 24 48
3 l l Q.20 A wheel has a speed of 1200 revolutions per minute and is
Ml 2 O
made to slow down at a rate of 4 radians /s2. The number of
(c)
6 revolutions it makes before coming to rest is
2 2 l (a) 143 (b) 272 (c) 314 (d) 722
(d) Ml
3
Q.15 The moment of inertia of a uniform circular ring, having a DIRECTIONS (Q.21-Q.23) : In the following questions,
more than one of the answers given are correct. Select
mass M and a radius R, about an axis tangential to the ring
the correct answers and mark it according to the following
and perpendicular to its plane, is
codes:
3 1
(a) 2MR 2 (b) MR 2 (c) MR 2 (d) MR 2 Codes:
2 2 (a) 1, 2 and 3 are correct (b) 1 and 2 are correct
Q.16 The moment of inertia of uniform rectangular plate about an (c) 2 and 4 are correct (d) 1 and 3 are correct
axis passing through its mid-point and parallel to its length Q.21 In pure rolling fraction of its total energy associated with
l is (b = breadth of rectangular plate) rotation is a for a ring and b for a solid sphere. Then
Mb2 Mb3 Mb3 Mb2 (1) a = 1 / 2 (2) b = 2 / 7 (3) b = 2 / 5 (4) a = 1 / 4
(a) (b) (c) (d) Q.22 One solid sphere and a disc of same radius are falling along
4 6 12 12
Q.17 The moment of inertia of a circular ring about an axis passing an inclined plane without slip. One reaches earlier than the
through its centre and normal to its plane is 200 gm × cm2. other due to
Then moment of inertia about its diameter is (1) different size
(2) different radius of gyration
(a) 400 gm × cm2 (b) 300 gm × cm2
(3) different moment of inertia
(c) 200 gm × cm2 (d) 100 gm × cm2 (4) different friction
Q.18 From a circular disc of radius R and mass 9 M, a small disc of Q.23 A body is rolling down an inclined plane. Its translational
radius R/3 is removed from the disc. The moment of inertia of and rotational kinetic energies are equal. The body is not a
the remaining disc about an axis perpendicular to the plane of (1) solid sphere (2) hollow sphere
the disc and passing through O is (3) solid cylinder (4) hollow cylinder

13. 14. 15. 16. 17.


RESPONSE
18. 19. 20. 21. 22.
GRID
23.

Space for Rough Work


t.me/Ebooks_Encyclopedia27. t.me/Magazines4all

EBD_7156
4 DPP/ P 17
DIRECTIONS (Q.24-Q.26) : Read the passage given below and (a) Statement-1 is True, Statement-2 is True; Statement-2 is a
answer the questions that follows : correct explanation for Statement-1.
A uniform solid cylinder of mass 2m and (b) Statement-1 is True, Statement-2 is True; Statement-2 is
radius R rolls on a rough inclined plane NOT a correct explanation for Statement-1.
2m
with its axis perpendicular to the line of (c) Statement -1 is False, Statement-2 is True.
R
the greatest slope. m (d) Statement -1 is True, Statement-2 is False.
System is released from rest and as q Q.27 Statement-1 : Two cylinders, one hollow (metal) and the
the cylinder rolls it winds up a light ////////////////////////// ////////////////
other solid (wood) with the same mass and identical
string which passes over a light pul- dimensions are simultaneously allowed to roll without
ley. slipping down an inclined plane from the same height. The
Q.24 The acceleration of block of mass m is - hollow cylinder will reach the bottom of the inclined plane
2 4
(a) g (1 - cos q) (b) g (1 - sin q) first.
7 7
(c)
2
g (1 - sin q ) (d)
2
g (1 + sin q)
Statement-2 : By the principle of conservation of energy,
7 14 the total kinetic energies of both the cylinders are identical
Q.25 The tension in the string is –
when they reach the bottom of the incline.
æ 4 + 3sin q ö æ 3 - 4sin q ö
(a) çè 7
÷ø mg (b) èç 7 ø÷
mg Q.28 Statement-1: The force of frction in the case of a disc
æ 3 + 4sin q ö 2 rolling without slipping down an inclined plane is 1/3 g
(c) çè ÷ø mg (d) (1 - sin q) mg
7 7 sin a.
Q.26 The frictional force acting on the cylinder is-
2 æ 6 - sin q ö Statement-2: When the disc rolls without slipping, friction
(a) (1 - sin q) mg (b) çè 7 ÷ø mg is required because for rolling condition velocity of point
7
æ 1 + 6 cos q ö æ 1 + 6 sin q ö of contact is zero.
(c) çè ÷ø mg (d) çè 7 ÷ø mg
7
Q.29 Statement-1: If two different axes are at same distance
DIRECTIONS (Q. 27-Q.29) : Each of these questions contains from the centre of mass of a rigid body, then moment of
two statements: Statement-1 (Assertion) and Statement-2 inertia of the given rigid body about both the axes will
(Reason). Each of these questions has four alternative choices, always be the same.
only one of which is the correct answer. You have to select Statement-2: From parallel axis theorem, I = Icm + md 2,
the correct choice. where all terms have usual meaning.

RESPONSE 24. 25. 26. 27. 28.


GRID 29.

DAILY PRA CTICE PROBLEM SHEET 17 - PHYSICS


Total Questions 29 Total Marks 116
Attempted Correct
Incorrect N et Score
Cut-off Score 28 Qualifying Score 44
Success Gap = Net Score – Qualifying Score
Net Score = (Correct × 4) – (Incorrect × 1)
Space for Rough Work
t.me/Ebooks_Encyclopedia27. t.me/Magazines4all

DPP - Daily Practice Problems


Name : Date :

Start Time : End Time :

SYLLABUS : Gravitation - 1 (The Universal law of gravitation, Acceleration due to gravity and its variation
18
with altitude and depth, Kepler's law of planetary motion)
Max. Marks : 116 Time : 60 min.
GENERAL INSTRUCTIONS
• The Daily Practice Problem Sheet contains 29 MCQ's. For each question only one option is correct. Darken the correct
circle/ bubble in the Response Grid provided on each page.
• You have to evaluate your Response Grids yourself with the help of solution booklet.
• Each correct answer will get you 4 marks and 1 mark shall be deduced for each incorrect answer. No mark will be given/
deducted if no bubble is filled. Keep a timer in front of you and stop immediately at the end of 60 min.
• The sheet follows a particular syllabus. Do not attempt the sheet before you have completed your preparation for that
syllabus. Refer syllabus sheet in the starting of the book for the syllabus of all the DPP sheets.
• After completing the sheet check your answers with the solution booklet and complete the Result Grid. Finally spend time
to analyse your performance and revise the areas which emerge out as weak in your evaluation.

DIRECTIONS (Q.1-Q.20) : There are 20 multiple choice Q.3 The speed with which the earth have to rotate on its axis so
questions. Each question has 4 choices (a), (b), (c) and (d), out that a person on the equator would weigh (3/5) th as much
as present will be -
of which ONLY ONE choice is correct.
(Take the equatorial radius as 6400 km.)
Q.1 A mass M splits into two parts m and (M – m), which are (a) 3.28 × 10–4 rad/sec (b) 7.826 × 10–4 rad/sec
then separated by a certain distance. What ratio of (m / M) (c) 3.28 × 10–3 rad/sec (d) 7.28 × 10–3 rad/sec
maximises the gravitational force between the parts. ? Q.4 On a planet (whose size is the same as that of earth and
(a) 2/3 (b) 3/4
mass 4 times to the earth) the energy needed to lift a 2kg
mass vertically upwards through 2m distance on the planet
(c) 1/2 (d) 1/3
is (g = 10m/sec2 on the surface of earth) -
Q.2 What would be the angular speed of earth, so that bodies (a) 16 J (b) 32 J (c) 160 J (d) 320 J
lying on equator may experience weightlessness ? Q.5 Two bodies of mass 102 kg and 103 kg are lying 1m apart.
(g = 10m/s2 and radius of earth = 6400 km) The gravitational potential at the mid-point of the line
(a) 1.25 × 10–3 rad/sec (b) 1.25 × 10–2 rad/sec joining them is -
–4
(c) 1.25 × 10 rad/sec (d) 1.25 × 10–1 rad/sec (a) 0 (b) –1.47 Joule/kg
(c) 1.47 Joule/kg (d) –147 × 10–7 Joule /kg

RESPONSE GRID 1. 2. 3. 4. 5.
Space for Rough Work
t.me/Ebooks_Encyclopedia27. t.me/Magazines4all

EBD_7156
2 DPP/ P 18
Q.6 If g is the acceleration due to gravity on the earth’s surface, Q.13 At what height above the earth’s surface does the force of
the gain in P.E. of an object of mass m raised from the surface gravity decrease by 10% ? Assume radius of earth to be
of the earth to a height of the radius R of the earth is - 6370 km.
1 1 (a) 350 km. (b) 250 km. (c) 150 km. (d) 300 km.
(a) mgR (b) 2mgR (c) mgR (d) mgR Q.14 A particle is suspended from a spring and it stretches the
2 4
Q.7 Four particles, each of mass m, are placed at the corners spring by 1 cm on the surface of earth. The same particle
of square and moving along a circle of radius r under the will stretches the same spring at a place 800 km above earth
influence of mutual gravitational attraction. The speed of surface by -
each particle will be - (a) 0.79 cm (b) 0.1 cm
(c) p / 6 rad/hr. (d) 2p / 7 rad/hr.
Gm Gm Q.15 The change in the value of acceleration of earth towards
(a) (2 2 + 1) (b)
r r sun, when the moon comes from the position of solar
Gm æ 2 2 + 1ö eclipse to the position on the other side of earth in line
2 2Gm
with sun is (Mass of moon = 7.36 × 1022 kg, the orbital
r çè 4 ÷ø
(c) (d)
r radius of moon 3.8 × 108m.
Q.8 Three particles of equal mass m are situated at the vertices (a) 6.73 × 10–2 m/s2 (b) 6.73 × 10–3 m/s2
of an equilateral triangle of side l. What should be the (c) 6.73 × 10 m/s–4 2 (d) 6.73 × 10–5 m/s2
velocity of each particle, so that they move on a circular Q.16 The radius of the earth is Re and the acceleration due to gravity
path without changing l ? at its surface is g. The work required in raising a body of
GM GM 2GM GM mass m to a height h form the surface of the earth will be -
(a) (b) (c) (d) mgh mgh mg
2l l l 3l (a)
mgh
(b) (c) (d)
Q.9 What will be the acceleration due to gravity on the surface æ h ö æ h ö
2
æ h ö æ h ö
of the moon if its radius is 1/4 th the radius of the earth and çè 1 - R ÷ø çè 1 + R ÷ø çè1 + R ÷ø çè1 + R ÷ø
e e e e
its mass is 1/80 th the mass of the earth ? Q.17 The masses and the radius of the earth and the moon are
(a) g/6 (b) g/5 (c) g/7 (d) g/8 M1, M2 and R1, R2 respectively their centres are at distance
Q.10 If the value of 'g' at a height h above the surface of the earth d apart. The minimum speed with which a particle of mass
is the same as at a depth x below it, then (both x and h being m should be projected form a point midway between the
much smaller than the radius of the earth) two centres so as to escape to infinity will be -
h
(a) x = h (b) x = 2h (c) x = (d) x = h2 G G
2 (a) 2 (M1 + M 2 ) (b) (M1 + M 2 )
d d
Q.11 At what height above the earth's surface the acceleration
due to gravity will be 1/9 th of its value at the earth’s G M1
G 2
surface? Radius of earth is 6400 km. (c) (M1 + M 2 ) (d)
2d d M2
(a) 12800 km (b) 1280 km
(c) 128000 km (d) 128 km Q.18 With what velocity must a body be thrown upward form the
Q.12 If the radius of the earth were to shrink by one percent, its surface of the earth so that it reaches a height of 10 Re?
mass remaining the same, the acceleration due to gravity earth’s mass Me = 6 × 1024 kg, radius Re = 6.4 × 106 m and
on the earth’s surface would - G = 6.67 × 10–11 N–m2/kg2.
(a) decrease (b) remain unchanged (a) 10.7 × 104 m/s (b) 10.7 × 103 m/s
5
(c) 10.7 × 10 m/s (d) 1.07 × 104 m/s
(c) increase (d) None of these

6. 7. 8. 9. 10.
RESPONSE
11. 12. 13. 14. 15.
GRID
16. 17. 18.

Space for Rough Work


t.me/Ebooks_Encyclopedia27. t.me/Magazines4all

DPP/ P 18 3
Q.19 Two concentric shells of uniform density having masses M1 Q.23 Spot the correct statements :
and M2 are situated as shown in the figure. The force on the The acceleration due to gravity ‘g’ decreases if
particle of mass m when it is located at r = b is (1) We go down from the surface of the earth towards its
centre
M1 (2) We go up from the surface of the earth
(3) The rotational velocity of the earth is increased
O (4) We go from the equator towards the poles on the
b
r surface of the earth
m
p DIRECTIONS (Q.24-Q.26) : Read the passage given below and
M2
answer the questions that follows :
GM1m GM 2 m
(a) (b) The orbit of Pluto is much more eccentric than the orbits of the
b2 b2
other planets. That is, instead of being nearly circular, the orbit
(M1 + M 2 )m (M1 - M 2 )m
(c) G (d) G is noticeably elliptical. The point in the orbit nearest to the sun
b2 b2 is called the perihelion and the point farthest from the sun is
Q.20 What is the mass of the planet that has a satellite whose called the aphelion.
time period is T and orbital radius is r?

4p 2 r 3 3p 2r 3 4p 2 r 3 4p2T Sun
(a) (b) (c) (d)
GT 2
GT 2
GT 3
GT 2 Aphelion Perihelion

DIRECTIONS (Q.21-Q.23) : In the following questions, more than


one of the answers given are correct. Select the correct answers
Q.24 At perihelion, the gravitational potential energy of Pluto
and mark it according to the following codes:
in its orbit has
Codes : (a) its maximum value
(a) 1, 2 and 3 are correct (b) 1 and 2 are correct
(c) 2 and 4 are correct (d) 1 and 3 are correct
(b) its minimum value
Q.21 The gravitational force between two point masses m1 and (c) the same value as at every other point in the orbit
mm (d) value which depends on sense of rotation
m2 at separation r is given by F = k 1 2 2
r Q.25 At perihelion, the mechanical energy of Pluto’s orbit has
The constant k doesn't (a) its maximum value
(1) depend on medium between masses (b) its minimum value
(2) depend on the place (c) the same value as at every other point in the orbit
(3) depend on time
(d) value which depends on sense of rotation
(4) depend on system of units
Q.26 As Pluto moves from the perihelion to the aphelion, the
Q.22 Which of the following statements about the gravitional
work done by gravitational pull of Sun on Pluto is
constant are false ?
(1) It is a force (a) is zero
(2) It has no unit (b) is positive
(3) It has same value in all system of units (c) is negative
(4) It doesn’t depend on the value of the masses (d) depends on sense of rotation

RESPONSE 19. 20. 21. 22. 23.


GRID 24. 25. 26.

Space for Rough Work


t.me/Ebooks_Encyclopedia27. t.me/Magazines4all

EBD_7156
4 DPP/ P 18
DIRECTIONS (Q. 27-Q.29) : Each of these questions contains two Q.27 Statement-1 : Gravitational force between two particles is
statements: Statement-1 (Assertion) and Statement-2 (Reason). negligibly small compared to the electrical force.
Each of these questions has four alternative choices, only one of Statement-2 :The electrical force is experienced by charged
which is the correct answer. You have to select the correct choice. particles only.
Q.28 Statement-1 :The universal gravitational constant is same
(a) Statement-1 is True, Statement-2 is True; Statement-2 is a as acceleration due to gravity.
correct explanation for Statement-1. Statement-2 :Gravitional constant and acceleration due to
(b) Statement-1 is True, Statement-2 is True; Statement-2 is gravity have different dimensional formula.
NOT a correct explanation for Statement-1. Q.29 Statement-1 :There is no effect of rotation of earth on the
(c) Statement -1 is False, Statement-2 is True. value of acceleration due to gravity at poles.
(d) Statement -1 is True, Statement-2 is False. Statement-2 :Rotation of earth is about polar axis.

RESPONSE GRID 27. 28. 29.

DAILY PRA CTICE PROBLEM SHEET 18 - PHYSICS


Total Questions 29 Total Marks 116
Attempted Correct
Incorrect N et Score
Cut-off Score 30 Qualifying Score 48
Success Gap = Net Score – Qualifying Score
Net Score = (Correct × 4) – (Incorrect × 1)

Space for Rough Work


t.me/Ebooks_Encyclopedia27. t.me/Magazines4all

DPP - Daily Practice Problems


Name : Date :

Start Time : End Time :

SYLLABUS : Gravitation - 2 (Gravitational potential energy, Gravitational potential,


19
Escape velocity & Orbital velocity of a satellite, Geo-stationary satellites)
Max. Marks : 120 Time : 60 min.
GENERAL INSTRUCTIONS
• The Daily Practice Problem Sheet contains 30 MCQ's. For each question only one option is correct. Darken the correct
circle/ bubble in the Response Grid provided on each page.
• You have to evaluate your Response Grids yourself with the help of solution booklet.
• Each correct answer will get you 4 marks and 1 mark shall be deduced for each incorrect answer. No mark will be given/
deducted if no bubble is filled. Keep a timer in front of you and stop immediately at the end of 60 min.
• The sheet follows a particular syllabus. Do not attempt the sheet before you have completed your preparation for that
syllabus. Refer syllabus sheet in the starting of the book for the syllabus of all the DPP sheets.
• After completing the sheet check your answers with the solution booklet and complete the Result Grid. Finally spend time
to analyse your performance and revise the areas which emerge out as weak in your evaluation.

DIRECTIONS (Q.1-Q.21) : There are 21 multiple choice centre of the earth will be- (The mass of the earth is 6.00 ×
questions. Each question has 4 choices (a), (b), (c) and (d), out 1024 kg and its angular velocity = 7.30 × 10–5 rad./sec.)
of which ONLY ONE choice is correct. (a) 2.66 × 103 m. (b) 2.66 × 105 m.
(c) 2.66 × 10 m.6 (d) 2.66 × 107 m.
Q.1 A body of mass 100 kg falls on the earth from infinity. Q.3 Two satellites S1 and S2 revolve round a planet in the same
What will be its energy on reaching the earth ? Radius of direction in circular orbits. Their periods of revolutions
the earth is 6400 km and g = 9.8 m/s2. Air friction is are 1 hour and 8 hour respectively. The radius of S1 is 104
negligible. km. The velocity of S2 with respect to S1 will be-
(a) 6.27 × 109 J (b) 6.27 × 1010 J (a) p × 104 km/hr (b) p/3 × 104 km/hr
(c) 6.27 × 10 J 10 (d) 6.27 × 107 J 4
(c) 2p × 10 km/hr (d) p/2 × 104 km/hr
Q.2 An artificial satellite of the earth is to be established in the Q.4 In the above example the angular velocity of S2 as actually
equatorial plane of the earth and to an observer at the equator observed by an astronaut in S1 is -
it is required that the satellite will move eastward, completing (a) p/3 rad/hr (b) p/3 rad/sec
one round trip per day. The distance of the satellite from the (c) p/6 rad/hr (d) 2p/7 rad/hr

RESPONSE GRID 1. 2. 3. 4.

Space for Rough Work


t.me/Ebooks_Encyclopedia27. t.me/Magazines4all

EBD_7156
2 DPP/ P 19
Q.5 The moon revolves round the earth 13 times in one year. If Q.12 The mean distance of mars from sun is 1.524 times the
the ratio of sun-earth distance to earth-moon distance is distance of the earth from the sun. The period of revolution
392, then the ratio of masses of sun and earth will be - of mars around sun will be-
(a) 365 (b) 356 (a) 2.88 earth year (b) 1.88 earth year
(c) 3.56 × 105 (d) 1
(c) 3.88 earth year (d) 4.88 earth year
Q.6 Two planets of radii in the ratio 2 : 3 are made from the
materials of density in the ratio 3 : 2. Then the ratio of Q.13 The semi-major axes of the orbits of mercury and mars are
acceleration due to gravity g1/g2 at the surface of two respectively 0.387 and 1.524 in astronomical unit. If the
planets will be period of Mercury is 0.241 year, what is the period of Mars.
4 (a) 1.2 years (b) 3.2 years
(a) 1 (b) 2 . 2 5 (c) (d) 0.12
9 (c) 3.9 years (d) 1.9 years
Q.7 A satellite of mass m is revolving in a circular orbit of radius Q.14 If a graph is plotted between T2 and r3 for a planet then its
r. The relation between the angular momentum J of satellite slope will be -
and mass m of earth will be -
(a) J = G.Mm 2 r (b) J = GMm 4p 2 GM
(a) (b)
GM 4p 2
mr
(c) J = GMmr (d) J = (c) 4p GM (d) 0
M
Q.8 A spaceship is launched into a circular orbit close to earth's Q.15 The mass and radius of earth and moon are M1, R1 and M2,
surface. What additional velocity has now to be imparted R2 respectively. Their centres are d distance apart. With
to the spaceship in the orbit to overcome the gravitational what velocity should a particle of mass m be projected from
pull? (Radius of earth = 6400 km, g = 9.8 m/sec2) the mid point of their centres so that it may escape out to
(a) 3.285 km/sec (b) 32.85 m/sec infinity -
(c) 11.32 km/sec (d) 7.32 m/sec
G(M1 + M 2 ) 2G(M1 + M 2 )
Q.9 The ratio of the radius of the Earth to that of the moon is (a) (b)
10. The ratio of g on earth to the moon is 6. The ratio of d d
the escape velocity from the earth’s surface to that from 4G(M1 + M 2 ) GM1M 2
the moon is approximately - (c) (d)
d d
(a) 10 (b) 8 (c) 4 (d) 2 Q.16 A satellite has to revolve round the earth in a circular orbit
Q.10 Acceleration due to gravity on a planet is 10 times the value of radius 8 × 103 km. The velocity of projection of the
on the earth. Escape velocity for the planet and the earth satellite in this orbit will be -
are Vp and Ve respectively. Assuming that the radii of the
(a) 16 km/sec (b) 8 km/sec
planet and the earth are the same, then -
(a) VP = 10 Ve (b) VP = 10 Ve (c) 3 km/sec (d) 7.08 km/sec
Ve Q.17 If the satellite is stopped suddenly in its orbit which is at a
(c) Vp = Ve (d) VP = distnace = radius of earth from earth’s surface and allowed
10 10
Q.11 The Jupiter’s period of revolution round the Sun is 12 times to fall freely into the earth, the speed with which it hits the
that of the Earth. Assuming the planetary orbits are circular, surface of earth will be -
how many times the distance between the Jupiter and Sun (a) 7.919 m/sec (b) 7.919 km/sec
exceeds that between the Earth and the sun. (c) 11.2 m/sec (d) 11.2 km/sec
(a) 5.242 (b) 4.242 (c) 3.242 (d) 2.242

5. 6. 7. 8. 9.
RESPONSE
10. 11. 12. 13. 14.
GRID
15. 16. 17.

Space for Rough Work


t.me/Ebooks_Encyclopedia27. t.me/Magazines4all

DPP/ P 19 3
Q.18 A projectile is fired vertically upward from the surface of Q.22 Gas escapes from the surface of a planet because it acquires
earth with a velocity K ve m/s where ve m/s is the escape an escape velocity. The escape velocity will depend on
velocity and K < 1. Neglecting air resistance, the maximum which of the following factors:
height to which it will rise measured from the centre of (1) Mass of the planet
the earth is - (where R = radius of earth) (2) Radius of the planet
2 2 (3) Mass of the particle escaping
(c) 1 - K
R R
(a) (b) (d) K (4) Temperature of the planet
2
1 - K2 K R R
Q.23 ve and vp denotes the escape velocity from the earth and
Q.19 A satellite is revolving in an orbit close to the earth’s
surface. Taking the radius of the earth as 6.4 × 106 metre, another planet having twice the radius and the same mean
the value of the orbital speed and the period of revolution density as the earth. Then which of the following is (are)
of the satellite will respectively be (g = 9.8 meter/sec2) wrong ?
(a) 7.2 km/sec., 84.6 minutes (1) ve = vp (2) ve = 2vp
(b) 2.7 km/sec., 8.6 minutes (3) ve = vp/4 (4) ve = vp / 2
(c) .72 km/sec., 84.6 minutes Q.24 Select the wrong statements from the following
(d) 7.2 km/sec., 8.6 minutes (1) The orbital velocity of a satellite increases with the
Q.20 If the period of revolution of an artificial satellite just above radius of the orbit
the earth be T second and the density of earth be r, kg/m3 (2) Escape velocity of a particle from the surface of the
then earth depends on the speed with which it is fired
(G = 6.67 × 10–11 m3/kg. second2) (3) The time period of a satellite does not depend on the
radius of the orbit
(a) rT2 is a universal constant
(4) The orbital velocity is inversely proportional to the
(b) rT2 varies with time square root of the radius of the orbit
3p
(c) rT2 = DIRECTIONS (Q.25-Q.27) : Read the passage given below and
G
answer the questions that follows :
(d) Both (a) and (c)
Q.21 Two satellites P and Q of same mass are revolving near the It can be assumed that orbits of earth and mars are nearly circular
earth surface in the equitorial plane. The satellite P moves around the sun. It is proposed to launch an artificial planet around
in the direction of rotation of earth whereas Q moves in the sun such that its apogee is at the orbit of mars while its perigee
the opposite direction. The ratio of their kinetic energies is at the orbit of earth. Let Te and Tm be periods of revolution of
with respect to a frame attached to earth will be - earth and mars. Further the variables are assigned the meanings as
2 2 follows.
æ 8363 ö æ 7437 ö æ 8363 ö æ 7437 ö Me ® Mass of earth
(a) ç (b) ç (c) çè ÷ (d) çè ÷
è 7437 ÷ø è 8363 ÷ø 7437 ø 8363 ø Mm ® Mass of mars.
Le ® Angular momentum of earth around the sun.
DIRECTIONS (Q.22-Q.24) : In the following questions, more than Lm ® Angular momentum of mars around the sun.
one of the answers given are correct. Select the correct answers
Re ® Semi major axis of earth’s orbit.
and mark it according to the following codes:
Rm ® Semi major axis of mars orbit.
Codes : M ® Mass of the artificial planet.
(a) 1, 2 and 3 are correct (b) 1 and 2 are correct
Ee ® Total energy of earth.
(c) 2 and 4 are correct (d) 1 and 3 are correct
Em ® Total energy of mars.

RESPONSE 18. 19. 20. 21. 22.


GRID 23. 24.

Space for Rough Work


t.me/Ebooks_Encyclopedia27. t.me/Magazines4all

EBD_7156
4 DPP/ P 19
Q.25 Time period of revolution of the artificial planet about sun DIRECTIONS (Q. 28-Q.30) : Each of these questions contains
will be (neglect gravitational effects of earth and mars) two statements: Statement-1 (Assertion) and Statement-2
Te + Tm (Reason). Each of these questions has four alternative choices,
(a) (b) TeTm only one of which is the correct answer. You have to select
2
3/ 2 the correct choice.
2TeTm é T 2 / 3 + Tm2 / 3 ù
(c) (d) ê e ú (a) Statement-1 is True, Statement-2 is True; Statement-2 is a
Te + Tm êë 2 úû correct explanation for Statement-1.
Q.26 The total energy of the artificial planet’s orbit will be (b) Statement-1 is True, Statement-2 is True; Statement-2 is
NOT a correct explanation for Statement-1.
2 M æ Re Ee ö 2M æ Re Ee ö (c) Statement-1 is False, Statement-2 is True.
(a) (b)
M e çè Re + Rm ÷ø M m çè Re + Rm ÷ø (d) Statement-1 is True, Statement-2 is False.
Q.28 Statement-1 :The speed of revolution of an artificial
2 Ee M æ Re + Rm ö æ ö
2 Ee M Re + Rm satellite revolving very near the earth is 8kms–1.
(c) (d) ç ÷
M m çè Rm ÷ø Me ç R2 + R2 ÷ Statement-2 : Orbital velocity of a satellite, become
è e mø independent of height of near satellite.
Q.27 Areal velocity of the artificial planet around the sun will Q.29 Statement-1 :If an earth satellite moves to a lower orbit,
be there is some dissipation of energy but the speed of
(a) less than that of earth gravitational satellite increases.
(b) more than that of mars Statement-2 :The speed of satellite is a constant quantity.
(c) more than that of earth Q.30 Statement-1 :Gravitational potential of earth at every place
(d) same as that of the earth on it is negative.
Statement-2 :Every body on earth is bound by the attraction
of earth.

RESPONSE 25. 26. 27. 28. 29.


GRID 30.

DAILY PRA CTICE PROBLEM SHEET 19 - PHYSICS


Total Questions 30 Total Marks 120
Attempted Correct
Incorrect N et Score
Cut-off Score 32 Qualifying Score 50
Success Gap = Net Score – Qualifying Score
Net Score = (Correct × 4) – (Incorrect × 1)
Space for Rough Work
t.me/Ebooks_Encyclopedia27. t.me/Magazines4all

DPP - Daily Practice Problems


Name : Date :

Start Time : End Time :

SYLLABUS : Mechanical Properties of solids


20
Max. Marks : 120 Time : 60 min.
GENERAL INSTRUCTIONS
• The Daily Practice Problem Sheet contains 30 MCQ's. For each question only one option is correct. Darken the correct
circle/ bubble in the Response Grid provided on each page.
• You have to evaluate your Response Grids yourself with the help of solution booklet.
• Each correct answer will get you 4 marks and 1 mark shall be deduced for each incorrect answer. No mark will be given/
deducted if no bubble is filled. Keep a timer in front of you and stop immediately at the end of 60 min.
• The sheet follows a particular syllabus. Do not attempt the sheet before you have completed your preparation for that
syllabus. Refer syllabus sheet in the starting of the book for the syllabus of all the DPP sheets.
• After completing the sheet check your answers with the solution booklet and complete the Result Grid. Finally spend time
to analyse your performance and revise the areas which emerge out as weak in your evaluation.

DIRECTIONS (Q.1-Q.21) : There are 21 multiple choice questions. (a) 44 N (b) 88 N


Each question has 4 choices (a), (b), (c) and (d), out of which ONLY (c) 132 N (d) 22 N
ONE choice is correct.
Q.1 Length of a wire is doubled, when 20 × 108 N/m2 stress is Q.3 The work done in increasing the length of a one metre long
applied on it. Its Young’s modulus of elasticity in N/m2 will wire of cross-sectional area 1mm2 through 1mm will be
be (Y = 2 × 1011 N/m2)
(a) 20 × 108 (b) 20 × 109
(c) 20 × 10 10 (d) 10 × 108 (a) 250 J (b) 10 J
Q.2 A steel wire of uniform cross-sectional area 2mm2 is heated (c) 5 J (d) 0.1 J
upto 50°C and clamped rigidly at two ends. If the
Q.4 A spring is stretched by 3cm when a load of 5.4 × 10 6 dyne
temperature of wire falls to 30° then change in tension in
is suspended from it. Work done will be-
the wire will be, if coefficient of linear expansion of steel
is 1.1 × 10–5 /°C and young's modulus of elasticity of steel (a) 8.1 × 106 J (b) 8 × 106 J
is 2 × 1011 N/m2.
(c) 8.0 × 106 erg (d) 8.1 ×106 erg

RESPONSE GRID 1. 2. 3. 4.

Space for Rough Work


t.me/Ebooks_Encyclopedia27. t.me/Magazines4all

EBD_7156
2 DPP/ P 20
Q.5 A wire of length 1m and area of cross section 4 × 10-8 m2 Q.12 The compressibility of water is 5 × 10–10 m2/N. If it is
increases in length by 0.2 cm when a force of 16 N is applied. subjected to a pressure of 15 MPa, the fractional decrease in
Value of Y for the material of the wire will be volume will be -
(a) 2 × 106 N/m2 (b) 2 × 1011 kg/m2 (a) 3.3 × 10–5 (b) 5.6 × 10–4
(c) 2 × 1011 N/mm2 (d) 2 × 1011 N/m2 (c) 7.5 × 10–3 (d) 1.5 × 10–2
Q.6 The volume of a solid rubber ball when it is carried from the Q.13 The Young’s modulus of steel is 2 × 1011 N/m2 and its
surface to the bottom of a 200 m deep lake decreases by coefficient of linear expansion is 1.1 × 10–5 per deg. The
0.1%. The value for bulk modulus of elasticity for rubber will pressure to be applied to the ends of a steel cylinder to
be keep its length constant on raising its temperature by 100ºC,
(a) 2 × 109 Pa (b) 2 × 106 Pa will be -
(c) 2 × 104 Pa (d) 2 × 10–4 Pa (a) 5.5 × 104 N/m2 (b) 1.8 × 106 N/m2
Q.7 A steel wire is 4.0 m long and 2 mm in diameter. Young’s (c) 2.2 × 108 N/m2 (d) 2.0 × 1011 N/m2
modulus of steel is 1.96 × 1011 N/m2. If a mass of 20 kg is
suspended from it the elongation produced will be - Q.14 For a given material, the Young’s modulus is 2.4 times that
of rigidity modulus. It’s poisson’s ratio is
(a) 2.54 mm (b) 1.27 mm
(a) 1.2 (b) 1.02
(c) 0.64 mm (d) 0.27 mm
Q.8 A brass rod is to support a load of 400 N. If its elastic limit is (c) 0.2 (d) 2
4.0 × 108 N/m2 its minimum diameter must be - Q.15 A wire of length 1m is stretched by a force of 10N. The
(a) 1.13 mm (b) 2.26 mm area of cross-section of the wire is 2 × 10–6 m2 and Y is
2 × 1011N/m2. Increase in length of the wire will be -
(c) 3.71 mm (d) 4.52 mm
(a) 2.5 × 10–5 cm (b) 2.5 × 10–5 mm
Q.9 A 4.0 m long copper wire of cross sectional area 1.2 cm 2 is
stretched by a force of 4.8 × 103 N stress will be - (c) 2.5 × 10–5 m (d) None of these
(a) 4.0 × 107 N/mm2 (b) 4.0 × 107 KN/m2 Q.16 A stress of 1kg/mm2 is applied on a wire. If the modulus of
(c) 4.0 × 107 N/m2 (d) None of these elasticity of the wire is 1010 dyne/cm2, then the percentage
increase in the length of the wire will be
Q.10 A copper rod 2m long is stretched by 1mm. Strain will be -
(a) 0.007 (b) 0.0098 (c) 98 (d) 9.8
(a) 10–4, volumetric (b) 5 × 10–4, volumetric
(c) 5 × 10–4, longitudinal (d) 5 × 10–3, volumetric Q.17 A uniform steel wire of density 7800kg/m3
is 2.5 m long
and weighs 15.6 × 10–3 kg. It extends by 1.25 mm when
Q.11 A wire of cross sectional area 3mm2 is just stretched between loaded by 8kg. Calculate the value of young’s modulus of
two fixed points at a temperature of 20°C. Determine the elasticity for steel.
tension when the temperature falls to 20°C. Coefficient of
linear expansion a = 10–5 /°C and Y = 2 × 1011 N/m2. (a) 1.96 × 1011 N/m2 (b) 19.6 × 1011 N/m2
(a) 120 KN (b) 20 N (c) 120 N (d) 102 N (c) 196 × 1011 N/m2 (d) None of these

5. 6. 7. 8. 9.
RESPONSE
10. 11. 12. 13. 14.
GRID
15. 16. 17.

Space for Rough Work


t.me/Ebooks_Encyclopedia27. t.me/Magazines4all

DPP/ P 20 3
Q.18 A metallic wire is suspended by suspending weight to it. If S (2) Reciprocal of bulk modulus of elasticity is called
is longitudinal strain and Y its young's modulus of elasticity compressibility
then potential energy per unit volume will be
(3) Hollow shaft is much stronger than a solid rod of same
1 2 2 1 2 length and same mass
(a) YS (b) YS
2 2 (4) It is difficult to twist a long rod as compared to small
rod
1 2
(c) YS (d) 2YS2 Q.23 Which statements are false for a metal?
2
Q.19 The lengths and radii of two wires of same material are (1) Y<h (2) Y=h
respectively L, 2L, and 2R, R. Equal weights are applied on
then. If the elongations produced in them are l1 and l2 (3) Y < 1/ h (4) Y>h
respectively then their ratio will be Q.24 Which of the following relations are false
(a) 2 : 1 (b) 4 : 1
(1) 3Y = K (1 - s ) (2) s = ( 6K + h) Y
(c) 8 : 1 (d) 1 : 8
9hY 0.5Y - h
Q.20 The ratio of radii of two wires of same material is 2 : 1. If (3) K = (4) s =
these wires are stretched by equal forces, then the ratio of Y+h h
stresses produced in them will be
DIRECTIONS (Q.25-Q.27) : Read the passage given below and
(a) 1 : 2 (b) 2 : 1 answer the questions that follows :

(c) 1 : 4 (d) 4 : 1 A bar of cross section A is subjected to equal and opposite tensile
Q.21 A rod of length l and area of cross-section A is heated from forces F at its ends. Consider a plane through the bar making an
0°C to 100°C. The rod is so placed that it is not allowed to angle q with a plane at right angles to the bar as shown in figure.
increase in length, then the force developed is proportional to
(a) l (b) l –1 (c) A (d) A–1
F q F
DIRECTIONS (Q.22-Q.24) : In the following questions,
more than one of the answers given are correct. Select the
correct answers and mark it according to the following Q.25 The tensile stress at this plane in terms of F, A and q is
codes:
Codes : Fcos2 q F
(a) (b)
A A cos2 q
(a) 1, 2 and 3 are correct (b) 1 and 2 are correct
Fsin 2 q F
(c) 2 and 4 are correct (d) 1 and 3 are correct (c) (d)
A A sin 2 q
Q.22 Mark the correct statements
Q.26 In the above problem, for what value of q is the tensile
(1) Sliding of molecular layer is much easier than stress maximum ?
compression or expansion
(a) Zero (b) 90° (c) 45° (d) 30°

RESPONSE 18. 19. 20. 21. 22.


GRID 23. 24. 25. 26.

Space for Rough Work


t.me/Ebooks_Encyclopedia27. t.me/Magazines4all

EBD_7156
4 DPP/ P 20
Q.27 The shearing stress at the plane, in terms of F, A and q is (c) Statement -1 is False, Statement-2 is True.

Fcos 2q Fsin 2q (d) Statement -1 is True, Statement-2 is False.


(a) (b)
2A 2A Q.28 Statement -1 : Steel is more elastic than rubber.
Fsin q F cos q Statement -2 : Under given deforming force, steel is
(c) (d) deformed less than rubber.
A A
DIRECTIONS (Qs. 28-Q.30) : Each of these questions contains two Q.29 Statement -1 : Bulk modulus of elasticity (K) represents
statements: Statement-1 (Assertion) and Statement-2 (Reason). Each incompressibility of the material.
of these questions has four alternative choices, only one of which is Statement -2 : Bulk modulus of elasticity is proportional
the correct answer. You have to select the correct choice.
to change in pressure.
(a) Statement-1 is True, Statement-2 is True; Statement-2 is a
Q.30 Statement -1 :The bridges declared unsafe after a long use.
correct explanation for Statement-1.
Statement -2 : Elastic strength of bridges losses with time.
(b) Statement-1 is True, Statement-2 is True; Statement-2 is
NOT a correct explanation for Statement-1.

RESPONSE GRID 27. 28. 29. 30.

DAILY PRA CTICE PROBLEM SHEET 20 - PHYSICS


Total Questions 30 Total Marks 120
Attempted Correct
Incorrect N et Score
Cut-off Score 32 Qualifying Score 52
Success Gap = Net Score – Qualifying Score
Net Score = (Correct × 4) – (Incorrect × 1)

Space for Rough Work


t.me/Ebooks_Encyclopedia27. t.me/Magazines4all

DPP - Daily Practice Problems


Name : Date :

Start Time : End Time :

SYLLABUS : Fluid Mechanics


21
Max. Marks : 112 Time : 60 min.
GENERAL INSTRUCTIONS
• The Daily Practice Problem Sheet contains 28 MCQ's. For each question only one option is correct. Darken the correct
circle/ bubble in the Response Grid provided on each page.
• You have to evaluate your Response Grids yourself with the help of solution booklet.
• Each correct answer will get you 4 marks and 1 mark shall be deduced for each incorrect answer. No mark will be given/
deducted if no bubble is filled. Keep a timer in front of you and stop immediately at the end of 60 min.
• The sheet follows a particular syllabus. Do not attempt the sheet before you have completed your preparation for that
syllabus. Refer syllabus sheet in the starting of the book for the syllabus of all the DPP sheets.
• After completing the sheet check your answers with the solution booklet and complete the Result Grid. Finally spend time
to analyse your performance and revise the areas which emerge out as weak in your evaluation.

DIRECTIONS (Q.1-Q.20) : There are 20 multiple choice Q.3 The amount of work done in blowing a soap bubble such that its
questions. Each question has 4 choices (a), (b), (c) and (d), out diameter increases from d to D is (T = surface tension of the
of which ONLY ONE choice is correct. solution)
Q.1 The force required to separate two glass plates of area
(a) 4p( D 2 - d 2 )T (b) 8p( D 2 - d 2 )T
10–2m–2 with a film of water 0.05 m thick between them, is
(Surface tension of water is 70 × 10–3 N/m) (c) p( D 2 - d 2 )T (d) 2p( D 2 - d 2 )T
(a) 28 N (b) 14 N (c) 50 N (d) 38 N
Q.4 A film of water is formed between two straight parallel
Q.2 A thin metal disc of radius r floats on water surface and bends wires of length 10 cm each separated by 0.5 cm. If their
the surface downwards along the perimeter making an angle q separation is increased by 1 mm while still maintaining their
with vertical edge of the disc. If the disc displaces a weight of parallelism, how much work will have to be done (Surface
water W and surface tension of water is T, then the weight of tension of water = 70 × 10–2 N/m)
metal disc is (a) 7.22 × 10–6 Joule (b) 1.44 × 10–5 Joule
(a) 2prT + W (b) 2prT cos q - W (c) 2.88 × 10–5 Joule (d) 5.76 × 10–5 Joule
(c) 2prT cos q + W (d) W - 2prT cos q

RESPONSE GRID 1. 2. 3. 4.
Space for Rough Work
t.me/Ebooks_Encyclopedia27. t.me/Magazines4all

EBD_7156
2 DPP/ P 21
Q.5 The liquid meniscus in capillary tube will be convex, if the Q.11 Which graph represents the variation of surface tension
angle of contact is with temperature over small temperature ranges for water ?
(a) Greater than 90° (b) Less than 90°
(c) Equal to 90° (d) Equal to 0°
Q.6 Two soap bubbles of radii r1 and r2 equal to 4 cm and 5 cm S.T. S.T.
(a) (b)
are touching each other over a common surface S1 S2
(shown in figure). Its radius will be S1 Temp. Temp.
(a) 4 cm
4 cm 5 cm
(b) 20 cm
(c) 5 cm S2 S.T. S.T.
(d) 4.5 cm (c) (d)
Q.7 The radii of two soap bubbles are r1 and r2 . In isothermal
Temp. Temp.
conditions, two meet together in vaccum. Then the radius Q.12 A solid sphere of density h ( > 1 ) times lighter than water
R of the resultant bubble is given by is suspended in a water tank by a string tied to its base as
(a) R = (r1 + r2 ) / 2 (b) R = r1 ( r1r2 + r2 ) shown in fig. If the mass of the sphere is m then the tension
(c) R 2 = r12 + r22 (d) R = r1 + r2 in the string is given by
Q.8 Two parallel glass plates are dipped partly in the liquid of (a) æ h - 1ö
çè h ÷ø mg
density ' d ' keeping them vertical. If the distance between
(b) hmg
the plates is ' x ' , surface tension for liquids is T and angle
mg
of contact is q , then rise of liquid between the plates due (c)
h -1
to capillary will be (d) ( h - 1)mg
T cos q 2T cos q 2T T cos q
(a) (b) (c) (d) Q.13 A candle of diameter d is floating on a liquid in a cylindrical
xd xdg xdg cos q xdg
container of diameter D ( D >> d ) as shown in figure. If it
Q.9 A capillary tube of radius R is immersed in water and water is burning at the rate of 2 cm/ hour then the top of the candle
rises in it to a height H . Mass of water in the capillary will
tube is M . If the radius of the tube is doubled, mass of
water that will rise in the capillary tube will now be
(a) M (b) 2M (c) M / 2 (d) 4M L

Q.10 In a surface tension experiment with a capillary tube water


L
rises upto 0.1 m. If the same experiment is repeated on an
artificial satellite, which is revolving around the earth, water d

will rise in the capillary tube upto a heights of


(a) 0.1 m D

(b) 0.2 m (a) Remain at the same height


(c) 0.98 m (b) Fall at the rate of 1 cm/hour
(d) Full length of the capillary tube (c) Fall at the rate of 2 cm/hour
(d) Go up the rate of 1 cm/hour

RESPONSE 5. 6. 7. 8. 9.
GRID 10. 11. 12. 13.

Space for Rough Work


t.me/Ebooks_Encyclopedia27. t.me/Magazines4all

DPP/ P 21 3
Q.14 A viscous fluid is flowing through a cylindrical tube. The circular hole of radius R at a depth 4y from the top. When
velocity distribution of the fluid is best represented by the the tank is completely filled with water the quantities of
diagram water flowing out per second from both the holes are the
same. Then R is equal to
(a) (b) L L
(a) 2pL (b) (c) L (d)
2p
2p
Q.18 Water is filled in a cylindrical container to a height of 3m.
(c) (d) None of these The ratio of the cross-sectional area of the orifice and the
beaker is 0.1. The square of the speed of the liquid coming
out from the orifice is (g = 10m/s2)
Q.15 When a body falls in air, the resistance of air depends to a (a) 50 m 2 / s 2
great extent on the shape of the body, 3 different shapes
are given. Identify the combination of air resistances which (b) 50.5 m 2 / s2
truly represents the physical situation. (The cross sectional 3m
(c) 51 m 2 / s 2 52.5 cm
areas are the same).
(d) 52 m / s 2 2
R R R
Q.19 An incompressible liquid flows through a horizontal tube
as shown in the following fig. Then the velocity v of the
fluid is
W W W v2 = 1.5 m/s
(1) (2) (3) A
Disc Ball Cigar shaped

v1 = 3 m/s A
(a) 1< 2 < 3 (b) 2 < 3 <1
1.5A
(c) 3 < 2 <1 (d) 3 <1< 2
v
Q.16 A homogeneous solid cylinder of length L( L < H / 2) .
Cross-sectional area A/5 is immersed such that it floats (a) 3.0 m/s (b) 1.5 m/s (c) 1.0 m/s (d) 2.25 m/s
with its axis vertical at the liquid-liquid interface with length Q.20 Radius of a capillary tube is 2 × 10–3m. A liquid of weight
L/4 in the denser liquid as shown in the fig. The lower 6.28 × 10 –4 N may remain in the capillary tube then the
density liquid is open to atmosphere having pressure P0. surface tension of liquid will be
Then density of solid is given by (a) 5 × 10–3 N/m (b) 5 × 10–2 N/m
(c) 5 N/m (d) 50 N/m
H/2 d
3L/4 DIRECTIONS (Q.21-Q.23) : In the following questions,
more than one of the answers given are correct. Select the
H/2 2d
correct answers and mark it according to the following
5 4 d codes:
(a) d (b) d (c) d (d)
4 5 5 Codes :
Q.17 A large open tank has two holes in the wall. One is a square (a) 1, 2 and 3 are correct (b) 1 and 2 are correct
hole of side L at a depth y from the top and the other is a (c) 2 and 4 are correct (d) 1 and 3 are correct

RESPONSE 14. 15. 16. 17. 18.


GRID 19. 20.

Space for Rough Work


t.me/Ebooks_Encyclopedia27. t.me/Magazines4all

EBD_7156
4 DPP/ P 21
Q.21 The temperature at which the surface tension of water is Q.24 The force exerted by air on surface P is
zero (a) 0.1 N (b) 1.0023 N (c) 105 N (d) 1.0 N
(1) 370°C Q.25 The force exerted by mercury drop on the surface P is
(2) 0°C (a) 0.1 N (b) 1.0023 N
(3) Slightly less than 647 K (c) 0.00023 N (d) 0.10023 N
(4) 277 K
Q.22 Which of the following statements are true in case when DIRECTIONS (Qs. 26-Q.28) : Each of these questions contains
two water drops coalesce and make a bigger drop? two statements: Statement-1 (Assertion) and Statement-2
(1) Energy is released. (Reason). Each of these questions has four alternative choices,
(2) Energy is absorbed. only one of which is the correct answer. You have to select the
(3) The surface area of the bigger drop is smaller than the correct choice.
sum of the surface areas of both the drops. (a) Statement-1 is True, Statement-2 is True; Statement-2 is a
(4) The surface area of the bigger drop is greater than the correct explanation for Statement-1.
sum of the surface areas of both the drops. (b) Statement-1 is True, Statement-2 is True; Statement-2 is
Q.23 An air bubble in a water tank rises from the bottom to the NOT a correct explanation for Statement-1.
top. Which of the following statements are true? (c) Statement -1 is False, Statement-2 is True.
(1) Bubble rises upwards because pressure at the bottom (d) Statement -1 is True, Statement-2 is False.
is greater than that at the top. Q.26 Statement-1 : A large soap bubble expands while a small
(2) As the bubble rises, its size increases. bubble shrinks, when they are connected to each other by a
(3) Bubble rises upwards because pressure at the bottom capillary tube.
is less than that at the top. Statement-2: The excess pressure inside bubble (or drop)
(4) As the bubble rises, its size decreases. is inverse ly proportional to the radius.
Q.27 Statement-1 : Bernoulli’s theorem holds for
DIRECTIONS (Q.24-Q.25) : Read the passage given below incompressible, non-viscous fluids.
and answer the questions that follows : v2
There is a small mercury drop of radius 4.0mm. A surface P of area Statement-2 : The factor is called velocity head.
2g
1.0 mm2 is placed at the top of the drop. Atmospheric pressure = Q.28 Statement-1 : The velocity increases, when water flowing
105 Pa. Surface tension of mercury = 0.465 N/m. Gravity ef- in broader pipe enter a narrow pipe.
fect is negligible. Statement-2 : According to equation of continuity, product
of area and velocity is constant.

RESPONSE 21. 22. 23. 24. 25.


GRID 26. 27. 28.

DAILY PRA CTICE PROBLEM SHEET 21 - PHYSICS


Total Questions 28 Total Marks 112
Attempted Correct
Incorrect N et Score
Cut-off Score 28 Qualifying Score 42
Success Gap = Net Score – Qualifying Score
Net Score = (Correct × 4) – (Incorrect × 1)
Space for Rough Work
t.me/Ebooks_Encyclopedia27. t.me/Magazines4all

DPP - Daily Practice Problems


Name : Date :

Start Time : End Time :

SYLLABUS : Thermal Expansion, Calorimetry and Change of State


22
Max. Marks : 116 Time : 60 min.
GENERAL INSTRUCTIONS
• The Daily Practice Problem Sheet contains 29 MCQ's. For each question only one option is correct. Darken the correct
circle/ bubble in the Response Grid provided on each page.
• You have to evaluate your Response Grids yourself with the help of solution booklet.
• Each correct answer will get you 4 marks and 1 mark shall be deduced for each incorrect answer. No mark will be given/
deducted if no bubble is filled. Keep a timer in front of you and stop immediately at the end of 60 min.
• The sheet follows a particular syllabus. Do not attempt the sheet before you have completed your preparation for that
syllabus. Refer syllabus sheet in the starting of the book for the syllabus of all the DPP sheets.
• After completing the sheet check your answers with the solution booklet and complete the Result Grid. Finally spend time
to analyse your performance and revise the areas which emerge out as weak in your evaluation.

DIRECTIONS (Q.1-Q.20) : There are 20 multiple choice Q.3 An iron tyre is to be fitted on to a wooden wheel 1m in
questions. Each question has 4 choices (a), (b), (c) and (d), out diameter. The diameter of tyre is 6mm smaller than that of
of which ONLY ONE choice is correct. wheel. The tyre should be heated so that its temperature
increases by a minimum of (the coefficient of cubical
Q.1 A glass flask is filled up to a mark with 50 cc of mercury at expansion of iron is 3.6 × 10–5/°C)
18°C. If the flask and contents are heated to 38°C, how much (a) 167°C (b) 334°C (c) 500°C (d) 1000°C
mercury will be above the mark? (a for glass is 9 × 10–6/°C Q.4 A rod of length 20 cm is made of metal. It expands by
and coefficient of real expansion of mercury is 180 × 10–6/ 0.075 cm when its temperature is raised from 0°C to
°C) 100°C. Another rod of a different metal B having the same
(a) 0.85 cc (b) 0.46 cc (c) 0.153 cc (d) 0.05 cc length expands by 0.045 cm for the same change in
Q.2 The coefficient of apparent expansion of mercury in a glass temperature. A third rod of the same length is composed
vessel is 153 × 10–6/°C and in a steel vessel is 144 × 10–6/ of two parts, one of metal A and the other of metal B. This
°C. If a for steel is 12 × 10–6/°C, then that of glass is rod expands by 0.060 cm for the same change in
(a) 9 × 10–6/°C (b) 6 × 10–6/°C temperature. The portion made of metal A has the length
(c) 36 × 10 /°C–6 (d) 27 × 10–6/°C (a) 20 cm (b) 10 cm (c) 15 cm (d) 18 cm

RESPONSE GRID 1. 2. 3. 4.

Space for Rough Work


t.me/Ebooks_Encyclopedia27. t.me/Magazines4all

EBD_7156
2 DPP/ P 22
Q.5 A glass flask of volume one litre at 0°C is filled, level full 0.03 cal/gm°C, latent heat of fusion of lead = 6 cal/gm
of mercury at this temperature. The flask and mercury are and J = 4.2 joule/cal)
now heated to 100°C. How much mercury will spill out, if (a) 410 m/sec (b) 1230 m/sec
coefficient of volume expansion of mercury is 1.82 × 10– (c) 307.5 m/sec (d) None of the above
4/°C andlinear expansion of glass is 0.1 × 10–4/°C? Q.11 The temperature of equal masses of three different liquids
(a) 21.2 cc (b) 15.2 cc (c) 1.52 cc (d) 2.12 cc A, B and C are 12°C, 19°C and 28°C respectively. The
Q.6 The apparent coefficient of expansion of a liquid when temperature when A and B are mixed is 16°C and when B
heated in a copper vessel is C and when heated in a silver and C are mixed is 23°C, The temperature when A and C
vessel is S. If A is the linear coefficient of expansion of are mixed is
copper, then the linear coefficient of expansion of silver (a) 18.2°C (b) 22°C (c) 20.2°C (d) 25.2°C
is Q.12 50 gm of copper is heated to increase its temperature by
C + S - 3A C + 3A - S 10°C. If the same quantity of heat is given to 10 gm of
(a) (b) water, the rise in its temperature is (Specific heat of copper
3 3
S + 3A - C C - S + 3A = 420 Joule-kg–1°C–1)
(c) (d) (a) 5°C (b) 6°C (c) 7°C (d) 8°C
3 3
Q.7 The coefficient of volumetric expansion of mercury is 18 Q.13 A beaker contains 200 gm of water. The heat capacity of
× 10–5/°C. A thermometer bulb has a volume 10–6 m3 and the beaker is equal to that of 20 gm of water. The initial
cross section of stem is 0.004 cm2. Assuming that bulb is temperature of water in the beaker is 20°C. If 440 gm of
filled with mercury at 0°C then the length of the mercury hot water at 92°C is poured in it, the final temperature
column at 100°C is (neglecting radiation loss) will be nearest to
(a) 18.8 mm (b) 9.2 mm (c) 7.4 cm (d) 4.5 cm (a) 58°C (b) 68°C (c) 73°C (d) 78°C
Q.8 A piece of metal weight 46 gm in air, when it is immersed Q.14 One calorie is defined as the amount of heat required to
in the liquid of specific gravity 1.24 at 27°C it weighs 30 raise temperature of 1g of water by 1°C and it is defined
gm. When the temperature of liquid is raised to 42°C the under which of the following condition
metal piece weighs 30.5 gm, specific gravity of the liquid (a) From 14.5°C to 15.5°C at 760 mm of Hg
at 42°C is 1.20, then the linear expansion of the metal will (b) From 98.5°C to 99.5°C at 760 mm of Hg
be (c) From 13.5°C to 14.5°C at 76 mm of Hg
(a) 3.316 × 10–5/°C (b) 2.316 × 10–5/°C (d) From 3.5°C to 4.5°C at 76 mm of Hg
(c) 4.316 × 10 /°C–5 (d) None of these Q.15 A bullet moving with a uniform velocity v, stops suddenly
Q.9 2 kg of ice at – 20°C is mixed with 5 kg of water at 20°C after hitting the target and the whole mass melts be m,
in an insulating vessel having a negligible heat capacity. specific heat S, initial temperature 25°C, melting point
Calculate the final mass of water remaining in the container. 475°C and the latent heat L. Then v is given by
It is given that the specific heats of water and ice are 1 1 mv 2
kcal/kg /°C and 0.5 kcal/kg/°C while the latent heat of fusion (a) mL = mS (475 - 25) + .
2 J
of ice is 80 kcal/kg mv 2
(a) 7 kg (b) 6 kg (c) 4 kg (d) 2 kg (b) mS (475 - 25) + mL =
2J
Q.10 A lead bullet at 27°C just melts when stopped by an
mv 2
obstacle. Assuming that 25% of heat is absorbed by the (c) mS (475 - 25) + mL =
J
obstacle, then the velocity of the bullet at the time of
mv 2
striking (M.P. of lead = 327°C, specific heat of lead = (d) mS (475 - 25) - mL =
2J

5. 6. 7. 8. 9.
RESPONSE
10. 11. 12. 13. 14.
GRID
15.

Space for Rough Work


t.me/Ebooks_Encyclopedia27. t.me/Magazines4all

DPP/ P 22 3
Q.16 A stationary object at 4°C and weighing 3.5 kg falls from a DIRECTIONS (Q.21-Q.23) : In the following questions,
height of 2000 m on a snow mountain at 0°C. If the more than one of the answers given are correct. Select the
temperature of the object just before hitting the snow is correct answers and mark it according to the following
0°C and the object comes to rest immediately? (g = 10m/ codes:
s2 and latent heat of ice = 3.5 × 105 joule/sec), then the
Codes :
object will melt
(a) 1, 2 and 3 are correct (b) 1 and 2 are correct
(a) 2 kg of ice (b) 200 gm of ice
(c) 2 and 4 are correct (d) 1 and 3 are correct
(c) 20 gm of ice (d) 2 gm of ice
Q.21 Heat is supplied to a certain homogenous sample of matter,
Q.17 Density of a substance at 0°C is 10 gm/cc and at 100°C,
at a uniform rate. Its temperature is plotted against time,
its density is 9.7 gm/cc. The coefficient of linear expansion
as shown. Which of the following conclusions can be
of the substance will be
drawn?
(a) 102 (b) 10–2 (c) 10–3 (d) 10–4
Q.18 The real coefficient of volume expansion of glycerine is

Temperature
0.000597 per °C and linear coefficient of expansion of
glass is 0.000009 per°C. Then the apparent volume
coefficient of expansion of glycerine is
(a) 0.000558 per°C (b) 0.00057 per°C Time
(c) 0.00027 per°C (d) 0.00066 per°C (1) Its specific heat capacity is greater in the liquid state
Q.19 A constant volume gas thermometer shows pressure than in the solid state
reading of 50 cm and 90 cm of mercury at 0°C and 100°C (2) Its latent heat of vaporization is greater than its latent
respectively. When the pressure reading is 60 cm of heat of fusion
mercury, the temperature is (3) Its specific heat capacity is greater in the solid state
(a) 25°C (b) 40°C (c) 15°C (d) 12.5°C than in the liquid state
Q.20 A student takes 50gm wax (specific heat = 0.6 kcal/kg°C) (4) Its latent heat of vaporization is smaller than its latent
and heats it till it boils. The graph between temperature and heat of fusion
time is as follows. Heat supplied to the wax per minute and Q.22 A bimetallic strip is formed out of two identical strips,
boiling point are respectively. one of copper and other of brass. The coefficients of linear
expansion of the two metals are αC and α B . On heating,
250
the temperature of the strip goes up by DT and the strip
bends to form an arc of radius of curvature R. Then R is
200 (1) inversely proportional to DT
Temperature (°C)

150 (2) proportional to α B - α C


(3) inversely proportional to α B - αC
100
(4) proportional to DT
50 Q.23 A bimetallic strip is heated. Choose wrong statements.
0 (1) does not bend at all
1 2 3 4 5 6 7 8
(2) gets twisted in the form of an helix
Time (Minute)
(3) bends in the form of an arc with the more expandable
(a) 500 cal, 50°C (b) 1000 cal, 100°C metal inside.
(c) 1500 cal, 200°C (d) 1000 cal, 200°C (4) bend in the form of an arc with the more expandable
metal outside

RESPONSE 16. 17. 18. 19. 20.


GRID 21. 22. 23.

Space for Rough Work


t.me/Ebooks_Encyclopedia27. t.me/Magazines4all

EBD_7156
4 DPP/ P 22
DIRECTIONS (Q.24-Q.26) : Read the passage given below DIRECTIONS (Q. 27-Q.29) : Each of these questions contains
and answer the questions that follows : two statements: Statement-1 (Assertion) and Statement-2
(Reason). Each of these questions has four alternative choices,
In a thermally insulated tube of cross sectional area 4cm2 a liquid only one of which is the correct answer. You have to select
of thermal expansion coefficient 10–3 K–1 is f lowing. Its the correct choice.
velocity at the entrance is 0.1 m/s. At the middle of the tube a
heater of a power of 10kW is heating the liquid. The specific (a) Statement-1 is True, Statement-2 is True; Statement-2 is a
heat capacity of the liquid is 1.5 kJ/(kg K), and its density is correct explanation for Statement-1.
1500 kg/m3 at the entrance. (b) Statement-1 is True, Statement-2 is True; Statement-2 is
Q.24 The rise in temperature of the liquid as it pass through the NOT a correct explanation for Statement-1.
tube is (c) Statement -1 is False, Statement-2 is True.
(d) Statement -1 is True, Statement-2 is False.
1000 1 500
(a) °C (b) °C (c) °C (d) None Q.27 Statement-1 : Fahrenheit is the smallest unit measuring
9 9 9
temperature.
Q.25 What is the density of liquid at the exit ?
Statement-2 : Fahrenheit was the first temperature scale
(a) 1450 kg/m3 (b) 1400 kg/m3
3 used for measuring temperature.
(c) 1350 kg/m (d) None of these
Q.28 Statement-1 : A brass disc is just fitted in a hole in a steel
Q.26 How much bigger is the volume rate of flow at the end of
plate. The system must be cooled to loosen the disc from
the tube than at the entrance in cubic meters ?
the hole.
1 Statement-2 : The coefficient of linear expansion for brass
(a) 9 × 10–5 (b) × 10–5
3 is greater than the coefficient of linear expansion for steel.
4 Q.29 Statement-1 : Laten t heat of fusion of ice is
(c) × 10–5 (d) None
9 336000 J kg–1.
Statement-2 : Latent heat refers to change of state without
any change in temperature.

RESPONSE 24. 25. 26. 27. 28.


GRID 29.

DAILY PRA CTICE PROBLEM SHEET 22 - PHYSICS


Total Questions 29 Total Marks 116
Attempted Correct
Incorrect N et Score
Cut-off Score 28 Qualifying Score 42
Success Gap = Net Score – Qualifying Score
Net Score = (Correct × 4) – (Incorrect × 1)
Space for Rough Work
t.me/Ebooks_Encyclopedia27. t.me/Magazines4all

DPP - Daily Practice Problems


Name : Date :

Start Time : End Time :

SYLLABUS : Heat transfer & Newton’s law of cooling


23
Max. Marks : 120 Time : 60 min.
GENERAL INSTRUCTIONS
• The Daily Practice Problem Sheet contains 30 MCQ's. For each question only one option is correct. Darken the correct
circle/ bubble in the Response Grid provided on each page.
• You have to evaluate your Response Grids yourself with the help of solution booklet.
• Each correct answer will get you 4 marks and 1 mark shall be deduced for each incorrect answer. No mark will be given/
deducted if no bubble is filled. Keep a timer in front of you and stop immediately at the end of 60 min.
• The sheet follows a particular syllabus. Do not attempt the sheet before you have completed your preparation for that
syllabus. Refer syllabus sheet in the starting of the book for the syllabus of all the DPP sheets.
• After completing the sheet check your answers with the solution booklet and complete the Result Grid. Finally spend time
to analyse your performance and revise the areas which emerge out as weak in your evaluation.

DIRECTIONS (Q.1-Q.21) : There are 21 multiple choice Q.2 A wall is made up of two layers A and B. The thickness of
questions. Each question has 4 choices (a), (b), (c) and (d), out the two layers is the same, but materials are different. The
of which ONLY ONE choice is correct. thermal conductivity of A is double than that of B. In thermal
equilibrium the temperature difference between the two
Q.1 Two rods (one semi-circular and other straight) of same ends is 36°C. Then the difference of temperature at the
material and of same cross-sectional area are joined as two surfaces of A will be
shown in the figure. The points A and B are maintained at (a) 6°C (b) 12°C (c) 18°C (d) 24°C
different temperature. The ratio of the heat transferred Q.3 A room is maintained at 20°C by a heater of resistance 20
through a cross-section of a semi-circular rod to the heat ohm connected to 200 volt mains. The temperature is
transferred through a cross section of the straight rod in a uniform through out the room and heat is transmitted
given time is through a glass window of area 1m2 and thickness 0.2 cm.
circular r
(a) 2 : p mi What will be the temperature outside? Given that thermal
od
Se

(b) 1 : 2 conductivity K for glass is 0.2 cal/m/°C/sec and J = 4.2 J/


cal
(c) p : 2
A Straight rod B (a) 15.24°C (b) 15.00°C
(d) 3 : 2 (c) 24.15°C (d) None of these

RESPONSE GRID 1. 2. 3.

Space for Rough Work


t.me/Ebooks_Encyclopedia27. t.me/Magazines4all

EBD_7156
2 DPP/ P 23
Q.4 A composite metal bar of uniform section is made up of Q.9 A cylindrical rod with one end in a steam chamber and the
length 25 cm of copper, 10 cm of nickel and 15 cm of other end in ice results in melting of 0.1 gm of ice per
aluminium. Each part being in perfect thermal contact with second. If the rod is replaced by another with half the length
the adjoining part. The copper end of the composite rod is and double the radius of the first and if the thermal
maintained at 100°C and the aluminium end at 0°C. The 1
conductivity of material of second rod is that of first,
whole rod is covered with belt so that there is no heat loss 4
occurs at the sides. If KCu = 2KAl and KAl = 3KNi, then the rate at which ice melts in gm/sec will be
what will be the temperatures of Cu – Ni and Ni – Al (a) 3.2 (b) 1.6 (c) 0.2 (d) 0.1
junctions respectively Q.10 An ice box used for keeping eatable cold has a total wall
area of 1 metre2 and a wall thickness of 5.0 cm. The thermal
Cu Ni Al conductivity of the ice box is K = 0.01 joule/metre °C. It
100° C 0° C is filled with ice at 0° C along with eatables on a day when
(a) 23.33°C and 78.8°C (b) 83.33°C and 20°C the temperature is 30°C. The latent heat of fusion of ice is
(c) 50°C and 30°C (d) 30°C and 50°C 334 × 103 joules/kg. The amount of ice melted in one day
Q.5 Three rods of the same dimension have thermal is (1 day = 86,400 seconds)
conductivities 3K, 2K and K. They are arranged as shown (a) 776 gm (b) 7760 gm
in fig. with their ends at 100°C, 50°C and 20°C. The (c) 11520 gm (d) 1552 gm
temperature of their junction is Q.11 A solid copper sphere (density r and specific heat capacity
50°C
c) of radius r at an initial temperature 200 K is suspended
(a) 60°C 2K
inside a chamber whose walls are at almost 0 K. The time
100°C
(b) 70°C required (in µ s) for the temperature of the sphere to drop
3K
(c) 50°C K to 100 K is
72 r rc
(b) 7 r rc (c) 27 rrc (d) 7 rrc
20°C
(d) 35°C (a)
7 s 72 s 7 s 27 s
Q.6 A black body is at a temperature of 2880 K. The energy of Q.12 Four rods of identical cross-sectional area and made from
radiation emitted by this object with wavelength between the same metal form the sides of square. The temperature
499 nm and 500 nm is U1, between 999 nm and 1000 nm
is U2 and between 1499 nm and 1500 nm is U3. The Wein's of two diagonally opposite points are T and 2T
constant b = 2.88 × 106 nm K. Then respectively in the steady state. Assuming that only heat
(a) U1 = 0 (b) U3 = 0 (c) U1 > U2 (d) U2 > U1 conduction takes place, what will be the temperature
Q.7 A body initially at 80° C cools to 64° C in 5 minutes and difference between other two points
to 52° C in 10 minutes. The temperature of the body after 2 +1 2
(a) T (b) T
15 minutes will be 2 2 +1
(a) 42.7° C (b) 35° C (c) 47° (d) 40° C (c) 0 (d) None of these
Q.8 A 5 cm thick ice block is there on the surface of water in a Q.13 Consider two hot bodies B1 and B2 which have temperature
lake. The temperature of air is – 10° C; how much time it 100° C and 80° C respectively at t = 0. The temperature
will take to double the thickness of the block of surroundings is 40° C. The ratio of the respective rates
(L = 80 cal/g, Kice = 0.004 erg/s-k, dice = 0.92 g cm–3) of cooling R1 and R2 of these two bodies at t = 0 will be
(a) 1 hour (b) 191 hours (a) R1 : R2 = 3 : 2 (b) R1 : R2 = 5 : 4
. (c) 19.1 hours (d) 1.91 hours (c) R1 : R2 = 2 : 3 (d) R1 : R2 = 4 : 5

RESPONSE 4. 5. 6. 7. 8.
GRID 9. 10. 11. 12. 13.

Space for Rough Work


t.me/Ebooks_Encyclopedia27. t.me/Magazines4all

DPP/ P 23 3
Q.14 A body cools from 60° C to 50° C in 10 minutes. If the Q.20 If between wavelength l and l + dl, el and al be the
room temperature is 25° C and assuming Newton's law of emissive and absorptive powers of a body and El be the
cooling to hold good, the temperature of the body at the emissive power of a perfectly black body, then according
end of the next 10 minutes will be to Kirchoff’s law, which is true
(a) 38.5° C (b) 40° C (c) 42.85° C (d) 45° C (a) el = al = El (b) el El = al
Q.15 The rates of cooling of two different liquids put in exactly (c) el = al El (d) el al El = constant
similar calorimeters and kept in identical surroundings are the
Q.21 Two thermometers A and B are exposed in sunlight. The
same if
bulb of A is painted black, But that of B is not painted. The
(a) The masses of the liquids are equal
(b) Equal masses of the liquids at the same temperature correct statement regarding this case is
are taken (a) Temperature of A will rise faster than B but the final
(c) Different volumes of the liquids at the same temperature will be the same in both
temperature are taken (b) Both A and B show equal rise in beginning
(d) Equal volumes of the liquids at the same temperature (c) Temperature of A will remain more than B
are taken (d) Temperature of B will rise faster
Q.16 For cooking the food, which of the following type of DIRECTIONS (Q.22-Q.24) : In the following questions, more
utensil is most suitable than one of the answers given are correct. Select the correct
(a) High specific heat and low conductivity
answers and mark it according to the following codes:
(b) High specific heat and high conductivity
(c) Low specific heat and low conductivity Codes :
(d) Low specific heat and high conductivity (a) 1, 2 and 3 are correct (b) 1 and 2 are correct
Q.17 Two rods A and B are of equal lengths. Their ends are kept (c) 2 and 4 are correct (d) 1 and 3 are correct
between the same temperature and their area of cross- Q.22 Two bodies A and B have thermal emissivities of 0.01 and
sections are A1 and A2 and thermal conductivities K1 and 0.81 respectively. The outer surface areas of the two bodies
K2. The rate of heat transmission in the two rods will be are the same. The two bodies emit total radiant power at
equal, if the same rate. The wavelength λ B corresponding to
(a) K1 A2 = K 2 A1 (b) K1 A1 = K2 A2
maximum spectral radiancy in the radiation from B is
(c) K1 = K2 (d) K1 A12 = K 2 A22 shifted from the wavelength corresponding to maximum
Q.18 While measuring the thermal conductivity of a liquid, we spectral radiancy in the radiation from A, by 1.00 mm. If
keep the upper part hot and lower part cool, so that the temperature of A is 5802 K
(a) Convection may be stopped (1) The temperature of B is 1934 K
(b) Radiation may be stopped (2) lB = 1.5 mm
(c) Heat conduction is easier downwards (3) The temperature of B is 11604 K
(d) It is easier and more convenient to do so (4) The temperature of B is 2901 K
Q.19 When fluids are heated from the bottom, convection
Q.23 A cane is taken out from a refrigerator at 0°C. The
currents are produced because
atmospheric temperature is 25°C. If t1 is the time taken to
(a) Molecular motion of fluid becomes aligned
heat from 0°C to 5°C and t2 is the time taken from 10°C
(b) Molecular collisions take place within the fluid
(c) Heated fluid becomes more dense than the cold fluid to 15°C, then the wrong statements are
above it (1) t1 > t2 (2) t1 = t2
(d) Heated fluid becomes less dense than the cold fluid (3) There is no relation (4) t1 < t2
above it

RESPONSE 14. 15. 16. 17. 18.


GRID 19. 20. 21. 22. 23.

Space for Rough Work


t.me/Ebooks_Encyclopedia27. t.me/Magazines4all

EBD_7156
4 DPP/ P 23
Q.24 The rate of loss of heat from a body cooling under conditions DIRECTIONS (Q.28-Q.30) : Each of these questions contains two
of forced convection is proportional to its statements: Statement-1 (Assertion) and Statement-2 (Reason). Each
(1) surface area of these questions has four alternative choices, only one of which is
(2) excess of temperature over that of surrounding the correct answer. You have to select the correct choice.
(3) heat capacity
(a) Statement-1 is True, Statement-2 is True; Statement-2 is a
(4) absolute temperature
correct explanation for Statement-1.
DIRECTIONS (Q.25-Q.27) : Read the passage given below (b) Statement-1 is True, Statement-2 is True; Statement-2 is
and answer the questions that follows : NOT a correct explanation for Statement-1.
A brass ball of mass 100g is heated to 100°C and then dropped (c) Statement -1 is False, Statement-2 is True.
into 200g of turpentine in a calorimeter at 15°C. The final (d) Statement -1 is True, Statement-2 is False.
temperature is found to be 23°C. Take specific heat of brass as Q.28 Statement-1 : The equivalent thermal conductivity of two
0.092 cal/g°C and water equivalent of calorimeter as 4g. plates of same thickness in contact (series) is less than
Q.25 The specific heat of turpentine is the smaller value of thermal conductivity.
(a) 0.42 cal/g°c (b) 0.96 cal/g°c Statement-2 : For two plates of equal thickness in contact
(c) 0.72 cal/g°c (d) 0.12 cal/g°c (series) the equivalent thermal conductivity is given by
Q.26 Heat lost by the ball is approximately 2 1 1
= +
(a) 810 cal (b) 610 cal K K1 K 2
(c) 710 cal (d) 510 cal Q.29 Statement-1 : The absorbance of a perfect black body is unity.
Q.27 Heat gained by turpentine and calorimeter is approximately Statement-2 : A perfect black body when heated emits
(a) 810 cal (b) 610 cal radiations of all possible wavelengths at that temperature.
(c) 710 cal (d) 510 cal Q.30 Statement-1 : As temperature of a black body is raised,
wavelength corresponding to maximum energy reduces.
Statement-2 : Higher temperature would mean higher
energy and hence higher wavelength.

RESPONSE 24. 25. 26. 27. 28.


GRID 29. 30.

DAILY PRA CTICE PROBLEM SHEET 23 - PHYSICS


Total Questions 30 Total Marks 120
Attem pted Correct
Incorrect N et Score
Cut-off Score 30 Qualifying Score 48
Success Gap = Net Score – Qualifying Score
Net Score = (Correct × 4) – (Incorrect × 1)
Space for Rough Work
t.me/Ebooks_Encyclopedia27. t.me/Magazines4all

DPP - Daily Practice Problems


Name : Date :

Start Time : End Time :

SYLLABUS : Thermodynamics-1 (Thermal equilibrium, zeroth law of thermodynamics,


24
concept of temperature, Heat, work and internal energy, Different thermodynamic processes)

Max. Marks : 116 Time : 60 min.


GENERAL INSTRUCTIONS
• The Daily Practice Problem Sheet contains 29 MCQ's. For each question only one option is correct. Darken the correct
circle/ bubble in the Response Grid provided on each page.
• You have to evaluate your Response Grids yourself with the help of solution booklet.
• Each correct answer will get you 4 marks and 1 mark shall be deduced for each incorrect answer. No mark will be given/
deducted if no bubble is filled. Keep a timer in front of you and stop immediately at the end of 60 min.
• The sheet follows a particular syllabus. Do not attempt the sheet before you have completed your preparation for that
syllabus. Refer syllabus sheet in the starting of the book for the syllabus of all the DPP sheets.
• After completing the sheet check your answers with the solution booklet and complete the Result Grid. Finally spend time
to analyse your performance and revise the areas which emerge out as weak in your evaluation.

DIRECTIONS (Q.1-Q.21) : There are 21 multiple choice questions. (c) 1.5 × 104 cal of heat flowed into the gas
Each question has 4 choices (a), (b), (c) and (d), out of which ONLY (d) 1.5 × 104 cal of heat flowed out from the gas
ONE choice is correct. Q.3 The latent heat of vaporisation of water is 2240 J/gm. If
Q.1 For an isothermal expansion of a perfect gas, the value of the work done in the process of expansion of 1 g of water
is 168 J, then increase in internal energy is
DP
is (a) 2408 J (b) 2240 J
P (c) 2072 J (d) 1904 J
1/ 2 DV DV DV 2 DV Q.4 One mole of an ideal gas expands at a constant temperature
(a) -g (b) - (c) -g (d) -g
V V V V of 300 K from an initial volume of 10 litres to a final
Q.2 When an ideal gas in a cylinder was compressed volume of 20 litres. The work done in expanding the gas is
isothermally by a piston, the work done on the gas was ( R = 8.31 J/mole - K)
found to be 1.5 × 104 Joule. During this process about
(a) 3.6 × 103 cal of heat flowed out from the gas (a) 750 Joules (b) 1728 Joules
(b) 3.6 × 103 cal of heat flowed into the gas (c) 1500 Joules (d) 3456 Joules

RESPONSE GRID 1. 2. 3. 4.

Space for Rough Work


t.me/Ebooks_Encyclopedia27. t.me/Magazines4all

EBD_7156
2 DPP/ P 24
Q.5 The pressure in the tyre of a car is four times the Q.11 For an adiabatic expansion of a perfect gas, the value of
atmospheric pressure at 300 K. If this tyre suddenly bursts,
DP
its new temperature will be ( g = 1.4) is equal to
P
-0.4 /1.4
(a) 300(4)1.4 / 0.4 (b) 300 æç 1 ö÷ (a) - g
DV
(b) -
DV
è4ø V V
(c) 300(2) - 0.4 /1.4 (d) 300(4)-0.4 /1.4 DV DV
(c) -g (d) -g 2
Q.6 A monoatomic gas ( g = 5 / 3) is suddenly compressed to V V
1 Q.12 If 300 ml of gas at 27°C is cooled to 7°C at constant
of its original volume adiabatically, then the pressure pressure, then its final volume will be
8
of the gas will change to (a) 540 ml (b) 350 ml
24 (c) 280 ml (d) 135 ml
(a) (b) 8 Q.13 A sample of gas expands from volume V1 to V2 . The amount
5
40 of work done by the gas is greatest when the expansion is
(c) (d) 32 times its initial pressure (a) isothermal (b) isobaric
3
(c) adiabatic (d) equal in all cases
8
Q.7 An ideal gas at 27°C is compressed adiabatically to of Q.14 How much work to be done in decreasing the volume of
27
5 and ideal gas by an amount of 2.4 ´ 10-4 m3 at normal
its original volume. If g = , then the rise in temperature
3 temperature and constant normal pressure of 1 ´ 105 N / m 2
is (a) 28 Joule (b) 27 Joule
(a) 450 K (b) 375 k (c) 225 K (d) 405 K
(c) 25 Joule (d) 24 Joule
Q.8 A given system undergoes a change in which the work done
Q.15 One mole of a perfect gas in a cylinder fitted with a piston has
by the system equals the decrease in its internal energy.
The system must have undergone an a pressure P , volume V and temperature T . If the temperature
(a) Isothermal change (b) Adiabatic change is increased by 1 K keeping pressure constant, the increase in
(c) Isobaric change (d) Isochoric change volume is
Q.9 Helium at 27° has a volume of 8 litres. It is suddenly 2V V V
compressed to a volume of 1 litre. The temperature of the (a) (b) (c) (d) V
273 91 273
gas will be [ g = 5 / 3] Q.16 Work done by 0.1 mole of a gas at 27o C to double its
(a) 108°C (b) 9327°C (c) 1200°C (d) 927°C
Q.10 One mole of an ideal gas at an initial temperature of TK volume at constant pressure is ( R = 2 cal mol -1o K -1 )
does 6 R joules of work adiabatically. If the ratio of (a) 54 cal (b) 600 cal (c) 60 cal (d) 546 cal
specific heats of this gas at constant pressure and at Q.17 When an ideal diatomic gas is heated at constant pressure,
constant volume is 5/3, the final temperature of gas will the fraction of the heat energy supplied which increases
be the internal energy of the gas, is
(a) (T + 2.4) K (b) (T - 2.4) K 2 3 3 5
(a) (b) (c) (d)
(c) (T + 4) K (d) (T - 4) K 5 5 7 7

5. 6. 7. 8. 9.
RESPONSE
10. 11. 12. 13. 14.
GRID
15. 16. 17.

Space for Rough Work


t.me/Ebooks_Encyclopedia27. t.me/Magazines4all

DPP/ P 24 3
Q.18 When heat is given to a gas in an isothermal change, the DIRECTIONS (Q.22 - Q.23) : In the following questions,
result will be more than one of the answers given are correct. Select the
(a) external work done correct answers and mark it according to the following
(b) rise in temperature codes:
(c) increase in internal energy
(d) external work done and also rise in temp. Codes : (a) 1, 2 and 3 are correct (b) 1 and 2 are correct
(c) 2 and 4 are correct (d) 1 and 3 are correct
Q.19 An ideal gas expands isothermally from a volume V1 to
Q.22 During the melting of a slab of ice at 273 K and one
V2 and then compressed to original volume V1 atmospheric pressure
(1) Positive work is done on the ice-water system by the
adiabatically. Initial pressure is P1 and final pressure is
atmosphere
P3 . The total work done is W . Then (2) Positive work is done by ice-water system on the
atmosphere
(a) P3 > P1 ,W > 0 (b) P3 < P1 , W < 0
(3) The internal energy of the ice-water system increases
(c) P3 > P1 ,W < 0 (d) P3 = P1 ,W = 0 (4) The internal energy of the ice-water system decreases
Q.20 An ideal gas expands in such a manner that its pressure and Q.23 One mole of an ideal monatomic gas is taken from A to C
along the path ABC. The temperature of the gas at A is T0.
volume can be related by equation PV 2 = constant. During For the process ABC –
this process, the gas is P
(a) heated 2P0 C

(b) cooled T0
P0 A
(c) neither heated nor cooled B
(d) first heated and then cooled
Q.21 In the following P - V diagram two adiabatics cut two
V0 2V0
isothermals at temperatures T1 and T2 (fig.). The value of (1) Work done by the gas is RT0
Va 11
will be (2) Change in internal energy of the gas is RT0
Vd 2
P
11
a b T1 (3) Heat absorbed by the gas is RT0
2
d
T2 13
(4) Heat absorbed by the gas is RT0
2
DIRECTIONS (Q.24-Q.26) : Read the passage given below
and answer the questions that follows :
Va Vd Vb Vc V
In the figure n mole of a monoatomic ideal gas undergo the
Vb Vc Vd
(a) (b) (c) (d) VbVc process ABC as shown in the P-V diagram. The process AB is
Vc Vb Va isothermal and BC is isochoric. The temperature of the gas at A
is T0. Total heat given to the gas during the process ABC is
measured to be Q.

RESPONSE 18. 19. 20. 21. 22.


GRID 23.

Space for Rough Work


t.me/Ebooks_Encyclopedia27. t.me/Magazines4all

EBD_7156
4 DPP/ P 24
DIRECTIONS (Q. 27-Q.29) : Each of these questions contains
P
A C two statements: Statement-1 (Assertion) and Statement-2
(Reason). Each of these questions has four alternative choices,
only one of which is the correct answer. You have to select the
correct choice.
B
(a) Statement-1 is True, Statement-2 is True; Statement-2 is a
2V 6V correct explanation for Statement-1.
Q.24 Temperature of the gas at C is equal to (b) Statement-1 is True, Statement-2 is True; Statement-2 is
NOT a correct explanation for Statement-1.
(a) T0 (b) 3 T0 (c) Statement -1 is False, Statement-2 is True.
(c) 6 T0 (d) 2 T0 (d) Statement -1 is True, Statement-2 is False.
Q.25 Heat absorbed by the gas in the process BC Q.27 Statement-1 : The isothermal curves intersect each other
(a) 3nRT0 (b) nRT0 at a certain point.
(c) 2nRT0 (d) 6nRT0 Statement-2 : The isothermal change takes place slowly,
so the isothermal curves have very little slope.
Q.26 The average molar heat capacity of the gas in process ABC
Q.28 Statement-1 : In adiabatic compression, the internal
is
energy and temperature of the system get increased.
Q Q Statement-2 : The adiabatic compression is a slow process.
(a) (b) Q.29 Statement-1 : The specific heat of a gas in an adiabatic
nT0 2nT0
process is zero and in an isothermal process is infinite.
Q 2Q Statement-2 : Specific heat of a gas is directly proportional
(c) (d)
3nT0 nT0 to change of heat in system and inversely proportional to
change in temperature.

RESPONSE 24. 25. 26. 27. 28.


GRID 29.

DAILY PRA CTICE PROBLEM SHEET 24 - PHYSICS


Total Questions 29 Total Marks 116
Attempted Correct
Incorrect N et Score
Cut-off Score 28 Qualifying Score 44
Success Gap = Net Score – Qualifying Score
Net Score = (Correct × 4) – (Incorrect × 1)
Space for Rough Work
t.me/Ebooks_Encyclopedia27. t.me/Magazines4all

DPP - Daily Practice Problems


Name : Date :

Start Time : End Time :

SYLLABUS : Thermodynamics-2 (1st and 2nd laws of thermodynamics, Reversible &


25
irreversible processes, Carnot engine and its efficiency)

Max. Marks : 112 Time : 60 min.


GENERAL INSTRUCTIONS
• The Daily Practice Problem Sheet contains 28 MCQ's. For each question only one option is correct. Darken the correct
circle/ bubble in the Response Grid provided on each page.
• You have to evaluate your Response Grids yourself with the help of solution booklet.
• Each correct answer will get you 4 marks and 1 mark shall be deduced for each incorrect answer. No mark will be given/
deducted if no bubble is filled. Keep a timer in front of you and stop immediately at the end of 60 min.
• The sheet follows a particular syllabus. Do not attempt the sheet before you have completed your preparation for that
syllabus. Refer syllabus sheet in the starting of the book for the syllabus of all the DPP sheets.
• After completing the sheet check your answers with the solution booklet and complete the Result Grid. Finally spend time
to analyse your performance and revise the areas which emerge out as weak in your evaluation.

DIRECTIONS (Q.1-Q.19) : There are 19 multiple choice questions. Q.2 An ideal gas is taken from point A to the point B , as
Each question has 4 choices (a), (b), (c) and (d), out of which ONLY shown in the P - V diagram, keeping the temperature
ONE choice is correct. constant. The work done in the process is
P
Q.1 Six moles of an ideal gas performs a cycle shown in figure.
A
If the temperature TA = 600 K , TB = 800 K , TC = 2200 K PA

and TD = 1200 K , the work done per cycle is


P
PB B
B C

V
A
O VA VB
D
1
(a) ( PA - PB )(VB - VA ) (b) ( PB - PA )(VB + V A )
2
T 1 1
(c) ( PB - PA )(VB - V A ) (d) ( PB + PA )(VB - V A )
(a) 20 kJ (b) 30 kJ (c) 40 kJ (d) 60 kJ 2 2

RESPONSE GRID 1. 2.
Space for Rough Work
t.me/Ebooks_Encyclopedia27. t.me/Magazines4all

EBD_7156
2 DPP/ P 25
Q.3 In the diagrams (i) to (iv) of variation of volume with Q.8 An ideal heat engine working between temperature T1 and
changing pressure is shown. A gas is taken along the path T2 has an efficiency h, the new efficiency of engine if both
ABCD. The change in internal energy of the gas will be the source and sink temperature are doubled, will be
V V
D
C D C h
(a) (b) h (c) 2 h (d) 3 h
(i) (ii) 2
A A B
Q.9 Efficiency of a Carnot engine is 50% when temperature
B
of outlet is 500 K. In order to increase efficiency up to
P P
V
D C
V D 60% keeping temperature of intake the same what will be
C
temperature of outlet
(iii) (iv) (a) 200 K (b) 400 K (c) 600 K (d) 800 K
A B A B Q.10 A scientist says that the efficiency of his heat engine which
P P operates at source temperature 120ºC and sink temperature
(a) Positive in all cases (i) to (iv) 27ºC is 26%, then
(b) Positive in cases (i), (ii) and (iii) but zero in case (iv) (a) It is impossible
(c) Negative in cases (i), (ii) and (iii) but zero in case (iv) (b) It is possible but less probable
(d) Zero in all cases (c) It is quite probable
Q.4 A monoatomic ideal gas, initially at temperature T1, is (d) Data are incomplete
enclosed in a cylinder fitted with a frictionless piston. The Q.11 The efficiency of Carnot’s engine operating between
gas is allowed to expand adiabatically to a temperature T2 reservoirs, maintained at temperatures 27°C and –123°C,
by releasing the piston suddenly. If L1and L2 are the lengths is
of the gas column before and after expansion respectively, (a) 50% (b) 24% (c) 0.75% (d) 0.4%
then T1 / T2 is given by Q.12 The temperature of sink of Carnot engine is 27°C and
æ L1 ö
2/3
L1 L2 æ L2 ö
2/3 Efficiency of engine is 25%. Then temperature of source
(a) ç ÷ (b) L2 (c) L1 (d) ç ÷ is
è L2 ø è L1 ø
(a) 227°C (b) 327°C (c) 127°C (d) 27°C
Q.5 A gas mixture consists of 2 moles of oxygen and 4 moles Q.13 In changing the state of thermodynamics from A to B state,
argon at temperature T. Neglecting all vibrational modes, the heat required is Q and the work done by the system is
the total internal energy of the system is
W. The change in its internal energy is
(a) 4 RT (b) 15 RT (c) 9 RT (d) 11 RT
Q.6 Two Carnot engines A and B are operated in succession. Q -W
(a) Q + W (b) Q – W (c) Q (d)
The first one, A receives heat from a source at T1 = 800 K 2
Q.14 The first law of thermodynamics is concerned with the
and rejects to sink at T2 K . The second engine B receives
conservation of
heat rejected by the first engine and rejects to another sink
(a) Momentum (b) Energy
at T3 = 300 K . If the work outputs of two engines are equal,
(c) Mass (d) Temperature
then the value of T2 will be Q.15 A system is given 300 calories of heat and it does 600
(a) 100K (b) 300K (c) 550K (d) 700 K
joules of work. The internal energy of the system change
Q.7 A Carnot engine whose low temperature reservoir is at 7°C
in this process is
has an efficiency of 50%. It is desired to increase the
(J = 4.18 Joule/cal)
efficiency to 70%. By how many degrees should the
temperature of the high temperature reservoir be increased (a) 654 Joule (b) 156.5 Joule
(a) 840K (b) 280 K (c) 560 K (d) 380K (c) –300 Joule (d) – 528.2 Joule

3. 4. 5. 6. 7.
RESPONSE
8. 9. 10. 11. 12.
GRID
13. 14. 15.

Space for Rough Work


t.me/Ebooks_Encyclopedia27. t.me/Magazines4all

DPP/ P 25 3
Q.16 110 J of heat is added to a gaseous system, whose internal (2) That the process should be too fast
energy change is 40 J, then the amount of external work (3) That the process should be slow so that the working
done is substance should remain in thermal and mechanical
(a) 150 J (b) 70 J (c) 110 J (d) 40 J equilibrium with the surroundings
Q.17 For free expansion of the gas which of the following is (4) The loss of energy should be zero and it should be
true quasistatic
(a) Q = W = 0 and DEint = 0 Q.22 One mole of an ideal gas is taken through the cyclic through
(b) Q = 0, W > 0 and DEint = -W the cyclic process shown in the V-T diagram, where V =
(c) W = 0, Q > 0 , and DEint = Q volume and T = absolute temperature of the gas. Which of
the following statements are correct
(d) W > 0, Q < 0 and DEint = 0
V
Q.18 In a given process for an ideal gas, dW = 0 and dQ < 0 . 2V0
A B

Then for the gas


(a) The temperature will decrease
(b) The volume will increase V0
D
C
(c) The pressure will remain constant
T
(d) The temperature will increase T0 2T0
Q.19 The specific heat of hydrogen gas at constant pressure is (1) Heat is given out by the gas
C p = 3.4 ´ 103 cal / kg oC and at constant volume is (2) Heat is absorbed by the gas
(3) The magnitude of the work done by the gas is RT0 (ln 2)
CV = 2.4 ´ 103 cal / kg oC . If one kilogram hydrogen gas is (4) The magnitude of the work done by the gas is V0T0
heated from 10°C to 20°C at constant pressure, the
external work done on the gas to maintain it at constant DIRECTIONS (Q.23-Q.25) : Read the passage given below and
pressure is answer the questions that follows :
(a) 105 cal (b) 104 cal
V-T graph of a process of monoatomic ideal gas is as shown in
(c) 103 cal (d) 5 × 103 cal
figure.
DIRECTIONS (Q.20-Q.22) : In the following questions, V

more than one of the answers given are correct. Select the a b

correct answers and mark it according to the following


codes: d c
T
Codes : (a) 1, 2 and 3 are correct (b) 1 and 2 are correct
(c) 2 and 4 are correct (d) 1 and 3 are correct Q.23 Sum of work done by the gas in process abcd is –
Q.20 Which of the following processes are irreversible? (a) zero (b) positive
(1) Transfer of heat by radiation (c) negative (d) data is insufficient
(2) Electrical heating of a nichrome wire Q.24 Heat is supplied to the gas in process(s) –
(3) Transfer of heat by conduction (a) da, ab and bc (b) da and ab only
(4) Isothermal compression (c) da only (d) ab and bc only
Q.21 For a reversible process, unnecessary conditions are Q.25 Change in internal energy of the gas is zero in process(s) –
(1) In the whole cycle of the system, the loss of any type (a) da, ab and bc (b) da and bc only
of heat energy should be zero (c) da only (d) da and ab only

RESPONSE 16. 17. 18. 19. 20.


GRID 21. 22. 23. 24. 25.

Space for Rough Work


t.me/Ebooks_Encyclopedia27. t.me/Magazines4all

EBD_7156
4 DPP/ P 25
DIRECTIONS (Q.26-Q.28) : Each of these questions contains two Q.26 Statement-1 : It is not possible for a system, unaided by an
statements: Statement-1 (Assertion) and Statement-2 (Reason). Each external agency to transfer heat from a body at lower
of these questions has four alternative choices, only one of which is temperature to another body at higher temperature.
the correct answer. You have to select the correct choice. Statement-2 : According to Clausius statement, “No
process is possible whose sole result is the transfer of
(a) Statement-1 is True, Statement-2 is True; Statement-2 is a heat from a cooled object to a hotter object.
correct explanation for Statement-1. Q.27 Statement-1 : A room can be warmed by opening the door
(b) Statement-1 is True, Statement-2 is True; Statement-2 is of a refrigerator in a closed room.
NOT a correct explanation for Statement-1. Statement-2 : Head flows from lower temperature
(c) Statement -1 is False, Statement-2 is True. (refrigerator) to higher temperature (room).
(d) Statement -1 is True, Statement-2 is False. Q.28 Statement-1 : In isothermal process whole of the heat
energy supplied to the body is converted into internal
energy.
Statement-2 : According to the first law of themodynamics
DQ = DU + PDV

RESPONSE GRID 26. 27. 28.

DAILY PRA CTICE PROBLEM SHEET 25 - PHYSICS


Total Questions 28 Total Marks 112
Attempted Correct
Incorrect N et Score
Cut-off Score 26 Qualifying Score 42
Success Gap = Net Score – Qualifying Score
Net Score = (Correct × 4) – (Incorrect × 1)

Space for Rough Work


t.me/Ebooks_Encyclopedia27. t.me/Magazines4all

DPP - Daily Practice Problems


Name : Date :

Start Time : End Time :

SYLLABUS : Kinetic Theory


26
Max. Marks : 120 Time : 60 min.
GENERAL INSTRUCTIONS
• The Daily Practice Problem Sheet contains 30 MCQ's. For each question only one option is correct. Darken the correct
circle/ bubble in the Response Grid provided on each page.
• You have to evaluate your Response Grids yourself with the help of solution booklet.
• Each correct answer will get you 4 marks and 1 mark shall be deduced for each incorrect answer. No mark will be given/
deducted if no bubble is filled. Keep a timer in front of you and stop immediately at the end of 60 min.
• The sheet follows a particular syllabus. Do not attempt the sheet before you have completed your preparation for that
syllabus. Refer syllabus sheet in the starting of the book for the syllabus of all the DPP sheets.
• After completing the sheet check your answers with the solution booklet and complete the Result Grid. Finally spend time
to analyse your performance and revise the areas which emerge out as weak in your evaluation.

DIRECTIONS (Q.1-Q.21) : There are 21 multiple choice questions. (c) Low pressure and high temperature
Each question has 4 choices (a), (b), (c) and (d), out of which ONLY (d) High pressure and low temperature
ONE choice is correct. Q.4 The pressure P, volume V and temperature T of a gas in
Q.1 If pressure of a gas contained in a closed vessel is increased the jar A and the other gas in the jar B at pressure 2 P ,
by 0.4% when heated by 1°C, the initial temperature must
V
be volume and temperature 2T, then the ratio of, the
(a) 250 K (b) 250°C (c) 2500 K (d) 25°C 4
Q.2 To double the volume of a given mass of an ideal gas at number of molecules in the jar A and B will be
27°C keeping the pressure constant, one must raise the (a) 1 : 1 (b) 1 : 2
temperature in degree centigrade to (c) 2 : 1 (d) 4 : 1
(a) 54 (b) 270 (c) 327 (d) 600 Q.5 A flask is filled with 13 gm of an ideal gas at 27°C and its
Q.3 Under which of the following conditions is the law temperature is raised to 52°C. The mass of the gas that has
PV = RT obeyed most closely by a real gas? to be released to maintain the temperature of the gas in
(a) High pressure and high temperature the flask at 52°C and the pressure remaining the same is
(b) Low pressure and low temperature (a) 2.5 g (b) 2.0 g (c) 1.5 g (d) 1.0 g

RESPONSE GRID 1. 2. 3. 4. 5.
Space for Rough Work
t.me/Ebooks_Encyclopedia27. t.me/Magazines4all

EBD_7156
2 DPP/ P 26
Q.6 The pressure and temperature of two different gases is Q.12 For a gas at a temperature T the root-mean-square velocity
P and T having the volume V for each. They are mixed vrms , the most probable speed vmp , and the average speed
keeping the same volume and temperature, the pressure of v au obey the relationship
the mixture will be
(a) vau > vrms > vmp (b) vrms > va u > vmp
(a) P / 2 (b) P (c) 2P (d) 4P
Q.7 The root mean square velocity of a gas molecule of mass (c) vmp > vau > vrms (d) vmp > vrms > va u
m at a given temperature is proportional to Q.13 One mole of ideal monoatomic gas ( g = 5 / 3) is mixed with
1 one mole of diatomic gas ( g = 7 / 5) . What is g for the
(a) mo (b) m (c) m (d)
m æ Cp ö
mixture? çè g = ÷
Q.8 Which of the following statements is true? Cv ø
(a) Absolute zero temperature is not zero energy (a) 3/2 (b) 23/15 (c) 35/23 (d) 4/3
temperature Q.14 The value of the gas constant (R) calculated from the
(b) Two different gases at the same temperature and perfect gas equation is 8.32 Joule/gm mol K , whereas its
pressure have equal root mean square velocities value calculated from the knowledge of Cp and CV of the
(c) The root mean square speed of the molecules of gas is 1.98 cal/gm mole K . From this data, the value of J
different ideal gases, maintained at the same is
temperature are the same (a) 4.16 J / cal (b) 4.18 J / cal
(d) Given sample of 1 cc of hydrogen and 1 cc of oxygen (c) 4.20 J / cal (d) 4.22 J / cal
both at NTP; oxygen sample has a large number of Q.15 Gas at a pressure P0 is contained in a vessel. If the masses
molecules
of all the molecules are halved and their speeds are
Q.9 At room temperature, the r.m.s. speed of the molecules of
doubled, the resulting pressure P will be equal to
certain diatomic gas is found to be 1930 m/s. The gas is
P0
(a) H 2 (b) F2 (c) O2 (d) Cl2 (a) 4P0 (b) 2P0 (c) P0 (d)
2
Q.10 Root mean square velocity of a particle is v at pressure P. Q.16 The relation between the gas pressure P and average kinetic
If pressure is increased two times, then the r.m.s. velocity energy per unit volume E is
becomes
2
(a) 2 v (b) 3 v (c) 0.5 v (d) v (a) P = 1 E (b) P = E (c) P = 3 E (d) P = E
Q.11 In the two vessels of same volume, atomic hydrogen and 2 2 3
helium at pressure 1 atm and 2 atm are filled. If temperature Q.17 Mean kinetic energy (or average energy) per gm molecule
of both the samples is same, then average speed of of a monoatomic gas is given by
3 1 1 3
hydrogen atoms < CH > will be related to that of helium (a) RT (b) KT (c) RT (d) KT
2 2 2 2
< CHe > as Q.18 At which of the following temperature would the molecules
of a gas have twice the average kinetic energy they have at
(a) < CH > = 2 < CHe > (b) < CH > = < CHe >
20°C?

(c) < CH > = 2 < CHe > (d) < CH > =


< CHe > (a) 40o C (b) 80 o C (c) 313o C (d) 586o C
2

6. 7. 8. 9. 10.
RESPONSE
11. 12. 13. 14. 15.
GRID
16. 17. 18.

Space for Rough Work


t.me/Ebooks_Encyclopedia27. t.me/Magazines4all

DPP/ P 26 3
Q.19 The kinetic energy of one gram molecule of a gas at normal
Q.23 Let v, vrms and v p respectively denote the mean speed,
temperature and pressure is (R = 8.31 J/mol - K)
root mean square speed and most probable speed of the
(a) 0.56 ´ 10 4 J (b) 1.3 ´ 10 2 J molecules in an ideal monoatomic gas at absolute
temperature T, the mass of a molecule is m. Then
(c) 2.7 ´ 10 2 J (d) 3.4 ´ 10 3 J (1) v p < v < v rms
Q.20 70 calories of heat are required to raise the temperature
3 2
o f (2) The average kinetic energy of a molecule is mv
2 moles of an ideal gas at constant pressure from 30°C to 4 p
35°C. The amount of heat required to raise the temperature (3) No molecule can have speed greater than 2vrms
of same gas through the same range (30o C to 35o C) at (4) No molecule can have speed less than v p / 2
Q.24 A gas in container A is in thermal equilibrium with another
constant volume (R = 2 cal/mol K) gas in container B, both contain equal masses of the two
(a) 30 cal (b) 50 cal (c) 70 cal (d) 90 cal gases in the respective containers. Which of the follow-
Q.21 A vessel contains a mixture of one mole of oxygen and ing can be true
two moles of nitrogen at 300 K . The ratio of the average (1) PA = PB , VA ¹ VB (2) PA VA = PB VB
rotational kinetic energy per O2 molecule to that per N2 PA PB
molecule is (3) PA ¹ PB , VA = VB (4) V = V
A B
(a) 1 : 1
(b) 1 : 2 DIRECTIONS (Q.25-Q.27) : Read the passage given below and
(c) 2 : 1 answer the questions that follows :
(d) Depends on the moments of inertia of the two A diathermic piston divides adiabatic cylinder of volume V0 into
molecules two equal parts as shown in the figure. Both parts contain ideal
DIRECTIONS (Q.22-Q.24) : In the following questions, more than monoatomic gases. The initial pressure and temperature of gas
one of the answers given are correct. Select the correct answers in left compartment are P0 and T0 while that in right compartment
and mark it according to the following codes: are 2P0 and 2T0. Initially the piston is kept fixed and the system
Codes : (a) 1, 2 and 3 are correct (b) 1 and 2 are correct is allowed to acquire a state of thermal equilibrium.
(c) 2 and 4 are correct (d) 1 and 3 are correct
Q.22 From the following statements, concerning ideal gas at any
given temperature T, select the correct one(s) P0 , T0 2 P0 , 2T0
(1) The coefficient of volume expansion at constant
pressure is same for all ideal gases
(2) In a gaseous mixture, the average translational kinetic Q.25 The pressure in left compartment after thermal equilibrium
energy of the molecules of each component is same is achieved is
(3) The mean free path of molecules increases with the 3
decrease in pressure (a) P0 (b) P0
2
(4) The average translational kinetic energy per molecule 4
of oxygen gas is 3KT (K being Boltzmann constant) (c) P0 (d) None of these
3

RESPONSE 19. 20. 21. 22. 23.


GRID 24. 25.

Space for Rough Work


t.me/Ebooks_Encyclopedia27. t.me/Magazines4all

EBD_7156
4 DPP/ P 26
Q.26 The heat that flown from right compartment to left (b) Statement-1 is True, Statement-2 is True; Statement-2 is
compartment before thermal equilibrium is achieved is NOT a correct explanation for Statement-1.
3 3 2 (c) Statement -1 is False, Statement-2 is True.
(a) P0V0 (b) P0V0 (c) P0V0 (d) P0V0 (d) Statement -1 is True, Statement-2 is False.
4 8 3
Q.27 If the pin which was keeping the piston fixed is removed Q.28 Statement-1 : Internal energy of an ideal gas does not
and the piston is allowed to slide slowly such that a state depend upon volume of the gas.
of mechanical equilibrium is achieved. The volume of left Statement-2 : Internal energy of an ideal gas depends on
compartment when piston is in equilibrium is temperature of gas.
Q.29 Statement-1 : Equal masses of helium and oxygen gases
3 V0 V0 2 are given equal quantities of heat. There will be a greater
(a) V0 (b) (c) (d) V0
4 4 2 3 rise in the temperature of helium compared to that of
DIRECTIONS (Qs. 28-Q.30) : Each of these questions contains two
oxygen.
statements: Statement-1 (Assertion) and Statement-2 (Reason). Each Statement-2 : The molecular weight of oxygen is more
of these questions has four alternative choices, only one of which is than the molecular weight of helium.
the correct answer. You have to select the correct choice. Q.30 Statement-1 : Maxwell speed distribution graph is
asymmetric about most probable speed.
(a) Statement-1 is True, Statement-2 is True; Statement-2 is a Statement-2 : rms speed of ideal gas, depends upon it’s
correct explanation for Statement-1. type (monoatomic, diatomic and polyatomic).

RESPONSE GRID 26. 27. 28. 29. 30.

DAILY PRA CTICE PROBLEM SHEET 26 - PHYSICS


Total Questions 30 Total Marks 120
Attempted Correct
Incorrect N et Score
Cut-off Score 30 Qualifying Score 48
Success Gap = Net Score – Qualifying Score
Net Score = (Correct × 4) – (Incorrect × 1)

Space for Rough Work


t.me/Ebooks_Encyclopedia27. t.me/Magazines4all

DPP - Daily Practice Problems


Name : Date :

Start Time : End Time :

SYLLABUS : Oscillations-1 (Periodic motion - period, Frequency, Displacement as a function of time. Periodic
27
functions, Simple harmonic motion and its equation, Energy in S.H.M. - kinetic and potential energies)
Max. Marks : 120 Time : 60 min.
GENERAL INSTRUCTIONS
• The Daily Practice Problem Sheet contains 30 MCQ's. For each question only one option is correct. Darken the correct
circle/ bubble in the Response Grid provided on each page.
• You have to evaluate your Response Grids yourself with the help of solution booklet.
• Each correct answer will get you 4 marks and 1 mark shall be deduced for each incorrect answer. No mark will be given/
deducted if no bubble is filled. Keep a timer in front of you and stop immediately at the end of 60 min.
• The sheet follows a particular syllabus. Do not attempt the sheet before you have completed your preparation for that
syllabus. Refer syllabus sheet in the starting of the book for the syllabus of all the DPP sheets.
• After completing the sheet check your answers with the solution booklet and complete the Result Grid. Finally spend time
to analyse your performance and revise the areas which emerge out as weak in your evaluation.

DIRECTIONS (Q.1-Q.22) : There are 22 multiple choice questions. 1 1 1 1


Each question has 4 choices (a), (b), (c) and (d), out of which ONLY (a) sec (b) sec (c) sec (d) sec
8 6 4 3
ONE choice is correct.
æ pö
Q.1 A simple h armonic motion is represented by Q.4 If x = a sin ç wt + ÷ and x ' = a cos wt , then what is the
è 6ø
F (t ) = 10sin(20t + 0.5) . The amplitude of the S.H.M. is phase difference between the two waves?
(a) a = 30 cm (b) a = 20 cm (a) p / 3 (b) p / 6
(c) a = 10 cm (d) a = 5 cm
(c) p / 2 (d) p
Q.2 A particle executes a simple harmonic motion of time
period T. Find the time taken by the particle to go directly Q.5 A body is executing S.H.M. when its displacement from
from its mean position to half the amplitude the mean position is 4 cm and 5 cm, the corresponding
velocity of the body is 10 cm/sec and 8 cm/sec. Then the
(a) T / 2 (b) T / 4 (c) T / 8 (d) T /12
Q.3 The periodic time of a body executing simple harmonic time period of the body is
motion is 3 sec. After how much time from time t = 0 , its (a) 2p sec (b) p / 2 sec
displacement will be half of its amplitude (c) p sec (d) 3p / 2sec

RESPONSE GRID 1. 2. 3. 4. 5.
Space for Rough Work
t.me/Ebooks_Encyclopedia27. t.me/Magazines4all

EBD_7156
2 DPP/ P 27
Q.6 If a simple pendulum oscillates with an amplitude of 50 Q.12 The total energy of a particle executing S.H.M. is
mm and time period of 2 sec, then its maximum velocity proportional to
is (a) Displacement from equilibrium position
(a) 0.10 m/s (b) 0.15 m/s (b) Frequency of oscillation
(c) 0.8 m/s (d) 0.26 m/s (c) Velocity in equilibrium position
Q.7 The maximum velocity and the maximum acceleration of (d) Square of amplitude of motion
a body moving in a simple harmonic oscillator are 2 m/s Q.13 When the displacement is half the amplitude, the ratio of
and 4 m/s2. Then angular velocity will be potential energy to the total energy is
(a) 3 rad/sec (b) 0.5 rad/sec 1 1 1
(a) (b) (c) 1 (d)
(c) 1 rad/sec (d) 2 rad/sec 2 4 8
Q.8 The amplitude of a particle executing SHM is 4 cm. At the Q.14 A particle is executing simple harmonic motion with
mean position the speed of the particle is 16 cm/sec. The frequency f . The frequency at which its kinetic energy
distance of the particle from the mean position at which changes into potential energy is
the speed of the particle becomes 8 3 cm/s, will be (a) f / 2 (b) f (c) 2 f (d) 4 f
(a) 2 3 cm (b) 3 cm
Q.15 A particle executes simple harmonic motion with a
(c) 1cm (d) 2 cm frequency f . The frequency with which its kinetic energy
Q.9 The amplitude of a particle executing S.H.M. with oscillates is
frequency of 60 Hz is 0.01 m. The maximum value of the (a) f / 2 (b) f (c) 2 f (d) 4 f
acceleration of the particle is Q.16 The kinetic energy of a particle executing S.H.M. is 16 J
(a) 144p 2 m/sec 2 (b) 144 m/sec 2 when it is in its mean position. If the amplitude of
oscillations is 25 cm and the mass of the particle is 5.12
144 kg, the time period of its oscillation is
(c) m/sec 2 (d) 288p 2 m/sec 2
p2 p
Q.10 A particle executes simple harmonic motion with an (a) sec (b) 2p sec (c) 20p sec (d) 5p sec
5
angular velocity and maximum acceleration of 3.5rad/sec Q.17 The displacement x (in metres) of a particle performing
and 7.5 m /s 2 respectively. The amplitude of oscillation simple harmonic motion is related to time t (in seconds) as
is æ pö
x = 0.05cos ç 4pt + ÷ . The frequency of the motion will
(a) 0.28 m (b) 0.36 m (c) 0.53 m (d) 0.61 m è 4ø
Q.11 What is the maximum acceleration of the particle doing be
é pt ù (a) 0.5 Hz (b) 1.0 Hz (c) 1.5 Hz (d) 2.0 Hz
the SHM y = 2sin ê + fú where y is in cm? Q.18 A particle executes simple harmonic motion
ë 2 û
[amplitude = A ] between x = - A and x = + A . The time
p p2
(a) cm/s 2 (b) cm/s 2 taken for it to go from 0 to A / 2 is T1 and to go from
2 2
p p A / 2 to A is T2 . Then
(c) cm/s 2 (d) cm/s 2
4 4 (a) T1 < T2 (b) T1 > T2 (c) T1 = T2 (d) T1 = 2T2

6. 7. 8. 9. 10.
RESPONSE
11. 12. 13. 14. 15.
GRID
16. 17. 18.

Space for Rough Work


t.me/Ebooks_Encyclopedia27. t.me/Magazines4all

DPP/ P 27 3

Q.19 A cylindrical piston of mass M slides smoothly inside a DIRECTIONS (Q.23-Q.25) : In the following questions, more than
long cylinder closed at one end, enclosing a certain mass one of the answers given are correct. Select the correct
of gas. The cylinder is kept with its axis horizontal. If the answers and mark it according to the following codes:
piston is disturbed from its equilibrium position, it
Codes : (a) 1, 2 and 3 are correct
oscillates simple harmonically. The period of oscillation
(b) 1 and 2 are correct
will be
(c) 2 and 4 are correct
h
(d) 1 and 3 are correct
Q.23 A particle constrained to move along the x-axis in a
M potential V = kx2, is subjected to an external time dependent
r
Gas
force f (t) , here k is a constant, x the distance from the
P A
origin, and t the time. At some time T, when the particle
has zero velocity at x = 0, the external force is removed.
(a) T = 2p æç Mh ö÷ (b) T = 2p æç MA ö÷ Choose the incorrect options –
è PA ø è Ph ø (1) Particle executes SHM
(2) Particle moves along +x direction
(c) T = 2p æç
M ö (3) Particle moves along – x direction
÷ (d) T = 2p MPhA
è PAh ø (4) Particle remains at rest
Q.24 Three simple harmonic motions in the same direction
Q.20 A particle is performing simple harmonic motion along having the same amplitude a and same period are
x-axis with amplitude 4 cm and time period 1.2 sec. The superposed. If each differs in phase from the next by 45°,
minimum time taken by the particle to move from x = 2 then –
cm to x = + 4 cm and back again is given by
(a) 0.6 sec (b) 0.4 sec (1) The resultant amplitude is (1 + 2) a
(c) 0.3 sec (d) 0.2 sec (2) The phase of the resultant motion relative to the first is
Q.21 A spring of force constant k is cut into two pieces such 90°
that one piece is double the length of the other. Then the (3) The energy associated with the resulting motion is
long piece will have a force constant of
(3 + 2 2) times the energy associated with any single
(a) (2/3)k (b) (3/2)k
motion
(c) 3k (d) 6k
(4) The resulting motion is not simple harmonic
Q.22 A simple pendulum has time period T 1 . The point of
Q.25 For a particle executing simple harmonic motion, which
suspension is now moved upward according to equation
of the following statements is correct?
y = kt2 where k =1m/sec2. If new time period is T2 then
(1) The total energy of the particle always remains the same
T12 (2) The restoring force always directed towards a fixed
ratio will be point
T22
(3) The restoring force is maximum at the extreme
(a) 2/3 (b) 5/6 positions
(c) 6/5 (d) 3/2 (4) The acceleration of the particle is maximum at the
equilibrium position

RESPONSE 19. 20. 21. 22. 23.


GRID 24. 25.

Space for Rough Work


t.me/Ebooks_Encyclopedia27. t.me/Magazines4all

EBD_7156
4 DPP/ P 27
DIRECTIONS (Q.26-Q.27) : Read the passage given below and DIRECTIONS (Q.28-Q.30) : Each of these questions contains two
answer the questions that follows : statements: Statement-1 (Assertion) and Statement-2 (Reason). Each
of these questions has four alternative choices, only one of which is
The differential equation of a particle undergoing SHM is given the correct answer. You have to select the correct choice.
d2 x (a) Statement-1 is True, Statement-2 is True; Statement-2 is a
by a
2
+ bx = 0 . The particle starts from the extreme position. correct explanation for Statement-1.
dt
Q.26 The ratio of the maximum acceleration to the maximum (b) Statement-1 is True, Statement-2 is True; Statement-2 is
velocity of the particle is – NOT a correct explanation for Statement-1.
(c) Statement -1 is False, Statement-2 is True.
b a (d) Statement -1 is True, Statement-2 is False.
(a) (b)
a b Q.28 Statement-1 : In S.H.M., the motion is ‘to and fro’ and
periodic.
a b Statement-2 : Velocity of the particle
(c) (d)
b a
Q.27 The equation of motion may be given by : ) w k 2 - x 2 (where x is the displacement and k is
(v=
amplitude)
æ bö
Q.29 Statement-1 : In simple harmonic motion, the velocity is
(a) x = Asin ç a ÷ t
è ø maximum when acceleration is minimum.
Statement-2 : Displacement and velocity of S.H.M. differ
æ bö
in phase by p / 2
(b) x = Acos ç a ÷ t
è ø Q.30 Statement-1 : The graph of total energy of a particle in
SHM w.r.t., position is a straight line with zero slope.
æ b ö
Statement-2 : Total energy of particle in SHM remains
(c) x = A sin ç a t + q÷ where q ¹ p/2
è ø constant throughout its motion.
(d) None of these

RESPONSE GRID 26. 27. 28. 29. 30.

DAILY PRA CTICE PROBLEM SHEET 27 - PHYSICS


Total Questions 30 Total Marks 120
Attempted Correct
Incorrect N et Score
Cut-off Score 30 Qualifying Score 45
Success Gap = Net Score – Qualifying Score
Net Score = (Correct × 4) – (Incorrect × 1)

Space for Rough Work


t.me/Ebooks_Encyclopedia27. t.me/Magazines4all

DPP - Daily Practice Problems


Name : Date :

Start Time : End Time :

SYLLABUS : Oscillations-2 (Oscillations of a spring, simple pendulum, free,


28
forced and damped oscillations, Resonance)
Max. Marks : 112 Time : 60 min.
GENERAL INSTRUCTIONS
• The Daily Practice Problem Sheet contains 28 MCQ's. For each question only one option is correct. Darken the correct
circle/ bubble in the Response Grid provided on each page.
• You have to evaluate your Response Grids yourself with the help of solution booklet.
• Each correct answer will get you 4 marks and 1 mark shall be deduced for each incorrect answer. No mark will be given/
deducted if no bubble is filled. Keep a timer in front of you and stop immediately at the end of 60 min.
• The sheet follows a particular syllabus. Do not attempt the sheet before you have completed your preparation for that
syllabus. Refer syllabus sheet in the starting of the book for the syllabus of all the DPP sheets.
• After completing the sheet check your answers with the solution booklet and complete the Result Grid. Finally spend time
to analyse your performance and revise the areas which emerge out as weak in your evaluation.

DIRECTIONS (Q.1-Q.20) : There are 20 multiple choice


2m m m m
questions. Each question has 4 choices (a), (b), (c) and (d), out (a) 2p (b) 2p (c) 2p (d) 2p
k 2k k 3k
of which ONLY ONE choice is correct.
Q.2 Three masses 700g, 500g, and 400g are suspended at the
Q.1 A particle of mass m is end of a spring as shown and are in equilibrium.
C B
attached to three identical When the 700g mass is removed, the system oscillates
springs A, B and C each of 90° with a period of 3 seconds, when the 500 gm mass is also
force constant k as shown removed, it will oscillate with a period of
in figure. m
(a) 1 s
O
If the particle of mass m (b) 2 s
is pushed slightly against (c) 3 s
700gm
the spring A and released
12 500gm
then the time period of (d) s
A 5 400gm
oscillations is

RESPONSE GRID 1. 2.
Space for Rough Work
t.me/Ebooks_Encyclopedia27. t.me/Magazines4all

EBD_7156
2 DPP/ P 28
Q.3 The bob of a simple pendulum is displaced from its Q.7 The time period of a second’s pendulum is 2 sec. The
equilibrium position O to a position Q which is at height h spherical bob which is empty from inside has a mass of 50
above O and the bob is then released. gm. This is now replaced by another solid bob of same
Assuming the mass of the bob to be m and time period of radius but having different mass of 100 gm. The new time
oscillations to be 2.0 sec, the tension in the string when period will be
the bob passes through O is (a) 4 sec (b) 1 sec (c) 2 sec (d) 8 sec
(a) m( g + p 2 g h) Q.8 The length of a simple pendulum is increased by 1%. Its
time period will
(b) m ( g + p2 g h ) (a) Increase by 1% (b) Increase by 0.5%
æ ö
(c) Decrease by 0.5% (d) Increase by 2%
p2
(c) mç g + g h÷ Q Q.9 The bob of a pendulum of length l is pulled aside from its
ç 2 ÷
è ø equilibrium position through an angle q and then released.
h
æ 2 ö The bob will then pass through its equilibrium position with
(d) m ç g + p g h ÷ O a speed v, where v equals
ç 3 ÷
è ø
(a) 2gl (1- sin q) (b) 2gl (1 + cos q)
Q.4 A spring of force constant k is cut into two pieces such
that one piece is double the length of the other. Then the (c) 2gl (1 - cos q) (d) 2gl (1+ sin q)
long piece will have a force constant of Q.10 A simple pendulum is executing simple harmonic motion
(a) (2 / 3)k (b) (3 / 2)k (c) 3k (d) 6k with a time period T. If the length of the pendulum is
Q.5 A pendulum suspended from the ceiling of a train has a increased by 21%, the percentage increase in the time
period T, when the train is at rest. When the train is period of the pendulum of is
accelerating with a uniform acceleration a, the period of (a) 10% (b) 21% (c) 30% (d) 50%
oscillation will Q.11 A chimpanzee swinging on a swing in a sitting position,
(a) increase (b) decrease stands up suddenly, the time period will
(c) remain unaffected (d) become infinite (a) Become infinite (b) Remain same
Q.6 A simple pendulum is set up in a trolley which moves to the (c) Increase (d) Decrease
right with an acceleration a on a horizontal plane. Then the Q.12 A simple pendulum consisting of a ball of mass m tied to a
thread of the pendulum in the mean position makes an angle thread of length l is made to swing on a circular arc of angle q
q with the vertical is in a vertical plane. At the end of this arc, another ball of mass
a m is placed at rest. The momentum transferred to this ball at
(a) tan -1 in the forward direction rest by the swinging ball is
g
g mq
a
(b) tan -1 in the backward direction (a) Zero (b) mq (c) l
(d) m 2p l
l l g l g
g
Q.13 The time period of a simple pendulum of length L as
g
(c) tan -1 in the backward direction measured in an elevator descending with acceleration g / 3
a is
-1 g
(d) tan in the forward direction æ 3L ö æ 3L ö
a (a) 2p 3L (b) p ç ÷ (c) 2p ç ÷ (d) 2p 2L
g è gø è 2g ø 3g

3. 4. 5. 6. 7.
RESPONSE
8. 9. 10. 11. 12.
GRID
13.

Space for Rough Work


t.me/Ebooks_Encyclopedia27. t.me/Magazines4all

DPP/ P 28 3
Q.14 A mass m is suspended from the two coupled springs is maximum for w = w2, then (where w0 natural frequency
connected in series. The force constant for springs are k1 of oscillation of particle)
and k2. The time period of the suspended mass will be (a) w1 = w0 and w 2 ¹ w 0 (b) w1 = w0 and w 2 = w 0
(a) æ m ö
(b) æ m ö (c) w1 ¹ w0 and w 2 = w 0 (d) w1 ¹ w0 and w 2 ¹ w 0
T = 2p ç ÷ T = 2p ç ÷
è k1 + k 2 ø è k1 + k 2 ø c
Q.20 Amplitude of a wave is represented by A =
æ mk k ö a +b-c
æ m(k1 + k2 ) ö T = 2p ç 1 2 ÷ Then resonance will occur when
(c) T = 2p ç ÷ (d)
è k1k2 ø è k1 + k 2 ø
(a) b = – c/2 (b) b = 0 & a = c
Q.15 A spring having a spring constant k is loaded with a mass
(c) b = – a/2 (d) None
m. The spring is cut into two equal parts and one of these is
loaded again with the same mass. The new spring constant is DIRECTIONS (Q.21-Q.23) : In the following questions,
(a) k / 2 (b) k (c) 2k (d) k2 more than one of the answers given are correct. Select the
Q.16 A mass m = 100 gm is attached at the end of a light spring correct answers and mark it according to the following
which oscillates on a frictionless horizontal table with an codes:
amplitude equal to 0.16 metre and time period equal to 2
sec. Initially the mass is released from rest at t = 0 and Codes :
displacement x = –0.16 metre. The expression for the (a) 1, 2 and 3 are correct (b) 1 and 2 are correct
displacement of mass at any time t is (c) 2 and 4 are correct (d) 1 and 3 are correct
(a) x = 0.16 cos(pt ) (b) x = - 0.16 cos(pt ) Q.21 Two blocks A and B each of mass m are connected by a
massless spring of natural length L and spring constant k.
(c) x = 0.16sin(pt + p) (d) x = - 0.16sin(pt + p ) The blocks are initially resting on a smooth horizontal floor
Q.17 Two masses m1 and m2 are suspended together by a massless with the spring at its natural length. A third identical block
spring of constant k. When the masses are in equilibrium, C also of mass m moves on the floor with a speed v along
m1 is removed without disturbing the system. The amplitude the line joining A and B and collides with A. Then
of oscillations is (1) The kinetic energy of the A – B system at maximum
m1g compression of the spring is mv2/4
(a)
k (2) The maximum compression of the spring is v m / 2k
m2 g
(b) (3) The kinetic energy of the A-B system at maximum
k
compression of the spring is zero
(m1 + m2 ) g
(c) k m1
(4) The maximum compression of the spring is v m / k
(m1 - m2 ) g m2 Q.22 A simple pendulum of length L and mass (bob) M is
(d) k oscillating in a plane about a vertical line between angular
Q.18 The composition of two simple harmonic motions of equal limits – f and + f. For an angular displacement θ ( θ < f) ,
periods at right angle to each other and with a phase the tension in the string and the velocity of the bob are T
difference of p results in the displacement of the particle and v respectively. The following relations hold good under
along the above conditions
(a) Straight line (b) Circle Mv 2
(1) T - Mg cos θ =
(c) Ellipse (d) Figure of 8 L
Q.19 A particle with restoring force proportional to (2) T cosθ = Mg
displacement and resisting force proportional to velocity (3) The magnitude of the tangential acceleration of the bob
is subjected to a force F sin wt. If the amplitude of the a T = g sin θ
particle is maximum for w = w1 and the energy of the particle (4) T = Mg cos q

RESPONSE 14. 15. 16. 17. 18.


GRID 19. 20. 21. 22.

Space for Rough Work


t.me/Ebooks_Encyclopedia27. t.me/Magazines4all

EBD_7156
4 DPP/ P 28
Q.23 Identify wrong statements among the following DIRECTIONS (Q.26-Q.28) : Each of these questions contains
(1) The greater the mass of a pendulum bob, the shorter two statements: Statement-1 (Assertion) and Statement-2
is its frequency of oscillation (Reason). Each of these questions has four alternative choices,
(2) A simple pendulum with a bob of mass M swings with
only one of which is the correct answer. You have to select the
an angular amplitude of 40°. When its angular
correct choice.
amplitude is 20°, the tension in the string is less than
Mgcos20°. (a) Statement-1 is True, Statement-2 is True; Statement-2 is a
(3) The fractional change in the time period of a pendulum correct explanation for Statement-1.
on changing the temperature is independent of the (b) Statement-1 is True, Statement-2 is True; Statement-2 is
length of the pendulum. NOT a correct explanation for Statement-1.
(4) As the length of a simple pendulum is increased, the (c) Statement -1 is False, Statement-2 is True.
maximum velocity of its bob during its oscillation will (d) Statement -1 is True, Statement-2 is False.
also decreases.
Q.26 Statement-1 : Consider motion for a mass spring system
DIRECTIONS (Q.24-Q.25) : Read the passage given below under gravity, motion of M is not a simple harmonic motion
and answer the questions that follows : unless Mg is negligibly small.
A particle performs linear SHM such that it is placed on plat- Statement-2 : For simple harmonic k = force constant

Ma = kx + Mg
form & platform along with particles oscillate vertically up and motion acceleration must be X

down with amplitude A = 1cm. If the particle does not loose proportional to displacement and is
contact with platform anywhere and mass of particle is 1 kg, directed towards the mean position. M = Mass

find : Q.27 Statement-1 : The periodic time of a hard spring is less as


Q.24 The minimum, possible time period (Take p = g ) compared to that of a soft spring.
(a) 0.1 sec. (b) 0.2 sec. Statement-2 : The periodic time depends upon the spring
(c) 0.3 sec. (d) 0.4 sec. constant, and spring constant is large for hard spring.
Q.25 For minimum time period condition average potential Q.28 Statement-1 : The percentage change in time period is
energy between t = 0 to t = 0.05 sec (Take g = 10 m/s2) 1.5%, if the length of simple pendulum increases by 3%
(a) 0.025 Joule (b) 0.1 Joule Statement-2:Time period is directly proportional to length of
(c) 0.08 Joule (d) 0.06 Joule pendulum.

RESPONSE 23. 24. 25. 26. 27.


GRID 28.

DAILY PRA CTICE PROBLEM SHEET 28 - PHYSICS


Total Questions 28 Total Marks 112
Attempted Correct
Incorrect N et Score
Cut-off Score 28 Qualifying Score 42
Success Gap = Net Score – Qualifying Score
Net Score = (Correct × 4) – (Incorrect × 1)
Space for Rough Work
t.me/Ebooks_Encyclopedia27. t.me/Magazines4all

DPP - Daily Practice Problems


Name : Date :

Start Time : End Time :

SYLLABUS : Waves-1 (Wave motion, longitudinal and transverse waves, speed of a wave, displacement relation for a
29
progressive wave, principle of superposition of waves, reflection of waves)

Max. Marks : 116 Time : 60 min.


GENERAL INSTRUCTIONS
• The Daily Practice Problem Sheet contains 29 MCQ's. For each question only one option is correct. Darken the correct
circle/ bubble in the Response Grid provided on each page.
• You have to evaluate your Response Grids yourself with the help of solution booklet.
• Each correct answer will get you 4 marks and 1 mark shall be deduced for each incorrect answer. No mark will be given/
deducted if no bubble is filled. Keep a timer in front of you and stop immediately at the end of 60 min.
• The sheet follows a particular syllabus. Do not attempt the sheet before you have completed your preparation for that
syllabus. Refer syllabus sheet in the starting of the book for the syllabus of all the DPP sheets.
• After completing the sheet check your answers with the solution booklet and complete the Result Grid. Finally spend time
to analyse your performance and revise the areas which emerge out as weak in your evaluation.

DIRECTIONS (Q.1-Q.20) : There are 20 multiple choice 3.5s respectively, the distance between the cliffs is
questions. Each question has 4 choices (a), (b), (c) and (d), out (Velocity of sound in air = 340 ms–1)
of which ONLY ONE choice is correct. (a) 1190 m
(b) 850 m
Q.1 A tuning fork makes 256 vibrations per second in air. When (c) 595 m
the velocity of sound is 330 m/s then wavelength of the tone (d) 510 m
emitted is Q.4 v1 and v2 are the velocities of sound at the same temperature
(a) 0.56 m (b) 0.89 m (c) 1.11 m (d) 1.29 m in two monoatomic gases of densities r 1 and r 2
Q.2 In a sinusoidal wave, the time required for a particular point
1
to move from maximum displacement to zero respectively. If r1 / r 2 = then the ratio of velocities v 1
4
displacement is 0.170 second. The frequency of the wave
and v2 will be
is
(a) 1 : 2 (b) 4 : 1 (c) 2 : 1 (d) 1 : 4
(a) 1.47Hz (b) 0.36 Hz (c) 0.73 Hz (d) 2.94 Hz
Q.5 A wave of frequency 500 Hz has velocity 360 m/sec. The
Q.3 A man is standing between two parallel cliffs and fires a
distance between two nearest points 60° out of phase, is
gun. If he hears first and second echoes after 1.5 s and
(a) 0.6 cm (b) 12 cm (c) 60 cm (d) 120 cm

RESPONSE GRID 1. 2. 3. 4. 5.
Space for Rough Work
t.me/Ebooks_Encyclopedia27. t.me/Magazines4all

EBD_7156
2 DPP/ P 29
Q.6 Two waves are given by y1 = a sin(wt - kx) and y2 = a cos Q.12 The equation of a plane progressive wave is given by
(wt – kx). The phase difference between the two waves is y = 0.025sin (100t + 0.25 x ) . The frequency of this wave
p p p would be
(a) (b) p (c) (d)
4 8 2 50 100
Q.7 The relation between time and displacement for two (a) Hz (b) Hz (c) 100Hz (d) 50Hz
p p
particles is given by
Q.13 A wave travelling in positive X-direction with A = 0.2m
y = 0.06sin 2 p(0.04t + f1 ), y2 = 0.03sin 2 p(1.04t + f 2 ) has a velocity of 360 m/sec. If l = 60m, then correct
The ratio of the intensities of the waves produced by the expression for the wave is
vibrations of the two particles will be
é æ x öù é æ x öù
(a) 2 : 1 (b) 1 : 2 (c) 4 : 1 (d) 1 : 4 (a) y = 0.2sin ê 2p ç 6t + ÷ ú (b) y = 0.2 sin ê p ç 6t + ÷ ú
Q.8 A tr ansverse wave is descr ibed by the equation ë è 60 ø û ë è 60 ø û

æ xö é æ x öù é æ x öù
Y = Y0 sin2p ç ft - ÷ . The maximum particle velocity is four y = 0.2sin ê 2p ç 6t - ÷ ú (d) y = 0.2 sin ê p ç 6t - ÷ ú
è l ø (c)
ë è 60 ø û ë è 60 ø û
times the wave velocity if
Q.14 The equation of a wave is given as
pY pY
(a) l = 0 (b) l = 0 y= 0.07 sin (12 px - 3000pt ). where x is in metre and t in
4 2
(c) l = pY0 (d) l = 2pY0 sec , then the correct statement is
Q.9 Which one of the following does not represent a travelling (a) l = 1/ 6m, v = 250 m/s (b) a = 0.07m, v = 300m/s
wave? (c) n = 1500, v = 200 m/s (d) None
(a) y = sin( x - vt ) (b) y = ym sin k ( x + vt ) Q.15 The equation of a progressive wave is given by
(c) y = ym log( x - vt ) (d) y = f ( x2 - vt 2 ) y = 0.5sin ( 20 x - 400t ) where x and y are in metre and
Q.10 The path difference between the two waves
t is in second. The velocity of the wave is
æ 2px ö æ 2px ö
y1 = a1 sin ç wt - ÷ and y2 = a2 cos ç wt - + f ÷ is (a) 10 m/s (b) 20 m/s (c) 200 m/s (d) 400 m/s
è l ø è l ø
Q.16 There is a destructive interference between the two waves
l l æ pö
(a) f (b) çf+ ÷ of wavelength l coming from two different paths at a
2p 2p è 2ø
point. To get maximum sound or constructive interference
2p æ pö 2p at that point, the path of one wave is to be increased by
(c) çf - ÷ (d) f
l è 2ø l l l
Q.11 A transverse wave is represented by the equation (a) (b)
4 2
2p 3l
y = y0 sin ( vt - x ) , Here v = wave velocity (c) (d) l
l
4
For what value of l , the maximum particle velocity equal Q.17 If two waves of same frequency and same amplitude on
to two times the wave velocity superimposition produced a resultant disturbance of the
(a) l = 2 py0 (b) l = py0 / 3 same amplitude, the waves differ in phase by
(c) l = py0 / 2 (d) l = py0 (a) p (b) 2p /3 (c) p / 2 (d) zero

6. 7. 8. 9. 10.
RESPONSE
11. 12. 13. 14. 15.
GRID
16. 17.

Space for Rough Work


t.me/Ebooks_Encyclopedia27. t.me/Magazines4all

DPP/ P 29 3
Q.18 Equation of motion in the same direction is given by (1) A wave travelling in the negative X direction with a
y1 = A sin( wt - kx ), y 2 = A sin(wt - kx - q) .The amplitude velocity of 1.5 m/sec
of the medium particle will be (2) A wave travelling in the negative X direction with a
wavelength of 0.2 m
q
(a) 2 A cos (b) 2 A cos q (3) A wave travelling in the positive X direction with a
2
velocity of 1.5 m/sec.
q
(c) 2 A cos (d) 2A cos q (4) A wave travelling in the positive X direction with a
2
Q.19 The amplitude of a wave, represented by displacement wavelength of 0.2 m
Q.23 It is usually more convenient to describe a sound wave in
1 1
equation y = sin wt ± cos wt will be terms of pressure wave as compared to displacement wave
a b because –
a+b a+ b a± b a+b (1) Two waves of same intensity but different frequencies
(a) (b) (c) (d) have different displacement amplitude but same
ab ab ab ab
Q.20 The displacement due to a wave moving in the positive pressure amplitude
1 (2) The human ear responds to the change in pressure and
x -direction is given by y = at time t = 0 and by
(1 + x2 ) not to the displacement wave.
1 (3) The electronic detector (microphone) does respond
y= at t = 2 seconds, where x and y are in to the change in pressure but not to the displacement.
[1 + ( x - 1)2 ]
metres. The velocity of the wave in m/s is (4) None of the above
(a) 0.5 (b) 1 (c) 2 (d) 4
DIRECTIONS (Q.24-Q.26) : Read the passage given below
DIRECTIONS (Q.21-Q.23) : In the following questions, and answer the questions that follows :
more than one of the answers given are correct. Select the
Sound from a point isotropic source spreads equally in all
correct answers and mark it according to the following
directions in homogeneous medium. Therefore its intensity
codes:
decreases with square of distance from the source. When
Codes : (a) 1, 2 and 3 are correct distance between observer and the source changes, apart from
(b) 1 and 2 are correct changes in intensity, the observer listens sound of pitch higher
(c) 2 and 4 are correct or lower than actual pitch depending upon the fact that the
(d) 1 and 3 are correct distance between the observer and source is decreasing or
Q.21 P, Q and R are three particles of a medium which lie on the increasing respectively. An observer O is at a distance 2R from
x-axis. A sine wave of wavelength l is travelling through centre of a circle of radius R. A point isotropic sound source S
the medium in the x-direction. P and Q always have the moves on the circle with uniform angular velocity w = p/3 rad/
same speed, while P and R always have the same velocity. s. Initially observer, source and centre of the circle are in same
The minimum distance between – line.
(1) P and Q is l (2) P and Q is l/2 2R
(3) P and R is l/2 (4) P and R is l R=1m
Q.22 A wave represented by the given equation
æ πö O S C
Y = A sin ç10πx + 15πt + ÷ , where x is in meter and t is
è 3ø 1R R
in second. The expression represents

RESPONSE 18. 19. 20. 21. 22.


GRID 23.

Space for Rough Work


t.me/Ebooks_Encyclopedia27. t.me/Magazines4all

EBD_7156
4 DPP/ P 29
Q.24 Starting from initial moment, the source moves through DIRECTIONS (Q. 27-Q.29) : Each of these questions contains
an angular displacement 180°. Intensity of the sound as two statements: Statement-1 (Assertion) and Statement-2
observed by the observer decreases by a factor of – (Reason). Each of these questions has four alternative choices,
(a) 2 (b) 3 only one of which is the correct answer. You have to select the
(c) 4 (d) 9
correct choice.
Q.25 During a complete round trip of star on the circle, the
observer listens a sound, whose – (a) Statement-1 is True, Statement-2 is True; Statement-2 is a
(a) wavelength first decreases to a maximum value then correct explanation for Statement-1.
increases to the original value (b) Statement-1 is True, Statement-2 is True; Statement-2 is
(b) wavelength first increases to a maximum value then NOT a correct explanation for Statement-1.
decreases to the original value (c) Statement -1 is False, Statement-2 is True.
(c) During the first half time wavelength increases then (d) Statement -1 is True, Statement-2 is False.
decreases to the original value Q.27 Statement-1 : Particle velocity and wave velocity both are
(d) None of the above is correct because in Doppler's independent of time.
effect, it is the pitch of sound which changes and not Statement-2 : For the propagation of wave motion, the
its wavelength, irrespective of motion of source or medium must have the properties of elasticity and inertia.
observer.
Wavelength
Q.26 Sound emitted by the source at two successive instants t1 Q.28 Statement-1 : Speed of wave =
Time period
and t 2 has minimum and maximum observed pitch
respectively, then – Statement-2 : Wavelength is the distance between two
(a) t1 = 1s, t2 = 5s nearest particles vibrating in phase.
(b) t1 = 5s, t2 = 7s Q.29 Statement-1 : Transverse waves are not produced in liquids
(c) t1 = 7s, t2 = 11s and gases.
(d) t1 = 5s, t2 = 11s Statement-2 : Light waves are transverse waves.

RESPONSE 24. 25. 26. 27. 28.


GRID 29.

DAILY PRA CTICE PROBLEM SHEET 29 - PHYSICS


Total Questions 29 Total Marks 116
Attempted Correct
Incorrect N et Score
Cut-off Score 28 Qualifying Score 44
Success Gap = Net Score – Qualifying Score
Net Score = (Correct × 4) – (Incorrect × 1)
Space for Rough Work
t.me/Ebooks_Encyclopedia27. t.me/Magazines4all

DPP - Daily Practice Problems


Name : Date :

Start Time : End Time :

SYLLABUS : Waves-2 (Standing waves in strings and organ pipes, Fundamental mode and
30
harmonics, Beats, Doppler effect in sound)
Max. Marks : 120 Time : 60 min.
GENERAL INSTRUCTIONS
• The Daily Practice Problem Sheet contains 30 MCQ's. For each question only one option is correct. Darken the correct
circle/ bubble in the Response Grid provided on each page.
• You have to evaluate your Response Grids yourself with the help of solution booklet.
• Each correct answer will get you 4 marks and 1 mark shall be deduced for each incorrect answer. No mark will be given/
deducted if no bubble is filled. Keep a timer in front of you and stop immediately at the end of 60 min.
• The sheet follows a particular syllabus. Do not attempt the sheet before you have completed your preparation for that
syllabus. Refer syllabus sheet in the starting of the book for the syllabus of all the DPP sheets.
• After completing the sheet check your answers with the solution booklet and complete the Result Grid. Finally spend time
to analyse your performance and revise the areas which emerge out as weak in your evaluation.

DIRECTIONS (Q.1-Q.22) : There are 22 multiple choice Q.2 A closed organ pipe of length L and an open organ pipe
questions. Each question has 4 choices (a), (b), (c) and (d), out contain gases of densities r1 and r2 respectively. The
of which ONLY ONE choice is correct. compressibility of gases are equal in both the pipes. Both
Q.1 A police car moving at 22 m/s, chases a motorcylist. The the pipes are vibrating in their first overtone with same
police man sounds his horn at 176 Hz, while both of them frequency. The length of the open organ pipe is
move towards a stationary siren of frequency 165 Hz. 4 L r2
L 4L 4 L r1
Calculate the speed of the motorcycle, if it is given that he (a) (b) (c) (d)
3 3 3 r2 3 r1
does not observes any beats
Police Car Motorcycle Q.3 Two whistles A and B produces notes of frequencies 660
Hz and 596 Hz respectively. There is a listener at the mid-
point of the line joining them. Now the whistle B and the
listener start moving with speed 30 m/s away from the
22 m/s Stationary siren whistle A. If speed of sound be 330 m/s, how many beats
(176 Hz) (165 Hz) will be heard by the listener
(a) 33 m/s (b) 22 m/s (c) Zero (d) 11 m/s (a) 2 (b) 4 (c) 6 (d) 8

RESPONSE GRID 1. 2. 3.
Space for Rough Work
t.me/Ebooks_Encyclopedia27. t.me/Magazines4all

EBD_7156
2 DPP/ P 30
Q.4 An open organ pipe is in resonance in its 2nd harmonic Q.10 An observer is moving away from source of sound of
with tuning fork of frequency f1. Now, it is closed at one frequency 100 Hz. His speed is 33 m/s. If speed of sound
end. If the frequency of the tuning fork is increased slowly is 330 m/s, then the observed frequency is
from f1 then again a resonance is obtained with a frequency (a) 90 Hz (b) 100 Hz (c) 91 Hz (d) 110 Hz
f2. If in this case the pipe vibrates nth hamonics then Q.11 A whistle giving out 450 Hz approaches a stationary
3 5 observer at a speed of 33 m/s. The frequency heard by the
(a) n = 3, f 2 = f1 (b) n = 3, f 2 = f1
4 4 observer in Hz is
5 3 (a) 409 (b) 429 (c) 517 (d) 500
(c) n = 5, f 2 = f1 (d) n = 5, f 2 = f1
4 4 Q.12 Two sirens situated one kilometre apart are producing sound
Q.5 The source producing sound and an observer both are of frequency 330 Hz. An observer starts moving from one
moving along the direction of propagation of sound waves. siren to the other with a speed of 2 m/s. If the speed of
If the respective velocities of sound, source and an sound be 330 m/s, what will be the beat frequency heard
observer are v, vs and v0, then the apparent frequency heard by the observer
by the observer will be (n = frequency of sound) (a) 8 (b) 4 (c) 6 (d) 1
n (v + v0 ) n(v - v0 ) Q.13 A small source of sound moves on a circle as shown in the
(a) (b)
v - v0 v - vs figure and an observer is standing on O. Let n1, n2 and n3 be
n (v - v0 ) n(v + v0 ) the frequencies heard when the source is at A, B and
(c) (d)
v + vs v + vs
C respectively. Then
Q.6 A whistle sends out 256 waves in a second. If the whistle
approaches the observer with velocity 1/3 of the velocity (a) n1 > n2 > n3
of sound in air, the number of waves per second the (b) n2 > n3 < n1
observer will receive (c) n1 = n2 > n3
(a) 384 (b) 192 (c) 300 (d) 200 (d) n2 > n1 > n3 O
Q.7 A source of sound emitting a note of frequency 200 Hz Q.14 A person carrying a whistle emitting continuously a note
moves towards an observer with a velocity v equal to the of 272 Hz is running towards a reflecting surface with a
velocity of sound. If the observer also moves away from speed of 18 km/hour. The speed of sound in air is 345ms–
the source with the same velocity v, the apparent frequency 1. The number of beats heard by him is
heard by the observer is (a) 4 (b) 6 (c) 8 (d) 3
(a) 50 Hz (b) 100 Hz (c) 150 Hz (d) 200 Hz Q.15 A source of sound of frequency 256 Hz is moving rapidly
Q.8 The speed of sound in air at a given temperature is 350 m/ towards a wall with a velocity of 5m/s. The speed of sound
s. An engine blows whistle at a frequency of 1200 cps. It is is 330 m/s. If the observer is between the wall and the
approaching the observer with velocity 50 m/s. The apparent source, then the beats heard per second will be
frequency in cps heard by the observer will be (a) 7.8 Hz (b) 7.7 Hz
(a) 600 (b) 1050 (c) 1400 (d) 2400 (c) 3.9 Hz (d) Zero
Q.9 A source of sound of frequency n is moving towards a Q.16 The harmonics which are present in a pipe open at one end
stationary observer with a speed S. If the speed of sound in are
air is V and the frequency heard by the observer is n1 ,the (a) odd harmonics
value of n1 / n is (b) even harmonics
(c) even as well as odd harmonics
(a) (V + S)/V (b) V/ (V + S)
(d) None of these
(c) (V – S)/V (d) V/ (V – S)

4. 5. 6. 7. 8.
RESPONSE
9. 10. 11. 12. 13.
GRID
14. 15. 16.

Space for Rough Work


t.me/Ebooks_Encyclopedia27. t.me/Magazines4all

DPP/ P 30 3
Q.17 A source of sound placed at the open end of a resonance
DIRECTIONS (Q.23-Q.25) : In the following questions,
column sends an acoustic wave of pressure amplitude P0
more than one of the answers given are correct. Select the
inside the tube. If the atmospheric pressure is PA, then the
correct answers and mark it according to the following
ratio of maximum and minimum pressure at the closed end codes:
of the tube will be
( PA + P0 ) ( PA + 2 P0 ) Codes :
(a) ( P - P ) (b) ( P - 2 P ) (a) 1, 2 and 3 are correct (b) 1 and 2 are correct
A 0 A 0 (c) 2 and 4 are correct (d) 1 and 3 are correct
æ 1 ö Q.23 You are given four tuning forks, the lowest frequency of
çè PA + P0 ÷ø the fork is 300Hz. By striking two tuning forks at a time
PA 2
(c) P (d) æ 1 ö any of 1, 2, 3, 5, 7 & 8 Hz beat frequencies are heard. The
A çè PA - P0 ÷ø possible frequencies of the other three forks –
2
(1) 301, 302 & 307 (2) 301, 303 & 308
Q.18 The frequency of fundamental tone in an open organ pipe (3) 300, 304 & 307 (4) 305, 307 & 308
of length 0.48 m is 320 Hz. Speed of sound is 320 m/sec. Q.24 Doppler shift in frequency depends upon
Frequency of fundamental tone in closed organ pipe will (1) the frequency of the wave produced
be (2) the velocity of the source
(a) 153.8 Hz (b) 160.0 Hz (3) the velocity of the observer
(c) 320.0 Hz (d) 143.2 Hz (4) distance from the source to the listener
Q.19 A standing wave having 3 nodes and 2 antinodes is formed Q.25 The ( x, y ) coordinates of the corners of a square plate are
between two atoms having a distance 1.21Å between them. (0, 0), (L, 0), (L, L) and (0, L). The edges of the plate are
The wavelength of the standing wave is clamped and transverse standing waves are set up in it. If
(a) 1.21 Å (b) 2.42 Å (c) 6.05 Å (d) 3.63 Å u ( x, y) denotes the displacement of the plate at the point
Q.20 A string on a musical instrument is 50 cm long and its
( x, y ) at some instant of time, the possible expression(s)
fundamental frequency is 270 Hz. If the desired frequency
of 1000 Hz, is to be produced, the required length of the for u is (are) (a = positive constant)
string is px py px 2 py
(1) a sin sin (2) a sin sin
(a) 13.5 cm (b) 2.7 cm (c) 5.4 cm (d) 10.3 cm L L L L
Q.21 The loudness and the pitch of a sound depends on px py 2px py
(a) intensity and velocity (3) a cos cos (4) a cos cos
2L 2L L L
(b) frequency and velocity
(c) intensity and frequency DIRECTIONS (Q.26-Q.27) : Read the passage given below
(d) frequency and number of harmonics and answer the questions that follows :
Q.22 If in an experiment for determination of velocity of sound
A plate was cut from a quartz crystal and is used to control the
by resonance tube method using a tuning fork of 512 Hz,
frequency of an oscillating electrical circuit. Longitudinal
first resonance was observed at 30.7 cm and second was
standing waves are set up in the plate with displacement antinodes
obtainded at 63.2 cm, then maximum possible error in
at opposite faces. The fundamental frequency of vibration is
velocity of sound is (consider actual speed of sound in air
is 332 m/s) 2.87 ´ 104
given by the equation f0 = . Here s is thickness of
(a) 204 cm/sec (b) 110 cm/sec s
(c) 58 cm/sec (d) 80 cm/sec the plate and density of quartz is 2658.76 kg/m3.

RESPONSE 17. 18. 19. 20. 21.


GRID 22. 23. 24. 25.

Space for Rough Work


t.me/Ebooks_Encyclopedia27. t.me/Magazines4all

EBD_7156
4 DPP/ P 30
Q.26 Young’s modulus of elasticity for quartz is – Q.28 Statement-1 : Beats cannot be produced by light sources.
(a) 7 × 1011 N/m2 (b) 8.76 × 1012 N/m2 Statement-2: Light sources have constant phase
12
(c) 2 × 10 N/m 2 (d) Information insufficient difference.
Q.27 If the quartz plate is vibrating in 3rd harmonic while
Q.29 Statement-1 : In the case of a stationary wave, a person
measuring the frequency of 1.2 × 106 Hz, then the thickness
hear a loud sound at the nodes as compared to the antinodes.
of the plateis
(a) 71.75cm (b) 7.175 cm Statement-2 : In a stationary wave all the particles of the
(c) 6.02 cm (d) 0.07 cm medium vibrate in phase.
Q.30 Statement-1 : Velocity of particles, while crossing mean
DIRECTIONS (Q.28-Q.30) : Each of these questions contains position (in stationary waves) varies from maximum at
two statements: Statement-1 (Assertion) and Statement-2 antinodes to zero at nodes.
(Reason). Each of these questions has four alternative choices,
Statement-2: Amplitude of vibration at antinodes is
only one of which is the correct answer. You have to select the
maximum and at nodes, the amplitude is zero, and all
correct choice.
particles between two successive nodes cross the mean
position together.

RESPONSE GRID 26. 27. 28. 29. 30.

DAILY PRA CTICE PROBLEM SHEET 30 - PHYSICS


Total Questions 30 Total Marks 120
Attempted Correct
Incorrect N et Score
Cut-off Score 30 Qualifying Score 48
Success Gap = Net Score – Qualifying Score
Net Score = (Correct × 4) – (Incorrect × 1)

Space for Rough Work


t.me/Ebooks_Encyclopedia27. t.me/Magazines4all

DPP - Daily Practice Problems


Name : Date :

Start Time : End Time :

SYLLABUS : Practical Physics - 1


31
Max. Marks : 120 Time : 60 min.
GENERAL INSTRUCTIONS
• The Daily Practice Problem Sheet contains 30 MCQ's. For each question only one option is correct. Darken the correct
circle/ bubble in the Response Grid provided on each page.
• You have to evaluate your Response Grids yourself with the help of solution booklet.
• Each correct answer will get you 4 marks and 1 mark shall be deduced for each incorrect answer. No mark will be given/
deducted if no bubble is filled. Keep a timer in front of you and stop immediately at the end of 60 min.
• The sheet follows a particular syllabus. Do not attempt the sheet before you have completed your preparation for that
syllabus. Refer syllabus sheet in the starting of the book for the syllabus of all the DPP sheets.
• After completing the sheet check your answers with the solution booklet and complete the Result Grid. Finally spend time
to analyse your performance and revise the areas which emerge out as weak in your evaluation.

DIRECTIONS (Q.1-Q.24) : There are 24 multiple choice (a) N (b) N – 1 (c) 1/10 N (d) 1/N – 1
questions. Each question has 4 choices (a), (b), (c) and (d), out Q.4 The figure shows a situation when the jaws of vernier are
of which ONLY ONE choice is correct. touching each other. Each main scale division is of 1 mm.
Find zero correction.
Q.1 One cm on the main scale of a vernier callipers is divided
(a) – 0.5 mm main
into ten equal parts. If 20 divisions of vernier scale coincide 0 1 2(cm)
(b) + 0.5 mm scale
with 8 small divisions of the main scale. What will be the
least count of callipers ? (c) – 0.4 mm 0 5 10 Vernier
(a) 0.05 cm (b) 0.06 cm (d) + 0.4 mm scale
(c) 0.04 cm (d) 0.01 cm
Q.2 The shape of stress vs strain graph within elastic limit is : Q.5 In an experiment for measurement of young’s modulus,
(a) parabolic (b) curve line following readings are taken. Load = 3.00 kg, length =
(c) straight line (d) ellipse 2.820 m, diameter = 0.041 cm and extension = 0.87.
Q.3 In a vernier calliper N divisions of vernier scale coincides Determine the percentage error in the measurement of Y.
with N – 1 divisions of main scale (in which length of one (a) ± 5% (b) ± 6.5%
division is 1 mm). The least count of the instrument should
be (c) ± 5.5% (d) ± 15%

RESPONSE GRID 1. 2. 3. 4. 5.
Space for Rough Work
t.me/Ebooks_Encyclopedia27. t.me/Magazines4all

EBD_7156
2 DPP/ P 31
Q.6 When the zero of the circular scale of a screw gauge that the screw gauge has a zero error of – 0.03 mm. While
coincides with the zero of the main scale before A and B measuring the diameter of a thin wire, a student notes the
come in contact then the instrument has
main scale reading of 3 mm and the number of circular
(a) positive zero error
scale divisions in line with the main scale as 35. The
(b) negative zero error AB
diameter of the wire is
(c) no zero error
(d) can't be said anything (a) 3.32 mm (b) 3.73 mm
Q.7 If h be the elevation or depression of a spherical surface (c) 3.67 mm (d) 3.38 mm
from the plane glass plate and c be the mean distance Q.12 In an experiment the angles are required to be measured
between two consecutive points corresponding to the using an instrument, 29 divisions of the main scale exactly
impressions made by the three legs of a spherometer then coincide with the 30 divisions of the vernier scale. If the
the radius of curvature is
smallest division of the main scale is half- a degree (=
c2 h c2 h 2 c2 h c2 2 0.5°), then the least count of the instrument is :
(a) - (b) + (c) + (d) +
6h 2 6h 2 6h 2 6h h (a) half minute (b) one degree
Q.8 The least count of a spherometer is given by
(c) half degree (d) one minute
(a) pitch × no. of circular divisions
pitch
Q.13 In a screw gauge, the zero of main scale coincides with
(b) fifth division of circular scale in figure (i). The circular
no. of circular divisions
no. of circular divisions divisions of screw gauge are 50. It moves 0.5 mm on main
(c)
pitch scale in one rotation. The diameter of the ball in figure (ii)
pitch is
(d) mean distance between two consecutive legs of the spherometer
Q.9 The specific heat of a solid is determined by the method 0 10
5
0 30
25

known as 0 20

(a) the method of fusion


(b) the method of mixture
(c) the method of vaporisation Figure (i) Figure (ii)

(d) the method of cooling (a) 2.25 mm (b) 2.20 mm


Q.10 Which principle is involved in the experiment to determine (c) 1.20 mm (d) 1.25 mm
the specific heat of a liquid by the method of mixture ?
Q.14 A student performs an experiment for determination of
(a) Heat gained by solid = Heat lost by calorimeter and
liquid. æ 4p 2 l ö
(b) Heat lost by solid = Heat gained by calorimeter and gç= ÷ . The error in length l is Dl and in time T is DT
è T2 ø
liquid.
(c) Heat lost by solid and liquid = Heat gained by and n is number of times the reading is taken. The
calorimeter. measurement of g is most accurate for
(d) Heat gained by solid and calorimeter = Heat lost by Dl DT n
liquid. (a) 5 mm 0.2 sec 10
Q.11 Two full turns of the circular scale of a screw gauge cover (b) 5 mm 0.2 sec 20
a distance of 1mm on its main scale. The total number of (c) 5 mm 0.1 sec 10
divisions on the circular scale is 50. Further, it is found (d) 1 mm 0.1 sec 50

RESPONSE 6. 7. 8. 9. 10.
GRID 11. 12. 13. 14.

Space for Rough Work


t.me/Ebooks_Encyclopedia27. t.me/Magazines4all

DPP/ P 31 3
Q.15 A student performs an experiment to determine the Young’s Vg (r1 – r2 ) Vgr1
modulus of a wire, exactly 2 m long, by Searle’s method. In (a) (b)
k k
a particular reading, the student measures the extension in Vg r1 Vg (r1 – r2 )
the length of the wire to be 0.8 mm with an uncertainty of ± (c) (d)
k k
0.05 mm at a load of exactly 1.0 kg. The student also Q.19 A jar is filled with two non-mixing liquids 1 and 2 having densities
measures the diameter of the wire to be 0.4 mm with an r1 and, r2 respectively. A solid ball, made of a material of density
uncertainty of ± 0.01 mm. Take g = 9.8 m/s2 (exact). The r3 , is dropped in the jar. It comes to equilibrium in the position
Young’s modulus obtained from the reading is shown in the figure.Which of the following is true for r1, r2and
(a) (2.0±0.3)×1011 N/m2 (b) (2.0 ± 0.2) × 1011 N/m2 r3?
(c) (2.0±0.1)×1011 N/m2 (d) (2.0 ± 0.05) × 1011 N/m2
(a) r3 < r1 < r2
r1

Q.16 Students I, II and III perform an experiment for measuring


the acceleration due to gravity (g) using a simple pendulum. (b) r1 > r3 > r2 r3

They use different lengths of the pendulum and /or record (c) r1 < r2 < r3
time for different number of oscillations. The observations (d) r1 < r3 < r2
are shown in the table. Q.20 A capillary tube (A) is dipped in water. Another identical
Least count for length = 0.1 cm tube (B) is dipped in a soap-water solution. Which of the
Least count for time = 0.1 s following shows the relative nature of the liquid columns
Student Length of the No. of Total time for Time in the two tubes?
pendulum (cm) oscillations (n) oscillations period (s) A B
(n) (s) A B

I 64.0 8 128.0 16.0


II 64.0 4 64.0 16.0 (a) (b)
III 20.0 4 36.0 9.0
If EI, EII and EIII are the percentage errors in g, i.e., A B A B

æ Dg ö
çè g ´ 100÷ø for students I, II and III, respectively, then (c) (d)
(a) EI = 0 (b) EI is minimum
(c) EI = EII (d) EII is maximum Q.21 Two wires are made of the same material and have the same
Q.17 If the terminal speed of a sphere of gold (density volume. However wire 1 has cross-sectional area A and
wire 2 has cross-sectional area 3A. If the length of wire 1
= 19.5 kg/m3) is 0.2 m/s in a viscous liquid (density = 1.5
increases by Dx on applying force F, how much force is
kg/m 3 ), find the terminal speed of a sphere of silver needed to stretch wire 2 by the same amount of energy?
(density = 10.5 kg/m3) of the same size in the same liquid (a) 4 F (b) 6 F (c) 9 F (d) 1 F
(a) 0.4 m/s (b) 0.133 m/s Q.22 The vernier constant of two vernier callipers A and B are
(c) 0.1 m/s (d) 0.2 m/s 0.01 cm and 0.01 mm respectively. Which one can measure
Q.18 A spherical solid ball of volume V is made of a material of the length of an object more accurately?
density r1. It is falling through a liquid of density r2 (r2< (a) Vernier A (b) Vernier B
r1). Assume that the liquid applies a viscous force on the (c) Accuracy in measurement does not depend on vernier
constant
ball that is proportional to the square of its speed v, i.e.,
(d) Both A and B are equally accurate.
Fviscous = –kv2 (k > 0). The terminal speed of the ball is

RESPONSE 15. 16. 17. 18. 19.


GRID 20. 21. 22.

Space for Rough Work


t.me/Ebooks_Encyclopedia27. t.me/Magazines4all

EBD_7156
4 DPP/ P 31
Q.23 The acceleration due to gravity at a place can be determined (2) Distilled water should be avoided.
with the help of a simple pendulum. For this purpose (3) Distilled water should be added.
effective length of the pendulum is considered. If 'l' be (4) Dirty liquid should be used.
the length of the string and 'd' the diameter of the bob then Q.27 The temperature-time variation graphs, as obtained by four
the effective length is equal to students 1, 2, 3 and 4 are as shown. The graphs, likely to be
d d wrong are
(a) l + d (b) l + (c) l - (d) l – d
2 2 80 80

Q.24 If x, y, p and q represent the increase in length, the original 70 70

Temperature
Temperature
length of the experimental wire, load applied to the wire (1)
60
(2)
60
50 50
and area of cross-section of the wire respectively then 40 40
Young's modulus of the wire is given by 30 30
xy xp py pq 0 5 10 10 20 25 30 35 0 5 10 10 20 25 30 35
(a) pq
(b) yq (c) xq (d) xy 80 Time 80 Time
70 70

Temperature

Temperature
DIRECTIONS (Q.25-Q.27) : In the following questions, (3)
60
(4)
60
50 50
more than one of the answers given are correct. Select the 40 40
correct answers and mark it according to the following 30 30

codes: 0 5 10 10
Time
20 25 30 35 0 5 10 10 20
Time
25 30 35

Codes : DIRECTIONS (Q.28-Q.30) : Read the passage given below


(a) 1, 2 and 3 are correct (b) 1 and 2 are correct and answer the questions that follows :
(c) 2 and 4 are correct (d) 1 and 3 are correct
Q.25 What is the function of a screw gauge in the experiment of The internal radius of a 1m long resonance tube is measured as
determining Young's modulus of a wire ? 3 cm. A tuning fork of frequency 2000 Hz is used. The first
(1) It measures the extension in the wire. resonating length is measured as 4.6 cm and the second
(2) It measures the load applied. resonating length is measured as 14.0 cm.
(3) It measures the length of the wire. Q.28 Calculate the maximum percentage error in measurement
(4) It measures the diameter of the wire. of e.
Q.26 Consider the following statements regarding the (a) 3.33% (b) 2.23% (c) 4.33% (d) 5.33%
experiment to determine the surface tension of water by Q.29 Calculate the speed of sound at the room temperature.
capillary rise method. Choose the correct statements. (a) 275 m/s (b) 376 m/s (c) 356 m/s (d) 330 m/s
(1) Capillary tube should be clean and liquid should be Q.30 Calculate the end correction.
free from dirt and grease. (a) 0.2 cm (b) 0.3 cm (c) 0.1 cm (d) 0.4 cm

RESPONSE 23. 24. 25. 26. 27.


GRID 28. 29. 30.

DAILY PRA CTICE PROBLEM SHEET 31 - PHYSICS


Total Questions 30 Total Marks 120
Attempted Correct
Incorrect N et Score
Cut-off Score 30 Qualifying Score 48
Success Gap = Net Score – Qualifying Score
Net Score = (Correct × 4) – (Incorrect × 1)
Space for Rough Work
t.me/Ebooks_Encyclopedia27. t.me/Magazines4all

DPP - Daily Practice Problems


Name : Date :

Start Time : End Time :

SYLLABUS : Electrostatics-1 (Coulomb's law, electric field, field lines, Gauss's law)
32
Max. Marks : 104 Time : 60 min.
GENERAL INSTRUCTIONS
• The Daily Practice Problem Sheet contains 26 MCQ's. For each question only one option is correct. Darken the correct
circle/ bubble in the Response Grid provided on each page.
• You have to evaluate your Response Grids yourself with the help of solution booklet.
• Each correct answer will get you 4 marks and 1 mark shall be deduced for each incorrect answer. No mark will be given/
deducted if no bubble is filled. Keep a timer in front of you and stop immediately at the end of 60 min.
• The sheet follows a particular syllabus. Do not attempt the sheet before you have completed your preparation for that
syllabus. Refer syllabus sheet in the starting of the book for the syllabus of all the DPP sheets.
• After completing the sheet check your answers with the solution booklet and complete the Result Grid. Finally spend time
to analyse your performance and revise the areas which emerge out as weak in your evaluation.

DIRECTIONS (Q.1-Q.18) : There are 18 multiple choice 1 Q2 1 Q2


(c) 180o , o
(d) 180 , 4p Î 2
questions. Each question has 4 choices (a), (b), (c) and (d), out 4p Î0 2 L2 0 L
of which ONLY ONE choice is correct. Q.3 Electric charges of 1mC , -1mC and 2mC are placed in air at
Q.1 A total charge Q is broken in two parts Q1 and Q 2 and they the corners A,B and C respectively of an equilateral triangle
ABC having length of each side 10 cm. The resultant force
are placed at a distance R from each other. The maximum on the charge at C is
force of repulsion between them will occur, when (a) 0.9 N (b) 1.8N (c) 2.7 N (d) 3.6 N
Q Q Q 2Q Q.4 An electron is moving round the nucleus of a hydrogen
(a) Q 2 = , Q1 = Q - (b) Q2 = , Q1 = Q -
R R 4 3 atom in a circular orbit of radius r. The coulomb force
Q 3Q Q Q ur 1
(c) Q 2 = , Q1 = (d) Q1 = , Q2 = F between the two is (here K = )
4 4 2 2 4 p Î0
Q.2 Two small balls each having the charge + Q are suspended e r e2 r
2 2
e e2
by insulating threads of length L from a hook. This (a) - K 3 r$ (b) K 3 r (c) - K 3 r (d) K 2 r$
r r r r
arrangement is taken in space where there is no
gravitational effect, then the angle between the two Q.5 Equal charges q are 2placed at the four corners A2, B, C , D of a
o 1 Q 1 Q
suspensions and the tension in each thread will be (a) 180
square ,
of length a . The
2 magnitude ofothe
(b) 90 , force on the charge
4 p Î0 (2 L ) 4p Î0 L
2

RESPONSE GRID 1. 2. 3. 4.
Space for Rough Work
t.me/Ebooks_Encyclopedia27. t.me/Magazines4all

EBD_7156
2 DPP/ P 32
at B will be of –3Q, it is
3q 2
q 2 (a) –E (b) E/3 (c) –3E (d) –E/3
(a) (b) Q.12 Charges q, 2q, 3q and 4q are placed at the corners A, B, C
4p Î0 a 2 p Î0 a 2
æ 1+ 2 2 ö q2
and D of a square as shown in the following figure. The
æ 1 ö q2
(c) çè ÷ (d) ç 2 + ÷ø direction of electric field at the centre of the square is
2 ø 4p Î0 a 2 è 2 4 p Î0 a 2 parallel to side.
Q.6 The charges on two spheres are +7mC and –5mC D C
(a) AB 4q 3q
respectively. They experience a force F. If each of them is
given an additional charge of –2mC, the new force of (b) CB
attraction will be O
(c) BD q 2q
(a) F (b) F/2 (c) F / 3 (d) 2F (d) AC A B
Q.7 Electric lines of force about negative point charge are Q.13 Three infinitely long non-conducting charge sheets are
(a) Circular, anticlockwise (b) Circular, clockwise placed as shown in figure. The electric field at point P is
(c) Radial, inward (d) Radial, outward 2s $
Q.8 Figure shows the electric lines of force emerging from a (a) k
e0 Z
charged body. If the electric field at A and B are E A and
2s $
(b) – k Z = 3a
EB respectively and if the distance between A and B is r then e0
P
(a) E A > EB 4s $ -2 s Z=a
(c) k
(b) E A < E B e0 x

E A B 4s $ Z=–a
(c) E A = B
r (d) – k
r e0
EB Q.14 Gauss's law is true only if force due to a charge varies as
(d) E A = 2 (a) r–1 (b) r–2 (c) r–3 (d) r–4
r
Q.9 The magnitude of electric field intensity E is such that, Q.15 The electric intensity due to an infinite cylinder of radius
an electron placed in it would experience an electrical R and having charge q per unit length at a distance r(r > R)
force equal to its weight is given by from its axis is
e e2
(a) Directly proprotional to r2
mg
(a) mge (b) (c) mg (d) 2 g (b) Directly proprotional to r3
e m
(c) Inversely proprotional to r
Q.10 A charge particle is free to move in an electric field. It
(d) Inversely proprotional to r2
will travel
Q.16 A sphere of radius R has a uniform distribution of electric
(a) Always along a line of force
charge in its volume. At a distance x from its centre, for
(b) Along a line of force, if its initial velocity is zero
x < R, the eletric field is directly proportional to
(c) Along a line of force, if it has some initial velocity in
the direction of an acute angle with the line of force 1 1
(d) None of the above (a) (b) (c) x (d) x2
x2 x
Q.11 Two point charges Q and –3Q are placed at some distance apart.
Q.17 A charged ball B hangs from a silk thread S, which makes an
If the electric field at the location of Q is E then at the locality
angle q with a large charged conducting sheet P, as shown

5. 6. 7. 8. 9.
RESPONSE
10. 11. 12. 13. 14.
GRID
15. 16.

Space for Rough Work


t.me/Ebooks_Encyclopedia27. t.me/Magazines4all

DPP/ P 32 3
in the figure. The surface charge density s of the sheet is (3) If a uniform downward electric field is switched on, T will
proportional to increase
+ (4) If a uniform downward electric field is switched on,
(a) sin q +
P + T will decrease
(b) tan q + q
+ DIRECTIONS (Q.21-Q.23) : Read the passage given below
(c) cos q + S
+ and answer the questions that follows :
(d) cot q B
Q.18 A charge q is placed at the centre of a cube. Then the flux A sphere of radius R contains charge density r (r ) = A( R - r ),
passing through one face of cube will be for 0 < r < R. The total electric charge inside the sphere is Q.
q q q q
(a) (b) (c) (d)
Î0 2 Î0 4 Î 0 6 Î0 Q.21 The value of A in terms of Q and R is
DIRECTIONS (Q.19-Q.20) : In the following questions, 2Q 2 3Q 3Q 2 3Q
(a) (b) (c) (d)
more than one of the answers given are correct. Select pR 4 pR 4 pR 3 pR
the correct answers and mark it according to the following Q.22 The electric field inside the sphere is
codes:
12Q2 é 1 æ r ö 1 æ r ö ù
2
3Q é 1 æ r ö 1 æ r ö ù
2
Codes : (a) ê ç ÷- ç ÷ ú (b) ê ç ÷- ç ÷ ú
Î0 R 2 êë 3 è R ø 4 è R ø úû R3 êë 3 è R ø 4 è R ø úû
(a) 1, 2 and 3 are correct (b) 1 and 2 are correct
120Q é 1 æ r ö 1 æ r ö ù 12 é 1 æ r ö 1 æ r ö ù
2 2
(c) 2 and 4 are correct (d) 1 and 3 are correct
(c) ê ç ÷ - ç ÷ ú (d) 2 ê ç ÷ - ç ÷ ú
Q.19 A solid sphere S1 is connected to a charge reservoir through 5 Î0 R 2 êë 4 è R ø 3 è R ø úû R Q êë 3 è R ø 4 è R ø úû
a heater H as shown in figure.
Q.23 The electric field outside the sphere is æç k =
S2 1 ö
S1
H è 4 p Î0 ÷ø
Solid Solid
Heater
sphere sphere kQ kQ kQ kQ 2
(a) (b) (c) (d)
Flux through a closed spherical surface around S1 is given r r2 r3 r2
by f = a t2 where a is a constant and t is time in seconds.
If resistance of heater is R then select correct statements DIRECTIONS (Q. 24-Q.26) : Each of these questions
(1) Power consumed by heater will be 4a2e02 Rt2. contains two statements: Statement-1 (Assertion) and
(2) Electric flux through a closed spherical surface around Statement-2 (Reason). Each of these questions has four
S2 will be – a t2. alternative choices, only one of which is the correct answer.
(3) Rate of change of electric flux through a closed You have to select the correct choice.
spherical surface around S2 will be –2a t (a) Statement-1 is True, Statement-2 is True; Statement-2 is a
(4) All of the above are correct correct explanation for Statement-1.
Q.20 A simple pendulum has a time period T. The bob is now (b) Statement-1 is True, Statement-2 is True; Statement-2 is
given some positive charge – NOT a correct explanation for Statement-1.
(1) If some positive charge is placed at the point of (c) Statement -1 is False, Statement-2 is True.
suspension, T will increases (d) Statement -1 is True, Statement-2 is False.
(2) If some positive charge is placed at the point of
suspension, T will not change Q.24 Statement-1 : Electric lines of force cross each other.
Statement-2 : Electric field at a point superimpose to give

RESPONSE 17. 18. 19. 20. 21.


GRID 22. 23.

Space for Rough Work


t.me/Ebooks_Encyclopedia27. t.me/Magazines4all

EBD_7156
4 DPP/ P 32
one resultant electric field. Q.26 Statement-1 : Direction of electric field at a point signifies
Q.25 Statement-1 : A point charge is brought in an electric field. The direction of force experienced by a point charge placed at
field at a nearby point will increase, whatever be the nature of that point.
the charge. Statement-2 : Electric field is a vector quantity.
Statement-2 : The electric field is dependent on the nature of
charge.

RESPONSE GRID 24. 25. 26.

DAILY PRA CTICE PROBLEM SHEET 32 - PHYSICS


Total Questions 26 Total Marks 104
Attempted Correct
Incorrect N et Score
Cut-off Score 25 Qualifying Score 40
Success Gap = Net Score – Qualifying Score
Net Score = (Correct × 4) – (Incorrect × 1)

Space for Rough Work


t.me/Ebooks_Encyclopedia27. t.me/Magazines4all

DPP - Daily Practice Problems


Name : Date :

Start Time : End Time :

SYLLABUS : Electrostatics-2 (Electric potential and potential difference,


33
equipotential surfaces, electric dipole)

Max. Marks : 104 Time : 60 min.


GENERAL INSTRUCTIONS
• The Daily Practice Problem Sheet contains 26 MCQ's. For each question only one option is correct. Darken the correct
circle/ bubble in the Response Grid provided on each page.
• You have to evaluate your Response Grids yourself with the help of solution booklet.
• Each correct answer will get you 4 marks and 1 mark shall be deduced for each incorrect answer. No mark will be given/
deducted if no bubble is filled. Keep a timer in front of you and stop immediately at the end of 60 min.
• The sheet follows a particular syllabus. Do not attempt the sheet before you have completed your preparation for that
syllabus. Refer syllabus sheet in the starting of the book for the syllabus of all the DPP sheets.
• After completing the sheet check your answers with the solution booklet and complete the Result Grid. Finally spend time
to analyse your performance and revise the areas which emerge out as weak in your evaluation.

DIRECTIONS (Q.1-Q.19) : There are 19 multiple choice Q2. Four equal charges Q are placed at the four corners of a
questions. Each question has 4 choices (a), (b), (c) and (d), square of each side ' a ' . Work done in removing a charge
out of which ONLY ONE choice is correct. – Q from its centre to infinity is
Q1. In the electric field of a point charge q , a certain charge is 2Q2 2Q2 Q2
carried from point A to B, C, D and E. Then the work done (a) 0 (b) (c) (d)
4pÎ0 a pÎ0 a 2pÎ0 a
by electric force is Q3. A particle A has charge +q and a particle B has charge +4q
A
(a) least along the path AB with each of them having the same mass m. When allowed
(b) least along the path AD to fall from rest through the same electric potential
(c) zero along all the paths AB, vA
B
+q
E difference, the ratio of their speed will become
AC, AD and AE vB
C D (a) 2 : 1 (b) 1 : 2
(d) least along AE
(c) 1 : 4 (d) 4 : 1

RESPONSE GRID 1. 2. 3.
Space for Rough Work
t.me/Ebooks_Encyclopedia27. t.me/Magazines4all

EBD_7156
2 DPP/ P 33
Q4. In the figure the charge Q is at the centre of the circle. Q9. Two electric charges 12µC and – 6µC are placed 20 cm
Work done (by electrostatic force on q) is maximum when apart in air. If there will be a point P on the line joining these
another charge q is taken from point P to (consider both charges and outside the region between them, at which the
the charges to be positive) electric potential is zero, then the distance of P from – 6µC
K
P charge is
(a) 0.10 m (b) 0.15 m (c) 0.20 m (d) 0.25 m
Q10. In the rectangle shown below, the two corners have charges
L
Q q1 = –5 mC and q2 = +2.0 mC. The work done by external
agent in moving a charge q = +3.0 mC slowly from B to A is
q1 A
M
N (Take 1/ 4pe 0 =1010 Nm2 /C2 )
(a) K (b) L (c) M (d) N (a) 2.8 J

5 cm
Q5. How much kinetic energy will be gained by an a- particle (b) 3.5 J
in going from a point at 70V to another point at 50V ? (c) 4.5 J
(a) 40 eV (b) 40 keV (c) 40 MeV (d) 0 eV (d) 5.5 J B 15 cm
q 2

Q6. Ten electrons are equally spaced and fixed around a circle Q11. Electric charges q, q, –2q are placed at the corners of an
of radius R. Relative to V = 0 at infinity, the electrostatic equilateral triangle ABC of side l. The magnitude of electric
potential V and the electric field E at the centre C are dipole moment of the system is
ur ur
(a) V ¹ 0 and E ¹ 0 (b) V ¹ 0 and E = 0 (a) ql (b) 2ql (c) 3ql (d) 4ql
ur ur
(c) V = 0 and E = 0 (d) V = 0 and E ¹ 0 Q12. A charge (–q) and another charge (+Q) are kept at two
Q7. The displacement of a charge Q in the electric field points A and B respectively. Keeping the charge (+Q) fixed
ur r at B, the charge (–q) at A is moved to another point C such
E = e1i$ + e2 $j + e3 k$ is r = ai$ + b $j . The work done is that ABC forms an equilateral triangle of side l . The net
work done by electrostatic field in moving the charge (–q)
(a) Q ( ae1 + be2 ) (b) Q ( ae1 )2 + (be2 )2 is
æ 2 2ö 1 Qq 1 Qq
(c) Q ( e1 + e2 ) a 2 + b 2 (d) Q ç e1 + e2 ÷ ( a + b ) (a) 4pÎ0 l (b) 4pÎ 2
è ø 0 l
Q8. As shown in the figure, charges + q and -q are placed at 1
(c) Qql (d) zero
the vertices B and C of an isosceles triangle. The potential 4p Î0
at the vertex A is Q13. In an hydrogen atom, the electron revolves around the
nucleus in an orbit of radius 0.53 × 10–10m. Then the
1 2q
. A electrical potential produced by the nucleus at the position
(a) 4pe0 a 2 + b2 of the electron is
(b) Zero (a) –13.6V (b) –27.2V (c) 27.2V (d) 13.6V
a
1 q Q14. Point charge q1 = 2mC and q 2 = -1mC are kept at points x =
(c) . b b
4pe0 a 2 + b2 B C 0 and x = 6 respectively. Electrical potential will be zero at
+q –q
points
(d)
1 ( -q ) (a) x = 2 and x = 9 (b) x = 1 and x = 5
.
4pe0 a 2 + b2 (c) x = 4 and x = 12 (d) x = -2 and x = 2

4. 5. 6. 7. 8.
RESPONSE
9. 10. 11. 12. 13.
GRID
14.

Space for Rough Work


t.me/Ebooks_Encyclopedia27. t.me/Magazines4all

DPP/ P 33 3
Q15. The distance between H+ and Cl– ions in HCl molecule is 1.28 DIRECTIONS (Q.20-Q.21) : In the following questions,
Å. What will be the potential due to this dipole at a distance of more than one of the answers given are correct. Select the
12 Å on the axis of dipole correct answers and mark it according to the following
(a) 0.13 V (b) 1.3 V (c) 13 V (d) 130 V codes:
Q16. Two identical thin rings each of radius R metres are
coaxially placed at a distance R metres apart. If Q1 coulomb Codes :
and Q2 coulomb are respectively the charges uniformly (a) 1, 2 and 3 are correct (b) 1 and 2 are correct
spread on the two rings, the work done by external agent in (c) 2 and 4 are correct (d) 1 and 3 are correct
moving a charge q slowly from the centre of ring with q q 2q
Q20. Consider a system of three charges , and - placed at
charge Q1 to that of other is 3 3 3
point A, B and C respectively, as shown in the figure. Take
q(Q2 - Q1 )( 2 - 1)
(a) zero (b) O to be the centre of the circle of radius R and angle
2.4 p Î0 R CAB = 60°. Choose the incorrect options
q 2(Q1 + Q2 ) q (Q1 + Q2 )( 2 + 1) q
(c) (d) (1) The electric field at point O is
4p Î0 R 4p Î0 R 8pe 0 R 2 y

Q17. Identical point charges, each having + q charge, are fixed directed along the negative x-axis B

at each of the points x = x0 , x = 3x0 , x = 5 x0 ......... (2) The potential energy of the
C
infinite, on the x-axis and a identical point charges, each system is zero O
x

having -q charge, ar e fixed at each of the points


60°
q
(3) The potential at point O is A
x = 2 x0 , x = 4 x0 , x = 6 x0 ... .... i nfinit e. Her e x0 is a 12pe 0 R
positive constant. Potential at the origin due to the above (4) The magnitude of the force between the charges at C
system of charges is q2
and B is
q 54pe 0 R 2
(a) 0 (b)
8pÎ0 x0 ln 2 Q21. For spherical symmetrical charge distribution, variation
q ln 2 of electric potential with distance from centre is given in
(c) ¥ (d)
4pÎ0 x0 q q
diagram. Given that : V = for r £ R0 and V =
Q18. A uniform electric field pointing in positive x - direction 4pe 0 R 0 4 pe 0r
V
exists in a region. Let A be the origin, B be the point on the for r ³ R0
x - axis at x = +1 cm and C be the point on the y - axis at Then which option (s) are correct :
y = +1 cm. Then the potentials at the points A, B and C (1) Total charge within 2R0 is q
(2) Total electrostatic energy r
satisfy
for r £ R0 is non-zero r = R0
(a) VA < VB (b) VA > VB (c) VA < VC (d) VA > VC (3) At r = R0 electric field is discontinuous
Q19. A point Q lies on the perpendicular bisector of an electrical (4) There will be no charge anywhere except at r < R0.
dipole of dipole moment p . If the distance of Q from the
DIRECTIONS (Q.22-Q.23) : Read the passage given below
dipole is r (much larger than the size of the dipole), then
and answer the questions that follows :
electric field at Q is proportional to
(a) P–1and r –2 (b) p and r–2 An electric dipole (AB) consisting of two particles of equal and
opposite charge and same mass is released in an electric field.
(c) P2 and r–3 (d) p and r–3
In the figure field lines are without considering effect of field
of dipole.

RESPONSE 15. 16. 17. 18. 19.


GRID 20. 21.

Space for Rough Work


t.me/Ebooks_Encyclopedia27. t.me/Magazines4all

EBD_7156
4 DPP/ P 33
DIRECTIONS (Q. 24-Q.26) : Each of these questions contains
B +q two statements: Statement-1 (Assertion) and Statement-2
y
(Reason). Each of these questions has four alternative choices,
–q only one of which is the correct answer. You have to select the
A correct choice.
x
(a) Statement-1 is True, Statement-2 is True; Statement-2 is a
correct explanation for Statement-1.
Q22. The centre of mass of the dipole (b) Statement-1 is True, Statement-2 is True; Statement-2 is
(a) Has no acceleration NOT a correct explanation for Statement-1.
(b) Has acceleration with positive x and y components (c) Statement -1 is False, Statement-2 is True.
(c) Has acceleration with positive x component and nega- (d) Statement -1 is True, Statement-2 is False.
tive y component Q24. Statement -1 : A bird perches on a high power line and
(d) Has acceleration with negative x component and posi- nothing happens to the bird.
tive y component Statement -2 : The level of bird is very high from the
Q23. Angular acceleration of the dipole, immediately after it is ground.
released Q25. Statement -1 : Electrons move away from a low potential
(a) is zero to high potential region.
(b) is clockwise Statement- 2 : Because electrons have negative charge
(c) is anticlockwise Q26. Statement -1 : Surface of a symmetrical conductor can
(d) cannot be determined from the given information. be treated as equipotential surface.
Statement -2 : Charges can easily flow in a conductor.

RESPONSE GRID 22. 23. 24. 25. 26.

DAILY PRA CTICE PROBLEM SHEET 33 - PHYSICS


Total Questions 26 Total Marks 104
Attempted Correct
Incorrect N et Score
Cut-off Score 26 Qualifying Score 42
Success Gap = Net Score – Qualifying Score
Net Score = (Correct × 4) – (Incorrect × 1)

Space for Rough Work


t.me/Ebooks_Encyclopedia27. t.me/Magazines4all

DPP - Daily Practice Problems


Name : Date :

Start Time : End Time :

SYLLABUS : ELECTROSTATICS -3 (Electrostatic Potential energy, conductors)


34
Max. Marks : 96 Time : 60 min.
GENERAL INSTRUCTIONS
• The Daily Practice Problem Sheet contains 24 MCQ's. For each question only one option is correct. Darken the correct
circle/ bubble in the Response Grid provided on each page.
• You have to evaluate your Response Grids yourself with the help of solution booklet.
• Each correct answer will get you 4 marks and 1 mark shall be deduced for each incorrect answer. No mark will be given/
deducted if no bubble is filled. Keep a timer in front of you and stop immediately at the end of 60 min.
• The sheet follows a particular syllabus. Do not attempt the sheet before you have completed your preparation for that
syllabus. Refer syllabus sheet in the starting of the book for the syllabus of all the DPP sheets.
• After completing the sheet check your answers with the solution booklet and complete the Result Grid. Finally spend time
to analyse your performance and revise the areas which emerge out as weak in your evaluation.

DIRECTIONS (Q.1-Q.16) : There are 16 multiple choice 1 q2 1 2q 2


questions. Each question has 4 choices (a), (b), (c) and (d), out (a) (b)
4pÎ0 l 4pÎ0 l
of which ONLY ONE choice is correct.
1 3q 2 1 4q 2
Q.1 Three charges Q, + q and + q are placed at the vertices of (c) 4pÎ0 l
(d) 4pÎ0 l
right-angled isosceles triangle as shown in the figure. The
Q.3 If identical charges (-q) are placed at each corner of a cube
net electrostatic energy of the configuration is zero if Q is
equal to Q of side b , then electric potential energy of charge
(+q) which is placed at centre of the cube will be
-q -2q
(a) (b) 8 2q2 -8 2q 2
1+ 2 2+ 2 (a) (b)
(c) –2q (d) + q
+q +q 4pÎ0 b p Î0 b
a

Q.2 Three charges of equal value ‘q’ are placed at the vertices -4 2q 2 -4q2
of an equilateral triangle. What is the net potential energy, (c) (d)
pÎ0 b 3pÎ0 b
if the side of equilateral Dis l ?

RESPONSE GRID 1. 2. 3.
Space for Rough Work
t.me/Ebooks_Encyclopedia27. t.me/Magazines4all

EBD_7156
2 DPP/ P 34
Q.4 Two charges q1 and q2 are placed 30 cm apart, shown in (a) decreases (b) increases
(c) remains unchanged (d) becomes zero
the figure. A third charge q3 is moved along the arc of a Q.9 Two identical charges are placed at the two corners of an
circle of radius 40 cm from C to D. The change in the equilateral triangle. The potential energy of the system is
U. The work done in bringing an identical charge from
q3
potential energy of the system is k , here k is infinity to the third vertex is
4pÎ0 (a) U (b) 2U (c) 3U (d) zero
(a) 8q2 q3
Q.10 Potential energy of two equal negative point charges 2mC
C held 1 m apart in air is
(b) 8q1 (a) 2 J (b) 2eV (c) 4 J (d) 0.036 J
(c) 6q2 40 cm
Q.11 Four charges + q, –q, + q and –q are put together on four
q2
corners of a square as shown in figure. The work done by
(d) 6q1 q1
D
external agent in slowly assembling this configuration is
A 30 cm B
+q –q

Q.5 Three particles, each having a charge of 10mC are placed


at the corners of an equilateral triangle of side 10 cm. The
electrostatic potential energy of the system is (Given
–q +q
1
= 9 ´109 N m 2 /C2 ) (a) zero (b) –2.59kq2/a
4p Î0
(c) +2.59kq2/a (d) none of these
(a) Zero (b) Infinite (c) 27 J (d) 100 J Q.12 As shown in figure a dust particle with mass m = 5.0 × 10 –9
Q.6 Two equal charges q are placed at a distance of 2a and a kg and charge q0 = 2.0 nC starts from rest at point a and
third charge -2q is placed at the midpoint. The potential moves in a straight line to point b . What is its speed v at
point b?
energy of the system is
3.0 nC a b –3.0 nC
q2 6q 2 –
(a) (b) 1.0 cm 1.0 cm 1.0 cm
8p Î0 a 8p Î0 a

7 q2 9q 2 (a) 26 ms–1 (b) 34 ms–1 (c) 46 ms–1 (d) 14 ms–1


(c) – (d)
8p Î0 a 8p Î0 a Q.13 Charges –q, Q and –q are placed at equal distance on a
Q.7 An electric dipole has the magnitude of its charge is q and straight line. If the total potential energy of the system of
its dipole moment is p . It is placed in a uniform electric three charges is zero, then find the ratio Q/q.
–q –q
field E. If its dipole moment is along the direction of the Q
r r
field, the force on it and its potential energy are
2r
respectively
(a) 1/2 (b) 1/4 (c) 2/3 (d) 3/4
(a) 2q.E and minimum (b) q.E and p.E Q.14 When the separation between two charges is increased,
(c) Zero and minimum (d) q.E and maximum the electric potential energy of the charges
Q8 In bringing an electron towards another electron, (a) increases (b) decreases
electrostatic potential energy of the system: (c) remains the same (d) may increase or decrease

4. 5. 6. 7. 8.
RESPONSE
9. 10. 11. 12. 13.
GRID
14.

Space for Rough Work


t.me/Ebooks_Encyclopedia27. t.me/Magazines4all

DPP/ P 34 3
Q.15 A positive charge is moved from a low potential point A to Q.18 A proton moves a distance d in a uniform electric field
high potential point B. Then the electric potential energy ®
E as shown in the figure. Then which of the following
of the system statements are correct ?
(a) increases ®
(b) decreases E
(c) will remain the same
(d) nothing definite can be predicted +
Q.16 If V and u are electric potential and energy density, p
respectively, at a distance r from a positive point charge, d
then which of the following graph is correct ? (1) Electric field do a negative work on the proton
4 4
V V (2) Electric potential energy of the proton increases
(3) Electric field do a positive work on the proton
(a) (b)
(4) Electric potential energy of the proton decreases
u u
V
4
V
4 DIRECTIONS (Q.19-Q.21) : Read the passage given below
and answer the questions that follows :
(c) (d) Three concentric spherical conductors A, B and C of radii R, 2R
u u
and 4R respectively. A and C is shorted and B is uniformly
DIRECTIONS (Q.17-Q.18) : In the following questions, charged.
more than one of the answers given are correct. Select the C
B +
correct answers and mark it according to the following +
+ +
+
A
codes: + +
+ +
Codes : 4R +
+
(a) 1, 2 and 3 are correct (b) 1 and 2 are correct R +
+
(c) 2 and 4 are correct (d) 1 and 3 are correct +
2R +
+
+
Q.17 S is a solid neutral conducting sphere. A point charge q of + +

1 × 10–6 C is placed at point A. C is the centre of sphere Q.19 Charge on conductor A is


and AB is a tangent. BC = 3m and AB = 4m. (a) Q/3 (b) – Q/3
A B (c) 2Q/3 (d) None of these
Q.20 Potential at A is
Q Q
(a) (b)
4pe 0 R 16 pe0 R
S Q
(c) (d) None of these
20pe0 R
(1) The electric potential of the conductor is 1.8 kV
Q.21 Potential at B is
(2) The electric potential of the conductor is 2.25 kV
Q Q
(3) The electric potential at B due to induced charges on (a) (b)
the sphere is – 0.45 kV 4 pe 0 R 16 pe0 R
(4) The electric potential at B due to induced charges on 5Q
(c) (d) None of these
the sphere is 0.45 kV 48pe0 R

RESPONSE 15. 16. 17. 18. 19.


GRID 20. 21.

Space for Rough Work


t.me/Ebooks_Encyclopedia27. t.me/Magazines4all

EBD_7156
4 DPP/ P 34
DIRECTIONS (Q. 22-Q.24) : Each of these questions contains Statement-2 : Inside the sphere electric potential is same
two statements: Statement-1 (Assertion) and Statement-2 at each point, but outside it is different for different points.
(Reason). Each of these questions has four alternative choices, Q.23 Statement-1 : Electric potential of earth is taken to be
only one of which is the correct answer. You have to select the zero as a reference.
correct choice. Statement-2 : The electric field produced by earth in
surrounding space is zero.
(a) Statement-1 is True, Statement-2 is True; Statement-2 is a
Q.24 Statement - 1 : The electric potential and the electric field
correct explanation for Statement-1.
intensity at the centre of a square having four fixed point
(b) Statement-1 is True, Statement-2 is True; Statement-2 is
charges at their vertices as shown in figure are zero.
NOT a correct explanation for Statement-1.
(c) Statement -1 is False, Statement-2 is True. +q –q
(d) Statement -1 is True, Statement-2 is False.
Q.22 Statement-1 : No work is done in taking a small positive
charge from one point to other inside a positively charged
metallic sphere while outside the sphere work is done in
taking the charge towards the sphere. Neglect induction
–q +q
due to small charge.
Statement - 2 : If electric potential at a point is zero then
the magnitude of electric field at that point must be zero.

RESPONSE GRID 22. 23. 24.

DAILY PRA CTICE PROBLEM SHEET 34 - PHYSICS


Total Questions 24 Total Marks 96
Attempted Correct
Incorrect N et Score
Cut-off Score 22 Qualifying Score 39
Success Gap = Net Score – Qualifying Score
Net Score = (Correct × 4) – (Incorrect × 1)

Space for Rough Work


t.me/Ebooks_Encyclopedia27. t.me/Magazines4all

DPP - Daily Practice Problems


Name : Date :

Start Time : End Time :

SYLLABUS : ELECTROSTATICS-4 (Capacitors, dielectrics)


35
Max. Marks : 96 Time : 60 min.
GENERAL INSTRUCTIONS
• The Daily Practice Problem Sheet contains 24 MCQ's. For each question only one option is correct. Darken the correct
circle/ bubble in the Response Grid provided on each page.
• You have to evaluate your Response Grids yourself with the help of solution booklet.
• Each correct answer will get you 4 marks and 1 mark shall be deduced for each incorrect answer. No mark will be given/
deducted if no bubble is filled. Keep a timer in front of you and stop immediately at the end of 60 min.
• The sheet follows a particular syllabus. Do not attempt the sheet before you have completed your preparation for that
syllabus. Refer syllabus sheet in the starting of the book for the syllabus of all the DPP sheets.
• After completing the sheet check your answers with the solution booklet and complete the Result Grid. Finally spend time
to analyse your performance and revise the areas which emerge out as weak in your evaluation.

DIRECTIONS (Q.1-Q.15) : There are 15 multiple choice e0 AV 2e0 AV e0 AV 2e0 AV


(a) , (b) ,
questions. Each question has 4 choices (a), (b), (c) and (d), out 2d 2d 2d 2d
of which ONLY ONE choice is correct. e 0 AV -2e0 AV
(c) ,
Q.1 A parallel plate capacitor is charged to a potential d d –
1 2 3 4 5
difference of 50V. It is discharged through a resistance. -e 0 AV -2e0 AV +
V

After 1 second, the potential difference between plates (d) ,


d d
becomes 40V. Then Q.3 Figure given below shows two identical parallel plate capacitors
(a) Fraction of stored energy after 1 second is 16/25 connected to a battery with switch S closed. The switch is now opened
(b) Potential difference between the plates after 2 and the free space between the plates of capacitors is filled with a
seconds will be 30V dielectric of dielectric constant 3. What will be the ratio of total
(c) Potential difference between the plates after 2 electrostatic energy stored in both capacitors before and after the
seconds will be 20V introduction of the dielectric?
(d) Fraction of stored energy after 1 second is 4/5 (a) 3:1
Q.2 Five identical plates each of area A are joined as shown in
(b) 5:1 V A B
the figure. The distance between the plates is d. The plates
are connected to a potential difference of V volts. The (c) 3:5
charge on plates 1 and 4 will be respectively (d) 5:3

RESPONSE GRID 1. 2. 3.
Space for Rough Work
t.me/Ebooks_Encyclopedia27. t.me/Magazines4all

EBD_7156
2 DPP/ P 35
Q.4 All six capacitors shown are identical, Each can withstand 2e 0 A
(a) ( K1 + K2 )
maximum 200 volts between its terminals. The maximum d
voltage that can be safely applied between A and B is 2e0 A æ K1 + K 2 ö
(a) 1200 V (b) ç ÷
d è K1 ´ K2 ø K1
(b) 400 V A B
d
(c) 800 V 4e0 A æ K1 ´ K 2 ö K2
(c) ç ÷
(d) 200 V d è K1 + K 2 ø
Q.5 A capacitor of capacity C1 is charged upto V volt and then 2e0 A æ K1 ´ K 2 ö
connected to an uncharged capacitor of capacity C2. Then (d) ç ÷
d è K1 + K 2 ø
final potential difference across each will be Q.10 Eight drops of mercury of equal radii possessing equal
C2V æ C2 ö C1V æ C2 ö charges combine to form a big drop. Then the capacitance
(a) C1 + C2
(b) çè 1 + C ÷ø V (c) C + C (d) çè 1 - C ÷ø V of bigger drop compared to each individual small drop is
1 1 2 1
Q.6 Two capacitors of capacitances 3mFand 6mFare charged to a (a) 8 times (b) 4 times
potential of 12V each. They are now connected to each other, (c) 2 times (d) 32 times
with the positive plate of each joined to the negative plate Q.11 Separation between the plates of a parallel plate capacitor
of the other. The potential difference across each will be is d and the area of each plate is A. When a slab of material
(a) 6 volt (b) 4 volt of dielectric constant k and thickness t(t < d) is introduced
(c) 3 volt (d) zero between the plates, its capacitance becomes
Q.7 In the figure a capacitor is filled with dielectrics K1, K2 e0 A e0 A
(a) (b) æ 1ö
and K3. The resultant capacitance is æ 1ö
d + t ç1 - ÷ d + t ç1 + ÷
è kø è kø
2e 0 A é 1 1 1 ù A/2 A/2
(a) ê + + ú (c)
e0 A
(d)
e0 A
d ë K1 K 2 K3 û d/2 K1 æ 1ö æ 1ö
d d - t ç1 - ÷ d - t ç1 + ÷
e0 A é 1 1 1 ù K3
è kø è kø
(b) ê + + ú K2
Q.12 There is an air filled 1pF parallel plate capacitor. When
d ë K1 K 2 K3 û
the plate separation is doubled and the space is filled with
2e 0 A
(c) [ K1 + K2 + K3 ] wax, the capacitance increases to 2pF. The dielectric
d
(d) None of these constant of wax is
Q.8 The resultant capacitance of given circut is (a) 2 (b) 4 (c) 6 (d) 8
Q.13 Between the plates of a parallel plate condenser, a plate of
(a) 3C P

2C
thickness t1 and dielectric constant k1 is placed. In the rest
(b) 2C 2C
of the space, there is another plate of thickness t 2 and
(c) C 2C
dielectric constant k2 . The potential difference across the
C
C C C condenser will be
(d)
3 Q
Q æ t1 t ö e 0Q æ t1 t ö
Q.9 Two dielectric slabs of constant K1 and K2 have been filled (a) ç + 2 ÷ (b) ç + 2 ÷
Ae0 è K1 K 2 ø A è K1 K 2 ø
in between the plates of a capacitor as shown below. What
will be the capacitance of the capacitor Q æ K1 K 2 ö e0 Q
(c) ç + ÷ (d) ( K1t1 + K 2 t2 )
Ae0 è t1 t2 ø A

RESPONSE 4. 5. 6. 7. 8.
GRID 9. 10. 11. 12. 13.

Space for Rough Work


t.me/Ebooks_Encyclopedia27. t.me/Magazines4all

DPP/ P 35 3
Q.14 A parallel plate capacitor is charged and the charging battery Q.18 The capacitance of a parallel plate condenser depends on
is then disconnected. If the plates of the capacitor are moved (1) Area of the plates
further apart by means of insulating handles, then (2) Medium between the plates
(a) The charge on the capacitor increases (3) Distance between the plates
(b) The voltage across the plates decreases (4) Metal of the plates
(c) The capacitance increases
(d) The electrostatic energy stored in the capacitor DIRECTIONS (Q.19-Q.21) : Read the passage given below and
increases answer the questions that follows :
Q.15 A parallel plate capacitor of plate area A and plates separation Capacitor C3 in the circuit is variable capacitor (its capacitance
distance d is charged by applying a potential V0 between the can be varied). Graph is plotted between potential difference V1
plates. The dielectric constant of the medium between the (across capacitor C1) versus C3.
plates is K. What is the uniform electric field E between the Electric potential V1 approaches on asymptote of 10 volts as
plates of the capacitor ? C3 ® ¥.
V1
CV0 V0
(a) E = Î0 (b) E= 10
KA Kd C1 8
6
V0 Kv 0 d V 4
C2 C3
(c) E= (d) E = 2
KA Î0 A C3
2 4 6 8
DIRECTIONS (Q.16-Q.18) : In the following questions, more
C1
than one of the answers given are correct. Select the correct Q.19 The ratio of the capacitance will be
answers and mark it according to the following codes: C2
(a) 2 / 3 (b) 4 / 3 (c) 3 / 4 (d) 3 / 2
Codes : Q.20 The value of C3 for which potential difference across C1
(a) 1, 2 and 3 are correct (b) 1 and 2 are correct will become 8V, is
(c) 2 and 4 are correct (d) 1 and 3 are correct (a) 1.5C1 (b) 2.5C1 (c) 3.5 C1 (d) 4.5 C1
Q.16 A parallel plate air condenser is connected with a battery. Q.21 The ratio of energy stored in capacitor C1 to that of total
Its charge, potenital difference, electric field and energy are energy when C3 ® ¥ is
Q0 , V0 , E0 , and U0 , respectively. In order to fill the complete (a) zero (b) 1/3
space between the plates a dielectric slab is inserted, the (c) 1 (d) Data insufficient
battery is still connected. Now the corresponding values Q,
V, E and U are in relation with the initially stated as DIRECTIONS (Q. 22-Q.24) : Each of these questions contains
(1) V > V0 (2) Q > Q0 (3) E > E0 (4) U > U0 two statements: Statement-1 (Assertion) and Statement-2
Q.17 The false statement are, on increasing the distance between (Reason). Each of these questions has four alternative choices,
the plates of a parallel plate condenser, only one of which is the correct answer. You have to select the
(1) The electric field intensity between the plates will correct choice.
decrease
(a) Statement-1 is True, Statement-2 is True; Statement-2 is a
(2) The electric field intensity between the plates will
correct explanation for Statement-1.
increase
(b) Statement-1 is True, Statement-2 is True; Statement-2 is
(3) The P. D. between the plates will decrease
(4) The electric field intensity between the plates will NOT a correct explanation for Statement-1.
remain unchanged (c) Statement-1 is False, Statement-2 is True.
(d) Statement-1 is True, Statement-2 is False.

RESPONSE 14. 15. 16. 17. 18.


GRID 19. 20. 21.

Space for Rough Work


t.me/Ebooks_Encyclopedia27. t.me/Magazines4all

EBD_7156
4 DPP/ P 35
Q.22 Statement-1 : The force with which one plate of a parallel Statement-2 : The capacitors are very delicate and so quickly
plate capacitor is attracted towards the other plate is equal break down.
to square of surface density per 2 Î0 per unit area. Q.24 Statement-1 : If the distance between parallel plates of a
Statement-2 : The electric field due to one charged plate capacitor is halved and dielectric constant is made three
of the capacitor at the location of the other is equal to times, then the capacitance becomes 6 times.
surface density per 2 Î0. Statement-1 : Capacitance of the capacitor does not
Q.23 Statement-1 : Circuit containing capacitors should be depend upon the nature of the material of plates.
handled cautiously even when there is no current.

RESPONSE GRID 22. 23. 24.

DAILY PRA CTICE PROBLEM SHEET 35 - PHYSICS


Total Questions 24 Total Marks 96
Attempted Correct
Incorrect N et Score
Cut-off Score 22 Qualifying Score 40
Success Gap = Net Score – Qualifying Score
Net Score = (Correct × 4) – (Incorrect × 1)

Space for Rough Work


t.me/Ebooks_Encyclopedia27. t.me/Magazines4all

DPP - Daily Practice Problems


Name : Date :

Start Time : End Time :

SYLLABUS : CURRENT ELECTRICITY – 1 (Electric Current, drift velocity, Ohm's law, Electrical resistance,
36
Resistances of different materials, V-I characteristics of Ohm and non-ohmic conductors, electrical energy and power,
Electrical resistivity, Colour code of resistors, Temperature dependance of resistance)
Max. Marks : 92 Time : 60 min.
GENERAL INSTRUCTIONS
• The Daily Practice Problem Sheet contains 23 MCQ's. For each question only one option is correct. Darken the correct
circle/ bubble in the Response Grid provided on each page.
• You have to evaluate your Response Grids yourself with the help of solution booklet.
• Each correct answer will get you 4 marks and 1 mark shall be deduced for each incorrect answer. No mark will be given/
deducted if no bubble is filled. Keep a timer in front of you and stop immediately at the end of 60 min.
• The sheet follows a particular syllabus. Do not attempt the sheet before you have completed your preparation for that
syllabus. Refer syllabus sheet in the starting of the book for the syllabus of all the DPP sheets.
• After completing the sheet check your answers with the solution booklet and complete the Result Grid. Finally spend time
to analyse your performance and revise the areas which emerge out as weak in your evaluation.
1W 1W 1W 1W
DIRECTIONS (Q.1-Q.14) : There are 14 multiple choice (a) 4 W A
questions. Each question has 4 choices (a), (b), (c) and (d), out (b) 8 W
2W 1W
of which ONLY ONE choice is correct. (c) 6 W 2W 2W

Q.1 In the following fig. the ratio of current in 3W and 1W (d) 2 W B


resistances is– Q.3 How will reading in the ammeter Switch
3W A be affected if an other
(a) 1/3 identical bulb Q is connected in Mains
3A 1W P Q
(b) 2/3 parallel to P as shown in the fig.
X Y Z
(c) 1 The voltage in the mains is
6W maintained at constant value A
(d) 2 (a) the reading will be reduced to one half.
Q.2 The resultant resistance between the points A and B in the (b) the reading will be double of previous one.
following fig. will be – (c) the reading will not be affected.
(d) the reading will increase four fold.
RESPONSE GRID 1. 2. 3.
Space for Rough Work
t.me/Ebooks_Encyclopedia27. t.me/Magazines4all

EBD_7156
2 DPP/ P 36
Q.4 In the circuit shown, the galvanometer G reads zero. If Q.10 In the fig. shown, Calculate the current through 3 ohm
batteries have negligible internal resistances, the value of resistor. The emf of battery is 2 volt and its internal
resistance X wil be – resistance is 2/3 ohm.
(a) 10 W
(b) 100 W
(c) 200 W
(d) 500 W
Q.5 A cylindrical wire is stretched to increase its length by
10%. The percentage increase in the resistance of the wire (a) 0.33 amp. (b) 0.44 amp.
will be– (c) 1.22 amp. (d) 0.88 amp.
(a) 20% (b) 21% (c) 22% (d) 24% Q.11 The current in the given circuit will be
Q.6 In the figure, the equivalent resistance between A and B is–
1
(a) 2R/3 (a) A
45 i
(b) R/3 1
(c) R (b) A 30 W
15 2V 30 W
(d) 3R 1
(c) A
Q.7 In the adjoining network of resistors, each is of resistance 10 30 W
r ohm, the equivalent resistance between points A and B is–
(a) 5r 1
(d) A
5
(b) 2r/3 Q.12 The equivalent resistance of the following infinite network
(c) r of resistance is
(d) r/2. 2W 2W 2W
Q.8 In the figure a carbon resistor has bands of different
2W 2W 2W
colours on its body as mentioned in the figure. The value
of the resistance is Silver
(a) 2.2 k W 2W 2W 2W
(b) 3.3 k W (a) Less than 4 W
(c) 5.6 k W (b) 4W
White
(d) 9.1 k W Brown Red (c) More than 4W but less than 12W
Q.9 Two wires of same material have length L and 2L and cross- (d) 12 W
sectional areas 4A and A respectively. The ratio of their Q.13 A heater coil connected to a supply of a 220 V is dissipating
specific resistance would be some power P1. The coil is cut into half and the two halves
(a) 1 : 2 (b) 8 : 1 (c) 1 : 8 (d) 1 : 1 are connected in parallel. The heater now dissipates a power
P2. The ratio of power P1 : P2 is
(a) 2 : 1 (b) 1 : 2 (c) 1 : 4 (d) 4 : 1

RESPONSE 4. 5. 6. 7. 8.
GRID 9. 10. 11. 12. 13.

Space for Rough Work


t.me/Ebooks_Encyclopedia27. t.me/Magazines4all

DPP/ P 36 3
Q.14 An electric lamp is marked 60 W, 230 V. The cost of a 1 (1) The potential difference across RL is 18 V
kWh of energy is ` 1.25. The cost of using this lamp 8 hrs (2) The current I through the battery is 7.5 mA
a day for 30 days is (approximately) (3) Ratio of powers dissipated in R1 and R2 is 3
(a) ` 10 (b) ` 16 (c) ` 18 (d) ` 20 (4) If R1 and R2 are interchanged magnitude of the power
dissipated in RL will decrease by a factor of 9
DIRECTIONS (Q.15-Q.17) : In the following questions,
more than one .of the answers given are correct. Select the DIRECTIONS (Q.18-Q.20) : Read the passage given below
correct answers and mark it according to the following and answer the questions that follows :
codes: In the circuit shown in the figure,
Codes : 1W 12 V
(a) 1, 2 and 3 are correct (b) 1 and 2 are correct
(c) 2 and 4 are correct (d) 1 and 3 are correct
Q.15 In the fig below the bulbs are identical, The bulbs, light
most brightly are
5W
E F Q.18 Rate of conversion of chemical energy within the battery
is
(a) 24 W (b) 20 W (c) 4 W (d) 14 W
A
B C Q.19 Rate of dissipation of electrical energy in battery is
(a) 24 W (b) 20 W (c) 4 W (d) 14 W
Q.20 Rate of dissipation of electrical energy in external resistor
P D Q
is
(a) 4 W (b) 20 W (c) 14 W (d) 24 W
(1) A (2) B (3) F (4) D DIRECTIONS (Q. 21-Q.23) : Each of these questions contains
Q.16 An electric kettle has two heating coils. When one of the two statements': Statement-1 (Assertion) and Statement-2
coils is switched on, the water begins to boil in 6 minutes. (Reason). Each of these questions has four alternative choices,
When the other is switched on, the boiling begins in 8 only one of which is the correct answer. You have to select the
minutes. The time when the boiling begin if both coils are correct choice.
switched on simultaneously is (i) in series (ii) in
parallel (a) Statement-1 is True, Statement-2 is True; Statement-2 is a
(1) 14 min in series (2) 3.43 min in parallel correct explanation for Statement-1.
(3) 3.43 min in series (4) 14 min in parallel (b) Statement-1 is True, Statement-2 is True; Statement-2 is
Q.17 For the circuit shown in the figure NOT a correct explanation for Statement-1.
(c) Statement -1 is False, Statement-2 is True.
(d) Statement -1 is True, Statement-2 is False.
Q.21 Statement-1 : The resistivity of a semiconductor decreases
with temperature.
Statement-2 : The atoms of a semiconductor vibrate with
larger amplitude at higher temperatures thereby increasing
its resistivity.

RESPONSE 14. 15. 16. 17. 18.


GRID 19. 20. 21.

Space for Rough Work


t.me/Ebooks_Encyclopedia27. t.me/Magazines4all

EBD_7156
4 DPP/ P 36
Q.22 Statement-1 : In a simple battery circuit the point of lowest Q.23 Statement-1 : The temperature coefficient of resistance is
potential is negative terminal of the battery. positive for metals and negative for p-type semiconductor.
Statement-2 : The current flows towards the point of the Statement-2 : The effective charge carriers in metals are
higher potential as it flows in such a circuit from the negatively charged whereas in p-type semiconductor they
negative to the positive terminal. are positively charged.

RESPONSE GRID 22. 23.

DAILY PRA CTICE PROBLEM SHEET 36 - PHYSICS


Total Questions 23 Total Marks 92
Attempted Correct
Incorrect N et Score
Cut-off Score 24 Qualifying Score 40
Success Gap = Net Score – Qualifying Score
Net Score = (Correct × 4) – (Incorrect × 1)

Space for Rough Work


t.me/Ebooks_Encyclopedia27. t.me/Magazines4all

DPP - Daily Practice Problems


Name : Date :

Start Time : End Time :

37
SYLLABUS : CURRENT ELECTRICITY – 2 Electrical cell and its internal resistance, Potential difference and E.M.F
of a cell, Combination of cells in series and in parallel, Kirchoff's laws and their applications, RC transient circuit,
Galvanometer, Ammeter, Voltmeter]
Max. Marks : 104 Time : 60 min.
GENERAL INSTRUCTIONS
• The Daily Practice Problem Sheet contains 26 MCQ's. For each question only one option is correct. Darken the correct
circle/ bubble in the Response Grid provided on each page.
• You have to evaluate your Response Grids yourself with the help of solution booklet.
• Each correct answer will get you 4 marks and 1 mark shall be deduced for each incorrect answer. No mark will be given/
deducted if no bubble is filled. Keep a timer in front of you and stop immediately at the end of 60 min.
• The sheet follows a particular syllabus. Do not attempt the sheet before you have completed your preparation for that
syllabus. Refer syllabus sheet in the starting of the book for the syllabus of all the DPP sheets.
• After completing the sheet check your answers with the solution booklet and complete the Result Grid. Finally spend time
to analyse your performance and revise the areas which emerge out as weak in your evaluation.

DIRECTIONS (Q.1-Q.18) : There are 18 multiple choice (a) G/10 (b) G/100 (c) G/99 (d) G/999
questions. Each question has 4 choices (a), (b), (c) and (d), out Q.3 The shunt required for 10% of main current to be sent
of which ONLY ONE choice is correct. through the moving coil galvanometer of resistance 99W
will be-
Q.1 The voltmeter shown in fig, reads 6V across the 60 W
(a) 0.9 W (b) 11 W (c) 90 W (d) 9.9 W
resistor. Then the resistance of the voltmeter is-
Q.4 The reading of voltmeter in the following circuit will be-
(a) 0 W
(b) ¥ W 20 W 80 W

(c) 200 W
V
(d) 300 W
®

80 W
Q.2 If only one hundredth part of total current flowing in the
+ –
circuit is to be passed through a galvanometer of resistance 2V
GW, Then the value of shunt resistance required will be- (a) 2 volt (b) 0.80 volt (c) 1.33 volt (d) 1.60 volt

RESPONSE GRID 1. 2. 3. 4.
Space for Rough Work
t.me/Ebooks_Encyclopedia27. t.me/Magazines4all

EBD_7156
2 DPP/ P 37
Q.5 The figure below shows currents in a part of electric circuit. Q.11 To get the maximum current from a parallel combination
The current i is 2 amp 1 amp of n identical cells each of internal resistance r in an
(a) 1.7 amp external resistance R,
(b) 3.7 amp 1.3 amp (a) R >> r (b) R << r (c) R > r (d) R = r
(c) 1.3 amp 2 amp Q.12 In the circuit shown below, if the value of R is increased
(d) 1 amp then what will be the effect on the reading of ammeter if
i
the internal resistance of cell is negligible-
Q.6 A voltmeter can measure upto 25 volt and its resistance is
1000 W. The resistance required to add with voltmeter to
measure upto 250 volt will be-
(a) 9000 W (b) 1000 W (c) 2500 W (d) 900 W
Q.7 When a Laclanche cell is connected to a 10W resistance
then a current of 0.25 ampere flows in the circuit. If the (a) The reading of ammeter will decrease
resistance is reduced to 4W then current becomes (b) The reading of ammeter will increase
0.5ampere. The internal resistance of galvanometer will be- (c) The reading of ammeter will remain unchanged
(a) 1.5 W (b) 0.5 W (c) 1 W (d) 2 W (d) The reading of ammeter will become zero.
Q.8 Consider the circuit shown in the figure. The value of Q.13 Twelve wires of equal length and same cross-section are
current I3 is connected in the form of a cube. If the resistance of each
28 W 54 W
(a) 5 A B of the wires is R, then the effective resistance between
the two diagonal ends would be
(b) 3 A (a) 2 R
6V
(c) – 3 A I3 (b) 12 R

(d) – 5/6 A 5
8V 12 V (c) R
6
Q.9 If VB – VA = 4 V in the given figure, then resistance X will
(d) 8 R
be
Q.14 The arrangement as shown in figure is called as
10 W 5V
(a) 5 W
(a) Potential divider ( )
(b) 10 W A B (b) Potential adder
Total P.D.
(c) 15 W
(c) Potential substracter
2V X
(d) 20 W
(d) Potential multiplier Variable P.D.
Q.10 In the given circuit the current I1 is
Q.15 When a cell of emf E and internal resistance r, is connected
(a) 0.4 A to the ends of a resistance R, then current through
30 W
I1 resistance is I. If the same cell is connected to the ends of
(b) – 0.4 A a resistance R/2 then the current would be-
I2
40 W 40 V I3 (a) less than I
(c) 0.8 A (b) I
40 W
(c) greater then I but less than 2I
(d) – 0.8 A 80 V
(d) greater than 2I

5. 6. 7. 8. 9.
RESPONSE
10. 11. 12. 13. 14.
GRID
15.

Space for Rough Work


t.me/Ebooks_Encyclopedia27. t.me/Magazines4all

DPP/ P 37 3
Q.16 The resistance of an ideal voltmeter is Q.21 A microammeter has a resistance of 100W and a full scale
(a) Zero (b) Very low range of 50 µA. It can be used as a voltmeter or a higher
(c) Very large (d) Infinite range ammeter provided a resistance is added to it. Pick
Q.17 An ammeter with internal resistance 90 W reads 1.85 A the correct range and resistance combination(s).
when connected in a circuit containing a battery and two (1) 10V range with 200 kW resistance in series.
resistors 700 W and 410 W in series. Actual current will (2) 50V range with 10 kW resistance in series.
be (3) 5 mA range with 1W resistance in parallel.
(a) 1.85 A (b) Greater than 1.85 A (4) 10 mA range with 1 kW resistance in parallel.
(c) Less than 1.85 A (d) None of these
Q.18 The figure shows a network of currents. The magnitude of DIRECTIONS (Q.22-Q.23) : Read the passage given below
currents is shown here. The current I will be and answer the questions that follows :
1A A 6V battery of negligible internal resistance is connected across
a uniform wire AB of length 100cm. The positive terminal of
another battery of emf 4V and internal resistance 1W is joined
10 A I to the point A as shown in figure. Take the potential at B to be
zero.
6A 6V

2A
A B
(a) 3 A (b) 9 A D C
(c) 13 A (d) 19 A
4V 1
DIRECTIONS (Q.19-Q.21) : In the following questions, Q.22 What are the potentials at points A and C ?
more than one of the answers given are correct. Select the (a) 6V, 2V (b) 8V, 4V
correct answers and mark it according to the following (c) 6V, 4V (d) 8V, 3V
codes: Q.23 If the points C and D are connected by a wire, what will be
Codes : the current through it ?
(a) 1, 2 and 3 are correct (b) 1 and 2 are correct (a) zero (b) 1A
(c) 2 and 4 are correct (d) 1 and 3 are correct (c) 2A (d) 3A
Q.19 In the figure,
DIRECTIONS (Qs. 24-Q.26) : Each of these questions contains
X E r Y two statements: Statement-1 (Assertion) and Statement-2
15 V 6V (Reason). Each of these questions has four alternative choices,
(1) current may flow from X to Y only one of which is the correct answer. You have to select the
(2) current may flow from Y to X correct choice.
(3) current’s direction depends on E (a) Statement-1 is True, Statement-2 is True; Statement-2 is a
(4) current’s direction depends on r correct explanation for Statement-1.
Q.20 Kirchoff’s laws are based on conservation of (b) Statement-1 is True, Statement-2 is True; Statement-2 is
(1) charge (2) potential NOT a correct explanation for Statement-1.
(3) energy (4) mass (c) Statement -1 is False, Statement-2 is True.
(d) Statement -1 is True, Statement-2 is False.

RESPONSE 16. 17. 18. 19. 20.


GRID 21. 22. 23.

Space for Rough Work


t.me/Ebooks_Encyclopedia27. t.me/Magazines4all

EBD_7156
4 DPP/ P 37
Q.24 Statement -1 : Voltameter measures current more Q.26 Statement - 1 : In the circuit shown, Vab or Va – Vb = 0,
accurately than ammeter. if
I = 2A.
Statement -2 : Relative error will be small if measured
from voltameter.
a 4V 2 b
Q.25 Statement - 1 : A larger dry cell has higher emf.
Statement - 2 : Potential difference across the
Statement - 2 : The emf of a dry cell is independent of its size. terminals of a non ideal battery is less than its emf when a
current flows through it.

RESPONSE GRID 24. 25. 26.

DAILY PRA CTICE PROBLEM SHEET 37 - PHYSICS


Total Questions 26 Total Marks 104
Attem pted Correct
Incorrect N et Score
Cut-off Score 26 Qualifying Score 42
Success Gap = Net Score – Qualifying Score
Net Score = (Correct × 4) – (Incorrect × 1)

Space for Rough Work


t.me/Ebooks_Encyclopedia27. t.me/Magazines4all

DPP - Daily Practice Problems


Name : Date :

Start Time : End Time :

SYLLABUS : CURRENT ELECTRICITY-3 : Wheatstone bridge, Meter bridge, Potentiometer-principle and its
38
applications.
Max. Marks : 100 Time : 60 min.
GENERAL INSTRUCTIONS
• The Daily Practice Problem Sheet contains 25 MCQ's. For each question only one option is correct. Darken the correct
circle/ bubble in the Response Grid provided on each page.
• You have to evaluate your Response Grids yourself with the help of solution booklet.
• Each correct answer will get you 4 marks and 1 mark shall be deduced for each incorrect answer. No mark will be given/
deducted if no bubble is filled. Keep a timer in front of you and stop immediately at the end of 60 min.
• The sheet follows a particular syllabus. Do not attempt the sheet before you have completed your preparation for that
syllabus. Refer syllabus sheet in the starting of the book for the syllabus of all the DPP sheets.
• After completing the sheet check your answers with the solution booklet and complete the Result Grid. Finally spend time
to analyse your performance and revise the areas which emerge out as weak in your evaluation.

DIRECTIONS (Q.1-Q.16) : There are 16 multiple choice Q.3 In a potentiometer experiment the balancing with a cell is
questions. Each question has 4 choices (a), (b), (c) and (d), out at length 240 cm. On shunting the cell with a resistance of
2 W, the balancing length becomes 120 cm. The internal
of which ONLY ONE choice is correct.
resistance of the cell is
Q.1 A Potentiometer wire of length 1 m is connected in series (a) 4 W (b) 2 W (c) 1 W (d) 0.5 W
with 490 W resistance and 2V battery. If 0.2 mV/cm is the Q.4 A potentiometer consists of a wire of length 4 m and
potential gradient, then resistance of the potentiometer resistance 10 W . It is connected to cell of emf 2 V. The
wire is potential difference per unit length of the wire will be
(a) 4.9 W (b) 7.9 W (c) 5.9 W (d) 6.9 W (a) 0.5 V/m (b) 10 V/m (c) 2 V/m (d) 5 V/m
Q.2 Two resistances are connected in two gaps of a metre Q.5 In given figure, the potentiometer wire AB has a resistance
bridge. The balance point is 20 cm from the zero end. A of 5 W and length 10 m. The balancing length AM for the
resistance of 15 ohms is connected in series with the emf of 0.4 V is
smaller of the two. The null point shifts to 40 cm. The (a) 0.4 m
(b) 4 m
value of the smaller resistance in ohm is
(c) 0.8 m
(a) 3 (b) 6 (c) 9 (d) 12 (d) 8 m

RESPONSE GRID 1. 2. 3. 4. 5.
Space for Rough Work
t.me/Ebooks_Encyclopedia27. t.me/Magazines4all

EBD_7156
2 DPP/ P 38
Q.6 In the circuit shown in the figure, the current flowing in (a) 33.3 cm (b) 66.67 cm (c) 25 cm (d) 50 cm
2W resistance Q.12 A potentiometer has uniform potential gradient across it.
10 W 2W
(a) 1.4 A Two cells connected in series (i) to support each other
1.4 A and (ii) to oppose each other are blanced over 6m and 2m
(b) 1.2 A G
respectively on the potentiometer wire. The e.m.f’s of the
(c) 0.4 A cells are in the ratio of
25 W 5W
(d) 1.0 A (a) 1 : 2 (b) 1 : 1 (c) 3 : 1 (d) 2 : 1
Q.7 For the post office box arrangement to determine the value Q.13 In a potentiometer experiment two cells of e.m.f E1 and E2
of unknown resistance the unknown resistance should be are used in series and in conjunction and the balancing
connected between length is found to be 58 cm of the wire. If the polarity of
E2 is reversed, then the balancing length becomes 29 cm.
E1
The ratio E of the e.m.f. of the two cells is
2
(a) 1:1 (b) 2 : 1 (c) 3 : 1 (d) 4 : 1
Q.14 The resistance of a 10 meter long potentiometer wire is
1 ohm/metre. A cell of e.m.f. 2.2 volts and a high resistance
box are connected in series with this wire. The value of
(a) B and C (b) C and D (c) A and D (d) B1and C1 resistance taken from resistance box for getting potential
Q.8 The e.m.f. of a standard cell balances across 150 cm length gradient of 2.2 millivolt/metre will be
of a wire of potentiometer. When a resistance of 2W is (a) 790 W (b) 810 W (c) 990 W (d) 1000 W
connected as a shunt with the cell, the balance point is Q.15 In the shown arrangement of the experiment of the meter
obtained at 100 cm. The internal resistance of the cell is bridge if AC correspondin g to null deflection of
(a) 0.1 W (b) 1W (c) 2W (d) 0.5W galvanometer is x, what would be its value if the radius of
Q.9 Five resistors are connected as shown in the diagram. The the wire AB is doubled
equivalent resistance between A and B is C
(a) 6 W 5W 4W

(b) 9 W 9W R1 R2
A B
(c) 12 W 10 W 8W
(d) 15 W D G
Q.10 A potentiometer has uniform potential gradient. The B
A x C
specific resistance of the material of the potentiometer
wire is 10–7 ohm-meter and the current passing through it (a) x (b) x / 4 (c) 4x (d) 2x
is 0.1 ampere; cross-section of the wire is 10–6m2. The Q.16 In meter bridge or Wheatstone bridge for measurement of
potential gradient along the potentiometer wire is resistance, the known and the unknown resistances are
(a) 10–4 V/m (b) 10–6 V/m interchanged. The error so removed is
–2
(c) 10 V/m (d) 10–8 V/m (a) End correction
Q.11 Resistance in the two gaps of a meter bridge are 10 ohm (b) Index error
and 30 ohm respectively. If the resistances are interchanged (c) Due to temperature effect
the balance point shifts by (d) Random error

6. 7. 8. 9. 10.
RESPONSE
11. 12. 13. 14. 15.
GRID
16.

Space for Rough Work


t.me/Ebooks_Encyclopedia27. t.me/Magazines4all

DPP/ P 38 3

DIRECTIONS (Q.17-Q.19) : In the following questions, DIRECTIONS (Q.20-Q.22) : Read the passage given below
more than one of the answers given are correct. Select the and answer the questions that follows :
correct answ]ers and mark it according to the following
codes: A battery is connected to a potentiometer and a balance point is
obtained at 84 cm along the wire. When its terminals are
Codes :
connected by a 5W resistor, the balance point changes to 70 cm
(a) 1, 2 and 3 are correct (b) 1 and 2 are correct
Q.20 Calculate the internal resistance of the cell.
(c) 2 and 4 are correct (d) 1 and 3 are correct
Q.17 Which of the following statements are correct? (a) 4 W (b) 2 W (c) 5 W (d) 1 W
(1) Voltmeter should have high resistance. Q.21 Find the new position of the balance point when 5W
(2) Ammeter should have low resistance. resistance is replaced by 4W resistor.
(3) Voltmeter is placed in parallel across the conductor (a) 26.5 cm (b) 52 cm (c) 67.2 cm (d) 83.3 cm
in a circuit. Q.22 How can we change a galvanometer with Re = 20.0W and
(4) Ammeter is placed in parallel across the conductor Ifs = 0.00100 A into a voltmeter with a maximum range of
in a circuit. 10.0 V?
Q.18 Which are correct statements? (a) By adding a resistance 9980 W in parallel with the
(1) The Wheatstone bridge is most sensitive when all the galvanometer
four resistances are of the same order (b) By adding a resistance 9980 W in series with the
(2) Kirchhoff’s first law (for currents meeting at a junction in galvanometer
an electric circuit) expresses the conservtion of charge. (c) By adding a resistance 8890 W in parallel with the
(3) The rheostat can be used as a potential divider. galvanometer
(4) In a balanced Wheatstone bridge, interchanging the (d) By adding a resistance 8890 W in series with the
positions of galvanometer and cell affects the balance galvanometer
of the bridge.
Q.19 Figure shows a balanced Wheatstone's bridge DIRECTIONS (Q. 23-Q.25) : Each of these questions contains
R = 5W Q = 50W two statements: Statement-1 (Assertion) and Statement-2
(Reason). Each of these questions has four alternative choices,
A C only one of which is the correct answer. You have to select the
S = 10W P = 100W correct choice.
(a) Statement-1 is True, Statement-2 is True; Statement-2 is a
correct explanation for Statement-1.
G (b) Statement-1 is True, Statement-2 is True; Statement-2 is
NOT a correct explanation for Statement-1.
(1) If P is slightly increased, the current in the
(c) Statement -1 is False, Statement-2 is True.
galvanometer flows from A to C.
(d) Statement -1 is True, Statement-2 is False.
(2) If P is slightly increased, the current in the
galvanometer flows from C to A. Q.23 Statement -1 : In meter bridge experiment, a high
(3) If Q is slightly increased, the current in the resistance is always connected in series with a
galvanometer flows from C to A. galvanometer.
(4) If Q is slightly increased, the current in the Statement -2 : As resistance increases current through the
galvanometer flows from A to C. circuit increases.

RESPONSE 17. 18. 19. 20. 21.


GRID 22. 23.

Space for Rough Work


t.me/Ebooks_Encyclopedia27. t.me/Magazines4all

EBD_7156
4 DPP/ P 38
Q.24 Statement -1: A potentiometer of longer length is used Q.25 Statement -1: The e.m.f. of the driver cell in the
for accurate measurement. potentiometer experiment should be greater than the e.m.f.
Statement -2: The potential gradient for a potentiometer of the cell to be determined.
of longer length with a given source of e.m.f. becomes Statement -2: The fall of potential across the
small. potentiometer wire should not be less than the e.m.f. of
the cell to be determined.

RESPONSE GRID 24. 25.

DAILY PRA CTICE PROBLEM SHEET 38 - PHYSICS


Total Questions 25 Total Marks 100
Attempted Correct
Incorrect N et Score
Cut-off Score 26 Qualifying Score 44
Success Gap = Net Score – Qualifying Score
Net Score = (Correct × 4) – (Incorrect × 1)

Space for Rough Work


t.me/Ebooks_Encyclopedia27. t.me/Magazines4all

DPP - Daily Practice Problems


Name : Date :

Start Time : End Time :

SYLLABUS : MAGNETIC EFFECTS OF CURRENT-1 (Magnetic field due to current carrying wires, Biot savart law)
39
Max. Marks : 108 Time : 60 min.
GENERAL INSTRUCTIONS
• The Daily Practice Problem Sheet contains 27 MCQ's. For each question only one option is correct. Darken the correct
circle/ bubble in the Response Grid provided on each page.
• You have to evaluate your Response Grids yourself with the help of solution booklet.
• Each correct answer will get you 4 marks and 1 mark shall be deduced for each incorrect answer. No mark will be given/
deducted if no bubble is filled. Keep a timer in front of you and stop immediately at the end of 60 min.
• The sheet follows a particular syllabus. Do not attempt the sheet before you have completed your preparation for that
syllabus. Refer syllabus sheet in the starting of the book for the syllabus of all the DPP sheets.
• After completing the sheet check your answers with the solution booklet and complete the Result Grid. Finally spend time
to analyse your performance and revise the areas which emerge out as weak in your evaluation.

DIRECTIONS (Q.1-Q.19) : There are 19 multiple choice Q.3 A current i is flowing in a conductor PQRST shaped as
questions. Each question has 4 choices (a), (b), (c) and (d), out shown in the figure. The radius of curved part QRS is r and
length of straight portions PQ and ST is very large. The
of which ONLY ONE choice is correct.
magnetic field at the centre O of the curved part is -
Q.1 The magnitude of magnetic field at a point having
perpendicular distance 50 mm from a long straight
conducting wire carrying a current of 3A is O Q i P
R y
r
(a) 0.12 G (b) 1.2 G 90° x
(c) 12 G (d) 0.012 G
S i T z
Q.2 A circular arc of wire of radius of curvature r subtends an
angle of p/4 radian at its centre. If i current is flowing in
µ0 i é 3 p ù ˆ µ0i é 3p ù ˆ
it then the magnetic induction at its centre is - (a) + 1ú k (b) - 1ú k
4 pr êë 2 û 4 pr êë 2 û
m0 i m i
(a) (b) 0
8r 4r µ0i é 3p ù ˆ µ0i é 3p ù ˆ
m0 i (c) + 1ú (- k) (d) - 1ú (- k)
(c) (d) 0 4pr êë 2 û 4pr êë 2 û
16r

RESPONSE GRID 1. 2. 3.
Space for Rough Work
t.me/Ebooks_Encyclopedia27. t.me/Magazines4all

EBD_7156
2 DPP/ P 39
Q.4 Consider the loop PQRSP, carrying clockwise current i, shown Q.9 A current of 1.0 ampere is flowing in the sides of an equilateral
in the figure. The magnitude of magnetic field at the centre O of triangle of side 4.5 × 10–2 m. Find the magnetic field at the
the curved portion is centroid of the triangle.
(Permeability constant m0 = 4p × 10–7 V-s/A-m).
m0 i (a) 4.0 × 10–5 weber/m2 (b) 6.0 × 10–8 weber/m2
(a) [p – f + tan f] S
2p r (c) 2.0 × 10–5 weber/m2 (d) 7.0 × 10–12 weber/m2
Q.10 An air-solenoid has 500 turns of wire in its 40 cm length.
m0 i O
(b) i i If the current in the wire be 1.0 ampere then the magnetic
2p r r field on the axis inside the solenoid is -

(
(c) 0 f f (a) 15.7 gauss (b) 1.57 gauss
P Q R (c) 0.157 gauss (d) 0.0157 gauss
m0 i Q.11 A solenoid of length 0.2m has 500 turns on it. If
(d) [p – f + tan f]
2p r 8.71 × 10–6 Weber/m2 be the magnetic field at an end of
the solenoid, then the current flowing in the solenoid is –
Q.5 A circular coil of 0.2 m diameter has 100 turns and carries a
current of 0.1 ampere. The intensity of magnetic field at the 0.174 0.0174 17.4 174
(a) A (b) A (c) A (d) A
centre of the coil is - p p p p
Q.12 A circular current carrying coil has a radius R. The distance
(a) 6.28 × 10–4 N/A.m (b) 62.8 × 10–4 N/A.m
from the centre of the coil on the axis where the magnetic
(c) 6.28 × 10 N/A.m
–5 (d) 62.8 × 10–5 N/A.m
1
Q.6 For the arrangement of two current carrying identical coils induction will be th to its value at the centre of the coil,
shown in the figure, the magnetic field at the center O is 8
is
(N and a represent number of turns and radius of each coil)- R 2
(a) (b) R 3 (c) 2 3R (d) R
3 3
y Q.13 The average radius of an air cored made toroid is 0.1 m
Coil-2 and it has 500 turns. If it carries 0.5 ampere current, then
x
the magnetic field inside it is :
I z
(a) 5 × 10–4 tesla (b) 5 × 10–3 tesla
I (c) 5 × 10 tesla
–2 (d) 2 × 10–3 tesla
Coil-1 Q.14 The straight long conductors AOB and COD are
m0 NI m 0 NI m 0 NI m NI perpendicular to each other and carry current i1 and i2. The
(a) (b) (c) (d) 0
2a 2 2a 2 2a magnitude of the magnetic induction at point P at a distance
Q.7 A current is flowing through a conducting hollow pipe a from the point O in a direction perpendicular to the plane
whose area of cross-section is shown in the fig. The value ACBD is B1
of magnetic induction will be zero at- μ0
•R (a) ( i +
1 2 i ) P B2
(a) Point P, Q and R 2πa
μ0 a
(b) Point R but not at P and Q •P (b) ( i1 - i2 )
(c) Point Q but not at P and R 2πa A i2

(
μ 0 2 2 1/ 2
)
•Q i1
(d) Point P but not at Q and R (c) i1 + i2 C D
Q.8 Dimensional formula of m0 is- 2πa B O
μ 0 i1i2
(a) MLT–2 A–2 (b) MLT–2A–2 (d) 2πa ( i + i )
1 2
(c) MLT A–2 2 (d) MLT2 A2

4. 5. 6. 7. 8.
RESPONSE
9. 10. 11. 12. 13.
GRID
14.

Space for Rough Work


t.me/Ebooks_Encyclopedia27. t.me/Magazines4all

DPP/ P 39 3
Q.15 A conducting circular loop of radius r carries a constant m 0 (I1 - I2 )
ur (1) into the plane of paper (If I1 > I2)
current i. It is placed in a uniform magnetic field B , such pR
ur (2)
m 0 (I 2 - I1 )
out of the plane of paper (if I2 > I1)
that B is perpendicular to the plane of the loop. The pR
magnetic force acting on the loop is m 0 (I1 - I 2 )
ur ur ur (3) out of the plane of paper (if I2 > I1)
(a) ir B (b) 2pri B (c) zero (d) pri B pR 2
Q.16 The radius of a circular loop is r and a current i is flowing m 0 (I 2 - I1 )
(4) into the plane of paper (if I1 > I2)
in it. The equivalent magnetic moment will be pR 2
1 Q.21 A wire of length L carrying current I is bent into a circle
(a) ir (b) 2pir (c) ipr 2 (d) of one turn. The field at the center of the coil is B1. A
r2 similar wire of length L carrying current I is bent into a
Q.17 A current of 30 A is flowing in a vertical straight wire. If square of one turn. The field at its center is B2. Then
the horizontal component of earth's magnetic field is 2 × B1
10–5 tesla then the distance of null point from wire is - (1) B1 > B2 (2) B1 = B2 (3) =2 (4) B1 < B2
B2
(a) 0.9 m (b) 0.3 mm (c) 0.3 cm (d) 0.3 m
Q.18 A charged particle is released from rest in a region of steady DIRECTIONS (Q.22-Q.24) : Read the passage given below
uniform electric and magnetic fields which are parallel to and answer the questions that follows :
each other. The particle will move in a A conducting wire is bent into a loop as shown in the figure. The
(a) Straight line (b) Circle segment AOB is parabolic given by the equation y2 = 2x while
(c) Helix (d) Cycloid segment BA is a straight line parallel to the y-axis.
Q.19 A 6.28m long wire is turned into a coil of diameter 0.2m ur
and a current of 1 amp is passed in it. The magnetic The magnetic field in the region is B = –8k$ and the current in
induction at its centre will be - the wire is 2A.
(a) 6.28 × 10–5 T (b) 0 T y
(c) 6.28 T (d) 6.28 × 10–3 T A
DIRECTIONS (Q.20-Q.21) : In the following questions, O C x
more than one of the answers given are correct. Select the 2m D
correct answers and mark it according to the following
B
codes:
Codes : Q.22 The torque on the loop will be
(a) 1, 2 and 3 are correct (b) 1 and 2 are correct
(a) 16 2 Nm (b) 16 Nm
(c) 2 and 4 are correct (d) 1 and 3 are correct
(c) 18 2 Nm (d) Zero
Q.20 Two long straight parallel wires carry Q.23 The field created by the current in the loop at point C will be
currents I1 and I2 respectively, in the same I
µ µ0 $
(a) – 0 k$
1 I2
direction (as shown). The distance (b) – k
between the wires is R. The magnetic 4p 2p
field at the centre of the two wires will µ0 2 $
be- R (c) – k (d) None of these
p

RESPONSE 15. 16. 17. 18. 19.


GRID 20. 21. 22. 23.

Space for Rough Work


t.me/Ebooks_Encyclopedia27. t.me/Magazines4all

EBD_7156
4 DPP/ P 39
Q.24 Magnetic field at point D due to segment AO of the loop is (b) Statement-1 is True, Statement-2 is True; Statement-2 is NOT
directed parallel to a correct explanation for Statement-1.
(c) Statement-1 is False, Statement-2 is True.
(a) k̂ (b) - k̂ (c) î (d) ĵ (d) Statement -1 is True, Statement-2 is False.
DIRECTIONS (Q. 25-Q.27) : Each of these questions contains Q.25 Statement -1: Cyclotron does not accelerate electron.
two statements: Statement-1 (Assertion) and Statement-2 Statement-2: Mass of the electron is very small.
(Reason). Each of these questions has four alternative choices, Q.26 Statement-1: The ion cannot move with a speed beyond a
only one of which is] the correct answer. You have to select the certain limit in a cyclotron.
correct choice. Statement-2: As velocity increases time taken by ion
increases.
(a) Statement-1 is True, Statement-2 is True; Statement-2 is a Q.27 Statement-1: If an electron, while coming vertically from
correct explanation for Statement-1. outerspace, enter the earth's magnetic field, it is deflected
towards west.
Statement-2: Electron has negative charge.

RESPONSE GRID 24. 25. 26. 27.

DAILY PRA CTICE PROBLEM SHEET 39 - PHYSICS


Total Questions 27 Total Marks 108
Attempted Correct
Incorrect N et Score
Cut-off Score 28 Qualifying Score 44
Success Gap = Net Score – Qualifying Score
Net Score = (Correct × 4) – (Incorrect × 1)

Space for Rough Work


t.me/Ebooks_Encyclopedia27. t.me/Magazines4all

DPP - Daily Practice Problems


Name : Date :

Start Time : End Time :

SYLLABUS : MAGNETIC EFFECTS OF CURRENT-2 :


40
(Motion of charge particle in a magnetic field, force between current carrying wires.)

Max. Marks : 104 Time : 60 min.


GENERAL INSTRUCTIONS
• The Daily Practice Problem Sheet contains 26 MCQ's. For each question only one option is correct. Darken the correct
circle/ bubble in the Response Grid provided on each page.
• You have to evaluate your Response Grids yourself with the help of solution booklet.
• Each correct answer will get you 4 marks and 1 mark shall be deduced for each incorrect answer. No mark will be given/
deducted if no bubble is filled. Keep a timer in front of you and stop immediately at the end of 60 min.
• The sheet follows a particular syllabus. Do not attempt the sheet before you have completed your preparation for that
syllabus. Refer syllabus sheet in the starting of the book for the syllabus of all the DPP sheets.
• After completing the sheet check your answers with the solution booklet and complete the Result Grid. Finally spend time
to analyse your performance and revise the areas which emerge out as weak in your evaluation.

DIRECTIONS (Q.1-Q.18) : There are 18 multiple choice Q.3 An electron is revolving in a circular path of radius
questions. Each question has 4 choices (a), (b), (c) and (d), out 2×10–10 m with a speed of 3×106 m/s. The magnetic field
at the centre of circular path will be-
of which ONLY ONE choice is correct.
(a) 1.2 T (b) 2.4 T (c) 0 (d) 3.6 T
Q.1 A proton, a deutron and an a-particle are accelerated Q.4 An a particle travels at an angle of 30º to a magnetic field
through same potential difference and then they enter a 0.8 T with a velocity of 105 m/s. The magnitude of force
normal uniform magnetic field. The ratio of their kinetic will be-
energies will be- (a) 1.28 × 10–14 N (b) (1.28)Ö3 ×10–4 N
(c) 1.28 × 10 N –4 (d) (12.8)Ö3 ×10–4 N
(a) 2 : 1 : 3 (b) 1 : 1 : 2 (c) 1 : 1 : 1 (d) 1 : 2 : 4
Q.2 A proton of energy 8eV is moving in a circular path in a Q.5 A beam of protons is moving horizontally towards you. As
uniform magnetic field. The energy of an a-particle it approaches, it passes through a magnetic field directed
moving in the same magnetic field and along the same path downward. The beam deflects- × × × ×
will be- (a) to your left side v
(a) 4eV (b) 2eV (b) to your right side × × × F
(c) 8eV (d) 6eV (c) does not deflect × × × ×
(d) nothing can be said × × × ×

RESPONSE GRID 1. 2. 3. 4. 5.
Space for Rough Work
t.me/Ebooks_Encyclopedia27. t.me/Magazines4all

EBD_7156
2 DPP/ P 40
Q.6 If a particle moves in a circular path in clockwise direction Bp
after entering into a downward vertical magnetic field. The (c) sin q =
qd
charge on the particle is-
(a) positive (b) negative pd
(d) sin q =
(c) nothing can be said (d) neutral Bq
Q.7 In the example above, after how much time, particle comes Q.11 An a particle is moving in a magnetic field of (3iˆ + 2ˆj)
to the starting point for the first time. (mass of particle =
m) tesla with a velocity of 5×105 $i m/s. The magnetic force
acting on the particle will be-
2p m
(a) (a) 3.2 × 10–13 dyne (b) 3.2 × 1013 N
3qB (c) 0 (d) 3.2 × 10–13 N
2p m Q.12 If an a-particle moving with velocity 'v' enters perpendicular
(b) to a magnetic field then the magnetic force acting on it will
qB
(c) Never be-
(d) It will leave the circular path before coming to the (a) evB (b) 2evB (c) 0 (d) 4evB
starting point Q.13 What is the net force on the square coil ?
Q.8 A current of 2.0 amp is flowing through a wire of length 10 cm
50 cm. If this wire be placed at an angle of 60º with the
direction of a uniform magnetic field of 5.0 × 10–4 N/Am 2A
the force on the wire will be- 1A 15 cm
(a) 4.33 × 10–4 N (b) 2.50 × 10–4 N
(c) 5.0 × 10 N –4 (d) 2.33 × 10–4 N
Q.9 A particle of mass m and charge q moves with a constant 2 cm
velocity v along the positive x direction. It enters a region (a) 25 ´ 10–7 N towards wire
containing a uniform magnetic field B directed along the (b) 25 ´ 10–7 N away from wire
negative z direction, extending from x = a to x = b. The (c) 35 ´ 10–7 N towards wire
minimum value of v required so that the particle can just (d) 35 ´ 10–7 N away from wire
enter the region x < b is Q.14 A proton is to circulate the earth along the equator with a
(a) qb B/m (b) q(b – a) B/m speed of 1.0 × 107 m/s. The minimum magnetic field which
(c) qa B/m (d) q(b + a) B/2m should be created at the equator for this purpose.
Q.10 A particle with charge q, moving with a momentum p, enters (The mass of proton = 1.7 × 10–27 kg and radius of earth
a uniform magnetic field normally. The magnetic field has = 6.37 × 106 m.) will be (in Wb/m2)
magnitude B and is confined to a region of width d, where (a) 1.6 × 10–19 (b) 1.67 × 10–8
(c) 1.0 × 10 –7 (d) 2 × 10–7
p
d < Bq , The particle is deflected by an angle q in crossing Q.15 An a-particle is describing a circle of radius 0.45 m in a
the field. Then × × × × field of magnetic induction 1.2 weber/m2. The potential
Bqd × × × × difference required to accelerate the particle, (The mass
(a) sin q = p B of a-particle is 6.8 × 10–27 kg and its charge is 3.2 × 10–
p × × × × 19 coulomb.) will be —
q
p × × × ×
(b) sin q = d (a) 6 × 106 V (b) 2.3 × 10–12 V
Bqd 6 (d) 3.2 × 10–12 V
× × × × (c) 7 × 10 V

RESPONSE 6. 7. 8. 9. 10.
GRID 11. 12. 13. 14. 15.

Space for Rough Work


t.me/Ebooks_Encyclopedia27. t.me/Magazines4all

DPP/ P 40 3
Q.16 An electron beam passes through a magnitic field of DIRECTIONS (Q.21-Q.23) : Read the passage given below
2 × 10–3 weber/m2 and an electric field of 1.0 × 104 volt/ and answer the questions that follows :
m both acting simultaneously. If the electric field is
removed, what will be the radius of the electron path ? r
A charge particles q enters in a magnetic field B = yiˆ + xjˆ with
(a) 1.43 cm. (b) 0.43 cm (c) 2.43 cm. (d) 3.43 cm. r
Q.17 A straight horizontal copper wire carries a current i = 30 the velocity v = xiˆ + yjˆ . Neglect any force other than magnetic
A. The linear mass density of the wire is 45 g/m. What is force. Now answer the following question.
the magnitude of the magnetic field needed to balance its Q.21 When particle arrives at any point P (2, 2) then force acting
weight? on it, will be –
(a) 147 G (b) 441 G (c) 14.7 G (d) 0 G (a) Zero (b) 4 2q (c) 8q (d) 2 2q
Q.18 A 1m long conducting wire is lying at right angles to the
Q.22 Magnetic force F acting on charge is proportional to –
magnetic field. A force of 1 kg. wt is acting on it in a
(a) F µ (x2 – y2)
magnetic field of 0.98 tesla. The current flowing in it will
(b) F µ (x2 + y2)
be-
(a) 100 A (b) 10 A (c) 1 A (d) 0 (c) Fµ x 2 + y2
DIRECTIONS (Q.19-Q.20) : In the following questions, (d) F does not depend on x or y co-ordinate
more than one of the answers given are correct. Select the Q.23 Which of the following is true for the direction of magnetic
correct answers and mark it according to the following force ?
codes: (a) if x > y then force works along (–z) direction
(b) if x < y then force works along (+z) direction
Codes :
(c) if x > y then force works along (+z) direction
(a) 1, 2 and 3 are correct (b) 1 and 2 are correct
(d) None of these
(c) 2 and 4 are correct (d) 1 and 3 are correct
Q.19 In the fig the two parallel wires PQ and ST are at 30 cm DIRECTIONS (Qs. 24-Q.26) : Each of these questions contains
apart. The currents flowing in the wires are according to two statements: Statement-1 (Assertion) and Statement-2
fig. The force acting over a length of 5m of the wires is- (Reason). Each of these questions has four alternative choices,
P 10A Q only one of which is the correct answer. You have to select the
correct choice.

S 15A T (a) Statement-1 is True, Statement-2 is True; Statement-2 is a


(1) 5 × 10–4 N (2) attraction correct explanation for Statement-1.
(3) 5 × 10–8 N (4) repulsion (b) Statement-1 is True, Statement-2 is True; Statement-2 is
Q.20 A beam of protons enters a uniform magnetic field of 0.3 NOT a correct explanation for Statement-1.
tesla with a velocity of 4 × 105 m/s at an angle of 60º to (c) Statement -1 is False, Statement-2 is True.
the field. Then, (d) Statement -1 is True, Statement-2 is False.
(Mass of the proton = 1.7 × 10–27 kg.) Q.24 Statement -1 : If two long wires, hanging freely are
(1) the radius of the helical path is 1.226 × 10–2 m connected to a battery in series, they come closer to each
(2) the pitch of the helix is 4.45 × 10–2 m other.
(3) the radius of the helical path is 1.226 × 10–3 m Statement -2 : Force of repulsion acts between the two
(4) the pitch of the helix is 4.45 × 10–4 m wires carrying current.

RESPONSE 16. 17. 18. 19. 20.


GRID 21. 22. 23. 24.

Space for Rough Work


t.me/Ebooks_Encyclopedia27. t.me/Magazines4all

EBD_7156
4 DPP/ P 40
Q.25 Statement - 1 : For a charged particle to pass through a Q.26 Statement - 1 : If an electron is not deflected while passing
uniform electro-magnetic field without change in velocity, through a certain region of space, then only possibility is
its velocity vector must be perpendicular to the magnetic that there is no magnetic region.
field. Statement - 2 : Magnetic force is directly proportional to
Statement - 2 : Net Lorentz force on the particle is given the magnetic field applied.
r r r r
by F = q [ E + v ´ B] .

RESPONSE GRID 25. 26.

DAILY PRA CTICE PROBLEM SHEET 40 - PHYSICS


Total Questions 26 Total Marks 104
Attempted Correct
Incorrect N et Score
Cut-off Score 26 Qualifying Score 46
Success Gap = Net Score – Qualifying Score
Net Score = (Correct × 4) – (Incorrect × 1)

Space for Rough Work


t.me/Ebooks_Encyclopedia27. t.me/Magazines4all

DPP - Daily Practice Problems


Name : Date :

Start Time : End Time :

SYLLABUS : MAGNETIC EFFECTS OF CURRENT-3 (Magnetic dipole, Current carrying loop in


41
magnetic field,Galvanometer )
Max. Marks : 116 Time : 60 min.
GENERAL INSTRUCTIONS
• The Daily Practice Problem Sheet contains 29 MCQ's. For each question only one option is correct. Darken the correct
circle/ bubble in the Response Grid provided on each page.
• You have to evaluate your Response Grids yourself with the help of solution booklet.
• Each correct answer will get you 4 marks and 1 mark shall be deduced for each incorrect answer. No mark will be given/
deducted if no bubble is filled. Keep a timer in front of you and stop immediately at the end of 60 min.
• The sheet follows a particular syllabus. Do not attempt the sheet before you have completed your preparation for that
syllabus. Refer syllabus sheet in the starting of the book for the syllabus of all the DPP sheets.
• After completing the sheet check your answers with the solution booklet and complete the Result Grid. Finally spend time
to analyse your performance and revise the areas which emerge out as weak in your evaluation.

DIRECTIONS (Q.1-Q.20) : There are 20 multiple choice questions. Q.3 The deflection in a moving coil galvanometer is
Each question has 4 choices (a), (b), (c) and (d), out of which ONLY (a) directly proportional to the torsional constant
ONE choice is correct. (b) directly proportional to the number of turns in the coil
Q.1 A circular coil has radius 4 cm and 20 number of turns (c) inversely proportional to the area of the coil
carries a current of 3 ampere. It is placed in a magnetic (d) inversely proportional to the current flowing
field of intensity 0.5 weber/m2. The magnetic dipole Q.4 A moving coil galvanometer has N number of turns in a
coil of effective area A. It carries a current I. The magnetic
æ 22 ö field B is radial. The torque acting on the coil is
moment of the coil is çè Take p = ÷ø
7 (a) NA2 B 2 I (b) NABI 2 (c) N 2 ABI (d) NABI
(a) 0.15 ampere m2 (b) 0.3 ampere m2 Q.5 A current carrying loop is free to turn in a uniform magnetic
(c) 0.45 ampere m2 (d) 0.6 ampere m2 field. The loop will then come into equilibrium when its
Q.2 A circular coil of radius 4 cm has 50 turns. In this coil a plane is inclined at
current of 2 A is flowing. It is placed in a magnetic field of (a) 0° to the direction of the field
0.1 weber/m2. The amount of work done in rotating it (b) 45° to the direction of the field
through 180° from its equilibrium position will be (c) 90° to the direction of the field
(a) 0.1 J (b) 0.2 J (c) 0.4 J (d) 0.8 J (d) 135° to the direction of the field

RESPONSE GRID 1. 2. 3. 4. 5.
Space for Rough Work
t.me/Ebooks_Encyclopedia27. t.me/Magazines4all

EBD_7156
2 DPP/ P 41
Q.6 A 100 turns coil shown in figure carries a current of 2 amp
LiB 2 Li 2 B L2 iB Li 2 B
in a magnetic field B = 0.2Wb/m2. The torque acting on (a) (b) (c) (d)
the coil is 2 2 4p 4p
Q.12 In ballistic galvanometer, the frame on which the coil is
A B wound is non-metallic. It is
(a) to avoid the production of induced e.m.f.

10 cm
N S (b) to avoid the production of eddy currents
(c) to increase the production of eddy currents
D C (d) to increase the production of induced e.m.f.
8 cm Q.13 A solenoid of length 0.4 m and having 500 turns of wire
(a) 0.32 Nm tending to rotate the side AD out of the page carries a current of 3 amp. A thin coil having 10 turns of wire
(b) 0.32 Nm tending to rotate the side AD into the page and of radius 0.01 m carries a current of 0.4 amp. The torque
(c) 0.0032 Nm tending to rotate the side AD out of the (in Nm) required to hold the coil in the middle of the solenoid
page with its axis perpendicular to the axis of the solenoid is
(d) 0.0032 Nm tending to rotate the side AD into the page (m0 = 4p × 10–7 V-s/A-m)
Q.7 A rectangular coil of size 20 cm × 20 cm has 100 turns
(a) 59.2 × 10–6 (b) 5.92 × 10–6
and carries a current of 1 A. It is placed in a uniform
magnetic field B = 0.5 T with the direction of magnetic (c) 0.592 × 10–6 (d) 0.592 × 10–4.
field parallel to the plane of the coil. The magnitude of the Q.14 If an electron is moving with velocity v in an orbit of radius
torque required to hold this coil in this position is r in a hydrogen atom, then the equivalent magnetic moment is
(a) zero (b) 200 Nm µ0 e ev ev´ 10,7 evr
(c) 2 Nm (d) 10 Nm (a) (b) 2 (c) 3 (d)
Q.8 A circular loop of area 0.01m2 carrying a current of 10 A, 2r r r 2
is held perpendicular to a magnetic field of intensity 0.1 Q.15 In a moving coil galvanometer, the deflection of the coil q is
T. The torque acting on the loop is related to the electrical current i by the relation
(a) zero (b) 0.01 Nm (a) i µ tan q (b) i µ q
(c) 0.001 Nm (d) 0.8 Nm (c) i µ q2 (d) i µ q
Q.9 The magnetic moment of a current carrying circular coil is
Q.16 A thin circular wire carrying a current I has a magnetic
(a) directly proportional to the length of the wire
(b) inversely proportional to the length of the wire moment M. The shape of the wire is changed to a square
(c) directly proportional to the square of the length of and it carries the same current. It will have a magnetic
the wire moment
(d) inversely proportional to the square of the length of 4 4 p
the wire (a) M (b) 2 M (c) M (d) M
p p 4
Q.10 What is the shape of magnet in moving coil galvanometer Q.17 A ring of radius R, made of an insulating material carries a
to make the radial magnetic field? charge Q uniformly distributed on it. If the ring rotates about
(a) Concave cylindrical (b) Horse shoe magnet the axis passing through its centre and normal to plane of
(c) Convex cylindrical (d) None of these the ring with constant angular speed w, then the magnitude
Q.11 Current i is carried in a wire of length L. If the wire is of the magnetic moment of the ring is
turned into a circular coil, the maximum magnitude of
torque in a given magnetic field B will be 1 1
(a) QwR 2 (b) QwR 2 (c) Qw 2 R (d) Qw 2 R
2 2

6. 7. 8. 9. 10.
RESPONSE
11. 12. 13. 14. 15.
GRID
16. 17.

Space for Rough Work


t.me/Ebooks_Encyclopedia27. t.me/Magazines4all

DPP/ P 41 3
Q.18 The (t - q) graph for a current carrying coil placed in a
uniform magnetic field is

P Q R S

(a) (b) (1) Couple on loop P will be the highest


(2) Couple on loop Q will be the highest
0° q 0° q (3) Couple on loop R will be the highest
90° 180° 90° 180°
(4) Couple on loop S will be the highest
Q.22 The sensitivity of a moving coil galvanometer can be
increased by
(1) decreasing the couple per unit twist of the suspension
(c) (d) (2) increasing the number of turns in the coil
q q
(3) decreasing the area of the coil
0° 90° 180° 0° 90° 180° (4) decreasing the magnetic field
Q.19 A rectangular loop carrying a current i is placed in a Q.23 A current carrying rectangular coil is placed in a uniform
uniform magnetic field B. The area enclosed by the loop magnetic field. In which orientation, the coil will tend to
is A. If there are n turns in the loop, the torque acting on rotate
the loop is given by (1) The magnetic field is parallel to the plane of the coil
r r r r
(a) ni A ´ B (b) ni A.B (2) The magnetic field is at 45° with the plane of the coil
1 r r 1 r r (3) In any orientation
(c) (i A ´ B) (d) (iA.B) (4) The magnetic field is perpendicular to the plane of
n n
the coil
Q.20 The pole pieces of the magnet used in a pivoted coil
galvanometer are DIRECTIONS (Q.24-Q.26) : Read the passage given below
(a) plane surfaces of a bar magnet and answer the questions that follows :
(b) plane surfaces of a horse-shoe magnet
y
(c) cylindrical surfaces of a bar magnet A wire carrying a 10 A current is
bent to pass through sides of a cube 2
(d) cylindrical surfaces of a horse-shoe magnet
of side 10 cm as shown in figure. A 1
DIRECTIONS (Q.21-Q.23) : In the following questions, r 3
more than one of the answers given are correct. Select the magnetic field B = (2iˆ - 3jˆ + k)T
ˆ is
x
present in the region. Then, find 4
correct answers and mark it according to the following z
codes: Q.24 The net force on the loop
r r
Codes : (a) Fnet = 0 (b) Fnet = (0.1iˆ - 0.2k)Nˆ
r r
(a) 1, 2 and 3 are correct (b) 1 and 2 are correct (c) Fnet = (0.3iˆ + 0.4k)N ˆ (d) Fnet = (0.36k)N ˆ
(c) 2 and 4 are correct (d) 1 and 3 are correct Q.25 The magnetic moment vector of the loop.
Q.21 Four wires each of length 2.0 metre are bent into four loops
(a) (0.1iˆ + 0.05jˆ - 0.05k)Am
ˆ 2 (b)
(0.1iˆ + 0.05jˆ + 0.05k)Am
ˆ 2
P, Q, R and S and then suspended into uniform magnetic
field. Same current is passed in each loop. Which (c) (0.1iˆ - 0.05jˆ + 0.05k)Am
ˆ 2 (d)
(0.1iˆ - 0.05jˆ - 0.05k)Am
ˆ 2

statements are incorrect?

RESPONSE 18. 19. 20. 21. 22.


GRID 23. 24. 25.

Space for Rough Work


t.me/Ebooks_Encyclopedia27. t.me/Magazines4all

EBD_7156
4 DPP/ P 41
Q.26 The net torque on the loop. (c) Statement-1 is False, Statement-2 is True.
(d) Statement-1 is True, Statement-2 is False.
(a) - 0.1iˆ + 0.4kˆ Nm (b) -0.1iˆ - 0.4kˆ Nm
Q.27 Statement-1 : The coil is bound over the metallic frame
(c) 0.1iˆ - 0.4kˆ Nm (d) 0.1iˆ - 0.4kˆ Nm in moving coil galvanometer.
Statement-2 : The metallic frame help in making steady
DIRECTIONS (Q. 27-Q.29) : Each of these questions contains deflection without any oscillation.
two statements: Statement-1 (Assertion) and Statement-2 Q.28 Statement-1 : Torque on the coil is maximum, when coil
(Reason). Each of these questions has four alternative choices, is suspended in a radial magnetic field.
only one of which is the correct answer. You have to select the Statement-2 : The torque tends to rotate the coil on its
correct choice. own axis.
(a) Statement-1 is True, Statement-2 is True; Statement-2 is a Q.29 Statement-1 : A current carr ying loop placed in
correct explanation for Statement-1. equilibrium in a uniform magnetic field starts oscillating
(b) Statement-1 is True, Statement-2 is True; Statement-2 is when disturbed from equilibrium.
NOT a correct explanation for Statement-1. Statement-2 : A system when disturbed slightly from stable
equilibrium oscillates.

RESPONSE GRID 26. 27. 28. 29.

DAILY PRA CTICE PROBLEM SHEET 41 - PHYSICS


Total Questions 29 Total Marks 116
Attempted Correct
Incorrect N et Score
Cut-off Score 30 Qualifying Score 48
Success Gap = Net Score – Qualifying Score
Net Score = (Correct × 4) – (Incorrect × 1)

Space for Rough Work


t.me/Ebooks_Encyclopedia27. t.me/Magazines4all

DPP - Daily Practice Problems


Name : Date :

Start Time : End Time :

SYLLABUS : MAGNETISM AND MATTER - 1 (Bar magnet as an equivalent solenoid, Magnetic field lines,
42
Earth's magnetic field and magnetic elements)

Max. Marks : 120 Time : 60 min.


GENERAL INSTRUCTIONS
• The Daily Practice Problem Sheet contains 30 MCQ's. For each question only one option is correct. Darken the correct
circle/ bubble in the Response Grid provided on each page.
• You have to evaluate your Response Grids yourself with the help of solution booklet.
• Each correct answer will get you 4 marks and 1 mark shall be deduced for each incorrect answer. No mark will be given/
deducted if no bubble is filled. Keep a timer in front of you and stop immediately at the end of 60 min.
• The sheet follows a particular syllabus. Do not attempt the sheet before you have completed your preparation for that
syllabus. Refer syllabus sheet in the starting of the book for the syllabus of all the DPP sheets.
• After completing the sheet check your answers with the solution booklet and complete the Result Grid. Finally spend time
to analyse your performance and revise the areas which emerge out as weak in your evaluation.

DIRECTIONS (Q.1-Q.21) : There are 21 mul1tiple choice wb/m–2 making an angle of 30° with the field. Find the
questions. Each question has 4 choices (a), (b), (c) and (d), out couple acting on it.
of which ONLY ONE choice is correct. (a) 2.5 Nm (b) 5.5 Nm (c) 7.5 Nm (d) 9.0 Nm
Q.4 The force experienced by a pole of strength 100 Am at a
Q.1 A north pole of strength 50 Am and south pole of strength distance of 0.2 m from a short magnet of length 5 cm and
100 Am are separated by a distance of 10 cm in air. Find pole strength of 200 Am on its axial line will be
the force between them. (a) 2.5 × 10-2 N (b) 2.5 × 10-3 N
(a) 50 × 10–3 N (b) 25 × 10–3 N (c) 5.0 × 10 N-2 (d) 5.0 × 10-3 N
(c) 20 × 10–6 N (d) 30 × 10–18 N Q.5 A magnet of moment M is lying in a magnetic field of induction
Q.2 Calculate magnetic induction at a distance of 20 cm from B. W1 is the work done in turning it from 0º to 60º and W2 is
a pole of strength 40 Am in air. the work done in turning it from 30º to 90º. Then
(a) 10–4 wb/m2 (b) 10–8 wb/m2
–1
(c) 10 wb/m 2 (d) 10–12 wb/m2 W1
(a) W2 = W1 (b) W2 =
Q.3 A bar magnet of length 0.2 m and pole strength 5 Am is 2
kept in a uniform magnetic induction field of strength 15 (c) W2 = 2W1 (d) W2 = 3 W1

RESPONSE GRID 1. 2. 3. 4. 5.
Space for Rough Work
t.me/Ebooks_Encyclopedia27. t.me/Magazines4all

EBD_7156
2 DPP/ P 42
Q.6 A bar magnet of magnetic moment 4.0 A-m2 is free to rotate comparison to the separation between them. A magnetic
about a vertical axis through its centre. The magnet is needle is placed between the magnets at point P which gets
released from rest from east-west position. Kinetic energy deflected through an angle q under the influence of
of the magnet in north-south position will be (Horizontal magnets. The ratio of distances d1 and d2 will be
component of earth’s magnetic field BH = 25mT) B
(a) (2 tan q)1/3
(a) 10-2 J (b) 10-4 J (c) 10-6 J (d) 0
Q.7 The length of a bar magnet is 10 cm and its pole strength is (b) (2 tan q)-1/3 A P
10-3 Weber. It is placed in a magnetic field of induction
(c) (2 cot q)1/3 q d
4 p × 10-3 Tesla in a direction making an angle 30º with d1 2
the field direction. The value of torque acting on the magnet (d) (2 cot q)-1/3
will be – Q.14 The period of oscillation of a freely suspended bar magnet
(a) 2p × 10-7 N-m (b) 2p × 10-5 N-m is 4 second. If it is cut into two equal parts length wise
2
(c) 0.5 × 10 N-m (d) None of these then the time period of each part will be
Q.8 At magnetic poles of earth, angle of dip is (a) 4 sec (b) 2 sec (c) 0.5 sec (d) 0.25 sec
(a) zero (b) 45° (c) 90 ° (d) 180 ° Q.15 The length, breadth and mass of two bar magnets are same
Q.9 A short bar magnet is placed with its north pole pointing but their magnetic moments are 3M and 2M respectively.
south. The neutral point is 10 cm away from the centre of These are joined pole to pole and are suspended by a string.
the magnet. If H = 0.4 gauss, calculate magnetic moment When oscillated in a magnetic field of strength B, the time
of the magnet. period obtained is 5s. If the poles of either of the magnets
(a) 2 Am2 (b) 1A m2 (c) 0.1 A m2 (d) 0.2 Am2 are reverse then the time period of the combination in the
Q.10 A bar magnet with its poles 25 cm apart and of pole strength same magnetic field will be –
24.0 A-m rests with its centre on a frictionless pivot. A
force F is applied on the magnet at a distance of 12 cm (a) 3 3 s (b) 2 2 s (c) 5 5 s (d) 1s
from the pivot, so that it is held in equilibrium at an angle Q.16 A thin magnetic needle oscillates in a horizontal plane with
of 30º with respect to a magnetic field of induction 0.25 a period T. It is broken into n equals parts. The time period
T. The value of force F is of each part will be
(a) 65.62 N (b) 2.56 N (c) 6.52 N (d) 6.25 N T T
Q.11 A small magnet of magnetic moment 4A-m2 is placed on a (a) T (b) (c) Tn2 (d) 2
n n
deflection magnetometer in tan-B position at a distance Q.17 A bar magnet made of steel has a magnetic moment of 2.5 A-
of 20 cm from the compass needle. At what distance from m2 and a mass of 6.6 × 103 kg. If the density of steel is
compass needle should another small magnet of moment 7.9 × 109 kg/m3, find the intensity of magnetization of the
0.5A-m2 be placed such that the deflection of the needle magnet.
remains zero ? (a) 3.0 × 106 A/m (b) 2.0 × 106 A/m
(a) 12 cm (b) 10 cm (c) 20 cm (d) 30 cm 6
(c) 5.0 × 10 A/m (d) 1.2 × 106 A/m
Q.12 The ratio of intensities of magnetic field, at distances x Q.18 A short magnet of length 4 cm is kept at a distance of 20
and 2x from the centre of magnet of length 2cm on its cm to the east of a compass box such that is axis is
axis, will be perpendicular to the magnetic meridian. If the deflection
(a) 4 : 1 (b) 4 : 1 approx produced is 45°, find the pole strength (H = 30 Am–1)
(c) 8 : 1 (d) 8 : 1 approx (a) 17.7 Am (b) 44.2 Am
Q.13 Two magnets A and B are identical and these are arranged (c) 27.7 Am (d) 37.7 Am
as shown in the figure. Their length is negligible in

6. 7. 8. 9. 10.
RESPONSE
11. 12. 13. 14. 15.
GRID
16. 17. 18.

Space for Rough Work


t.me/Ebooks_Encyclopedia27. t.me/Magazines4all

DPP/ P 42 3
Q.19 A 10 cm long bar magnet of magnetic moment 1.34 Am2 Q.23 The correct statements regarding the lines of force of the
is placed in the magnetic meridian with its south pole magnetic field B are
pointing geographical south. The neutral point is obtained (1) Magnetic intensity is a measure of lines of force
at a distance of 15 cm from the centre of the magnet. passing through unit area held normal to it
Calculate the horizontal component of earth’s magnetic (2) Magnetic lines of force form a closed curve
field. (3) Due to a magnet magnetic lines of force never cut
(a) 0.12 × 10–4 T (b) 0.21× 10–4 T each other
(c) 0.34 × 10 T –4 (d) 0.87 × 10–7 T (4) Inside a magnet, its magnetic lines of force move from
Q.20 A 30 cm long bar magnet is placed in the magnetic meridian north pole of a magnet towards its south pole
with its north pole pointing south. The neutral point is Q.24 A short bar magnet of magnetic moment 5.25 × 10–2 JT–1
obtained at a distance of 40 cm from the centre of the is placed with its axis perpendicular to the earth’s field
magnet. Pole strength of the magnet is (The horizontal direction. Magnitude of the earth’s field at the place is given
component of earth’s magnetic field is 0.34 Gauss) to be 0.42 G. Ignore the length of the magnet in comparison
(a) 26.7 Am (b) 16.7 Am (c) 12.7 Am (d) 15.2 Am to the distance involved. Then
Q.21 A long straight horizontal cable carries a current of 2.5 A (1) the distance from the centre of the magnet on its
in the direction 10° south of west to 10° north of east. The normal bisector at which the resultant field is inclined
magnetic meridian of the place happens to be 10° west of at 45° with the earth's field is 5 cm
the geographic meridian. The earth’s magnetic field at the (2) the distance from the centre of the magnet on its axis
location is 0.33 Gauss, and the angle of dip is zero. at which the resultant field inclined at 45° with the
Distance of the line of neutral points from the cable is earth's field is 6.3 cm
(Ignore the thickness of the cable). (3) the distance from the centre of the magnet on its
(a) 1.5 cm (b) 2.5 cm (c) 3.5 cm (d) 2.0 cm normal bisector at which the resultant field inclined
at 45° with the earth's field is 8.3 cm
DIRECTIONS (Q.22-Q.24) : In the following questions, (4) the distance from the centre of the magnet on its axis
more than one of the answers given are correct. Select the at which the resultant field inclined at 45° with the
correct answers and mark it according to the following earth's field is 8 cm
codes:
DIRECTIONS (Q.25-Q.27) : Read the passage given below
Codes : and answer the questions that follows :
(a) 1, 2 and 3 are correct (b) 1 and 2 are correct
A telephone cable at a place has four long, straight horizontal wires
(c) 2 and 4 are correct (d) 1 and 3 are correct carrying a current of 1.0 A in the same direction east to west. The
Q.22 Which of the following is/are not the main difference
earth’s magnetic field at the place is 0.39 Gauss, and the angle of
between electric lines of force and magnetic lines of force dip is 35°. The magnetic declination is nearly zero. (cos 35° =
? 0.82, sin 35° = 0.57)
(1) Electric lines of force are closed curves whereas Q.25 The magnetic field produced by four current carrying
magnetic lines of force are open curves. straight cable wires at a distance 4 cm is
(2) Magnetic lines of force cut each other whereas (a) 0.2 Gauss (b) 0.3 Gauss
electric lines of force do not cut. (c) 0.4 Gauss (d) 0.5 Gauss
(3) Electric lines of force cut each other whereas Q.26 The resultant magnetic field below at points 4cm and above
magnetic lines of force do not cut. the cable are
(4) Electric lines of force are open curves whereas (a) 0.25, 0.56 Gauss (b) 0.14, 0.32 Gauss
magnetic lines of force are closed curves. (c) 0.23, 0.34 Gauss (d) 0.52, 0.62 Gauss

RESPONSE 19. 20. 21. 22. 23.


GRID 24. 25. 26.

Space for Rough Work


t.me/Ebooks_Encyclopedia27. t.me/Magazines4all

EBD_7156
4 DPP/ P 42
Q.27 The angle that resultant makes with horizontal in case below (c) Statement-1 is False, Statement-2 is True.
and above the cable respectively, are (d) Statement-1 is True, Statement-2 is False.
(a) 30°, 45° (b) tan–1 1.8, tan–1 0.43 Q.28 Statement-1 : Gauss theorem is not applicable in
magnetism.
(c) tan–1 2, tan–1 2 (d) sin–1 0.7, sin–1 0.9
Statement-2 : Mono magnetic pole does not exist.
DIRECTIONS (Q. 28-Q.30) : Each of these questions contains Q.29 Statement-1 : A compass needle when placed on the
two statements: Statement-1 (Assertion) and Statement-2 magnetic north pole of the earth cannot rotate in vertical
(Reason). Each of these questions has four alternative choices, direction.
only one of which is the correct answer. You have to select the Statement-2 : The earth has only horizontal component
correct choice. of its magnetic field at the north poles.
Q.30 Statement-1 : We cannot think of magnetic field
(a) Statement-1 is True, Statement-2 is True; Statement-2 is a configuration with three poles.
correct explanation for Statement-1. Statement-2 : A bar magnet does not exert a torque on
(b) Statement-1 is True, Statement-2 is True; Statement-2 is itself due to its own field.
NOT a correct explanation for Statement-1.

RESPONSE GRID 27. 28. 29. 30.

DAILY PRA CTICE PROBLEM SHEET 42 - PHYSICS


Total Questions 30 Total Marks 120
Attempted Correct
Incorrect N et Score
Cut-off Score 30 Qualifying Score 50
Success Gap = Net Score – Qualifying Score
Net Score = (Correct × 4) – (Incorrect × 1)

Space for Rough Work


t.me/Ebooks_Encyclopedia27. t.me/Magazines4all

DPP - Daily Practice Problems


Name : Date :

Start Time : End Time :

SYLLABUS : MAGNETISM & MATTER-2 (Para, dia and ferro-magnetic substances, magnetic susceptibility and
43
permeability, Hysteresis, Electromagnets and permanent magnets.)

Max. Marks : 120 Time : 60 min.


GENERAL INSTRUCTIONS
• The Daily Practice Problem Sheet contains 30 MCQ's. For each question only one option is correct. Darken the correct
circle/ bubble in the Response Grid provided on each page.
• You have to evaluate your Response Grids yourself with the help of solution booklet.
• Each correct answer will get you 4 marks and 1 mark shall be deduced for each incorrect answer. No mark will be given/
deducted if no bubble is filled. Keep a timer in front of you and stop immediately at the end of 60 min.
• The sheet follows a particular syllabus. Do not attempt the sheet before you have completed your preparation for that
syllabus. Refer syllabus sheet in the starting of the book for the syllabus of all the DPP sheets.
• After completing the sheet check your answers with the solution booklet and complete the Result Grid. Finally spend time
to analyse your performance and revise the areas which emerge out as weak in your evaluation.

DIRECTIONS (Q.1-Q.24) : There are 24 multiple choice Q.3 If the magnetic dipole moment of an atom of diamagnetic
questions. Each question has 4 choices (a), (b), (c) and (d), out material, paramagnetic material and ferromagnetic material
of which ONLY ONE choice is correct. denoted by m d , m p , m f respectively then
Q.1 Susceptibility of ferromagnetic substance is
(a) > 1 (b) < 1 (c) 0 (d) 1 (a) md , ¹ 0 and m f ¹ 0 (b) m p = 0 and m f ¹ 0
Q.2 Among the following properties describing diamagnetism
identify the property that is wrongly stated. (c) m d = 0 and m p ¹ 0 (d) m d ¹ 0 and m p = 0
(a) Diamagnetic material do not have perman ent
magnetic moment Q.4 When a piece of a ferromagnetic substance is put in a
(b) Diamagnetism is explained in terms of uniform magnetic field, the flux density inside it is four
electromagnetic induction. times the flux density away from the piece. The magnetic
(c) Diamagnetic materials have a small positive permeability of the material is
susceptibility
(d) The magnetic moment of individual electrons (a) 1 (b) 2
neutralize each other (c) 3 (d) 4

RESPONSE GRID 1. 2. 3. 4.
Space for Rough Work
t.me/Ebooks_Encyclopedia27. t.me/Magazines4all

EBD_7156
2 DPP/ P 43
Q.5 The given figure represents a material which is Q.13 The relative permeability is represented by µr and the
(a) Paramagnetic susceptibility is denoted by c for a magnetic substance.
Then for a paramagnetic substance
(b) Diamagnetic
(a) m r < 1, c < 0 (b) m r < 1, c > 0
(c) Ferromagnetic
(d) None of these (c) m r > 1, c < 0 (d) m r > 1, c > 0

Q.6 Liquid oxygen remains suspended between two pole faces Q.14 The use of study of hysteresis curve for a given material is
to estimate the
of a magnet because it is
(a) voltage loss (b) hysteresis loss
(a) diamagnetic (b) paramagnetic
(c) current loss (d) all of these
(c) ferromagnetic (d) antiferromagnetic
Q.15 The magnetic moment of atomic neon is
Q.7 A superconductor exhibits perfect
(a) zero (b) mB/2
(a) ferrimagnetism (b) ferromagnetism (c) mB (d) 3mB/2
(c) paramagnetism (d) diamagnetism Q.16 A ferromagnetic material is heated above its Curie
Q.8 Which of the following is the most suitable for the core temperature, then which one is a correct statement ?
of electromagnets? (a) Ferromagnetic domains are perfectly arranged
(a) Soft iron (b) Steel (b) Ferromagnetic domains becomes random
(c) Copper-nickel alloy (d) Air (c) Ferromagnetic domains are not influenced
Q.9 The universal property of all substances is (d) Ferromagnetic material changes itself into
(a) diamagnetism (b) ferromagnetism diamagnetic material
(c) paramagnetism (d) all of these Q.17 If a diamagnetic substance is brought near north or south
pole of a bar magnet, it is
Q.10 If a magnetic substance is kept in a magnetic field, then
(a) attracted by the poles
which of the following substance is thrown out ?
(b) repelled by the poles
(a) Paramagnetic (b) Ferromagnetic
(c) repelled by the north pole and attracted by the south pole
(c) Diamagnetic (d) Antiferromagnetic
(d) attracted by the north pole and repelled by the south pole
Q.11 In the hysteresis cycle, the value of H needed to make the
Q.18 The material of permanent magnet has
intensity of magnetisation zero is called
(a) high retentivity, low coercivity
(a) Retentivity (b) Coercive force
(b) low retentivity, high coercivity
(c) Lorentz force (d) None of these (c) low retentivity, low coercivity
Q.12 If a diamagnetic solution is poured into a U-tube and one (d) high retentivity, high coercivity
arm of this U-tube placed between the poles of a strong Q.19 Diamagnetic substances are
magnet with the meniscus in a line with field, then the level (a) feebly attracted by magnets
of the solution will
(b) strongly attracted by magnets
(a) rise (b) fall (c) feebly repelled by magnets
(c) oscillate slowly (d) remain as such (d) strongly repelled by magnets

5. 6. 7. 8. 9.
RESPONSE
10. 11. 12. 13. 14.
GRID
15. 16. 17. 18. 19.

Space for Rough Work


t.me/Ebooks_Encyclopedia27. t.me/Magazines4all

DPP/ P 43 3
Q.20 For an isotropic medium B, µ, H and M are related as (where Q.25 A magnetising field of 1600 Am–1 produces a magnetic
B, µ0, H and M have their usual meaning in the context of flux of 2.4 × 10–5 weber in a bar of iron of area of cross-
magnetic material) section 0.2 cm2. Then,
(a) (B – M) = µ0H (b) M = µ0(H + M) (1) the magnetic permeability of th e bar is
(c) H = µ0(H + M) (d) B = µ0(H + M) 7.5 × 10–4 TA–1m
Q.21 Relative permeability of iron is 5500, then its magnetic (2) the susceptibility of the bar is 596.1
susceptibility will be (3) the magnetic permeability of the bar is 4.1 Wbm–2
(a) 5500 × 107 (b) 5500 × 10–7 (4) the susceptibility of the bar is 496.1
(c) 5501 (d) 5499 Q.26 Which of the following statements are correct about
Q.22 A magnetising field of 2 × 103 amp/m produces a magnetic hysteresis?
flux density of 8p Tesla in an iron rod. The relative
(1) This effect is common to all ferromagnetic substances
permeability of the rod will be
(2) The hysteresis loop area is proportional to the thermal
(a) 102 (b) 100 (c) 104 (d) 101
energy developed per unit volume of the material
Q.23 The mass of a specimen of a ferromagnetic material is 0.6
(3) The shape of the hysteresis loop is characteristic of
kg. and its density is 7.8 × 103 kg/m 3 . If the area of
hysteresis loop of alternating magnetising field of the material
frequency 50Hz is 0.722 MKS units then the hysteresis (4) The hysteresis loop area is independent of the thermal
loss per second will be Y energy developed per unit volume of the material
Soft iron Q.27 Which of the following statments are false about the
(a) 277.7 × 10-5 Joule I
Steel
magnetic susceptibility c m of paramagnetic substance?
(b) 277.7 × 10-6 Joule
X'
H
X (1) Value of c m is directly proportional to the absolute
(c) 277.7 × 10-4 Joule temperature of the sample
(d) 27.77 × 10-4 Joule (2) c m is negative at all temperature
Y'
(3) c m does not depend on the temperature of the sample
Q.24 A diamagnetic material in a magnetic field moves
(a) from weaker to the stronger parts of the field (4) c m is positive at all temperature
(b) perpendicular to the field
DIRECTIONS (Q. 28-Q.30) : Each of these questions contains
(c) from stronger to the weaker parts of the field
(d) None of these two statements: Statement-1 (Assertion) and Statement-2
(Reason). Each of these questions has four alternative choices,
DIRECTIONS (Q.25-Q.27) : In the following questions, only one of which is the correct answer. You have to select the
more than one of the answers given are correct. Select the correct choice.
correct answers and mark it according to the following
codes: (a) Statement-1 is True, Statement-2 is True; Statement-2 is a
correct explanation for Statement-1.
Codes : (b) Statement-1 is True, Statement-2 is True; Statement-2 is
(a) 1, 2 and 3 are correct (b) 1 and 2 are correct NOT a correct explanation for Statement-1.
(c) 2 and 4 are correct (d) 1 and 3 are correct
(c) Statement -1 is False, Statement-2 is True.
(d) Statement -1 is True, Statement-2 is False.

RESPONSE 20. 21. 22. 23. 24.


GRID 25. 26. 27.

Space for Rough Work


t.me/Ebooks_Encyclopedia27. t.me/Magazines4all

EBD_7156
4 DPP/ P 43
Q.28 Statement-1 : The ferromagnetic substance do not obey Statement-2 : The magnetisation does not depend on
Curie’s law. temperature.
Statement-2 : At Curie point a ferromagnetic substance Q.30 Statement-1 : The permeability of a ferromagnetic
start behaving as a paramagnetic substance. material dependent on the magnetic field.
Q.29 Statement-1 : A paramagnetic sample displays greater Statement-2 : Permeability of a material is a constant
magnetisation (for the same magnetising field) when quantity.
cooled.

RESPONSE GRID 28. 29. 30.

DAILY PRA CTICE PROBLEM SHEET 43 - PHYSICS


Total Questions 30 Total Marks 120
Attempted Correct
Incorrect N et Score
Cut-off Score 28 Qualifying Score 46
Success Gap = Net Score – Qualifying Score
Net Score = (Correct × 4) – (Incorrect × 1)

Space for Rough Work


t.me/Ebooks_Encyclopedia27. t.me/Magazines4all

DPP - Daily Practice Problems


Name : Date :

Start Time : End Time :

SYLLABUS : ELECTROMAGNETIC INDUCTION-1 (Magnetic flux, Faraday's law of electromagnetic induction, Lenz's
44
law, motional e.m.f.)

Max. Marks : 116 Time : 60 min.


GENERAL INSTRUCTIONS
• The Daily Practice Problem Sheet contains 29 MCQ's. For each question only one option is correct. Darken the correct
circle/ bubble in the Response Grid provided on each page.
• You have to evaluate your Response Grids yourself with the help of solution booklet.
• Each correct answer will get you 4 marks and 1 mark shall be deduced for each incorrect answer. No mark will be given/
deducted if no bubble is filled. Keep a timer in front of you and stop immediately at the end of 60 min.
• The sheet follows a particular syllabus. Do not attempt the sheet before you have completed your preparation for that
syllabus. Refer syllabus sheet in the starting of the book for the syllabus of all the DPP sheets.
• After completing the sheet check your answers with the solution booklet and complete the Result Grid. Finally spend time
to analyse your performance and revise the areas which emerge out as weak in your evaluation.

DIRECTIONS (Q.1-Q.20) : There are 20 multiple choice (a) 31 mV


questions. Each question has 4 choices (a), (b), (c) and (d), out (b) 19 mV
of which ONLY ONE choice is correct. (c) 14 mV
r (d) 6 mV
Q.1 A loop of wire is placed in a magnetic field B = 0.02iˆ T..
Then the flux through the loop if its area vector Q.3 A current carrying solenoid is approaching a conducting
r loop as shown in the figure. The direction of induced
A = 30iˆ + 16ˆj + 23kˆ cm2 is
current as observed by an observer on the other side of the
(a) 60 mWb (b) 32 m Wb loop will be -
(c) 46 m Wb (d) 138 m Wb (a) anti-clockwise
Q.2 The magnetic flux passing perpendicular to the plane of (b) clockwise V
the coil and directed into the paper is varying according to (c) east
the relation f = 3t2 + 2t + 3, where f is in milliweber and (d) west
t is in second. Then the magnitude of emf induced in the
loop when t = 2 second is-

RESPONSE GRID 1. 2. 3.

Space for Rough Work


t.me/Ebooks_Encyclopedia27. t.me/Magazines4all

EBD_7156
2 DPP/ P 44
Q.4 Consider the arrangement shown in figure in which the north (a) Zero
pole of a magnet is moved away from a thick conducting × × × × ×
(b) B vR2/2 and M is at higher potential × B × N × ×
loop containing capacitor. Then excess positive charge will
(c) pRBV and Q is at higher potential × × ×V × ×
arrive on × × × × ×
(a) plate a (d) 2RBV and Q is at higher potential M Q
(b) plate b Q.10 An aeroplane having a distance of 50 metre between the
(c) both plates a and b edges of its wings is flying horizontally with a speed of
(d) neither a nor b plates 360km/hour. If the vertical component of earth’s magnetic
Q.5 An electron moves along the line AB, which lies in the field is 4 × 10–4 weber/m2, then the induced emf between
same plane as a circular loop of conducting wires as shown the edges of its wings will be –
in the diagram. What will be the direction of current (a) 2 mV (b) 2 V (c) 0.2 V (d) 20 V
induced if any, in the loop? Q.11 At certain location in the northern hemisphere, the earth's
(a) No current will be induced magnetic field has a magnitude of 42 mT and points down
(b) The current will be clockwise ward at 57º to vertical. The flux through a horizontal surface
A B
(c) The current will be anticlockwise of area 2.5 m2 will be- (Given cos 33º = 0.839, cos 57º =
(d) The current will change direction as the electron passes by 0.545)
Q.6 When a small piece of wire passes between the magnetic (a) 42 × 10–6 Wb/m2 (b) 42 × 10–6 Wb/m2
poles of a horse-shoe magnet in 0.1 sec, emf of 4 × 10 –3 –6
(c) 57 × 10 Wb/m 2 (d) 57 × 10–6 Wb/m2
volt is induced in it. The magnetic flux between the poles Q.12 A square loop of side a is rotating about its diagonal with
is : angular velocity w in a perpendicular magnetic field as shown
(a) 4 × 10–2 weber (b) 4 × 10–3 weber in the figure. If the number of turns in it is × × × × × ×
–4 × × × × × ×
(c) 4 × 10 weber (d) 4 × 10–6 weber 10 then the magnetic flux linked with the × × × × × ×
Q.7 The normal magnetic flux passing through a coil changes loop at any instant will be– × × × × ××
× × × × ××
with time according to following equation (a) 10Ba2 cos wt (b) 10Ba × × w× × × ×
× × × × ××
f = 10t2 + 5t + 1 2 2 × × × × ××
(c) 10Ba (d) 20Ba
where f is in milliweber and t is in second. The value of
induced e.m.f. produced in the coil at t = 5s will be – Q.13 Two identical coaxial circular loops carry current i each
(a) zero (b) 1 V (c) 2 V (d) 0.105 V circulating in the clockwise direction. If the loops are
Q.8 A bicycle wheel of radius 0.5 m has 32 spokes. It is rotating approaching each other, then
at the rate of 120 revolutions per minute, perpendicular to (a) Current in each loop increases
the horizontal component of earth's magnetic field (b) Current in each loop remains the same
BH = 4 × 10–5 tesla. The emf induced between the rim and (c) Current in each loop decreases
the centre of the wheel will be- (d) Current in one-loop increases and in the other it
(a) 6.28 × 10–5 V (b) 4.8 × 10–5 V decreases
(c) 6.0 × 10 V –5 (d) 1.6 × 10–5 V Q.14 The distance between the ends of wings of an aeroplane is
Q.9 A thin semicircular conducting ring of radius R is falling 3m. This aeroplane is descending down with a speed of
with its plane vertical in a horizontal magnetic induction 300 km/hour. If the horizontal component of earths
B. At the position MNQ shown in the fig, the speed of the magnetic field is 0.4 gauss then the value of e.m.f. induced
ring is V. The potential difference developed across the in the wings of the plane will be –
semicircular ring is (a) 1 V (b) 2 V (c) 0.01 V (d) 0.1 V

4. 5. 6. 7. 8.
RESPONSE
9. 10. 11. 12. 13.
GRID
14.

Space for Rough Work


t.me/Ebooks_Encyclopedia27. t.me/Magazines4all

DPP/ P 44 3
Q.15 A gramophone disc of brass of diameter 30 cm rotates (a) zero
horizontally at the rate of 100/3 revolutions per minute. If
(b) 2 Blv
the vertical component of the earth's magnetic field be 0.01
weber/metre2, then the emf induced between the centre (c) 2 Blv sin (q/2)
and the rim of the disc will be- (d) 2 Blv cos (q/2)
(a) 7.065 × 10–4 V (b) 3.9 × 10–4 V
(c) 2.32 × 10 V –4 (d) None of the above DIRECTIONS (Q.21-Q.23) : In the following questions,
Q.16 A closed coil consists of 500 turns on a rectangular frame more than one of the answers given are correct. Select the
of area 4.0 cm2 and has a resistance of 50 ohm. The coil is correct answers and mark it according to the following
kept with its plane perpendicular to a uniform magnetic codes:
field of 0.2 weber/meter 2. The amount of charge flowing
Codes :
through the coil if it is turned over (rotated through 180º) (a) 1, 2 and 3 are correct (b) 1 and 2 are correct
will be - (c) 2 and 4 are correct (d) 1 and 3 are correct
(a) 1.6 × 10–19 C (b) 1.6 × 10–9 C Q.21 A rectangular coil of size 10 cm ×20 cm has 60 turns. It is
(c) 1.6 × 10 C –3 (d) 1.6 × 10–2 C rotating about one of its diagonals in magnetic field
Q.17 A copper disc of radius 0.1 m rotates about its centre with 0.5 Wb/m2 with a rate of 1800 revolution per minute. The
10 revolution per second in 'a uniform magnetic field of induced e.m.f. in the coil can be
0.1 T. The emf induced across the radius of the disc is - (1) 111 V (2) 112 V (3) 113 V (4) 114 V
(a) p/10 V (b) 2p/10 V (c) 10p mV (d) 20p mV Q.22 A closed coil of copper whose area is 1m × 1m is free to
Q.18 Two rail tracks, insulated from each other and the ground, rotate about an axis. The coil is placed perpendicular to a
are connected to milli voltmeter. What is the reading of magnetic field of 0.10 Wb/m2. It is rotated through 180º
the milli voltmeter when a train passes at a speed of 180 in 0.01 second. Then (The resistance of the coil is 2.0W)
km/hr along the track ? Given that – the horizontal (1) The induced e.m.f. in the coil is 20 V
component of earth’s magnetic field BH is 0.2 × 10–4 Wb/ (2) The induced current in the coil is 10 A
m2 and rails are separated by 1 metre. (3) The induced e.m.f. in the coil is 10 V
(a) 1 mV (b) 10 mV (c) 100 mV (d) 1 V (4) The induced current in the coil is 20 A
Q.19 The annular disc of copper, with inner radius a and outer Q.23 5.5 × 10–4 magnetic flux lines are passing through a coil
radius b is rotating with a uniform angular speed w, in a of resistance 10 ohm and number of turns 1000. If the
number of flux lines reduces to 5 × 10–5 in 0.1 sec. Then
region where a uniform magnetic field B along the axis of
(1) The electromotive force induced in the coil is 5V
rotation exists. Then, the emf induced between inner side
(2) The electromotive force induced in the coil is
and the outer rim of the disc is- 5 × 10–4 V
1 (3) The current induced in the coil is 0.5 A
(a) Zero (b) Bwa2 (4) The current induced in the coil is 10 A
2
1 1 DIRECTIONS (Q.24-Q.26) : Read the passage given below
(c) Bwb2 (d) Bw (b2 – a2) and answer the questions that follows :
2 2
Q.20 A conducting wire in the shape of Y with each side of length In the figure shown, the rod has a resistance R, the horizontal
l is moving in a uniform magnetic field B, with a uniform rails have negligible friction. Magnetic field of intensity B is
speed v as shown in fig. The induced emf at the two ends X directed perpendicular into the plane of paper. A cell of e.m.f. E
and negligible internal resistance is connected between points a
and Y of the wire will be-
and b. The rod is initially at rest.

RESPONSE 15. 16. 17. 18. 19.


GRID 20. 21. 22. 23.

Space for Rough Work


t.me/Ebooks_Encyclopedia27. t.me/Magazines4all

EBD_7156
4 DPP/ P 44
ROD
B DIRECTIONS (Q. 27-Q.29) : Each of these questions

a
× × × × contains two statements: Statement-1 (Assertion) and
Statement-2 (Reason). Each of these questions has four
× × × × alternative choices, only one of which is the correct answer.
You have to select the correct choice.
E L
(a) Statement-1 is True, Statement-2 is True; Statement-2 is a
× × × × correct explanation for Statement-1.
b (b) Statement-1 is True, Statement-2 is True; Statement-2 is
× × × × NOT a correct explanation for Statement-1.
Q.24 The velocity of the rod as a function of time t (where (c) Statement -1 is False, Statement-2 is True.
t = mR/Bl2) is (d) Statement -1 is True, Statement-2 is False.
E E Q.27 Statement-1 : The induced e.m.f. and current will be same
(a) (1 - e - t / t ) (b) (1 + e - t / t ) in two identical loops of copper and aluminium, when
Bl Bl
E rotated with same speed in the same magnetic field.
(c) 3 E (1 - e- t / t ) (d) (1 - e - t / t ) Statement-2 : Induced e.m.f. is proportional to rate of
2 Bl 2Bl
Q.25 After some time the rod will approach a terminal speed. change of magnetic field while induced current depends
The speed is on resistance of wire.
Q.28 Statement-1 : An aircraft flies along the meridian, the
3 E E E 2E
(a) (b) (c) (d) potential at the ends of its wings will be the same.
2 Bl 2Bl Bl Bl
Q.26 The current when the rod attains its terminal speed is Statement-2 : Whenever there is change in the magnetic
flux e.m.f. induces.
2E E 3E
(a) (b) (c) (d) zero Q.29 Statement-1 : Lenz’s law violates the principle of
R R 2R
conservation of energy.
Statement-2 : Induced e.m.f. opposes the change in
magnetic flux responsible for its production.

RESPONSE 24. 25. 26. 27. 28.


GRID 29.

DAILY PRA CTICE PROBLEM SHEET 44 - PHYSICS


Total Questions 29 Total Marks 116
Attempted Correct
Incorrect N et Score
Cut-off Score 26 Qualifying Score 46
Success Gap = Net Score – Qualifying Score
Net Score = (Correct × 4) – (Incorrect × 1)

Space for Rough Work


t.me/Ebooks_Encyclopedia27. t.me/Magazines4all

DPP - Daily Practice Problems


Name : Date :

Start Time : End Time :

SYLLABUS : ELECTROMAGNETIC INDUCTION - 2 : Self inductance, mutual inductance,


45
Growth and decay of current in L.R. circuit, Transformer, Electric motor, Generator
Max. Marks : 116 Time : 60 min.
GENERAL INSTRUCTIONS
• The Daily Practice Problem Sheet contains 29 MCQ's. For each question only one option is correct. Darken the correct
circle/ bubble in the Response Grid provided on each page.
• You have to evaluate your Response Grids yourself with the help of solution booklet.
• Each correct answer will get you 4 marks and 1 mark shall be deduced for each incorrect answer. No mark will be given/
deducted if no bubble is filled. Keep a timer in front of you and stop immediately at the end of 60 min.
• The sheet follows a particular syllabus. Do not attempt the sheet before you have completed your preparation for that
syllabus. Refer syllabus sheet in the starting of the book for the syllabus of all the DPP sheets.
• After completing the sheet check your answers with the solution booklet and complete the Result Grid. Finally spend time
to analyse your performance and revise the areas which emerge out as weak in your evaluation.

DIRECTIONS (Q.1-Q.20) : There are 20 multiple choice


questions. Each question has 4 choices (a), (b), (c) and (d), out
of which ONLY ONE choice is correct.
Q.1 A current increases uniformly from zero to one ampere in (A) (B) (C)
0.01 second, in a coil of inductance 10 mH. The induced (a) Maximum in situation (A)
e.m.f. will be - (b) Maximum in situation (B)
(a) 1 V (b) 2 V (c) 3 V (d) 4 V (c) Maximum in situation (C)
Q.2 The current in a coil varies with respect to time t as I = 3t2 + (d) The same in all situations
2t. If the inductance of coil be 10 mH, the value of induced Q.4 A current of 10 A in the primary coil of a circuit is reduced
e.m.f. at t = 2s will be- to zero at a uniform rate in 10–3 second. If the coefficient
(a) 0.14 V (b) 0.12 V (c) 0.11 V (d) 0.13 V of mutual inductance is 3H, the induced e.m.f. in the
Q.3 Two circular coils can be arranged in any of the three secondary coil will be-
situations shown in the figure. Their mutual inductance will (a) 3 kV (b) 30 kV (c) 2 kV (d) 20 kV
be
RESPONSE GRID 1. 2. 3. 4.
Space for Rough Work
t.me/Ebooks_Encyclopedia27. t.me/Magazines4all

EBD_7156
2 DPP/ P 45
Q.5 Three inductances are connected as shown below.
Assuming no coupling, the resultant inductance will be-
(L1 = 0.75 H, L2 = L3 = 0.5 H) L2 L
(a) 0.25 H L1
(b) 0.75 H
(c) 0.01 H
(d) 1 H L3
Q.6 A solenoid has an inductance of 50 mH and a resistance of
0.025 W. If it is connected to a battery, how long will it (a) 2.0A (b) 3.0A
take for the current to reach one half of its final equilibrium (c) 5.0A (d) 6.0A
value? Q.12 Two inductors L1 and L2 are at a sufficient distance apart.
(a) 1.34 s (b) 1.38 s (c) 1.38 ms (d) 0.23 s Equivalent inductance when they are connected (i) in series
Q.7 The current in the primary coil of a transformer (assuming (ii) in parallel are
no power loss) as shown in fig. will be – L1L 2 L1L2
(a) L1 + L2, (b) L1 – L2,
(a) 0.01 A P S L1 + L2 L1 - L2
(b) 1.0 A L1 + L2

/\/\/\/
VP = 220 V 220 W (c) L1 L2, L L (d) None of these
(c) 0.1 A 1 2
(d) 10–6 A. P S Q.13 A small coil of N1 turns, l1 length is tightly wound over the
Q.8 A current of 5A is flowing at 220V in the primary coil of a centre of a long solenoid of length l2, area of cross-section
transformer. If the voltage produced in the secondary coil A and number of turns N2. If a current I flows in the small
is 2200V and 50% of power is lost, then the current in the coil, then the flux through the long solenoid is
secondary coil will be –
m 0 N12 AI
(a) 2.5A (b) 5A (c) 0.25A (d) 0.025A (a) zero (b)
Q.9 An inductor (L = 100 mH), a resistor (R = 100W) and a l1
battery (E = 100 V) are initially connected in series as m 0 N1 N 2 AI
shown in the figure. After a long time the battery is (c) inifinite (d)
l2
disconnected after short circuiting the points A and B. The
current in the circuit 1 ms after the short circuit is Q.14 If the current in the primary coil is reduced from 3.0
L ampere to zero in 0.001 second, the induced e.m.f in the
(a) e A secondary coil is 1500 volt. The mutual inductance of the
(b) 0.1 A two coils will be-
R
(c) 1 A (a) 0.5 H (b) 0.05 H (c) 0.005 H (d) 0.0005 H
A B Q.15 A 50 Hz a.c. current of crest value 1A flows through the
(d) 1/e A E
primary of a transformer. If the mutual inductance between
Q.10 Which of the following is constructed on the principle of
the primary and secondary be 1.5 H, the crest voltage
electromagnetic induction ?
induced in secondary is-
(a) Galvanometer (b) Electric motor
(a) 75 V (b) 150 V (c) 471 V (d) 300 V
(c) Generator (d) Voltmeter
Q.16 In an inductor of inductance L = 100 mH, a current of I =
Q.11 In the circuit, E = 10 volt, R1 = 5.0 ohm, R2 = 10 ohm and
10A is flowing. The energy stored in the inductor is
L = 5.0 henry. The current just after the switch S is pressed
(a) 5 J (b) 10 J (c) 100 J (d) 1000 J
is.

5. 6. 7. 8. 9.
RESPONSE
10. 11. 12. 13. 14.
GRID
15. 16.

Space for Rough Work


t.me/Ebooks_Encyclopedia27. t.me/Magazines4all

DPP/ P 45 3
Q.17 A step up transformer has transformation ratio 5 : 3. What DIRECTIONS (Q.24-Q.26) : Read the passage given below
is voltage in secondary if voltage in primary is 60 V and answer the questions that follows :
(a) 20 V (b) 60 V (c) 100 V (d) 180 V
Q.18 A transformer has turn ratio 100 : 1. If secondary coil has In Fig., there is a conducting loop ABCDEFA, of resistance l
4 amp current then current in primary coil is per unit length placed near a long straight current-carrying wire.
(a) 4 A (b) 0.04 A (c) 0.4 A (d) 400 A The dimensions are shown in the figure. The long wire lies in
Q.19 A step-down transformer is used on a 1000 V line to deliver the plane of the loop. The current in the long wire varies as I = I0
20 A at 120 V at the secondary coil. If the efficiency of t.
the transformer is 80%, the current drawn from the line
is: A
a
B
(a) 3 A (b) 30 A (c) 0.3 A (d) 2.4 A
l a
Q.20 Energy stored in an inductor is proportional to (i = current
in the inductor) C
F

(a) i (b) i (c) i2 (d) i3 a

E
DIRECTIONS (Q.21-Q.23) : In the following questions, a D

more than one of the answers given are correct. Select the I

correct answers and mark it according to the following


codes: Q.24 The mutual inductance of the pair is
m0a æ 2a + l ö m0a æ 2a - l ö
Codes : (a) 1n ç ÷ (b) 1n ç ÷
(a) 1, 2 and 3 are correct (b) 1 and 2 are correct 2p è l ø 2p è l ø
2m0a æ a + l ö
(c) 2 and 4 are correct (d) 1 and 3 are correct (c) 1n ç ÷ (d) m0 a 1n æç a + l ö÷
Q.21 Voltage (r. m. s) in the secondary coil of a transformer p è l ø p è l ø
depends upon Q.25 The e.m.f. induced in the closed loop is
(1) voltage in the primary coil m0 I0 a æ 2a + l ö m0 I0 a æ 2a - l ö
(2) ratio of number of turns in the two coils (a) 1n ç ÷ (b) 1n ç ÷
(3) frequency of the source
2p è l ø 2p è l ø
(4) time-period of the source 2m0 I0 a æ a + l ö m0 I0 a æ a + l ö
(c) 1n ç ÷ (d) 1n ç ÷
Q.22 Core of a transformer can’t be made up of p è l ø p è l ø
(1) steel (2) alnico (3) iron (4) soft iron Q.26 The heat produced in the loop in time t is
Q.23 Large transformer, when used for some time, become hot 2 2
é m0 æ a + l ö ù é m0 æ 2a + l ö ù
and are cooled by circulating oil. The heating of ê 1n ç ÷ I0 ú at ê 1n ç ÷ I0 ú at
transformer is due to (a) ë 2p è l ø û (b) ë 2 p è l ø û
(1) heating effect of current 4l 8l
2 2
(2) hysteresis loss é 2m 0 æ a + l ö ù é m 0 æ 3a + l ö ù
(3) chemical effect of current ê p 1n ç l ÷ I0 ú at ê 1n ç ÷ I0 ú at
(c) ë è ø û (d) ë 2p è l ø û
(4) magnetic effect of current 3l 6l

RESPONSE 17. 18. 19. 20. 21.


GRID 22. 23. 24. 25. 26.

Space for Rough Work


t.me/Ebooks_Encyclopedia27. t.me/Magazines4all

EBD_7156
4 DPP/ P 45
DIRECTIONS (Qs. 27-Q.29) : Each of these questions contains Q.27 Statement-1 : Soft iron is used as a core of transformer.
two statements: Statement-1 (Assertion) and Statement-2 Statement-2 : Area of hysteresis loop for soft iron is
(Reason). Each of these questions has four alternative choices, small.
only one of which is the correct answer. You have to select the Q.28 Statement-1 : An electric motor will have maximum
correct choice. efficiency when back e.m.f. is equal to half of the applied
(a) Statement-1 is True, Statement-2 is True; Statement-2 is a e.m.f.
correct explanation for Statement-1. Statement-2 : Efficiency of electric motor depends only
(b) Statement-1 is True, Statement-2 is True; Statement-2 is on magnitude of back e.m.f.
NOT a correct explanation for Statement-1. Q.29 Statement-1 : A transformer cannot work on dc supply.
(c) Statement-1 is False, Statement-2 is True.
Statement-2 : dc changes neither in magnitude nor in
(d) Statement-1 is True, Statement-2 is False.
direction.

RESPONSE GRID 27. 28. 29.

DAILY PRA CTICE PROBLEM SHEET 45 - PHYSICS


Total Questions 29 Total Marks 116
Attempted Correct
Incorrect N et Score
Cut-off Score 28 Qualifying Score 48
Success Gap = Net Score – Qualifying Score
Net Score = (Correct × 4) – (Incorrect × 1)

Space for Rough Work


t.me/Ebooks_Encyclopedia27. t.me/Magazines4all

DPP - Daily Practice Problems


Name : Date :

Start Time : End Time :

SYLLABUS : ALTERNATING CURRENT - 1 (Alternating currents, peak and rms value of


46
alternating current/voltage; reactance and impedance, Pure circuits, LR, CR ac circuits.)
Max. Marks : 116 Time : 60 min.
GENERAL INSTRUCTIONS
• The Daily Practice Problem Sheet contains 29 MCQ's. For each question only one option is correct. Darken the correct
circle/ bubble in the Response Grid provided on each page.
• You have to evaluate your Response Grids yourself with the help of solution booklet.
• Each correct answer will get you 4 marks and 1 mark shall be deduced for each incorrect answer. No mark will be given/
deducted if no bubble is filled. Keep a timer in front of you and stop immediately at the end of 60 min.
• The sheet follows a particular syllabus. Do not attempt the sheet before you have completed your preparation for that
syllabus. Refer syllabus sheet in the starting of the book for the syllabus of all the DPP sheets.
• After completing the sheet check your answers with the solution booklet and complete the Result Grid. Finally spend time
to analyse your performance and revise the areas which emerge out as weak in your evaluation.

DIRECTIONS (Q.1-Q.21) : There are 21 multiple choice Q.3 In an ac circuit I = 100 sin 200pt. The time required for
questions. Each question has 4 choices (a), (b), (c) and (d), out the current to achieve its peak value will be
of which ONLY ONE choice is correct. 1 1 1 1
(a) sec (b) sec (c) sec (d) sec
Q.1 The resistance of a coil for dc is 5 ohm. In ac, the resistance 100 200 300 400
will Q.4 The frequency of ac mains in India is
(a) remain same (b) increase (a) 30 c/s or Hz (b) 50 c/s or Hz
(c) decrease (d) be zero (c) 60 c/s or Hz (d) 120 c/s or Hz
Q.5 The peak value of an alternating e.m.f. E given by
Q.2 If instantaneous current is given by i = 4 cos(wt + f)
amperes, then the r.m.s. value of current is E = E0 cos wt is 10 volts and its frequency is 50 Hz. At

(a) 4 ampere (b) 2 2 ampere 1


time t = sec, the instantaneous e.m.f. is
(c) 4 2 ampere (d) zero ampere 600
(a) 10 V (b) 5 3 V (c) 5 V (d) 1 V

RESPONSE GRID 1. 2. 3. 4. 5.
Space for Rough Work
t.me/Ebooks_Encyclopedia27. t.me/Magazines4all

EBD_7156
2 DPP/ P 46
Q.6 An alternating current is given by the equation (a) 0.32 A (b) 0.016 A
i = i1 cos wt + i2 sin wt . The r.m.s. current is given by (c) 0.48 A (d) 0.80 A
Q.13 A 20 volt ac is applied to a circuit consisting of a resistance
1 1 and a coil with negligible resistance. If the voltage across
(a) (i1 + i2 ) (b) (i1 + i2 ) 2
2 2 the resistance is 12 V , the voltage across the coil is
(a) 16 volt (b) 10 volt
1 1 2 2 1/ 2 (c) 8 volt (d) 6 volt
(c) (i12 + i22 )1/ 2 (d) (i1 + i2 )
2 2 Q.14 An alternating voltage E = 200 2 sin(100t ) is connected
Q.7 In a circuit, the value of alternating current is measured by to a 1 microfarad capacitor through an ac ammeter. The
hot wire ammeter as 10 ampere. Its peak value will be reading of the ammeter will be
(a) 10 A (b) 20 A (c) 14.14 A (d) 7.07 A (a) 10 mA (b) 20 mA
Q.8 The frequency of an alternating voltage is 50 cycles/sec (c) 40 mA (d) 80 mA
and its amplitude is 120V. Then the r.m.s. value of voltage Q.15 A resistor and a capacitor are connected in series with an
is a.c. source. If the potential drop across the capacitor is 5
(a) 101.3V (b) 84.8V (c) 70.7V (d) 56.5V V and that across resistor is 12 V, applied voltage is
Q.9 A resistance of 20W is connected to a source of an (a) 13 V (b) 17 V
alternating potential V = 220sin(100pt ) . The time taken (c) 5 V (d) 12 V
by the current to change from its peak value to r.m.s. value Q.16 A 120 volt ac source is connected across a pure inductor
is of inductance 0.70 henry. If the frequency of the ac source
(a) 0.2 sec (b) 0.25 sec is 60 Hz, the current passing through the inductor is
(c) 25 × 10–3 sec (d) 2.5 × 10–3 sec (a) 4.55 amp (b) 0.355 amp
Q.10 An alternating current of frequency f is flowing in a circuit (c) 0.455 amp (d) 3.55 amp
containing a resistor of resistance R and a choke of Q.17 The instantaneous value of current in an A.C. circuit is
inductance L in series. The impedance of this circuit is I = 2 sin (100 pt + p/3)A. The current will be maximum
for the first time at
(a) R + 2pfL (b) R 2 + 4p2 f 2 L2 1 1
(a) t = s (b) t = s
100 200
(c) R 2 + L2 (d) R 2 + 2pfL
1 1
Q.11 An alternating voltage is connected in series with a (c) t = s (d) t = s
400 600
resistance R and an inductance L. If the potential drop across
the resistance is 200 V and across the inductance is 150 V, æ 0.4 ö
Q.18 In an L – R circuit, the value of L is ç ÷ henry and the
then the applied voltage is è p ø
(a) 350 V (b) 250 V value of R is 30 ohm. If in the circuit, an alternating e.m.f.
(c) 500 V (d) 300 V of 200 volt at 50 cycles per sec is connected, the
Q.12 An inductive circuit contains resistance of 10W and an impedance of the circuit and current will be
inductance of 20 H. If an ac voltage of 120 V and frequency (a) 11.4W,17.5A (b) 30.7 W,6.5A
60 Hz is applied to this circuit, the current would be nearly
(c) 40.4W,5A (d) 50W, 4A

6. 7. 8. 9. 10.
RESPONSE
11. 12. 13. 14. 15.
GRID
16. 17. 18.

Space for Rough Work


t.me/Ebooks_Encyclopedia27. t.me/Magazines4all

DPP/ P 46 3
Q.19 The voltage across a pure inductor is represented by the DIRECTIONS (Q.22-Q.23) : In the following questions,
following diagram. Which one of the following diagrams more than one of the answers given are correct. Select the
will represent the current? correct answers and mark it according to the following
V codes:
Codes :
(a) 1, 2 and 3 are correct
t (b) 1 and 2 are correct
(c) 2 and 4 are correct
(d) 1 and 3 are correct
Q.22 If an alternating voltage is represented as E = 141 sin
Ai Ai
(628t), then
(1) the rms voltage is 141V
(a) (b) (2) the rms voltage is 100V
t t
(3) the frequency is 50 Hz
(4) the frequency is 100 Hz
Q.23 The r.m.s. value of an ac of 50 Hz is 10 A.
Ai Ai (1) The time taken by the alternating current in reaching
from zero to maximum value is 5×10–3 sec
(c) (d) (2) The time taken by the alternating current in reaching
t t from zero to maximum value is 2×10–3 sec
(3) The peak current is 14.14 A
(4) The peak current is 7.07 A
Q.20 One 10 V, 60 W bulb is to be connected to 100 V line. The
required induction coil has self-inductance of value DIRECTIONS (Q.24-Q.26) : Read the passage given below
( f = 50 Hz) and answer the questions that follows :
(a) 0.052 H (b) 2.42 H If the voltage in an ac circuit is represented by the equation,
(c) 16.2 mH (d) 1.62 mH
V = 220 2 sin (314t – f), then
1
Q.21 A resistance of 300W and an inductance of henry are Q.24 RMS value of the voltage is
p (a) 220 V (b) 314 V
connected in series to a ac voltage of 20 volt and 200 Hz (c) 220 2 V (d) 200 / 2 V
frequency. The phase angle between the voltage and current Q.25 Average voltage is
is (a) 220 V (b) 622/pV
-1 4 -1 3 (c) 220 2 V (d) 200 / 2 V
(a) tan (b) tan
3 4 Q.26 Frequency of ac is
-1 3 -1 2 (a) 50 Hz (b) 50 2 Hz
(c) tan (d) tan
2 5 (c) 50 2 Hz (d) 75 Hz

RESPONSE 19. 20. 21. 22. 23.


GRID 24. 25. 26.

Space for Rough Work


t.me/Ebooks_Encyclopedia27. t.me/Magazines4all

EBD_7156
4 DPP/ P 46
DIRECTIONS (Q. 27-Q.29) : Each of these questions contains Q27 Statement-1: The alternating current lags behind the em.f.
two statements: Statement-1 (Assertion) and Statement-2 by a phase angle of p/2, when ac flows through an inductor.
(Reason). Each of these questions has four alternative choices, Statement-2 : The inductive reactance increases as the
only one of which is the correct answer. You have to select the frequency of ac source decreases.
correct choice. Q.28 Statement-1 : An alternating current does not show any
magnetic effect.
(a) Statement-1 is True, Statement-2 is True; Statement-2 is a Statement-2: Alternating current varies with time.
correct explanation for Statement-1. Q.29 Statement-1 : A capacitor of suitable capacitance can be
(b) Statement-1 is True, Statement-2 is True; Statement-2 is used in an ac circuit in place of the choke coil.
NOT a correct explanation for Statement-1. Statement-2 : A capacitor blocks dc and allows ac only.
(c) Statement -1 is False, Statement-2 is True.
(d) Statement -1 is True, Statement-2 is False.

RESPONSE GRID 27. 28. 29.

DAILY PRA CTICE PROBLEM SHEET 46 - PHYSICS


Total Questions 29 Total Marks 116
Attempted Correct
Incorrect N et Score
Cut-off Score 26 Qualifying Score 44
Success Gap = Net Score – Qualifying Score
Net Score = (Correct × 4) – (Incorrect × 1)

Space for Rough Work


t.me/Ebooks_Encyclopedia27. t.me/Magazines4all

DPP - Daily Practice Problems


Name : Date :

Start Time : End Time :

SYLLABUS : ALTERNATING CURRENT - 2 (LCR series circuit, resonance, quality factor, power in AC circuits,
47
wattless and power current)
Max. Marks : 116 Time : 60 min.
GENERAL INSTRUCTIONS
• The Daily Practice Problem Sheet contains 29 MCQ's. For each question only one option is correct. Darken the correct
circle/ bubble in the Response Grid provided on each page.
• You have to evaluate your Response Grids yourself with the help of solution booklet.
• Each correct answer will get you 4 marks and 1 mark shall be deduced for each incorrect answer. No mark will be given/
deducted if no bubble is filled. Keep a timer in front of you and stop immediately at the end of 60 min.
• The sheet follows a particular syllabus. Do not attempt the sheet before you have completed your preparation for that
syllabus. Refer syllabus sheet in the starting of the book for the syllabus of all the DPP sheets.
• After completing the sheet check your answers with the solution booklet and complete the Result Grid. Finally spend time
to analyse your performance and revise the areas which emerge out as weak in your evaluation.

DIRECTIONS (Q.1-Q.20) : There are 20 multiple choice 40 volt respectively. The electromotive force of alternating
questions. Each question has 4 choices (a), (b), (c) and (d), out current source in volt is
of which ONLY ONE choice is correct. (a) 94 (b) 14 (c) 10 (d) 76
Q.4 A 10 ohm resistance, 5 mH inductance coil and 10 mF
Q.1 In a series LCR circuit capacitance is changed from C to 2 capacitor are joined in series. When a suitable frequency
C. For the resonant frequency to remain unchanged, the alternating current source is joined to this combination,
inductance would be changed from L to the circuit resonates. If the resistance is halved, the
(a) L/2 (b) 2 L (c) 4 L (d) L/4 resonance frequency
Q.2 The power factor of LCR circuit at resonance is (a) is halved (b) is doubled
(a) 0.707 (b) 1 (c) Zero (d) 0.5 (c) remains unchanged (d) is quadrupled
Q.3 An alternating current source of frequency 100 Hz is Q.5 The phase difference between the current and voltage of
joined to a combination of a resistance, a capacitance and LCR circuit in series combination at resonance is
a inductance in series. The potential difference across the (a) 0° (b) p/2
inductance, the resistance and the capacitor is 46, 8 and (c) p (d) – p

RESPONSE GRID 1. 2. 3. 4. 5.
Space for Rough Work
t.me/Ebooks_Encyclopedia27. t.me/Magazines4all

EBD_7156
2 DPP/ P 47
Q.6 The coefficient of induction of a choke coil is 0.1H and Q.12 For the series LCR circuit shown in the figure, what is the
resistance is 12W. If it is connected to an alternating current resonance frequency and the amplitude of the current at
source of frequen cy 60 Hz, then power factor is the resonating frequency
approximately 8 mH
(a) 0.4 (b) 0.30 (c) 0.2 (d) 0.1
Q.7 The resonant frequency of a circuit is f. If the capacitance
is made 4 times the initial values, then the resonant 220 V 20µF
frequency will become
(a) f /2 (b) 2f (c) f (d) f/4
Q.8 In the non-resonant circuit, what will be the nature of the
44 W
circuit for frequencies higher than the resonant frequency?
(a) Resistive (b) Capacitive (a) 2500 rad s -1 and 5 2A (b) 2500rad s -1 and 5A
(c) Inductive (d) None of the above 5
(c) 2500rad s -1 and A (d) 25rad s -1 and 5 2A
Q.9 In a series LCR circuit, resistance R = 10W and the impedance 2
Z = 20W . The phase difference between the current and the Q.13 In an ac circuit, V and I are given by V = 100sin(100 t ) volt,
voltage is æ pö
(a) 30° (b) 45° (c) 60° (d) 90° I = sin ç 100t + ÷ mA . The average power dissipated in
è 3ø
æ 1 ö circuit is
Q.10 An alternating e.m.f. of frequency n ç = is
è 2 p LC ÷ø (a) 104 watt (b) 10 watt
applied to a series LCR circuit. For this frequency of the (c) 0.025 watt (d) 2.5 watt
applied e.m.f. Q.14 For a series LCR circuit R = XL = 2XC. The impedance of
(a) The circuit is at resonance and its impedance is made the circuit and phase difference between V and I
up only of a reactive part respectively will be
(b) The current in the circuit is not in phase with the 5R 5R
(a) , tan -1 (2) (b) , tan -1 (1 / 2)
applied e.m.f. and the voltage across R equals this 2 2
applied emf (c) 5X C , tan -1 (2) (d) 5R, tan -1 (1/ 2)
(c) The sum of the p.d.’s across the inductance and
æ pö
capacitance equals the applied e.m.f. which is 180° Q.15 If a current I given by I0 sin ç wt - ÷ flows in an ac circuit
ahead of phase of the current in the circuit è 2ø
across which an ac potential of E = E0 sin wt has been
(d) The quality factor of the circuit is wL / R or 1/ wCR
and this is a measure of the voltage magnification applied, then the average power consumption P in the circuit
(produced by the circuit at resonance) as well as the will be
E I
sharpness of resonance of the circuit (a) P = 0 0 (b) P = 2 E0l0
Q.11 In a circuit L,C and R are connected in series with an 2
alternating voltage source of frequency f. The current leads (c) P = E0 I 0 (d) P = 0
the voltage by 45°. The value of C is 2
1 1 Q.16 An ac supply gives 30V r.m.s. which passes through a 10 W
(a) 2pf (2pfL + R ) (b) resistance. The power dissipated in it is
pf (2pfL + R )
1 1 (a) 90 2 W (b) 90 W (c) 45 2 W (d) 45 W
(c) (d)
2pf (2pfL - R ) pf (2pfL - R)

6. 7. 8. 9. 10.
RESPONSE
11. 12. 13. 14. 15.
GRID
16.

Space for Rough Work


t.me/Ebooks_Encyclopedia27. t.me/Magazines4all

DPP/ P 47 3

Q.17 The figure shows variation of R,XL and X C with frequency Q.21 For series LCR circuit, correct statements are
(1) Applied e.m.f. and potential difference across
f in a series L, C , R circuit. Then for what frequency resistance may be in phase
point, the circuit is inductive (2) Applied e.m.f. and potential difference at inductor coil
XC XL
have phase difference of p/2
(3) Potential difference across resitance and capacitor
have phase difference of p/2
(4) Potential difference at capacitor and inductor have
R phase difference of p/2
Q.22 An ac source is connected to a resistive circuits. Which
A B C
f of the following statements are false?
(a) A (b) B (c) C (d) All points (1) Current leads the voltage
Q.18 An alternating e.m.f. of angular frequency w is applied (2) Current lags behind the voltage
across an inductance. The instantaneous power developed (3) Any of (1) or (2) may be true depending upon the value
in the circuit has an angular frequency of resistance
w w (4) Current and voltage are in same phase
(a) (b) (c) w (d) 2w Q.23 A series LCR arrangement with XL = 80 W, XC = 50 W, R = 40 W
4 2
Q.19 In the circuit shown in figure neglecting source resistance is applied across a.c. source of 200 V. Choose the correct
the voltmeter and ammeter reading will respectively, be options.
(1) Wattless current = 3.2 A
V (2) Power current = 3.2 A
(3) Power factor = 0.6
(4) Impedance of circuit = 50 W
R = 30W X C = 25W
A X L = 25W
DIRECTIONS (Q.24-Q.26) : Read the passage given below
and answer the questions that follows :

240 V A student constructs a series RLC circuit. While operating the


(a) 0V ,3 A (b) 150V ,3 A circuit at a frequency f she uses an AC voltmeter and measures
(c) 150V , 6 A (d) 0V ,8 A the potential difference across each device as (DVR) = 8.8 V,
(DVL) = 2.6V and (DVC) = 7.4V.
Q.20 In an LCR circuit, the sharpness of resonance depends on
Q.24 The circuit is constructed so that the inductor is next to
(a) Inductance (L) (b) Capacitance (C)
the capacitor. What result should the student expect for
(c) Resistance (R) (d) All of these
a measurement of the combined potential difference
DIRECTIONS (Q.21-Q.23) : In the following questions, (DVL + DVC) across the inductor and capacitor ?
more than one of the answers given are correct. Select the (a) 10.0 V (b) 7.8 V
correct answers and mark it according to the following (c) 7.4 V (d) 4.8 V
codes: Q.25 What result should the student expect for a measurement of
Codes : the amplitude Em of the potential difference across the power
(a) 1, 2 and 3 are correct (b) 1 and 2 are correct supply ?
(c) 2 and 4 are correct (d) 1 and 3 are correct (a) 18.8 V (b) 13.6 V (c) 10.0 V (d) 4.0 V

RESPONSE 17. 18. 19. 20. 21.


GRID 22. 23. 24. 25.

Space for Rough Work


t.me/Ebooks_Encyclopedia27. t.me/Magazines4all

EBD_7156
4 DPP/ P 47
Q.26 What will happen to the value of (DVL) if the frequency is (b) Statement-1 is True, Statement-2 is True; Statement-2 is
adjusted to increase the current through the circuit? NOT a correct explanation for Statement-1.
(a) (DVL) will increase. (c) Statement -1 is False, Statement-2 is True.
(b) (DVL) will decrease. (d) Statement -1 is True, Statement-2 is False.
(c) (DVL) will remain the same regardless of any changes Q.27 Statement-1 : For an electric lamp connected in series
to f. with a variable capacitor and ac source, its brightness
(d) There is not enough information to answer the increases with increase in capacitance.
question. Statement-2 : Capacitive reactance decreases with
increase in capacitance of capacitor.
DIRECTIONS (Q. 27-Q.29) : Each of these questions contains
Q.28 Statement-1 : When capacitive reactance is smaller than
two statements: Statement-1 (Assertion) and Statement-2
the inductive reactance in LCR current, e.m.f. leads the
(Reason). Each of these questions has four alternative choices,
only one of which is the correct answer. You have to select the current.
correct choice. Statement-2 : The phase angle is the angle between the
alternating e.m.f. and alternating current of the circuit.
(a) Statement-1 is True, Statement-2 is True; Statement-2 is a Q.29 Statement-1 : Choke coil is preferred over a resistor to
correct explanation for Statement-1. adjust current in an ac circuit.
Statement-2 : Power factor for inductance is zero.

RESPONSE GRID 26. 27. 27. 28. 29.

DAILY PRA CTICE PROBLEM SHEET 47 - PHYSICS


Total Questions 29 Total Marks 116
Attempted Correct
Incorrect N et Score
Cut-off Score 26 Qualifying Score 46
Success Gap = Net Score – Qualifying Score
Net Score = (Correct × 4) – (Incorrect × 1)

Space for Rough Work


t.me/Ebooks_Encyclopedia27. t.me/Magazines4all

DPP - Daily Practice Problems


Name : Date :

Start Time : End Time :

SYLLABUS : EM Waves
48
Max. Marks : 116 Time : 60 min.
GENERAL INSTRUCTIONS
• The Daily Practice Problem Sheet contains 29 MCQ's. For each question only one option is correct. Darken the correct
circle/ bubble in the Response Grid provided on each page.
• You have to evaluate your Response Grids yourself with the help of solution booklet.
• Each correct answer will get you 4 marks and 1 mark shall be deduced for each incorrect answer. No mark will be given/
deducted if no bubble is filled. Keep a timer in front of you and stop immediately at the end of 60 min.
• The sheet follows a particular syllabus. Do not attempt the sheet before you have completed your preparation for that
syllabus. Refer syllabus sheet in the starting of the book for the syllabus of all the DPP sheets.
• After completing the sheet check your answers with the solution booklet and complete the Result Grid. Finally spend time
to analyse your performance and revise the areas which emerge out as weak in your evaluation.

DIRECTIONS (Q.1-Q.20) : There are 20 multiple choice Q.3 Which of the following radiations has the least wavelength?
questions. Each question has 4 choices (a), (b), (c) and (d), out (a) g-rays (b) b-rays
of which ONLY ONE choice is correct. (c) a-rays (d) X -rays
Q.1 Light is an electromagnetic wave. Its speed in vacuum is Q.4 A parallel plate capacitor with plate area A and seperation
given by the expression between the plates d, is charged by a constant current i.
Consider a plane surface of area A/4 parallel to the plates
mo eo 1
(a) m o eo (b) (c) (d) and drawn symetrically between the plates, what is the
eo mo m o eo displacement current through this area?
Q.2 The range of wavelength of the visible light is (a) i (b) 2 i (c) i /4 (d) i / 2
(a) 10 Å to 100 Å Q.5 The charging current for a capacitor is 1 A, then the
(b) 4,000 Å to 8,000 Å displacement current is
(c) 8,000 Å to 10,000 Å (a) 1 A (b) 0.5 A
(d) 10,000 Å to 15000 Å (c) 0 (d) 2 A

RESPONSE GRID 1. 2. 3. 4. 5.

Space for Rough Work


t.me/Ebooks_Encyclopedia27. t.me/Magazines4all

EBD_7156
2 DPP/ P 48
r r Q.12 Elecromagnetic waves travel in a medium with a speed of
Q.6 If E and B be the electric and magnetic field of E.M.
wave then the direction of propogation of E.M. wave is 2 × 108 m/s. The relative permeability of the medium is 1.
along the direction. What is the relative permittivity of the medium ?
r r (a) 2.25 (b) 1.25 (c) 3.25 (d) 0.25
(a) E (b) B
r r r r Q.13 A magnetic field of a plane electromagnetic wave is given
(c) E × B (d) B × E by By = 2 × 10–7 sin(0.5 × 103x + 1.5 × 1011t) T. Fequency
Q.7 Which of the following pairs of space and time varying E of the wave is
and B fields would generate a plane electromagnetic wave (a) 23.9 Hz (b) 13.9 Hz
travelling in (–Z) direction ? (c) 33.9 Hz (d) 12.9 Hz
(a) Ex, By (b) Ey, Bx Q.14 The electric field of a plane electromagnetic wave
(c) Ez, By (d) Ey, Bz in vacuum is represented by
Q.8 Choose the wrong statement for E.M. wave. They- Ex = 0, Ey = 0.5 cos[2p × 108 (t – x/c)] and Ez = 0.
(a) are transverse Determine the wavelength of the wave.
(b) travel in vacuum with the speed of light (a) 4 m (b) 5 m (c) 3 m (d) 6 m
(c) are produced by accelerated charges Q.15 A light beam travelling in the X-direction is described by
(d) travel with same speed in all medium the electric field Ey = (300 V/m)sinw(t – x/c). An electron
is constrained to move along the Y-direction with a speed
Q.9 The intensity of light from a source is 500/p W/m2. Find
of 2.0 × 107 m/s. Find the maximum magnetic force (in N)
the amplitude of electric field in this wave-
on the electron.
(a) 3 × 102 N/C (b) 2 3 × 102 N/C (a) 3.2 × 10–18 (b) 5.1 × 10–16
(c) 6.5 × 10–11 (d) 7.8 × 10–12
3
(c) × 102 N/C (d) 2 3 ×101 N/C Q.16 Which of the following waves have minimum frequency ?
2
(a) Microwaves (b) Audible waves
Q.10 A point source of 2 watt is radiating uniformly in all
direction in vacuum. Find the amplitude of electric field (c) Ultrasonic waves (d) Radiowaves
at a distance 2m from it- Q.17 Electromagnetic waves travel in a medium which has
(a) 3 × 10–4 (b) relative permeability 1.3 and relative permittivity 2.14.
30
Then the speed of the electromagnetic wave in the medium
(c) 3 × 10–4 (d) 3 × 10–2 will be
Q.11 In a EM wave the amplitude of electric field is 10 V/m. (a) 13.6 × 106 m/s (b) 1.8 × 102 m/s
The frequency of wave is 5 × 10 4 Hz. The wave is (c) 3.6 × 108 m/s (d) 1.8 × 108 m/s
propagating along Z-axis. Then the average energy density Q.18 If lv, lx and lm represent the wavelength of visible light
of magnetic field is- x-rays and microwaves respectively, then
(a) 2.21 × 10–10 J/m3 (b) 2.21 × 10–8 J/m3 (a) lm > lx > lv (b) lv > lm > lx
(c) 2 × 10–8 J/m3 (d) 2 × 10–10 J/m3 (c) lm > lv > lx (d) lv > lx > lm

6. 7. 8. 9. 10.
RESPONSE
11. 12. 13. 14. 15.
GRID
16. 17. 18.

Space for Rough Work


t.me/Ebooks_Encyclopedia27. t.me/Magazines4all

DPP/ P 48 3
Q.19 Light wave is travelling along + y-direction. If the Q.23 An electromagnetic wave of frequency n = 3.0 MHz passes
r from vacuum into a dielectric medium with permitivity
corresponding E vector at that time is along + x-direction,
r e = 4.0. Then the wrong statements are
B vector must be directed along.
y (1) Wavelength is doubled and the frequency remains
(a) y-axis
unchanged
(b) x-axis (2) Wavelength is doubled and frequency becomes half
x
(c) + z-axis (3) Wavelength and frequency both remain unchanged
z
(d) – z axis (4) Wavelength is halved and frequency remains
Q.20 A wave is propagating in a medium of dielectric constant 2 unchanged
and relative magnetic permeability 50. The wave impedance DIRECTIONS (Q.24-Q.26) : Read the passage given below
of such a medium is and answer the questions that follows :
(a) 5W (b) 376.6W
The electron density of a layer of ionosphere at a height 150
(c) 1883 W (d) 3776 W km from the earth surface is 9 ×1010 per m3. For the sky wave
transmission from this layer upto a range of 250 km, find
DIRECTIONS (Q.21-Q.23) : In the following questions,
Q.24 The critical frequency of the layer
more than one of the answers given are correct. Select the
correct answers and mark it according to the following (a) 2.7 × 106 Hz (b) 2.7 × 105 Hz
codes: (c) 4.7 × 106 Hz (d) 4.8 × 105Hz
Q.25 The maximum usuable frequency
Codes :
(a) 3.17 × 108 Hz (b) 3.17 × 106 Hz
(a) 1, 2 and 3 are correct
(c) 4.57 × 106 Hz (d) 4.57 × 106Hz
(b) 1 and 2 are correct
Q.26 The angle of incidence of this layer
(c) 2 and 4 are correct
(a) 34.5° (b) 25.2°
(d) 1 and 3 are correct
(c) 31.6° (d) 40°
Q.21 Which of the following statements are true ?
(1) Photographic plates are sensitive to ultraviolet rays. DIRECTIONS (Q. 27-Q.29) : Each of these questions contains
(2) Photographic plates can be made sensitive to infrared two statements: Statement-1 (Assertion) and Statement-2
rays. (Reason). Each of these questions has four alternative choices,
only one of which is the correct answer. You have to select the
(3) Infrared rays are emitted by hot objects. correct choice.
(4) Infrared photon has more energy than the photon of (a) Statement-1 is True, Statement-2 is True; Statement-2 is a
visible light. correct explanation for Statement-1.
Q.22 Which of the following are electromagnetic waves ? (b) Statement-1 is True, Statement-2 is True; Statement-2 is
(1) Cosmic rays (2) Gamma rays NOT a correct explanation for Statement-1.
(3) X-rays (4) b- rays (c) Statement -1 is False, Statement-2 is True.
(d) Statement -1 is True, Statement-2 is False.

RESPONSE 19. 20. 21. 22. 23.


GRID 24. 25. 26.

Space for Rough Work


t.me/Ebooks_Encyclopedia27. t.me/Magazines4all

EBD_7156
4 DPP/ P 48
Q.27 Statement-1: The electromagnetic waves of shorter Q.28 Statement-1: Ultraviolet radiation are of higher frequency
wavelength can travel longer distances on earth's surface waves and are dangerous to human being.
than those of longer wavelengths. Statement-2: Ultraviolet radiation are absorbed by the
Statement-2: Shorter the wavelength, the larger is the atmosphere.
velocity of wave propagation. Q.29 Statement-1: Radio waves can be polarised.
Statement-2: Sound waves in air are longitudinal in nature.

RESPONSE GRID 27. 28. 29.

DAILY PRA CTICE PROBLEM SHEET 48 - PHYSICS


Total Questions 29 Total Marks 116
Attempted Correct
Incorrect N et Score
Cut-off Score 30 Qualifying Score 48
Success Gap = Net Score – Qualifying Score
Net Score = (Correct × 4) – (Incorrect × 1)

Space for Rough Work


t.me/Ebooks_Encyclopedia27. t.me/Magazines4all

DPP - Daily Practice Problems


Name : Date :

Start Time : End Time :

SYLLABUS : RAY OPTICS-1 (Reflection on plane mirrors and curved mirrors)


49
Max. Marks : 120 Time : 60 min.
GENERAL INSTRUCTIONS
• The Daily Practice Problem Sheet contains 30 MCQ's. For each question only one option is correct. Darken the correct
circle/ bubble in the Response Grid provided on each page.
• You have to evaluate your Response Grids yourself with the help of solution booklet.
• Each correct answer will get you 4 marks and 1 mark shall be deduced for each incorrect answer. No mark will be given/
deducted if no bubble is filled. Keep a timer in front of you and stop immediately at the end of 60 min.
• The sheet follows a particular syllabus. Do not attempt the sheet before you have completed your preparation for that
syllabus. Refer syllabus sheet in the starting of the book for the syllabus of all the DPP sheets.
• After completing the sheet check your answers with the solution booklet and complete the Result Grid. Finally spend time
to analyse your performance and revise the areas which emerge out as weak in your evaluation.

DIRECTIONS (Q.1-Q.22) : There are 22 multiple choice (a) 3 (b) 6 (c) 1 (d) 7
questions. Each question has 4 choices (a), (b), (c) and (d), out Q.4 Two plane mirrors are inclined at an angle q. A ray of light
of which ONLY ONE choice is correct. is incident on one mirror at an angle of incidence i. The
Q.1 Find the number of images formed by two mutually ray is reflected from this mirror, falls on the second mir-
perpendicular mirrors – ror from where it is reflected parallel to the first mirror.
What is the value of i, the angle of incidence in terms q ?
(a) 3 (b) 4 (c) 1 (d) 2
(a) 2q - 90º (b) 4q - 90º
Q.2 The angle q between two plane mirrors producing five
images of a given object is given by. (c) q - 90º (d) 3q - 90º
(a) 30º £ q £ 72º (b) 45º £ q £ 72º Q.5 A girl stands at a distance 30 cm from the mirror. She is
able to see her erect image but of 1/5 height of actual
(c) 60º £ q £ 72º (d) 15º £ q £ 72º
height. The mirror will be :
Q.3 Two mirrors are inclined at an angle of 50º. Then what is
the number of images formed for an object placed in be- (a) plane mirror (b) concave mirror
tween the mirrors ? (c) convex mirror (d) plane convex mirror

RESPONSE GRID 1. 2. 3. 4. 5.
Space for Rough Work
t.me/Ebooks_Encyclopedia27. t.me/Magazines4all

EBD_7156
2 DPP/ P 49
Q.6 An object is placed at a distance of 50 cm from a convex Q.11 A 0.5 cm high object is placed at 30 cm from a convex
mirror. A plane mirror is placed in front of the convex mirror whose focal length is 20 cm. Find the position and
mirror in such a way that it convers half of the convex size of the image.
mirror. If the distance between object and plane mirror is (a) 12 cm, 0.2 cm (b) 18 cm, 0.2 cm
30 cm then there is no parallax between the images formed
(c) 6 cm, 0.5 cm (d) 5 cm, 0.1 cm
by two mirrors , the radius of curvature of convex mirror
will be : Q.12 There is a convex mirror of radius 50 cm. The image of a
point at a distance 50 cm from the pole of mirror on its
(a) 50 cm (b) 25 cm (c) 12.5 cm (d) 100cm axis will be formed at :
//
Q.7 Two plane mirrors are inclined at an angle of 30º. Then the (a) infinity ////

//// ////
O· ·C
first four images of an object O placed between the two (b) pole 50 cm 50 cm

////
mirrors are correctly represented by /// ////
(c) focus
M1 I1 M1 (d) 16.67 cm behind the mirror
I1 I4
I4
Q.13 A particle is moving at a constant speed v0 from a large
distance towards a concave mirror of radius R along its
(a) M2 (b) M2 principle axis. Find the speed of the image formed by the
I2 I2 I3 mirror as a function of the distance u of the particles from
I3
the mirror.
I3 M1 I1 I3 2 2
M1 æ R ö æ R ö
(a) ç
è 2u - R ÷ø 0
.v (b) ç
è 2u + R ÷ø 0
I1 .v

(c) M2 (d) M2 2
I2 æ 2R ö
(c) ç
è 2u - R ÷ø 0
.v (d) None of these
I4 I2 I4
Q.8 The plane of a mirror makes an angle of 30º with horizontal. Q.14 A short linear object of length b lies along the axis of a
If a vertical ray is incident on a mirror, then what is the concave mirror of focal length f at a distance u from the
angle between mirror and reflected ray ? pole of the mirror. Find the approximate size of the image.
2 2
(a) 60º (b) 90º (c) 45º (d) 30º æ f ö æ f ö
(a) b ç (b) b ç
è u - f ÷ø è u + f ÷ø
Q.9 Two plane mirrors are placed at an angle a so that a ray paral-
2
lel to one mirror gets reflected parallel to the second mirror æ 2f ö
(c) b ç (d) None of these
after two consecutive reflections. The value of a will be è u - f ÷ø
(a) 30º (b) 60º (c) 75º (d) 90º Q.15 The relation between the linear magnification m, the object
Q.10 A 0.2 cm high object is placed 15 cm from a concave mirror distance u and the focal length f for a spherical mirror is
of focal length 5 cm. Find position and size of the image. f -u f
(a) m = f
(b) m = f - u
(a) 7.5 cm, 0.1 cm. (b) 7.5 cm, 0.4 cm.
f +u f
(c) 10.0 cm, 0.5 cm. (d) 7.5 cm, 0.4 cm. (c) m = f
(d) m = f + u

RESPONSE 6. 7. 8. 9. 10.
GRID 11. 12. 13. 14. 15.

Space for Rough Work


t.me/Ebooks_Encyclopedia27. t.me/Magazines4all

DPP/ P 49 3
Q.16 An object of length 1 cm is placed at a distance of 15 cm Q.21 A concave mirror of focal length f0(in magnitude) produces
from a concave mirror of focal length 10 cm. The nature a real image n time the size of the object. What is the
and size of the image are distance of the object from the mirror?
(a) real, inverted, 1.0 cm (b) real, inverted, 2.0 cm
-(n +1) (n +1) (n - 1) -(n2 +1)
(c) virtual, erect, 0.5 cm (d) virtual, erect, 1.0 cm (a) f0 (b) f0 (c) f0 (d) f0
n n n n
Q.17 In an experiment to determine the focal length
(f ) of a concave mirror by the u - v method, a student places Q.22 The focal length of a concave mirror is 30 cm. Find the
the object pin A on the principal axis at a distance x from position of the object in front of the mirror, so that the
the pole P. The student looks at the pin and its inverted image is three times the size of the object.
image from a distance keeping his/her eye in line with PA. (a) 20 cm (only) (b) 40 cm (only)
When the student shifts his/her eye towards left, the image
appears to the right of the object pin. Then, (c) 30 cm (only) (d) 20 cm or 40 cm
(a) x < f (b) f< x< 2f (c) x = 2f (d) x > 2f DIRECTIONS (Q.23-Q.25) : In the following questions,
Q.18 Two plane mirrors are inclined to each other at some angle. more than one of the answers given are correct. Select the
A ray of light incident at 30º on one, after reflection from correct answers and mark it according to the following
the other retraces its path. The angle between the mirrors codes:
is Codes :
(a) 30º (b) 45º (c) 60º (d) 90º
(a) 1, 2 and 3 are correct (b) 1 and 2 are correct
Q.19 A convex mirror is used to form the image of an object.
Which of the following statements is wrong ? (c) 2 and 4 are correct (d) 1 and 3 are correct
(a) The image lies between the pole and the focus Q.23 A plane mirror reflecting a ray of incident light is rotated
(b) The image is diminished in size through an angle q about an axis through the point of
(c) The image is erect incidence in the plane of the mirror perpendicular to the
plane of incidence, then
(d) The image is real
Q.20 A point source of light B is placed at a distance L in front (1) The reflected ray rotates through an angle 2q
of the centre of a mirror of width 'd' hung vertically on a (2) The incident ray is fixed
wall. A man walks in front of the mirror along a line parallel (3) The reflected ray does not rotate
to the mirror at a distance 2L from it as shown in fig. The
(4) The reflected ray rotates through an angle q
greatest distance over which he can see the image of the
light source in the mirror is Q.24 The light reflected by a plane mirror will not form a real
B image
d (1) If the rays incident on the mirror are diverging
(2) Under no circumstances
L
(3) If the object is real
2L
(4) If the rays incident on the mirror are converging
(a) d/2 (b) d (c) 2d (d) 3d

RESPONSE 16. 17. 18. 19. 20.


GRID 21. 22. 23. 24.

Space for Rough Work


t.me/Ebooks_Encyclopedia27. t.me/Magazines4all

EBD_7156
4 DPP/ P 49
Q.25 Which of the following form(s) a virtual and erect image DIRECTIONS (Qs. 28-Q.30) : Each of these questions contains
for all positions of the object? two statements: Statement-1 (Assertion) and Statement-2
(1) Convex lens (2) Concave lens (Reason). Each of these questions has four alternative choices,
only one of which is the correct answer. You have to select the
(3) Convex mirror (4) Concave mirror correct choice.
DIRECTIONS (Q.26-Q.27) : Read the passage given below (a) Statement-1 is True, Statement-2 is True; Statement-2 is a
and answer the questions that follows : correct explanation for Statement-1.
(b) Statement-1 is True, Statement-2 is True; Statement-2 is
A plane mirror (M1) and a concave mirror Y M2
NOT a correct explanation for Statement-1.
(M2) of focal length 10 cm are arranged as 20 cm
10 cm (c) Statement -1 is False, Statement-2 is True.
shown in figure. An object is kept at origin.
Answer the following questions. (Consider (d) Statement -1 is True, Statement-2 is False.
45°
image formed by single reflection in all O X Q.28 Statement-1: The mirrors used in search lights are
cases) M1
parabolic and not concave spherical.
Q.26 The co-ordinates of image formed by plane mirror are Statement-2: In a concave spherical mirror the image
formed is always virtual.
(a) (–20 cm, 0) (b) (10 cm, – 60 cm) Q.29 Statement-1: When an object is placed between two plane
(c) (10 cm, –10 cm) (d) (10 cm, 10 cm) parallel mirors, then all the images found are of different
intensity.
Q.27 The co-ordinates of image formed by concave mirror are
Statement-2: In case of plane parallel mirrors, only two
(a) (10 cm, – 40 cm) (b) (10 cm, – 60 cm) images are possible.
(c) (10 cm, 8 cm) (d) None of these Q.30 Statement-1: The size of the mirror doesn't affect the
nature of the image.
Statement-2: Small mirror always forms a virtual image.

RESPONSE 25. 26. 27. 28. 29.


GRID 30.

DAILY PRA CTICE PROBLEM SHEET 49 - PHYSICS


Total Questions 30 Total Marks 120
Attempted Correct
Incorrect N et Score
Cut-off Score 30 Qualifying Score 50
Success Gap = Net Score – Qualifying Score
Net Score = (Correct × 4) – (Incorrect × 1)

Space for Rough Work


t.me/Ebooks_Encyclopedia27. t.me/Magazines4all

DPP - Daily Practice Problems


Name : Date :

Start Time : End Time :

SYLLABUS : RAY OPTICS - II (Refraction on plane surface, total internal reflection, prism)
50
Max. Marks : 108 Time : 60 min.
GENERAL INSTRUCTIONS
• The Daily Practice Problem Sheet contains 27 MCQ's. For each question only one option is correct. Darken the correct
circle/ bubble in the Response Grid provided on each page.
• You have to evaluate your Response Grids yourself with the help of solution booklet.
• Each correct answer will get you 4 marks and 1 mark shall be deduced for each incorrect answer. No mark will be given/
deducted if no bubble is filled. Keep a timer in front of you and stop immediately at the end of 60 min.
• The sheet follows a particular syllabus. Do not attempt the sheet before you have completed your preparation for that
syllabus. Refer syllabus sheet in the starting of the book for the syllabus of all the DPP sheets.
• After completing the sheet check your answers with the solution booklet and complete the Result Grid. Finally spend time
to analyse your performance and revise the areas which emerge out as weak in your evaluation.

DIRECTIONS (Q.1-Q.21) : There are 21 multiple choice Q.3 Light of wavelength 6000Å enters from air into water
questions. Each question has 4 choices (a), (b), (c) and (d), out (a medium of refractive index 4/3). Find the speed and
of which ONLY ONE choice is correct. wavelength [c = 3 × 108 m/s]
(a) 2.25 × 108 m/s, 4500Å (b) 1.25 × 108 m/s, 2500Å
Q.1 A thin prism of angle A = 6° produces a deviation d = 3°.
Find the refractive index of the material of prism. (c) 3.15 × 108 m/s, 3500Å (d) 3.45 × 108 m/s, 5500Å
(a) 1.5 (b) 1.0 Q.4 A ray of light is incident on a transparent glass-slab of
(c) 2.5 (d) 0.5 refractive index 1.5. If the reflected and refracted rays are
Q.2 A ray of light is incident at an angle of 60° on one face of mutually perpendicular, what is the angle of incidence ?
a prism which has an angle of 30°. The ray emerging out of
the prism makes an angle of 30° with the incident ray. -1 2
(a) 30° (b) sin
Calculate the refractive index of the material of the prism 3
(a) 1 (b) 2 2
-1 -1 3
(c) tan (d) tan
(c) 3 (d) 2 3 2

RESPONSE GRID 1. 2. 3. 4.

Space for Rough Work


t.me/Ebooks_Encyclopedia27. t.me/Magazines4all

EBD_7156
2 DPP/ P 50
Q.5 An optical fibre consists of core of m1 surrounded by a Region I Region II Region III Region IV
cladding of m2 < m1. A beam of light enters from air at an
angle a with axis of fibre. The highest a for which ray can
be travelled through fibre is q n0 n0 n0
n0 2 6 8
(a) cos -1 m 22 - m12
m2 0 0.2 m 0.6 m
(b) sin -1
m12 - m 22 a m1 æ 3ö -1 æ 1 ö
(a) sin -1 ç ÷ (b) sin ç ÷
4
è ø è8ø
(c) tan -1 m12 - m 22 -1 æ ö
1 -1 æ 1 ö
(c) sin ç ÷ (d) sin ç ÷
è4ø è3ø
-1
(d) sec m12 - m 22 Q.10 The refractive index of the material of a prism is 2 and
Q.6 A glass plate 4 mm thick is viewed from the above through its prism angle is 30º. One of its refracting faces is polished.
a microscope. The microscope must be lowered 2.58 mm The incident beam of light will return back for the angle of
as the operator shifts from viewing the top surface to incidence
viewing the bottom surface through the glass. What is the (a) 60º (b) 45º (c) 30º (d) 0º
index of refraction of the glass ? Q.11 A ray of light incident on a prism surface at an angle of 50º
(a) 1.61 (b) 1.55 (c) 3.24 (d) 1.21 in the minimum deviation position. If the angle of prism is
Q.7 A vertical microscope is focussed on a point at the bottom 60º then the values of dm and m will be respectively –
of an empty tank. Water (m = 4/3) is then poured into the (sin 50º = 0.766)
tank. The height of the water column is 4cm. Another lighter (a) 40º and 1.532 (b) 60º and 1.532
liquid, which does not mix with water and which has (c) 90º and 1.532 (d) 0º and 1.532
refractive index 3/2 is then poured over the water.The height Q.12 A glass prism of refractive index 1.5 and angle of prism 6º
of liquid column is 2cm. What is the vertical distance is put in contact with another prism of refractive index 1.6
through which the microscope must be moved to bring the when a ray of light is made incident on this combination
object in focus again ? normally then it emerges out undeviated. The angle of
(a) 2.61 m (b) 1.55 m (c) 3.12 m (d) 1.67 m second prism will be –
Q.8 Light from a sodium lamp (l0 = 589 nm) passes through a (a) 6º (b) 5º (c) 4º (d) 3º
tank of glycerin (refractive index 1.47) 20m long in a time Q.13 A crown glass prism of angle 5° is to be combined with a
t1. If it takes a time t2 to traverse the same tank when filled flint glass prism in such a way that the mean ray passes
with carbon disulphide (index 1.63), then the difference t2 – undeviated. Find the angle of the flint glass prism needed
t1 is and the angular dispersion produced by the combination
(a) 6.67 × 10-8 sec (b) 1.09 × 10-7 sec when white light goes through it. Refractive indices for
(c) 2.07 × 10 sec -7 (d) 1.07 × 10-8 sec red, yellow and violet light are 1.514, 1.517 and 1.523
Q.9 A light beam is travelling from Region I to Region IV (Refer respectively for crown glass and 1.613, 1.620 and 1.632
Figure). The refractive index in Regions I, II, III and IV are for flint glass.
n n n (a) 4.2°, 0.0348° (b) 4.2°, 0.0138°
n0, 0 , 0 and 0 , respectively. The angle of incidence q (c) 1.2°, 0.0348° (d) 4.4°, 0.0218°
2 6 8
Q.14 Calculate the dispersive power for crown glass from the
for which the beam just misses entering Region IV is (given data : mv= 1.5230, m r =1.5145)
(a) 0.0163 (b) 0.0183 (c) 0.0142 (d) 0.0112

RESPONSE 5. 6. 7. 8. 9.
GRID 10. 11. 12. 13. 14.

Space for Rough Work


t.me/Ebooks_Encyclopedia27. t.me/Magazines4all

DPP/ P 50 3
Q.15 A prism of dispersive power 0.021 and refractive index Q.21 A prism having an apex angle 4° and refraction index 1.5
1.53 form an achromatic combination with prism of angle is located in front of a vertical plane mirror as shown in
4.2° and dispersive power 0.045 having refractive index figure. Through what total angle is the ray deviated after
1.65. Find the resultant deviation. reflection from the mirror
(a) 1.12° (b) 2.16° (c) 3.12° (d) 4.18°
Q.16 A ray of light fall normally on a refracting face of a prism 90° 4°
of refractive index 1.5. Find the angle of the prism if the
ray just fails to emerge from the prism.
(a) 55° (b) 22° (c) 12° (d) 42°
Q.17 The refractive indices of material of a prism for blue and (a) 176° (b) 4°
red colours are 1.532 and 1.514 respectively. Calculate (c) 178° (d) 2°
angular dispersion produced by the prism if angle of prism
is 8°. DIRECTIONS (Q.22-Q.24) : In the following questions,
(a) 0.144° (b) 0.122° (c) 0.133° (d) 0.111° more than one of the answers given are correct. Select the
Q.18 A ball is dropped from a height of 20 m above the surface correct answers and mark it according to the following
of water in a lake. The refractive index of water is 4/3. A codes:
fish inside the lake, in the line of fall of the ball, is looking
Codes :
at the ball. At an instant, when the ball is 12.8 m above the
(a) 1, 2 and 3 are correct (b) 1 and 2 are correct
water surface, the fish sees the speed of ball as [g = 10 m/
(c) 2 and 4 are correct (d) 1 and 3 are correct
s2]
Q.22 A ray of monochromatic light is incident on the plane
(a) 9 m/s (b) 12 m/s (c) 16 m/s (d) 21.33 m/s
surface of separation between two media x and y with angle
Q.19 The dispersive powers of crown and flint glasses are 0.03
of incidence i in the medium x and angle of refraction r in
and 0.05 respectively. The refractive indices for yellow
the medium y. The graph shows the relation between sin i
light for these glasses are 1.517 and 1.621 respectively .
and sin r.
It is desired to form an achromatic combination of prisms
of crown and flint glasses which can produce a deviation sin r
of 1° in the yellow ray. The refracting angle of the flint
glass prism is
(a) 2.4° (b) 1.4° (c) 3.4° (d) 5.2°
Q.20 A glass prism (m = 1.5) is dipped in water ( m = 4/3) as
shown in figure. A light ray is incident normally on the 30°
surface AB. It reaches the surface BC after T.I.R
sin i
B A (1) The speed of light in the medium y is 3 times than in
q
medium x
1
(2) The speed of light in the medium y is times than in
3
C medium x.
(3) The total internal reflection can take place when the
incidence is in x.
(a) sin q ³ 8/9 (b) 2/3 < sin q < 8/9
(4) The total internal reflection can take place when the
(c) sin q £ 2/3 (d) It is not possible
incidence is in y

RESPONSE 15. 16. 17. 18. 19.


GRID 20. 21. 22.

Space for Rough Work


t.me/Ebooks_Encyclopedia27. t.me/Magazines4all

EBD_7156
4 DPP/ P 50
Q.23 Dispersive power does not depend upon (b) Statement-1 is True, Statement-2 is True; Statement-2 is
(1) The shape of prism (2) Angle of prism NOT a correct explanation for Statement-1.
(3) Height of the prism (4) Material of prism
(c) Statement -1 is False, Statement-2 is True.
Q.24 The wrong statements are
(1) The order of colours in the primary and the secondary (d) Statement -1 is True, Statement-2 is False.
rainbows is the same Q.25 Statement-1: There is no dispersion of light refracted
(2) The intensity of colours in the primary and the through a rectangular glass slab.
secondary rainbows is the same
Statement-2: Dispersion of light is the phenomenon of
(3) The intensity of light in the primary rainbow is greater
and the order of colours is the same than the secondary splitting of a beam of white light into its constituent
rainbow colours.
(4) The intensity of light for different colours in primary Q.26 Statement-1: Dispersion of light occurs because velocity
rainbow is greater and the order of colours is reverse of light in a material depends upon its colour.
as that in the secondary rainbow Statement-2: The dispersive power depends only upon the
material of the prism, not upon the refracting angle of the
DIRECTIONS (Qs. 25-Q.27) : Each of these questions contains
two statements: Statement-1 (Assertion) and Statement-2 prism.
(Reason). Each of these questions has four alternative choices, Q.27 Statement-1: If a plane glass slab is placed on the letters
only one of which is the correct answer. You have to select the of different colours all the letters appear to be raised up to
correct choice. the same height.
(a) Statement-1 is True, Statement-2 is True; Statement-2 is a Statement-2: Different colours h ave differen t
correct explanation for Statement-1. wavelengths.

RESPONSE GRID 23. 24. 25. 26. 27.

DAILY PRA CTICE PROBLEM SHEET 50 - PHYSICS


Total Questions 27 Total Marks 108
Attempted Correct
Incorrect N et Score
Cut-off Score 28 Qualifying Score 46
Success Gap = Net Score – Qualifying Score
Net Score = (Correct × 4) – (Incorrect × 1)

Space for Rough Work


t.me/Ebooks_Encyclopedia27. t.me/Magazines4all

DPP - Daily Practice Problems


Name : Date :

Start Time : End Time :

SYLLABUS : RAY OPTICS - 3 (Refraction on curved surface lens, Optical instrument)


51
Max. Marks : 120 Time : 60 min.
GENERAL INSTRUCTIONS
• The Daily Practice Problem Sheet contains 30 MCQ's. For each question only one option is correct. Darken the correct
circle/ bubble in the Response Grid provided on each page.
• You have to evaluate your Response Grids yourself with the help of solution booklet.
• Each correct answer will get you 4 marks and 1 mark shall be deduced for each incorrect answer. No mark will be given/
deducted if no bubble is filled. Keep a timer in front of you and stop immediately at the end of 60 min.
• The sheet follows a particular syllabus. Do not attempt the sheet before you have completed your preparation for that
syllabus. Refer syllabus sheet in the starting of the book for the syllabus of all the DPP sheets.
• After completing the sheet check your answers with the solution booklet and complete the Result Grid. Finally spend time
to analyse your performance and revise the areas which emerge out as weak in your evaluation.

DIRECTIONS (Q.1-Q.21) : There are 21 multiple choice Q.3 In case of thin lens of focal length f an object is placed at
questions. Each question has 4 choices (a), (b), (c) and (d), out a distance x1 from first focus and its image is formed at a
of which ONLY ONE choice is correct. distance x2 from the second focus, find x1 x2
(a) f (b) f 3
Q.1 A small point object is placed at O, at a distance of 0.60
(c) f 2 (d) 1/f
metre in air from a convex spherical surface of refractive
Q.4 What is the refractive index of material of a plano-convex
index 1.5. If the radius of the curvature is 25 cm, then what
lens , if the radius of curvature of the convex surface is
is the position of the image on the principal axis ?
10 cm and focal length of the lens is 30 cm ?
(a) 4.5 m (b) 2.5 m
(a) 1/3 (b) 4/3
(c) 1.5 m (d) 5.5 m
(c) 2/3 (d) 1/4
Q.2 The radius of a glass ball is 5 cm. There is an air bubble at
Q.5 A convex lens of focal length 10.0 cm is placed in contact
1cm from the centre of the ball and refractive index of
with a convex lens of 15.0 cm focal length. What is the
glass is 1.5. The position of image viewed from surface
focal length of the combination ?
near the bubble is.
(a) 6 cm (b) 12 cm
(a) 3.63 cm (b) 4.63 cm
(c) 8 cm (d) 4 cm
(c) 2.12 cm (d) 5.12 cm

RESPONSE GRID 1. 2. 3. 4. 5.
Space for Rough Work
t.me/Ebooks_Encyclopedia27. t.me/Magazines4all

EBD_7156
2 DPP/ P 51
Q.6 A convex lens of focal length 20 cm is placed in contact (a) 25.0 cm (b) 16.7 cm (c) 50.3 cm (d) 33.3 cm
with a diverging lens of unknown focal length. The lens Q.12 A lens is placed between a source of light and a wall. It
combination acts as a converging lens and has a focal length forms images of area A1 and A2 on the wall for its two
of 30 cm. What is the focal length of the diverging lens ? different positions. The area of the source of light is
(a) – 10 cm (b) – 30 cm (c) – 60 cm (d) – 90 cm -1
A1 +A 2 é 1 1 ù
Q.7 A pin is placed 10 cm in front of a convex lens of focal (a) (b) ê + ú
length 20 cm, made of material having refractive index 1.5. 2 ë A1 A 2 û
The surface of the lens farther away from the pin is silvered 2
é A1 + A 2 ù
and has a radius of curvature 22 cm. Determine the position (c) A1A 2 (d) ê ú
of the final image. ëê 2 ûú
(a) 11 cm in front (b) 21 cm in front Q.13 A convex lens of power 4D is kept in contact with a concave
(c) 15 cm in front (d) 31 cm in front lens of power 3D, the effective power of combination will
Q.8 An image is formed on the screen by a convex lens. When be
upper half part of lens is covered with black paper, then : (a) 7D (b) 4D/3
(a) half image is formed (c) 1D (d) 3D/4
(b) full image is formed Q.14 The power of a plano-convex lens is P. If this lens is cut
(c) intensity of image will be enhanced longitudinally along its principal axis into two equal parts
(d) None of these and then they are joined as given in the figure. The power
Q.9 A convex lens is made out of a substance of 1.2 refractive of combination will be :
index. The two surfaces of lens are convex. If this lens is (a) P
placed in water whose refractive index is 1.33, it will (b) 2P
behave as : (c) P/2
(a) convergent lens (b) divergent lens (d) zero
(c) plane glass plate (d) like a prism Q.15 The plane surface of a planoconvex lens is silvered. If
Q.10 An equiconvex lens has a power of 5 diopter. If it is made radius of curved surfaceisR and refractiveindex is m, then
of glass of refractive index 1.5 then the radius of the the system behaves like a concave mirror whose radius
curvature of each surface will be will be
(a) 20 cm (b) 10 cm (c) 5 cm (d) zero R
Q.11 A convex lens when placed in the first position forms a (a) (b) Rm
m
real image of an object on a fixed screen. The distance
between the object and the screen is 75 cm. On displacing R
(c) (d) R (m – 1)
the lens from first position by 25 cm to the second position, m -1
again a real image is formed on the screen. Then the focal Q.16 A slide projector lens has a focal length 10 cm. It throws
length of the lens is an image of a 2cm × 2 cm slide on a screen 5 m from the
D = 75 cm lens. Find the size of the picture on the screen.
x = 25 cm (a) ( 98 × 98) cm2 (b) (88 × 88) cm2
(c) (64 × 64) cm 2 (d) (78 × 78) cm2
Q.17 If the focal length of a magnifier is 5 cm calculate the
Object I2 power of the lens.
(a) 20D (b) 10D
I1
First position Second position (c) 5D (d) 15D

6. 7. 8. 9. 10.
RESPONSE
11. 12. 13. 14. 15.
GRID
16. 17.

Space for Rough Work


t.me/Ebooks_Encyclopedia27. t.me/Magazines4all

DPP/ P 51 3
Q.18 In the above question, find the magnifying power of the Q.23 Resolving power of a microscope doesn't depend upon
lens for relaxed and strained eye. (1) Velocity of light used
(a) 2×, 3× (b) 5×, 6× (2) Frequency of light used
(c) 4×, 2× (d) 1×, 2× (3) Focal length of objective
Q.19 A 35 mm film is to be projected on a 20 m wide screen (4) Wavelength of light used
situated at a distance of 40 m from the film-projector. Q.24 The light gathering power of a camera lens
Calculate the focal length of projection lens. doesn't depend on
(a) 70 mm (b) 35 mm (1) Ratio of focal length and diameter
(c) 40 mm (d) 20 mm (2) Product of focal length and diameter
Q.20 In a compound microscope the objective and the eye- piece (3) Wavelength of light used
have focal lengths of 0.95 cm and 5 cm respectively, and (4) Its diameter
are kept at a distance of 20 cm. The last image is formed at
a distance of 25 cm from the eye- piece. Calculate the DIRECTIONS (Q.25-Q.27) : Read the passage given below
total magnification. and answer the questions that follows :
(a) 94 (b) 84
(c) 75 (d) 88
Q.21 A Galilean telescope consists of an objective of focal
length 12 cm and eye- piece of focal length 4 cm. What m1 m2
mw
should be the separation of the two lenses when the virtual
image of a distant object is formed at a distance of 24 cm
from the eye- piece? O
L1 L2
(a) 7.2 cm (b) 8.2 cm.
(c) 12.4 cm. (d) 2.8cm.

DIRECTIONS (Q.22-Q.24) : In the following questions, A cylindrical tube filled with water (µw = 4/3) is closed at its
more than one of the answers given are correct. Select the both ends by two silvered plano convex lenses as shown in the
correct answers and mark it according to the following figure. Refractiveindex of lenses L1 and L2 are 2.0 and 1.5 while
codes: their radii of curvature are 5 cm and 9 cm respectively. A point
object is placed somewhere at a point O on the axis of cylindrical
Codes : tube. It is found that the object and image coincide each other.
(a) 1, 2 and 3 are correct (b) 1 and 2 are correct Q.25 The position of object w.r.t lens L1 is
(c) 2 and 4 are correct (d) 1 and 3 are correct (a) 8 cm (b) 10 cm
Q.22 An astronomical telescope has an angular magnification (c) 12 cm (d) 14 cm
of magnitude 5 for distant objects. The separation between Q.26 The position of object w.r.t lens L2 is
the objective and eye- piece is 36 cm and the final image (a) 8 cm (b) 10 cm
is formed at infinity. (c) 12 cm (d) 14 cm
(1) the focal length of objective is 30 cm Q.27 The length of the cylindrical tube is
(2) the focal length of objective is 25 cm (a) 16 cm (b) 18 cm
(3) the focal length of eye piece is 6 cm
(c) 20 cm (d) 22 cm
(4) the focal length of eye piece is 12 cm

RESPONSE 18. 19. 20. 21. 22.


GRID 23. 24. 25. 26. 27.

Space for Rough Work


t.me/Ebooks_Encyclopedia27. t.me/Magazines4all

EBD_7156
4 DPP/ P 51
DIRECTIONS (Q. 28-Q.30) : Each of these questions contains Q.28 Statement-1: A double convex lens (m = 1.5) has focal
two statements: Statement-1 (Assertion) and Statement-2 length 10 cm. When the lens is immersed in water (m = 4/
(Reason). Each of these questions has four alternative choices, 3) its focal length becomes 40 cm.
only one of which is the correct answer. You have to select the 1 ml - mm æ 1 1ö
correct choice. Statement-2: = ç - ÷
f m m è R1 R2 ø
(a) Statement-1 is True, Statement-2 is True; Statement-2 is a Q.29 Statement-1: The focal length of lens changes when red
correct explanation for Statement-1. light is replaced by blue light.
(b) Statement-1 is True, Statement-2 is True; Statement-2 is Statement-2: The focal length of lens does not depend on
NOT a correct explanation for Statement-1. colour of light used.
Q.30 Statement-1: By increasing the diameter of the objective
(c) Statement -1 is False, Statement-2 is True.
of telescope, we can increase its range.
(d) Statement -1 is True, Statement-2 is False. Statement-2: The range of a telescope tells us how far
away a star of some standard brightness can be spotted by
telescope.

RESPONSE GRID 28. 29. 30.

DAILY PRA CTICE PROBLEM SHEET 51 - PHYSICS


Total Questions 30 Total Marks 120
Attempted Correct
Incorrect N et Score
Cut-off Score 30 Qualifying Score 48
Success Gap = Net Score – Qualifying Score
Net Score = (Correct × 4) – (Incorrect × 1)

Space for Rough Work


t.me/Ebooks_Encyclopedia27. t.me/Magazines4all

DPP - Daily Practice Problems


Name : Date :

Start Time : End Time :

SYLLABUS : WAVE OPTICS - I (Interference of Light)


52
Max. Marks : 116 Time : 60 min.
GENERAL INSTRUCTIONS
• The Daily Practice Problem Sheet contains 29 MCQ's. For each question only one option is correct. Darken the correct
circle/ bubble in the Response Grid provided on each page.
• You have to evaluate your Response Grids yourself with the help of solution booklet.
• Each correct answer will get you 4 marks and 1 mark shall be deduced for each incorrect answer. No mark will be given/
deducted if no bubble is filled. Keep a timer in front of you and stop immediately at the end of 60 min.
• The sheet follows a particular syllabus. Do not attempt the sheet before you have completed your preparation for that
syllabus. Refer syllabus sheet in the starting of the book for the syllabus of all the DPP sheets.
• After completing the sheet check your answers with the solution booklet and complete the Result Grid. Finally spend time
to analyse your performance and revise the areas which emerge out as weak in your evaluation.

DIRECTIONS (Q.1-Q.20) : There are 20 multiple choice nm and the interference pattern is observed on a screen
questions. Each question has 4 choices (a), (b), (c) and (d), out 1.0 m away. Find the separation between the successive
of which ONLY ONE choice is correct. bright fringes.
(a) 6.6 mm (b) 6.0 mm (c) 6 m (d) 6 cm.
Q.1 The intensity ratio of two waves is 9 : 1. These waves Q.4 In Young’s double slit experiment the two slits are
produce the event of interference. The ratio of maximum illuminated by light of wavelength 5890 Å and the angular
to minimum intensity will be separation between the fringes obtained on the screen is
(a) 1 : 9 (b) 9 : 1 (c) 1 : 4 (d) 4 : 1 0.2°. If the whole apparatus is immersed in water then the
Q.2 The equation of two light waves are y1 = 6cos wt, angular fringe width will be, if the refractive index of water
y2 = 8cos(wt + f). The ratio of maximum to minimum is 4/3?
intensities produced by the superposition of these waves (a) 0.30° (b) 0.15° (c) 15° (d) 30°
will be Q.5 The intensities of two light sources are I and 9I respectively.
(a) 49 : 1 (b) 1 : 49 If the phase difference between the waves emitted by them
(c) 1 : 7 (d) 7 : 1 is p then the resultant intensity at the point of observation
Q.3 In a Young’s double slit experiment, the separation between will be –
the slits is 0.10 mm, the wavelength of light used is 600 (a) 3I (b) 4I (c) 10I (d) 82I

RESPONSE GRID 1. 2. 3. 4. 5.
Space for Rough Work
t.me/Ebooks_Encyclopedia27. t.me/Magazines4all

EBD_7156
2 DPP/ P 52
Q.6 In Fresnel’s biprism experiment the width of 10 fringes is (a) 12 th (b) 14 th
2cm which are formed at a distance of 2 meter from the (c) 16 th (d) 18 th
slit. If the wavelength of light is 5100Å then the distance Q.12 In Young’s double slit experiment, white light is used. The
between two coherent sources will be separation between the slits is b. The screen is at a distance
(a) 5.1 × 10–4 m (b) 5.1 × 104 cm d (d > > b) from the slits. Some wavelengths are missing
(c) 5.1 × 10 mm–4 (d) 10.1 × 10–4cm exactly in front of one slit. One of these wavelengths is
Q.7 Two coherent sources of intensity ratio 1 : 4 produce an
b2 2b 2 b2 2b 2
interference pattern. The fringe visibility will be - (a) l = (b) l = (c) l = (d) l =
(a) 1 (b) 0.8 (c) 0.4 (d) 0.6 6d d 3d 3d
th
Q.13 In Fresnel’s biprism experiment distance of m bright
Q.8 When a mica sheet (m = 1.6) of thickness 7 microns is
placed in the path of one of interfering beams in the biprism fringe from zeroth order fringe will be –
experiment then the central fringe gets shifted at the lD mDl
position of seventh bright fringe. The wavelength of light (a) (2m – 1) (b)
2d d
used will be –
md lD
(a) 4000Å (b) 5000Å (c) 6000 Å (d) 7000Å (c) (d) (2m + 1)
Q.9 In Young’s double slit experiment, the distance between lD 2d
two slits is made three times then the fringe width will Q.14 Consider interference between waves from two sources
become – of Intensites I & 4I. Find intensities at points where the
(a) 9 times (b) 1/9 times (c) 3 times (d) 1/3 times p
Q.10 In the given diagram, CP represents a wavefront and AO & phase difference is .
2
BP, the corresponding two rays. Find the condition on q
(a) I (b) 5 I (c) 4 I (d) 3 I
for constructive interference at P between the ray BP and
Q.15 The width of one of the two slits in a Young’s double slit
reflected ray OP
experiment is double of the other slit. Assuming that the
O
Q R amplitude of the light coming from a slit is proportional
to slit-width. Find the ratio of the maximum to the
qq
minimum intensity in the interference pattern.
C (a) 34 : 1 (b) 9 : 1 (c) 4 : 1 (d) 16 : 1
d
Q.16 The intensity of the light coming from one of the slits in a
young’s double slit experiment is double the intensity from
A the other slit. Find the ratio of the maximum intensity to
P
B the minimum intensity in the interference fringe pattern
observed.
(a) cos q = 3l/2d (b) cos q = l/4d
(a) 9 : 1 (b) 34 : 1 (c) 4 : 1 (d) 16 : 1
(c) sec q – cosq = l/d (d) sec q – cosl = 4l/d
Q.17 Two waves originating from source S1 and S2 having zero
Q.11 In Young’s double slit experiment 10th order maximum is
phase difference and common wavelength l will show
obtained at the point of observation in the interference
completely destructive interference at a point P if
pattern for l = 7000 Å. If the source is replaced by another
(S1 P – S2 P) is-
one of wavelength 5000 Å then the order of maximum at
(a) 5l (b) 3l/4
the same point will be–
(c) 2l (d) 11l/2

6. 7. 8. 9. 10.
RESPONSE
11. 12. 13. 14. 15.
GRID
16. 17.

Space for Rough Work


t.me/Ebooks_Encyclopedia27. t.me/Magazines4all

DPP/ P 52 3
Q.18 In an interference pattern, at a point we observe the 16th (1) electron gun voltage be increased.
order maximum for l1= 6000Å. What order will be visible (2) the slits be moved away from each other.
here if the source is replaced by light of wavelength ? (3) the screen be moved closer to interfering slits.
l2 = 4800 Å. (4) electron gun voltage be decreased.
(a) 40 (b) 20 Q.23 Interference fringes were produced in Young’s double slit
(c) 10 (d) 80 experiment using light of wave length 5000 Å. When a film
Q.19 In Young’s experiment the wavelength of red light is of material 2.5 × 10–3 cm thick was placed over one of the
7.5 × 10–5 cm. and that of blue light 5.0 × 10–5 cm. The slits, the fringe pettern shifted by a distance equal to 20
value of n for which (n + 1)th blue bright band coincides fringe width. The refractive index of the material of the
with nth red bright band is- film cannot be
(a) 8 (b) 4 (1) 1.25 (2) 1.33
(c) 2 (d) 1 (3) 1.5 (4) 1.4
Q.20 In Young’s double slit experiment, carried out with light of
wavelength l = 5000Å, the distance between the slits is DIRECTIONS (Q.24-Q.26) : Read the passage given below
0.2mm and the screen is at 200 cm from the slits. The and answer the questions that follows :
central maximum is at x = 0. The third maximum will be at
In a Young’s double slit experiment a monochromatic light whose
x equal to.
wavelength is l strikes on the slits, separated by distance d, as
(a) 1.67 cm (b) 1.5 cm
shown in the figure. Refractive index of the medium between
(c) 0.5 cm (d) 5.0 cm
slits and screen varies with time t as n = n0 + kt. Here n0 and k
DIRECTIONS (Q.21-Q.23) : In the following questions, are positive constants. Position of any point P on screen is
more than one of the answers given are correct. Select the measure by its y-coordinate as shown.
correct answers and mark it according to the following P
codes:
AIR
Codes : y
2 n = (n0+ kt)
(a) 1, 2 and 3 are correct (b) 1 and 2 are correct
(c) 2 and 4 are correct (d) 1 and 3 are correct
d f O
Q.21 The Young’s double slit experiment, the ratio of intensities
of bright and dark fringes is 9. This means that l
S1
(1) The intensities of individual sources are 5 and 4 units
respectively
(2) The intensities of individual sources are 4 and 1 units D
respectively
(3) The ratio of the their amplitudes is 3 Q.24 The y co-ordinate of central maxima at any time t is
(4) The ratio of their amplitude is 2 D sin f D cos f
Q.22 In an experiment similar to Young’s experiment, (a) (b)
n0 + kt n0 + kt
interference is observed using waves associated with
electrons. The electrons are being produced in an electron D sin f D cos f
(c) 2
(d)
gun. In order to decrease the fringe width (n0 + kt ) (n0 + kt )2

RESPONSE 18. 19. 20. 21. 22.


GRID 23. 24.

Space for Rough Work


t.me/Ebooks_Encyclopedia27. t.me/Magazines4all

EBD_7156
4 DPP/ P 52
Q.25 The velocity of central maxima at any time t as a function (b) Statement-1 is True, Statement-2 is True; Statement-2 is
of time t is NOT a correct explanation for Statement-1.
–2kD sin f – kD sin f (c) Statement -1 is False, Statement-2 is True.
(a) 2
(b) (d) Statement -1 is True, Statement-2 is False.
(n0 + kt ) (n0 + kt )2
Q.27 Statement-1 : No interference pattern is detected when
–2kD sin f – kD sin f two coherent sources are infinitely close to each other.
(c) (d)
(n0 + kt ) (n0 + kt ) Statement-2 : The fringe width is directly proportional to
Q.26 If a glass plate of small thickness b is placed in front of S1. the distance between the two slits.
How should its refractive index vary with time so that Q.28 Statement-1 : In Young’s experiment, the fringe width for
central maxima is formed at O. dark fringes is same as that for white fringes.
2 d sin f 2d sin f Statement-2 : In Young’s double slit experiment performed
(a) n0 + kt + (b) n0 + kt – with a source of white light, only black and bright fringes
b b
are observed.
d sin f d sin f
(c) n0 + kt – (d) n0 + kt + Q.29 Statement-1 : In Young’s double slit experiment, the fringes
b b become indistinct if one of the slits is covered with
DIRECTIONS (Q. 27-Q.29) : Each of these questions contains cellophane paper.
two statements: Statement-1 (Assertion) and Statement-2 Statement-2 : The cellophane paper decreases the
(Reason). Each of these questions has four alternative choices, wavelength of light.
only one of which is the correct answer. You have to select the
correct choice.
(a) Statement-1 is True, Statement-2 is True; Statement-2 is a
correct explanation for Statement-1.

RESPONSE GRID 25. 26. 27. 28. 29.

DAILY PRA CTICE PROBLEM SHEET 52 - PHYSICS


Total Questions 29 Total Marks 116
Attempted Correct
Incorrect N et Score
Cut-off Score 28 Qualifying Score 46
Success Gap = Net Score – Qualifying Score
Net Score = (Correct × 4) – (Incorrect × 1)

Space for Rough Work


t.me/Ebooks_Encyclopedia27. t.me/Magazines4all

DPP - Daily Practice Problems


Name : Date :

Start Time : End Time :

SYLLABUS : WAVE OPTICS - II (Diffraction and polarisation of light)


53
Max. Marks : 120 Time : 60 min.
GENERAL INSTRUCTIONS
• The Daily Practice Problem Sheet contains 30 MCQ's. For each question only one option is correct. Darken the correct
circle/ bubble in the Response Grid provided on each page.
• You have to evaluate your Response Grids yourself with the help of solution booklet.
• Each correct answer will get you 4 marks and 1 mark shall be deduced for each incorrect answer. No mark will be given/
deducted if no bubble is filled. Keep a timer in front of you and stop immediately at the end of 60 min.
• The sheet follows a particular syllabus. Do not attempt the sheet before you have completed your preparation for that
syllabus. Refer syllabus sheet in the starting of the book for the syllabus of all the DPP sheets.
• After completing the sheet check your answers with the solution booklet and complete the Result Grid. Finally spend time
to analyse your performance and revise the areas which emerge out as weak in your evaluation.

DIRECTIONS (Q.1-Q.22) : There are 22 multiple choice Q.3 Width of slit is 0.3mm. Fraunhoffer diffraction is observed
questions. Each question has 4 choices (a), (b), (c) and (d), out in focus plane of lense of a lense of focal length 1 m. If
of which ONLY ONE choice is correct. third minima is at 5 mm distance from central maxima,
then wavelength of light is-
Q.1 The first diffraction minima due to a single slit diffraction
(a) 7000Å (b) 6500Å (c) 6000Å (d) 5000Å
is at q = 30° for a light of wavelength 5000Å. The width of
the slit is- Q.4 When a wave of wavelength 0.2 cm is made incident
(a) 5 × 10–5 cm (b) 1.0 × 10–4 cm normally on a slit of width 0.004m, then the semi-angular
(c) 2.5 × 10 cm–5 (d) 1.25 × 10–5 cm width of central maximum of diffraction pattern will be-
Q.2 Two spectral line of sodium D1 & D2 have wavelengths of (a) 60° (b) 30° (c) 90° (d) 0°
approximately 5890Å and 5896Å. A sodium lamp sends Q.5 A parallel beam of monochromatic light is incident on a
incident plane wave on to a slit of width 2 micrometre. A narrow rectangular slit of width 1mm. When the diffraction
screen is located at 2m from the slit. Find the spacing pattern is seen on a screen placed at a distance of 2m. the
between the first maxima of two sodium lines as measured width of principal maxima is found to be 2.5 mm. The wave
on the screen. length of light is-
(a) 10–4 m (b) 9 × 10–4 m (a) 6250 nm (b) 6200 nm
(c) 9 × 10 m 4 (d) None (c) 5890 nm (d) 6000 nm

RESPONSE GRID 1. 2. 3. 4. 5.
Space for Rough Work
t.me/Ebooks_Encyclopedia27. t.me/Magazines4all

EBD_7156
2 DPP/ P 53
Q.6 Light of wavelength 6328Å is incident normally on slit Q.12 Image of sun formed due to reflection at air water interface
having a width of 0.2 mm. The width of the central is found to be very highly polarised. Refractive index of
maximum measured from minimum to minimum of water being m = 4/3, find the angle of sun above the horizon.
diffraction pattern on a screen 9.0 meters away will be
about - (a) 36.9º (b) 26.9º
(a) 0.36° (b) 0.18° (c) 16.9º (d) 46.9º
(c) 0.72° (d) 0.09° Q.13 When light of a certain wavelength is incident on a plane
Q.7 A screen is placed 2m away from the single narrow slit. surface of a material at a glancing angle 30º, the reflected
Calculate the slit width if the first minimum lies 5mm on light is found to be completely plane polarised. Determine
either side of the central maximum. Incident plane waves refractive index of given material –
have a wavelenght of 5000Å.
(a) 2 × 10–4 m (b) 2 × 10–3 cm (a) 3 (b) 2
(c) 2 × 10 cm–4 (d) None
(c) 1 / 2 (d) 2
Q.8 Red light of wavelength 6500Å from a distant source falls
on a slit 0.5 mm wide. What is the distance between two Q.14 Two polaroids are oriented with their planes perpendicular
dark bands on each side of central bright band of diffraction to incident light and transmission axis making an angle of
pattern observed on a screen placed 1.8 m from the slit. 30º with each other. What fraction of incident unpolarised
(a) 4.68 × 10–3 cm (b) 4.68 × 10–3 mm light is transmitted ?
(c) 4.68 × 10 nm–3 (d) 4.68 × 10–3 m (a) 57.5 % (b) 17.5 %
Q.9 Fraunhoffer diffraction pattern is observed at a distance (c) 27.5 % (d) 37.5 %
of 2m on screen, when a plane-wavefront of 6000Å is
incident perpendicularly on 0.2 mm wide slit.Width of Q.15 Unpolarised light of intensity 32 Wm–2 passes through
central maxima is: three polarisers such that the transmission axis of the last
polariser is crossed with the first. If the intensity of the
(a) 10 mm (b) 6 mm
emerging light is 3 Wm –2 . At what angle will the
(c) 12 mm (d) None of these
transmitted intensity be maximum ?
Q.10 A diffraction pattern is produced by a single slit of width
0.5mm with the help of a convex lens of focal length 40cm. (a) 45º (b) 15º
If the wavelength of light used is 5896Å. then the distance (c) 35º (d) 75º
of first dark fringe from the axis will be-
Q.16 V0 and VE represent the velocities, m0 and mE the refractive
(a) 0.047 cm (b) 0.047 m indices of ordinary and extraordinary rays for a doubly
(c) 0.047 mm (d) 47 cm refracting crystal. Then
Q.11 What should be the size of the aperture of the objective of
(a) V0 ³ VE, m0 £ mE if the crystal is calcite
telescope which can just resolve the two stars of angular
width of 10–3 degree by light of wavelength 5000Å? (b) V0 £ VE, m0 £ mE if the crystal is quartz
(a) 3.5 cm (b) 3.5 mm (c) V0 £ VE, m0 ³ mE if the crystal is calcite
(c) 3.5 m (d) 3.5 km (d) V0 ³ VE, m0 ³ mE if the crystal is quartz

6. 7. 8. 9. 10.
RESPONSE
11. 12. 13. 14. 15.
GRID
16.

Space for Rough Work


t.me/Ebooks_Encyclopedia27. t.me/Magazines4all

DPP/ P 53 3
Q.17 A ray of light is incident on the surface of a glass plate at DIRECTIONS (Q.23-Q.25) : In the following questions,
an angle of incidence equal to Brewster’s angle f. If m more than one of the answers given are correct. Select the
represents the refractive index of glass with respect to air. correct answers and mark it according to the following
then the angle between reflected and refracted rays is codes:
(a) 90° + f (b) sin–1 (m cosf) Codes :
(c) 90° (d) 90° – sin–1 ( cosf/m)
(a) 1, 2 and 3 are correct (b) 1 and 2 are correct
Q.18 A light has amplitude A and angle between analyser and
(c) 2 and 4 are correct (d) 1 and 3 are correct
polariser is 60°. Light transmitted by analyser has
amplitude Q.23 Plane polarised light is passed through a polaroid. On
viewing through the polaroid we find that when the polariod
(a) A 2 (b) A / 2 (c) 3 A / 2 (d) A/2 is given one complete rotation about the direction of the
light, which of the following is not observed ?
Q.19 A slit of size 0.15 cm is placed at 2.1 m from a screen. On
illuminating it by a light of wavelength 5×10–5 cm, the width (1) The intensity of light gradually decreases to zero and
of central maxima will be remains at zero
(2) The intensity of light gradually increases to a
(a) 70 mm (b) 0.14 mm (c) 1.4 mm (d) 0.14 cm
maximum and remains at maximum
Q.20 What will be the angle of diffraction for the first minimum (3) There is no change in intensity
due to Fraunhoffer diffraction with sources of light of wave
lenght 550 nm and slit width 0.55 mm ? (4) The intensity of light is twice maximum and twice
zero
(a) 0.001 rad (b) 0.01 rad Q.24 Out of the following statements which are correct ?
(c) 1 rad (d) 0.1 rad (1) Nicol’s prism works on the principle of double
Q.21 In Fresnel diffraction, if the distance between the disc and refraction and total internal reflection
the screen is decreased, the intensity of central bright spot (2) Nicol’s prism can be used to produce and analyse
will polarised light
(a) increase (b) decrease (3) Calcite and Quartz are both doubly refracting crystals
(c) remain constant (d) none of these (4) When unpolarised light passes through a Nicol’s
prism, the emergent light is elliptically polarised
22. When an unpolarized light of intensity I 0 is incident on a Q.25 Which statements are incorrect for a zone plate and a lens?
polarizing sheet, the intensity of the light which does not (1) Zone plate has one focus whereas lens has multiple
get transmitted is focii

1 1 (2) Both zone plate and lens have multi focii


(a) I0 (b) I0 (c) I 0 (d) zero (3) Zone plate has one focus whereas a lens has infinite
4 2
(4) Zone plate has multi focii whereas lens has one

RESPONSE 17. 18. 19. 20. 21.


GRID 22. 23. 24. 25.

Space for Rough Work


t.me/Ebooks_Encyclopedia27. t.me/Magazines4all

EBD_7156
4 DPP/ P 53
DIRECTIONS (Q.26-Q.27) : Read the passage given below (b) Statement-1 is True, Statement-2 is True; Statement-2 is
and answer the questions that follows : NOT a correct explanation for Statement-1.
Angular width of central maximum in the Fraunhoffer-diffraction (c) Statement-1 is False, Statement-2 is True.
pattern of a slit is measured. The slit is illuminated by light of (d) Statement-1 is True, Statement-2 is False.
wavelength 6000 Å. When the slit is illuminated by light of Q.28 Statement-1 : The unpolarised light and polarised light
another wavelength, the angular width decreases by 30%. can be distinguished from each other by using polaroid.
Q.26 The wavelength of the light is Statement-2 : A polaroid is capable of producing plane
(a) 4200Å (b) 3500Å (c) 5000 Å (d) 5200Å polarised beams of light.
Q.27 The same decrease in the angular width of central maximum Q.29 Statement-1 : Nicol prism is used to produce and analyse
is obtained when the original apparatus is immersed in a plane polarised light.
liquid. Find refractive index of the liquid. Statement-2 : Nicol prism reduces the intensity of light
(a) 1.23 (b) 1.43 (c) 2.2 (d) 2.43 to zero.
Q.30 Statement-1 : The cloud in sky generally appear to be
DIRECTIONS (Q. 28-Q.30) : Each of these questions contains
whitish.
two statements: Statement-1 (Assertion) and Statement-2
(Reason). Each of these questions has four alternative choices, Statement-2 : Diffraction due to clouds is efficient in equal
only one of which is the correct answer. You have to select the measure at all wavelengths.
correct choice.
(a) Statement-1 is True, Statement-2 is True; Statement-2 is a
correct explanation for Statement-1.

RESPONSE GRID 26. 27. 28. 29. 30.

DAILY PRA CTICE PROBLEM SHEET 53 - PHYSICS


Total Questions 30 Total Marks 120
Attempted Correct
Incorrect N et Score
Cut-off Score 26 Qualifying Score 46
Success Gap = Net Score – Qualifying Score
Net Score = (Correct × 4) – (Incorrect × 1)

Space for Rough Work


t.me/Ebooks_Encyclopedia27. t.me/Magazines4all

DPP - Daily Practice Problems


Name : Date :

Start Time : End Time :

SYLLABUS : DUAL NATURE OF MATTER & RADIATION (Matter Waves, Photon, Photoelectric effect, X-ray)
54
Max. Marks : 120 Time : 60 min.
GENERAL INSTRUCTIONS
• The Daily Practice Problem Sheet contains 30 MCQ's. For each question only one option is correct. Darken the correct
circle/ bubble in the Response Grid provided on each page.
• You have to evaluate your Response Grids yourself with the help of solution booklet.
• Each correct answer will get you 4 marks and 1 mark shall be deduced for each incorrect answer. No mark will be given/
deducted if no bubble is filled. Keep a timer in front of you and stop immediately at the end of 60 min.
• The sheet follows a particular syllabus. Do not attempt the sheet before you have completed your preparation for that
syllabus. Refer syllabus sheet in the starting of the book for the syllabus of all the DPP sheets.
• After completing the sheet check your answers with the solution booklet and complete the Result Grid. Finally spend time
to analyse your performance and revise the areas which emerge out as weak in your evaluation.

DIRECTIONS (Q.1-Q.21) : There are 21 multiple choice Q.3 One electron & one proton is accelerated by equal potential.
questions. Each question has 4 choices (a), (b), (c) and (d), out Ratio of their de-Broglie wavelengths is-
of which ONLY ONE choice is correct. mp me mp
Q.1 Energy of a a-particle, having de broglie wavelength of (a) (b) mp (c) (d) 1
me me
0.004 Å is approximately.
Q.4 de-Broglie wavelength of an electron is 10 Å then velocity
(a) 1275 eV (b) 1200 KeV will be-
(c) 1200 MeV (d) 1200 GeV (a) 7.2 × 107 m/s (b) 7.2 × 106 m/s
5
(c) 7.2 × 10 m/s (d) 7.2 × 104 m/s
Q.2 Velocity of a proton is c/20. Associated de-Broglie Q.5 One electron & one proton have equal energies then ratio
wavelength is (Take h = 6.626 × 10–34 J–s) of associated de-Broglie wavelength will be-
(a) 2.64 × 10–24 mm (b) 2.64 × 10–24 cm (a) 1 : (1836)2 (b) 1836 :1
(c) 2.64 × 10–14 Å (d) 2.64 × 10–14 m (c) 1836 : 1 (d) (1836)2 : 1

RESPONSE GRID 1. 2. 3. 4. 5.

Space for Rough Work


t.me/Ebooks_Encyclopedia27. t.me/Magazines4all

EBD_7156
2 DPP/ P 54
Q.6 The ratio of wavelength of deutron & proton accelerated Q.13 A photon and an electron have equal energy E. lphoton/
by an equal potential is lelectron is proportional to
1 2 1
(a) (b) (a) E (b)
2 1 E
1 2
(c) (d) 1
2 1 (c) (d) Does not depend upon E.
E
Q.7 In photoelectric effect if intensity of light is doubled then
maximum kinetic energy of photoelectrons will become Q.14 In a photoemissive cell with exciting wavelength l, the
(a) Double (b) Half fastest electron has speed v. If the exciting wavelength is
(c) Four time (d) No change changed to 3l/4, the speed of the fastest emitted electron
Q.8 Quantum nature of light is explained by which of the will be
following phenomenon? (a) v (3/4)1/2 (b) v (4/3)1/2
(a) Huygen wave theory (c) Less than v (4/3)1/2 (d) Greater than v (4/3)1/2
(b) Photoelectric effect Q.15 Which of the following figure repesents variation of
(c) Maxwell electromagnetic theory particle momentum and the associated de-Broglie
(d) de- Broglie theory wavelength?
Q.9 From rest an electron is accelerated between two such
points which has potential 20 & 40 volts respectively. p p
Associated de-Broglie wavelength of electron is-
(a) 0.75 Å (b) 7.5 Å
(c) 2.75 Å (d) 2.75 m (a) (b)
Q.10 An electron microscope uses 40 keV electrons. Find its
resolving limit on the assumption that it is equal to the l l
wavelength of the electron-
(a) 0.61 Å (b) 0.6 Å p p
(c) 0.06 Å (d) 0.061 Å
Q.11 A hydrogen atom moving at a speed v absorbs a photon of
wavelength 122 nm and stops. Find the value of v. (c) (d)
(Mass of hydrogen atom = 1.67 × 10–27 kg)
l l
(a) 3.5 m/s (b) 32.5 m/s
(c) 3.05 m/s (d) 3.25 m/s Q.16 The work function for the surface of aluminium is 4.2 eV.
Q.12 The de-Broglie wavelength of an electron is 0.2 Å. Calculate What will be the wavelength of that incident light for which
the potential difference (approximate) required to retard the stopping potential will be zero.
it to rest- (h » 6.6 × 10–34 J–s e » 1.6 × 10–19 C)
(a) 3.76 × 10–3 V (b) 3.76 × 103 V
3 (a) 2496 Å (b) 2946 × 10–7 m
(c) 3.76 × 10 eV (d) 376.5 V
(c) 2649 Å (d) 2946 Å

6. 7. 8. 9. 10.
RESPONSE
11. 12. 13. 14. 15.
GRID
16.

Space for Rough Work


t.me/Ebooks_Encyclopedia27. t.me/Magazines4all

DPP/ P 54 3
Q.17 Slope of V0 – n curve is- Q.22 Ultraviolet light of wavelength 280 nm is used in an
(where V0 = Stopping potential and n = frequency) experiment on photo electric effect with lithium (f = 2.5
h eV) cathode.
(a) e (b) (c) f0 (d) h (1) The maximum kinetic energy is 1 .9 eV
e
Q.18 A radio station is transmitting waves of wavelength 300 m. (2) The stopping potential is1.9 V
If diffracting power of transmitter is 10 kW, then numbers (3) The maximum kinetic energy is 4.4 V
of photons diffracted per second is- (4) The stopping potential is 4.4 eV
(a) 1.5 × 1035 (b) 1.5 × 1031 Q.23 The separation between Bragg’s planes in a crystal is
(c) 1.5 × 10 29 (d) 1.5 × 1033 10 Å. Then the wavelength of those X-rays which can be
Q.19 Light of wavelength 3320 Å is incident on metal surface diffracted by this crystal is-
(work function = 1.07 eV). To stop emission of photo (1) 5 Å (2) 10 Å
electron, retarding potential required to be
(3) 20 Å (4) 25 Å
(Take hc » 12420 eV – Å)
(a) 3.74 V (b) 2.67 V (c) 1.07 V (d) 4.81 V Q.24 Electrons are accelerated in television tubes through
Q.20 The figure shows the variation of photocurrent with anode potential difference of about 10 KV.
potential for a photo-sensitive surface for three different (1) The lowest wavelength of the emitted X-rays is 12.4Å
radiations. Let Ia, Ib and Ic be the intensities and fa, fb and fc (2) The lowest wavelength of the emitted X-rays is 1.24Å
be the frequencies for the curves a, b and c respectively. (3) The highest frequency of the emitted X-rays is
Then 2.4 × 108 Hz
(a) fa = fb and Ia ¹ Ib (4) The highest frequency of the emitted X-rays is
c b 2.4 × 1018 Hz
(b) fa = fc and Ia = Ic a
DIRECTIONS (Q.25-Q.27) : Read the passage given below
(c) fa = fb and Ia = Ib and answer the questions that follows :
(d) fa = fb and Ia = Ic A physicist wishes to eject electrons by shining light on a metal
O
Q.21 An electromagnetic radiation of frequency 3 × 1015 cycles surface. The light source emits light of wavelength of 450 nm.
per second falls on a photo electric surface whose work The table lists the only available metals and their work functions.
function is 4.0 eV. Find out the maximum velocity of the Metal W0 (eV)
photo electrons emitted by the surface- Barium 2.5
(a) 13.4 × 10–19 m/s (b) 19.8 × 10–19m/s Lithium 2.3
6
(c) 1.73 × 10 m/s (d) None Tantalum 4.2
Tungsten 4.5
DIRECTIONS (Q.22-Q.24) : In the following questions,
more than one of the answers given are correct. Select Q.25 Which metal(s) can be used to produce electrons by the
the correct answers and mark it according to the following photoelectric effect from given source of light ?
codes: (a) Barium only
(b) Barium or lithium
Codes :
(a) 1, 2 and 3 are correct (b) 1 and 2 are correct (c) Lithium, tantalum or tungsten
(c) 2 and 4 are correct (d) 1 and 3 are correct (d) Tungsten or tantalum

RESPONSE 17. 18. 19. 20. 21.


GRID 22. 23. 24. 25.

Space for Rough Work


t.me/Ebooks_Encyclopedia27. t.me/Magazines4all

EBD_7156
4 DPP/ P 54
Q.26 Which option correctly identifies the metal that will (b) Statement-1 is True, Statement-2 is True; Statement-2 is
produce the most energetic electrons and their energies ? NOT a correct explanation for Statement-1.
(a) Lithium, 0.45 eV (b) Tungsten, 1.75 eV (c) Statement -1 is False, Statement-2 is True.
(c) Lithium, 2.30 eV (d) Tungsten, 2.75 eV (d) Statement -1 is True, Statement-2 is False.
Q.27 Suppose photoelectric experiment is done separately with
Q.28 Statement -1 : Mass of moving photon varies directly as
these metals with light of wavelength 450 nm. The
the wavelength.
maximum magnitude of stopping potential amongst all the
Statement -2 : Energy of the particle = Mass × (Speed of
metals is-
light)2
(a) 2.75 volt (b) 4.5 volt
Q.29 Statement -1 : Photosensitivity of a metal is large if its
(c) 0.45 volt (d) 0.25 volt
work function is small.
DIRECTIONS (Q. 28-Q.30) : Each of these questions contains Statement -2 : Work function = hf 0 where f 0 is the
two statements: Statement-1 (Assertion) and Statement-2 threshold frequency.
(Reason). Each of these questions has four alternative choices, Q.30 Statement -1 : The de-Broglie wavelength of a molecule
only one of which is the correct answer. You have to select varies inversely as the square root of temperature.
the correct choice. Statement -2 : The root mean square velocity of the
(a) Statement-1 is True, Statement-2 is True; Statement-2 is a molecule is proportional to square root of absolute
correct explanation for Statement-1. temperature.

RESPONSE GRID 26. 27. 28. 29. 30.

DAILY PRA CTICE PROBLEM SHEET 54 - PHYSICS


Total Questions 30 Total Marks 120
Attempted Correct
Incorrect N et Score
Cut-off Score 30 Qualifying Score 50
Success Gap = Net Score – Qualifying Score
Net Score = (Correct × 4) – (Incorrect × 1)

Space for Rough Work


t.me/Ebooks_Encyclopedia27. t.me/Magazines4all

DPP - Daily Practice Problems


Name : Date :

Start Time : End Time :

SYLLABUS : Atoms
55
Max. Marks : 120 Time : 60 min.
GENERAL INSTRUCTIONS
• The Daily Practice Problem Sheet contains 30 MCQ's. For each question only one option is correct. Darken the correct
circle/ bubble in the Response Grid provided on each page.
• You have to evaluate your Response Grids yourself with the help of solution booklet.
• Each correct answer will get you 4 marks and 1 mark shall be deduced for each incorrect answer. No mark will be given/
deducted if no bubble is filled. Keep a timer in front of you and stop immediately at the end of 60 min.
• The sheet follows a particular syllabus. Do not attempt the sheet before you have completed your preparation for that
syllabus. Refer syllabus sheet in the starting of the book for the syllabus of all the DPP sheets.
• After completing the sheet check your answers with the solution booklet and complete the Result Grid. Finally spend time
to analyse your performance and revise the areas which emerge out as weak in your evaluation.

DIRECTIONS (Q.1-Q.21) : There are 21 multiple choice Q.4 The energy levels of the hydrogen spectrum is shown in
questions. Each question has 4 choices (a), (b), (c) and (d), out figure. There are some transitions. A,B,C,D and E.
of which ONLY ONE choice is correct. Transition A, B and C respectively represent
(a) First spectral line of
Q.1 In nature there may not be an element for which the Lyman series, third
principal quantum number n > 4 , then the total possible spectral line of Balmer
number of elements will be series and the second
(a) 60 (b) 32 (c) 4 (d) 64 spectral line of Paschen
Q.2 In the following atoms and molecule for the transition series.
from n = 2 to n = 1 , the spectral line of minimum (b) Ionization potential of hydrogen, second spectral line
wavelength will be produced by of Balmer series and third spectral line of Paschen
(a) Hydrogen atom (b) Deuterium atom series
(c) Series limit of Lyman series, third spectral line of
(c) Uni-ionized helium (d) Di-ionized lithium
Balmer series and second spectral line of Paschen
Q.3 The Lyman series of hydrogen sperctum lies in the region series
(a) Infrared (b) Visible (d) Series limit of Lyman series, second spectral line of
(c) Ultraviolet (d) X - rays Balmer series and third spectral line of Paschen series

RESPONSE GRID 1. 2. 3. 4.
Space for Rough Work
t.me/Ebooks_Encyclopedia27. t.me/Magazines4all

EBD_7156
2 DPP/ P 55
Q.5 Energy levels A, B, C of a certain atom corresponding to Q.11 According to Bohr’s theory the moment of momentum of
increasing values of energy i.e. E A < EB < EC . If an electron revolving in second orbit of hydrogen atom will
be
l1 , l 2 , l3 are the wavelengths of radiations corresponding
p h
to the transitions C to B, B to A and C to A respectively, (a) 2ph (b) ph (c) (d)
which of the following statements is correct? h p
C Q.12 In the Bohr model of a hydrogen atom, the centripetal force
l1 is furnished by the coulomb attraction between the proton
B
l3
l2 and the electron. If a0 is the radius of the ground state orbit,
A m is the mass, e is the charge on the electron and e0 is the
l1l 2 vacuum permittivity, the speed of the electron is
(a) l3 = l1 + l 2 (b) l3 = l + l
1 2 e
(a) 0 (b) e a m
(c) l1 + l 2 + l3 = 0 (d) l32 = l12 + l 22 0 0
Q.6 If m is mass of electron, v its velocity, r the radius of e 4pe0 a0 m
stationary circular orbit around a nucleus with charge Ze, (c) 4pe0 a0 m (d)
e
then from Bohr’s first postulate, the kinetic energy
Q.13 Which of the following transitions in hydrogen atoms emit
1
K = mv2 of the electron in C.G.S . system is equal to photons of highest frequency?
2 (a) n = 1 to n = 2 (b) n = 2 to n = 6
1 Ze 2 1 Ze 2 Ze2 Ze (c) n = 6 to n = 2 (d) n = 2 to n = 1
(a) (b) (c) (d)
2 r 2 r 2
r r2 Q.14 As per Bohr model, the minimum energy (in eV ) required
Q.7 When a hydrogen atom is raised from the ground state to to remove an electron from the ground state of doubly
an excited state ionized Li atom ( Z = 3) is
(a) P. E. increases and K. E. decreases
(a) 1.51 (b) 13.6
(b) P. E. decreases and K. E. increases
(c) 40.8 (d) 122.4
(c) Both kinetic energy and potential energy increase
Q.15 The third line of Balmer series of an ion equivalent to
(d) Both K. E. and P. E. decrease
hydrogen atom has wavelength of 108.5 nm. The ground
Q.8 The value of the kinetic energy divided by the total energy
state energy of an electron of this ion will be
of an electron in a Bohr orbit is
(a) 3.4 eV (b) 13.6 eV
(a) – 1 (b) 2
(c) 54.4 eV (d) 122.4 eV
(c) 0.5 (d) None of these
Q.16 The wavelength of radiation emitted is l0 when an electron
Q.9 The ratio of the frequencies of the long wavelength limits
jumps from the third to the second orbit of hydrogen atom.
of Lyman and Balmer series of hydrogen spectrum is
For the electron jump from the fourth to the second orbit of
(a) 27 : 5 (b) 5 : 27
the hydrogen atom, the wavelength of radiation emitted will
(c) 4 : 1 (d) 1 : 4
be
Q.10 Ratio of the wavelengths of first line of Lyman series and
first line of Balmer series is 16 20 27 25
(a) l 0 (b) l0 (c) l0 (d) l0
(a) 1 : 3 (b) 27 : 5 (c) 5 : 27 (d) 4 : 9 25 27 20 16

5. 6. 7. 8. 9.
RESPONSE
10. 11. 12. 13. 14.
GRID
15. 16.

Space for Rough Work


t.me/Ebooks_Encyclopedia27. t.me/Magazines4all

DPP/ P 55 3

Q.17 The energy of electron in the nth orbit of hydrogen atom DIRECTIONS (Q.22-Q.24) : In the following questions,
-13.6 more than one of the answers given are correct. Select
is expressed as En = eV . The shortest and longest the correct answers and mark it according to the following
n2 codes:
wavelength of Lyman series will be
(a) 910 Å, 1213 Å (b) 5463 Å, 7858 Å Codes :
(c) 1315 Å , 1530 Å (d) None of these (a) 1, 2 and 3 are correct (b) 1 and 2 are correct
(c) 2 and 4 are correct (d) 1 and 3 are correct
Q.18 Consider a hydrogen like atom whose energy in nth exicited
Q.22 The electron in a hydrogen atom makes a transition n 1 ®
13.6 Z 2
state is given by En = - when this excited atom n2, where n 1 and n2 are the principal quantum numbers of
n2 two states. Assume the Bohr model to be valid. The time
makes a transition from excited state to ground state, most period of the electron in the initial state is eight times that
energetic photons have energy Emax = 52.224 eV and least in the final state. Then
energetic photons have energy Emin = 1.224 eV . The (1) n1 = 4 (2) n2 = 2 (3) n2 = 5 (4) n1 = 5
atomic number of atom is Q.23 A free hydrogen atom in ground state is at rest. A neutron
(a) 2 (b) 5 of kinetic energy K collides with the hydrogen atom.
(c) 4 (d) None of these After collision hydrogen atom emits two photons in
Q.19 In the Bohr model of the hydrogen atom, let R, v and E succession one of which has energy 2.55eV. Assume that
represent the radius of the orbit, the speed of electron and the hydrogen atom and neutron has same mass.
the total energy of the electron respectively. Which of the (1) Minimum value of K is 25.5 eV
following quantity is proportional to the quantum number (2) Minimum value of K is 12.75 eV
n (3) The other photon has energy 10.2eV
(4) The upper energy level is of excitation energy 12.5
(a) R / E (b) E / v eV
(c) RE (d) vR Q.24 Which of the series of hydrogen spectrum are not in the
Q.20 An a - particle of 5 MeV energy strikes with a nucleus visible region?
of uranium at stationary at an scattering angle of 180°. The (1) Lyman series (2) Paschen series
nearest distance upto which a - particle reaches the (3) Bracket series (4) Balmer series
nucleus will be closest to
DIRECTIONS (Q.25-Q.27) : Read the passage given below
(a) 1 Å (b) 10–10cm and answer the questions that follows :
–12
(c) 10 cm (d) 10–15cm
Q.21 In a hypothetical Bohr hydrogen, the mass of the electron A gas of identical hydrogen like atoms has some atoms in ground
state and some atoms in a particular excited state and there are
is doubled. The energy E0 and the radius r0 of the first no atoms in any other energy level. The atoms of the gas make
orbit will be ( a0 is the Bohr radius) transition to a higher energy state by absorbing monochromatic
light of wavelength 304Å. Subsequently, the atoms emit radia-
(a) E0 = -27.2eV ; r0 = a0 /2 tion of only six different photon energies. Some of emitted pho-
(b) E0 = -27.2eV ; r0 = a0 tons have wavelength 304 Å, some have wavelength more and
some have less than 304Å (Take hc = 12420 eV–Å)
(c) E0 = -13.6eV ; r0 = a0 /2
Q.25 Find the principal quantum number of the initially excited
(d) E0 = -13.6eV ; r0 = a0 state.
(a) 1 (b) 2 (c) 3 (d) 4

RESPONSE 17. 18. 19. 20. 21.


GRID 22. 23. 24. 25.

Space for Rough Work


t.me/Ebooks_Encyclopedia27. t.me/Magazines4all

EBD_7156
4 DPP/ P 55
Q.26 Identify the gas (Z = ?) (c) Statement -1 is False, Statement-2 is True.
(a) 1 (b) 2 (c) 3 (d) 4 (d) Statement -1 is True, Statement-2 is False.
Q.27 Find the maximum and minimum energies of emitted Q.28 Statement-1 : Bohr postulated that the electrons in
photons (in eV) stationary orbits around the nucleus do not radiate energy.
(a) 20.4, 10.6 (b) 10.4, 3.6 Statement-2 : According to classical physics all moving
(c) 40.8, 10.6 (d) None of these electrons radiate energy.
Q.29 Statement-1 : The force of repulsion between atomic
DIRECTIONS (Q.28-Q.30) : Each of these questions contains nucleus and a-particle varies with distance according to
two statements: Statement-1 (Assertion) and Statement-2 inverse square law.
(Reason). Each of these questions has four alternative choices, Statement-2 : Rutherford did a-particle scattering
only one of which is the correct answer. You have to select experiment.
the correct choice. Q.30 Statement-1 : Hydrogen atom consists of only one
(a) Statement-1 is True, Statement-2 is True; Statement-2 is a electron but its emission spectrum has many lines.
correct explanation for Statement-1. Statement-2 : Only Lyman series is found in the absorption
(b) Statement-1 is True, Statement-2 is True; Statement-2 is spectrum of hydrogen atom whereas in the emission
NOT a correct explanation for Statement-1. spectrum, all the series are found.

RESPONSE GRID 26. 27. 28. 29. 30.

DAILY PRA CTICE PROBLEM SHEET 55 - PHYSICS


Total Questions 30 Total Marks 120
Attempted Correct
Incorrect N et Score
Cut-off Score 30 Qualifying Score 50
Success Gap = Net Score – Qualifying Score
Net Score = (Correct × 4) – (Incorrect × 1)

Space for Rough Work


t.me/Ebooks_Encyclopedia27. t.me/Magazines4all

DPP - Daily Practice Problems


Name : Date :

Start Time : End Time :

SYLLABUS : Nuclei
56
Max. Marks : 120 Time : 60 min.
GENERAL INSTRUCTIONS
• The Daily Practice Problem Sheet contains 30 MCQ's. For each question only one option is correct. Darken the correct
circle/ bubble in the Response Grid provided on each page.
• You have to evaluate your Response Grids yourself with the help of solution booklet.
• Each correct answer will get you 4 marks and 1 mark shall be deduced for each incorrect answer. No mark will be given/
deducted if no bubble is filled. Keep a timer in front of you and stop immediately at the end of 60 min.
• The sheet follows a particular syllabus. Do not attempt the sheet before you have completed your preparation for that
syllabus. Refer syllabus sheet in the starting of the book for the syllabus of all the DPP sheets.
• After completing the sheet check your answers with the solution booklet and complete the Result Grid. Finally spend time
to analyse your performance and revise the areas which emerge out as weak in your evaluation.

DIRECTIONS (Q.1-Q.21) : There are 21 multiple choice (a) 6 × 1023, 6 × 1023 , 6 × 1023
questions. Each question has 4 choices (a), (b), (c) and (d), out (b) 36 × 1023 , 36 × 1023 , 36 × 1023
of which ONLY ONE choice is correct. (c) 12 × 1023, 12 × 1023, 12 × 1023
(d) 18 × 1023, 18 × 1023, 18 × 1023
Q.1 The energy released per fission of uranium 235 is about Q.4 Nuclear radius of 8O16 is 3 × 10–15 m. Find the density of
200 MeV. A reactor using U-235 as fuel is producing 1000 nuclear matter.
kilowatt power. The number of U-235 nuclei undergoing (a) 7.5 × 1017 kg m–3 (b) 5.7 × 1017 kg m–3
fission per sec is, approximately- 17
(c) 2.3 × 10 kg m –3 (d) 1.66 × 1017 kg m–3
(a) 106 (b) 2 × 108 (c) 3 × 1016 (d) 931 Q.5 Consider the decay of radium-226 atom into an alpha
Q.2 Power output of 92U235 reactor if it takes 30 days to use up particle and radon-222. Then, what is the mass defect of the
2kg of fuel, and if each fission gives 185 MeV of useable reaction-
energy is- Mass of radium -226 atom = 226.0256 u
(a) 5.846 kW (b) 58.46 MW Mass of radon - 222 atom = 222.0715 u
(c) .5846 kW (d) None Mass of helium - 4 atom = 4.0026 u
Q.3 How many electrons, protons and neutrons are there in a (a) 0.0053 u (b) 0.0083 u
6 gm of 6C12. (c) 0.083 u (d) None

RESPONSE GRID 1. 2. 3. 4. 5.
Space for Rough Work
t.me/Ebooks_Encyclopedia27. t.me/Magazines4all

EBD_7156
2 DPP/ P 56
Q.6 If mass equivalent to one mass of proton is completely Q.12 If the binding energy of deuterium is 2.23 MeV, then the
converted into energy then determine the energy produced? mass defect will be- (in a.m.u.)

(a) 931.49 MeV (b) 731.49 MeV (a) 0.0024 (b) – 0.0024

(c) 911.49 MeV (d) 431.49 MeV (c) – 0.0012 (d) 0.0012

Q.13 The ratio of the radii of the nuclei 27 e125


Q.7 If mass equivalent to one mass of electron is completely 13 Al and 52Te is
converted into energy then determine the energy liberated. approximately -
(a) 1.51 MeV (b) 0.51 MeV (a) 6 : 10 (b) 13 : 52
(c) 3.12 MeV (d) 2.12 MeV (c) 40 : 177 (d) 14 : 73
Q.8 If the mass defect in the formation of helium from Q.14 The radius of the 30 Zn64 nucleus is nearly (in fm)-
hydrogen is 0.5%, then the energy obtained, in kWH, in (a) 1.2 (b) 2.4
forming helium from 1 kg of hydrogen will be-
(c) 3.7 (d) 4.8
(a) 1.25 (b) 125 × 104
Q.15 How many electrons, protons, and neutrons are there in a
(c) 1.25 × 108 (d) 1.25 × 106 nucleus of atomic number 11 and mass number 24?
Q.9 The half life of radioactive Radon is 3.8 days. The time at (a) 11, 12, 13 (b) 11, 11, 13
the end of which 1/20th of the Radon sample will remain
(c) 12, 11, 13 (d) 11, 13, 12
undecayed is
Q.16 Energy of each photon obtained in the pair production
(Given log 10e = 0.4343)
process will be, if the mass of electron or positron is
(a) 3.8 days (b) 16.5 days
1/2000 a.m.u-
(c) 33 days (d) 76 days (a) 0.213 MeV (b) 0.123 MeV
Q.10 In the nuclear reaction, (c) 0.321 MeV (d) 0.465 MeV
92 U238 ® A 4
ZTh + 2He , the values of A and Z are- Q.17 Deuterium is an isotope of hydrogen having a mass of
(a) A = 234, Z = 94 (b) A = 234, Z = 90 2.01470 amu. Find binding energy in MeV of this isotope

(c) A = 238, Z = 94 (d) A = 238, Z = 90 (a) 2.741 MeV (b) 2.174 MeV

Q.11 The mass of helium nucleus is less than that of its (c) 1.741 MeV (d) 0.741 MeV
constituent particles by 0.03 a.m.u. The binding energy per Q.18 The binding energy per nucleon for 3Li7 will be, if the mass
nucleon of 2He4 nucleus will be- of 3Li7 is 7.0163 a.m.u.
(a) 7 MeV (b) 14 MeV (a) 5.6 MeV (b) 39.25 MeV
(c) 3.5 MeV (d) 21 MeV (c) 1 MeV (d) zero

6. 7. 8. 9. 10.
RESPONSE
11. 12. 13. 14. 15.
GRID
16. 17. 18.

Space for Rough Work


t.me/Ebooks_Encyclopedia27. t.me/Magazines4all

DPP/ P 56 3
Q.19 Sun radiates energy in all direction. The average energy (1) atoms disintegrated per second in r eactor is
recieved at earth is 1.4 kW/m2. The average distance 3.125 ×1016
between the earth and the sun is 1.5 × 10 11 m. If this energy (2) atoms disintegrated per second in r eactor is
is released by conservation of mass into energy, then the 3.125 ×1018
mass lost per day by the sun is approximately
(3) decay in mass per hour is 4 × 10–8 kg
(Use 1day = 86400 sec)
(4) decay in mass per hour is 4 × 10–6 kg
(a) 4.4 × 109 kg (b) 7.6 × 1014 kg Q.23 Which of the following are not examples of nuclear fusion?
(c) 3.8 × 1012 kg (d) 3.8 × 1014 kg (1) Formation of Ba and Kr from U235
Q.20 Fission of nuclei is possible because the binding energy (2) Formation of Pu – 235 from U–235
per nucleon in them
(3) Formation of water from hydrogen and oxygen
(a) increases with mass number at high mass number
(4) Formation of He from H
(b) decreases with mass number at high mass number
Q.24 Which of the following are mode of radioactive decay?
(c) increases with mass number at low mass numbers (1) Positron emission (2) Electron capture
(d) decreases with mass number at low mass numbers (3) Alpha decay (4) Fusion
Q.21 Half life of Bi210 is 5 days. If we start with 50,000 atoms
DIRECTIONS (Q.25-Q.27) : Read the passage given below
of this isotope, the number of atoms left over after 10 days
and answer the questions that follows :
is
(a) 5,000 (b) 25,000 In a living organism, the quantity of C14 is the same as in the
atmosphere. But in organisms which are dead, no exchange takes
(c) 12,500 (d) 20,000 place with the atmosphere and by measuring the decay rate of
14C in the old bones or wood, the time taken for the activity to
DIRECTIONS (Q.22-Q.24) : In the following questions,
reduce to this level can be calculated. This gives the age of the
more than one of the answers given are correct. Select the
wood or bone.
correct answers and mark it according to the following
codes: Given : T1/2 for 14 C is 5370 years and the ratio of
14 C/ 12 C is 1.3 × 10 –12 .
Codes :
Q.25 The decay rate of 14C in 1g of carbon in a living organism
(a) 1, 2 and 3 are correct (b) 1 and 2 are correct
is
(c) 2 and 4 are correct (d) 1 and 3 are correct
(a) 25 Bq (b) 2.5 Bq
Q.22 On disintegration of one atom of U235 the amount of energy
(c) 0.25 Bq (d) 5 Bq
obtained is 200 MeV. The power obtained in a reactor is
1000 KW. Then

RESPONSE 19. 20. 21. 22. 23.


GRID 24. 25.

Space for Rough Work

https://rebrand.ly/DownloadMore
t.me/Ebooks_Encyclopedia27. t.me/Magazines4all

EBD_7156
4 DPP/ P 56
Q.26 If in an old sample of wood of 10g the decay rate is 30 (b) Statement-1 is True, Statement-2 is True; Statement-2 is
decays per minute, the age of the wood is NOT a correct explanation for Statement-1.
(a) 50 years (b) 1000 years (c) Statement -1 is False, Statement-2 is True.
(c) 13310 years (d) 15300 years (d) Statement -1 is True, Statement-2 is False.
Q.27 The decay rate in another piece is found to be 0.30 Bq per Q.28 Statement-1 : Amongst alpha, beta and gamma rays, g-has
gm then we can conclude maximum penetrating power.
Statement-2 : The alpha particle is heavier than beta and
(a) the sample is very recent
gamma rays.
(b) the observed decay is not that of 14C alone Q.29 Statement-1 : The mass of b-particles when they are
(c) there is a statistical error emitted is higher than the mass of electrons obtained by
other means.
(d) all of these Statement-2 : b-particle and electron, both are similar
particles.
DIRECTIONS (Q. 28-Q.30) : Each of these questions contains
two statements: Statement-1 (Assertion) and Statement-2 Q.30 Statement-1 : Electron capture occurs more often than
(Reason). Each of these questions has four alternative choices, positron emission in heavy elements.
only one of which is the correct answer. You have to select the Statement-2 : Heavy elements exhibit radioactivity.
correct choice.
(a) Statement-1 is True, Statement-2 is True; Statement-2 is a
correct explanation for Statement-1.

RESPONSE GRID 26. 27. 28. 29. 30.

DAILY PRA CTICE PROBLEM SHEET 56 - PHYSICS


Total Questions 30 Total Marks 120
Attempted Correct
Incorrect N et Score
Cut-off Score 28 Qualifying Score 48
Success Gap = Net Score – Qualifying Score
Net Score = (Correct × 4) – (Incorrect × 1)

Space for Rough Work


t.me/Ebooks_Encyclopedia27. t.me/Magazines4all

DPP - Daily Practice Problems


Name : Date :

Start Time : End Time :

SYLLABUS : SEMICONDUCTOR ELECTRONICS - 1 (Semiconductors, LED, Photodiode, Zener diode)


57
Max. Marks : 120 Time : 60 min.
GENERAL INSTRUCTIONS
• The Daily Practice Problem Sheet contains 30 MCQ's. For each question only one option is correct. Darken the correct
circle/ bubble in the Response Grid provided on each page.
• You have to evaluate your Response Grids yourself with the help of solution booklet.
• Each correct answer will get you 4 marks and 1 mark shall be deduced for each incorrect answer. No mark will be given/
deducted if no bubble is filled. Keep a timer in front of you and stop immediately at the end of 60 min.
• The sheet follows a particular syllabus. Do not attempt the sheet before you have completed your preparation for that
syllabus. Refer syllabus sheet in the starting of the book for the syllabus of all the DPP sheets.
• After completing the sheet check your answers with the solution booklet and complete the Result Grid. Finally spend time
to analyse your performance and revise the areas which emerge out as weak in your evaluation.

DIRECTIONS (Q.1-Q.21) : There are 21 multiple choice Q.4 Let nh and ne be the number of holes and conduction
questions. Each question has 4 choices (a), (b), (c) and (d), out electrons respectively in a semiconductor. Then
of which ONLY ONE choice is correct. (a) nh > nein an intrinsic semiconductor
Q.1 When a semiconductor is heated, its resistance (b) nh = nein an extrinsic semiconductor
(a) decreases (b) increases (c) nh = ne in an intrinsic semiconductor
(c) reamins unchanged (d) nothing is definite (d) ne > nh in an intrinsic semiconductor
Q.2 The energy band gap of Si is Q.5 Which statement is correct?
(a) 0.70 eV (a) N-type germanium is negatively charged and P-type
(b) 1.1 eV
germanium is positively charged
(c) between 0.70 eV to 1.1eV
(d) 5 eV (b) Both N-type and P-type germanium are neutral
Q.3 The forbidden energy ban d gap in conductors, (c) N-type germanium is positively charged and P-type
semiconductors and insulators are EG1, EG2 and EG3 germanium is negatively charged
respectively. The relation among them is (d) Both N-type and P-type germanium are negatively
(a) EG1 = EG2 = EG3 (b) EG1 < EG2 < EG3 charged
(c) EG1 > EG2 > EG3 (d) EG1 < EG2 > EG3

RESPONSE GRID 1. 2. 3. 4. 5.

Space for Rough Work


t.me/Ebooks_Encyclopedia27. t.me/Magazines4all

EBD_7156
2 DPP/ P 57
Q.6 Wires P and Q have the same resistance at ordinary (room) Q.12 The energy gap for diamond is nearly
temperature. When heated, resistance of P increases and (a) 1 ev (b) 2 ev (c) 4 ev (d) 6 ev
that of Q decreases. We conclude that Q.13 The valence band and conduction band of a solid overlap at
(a) P and Q are conductors of different materials low temperature, the solid may be
(b) P is n-type semiconductor and Q is p-type (a) metal (b) semiconductor
semiconductor (c) insulator (d) None of these
(c) P is semiconductor and Q is conductor Q.14 Choose the correct statement
(a) When we heat a semiconductor its resistance
(d) P is conductor and Q is semiconductor
increases
Q.7 In extrinsic P and N-type, semiconductor materials, the (b) When we heat a semiconductor its resistance
ratio of the impurity atoms to the pure semiconductor decreases
atoms is about (c) When we cool a semiconductor to 0 K then it becomes
(a) 1 (b) 10–1 (c) 10–4 (d) 10–7 super conductor
Q.8 At zero Kelvin a piece of germanium (d) Resistance of a semiconductor is independent of
(a) becomes semiconductor temperature
(b) becomes good conductor Q.15 If ne and vd be the number of electrons and drift velocity
(c) becomes bad conductor in a semiconductor. When the temperature is increased
(d) has maximum conductivity (a) ne increases and vd decreases
Q.9 Electronic configuration of germanium is 2, 8, 18 and 4, (b) ne decreases and vd increases
To make it extrinsic semiconductor small quantity of (c) Both ne and vd increases
antimony is added (d) Both ne and vd decreases
(a) The material obtained will be N-type germanium in Q.16 The reverse biasing in a PN junction diode
which electrons and holes are equal in number (a) decreases the potential barrier
(b) The material obtained will be P-type germanium (b) increases the potential barrier
(c) The material obtanied will be N-type germanium (c) increases the number of minority charge carriers
which has more electrons than holes at room (d) increases the number of majority charge carriers
temperature Q.17 Two PN-junctions can be connected in series by three
(d) The material obtained will be N-type germanium different methods as shown in the figure. If the potential
which h as less electrons than holes at room difference in the junctions is the same, then the correct
temperature connections will be
Q.10 The intrinsic semiconductor becomes an insulator at P N N P P N P N N P N P
(a) 0°C (b) –100°C
(c) 300 K (d) 0 K
Q.11 Energy bands in solids are a consequence of + – + – + –
(a) Ohm’s Law 1 2 3
(b) Pauli’s exclusion principle (a) In the circuit (1) and (2)
(c) Bohr’s theory (b) In the circuit (2) and (3)
(c) In the circuit (1) and (3)
(d) Heisenberg’s uncertainty principle
(d) Only in the circuit (1)

6. 7. 8. 9. 10.
RESPONSE
11. 12. 13. 14. 15.
GRID
16. 17.

Space for Rough Work


t.me/Ebooks_Encyclopedia27. t.me/Magazines4all

DPP/ P 57 3
Q.18 The approximate ratio of resistances in the forward and Q.22 In the given figure, which of the diodes are forward biased?
reverse bias of the PN-junction diode is +10V
(a) 102 : 1 (b) 10–2 : 1 (1) (2) –12V
–4 R
(c) 1 : 10 (d) 1 : 104
Q.19 The dominant mechanisms for motion of charge carriers
in forward and reverse biased silicon P-N junctions are +5V R –5V
(a) Drift in forward bias, diffusion in reverse bias R
(b) Diffusion in forward bias, drift in reverse bias (3) (4) –10V
R
(c) Diffusion in both forward and reverse bias
(d) Drift in both forward and reverse bias
Q.20 In a triclinic crystal system –10V
(a) a ¹ b ¹ c, a ¹ b ¹ g Q.23 Which of the following materials are crystalline?
(b) a = b = c, a ¹ b ¹ g (1) Copper (2) Sodium chloride
(c) a ¹ b ¹ c, a ¹ b = g (3) Diamond (4) Wood
(d) a = b = c, a = b = g Q.24 A piece of copper and the other of germanium are cooled
from the room temperature to 80 K, then which of the
Q.21 The correct cymbol for zener diode is
following would be wrong statements?
(1) Resistance of each increases
(a)
(2) Resistance of each decreases
(3) Resistance of copper increases while that of
+ – germanium decreases
(b)
(4) Resistan ce of copper decreases while that of
+ – germanium increases
(c)
DIRECTIONS (Q.25-Q.27) : Read the passage given below
and answer the questions that follows :
+ – A student performs an experiment for drawing the static
(d)
characteristic curve of a triode valve in the laboratory. The
following data were obtained from the linear portion of the
DIRECTIONS (Q.22-Q.24) : In the following questions, curves:
more than one of the answers given are correct. Select the Grid voltage Vg (volt) –2.0 –3.5 –2.0
correct answers and mark it according to the following
Plate voltage Vp (volt) 180 180 120
codes:
Plate current IP (mA) 15 7 10
Codes :
Q.25 Calculate the plate resistance rp of the triode valve?
(a) 1, 2 and 3 are correct (b) 1 and 2 are correct
(a) 0.12 × 104 ohm (b) 1.2 × 10 4 ohm
(c) 2 and 4 are correct (d) 1 and 3 are correct 4
(c) 1.3 × 10 ohm (d) 1.4 × 10 4 ohm

RESPONSE 18. 19. 20. 21. 22.


GRID 23. 24. 25.

Space for Rough Work


t.me/Ebooks_Encyclopedia27. t.me/Magazines4all

EBD_7156
4 DPP/ P 57
Q.26 Calculate the mutual conductance gm of the triode valve? (c) Statement -1 is False, Statement-2 is True.
(a) 5.33 × 10–3 ohm–1 (b) 53.3 × 10–3 ohm–1 (d) Statement -1 is True, Statement-2 is False.
–3
(c) 4.32 × 10 ohm –1 (d) 5.00 × 10–3 ohm–1 Q.28 Statement-1 : The number of electrons in a P-type silicon
Q.27 Calculate the amplification factor µ, of the triode valve? semiconductor is less than the number of electrons in a
(a) 64 (b) 52 pure silicon semiconductor at room temperature.
(c) 54 (d) 62 Statement-2: It is due to law of mass action.
Q.29 Statement-1 : The resistivity of a semiconductor decreases
DIRECTIONS (Q. 28-Q.30) : Each of these questions contains
with temperature.
two statements: Statement-1 (Assertion) and Statement-2
(Reason). Each of these questions has four alternative choices, Statement-2 : The atoms of a semiconductor vibrate with
only one of which is the correct answer. You have to select the larger amplitude at higher temperature there by increasing
correct choice. its resistivity.
(a) Statement-1 is True, Statement-2 is True; Statement-2 is a Q.30 Statement-1 : We can measure the potential barrier of a
correct explanation for Statement-1. PN junction by putting a sensitive voltmeter across its
terminals.
(b) Statement-1 is True, Statement-2 is True; Statement-2 is
NOT a correct explanation for Statement-1. Statement-2 : The current through the PN junction is not
same in forward and reversed bias.

RESPONSE GRID 26. 27. 28. 29. 30.

DAILY PRA CTICE PROBLEM SHEET 57 - PHYSICS


Total Questions 30 Total Marks 120
Attempted Correct
Incorrect N et Score
Cut-off Score 28 Qualifying Score 48
Success Gap = Net Score – Qualifying Score
Net Score = (Correct × 4) – (Incorrect × 1)

Space for Rough Work


t.me/Ebooks_Encyclopedia27. t.me/Magazines4all

DPP - Daily Practice Problems


Name : Date :

Start Time : End Time :

SYLLABUS : SEMICONDUCTOR ELECTRONICS-2 (Junction transistor, transistor action, characteristics of a


58
transistor, transistor as an amplifier, logic gates)

Max. Marks : 112 Time : 60 min.


GENERAL INSTRUCTIONS
• The Daily Practice Problem Sheet contains 28 MCQ's. For each question only one option is correct. Darken the correct
circle/ bubble in the Response Grid provided on each page.
• You have to evaluate your Response Grids yourself with the help of solution booklet.
• Each correct answer will get you 4 marks and 1 mark shall be deduced for each incorrect answer. No mark will be given/
deducted if no bubble is filled. Keep a timer in front of you and stop immediately at the end of 60 min.
• The sheet follows a particular syllabus. Do not attempt the sheet before you have completed your preparation for that
syllabus. Refer syllabus sheet in the starting of the book for the syllabus of all the DPP sheets.
• After completing the sheet check your answers with the solution booklet and complete the Result Grid. Finally spend time
to analyse your performance and revise the areas which emerge out as weak in your evaluation.

DIRECTIONS (Q.1-Q.20) : There are 20 multiple choice Q.3 In an NPN transistor 1010 electrons enter the emitter in
questions. Each] question has 4 choices (a), (b), (c) and (d), 10–6 s and 2% electrons recombine with holes in base,
out of which ONLY ONE choice is correct. then a and b respectively are
(a) a = 0.98, b = 49 (b) a = 49, b = 0.98
Q.1 A NPN transistor conducts when (c) a = 0.49, b = 98 (d) a = 98, b = 0.49
(a) both collector and emitter are positive with respect
Q.4 If l1, l2 , l3 are the lengths of the emitter, base and collector
to the base
(b) collector is positive and emitter is negative with of a transistor then
respect to the base (a) l1 = l2 = l3 (b) l3 < l2 > l1
(c) collector is positive and emitter is at same potential (c) l3 < l1 < l2 (d) l3 > l1 > l2
as the base Q.5 In an NPN transistor circuit, the collector current is 10
(d) both collector and emitter are negative with respect mA. If 90% of the electrons emitted reach the collector,
to the base the emitter current (iE) and base current (iB) are given by
Q.2 In the case of constants a and b of a transistor (a) iE = -1mA, iB = 9mA (b) iE = 9mA, iB = -1mA
(a) a = b (b) b < 1, a > 1
(c) iE = 1mA, iB = 11mA (d) iE = 11mA, iB = 1mA
(c) a = b2 (d) b > 1, a < 1

RESPONSE GRID 1. 2. 3. 4. 5.
Space for Rough Work
t.me/Ebooks_Encyclopedia27. t.me/Magazines4all

EBD_7156
2 DPP/ P 58
Q.6 The transfer ratio of a transistor is 50. The input resistance Q.12 For the given combination of gates, if the logic states of
of the transistor when used in the common-emitter inputs A, B, C are as follows A = B = C = 0 and
configuration is 1 kW. The peak value for an A.C input
A = B = 1,
voltage of 0.01 V peak is
C = 0 then the logic states of output D are
(a) 100 μA (b) 0.01 mA
(c) 0.25 mA (d) 500 μA
Q.7 For transistor, the current amplification factor is 0.8. The
transistor is connected in common emitter configuration. (a) 0, 0 (b) 0, 1 (c) 1, 0 (d) 1, 1
The change in the collector current when the base current Q.13 Correct statement for ‘NOR’ gate is that, it gives
changes by 6 mA is (a) high output when both the inputs are low
(a) 6 mA (b) 4.8 mA (b) low output when both the inputs are low
(c) 24 mA (d) 8 mA (c) high output when both the inputs are high
Q.8 In NPN transistor the collector current is 10 mA. If 90% (d) None of these
of electrons emitted reach the collector, then Q.14 A gate has the following truth table
(a) emitter current will be 9 mA P 1 1 0 0
(b) emitter current will be 11.1 mA Q 1 0 1 0
(c) base current will be 0.1 mA
(d) base current will be 0.01 mA R 1 0 0 0
Q.9 In a transistor in CE configuration, the ratio of power gain The gate is
to voltage gain is (a) NOR (b) OR (c) NAND (d) AND
(a) a (b) b / a Q.15 What will be the input of A and B for the Boolean
(c) ba (d) b expression ( A + B).( A.B) = 1
Q.10 The following truth table corresponds to the logic gate (a) 0, 0 (b) 0, 1 (c) 1, 0 (d) 1, 1
Q.16 To get an output 1 from the circuit shown in the figure, the
A 0 0 1 1 input can be
B 0 1 0 1
X 0 1 1 1
(a) NAND (b) OR
(c) AND (d) XOR
Q.11 The truth table shown in figure is for (a) A = 0, B = 1, C = 0 (b) A = 1, B = 0, C = 0
A 0 0 1 1 (c) A = 1, B = 0, C = 1 (d) A = 1, B = 1, C = 0
Q.17 The truth-table given below is for which gate?
B 0 1 0 1
A 0 0 1 1
Y 1 0 0 1
B 0 1 0 1
(a) XOR (b) AND
C 1 1 1 0
(c) XNOR (d) OR
(a) XOR (b) OR (c) AND (d) NAND

6. 7. 8. 9. 10.
RESPONSE
11. 12. 13. 14. 15.
GRID
16. 17.

Space for Rough Work


t.me/Ebooks_Encyclopedia27. t.me/Magazines4all

DPP/ P 58 3
Q.18 The combination of gates shown below produces Q.22 Given below are symbols for some logic gates. The XOR
gate and NOR gate are respectively
(1) (2)

(3) (4)
(a) AND gate (b) XOR gate Q.23 Given below are four logic gates symbol (figure). Those
(c) NOR gate (d) NAND gate for OR, NOR and NAND are respectively
Q.19 Figure gives a system of logic gates. From the study of A y A y
(1) (2) B
truth table it can be found that to produce a high output (1) B
at R, we must have A y A y
(3) B (4) B

DIRECTIONS (Q.24-Q.25) : Read the passage given below


and answer the questions that follows :
Doping changes the fermi energy of a semiconductor. Consider
(a) X = 0, Y = 1 (b) X = 1, Y = 1 silicon, with a gap of 1.11 eV between the top of the valence
(c) X = 1, Y = 0 (d) X = 0, Y = 0 bond and the bottom of the conduction band. At 300K the Fermi
Q.20 In the case of a common emitter transistor as/an amplifier, level of the pure material is nearly at the mid-point of the gap.
the ratio Ic /Ie is 0.96, then the current gain (b) of the amplifier Suppose that silicon is doped with donor atoms, each of which
is has a state 0.15 eV below the bottom of the silicon conduction
(a) 6 (b) 48 band, and suppose further that doping raises the Fermi level to
(c) 24 (d) 12 0.11 eV below the bottom of that band.
DIRECTIONS (Q.21-Q.23) : In the following questions, Conduction Band

more than one of the answers given are correct. Select the
correct answers and mark it according to the following Fermi Donor
1.11eV level level
codes:
Codes :
(a) 1, 2 and 3 are correct (b) 1 and 2 are correct
(c) 2 and 4 are correct (d) 1 and 3 are correct Valence Band

Q.21 Which of the following are false? Q.24 For both pure and doped silicon, calculate the probability
(1) Common base transistor is commonly used because that a state at the bottom of the silicon conduction band is
current gain is maximum occupied? (e4.524 = 70.38)
(2) Common collector is commonly used because current (a) 5.20 × 10–2 (b) 1.40 × 10–2
gain is maximum (c) 10.5 × 10 –2 (d) 14 × 10–2
(3) Common emitter is the least used transistor Q.25 Calculate the probability that a donor state in the doped
(4) Common emitter is commonly used because current material is occupied? e–1.547 = 0.212
gain is maximum (a) 0.824 (b) 0.08
(c) 0.008 (d) 8.2

RESPONSE 18. 19. 20. 21. 22.


GRID 23. 24. 25.

Space for Rough Work


t.me/Ebooks_Encyclopedia27. t.me/Magazines4all

EBD_7156
4 DPP/ P 58
DIRECTIONS (Q. 26-Q.28) : Each of these questions contains Q.27 Statement -1 : The following circuit represents ‘OR’ gate
two statements: Statement-1 (Assertion) and Statement-2
(Reason). Each of these questions has four alternative choices,
only one of which is the correct answer. You have to select the
correct choice. Statement-2 : For the above circuit Y = X = A + B = A + B

(a) Statement-1 is True, Statement-2 is True; Statement-2 is a Q.28 Statement -1 : De-morgan’s theorem A + B = A.B may be
correct explanation for Statement-1. explained by the following circuit
(b) Statement-1 is True, Statement-2 is True; Statement-2 is
NOT a correct explanation for Statement-1.
(c) Statement -1 is False, Statement-2 is True.
(d) Statement -1 is True, Statement-2 is False.
Q.26 Statement -1 : The logic gate NOT cannot be built by using
diode.
Statement -2 : In the following circuit, for output 1 inputs
Statement -2 : The output voltage and the input voltage of
A,B,C are 1, 0, 1.
the diode have 180° phase difference.

RESPONSE GRID 26. 27. 28.

DAILY PRA CTICE PROBLEM SHEET 58 - PHYSICS


Total Questions 28 Total Marks 112
Attempted Correct
Incorrect N et Score
Cut-off Score 26 Qualifying Score 46
Success Gap = Net Score – Qualifying Score
Net Score = (Correct × 4) – (Incorrect × 1)

Space for Rough Work


t.me/Ebooks_Encyclopedia27. t.me/Magazines4all

DPP - Daily Practice Problems


Name : Date :

Start Time : End Time :

SYLLABUS : Communication Systems, Laser


59
Max. Marks : 120 Time : 60 min.
GENERAL INSTRUCTIONS
• The Daily Practice Problem Sheet contains 30 MCQ's. For each question only one option is correct. Darken the correct
circle/ bubble in the Response Grid provided on each page.
• You have to evaluate your Response Grids yourself with the help of solution booklet.
• Each correct answer will get you 4 marks and 1 mark shall be deduced for each incorrect answer. No mark will be given/
deducted if no bubble is filled. Keep a timer in front of you and stop immediately at the end of 60 min.
• The sheet follows a particular syllabus. Do not attempt the sheet before you have completed your preparation for that
syllabus. Refer syllabus sheet in the starting of the book for the syllabus of all the DPP sheets.
• After completing the sheet check your answers with the solution booklet and complete the Result Grid. Finally spend time
to analyse your performance and revise the areas which emerge out as weak in your evaluation.

DIRECTIONS (Q.1-Q.21) : There are 21 multiple choice Q.3 Range of frequencies allotted for commercial FM radio
questions. Each question has 4 choices (a), (b), (c) and (d), out broadcast is
of which ONLY ONE choice is correct. (a) 88 to 108 MHz (b) 88 to 108 kHz
Q.1 In short wave communication which of the following (c) 8 to 88 MHz (d) 88 to 108 GHz
frequencies will be reflected back by the ionospheric layer, Q.4 The process of superimposing signal frequency (i.e. audio
having electron density 1011 per m3. wave) on the carrier wave is known as
(a) 2.84 MHz (b) 10.42 MHz
(a) Transmission (b) Reception
(c) 12.24 MHz (d) 18.1 MHz
(c) Modulation (d) Detection
Q.2 In an amplitude modulated wave for audio frequency of
500 cycle/second, the appropriate carrier frequency will Q.5 The characteristic impedance of a coaxial cable is of the
be order of
(a) 50 cycles/sec (b) 100 cycles/sec (a) 50W (b) 200W
(c) 500 cycles/sec (d) 50, 000 cycles/sec (c) 270W (d) None of these

RESPONSE GRID 1. 2. 3. 4. 5.

Space for Rough Work


t.me/Ebooks_Encyclopedia27. t.me/Magazines4all

EBD_7156
2 DPP/ P 59
Q.6 If m1 and m 2 are the refractive indices of the materials of (b) that converts radio frequency signal into
electromagnetic energy
core and cladding of an optical fibre, then the loss of light
(c) that converts guided electromagnetic waves into free
due to its leakage can be minimised by having
space electromagnetic waves and vice-versa
(a) m1 > m2 (b) m1 < m 2 (d) None of these
(c) m1 = m2 (d) None of these Q.14 While tuning in a certain broadcast station with a receiver,
we are actually
Q.7 Maximum usable frequency (MUF) in F-region layer is x,
(a) varying the local oscillator frequency
when the critical frequency is 60 MHz and the angle of
(b) varying the frequency of the radio signal to be picked
incidence is 70o . Then x is (cos 70° = 0.34) up
(a) 150 MHz (b) 170 MHz (c) tuning the antenna
(c) 175 MHz (d) 190 MHz (d) None of these
Q.8 A laser is a coherent source because it contains Q.15 In a communication system, noise is most likely to affect
(a) many wavelengths the signal
(b) uncoordinated wave of a particular wavelength (a) At the transmitter
(c) coordinated wave of many wavelengths (b) In the channel or in the transmission line
(d) coordinated waves of a particular wavelength (c) In the information source
Q.9 A laser beam is used for carrying out surgery because it (d) At the receiver
(a) is highly monochromatic Q.16 In an FM system a 7 kHz signal modulates 108 MHz
(b) is highly coherent
carrier so that frequency deviation is 50 kHz . The carrier
(c) is highly directional
swing is
(d) can be sharply focussed
Q.10 Laser beams are used to measure long distances because (a) 7.143 (b) 8
(a) they are monochromatic (c) 0.71 (d) 350
(b) they are highly polarised Q.17 The phenomenon by which light travels in an optical fibres
(c) they are coherent is
(d) they have high degree of parallelism (a) Reflection
Q.11 An oscillator is producing FM waves of frequency (b) Refraction
2 kHz with a variation of 10 kHz . What is the modulating (c) Total internal reflection
index? (d) Transmission
(a) 0.20 (b) 5.0 (c) 0.67 (d) 1.5 Q.18 In frequency modulation
Q.12 If f a and ff represent the carrier wave frequencies for (a) The amplitude of modulated wave varies as frequency
amplitude and frequency modulations respectively, then of carrier wave
(a) fa > f f (b) fa < f f (b) The frequency of modulated wave varies as amplitude
of modulating wave
(c) fa = f f (d) fa ³ f f (c) The amplitude of modulated wave varies as amplitude
Q.13 An antenna is a device of carrier wave
(a) that converts electromagnetic energy into radio (d) The frequency of modulated wave varies as frequency
frequency signal of modulating wave

6. 7. 8. 9. 10.
RESPONSE
11. 12. 13. 14. 15.
GRID
16. 17. 18.

Space for Rough Work


t.me/Ebooks_Encyclopedia27. t.me/Magazines4all

DPP/ P 59 3
Q.19 Audio signal cannot be transmitted because Q.23 What type of modulation is not employed in India for radio
(a) The signal has more noise transmission?
(b) The signal cannot be amplified for distance (1) A mixture of both frequency and pulse modulation.
communication
(c) The transmitting antenna length is very small to design (2) Pulse modulation
(d) The transmitting antenna length is very large and (3) Frequency modulation
impracticable (4) Amplitude modulation
Q.20 For sky wave propagation of a 10 MHz signal, what should
be the minimum electron density in ionosphere Q.24 Which of the following are the characteristics of Laser
beams?
(a) ~ 1.2 ´ 1012 m -3
(1) They are monochromatic
(b) ~ 106 m -3
(2) They are coherent
(c) ~ 1014 m-3 (3) They have high degree of parallelism
(d) ~ 10 22 m -3 (4) They are not monochromatic
Q.21 What should be the maximum acceptance angle at the
DIRECTIONS (Q.25-Q.27) : Read the passage given below
aircore interface of an optical fibre if n1 and n2 are the and answer the questions that follows :
refractive indices of the core and the cladding, respectively
The electron density of a layer of ionosphere at a height 150 km
(a) sin -1 (n2 / n1 ) (b) sin -1 n12 - n22 from the earth's surface is 9 × 10 9 per m 3 . For the sky
transmission from this layer up to a range of 250 km,
é -1 n2 ù é -1 n1 ù
(c) ê tan n ú (d) ê tan n ú Q.25 The critical frequency of the layer is
ë 1û ë 2û
(a) 2 Hz (b) 2.7 Hz
DIRECTIONS (Q.22-Q.24) : In the following questions,
more than one of the answers given are correct. Select the (c) 2.78 kHz (d) 2.7 MHz
correct answers and mark it according to the following Q.26 Maximum usable frequency is
codes:
(a) 3.17 Hz
Codes :
(a) 1, 2 and 3 are correct (b) 1 and 2 are correct (b) 3.17 × 106 HZ
(c) 2 and 4 are correct (d) 1 and 3 are correct (c) 3.17×10 3 Hz
Q.22 In which of the following remote sensing technique is used?
(d) 3.17 × 1010Hz
(1) Forest density
(2) Pollution Q.27 Angle of incidence of this layer is
(3) Wetland mapping (a) 0° (b) sec–1 (1.51)
(4) Medical treatment (c) sec–1 (1.17) (d) 37°

RESPONSE 19. 20. 21. 22. 23.


GRID 24. 25. 26. 27.

Space for Rough Work


t.me/Ebooks_Encyclopedia27. t.me/Magazines4all

EBD_7156
4 DPP/ P 59
DIRECTIONS (Qs. 28-Q.30) : Each of these questions contains Q.28 Statement-1 : Television signals are not received through
two statements: Statement-1 (Assertion) and Statement-2 sky-wave propagation.
(Reason). Each of these questions has four alternative choices, Statement-2 : The ionosphere reflects electromagnetic
only one of which is the correct answer. You have to select the waves of frequencies greater than a certain critical
correct choice. frequency.
Q.29 Statement-1 : The electromagnetic waves of shorter
(a) Statement-1 is True, Statement-2 is True; Statement-2 is a wavelength can travel longer distances on earth’s surface
correct explanation for Statement-1. than those of longer wavelengths.
(b) Statement-1 is True, Statement-2 is True; Statement-2 is Statement-2 : Shorter the wavelength, the larger is the
NOT a correct explanation for Statement-1. velocity of wave propagation.
(c) Statement -1 is False, Statement-2 is True. Q.30 Statement-1 : A dish antenna is highly directional.
(d) Statement -1 is True, Statement-2 is False. Statement-2 : This is because a dipole antenna is omni
directional.

RESPONSE GRID 28. 29. 30.

DAILY PRA CTICE PROBLEM SHEET 59 - PHYSICS


Total Questions 30 Total Marks 120
Attempted Correct
Incorrect N et Score
Cut-off Score 30 Qualifying Score 50
Success Gap = Net Score – Qualifying Score
Net Score = (Correct × 4) – (Incorrect × 1)

Space for Rough Work


t.me/Ebooks_Encyclopedia27. t.me/Magazines4all

DPP - Daily Practice Problems


Name : Date :

Start Time : End Time :

SYLLABUS : Practical Physics - 2


60
Max. Marks : 112 Time : 60 min.
GENERAL INSTRUCTIONS
• The Daily Practice Problem Sheet contains 28 MCQ's. For each question only one option is correct. Darken the correct
circle/ bubble in the Response Grid provided on each page.
• You have to evaluate your Response Grids yourself with the help of solution booklet.
• Each correct answer will get you 4 marks and 1 mark shall be deduced for each incorrect answer. No mark will be given/
deducted if no bubble is filled. Keep a timer in front of you and stop immediately at the end of 60 min.
• The sheet follows a particular syllabus. Do not attempt the sheet before you have completed your preparation for that
syllabus. Refer syllabus sheet in the starting of the book for the syllabus of all the DPP sheets.
• After completing the sheet check your answers with the solution booklet and complete the Result Grid. Finally spend time
to analyse your performance and revise the areas which emerge out as weak in your evaluation.

DIRECTIONS (Q.1-Q.26) : There are 26 multiple choice (b) To include the resistance of the connecting wire and
questions. Each question has 4 choices (a), (b), (c) and (d), out copper strip
of which ONLY ONE choice is correct. (c) To balance the known and unknown resistance.
(d) To eliminate the resistance of the gap.
Q.1 In making an Ohm’s law circuit, which of the following Q.3 Potential gradient of a potentiometer is equal to
connection is correct? (a) e.m.f per unit length
(a) Voltmeter in series and ammeter in parallel (b) potential drop per unit length
(b) Voltmeter in parallel and ammeter in series (c) current per unit length
(c) Voltmeter and ammeter both are in parallel (d) resistance per unit length
(d) Voltmeter and ammeter both are in series Q.4 The refractive index of the material of a prism does not
Q.2 To calculate an unknown resistance with the help of a meter depend on which of the following factor?
bridge why is it advised to change the gap with the known (a) Nature of the material
and unknown resistance? (b) Wavelength or colour of light
(a) To eliminate the resistance of the connecting wire and (c) Temperature
copper strip (d) Angle of the prism.

RESPONSE GRID 1. 2. 3. 4.

Space for Rough Work


t.me/Ebooks_Encyclopedia27. t.me/Magazines4all

EBD_7156
2 DPP/ P 60
Q.5 A meter-bridge is based on the principle of Q.10 Current gain is maximum in which of the following
(a) Wheatstone bridge configuration of a transistor ?
(b) Variation of resistance with temperature (a) common emitter configuration
(c) Galvanometer (b) common base configuration
(d) None of these (c) common collector configuration
Q.6 A potentiometer works on the principle that (d) equal in both common emitter and common base
(a) when a current flows through a wire of uniform configuration
thickness and material, potential difference between Q.11 Which of the following operations will not increase the
its two points is directly proportional to the length of sensitivity of a potentiometer?
the wire between the two points (a) Increase in the number of wires of the potentiometer.
(b) when a current flows through a wire of uniform (b) Reducing the potential gradient.
thickness and material, potential difference between (c) Increasing the current through the potentiometer
its two points is inversely proportional to the length (d) Increasing the sensitivity of the galvanometer.
of the wire between the points Q.12 Which two circuit components are connected in parallel
(c) when a current flows through a wire of uniform in the following circuit diagram ?
thickness and material, potential difference between (a) Rheostat and voltmeter
its two points doesn't depend on the length of the wire (b) Voltmeter and ammeter
between the points
(c) Voltmeter and resistor
(d) none of these
(d) Ammeter and resistor
Q.7 Which of the following statement is wrong regarding a
Q.13 A current of 4A produces a deflection of 30° in the
p–n junction diode?
galvanometer. The figure of merit is
(a) When the p-type section is connected to the positive
(a) 6.5 A/rad (b) 7.6 A/rad
terminal and the n-type section to the negative terminal
(c) 7.5 A/rad (d) 8.0 A/rad
of the battery the diode is called forward biased
(b) When the p-type section is connected to the negative Q.14 Two potentiometers A and B having 4 wires and 10 wires,
terminal and the n-type section to the positive terminal each having 100 cm in length are used to compare e.m.f.
of the battery the diode is said to be reverse biased of 2 cells. Which one will give a larger balancing length?
(c) When the diode is in reverse biased mode a forward (a) Balancing length doesn't depend on the total length
current flows of the wire.
(d) When the diode is in forward biased mode a forward (b) Both A and B will give same balancing length
current flows. (c) Potentiometer B
Q.8 A Zener diode operates on which of the following bias? (d) Potentiometer A
(a) Forward bias Q.15 An LED operates under which biasing condition?
(b) Reverse bias (a) Forward bias
(c) Both forward and reverse bias. (b) Reverse bias
(d) No biasing is required for it. (c) Can operate both in forward and reverse bias
Q.9 The transfer characteristics of a transistor means a plot of (d) No biasing is required.
(a) input voltage versus input current Q.16 How are the currents flowing in the emitter, base and the
(b) output voltage versus output current. collector related to each other?
(c) output voltage versus input voltage (a) Ic =Ib + Ie (b) Ie =Ib + Ic
(d) input current versus output current. (c) Ib =Ie + Ic (d) Ie =Ic – Ib

5. 6. 7. 8. 9.
RESPONSE
10. 11. 12. 13. 14.
GRID
15. 16.

Space for Rough Work


t.me/Ebooks_Encyclopedia27. t.me/Magazines4all

DPP/ P 60 3
Q.17 The potential gradient of a potentiometer can be increased (a) (1.5 ± 0.05)kW R H
by which of the following operation? (b) (1.2 ± 0.05) k W E

(a) By increasing the area of cross-section of the (c) (1.2 ± 0.08) kW V R

potentiometer wire. (d) (1.5 ± 0.08) kW A

(b) By decreasing the area of cross-section of the Q.21 In an experiment to measure the focal length of a concave
potentiometer wire. mirror, it was found that for an object distance of 0.30 m,
(c) By decreasing the current through it. the image distance come out to be 0.60 m. Determine the
(d) By using a wire of material of low specific resistance. focal length.
when A is decreased, k will increase. (a) (0.2 ± 0.01) m (b) (0.1 ± 0.01) m
Q.18 Of the diodes shown in the following diagrams, which one (c) (0.2 ± 0.02) m (d) (0.1 ± 0.02) m
is reverse biased ? Q.22 In an experiment to determine an unknown resistance, a
+10 V 100 cm long resistance wire is used. The unknown
resistance is kept in the left gap and a known resistance is
–12 V R
(a) R
(b) put into the right gap. The scale used to measure length has
+5 V –5 V a least count 1 mm. The null point B is obtained at 40.0 cm
+5 V from the left gap. Determine the percentage error in the
(c) (d) R
computation of unknown resistance.
R
(a) 0.24% (b) 0.28% (c) 0.50% (d) 0.42%
–10 V
Q.23 In an experiment to determine the focal length
Q.19 To determine the equivalent resistance of two resistors (f ) of a concave mirror by the u - v method, a student places
when connected in series, a student arranged the circuit the object pin A on the principal axis at a distance x from
components as shown in the diagram. But he did not the pole P. The student looks at the pin and its inverted
succeed to achieve the objective. image from a distance keeping his/her eye in line with PA.
When the student shifts his/her eye towards left, the image
appears to the right of the object pin. Then,
(a) x < f (b) f < x < 2f
(c) x = 2f (d) x > 2f
æ 1ö æ 1ö
Which of the following mistakes has been committed by Q.24 For a convex spherical mirror, the graph of çè v ÷ø verses çè u ÷ø
him in setting up the circuit ?
is
(a) Position of voltmeter is incorrect (a) 1/ v (b) 1/ v
(b) Position of ammeter is incorrect
(c) Terminals of voltmeter are wrongly connected
(d) Terminals of ammeter are wrongly connected O 1/ u
1/ u
Q.20 In the circuit shown, voltmeter is ideal and its least count O
(c) 1/ v (d) 1/ v
is 0.1 V. The least count of ammeter is 1 mA. Let reading
of the voltmeter be 30.0 V and the reading of ammeter is
0.020 A. Calculate the value of resistance R within error
1/ u 1/ u
limits. O O

RESPONSE 17. 18. 19. 20. 21.


GRID 22. 23. 24.

Space for Rough Work


t.me/Ebooks_Encyclopedia27. t.me/Magazines4all

EBD_7156
4 DPP/ P 60
Q.25 If the wire in the experiment to determine the resistivity (ii) Calculate field E(r) at distance ‘r’ from A by using Ohm’s
of a material using meter bridge is replaced by copper or law E = r j, where j is the current per unit area at ‘r’.
hollow wire the balance point i.e. null point shifts to (iii) From the ‘r’ dependence of E(r), I DV I

(a) right (b) left obtain the potential V(r) at r.


(c) at same point (d) none of these (iv) Repeat (i), (ii) and (iii) for current a b a
A B C D
Q.26 Which device is used to measure the potential difference ‘I’ leaving ‘D’ and superpose
between two points of a conductor in the laboratory ? results for ‘A’ and ‘D’.
(a) Voltameter (b) Ammeter
(c) Potentiometer (d) Galvanometer Q.27 DV measured between B and C is
DIRECTIONS (Q.27-Q.28) : Read the passage given below rI rI rI rI
(a) – (b) –
and answer the questions that follows : pa p(a + b) a (a + b)
Consider a block of conducting material of resistivity ‘r’ shown rI rI rI
in the figure. Current ‘I’ enters at ‘A’ and leaves from ‘D’. We (c) – (d)
2pa 2p(a + b) 2p(a - b)
apply superposition principle to find voltage ‘DV’ developed Q.28 For current entering at A, the electric field at a distance ‘r’
between ‘B’ and ‘C’. The calculation is done in the following from A is
steps:
(i) Take current ‘I’ entering from ‘A’ and assume it to spread rI rI rI rI
(a) 2 (b) 2 (c) 2 (d)
over a hemispherical surface in the block. 8pr r 2pr 4pr 2

RESPONSE GRID 25. 26. 27. 28.

DAILY PRA CTICE PROBLEM SHEET 60 - PHYSICS


Total Questions 28 Total Marks 112
Attempted Correct
Incorrect N et Score
Cut-off Score 26 Qualifying Score 44
Success Gap = Net Score – Qualifying Score
Net Score = (Correct × 4) – (Incorrect × 1)

Space for Rough Work


t.me/Ebooks_Encyclopedia27. t.me/Magazines4all

DAILY PRACTICE
PROBLEMS
PHYSICS
SOLUTIONS 01
1 8. (a) Let radius of gyration [ k ] µ [ h]x [c ] y [G ]z
1. (d) f =
2p LC By substituting the dimension of
æ Cö [k] = [L] [h] = [ ML2T -1 ],[c ] = [ LT -1 ],
\ç ÷ does not represent the dimension of frequency..
è Lø
[G ] = [M -1 L3T -2 ]
2. (d) [n] = Number of particles crossing a unit area in unit
time = [L –2T –1] and by comparing the power of both sides
[n2] = [n1] = number of particles per unit volume = [L–3] we can get x = 1/2, y = – 3/2, z = 1/2
[x2] = [x1] = positions So dimension of radius of gyration are
[h]1/2 [c]–3/2[G]1/2
é L-2T -1 ù ´ [ L]
[n][ x2 - x1 ] 9. (b) Because magnitude is absolute.
\ D= =ë û = [ L2T -1 ]
[n2 - n1 ] -3 E
[L ] 10. (c) Stefan's law is E = s(T 4 ) Þ s =
T4
X M -1L-2T 4 A2
3. (d) Y = = = [M -3 L-2T 8 A4 ] Energy Watt
3Z 2 [ MT -2 A-1 ]2 where, E = = 2
Area ´ Time m
4. (a) In given equation, aZ should be dimensionless
kq Watt - m-2
s= = Watt - m-2 K -4
4
K
kq [ML2T -2 K -1 ´ K ]
\ a= Þ [a ] = = [ MLT -2 ] 11. (d) ct2 must have dimensions of L
Z [ L] Þ c must have dimensions of L/T2 = LT
T–2
a -2 12. (b) 6 × 10–5 = 60 × 10–6 = 60 microns
é a ù [ MLT ]
and P = Þ [b] = ê ú = = [ M 0 L2T 0 ]
b ë P û [ ML-1T -2 ] 1 q1q2 1 q1q2
13. (d) F= Þ Î0 = = C 2 m-2 N -1
4p Î0 r 2 4p Fr 2
1/ 2
P éF ù 2 P2 é F ù F 14. (b) According to the defition.
5. (c) v= Þ v = 2 êë m úû
\m µ
2l êë m úû 4l 2 2
l v 15. (b) Pyrometer is used for measurement of temperature.
16. (d) x = Ay + B tan Cz,
é MLT -2 ù -1 0 From the dimensional homogenity
Þ [m] = ê 2 -2 ú = [ ML T ]
ëê L T ûú éxù éBù
[ x ] = [ Ay] = [ B] Þ ê ú = [ y] = ê ú
6. (d) By substituting the dimensions of mass [M], length [L] ë Aû ë Aû
and coefficient of rigidity [ML–1T–2] we get [Cz] = [M0L0T0] = Dimensionless
M B
T = 2p is the right formula for time period of x and B; C and z–1; y and have the same dimension
hL A
oscillations. but x and A have the different dimensions.
7. (a) Time µ c x G y h z Þ T = kc x G y h z 17. (a) Let T µ S x r y r z
Putting the dimensions in the above relation by substituting the dimension of
Þ [ M 0 L0T 1 ] = [ LT -1 ]x [ M -1L3T -2 ] y [ ML2T -1 ]z [T] = [T], [ S ] = [ MT -2 ],[r ] = [ L ],[r] = [ML-3 ]
Þ [ M 0 L0T 1 ] = [ M - y + z Lx+ 3 y + 2 z T - x- 2 y - z ] and by comparing the power of both the sides
x = –1/2, y = 3/2, z = 1/2
Comparing the powers of M, L and T
–y+z=0 ....(i) 3
x + 3y + 2z = 0 ....(ii) so T µ rr 3 / S Þ T = k rr
– x – 2y – z = 1 ....(iii) S
On solving equations (i) and (ii) and (iii) 18. (c) E = hv Þ [ML2T -2 ] = [h][T -1 ] Þ [h] = [ ML2T -1 ]
-5 1
x= ,y=z=
2 2 ur ur ur
19. (c) P = A + B
Hence, dimension of time are [G1/2 h1/2c–5/2]. ur ur ur
Q= A–B
t.me/Ebooks_Encyclopedia27. t.me/Magazines4all

EBD_7156
2 DPP/ P 01
ur ur ur ur ur ur
P.Q = 0 Þ (A + B).(A – B) = 0 hc 6.6 ´10 -34 ´ 3 ´108
ur ur 27. (a) E= = = 4.95 ´10-19 J
Þ A2 – B2 = 0 Þ | A |=| B | l 4 ´10-7
= 3.0 eV
QP ^ Q 28. (a) Both statement -1 and statement -2 are correct and
20. (a) By substituting dimension of each quantitity in R.H.S. statement -1 follows from statement -2
of option (a) we get 29. (c) Let us write the dimension of various quantities on
two sides of the given relation.
é mg ù é M ´ LT -2 ù -1
ê hr ú = ê -1 -1 ú = [ LT ]. L.H.S. = T = [T],
ë û ëê ML T ´ L ûú
LT -2
This option gives the dimension of velocity. R.H.S. = 2p g / l = = [T -1 ]
21. (a) By principle of dimensional homogeneity L
(\ 2p has no dimension).
é a ù As dimension of L.H.S. is not equal to dimension of
ê 2 ú = [ P] R.H.S. therefore according to principle of homogeneity
ëV û
the relation.
\ [a ] = [ P ][V ]2 = [ ML-1T -2 ] ´ [ L6 ] = [ ML5T -2 ]
T = 2p g / l .
22. (c) f = c mx ky;
30. (a) Unit of quantity (L/R) is Henry/ohm.
Spring constant k = force/length.
As Henry = ohm × sec,
[M0L0T–1] = [Mx (MT–2)y] =[ Mx + y T–2y]
hence unit of L/R is sec
1 i.e. [L/R] = [T].
Þ x + y = 0, - 2 y = -1 or y =
2 Similarly, unit of product CR is farad × ohm
1 Coulomb Volt
Therefore, x = – or ×
2 Volt Amp
23. (a) Try out the given alternatives.
When x = 1, y = –1, z = 1 Sec×Amp
or = second
Amp
PC
P x S y C z = P1S-1C1 = i.e. [CR] = [T]
S
therefore, [L/R] and [CR] both have the same
[ML–1T –2 ] [LT –1 ] dimension.
= = [M 0 L0 T 0 ]
2 –2 2
[ML T / L T]

24. (c) Dimensions of angular momentum, [ L] = éë ML T ùû


2 -1

Dimensions of work, [ W ] = éë ML T ùû
2 -2

Dimensions of torque, [ t ] = éë ML T ùû
2 -2

Dimensions of energy, [ E ] = éë ML T ùû
2 -2

Dimensions of Young¢s modulus,

[ Y] = éëML-1T-2 ùû
Dimensions of light year = [L]
Dimension of wavelength = [L]
25. (d), 26. (b)

hG kgm 2s-1 ´ m3kg -1s-2


5
= 5 5
= s2 = s
c m /s
Putting the values of h, G and c in above relation
Planck time = 1.3 × 10– 43 s.
t.me/Ebooks_Encyclopedia27. t.me/Magazines4all

DPP/ P 02 3

DAILY PRACTICE
PROBLEMS
PHYSICS
SOLUTIONS 02
M M l
1. (d) Density r = = 2 10. (c) T = 2p
V pr L g

Dr DM Dr DL l 4p 2 l
Þ = +2 + Þ T 2 = 4p 2 Þ g= 2
r M r L g T
0.003 0.005 0.06 1mm 0.1
= + 2´ + Here % error in l = ´ 100 = ´ 100 = 0.1%
0.3 0.5 6 100cm 100
= 0.01 + 0.02 + 0.01 = 0.04
0.1
Dr and % error in T = ´ 100 = 0.05%
\ Percentage error = ´ 100 = 0.04 ´100 = 4% 2 ´ 100
r
\ % error in g
2. (a) In division (or multiplication), the number of significant = % error in l + 2(% error in T)
digits in the answer is equal to the number of = 0.1 + 2 × 0.05 = 0.2%
significant digits which is the minimum in the given 11. (c) Mean time period T = 2.00 sec
numbers. & Mean absolute error DT = 0.05 sec.
3. (d) Percentage error in A To express maximum estimate of error, the time period
should be written as (2.00 ± 0.05) sec
æ 1 ö
= ç 2 ´ 1 + 3 ´ 3 + 1´ 2 + ´ 2 ÷ % = 14%
è 2 ø 12. (b) V = 4 pr 3
4. (a) Percentage error in X = aa+ bb + cg 3
5. (c) Errors in A and B will be added. \ % error in volume = 3 × % error in radius
6. (c) Given, L = 2.331 cm = 3 × 1 = 3%
= 2.33 (correct upto two decimal places) 13. (a) Weight in air = (5.00 ± 0.05)N
and B = 2.1 cm = 2.10 cm Weight in water = (4.00 ± 0.05)N
\ L + B = 2.33 + 2.10 = 4.43 cm. = 4.4 cm Loss of weight in water = (1.00 ± 0.1)N
Since minimum significant figure is 2.
weight in air
7. (d) The number of significant figures in all of the given Now relative density =
number is 4. weight loss in water

8. (c) 4MgL so maximum permissible error in Y 5.00 ± 0.05


Y= i.e. R.D =
pD 2l 1.00 ± 0.1
Now relative density with max permissible error
DY æ DM Dg DL 2DD Dl ö
= ´ 100 = ç + + + + ÷ ´ 100 5.00 æ 0.05 0.1 ö
Y è M g L D l ø = ±ç ± ÷ ´100 = 5.0 ± (1 + 10)%
1.00 è 5.00 1.00 ø
= æç
1 1 1 1 1 ö = 5.0 ± 11%
+ + + 2 ´ + ÷ ´ 100
è 300 981 2820 41 87 ø
æ DR ö DV Dl
= 0.065 × 100 = 6.5% 14. (b) \ ç ´100 ÷ = ´100 + ´ 100
è R ømax V l
b a cb
9. (d) a = 5 0.2
d g ed = ´ 100 + ´100 = (5 + 2)% = 7%
100 10
So, maximum error in a is given by
15. (c) Volume of cylinder V = pr2l
æ Da ö Db Dc Percentage error in volume =
ç ´ 100 ÷ = a. ´ 100 + b. ´100
è a ømax b c DV 2Dr Dl
´ 100 = ´ 100 + ´ 100
Dd De V r l
+ g. ´ 100 + d. ´ 100
d e æ 0.01 0.1 ö
=ç 2´ ´100 + ´ 100 ÷ = (1 + 2)% = 3%
= (ab1+ bc1+ gd1+ de1)% è 2.0 5.0 ø
t.me/Ebooks_Encyclopedia27. t.me/Magazines4all

EBD_7156
4 DPP/ P 02
24. (d) Here, s = (13.8 ± 0.2) m
DV 2Dr Dl
t = (4.0 ± 0.3) s
´ 100 = ´ 100 + ´ 100
V r l s 13.8
velocity, v = = = 3.45 ms–1 = 3.5 ms–1
t 4.0
æ 0.01 0.1 ö
= ç 2´ ´100 + ´ 100 ÷ = (1 + 2)% = 3% (rounding off to two significant figures)
è 2.0 5.0 ø

16. (b) H = I2Rt Dv æ Ds Dt ö


= ±ç + ÷ = ±ç
æ 0.2 0.3 ö ( 0.8 + 4.14)
+ ÷=±
v è s t ø è 13.8 4.0 ø 13.8 ´ 4.0
DH æ 2D I D R D t ö
´ 100 = ç + + ÷ ´ 100
H è I R t ø Dv 4.94
Þ =± = ±0.0895
= (2 × 3 + 4 + 6)% = 16% v 13.8 ´ 4.0
17. (c) Quantity C has maximum power. So it brings maximum D v = ± 0.0895 × v = ± 0.0895 × 3.45 = ± 0.3087 = ± 0.31
error in P. (rounding off to two significant fig.)
1
Hence, v = (3.5 ± 0.31) ms–1
18. (d) Kinetic energy E = mv 2
2 Dv
% age error in velocity = ´ 100 = ± 0.0895 × 100 = ± 8.95
v
DE v '2 - v 2 % = ± 9%
\ ´ 100 = ´ 100 = [(1.5)2 – 1] × 100
E v2 25. (a) Maximum percentage error in measurement of e, as given
by Reyleigh’s formula.
DE
\ ´100 = 125% (Given error is measurement of radius is 0.1 cm)
E De = 0.6 DR = 0.6 × 0.1 = 0.06 cm.
x De 0.06
19. (b) Required random error = Percentage error is ´ 100 = ´ 100 = 3.33%
4 e 0.6 ´ 3
1 2 26. (b) Speed of sound at the room temperature.
20. (b) \ E = mv l1 = 4.6 cm, l2 = 14.0 cm.,
2
\ % Error in K.E. l = 2 (l2 – l1) = 2 (14.0 – 4.6) = 18.8 cm.
= % error in mass + 2 × % error in velocity 18.8
= 2 + 2 × 3 = 8% v = f l = 2000 ´ = 376 m / s
100
21. (c)
27. (c) End correction obtained in the experiment.
22. (d) Since for 50.14 cm, significant number = 4 and for
0.00025, significant numbers = 2 l 2 - 3l1 14.0 - 3 ´ 4.6
e= = = 0.1 cm.
23. (a) Since percentage increase in length = 2% 2 2
Hence, percentage increase in area of square sheet 28. (d) Since zeros placed to the left of the number are never
= 2 × 2% = 4% significant, but zeros placed to right of the number are
significant.
29. (b) The last number is most accurate because it has
greatest significant figure (3).
30. (a) As the distance of star increases, the parallex angle
decreases, and great degree of accuracy is required for
its measurement. Keeping in view the practical
limitation in measuring the parallex angle, the maximum
distance of a star we can measure is limited to 100 light
year.
t.me/Ebooks_Encyclopedia27. t.me/Magazines4all

DPP/ P 03 5

DAILY PRACTICE
PROBLEMS
PHYSICS
SOLUTIONS 03
1. (a) If t1 and 2 t2 are the time taken by particle to cover first 4. (a) The body starts from rest at x = 0 and then again comes
and second half distance respectively. to rest at x = 1. It means initially acceleration is positive
x/2 x and then negative.
t= = ...(i) So we can conclude that a can not remain positive for
3 6
x1 = 4.5t2 and x2 = 7.5 t2 all t in the interval 0 £ t £ 1 i.e. a must change sign
during the motion.
x x 5. (b) The area under acceleration time graph gives change
So, x1 + x2 = Þ 4.5t2 + 7.5t2 =
2 2 in velocity. As acceleration is zero at the end of 11 sec.
x i.e. vmax= Area of DOAB
t2 = ...(ii)
24 1
= ´11´10 = 55m/s
Total time t = t1 + 2t2 = x + x = x 2
6 12 4
So,average speed = 4 m/sec. a
2 B
dv bt 2 10 m/s
2. (c) = bt Þ dv = bt dt Þ v = + K1
dt 2
At t = 0, v = v0 Þ K1 = v0 A
t
1 2 O 11 sec.
We get v = bt + v0
2
6. (d) Let the car accelerate at rate a for time t1 then maximum
dx 1 2 1 bt 2 velocity attained, v = 0 + at1 = at1
Again = bt + v0 Þ x = + v0t + K 2
dt 2 2 3 Now, the car decelerates at a rate b for time (t – t1) and
finally comes to rest. Then,
At t = 0, x = 0 Þ K 2 = 0
0 = v – b(t – t1)
1
x = bt 3 + v0 t Þ 0 = at1 – bt + bt1
\
6
b
dv dv Þ t1 = t
3. (d) = 6 - 3v Þ = dt a +b
dt 6 - 3v
ab
dv Þ v= t
Integrating both sides, ò (6 - 3v) = ò dt a+b

log e (6 - 3v ) a
Þ = t + K1 7. (c) Sn = u + (2 n - 1)
-3 2
Þ loge(6 – 3v) = – 3t + K2 ....(i) a
At t = 0, v = 0 Þ loge 6 = K2 = (2 n - 1) (Qu = 0)
Substituting the value of K2 in equation (i) 2
loge(6 – 3v) = – 3t + loge 6 a a
Sn+1 = [ 2(n + 1) - 1] = (2n + 1)
æ 6 - 3v ö -3t 6 - 3v 2 2
Þ loge ç ÷ = -3t Þ e =
è 6 ø 6
Sn 2n - 1
\ =
Þ 6 - 3v = 6e-3t Þ 3v = 6(1 - e -3t ) Sn+1 2n + 1
Þ v = 2(1 - e-3t ) 8. (b) Distance = Area under v – t graph
\ vterminal = 2 m/s (when t = ¥) = A1 + A2 + A3 + A4

dv d é 1 1
Acceleration a = = 2(1 - e-3t ) ù = 6e -3t = ´1 ´ 20 + (20 ´ 1) + (20 + 10) ´1 + (10 ´ 1)
dt dt ë û 2 2
2
Initial acceleration = 6 m/s . = 10 + 20 + 15 + 10 = 55 m
t.me/Ebooks_Encyclopedia27. t.me/Magazines4all

EBD_7156
6 DPP/ P 03
æ t 2 t3 ö
Þ x = a0 ç - ÷ Qa=0Þ t=T
30 è 2 6T ø
Velocity (m/s)

20
displacement
Average velocity =
time
10 A2 æ T 2 T3 ö
A1 A3 A4 a0 ç - ÷
0 è 2 6T ø a 0 T
1 2 3 4 = =
T 3
Time (in second)

1 2 14. (b) Let u1, u2, u3 and u4 be velocities at time t = 0, t1,


9. (c) As acc. is constant so from s = ut + at we have (t1 + t2) and (t1 + t2 + t3) respectively and acceleration
2 is a then
1 2
x= at [u = 0] ....(i) u1 + u2 u + u3 u3 + u4
2 v1 = , v2 = 2 and v3 =
2 2 2
Now if it travels a distance y in next t sec.
in 2t sec total distance travelled Also u2 = u1 + at1, u3 = u1 + a(t1 + t2)
and u4 = u1 + a(t1 + t2 + t3)
1 By solving, we get
x+y= a(2t)2 ....(ii) (t + t = 2t)
2
v1 - v2 (t1 + t2 )
x+y =
Dividing eqn. (ii) by eqn (i), = 4 or y = 3x v2 - v3 (t2 + t3 )
x
dx d
g 15. (c) vx = = (3t2 – 6t) = 6t – 6. At t = 1, vx = 0
(2 ´ 4 - 1) dt dt
x ( 4) 2 7
= = a
10. (b) x (5) g 9 [QS = u+ (2n - 1)] dy d 2
(2 ´ 5 - 1) n th 2 vy = = (t – 2t) = 2t – 2. At t = 1, vy = 0
2 dt dt
11. (c) Let body takes T sec to reach maximum height.
Then v = u – gT B v=0 Hence v = v x2 + v 2y = 0
v = 0, at highest point.
16. (c) Initial relative velocity = v1 – v2.
u C Final relative velocity = 0
T= ...................(1]
g From v2 = u2 – 2as
Velocity attained by body in (T – t) sec
H
Þ 0 = (v1 – v2)2 – 2 × a × s
v = u – g (T – t)
u ( v1 - v2 ) 2
u Þ s=
+ gt 2a
= u – gT + gt = u – g
g A
If the distance between two cars is 's' then collision will
or v = gt ...................(2) take place. To avoid collision d > s
\Distance travelled in last t sec of its ascent
(v1 - v2 ) 2
1 1 \ d>
S = (gt ) t - gt 2 = gt 2 2a
2 2 where d = actual initial distance between two cars.
dx d 17. (c) If the body starts from rest and moves with constant
12. (c) v = = (2t2 + t + 5) = 4t + 1 m/s
dt dt acceleration then the ratio of distances in consecutive
dv d equal time interval S1 : S2 : S3 = 1 : 3 : 5
and a = = (4t + 1) ; a = 4 m/s2
dt dt Dx
18. (c) Instantaneous velocity v =
dv æ tö Dt
13. (a) = a0 çè1 - ÷ø
dt T By using the data from the table
t v
æ tö æ t2 ö 0 - ( -2) 6-0
v1 = = 2m/s, v2 = = 6m/s,
Þò dv = ò è Tø
a 0 ç 1 - ÷ dt Þ v = a 0 ç t - ÷ 1 1
0 0 è 2T ø
16 - 6
dx æ t
t2 ö v3 = = 10m/s
Q = v so, ò dx = ò v dt Þ x = ò a 0 çè t - 2T ÷ø dt 1
dt So, motion is non-uniform but accelerated.
0
t.me/Ebooks_Encyclopedia27. t.me/Magazines4all

DPP/ P 03 7
23. (c) For an inertial frame of reference, its acceleration should
19. (a) 1 2 2v
at = vt Þ t = be zero. As reference frame attached to the earth i.e. a
2 a rotating or revolving frame is accelerating, therefore, it
20. (b) x = 4(t – 2) + a(t – 2)2 will be non-inertial.
At t = 0, x = – 8 + 4a = 4a – 8 Thus (2) & (4) are correct, so correct answer is (c).
dx 24. (c) Average speed
v= = 4 + 2 a (t - 2)
dt Total distance 2x 2v1v 2
= = =
At t = 0, v = 4 – 4a = 4(1 – a) Total time x x v1 + v 2
+
v1 v 2
d2x
But acceleration, a = = 2a
dt 2 Total displacement
Average velocity =
-at bt Total time
21. (d) x = ae + be
Q It comes back to its initial position
dx d
Velocity v = = (ae -at + bebt ) \ Total displacement is zero.
dt dt Hence, average velocity is zero.
= a.e-at (-a) + bebt (b) = -aae-at + bbebt Sol. For Qs. 25-27. a = sin pt

-at bt 2
Acceleration = -aae (-a ) + bbe .b \ ò dv = ò 2sin pt dt or v = - cos pt + C
p
= aa 2 e-at + bb2 eb t 2 2
Acceleration is positive so velocity goes on increasing At t = 0, v = 0 \ C = or v = (1 - cos pt)
p p
with time.
Velocity is always non-negative, hence particle always moves
22. (d) (1) a = 3 sin 4t
along positive x-direction.
dv \ Distance from time t = 0 to t = t is
Þ = 3sin 4t
dt
t t
2 2æ 1 ö 2 2
S=ò (1 - cos pt) dt = ç t - sin pt ÷ = t - 2 sin pt
Þ ò dv = ò 3sin 4t dt + c 0
p pè p ø0 p p

-3
Þ v= cos 4t + c 2t 2
4 Also displacement from time t = 0 to t = - sin pt
p p2
For initial velocity, t = 0
3 2
v0 = - +C Distance from time t = t to t = 1s = meters.
4 p
25. (a) 26. (b) 27. (b)
3
At particular value of C = , v0 = 0 28. (d) Negative slope of position time graph represents that
4 the body is moving towards the negative direction and
Therefore, initial velocity may or may not be zero. if the slope of the graph decrease with time then it
(2) Acceleration = 0 represents the decrease in speed i.e. retardation in
Þ a = 3 sin4t = 0 Þ sin 4t = 0 motion.
np 29. (c) As per definition, acceleration is the rate of change of
Þ 4t = np Þ t= velocity,
4
r
where n = 0, 1, 2, ................. r dv
i.e. a = .
Therefore, the acceleration of the particle becomes zero dt
p If velocity is constant
after each interval of second. r
4 dv r
= 0, \ a = 0
(3) As acceleration is sinusoidal function of time, so dt
particle repeats its path periodically. Thus, the particle Therefore, if a body has constant velocity it cannot
comes to its initial position after sometime (period of have non zero acceleration.
function). 30. (d) The displacement is the shortest distance between
(4) The particle moves in a straight line path as it initial and final position. When final position of a body
performs S.H.M. coincides with its initial position, displacement is zero,
Since (1) & (3) are correct, hence correct answer is (d). but the distance travelled is not zero.
t.me/Ebooks_Encyclopedia27. t.me/Magazines4all

EBD_7156
8 DPP/ P 04
DAILY PRACTICE
PROBLEMS
PHYSICS
SOLUTIONS 04
1. (c) If a stone is dropped from height h
-2 2 g ± 8 g + 8 g - 2 ± 2
1 2 t2 = =
then h = gt ......... (i) 2g g
2
If a stone is thrown upward with velocity u then Taking + ve sign t2 = (2 - 2) / g
1 2
h = -ut1 + gt1 ......... (ii) t1 2/ g 1
2 \ = = and so on.
t2 (2 - 2) / g 2 -1
If a stone is thrown downward with velocity u then
4. (d) Interval of ball throw = 2 sec.
1 2
h = ut2 + gt2 ......... (iii) If we want that minimum three (more than two) ball
2 remain in air then time of flight of first ball must be
From (i), (ii) and (iii) we get greater than 4 sec.
1 2 1 2 T > 4 sec.
-ut1 + gt1 = gt ......... (iv)
2 2 2u
> 4sec Þ u > 19.6m / s
1 2 1 2 g
ut2 + gt2 = gt ......... (v)
2 2 for u = 19.6 First ball will just strike the ground(in sky)
Dividing (iv) and (v) we get Second ball will be at highest point (in sky)
Third ball will be at point of projection or at ground
1 (not in sky)
g (t 2 - t12 )
-ut1 2 5. (a) The distance covered by the ball during the last t
\ =
ut2 1 seconds of its upward motion = Distance covered by it
g (t 2 - t22 )
2 in first t seconds of its downward motion
1 2
t t2 - t2 From h = ut + gt
or - 1 = 2 12 2
t2 t - t2
1 2
h= gt [As u = 0 for it downward motion]
By solving t = t1t2 2
2. (c) Since direction of v is opposite to the direction of g 6. (d) In the positive region the velocity decreases linearly
and h so from equation of motion (during rise) and in the negative region velocity
increases linearly (during fall) and the direction is
1 2 opposite to each other during rise and fall, hence fall is
h = -nt + gt
2 shown in the negative region.
7. (a) For the given condition initial height h = d and velocity
Þ gt 2 - 2nt - 2h = 0
of the ball is zero. When the ball moves downward its
velocity increases and it will be maximum when the ball
2n ± 4n2 + 8 gh
Þ t= hits the ground & just after the collision it becomes
2g half and in opposite direction. As the ball moves
upward its velocity again decreases and becomes zero
né 2 gh ù at height d/2. This explanation match with graph (a).
Þ t= ê1 + 1 + 2 ú
gë n û 8. (c) Acceleration of body along AB is g cos q
1 2
1 2 1 Distance travelled in time t sec = AB = ( g cos q)t
3. (c) h = ut + gt Þ 1 = 0 ´ t1 + gt12 Þ t1 = 2 / g 2
2 2
Velocity after travelling 1m distance 1
From DABC, AB = 2R cosq; 2 R cos q = g cos qt 2
2
v 2 = u 2 + 2 gh Þ n 2 = (0) 2 + 2 g ´ 1 Þ n = 2 g
4R R
For second 1 meter distance t2 = or t = 2
g g
1 2 9. (b) It has lesser initial upward velocity.
1 = 2 g ´ t2 + gt2 Þ gt22 + 2 2 gt2 - 2 = 0
2 10. (b) At maximum height velocity n = 0
t.me/Ebooks_Encyclopedia27. t.me/Magazines4all

DPP/ P 04 9
We know that n = u + at, hence uuur
Velocity of car w.r.t. train (nct ) is towards
0 = u - gT Þ u = gT West – North
u 21. (a) As the trains are moving in the same direction. So the
When n = , then initial relative speed (v1 – v2 ) and by applying
2 retardation final relative speed becomes zero.
u u gT T
= u - gt Þ gt = Þ gt = Þt = From v = u - at Þ 0 = (v1 - v2 ) - at Þ t = v1 - v2
2 2 2 2 a
r r
T u 22. (d) Let v A and v B be the respective velocities of the
Hence at t = , it acquires velocity .
2 2 particles at A and B. The relative velocity of particle at
A. w.r.t. to that at B is given by
2h t h r r r r
11. (b) t= Þ 1 = 1 v A - v B = v A + (- v B )
g t2 h2

12. (c) Speed of the object at reaching the ground n = 2 gh


If heights are equal then velocity will also be equal.
A B
10 vA=25m/s
13. (b) S = 10 + (2 ´ 3 - 1) = 35m
3rd 2 300m

10 S rd 7
S nd = 10 + (2 ´ 2 - 1) = 25 Þ 3 = (see figure). From triangle law of velocities if OP and
2 2 S nd 5
2 r r
PQ represent v A and - v B , then the required relative
14. (c) Sn µ (2n - 1) . In equal time interval of 2 seconds r
velocity v R is given by OQ .
Ratio of distance = 1 : 3 : 5
15. (c) Net acceleration of a body when thrown upward r
| vR | = 252 + 202 = 625 + 400 = 32.02 m / s
= acceleration of body – acceleration due to gravity
=a–g
16. (d) The initial velocity of aeroplane is horizontal, then the 25 æ 5ö
vertical component of velocity of packet will be zero. If Ð PQO = q, then tan q = Þ q = tan -1 ç ÷
20 è 4ø
2h
So t = vA 25
g P
vB=20m/s O
Totallength 50 + 50 100
17. (b) Time = = = = 4sec
Relative velocity 10 + 15 25 20
18. (d) Relative velocity
= 10 + 5 = 15 m/sec
Q
150
\t = = 10 sec Thus, the particle at A, appears to approach B, in a direction
15 making an angle of tan–1 (5/4) with its direction of motion.
19. (a) When two particles moves towards each other then
Let us draw a line from A, as AC, such that Ð BCA is equal
v1 + v2 = 4 ....... (i)
When these particles moves in the same direction then to q .
v1 – v2 = 4 ....... (ii) 300m B
By solving v1 = 5 and v2 = 1 m/s A
uuur uur uur
20. (b) nct = nc - nt
uuur uur uur
n ct = n c + (-nt ) M
vc
C
vct Thus, to B, A appears to move along AC. From B, draw a
perpendicular to AC as BM.
BM is the shortest distance between them.
4
45° \ BM = ABcos q = 300 ´ = 187.41 m
41
Also, AM = AB sin q = 234 .26 m
-vt vt \ time taken to cover a distance
t.me/Ebooks_Encyclopedia27. t.me/Magazines4all

EBD_7156
10 DPP/ P 04
AB = 234.26 m with a velocity of 32.02 m/s 25. (b) Vplane = 100 m/s
234.26
= = 7.32 sec.
32.02
23. (d) Since the wind is blowing toward the east, the plane
must head west of north as shown in figure. The velocity
of the plane relative to the ground vr pg will be the sum of vplane
the velocity of the plane relative to the air vr pa and the
velocity of the air relative to the ground vr ag.
(i) 1. The velocity of the plane relative to the ground is
given by equation :
r r r
v pg = v pa + v ag vwind vtotal
2. The sine of the angle q between the velocity of
the plane and north equals the ratio of vag and
vpa . Vtotal = (20)2 + (100)2 + 2 ´ 20 ´ 100 ´ cos135°

v ag 90 km / h æ 1 ö
sin q = = = 0.45 \ q = 26.74 = 400 + 10000 + 2 ´ 20 ´ 100 ´ ç - ÷
v pa 200 km / h è 2ø
(ii) Since vag and vpg are perpendicular, we can use the = 87 m/s
Pythagorean theorem to find the magnitude of vr pg.
v2pg = v2ag + v2pg v wind
26. (c) tan f = vwind
vplane vtotal
vpg = v 2pa 2
- v ag = (200 km / h )2 - (90 km / h )2
= 179 km/h. 20
=
100
r r 1r
24. (a) Using, S = uT + aT 2
2 æ 20 ö
\ f = tan -1 ç
è 100 ÷ø f
1 2 vplane
(i) -40 = 10T - gT u=10m/s
2 u vplane
2 27. (d) cos f =
or -40 = 10T - 5T 40m v total

v v plane 100
\ v total = = m /s
or 5T 2 - 10T - 40 = 0 h cos f cos f
28. (a) If components of velocities of boat relative to river is
2
10 + 10 - 4 ´ 5(-40) 10 + 100 + 800 same normal to river flow (as shown in figure) are same,
or T= = both boats reach other bank simultaneously.
2´5 10
10 + 30
= = 4 sec.
10 Boat 2 v Boat 1
v q q
2 ´ 10 River
(ii) t = = 2 sec.
g
29. (a) Both statement - 1 & statement - 2 are correct and
(iii) v = 10 + g × 2 = 30 m/s
statement - 2 is correct explanation of statement - 2
30. (d) Statement - 1 is true but statement - 2 is false.
t.me/Ebooks_Encyclopedia27. t.me/Magazines4all

DPP/ P 05 11

DAILY PRACTICE
PROBLEMS
PHYSICS
SOLUTIONS 05
5. (b) Time taken to cross the river along shortest possible
æ 90° ö path is given by
1. (d) Dv = 2v sin ç
è 2 ÷ø
d
t=
1 v - u2
2
= 2v sin 45° = 2v ´ = 2v
2 v = velocity of boat in still water
u = velocity of river water
2p 2p d = width of river
= 2 ´ rw = 2 ´ 1 ´ = cm/s
60 30 15 1
\ =
10 60 5 - u2
2
æ qö
2. (b) Dv = 2v sin ç ÷ = 2 ´ 5 ´ sin 45° =
è2ø 2 Þ u = 3 km/h
320
Dv 10 / 2 1 6. (d) Here d = 320 m = km
\ a= = = m/s2 1000
Dt 10 2 t = 4 min
3. (c) For motion of the particle from (0, 0) to (a, 0) 5
ur ur v= u
F = - K (0iˆ + ajˆ) Þ F = - Ka $j 3
r
Displacement r = (aiˆ + 0 ˆj ) - (0iˆ + 0 ˆj ) = ai$ Putting values in t =
d
, u = 60 m/min
So work done from (0, 0) to (a, 0) is given by v - u2
2

ur r
W = F . r = - Ka $j . ai$ = 0 P Q R
7. (c) = =
For motion (a, 0) to (a, a) sin q1 sin q2 sin150°
ur
F = - K (ai$ + a $j ) and displacement 1.93 R
Þ =
r sin q1 sin150°
r = (aiˆ + ajˆ) - (aiˆ + 0 ˆj ) = a $j
So work done from (a, 0) to (a, a) 1.93 ´ sin150° 1.93 ´ 0.5
Þ R= = =1
ur r sin q1 0.9659
W = F . r = - K (ai$ + a $j ). a $j = - Ka 2
150°
So total work done = - Ka 2
P Q
4. (d) q2 q1
T T cos q
R
q

T sin q 8. (b)
P T
T cos 30°
W
30°
As the metal sphere is in equilibrium under the effect
of the three forces therefore
ur ur uur
T + P +W = 0 T sin 30° 30 N
From the figure
T cos q = W ................... (i)
T sin q = P ................... (ii)
From equation (i) and (ii) we get
P = W tan q and T 2 = P 2 + W 2
W
t.me/Ebooks_Encyclopedia27. t.me/Magazines4all

EBD_7156
12 DPP/ P 05
From the figure ur ur ur ur
T sin 30° = 30 ...(i) 16. (b) | A ´ B | = A. B
T cos 30° = W ...(ii) Þ AB sin q = AB cos q
By solving equation (i) and (ii) we get Þ tan q = 1
W = 30 3 N and T = 60 N \ q = 45°
ur ur
9. (c) Relative velocity = (3iˆ + 4 ˆj ) - (-3iˆ - 4 ˆj ) = 6iˆ + 8 ˆj 17. (a) P . Q = 0

10. (c) Þ a 2 - 2a - 3 = 0
vr
Þa=3
r uur ur
18. (a) S = r2 - r1
ur ur
vm W = F .S
30°
= (4$i + $j + 3k$ ).(11$i + 11$j + 15k$ )
90°
= (4 × 11 + 1 × 11 + 3 × 15) = 100 J
ur ur ur
19. (c) A = 3iˆ - 2 ˆj + kˆ, B = iˆ - 3 ˆj + 5kˆ , C = 2iˆ + ˆj - 4kˆ

vr 1 r
sin 30° = = A = 32 + ( -2) 2 + 12 = 9 + 4 + 1 = 14
vm 2
r
vm 0.5 B = 12 + ( -3) 2 + 52 = 1 + 9 + 25 = 35
Þ vr = = = 0.25 m/s
2 2 r
11. (a) To cross the river in minimum time, the shift is given C = 2 2 + 12 + ( -4) 2 = 4 + 1 + 16 = 21

du
by . As B = A2 + C 2 therefore ABC will be right angled
v triangle.
12. (d) Relative velocity = 10 + 5 = 15 m/s. r r
F1.F2
120 20. (b) cos q =
Time taken by the bird to cross the train = = 8 sec F1 F2
15
r ur r
13. (d) v = w ´ r (5iˆ + 10 ˆj - 20kˆ).(10iˆ - 5 ˆj - 15kˆ)
=
$i $j k$ 25 + 100 + 400. 100 + 25 + 225
= 3 -4 1 = -18i$ - 13 $j + 2 k$ 50 - 50 + 300
=
5 -6 6 525. 350
ur ur ur ur 1
14. (d) | A ´ B | = 3 ( A . B) Þ cos q =
2
AB sin q = 3 AB cos q
\ q = 45°
Þ tan q = 3
r r r r
Þ q = 60° 21. (a) r = a + b + c = 4iˆ - ˆj - 3iˆ + 2 ˆj - kˆ = iˆ + ˆj - kˆ
ur ur ur
Now | R | = | A + B | r
r iˆ + ˆj - kˆ iˆ + ˆj - kˆ
rˆ = = =
|r| 2 2
1 + 1 + (-1) 2
= A2 + B 2 + 2 AB cos q 3

æ1ö
= A2 + B 2 + 2 AB ç ÷
è 2ø D C
2 2 1/ 2
= ( A + B + AB )
r r ur 300 m
15. (a) t = r ´ F = (7$i + 3 $j + k$ )(-3$i + $j + 5k$ )
22. (a)
$i $j k$
r A B
t= 7 3 1 = 14$i - 38 $j + 16k$ 400 m
uuur uuur uuur
-3 1 5 Displacement AC = AB + BC
t.me/Ebooks_Encyclopedia27. t.me/Magazines4all

DPP/ P 05 13

AC = (AB)2 + (BC) 2 = (400)2 + (300) 2 = 500 m r r æ - ˆj + kˆ ö


Unit vector perpendicular to A and B is ç ÷.
Distance = AB+BC =400+300=700 m è 2 ø
r r r
23. (a) A = 3iˆ - 2ˆj + k,
ˆ B = ˆi - 3jˆ + 5k,
ˆ C = 2iˆ - ˆj + 4kˆ Any vector whose magnitude is k (constant) times
r r 2iˆ + ˆj + kˆ
A = 32 + ( -2) 2 + 12 = 9 + 4 + 1 = 14 (2iˆ + ˆj + kˆ) is parallel to A so, unit vector
6
r r
B = 12 + ( -3) 2 + 52 = 1 + 9 + 25 = 35
is parallel to A .
uv
C = 22 + 12 + ( -4) 2 = 4 + 1 + 16 = 21 r r r r
28. (b) A+ B = A- B
As B = A 2 + C2 therefore ABC will be right angled Þ A2 + B2 + 2AB cosq = A2 + B2 + 2AB cosq
triangle. Hence cosq = 0 which gives q = 90°
r r Also vector addition is commutative.
24. (a) A ´ B = 0 \ sin q = 0 \q = 0°
r r r r
Two vectors will be parallel to each other. Hence A + B = B + A
25. (a), 26 (b), 27. (b) r r
29. (a) Let P and Q are two vectors in opposite direction,
iˆ ˆj kˆ r r r r
r r then their sum P + (-Q) = P - Q
A´ B = 2 1 1
= iˆ(1 - 1) - ˆj(2 - 1) + kˆ(2 - 1) r r
If P = Q then sum equal to zero.
1 1 1
30. (d) The resultant of two vectors of unequal magnitude
= - ĵ + kˆ given by R = A2 + B 2 + 2 AB cos q cannot be zero
for any value of q.
t.me/Ebooks_Encyclopedia27. t.me/Magazines4all

EBD_7156
14 DPP/ P 06
DAILY PRACTICE
PROBLEMS
PHYSICS
SOLUTIONS 06
1. (b) Given, that y = 3 x – (1/2) x2 ....(1) 2u sin q 2u sin q
8. (b) T = Þ2= Þ u sin q = g
The above equation is similar to equation of trajectory g g
of the projectiles
u 2 sin 2 q g 2 g
g H= = = =5m
y = tan q x – 1/2 x2 ....(2) 2g 2g 2
u 2 cos 2 q
Comparing (1) & (2) we get 9. (b) Let u1 and u2 be the initial velocities respectively. If h1
and h 2 are the heights attained by them, then
tan q = 3 Þ q = 60º
u12 u 22 sin 2 q
g h1 = and h2 = ...(1)
and 1/2 = (1/2) 2 2
2g 2g
u cos q
The times of ascent of balls are equal,
Þ u2 cos2 q = g Þ u2 cos2 60° = 10
we have t = u1/g = u2 sin q/g
Þ u2 (1/4) = 10 Þ u2 = 40 Þ u = 2 10 m/s \ u1 = u2 sin q ... (2)
2. (a) For angle of elevation of 60º, we have maximum height 2
h1 u
From eq. (1) = 2 12 ... (3)
u 2 sin 2 60º 3u 2 h2 u 2 sin q
h1 = =
2g 8g
h1 1
For angle of elevation of 30º, we have maximum height From (2) & (3), =
h2 1
u 2 sin 2 30 º u 2 h1 3
h2 = = ; = u 2 sin 2 q u 2 sin 2 (90 - q)
2g 8g h 2 1
10. (a) h1 = and h2 =
3. (b) Total time of flight = 2 × time taken to reach max. height 2g 2g
Þ t2 = 2t1 Þ t2/t1 = 2/1 \ h1 + h2 = u2/2g (sin 2q + cos2q)
4. (d) vx = dx/dt = 2ct, vy = dy/dt = 2bt
982
\v= v 2x + v 2y = u2/2g = = 490
= 2t c2 + b 2 2 ´ 10
5. (c) vy = dy/dt = 8 – 10 t = 8, when t = 0 (at the time of h1 – h 2 = 50, \ h1 = 270 m and h 2 = 220 m
projection.) 11. (c) Greatest height attained

v 2x + v 2y = u 2 sin 2 q
vx = dx/dt = 6, v = 8 2 + 6 2 = 10 m/s h= ... (1)
2g
6. (a) Horizontal component of velocity
Horizontal range
vx = ux = u cos q = 30 × cos 30º = 15 3 m/s
2
Vertical component of the velocity u 2 sin 2q 2u sin q cos q
R= = ... (2)
vy = u sin q – gt = 30 sin 30º – 10 × 1 = 5 m/s g g
v2 = vx2 + vy2 = 700 Þ u = 10 7 m/s Given that R=2h
7. (a) Let ux and uy be the components of the velocity of the
2u 2 sin q cos q 2 u 2 sin 2 q
particle along the x- and y-directions. Then Þ =
ux = dx/dt = u0 and uy = dy/dt = wa cos wt g 2g
Integration : x = u0 t and y = a sin wt Þ tan q = 2 ... (3)
Eliminating t : y = a sin (wx/u0)
This is the equation of the trajectory Hence sin q = 2/ 5 , cos q = 1/ 5 ,
At t = 3p/2w, we have, \ From (2) R = 4u2/5g
x = u0 3p/2w and y = a sin 3p/2 = – a
u 2 sin 2q
\ The distance of the particle from the origin is 12. (d) R = = (19.6)2 sin 90º /10 = 39.2 m
g
éæ 3pu ö 2 ù Man must run (67.4m – 39.2 m) = 28.2 m in the time
2
x +y 2 = êç 0
÷ + a2 ú taken by the ball to come to ground. Time taken by the
êëè 2w ø úû
ball.
t.me/Ebooks_Encyclopedia27. t.me/Magazines4all

DPP/ P 06 15
(b) Let at the time of impact, vx and vy be the horizontal
2u sin q 2´ 19.6 sin 45º 4
t= = = = 2.82 sec and vertical velocities respectively, then
g 9.8 2 vx = 700 × cos 37° = 700 × 0.8 = 560 cm/s
28 . 2 m and vy = – 700 × sin 37° + 1000 × (5/7)
Velocity of man = = 10 m/s = – 700 × 0.6 + (5000/7) = – 420 + 714.3
2 . 82 sec
= + 294.3 cm/sec (downward)
13. (a) Referring to (fig.) let P be a point on the trajectory B
whose co-ordinates are (4, 4). As the ball strikes the
ground at a distance 14 metre from the wall, the range is m
0c h
4 + 14 = 18 metre. The equation of trajectory is 50
vx
x2 / s
y = x tan q -(1/2) g cm
0
70 v
u 2 cos 2 q 37º
vy
A C
Y O 40 0cm

Velocity of the ball at the time of collision

v sina v= ( v 2x + v 2y )
v
·
usinq

u P v cosa \v= [(560 ) 2 + ( 294 .3) 2 ] = 632.6 cm/sec


q h
vy 294 .3
A ucosq B X Again tan q = =
vx 560

é gx ù æ 294 .3 ö
or y = x tan q ê1 - 2 2 ú or q = tan–1 ç ÷ = 27° 43'
ë 2 u cos q. tan q û è 560 ø
15. (d) Initial velocity is constant let the ball touches the
é 2u 2 x ù
ground at an angle q and velocity 3u
or y = x tan q ê1 - g sin q cos q ú
ëê ûú Hence 3u cos q = u or cos q = 1/3 or sin q = 8 /3
é xù The vertical component of velocity at the ground
= x tan q ê1 - ú ... (1)
ë Rû 3 8
Here x = 4, y = 4 and R = 18 = 3u sin q = = 8u
u 3
é 4ù æ7ö
\ 4 = 4 tan q ê1 - ú = 4 tan q ç ÷
ë 18 û è9ø 20 m
or tan q = 9/7, sin q = 9/ 130 and

cos q = 7/ 130 q u
Again R = (2/g) u2 sin q cos q
= (2/9.8) × u2 × (9/ 130 ) × (7/ 130 )
For a freely falling body it covers 20 m to acquire veloc-
18´ 9.8 ´ 130 ´ 130 98 ´ 13
u2 = = = 182, ity 8u
2´ 9 ´ 7 7
\ ( 8 u)2 – 0 = 2 × 9.8 × 20 or u = 7 m/s
u = 182 metre per second.
16. (b) The horizontal range of the projectile on the ground
14. (b) The situation is shown in fig.
(a) Let the ball collide after t sec 2h 2 ´ 10
From fig. OC = OB cos 37° = 500 cos 37° R= u g ÞR= 2 10
= 2 . 2 = 2m
= 500 × 0.8 = 400 cm ...(1)
Horizontal velocity = 700 × cos 37º 17. (a) R = ut Þ t = R/u = 12/8
\ OC = 700 × cos 37° × t Now h = (1/2) gt2 = (1/2) × 9.8 × (12/8)2 = 11 m
= 700 × 0.8 × t = 560 t ... (2) 18. (b) The situation is shown in the adjoining figure.
From eqs. (1) and (2) 560 t = 400 The time taken by the body is equal to the time taken
or t = (5/7) sec. by the freely falling body from the height 29.4 m. Initial
Now h = (1/2) g t2 = (1/2) × 1000 × (5/7)2 = 255.1 cm velocity of body
t.me/Ebooks_Encyclopedia27. t.me/Magazines4all

EBD_7156
16 DPP/ P 06
Here uy = – uy' = – 5.0 m/s and t = 2.0 s
30º \ h = (–5.0) × 2.0 + 1/2 × 10 × (2.0)2
v = – 10 + 20 = 10 meter
29.4 m 21. (b) Let the ball B hits the ball A after t sec
The X-component of velocity of A is
v0 cos 37º = 700 cos 37º
A B The X-compoment of position of B is 300 cos 37º
u sin q = 9.8 sin 30º = 4.9 m/s
The collision will take place when the X-coordinate of
From the relation, h = u sin q t + (1/2) gt2,
A is the same as that of B.
we get 29.4 = 4.9 t + (1/2) × 9.8 t2 Þ t = 2 sec
19. (b) The horizontal and vertical velocities of the bomb are As the collision takes place at a time t, hence
independent to each other. The time taken by the bomb 700 cos 37º × t = 300 cos 37º
to hit the target can be calculated by its vertical mo- or t = (300/700) = (3/7) sec
tion. Let this time be t. Putting h = 490 m and g = 9.8 m/
In this time the ball B has fallen through a distance
s2 in the formula h = 1/2 gt2,
we have 490 = (1/2) × 9.8 × t2, y = – 1/2 gt2 (Free fall of body B)
= – 1/2 × 980 × (3 / 7)2 = – 90 cm
2 ´ 490
\t= = 10 sec Hence the ball B falls a distance 90 cm
9.8
22. (b)
The bomb will hit the target after 10 sec of its dropping. (1) Because force is constant hence acceleration will be
The horizontal velocity of the bomb is 60 km/hr which constant. When force is in oblique direction with initial
is constant. Hence the horizontal distance travelled by velocity, the resultant path is parabolic path.
the bomb in 10 sec (horizontal velocity × time)
= 60 km/hr × 10 sec 2u sin q
(2) Total time of flight = T = ,
= 60 km/hr × 10/(60 × 60) hr = 1/6 km g
Hence the distance of aeroplane from the enemy post
is 1/6 km = 1000/6 m = 500/3 meter. u 2 sin 2 q
The trajectory of the bomb as seen by an observer on Maximum height attained H =
2g
the ground is parabola. Since the horizontal velocity of
the bomb is the same as that of the aeroplane, the fall- H u sin q
ing bomb will always remain below the aeroplane. Hence Now =
T 4
the person sitting inside the aeroplane will observe the
bomb falling vertically downward. (3) Initially the height of the monkey = MB = y = x tan q
20. (a) The angle of projection of the ball is q0 ( = 30º) and the Let the monkey drop to along line MA and the bullet
velocity of projection is u ( = 10 m/s). Resolving u in reach along the parabolic path OA. If both reach at A
horizontal and vertical components, simultaneously, the monkey is hit by the bullet.
we have horizontal component,
gx 2
ux = u cos q0 = 10 cos 30º = 8.65 m/s AB = x tan q – ,
and vertical component (upward), 2u 2 cos 2 q
uy = u sin 30º = 5.0 m/s \ MA = MB – AB
If the ball hit the ground after t sec of projection, then
the horizontal range is R = ux × t = 8.65 t meter gx 2
MA = x tan q – x tan q +
R 17 . 3m 2u 2 cos 2 q
\t= = = 2.0 s
8 .65 8 .65 m / s
gx 2
If h be the height of the tower, = ....(i)
then h = u'y t + (1/2) g t2, 2u 2 cos 2 q
where uy' is the vertical component M
v
(downward) of the velocity of the ball. A
Y
u
uy
X
ux
q
O B
h
Tower Time taken by the bullet to reach point A,

X x
t= ....(ii)
R=17.3m u cos q
t.me/Ebooks_Encyclopedia27. t.me/Magazines4all

DPP/ P 06 17
Hence from (1), MA = (1/2) gt2 25. (c) If the ball hits the n th step, the horizontal
The monkey drops through distance (1/2) gt2 in the and vertical distances traversed are nb and nh
same time. So the monkey is hit by the bullet. respectively.

u 2 sin 2q u
(4) The range R =
g h 1
b
u2 2
\ Maximum range Rmax = d = ....(iii)
g

u 2 sin 2 q
Height H = nth step
2g
\ Maximum height R

u2 Let t be the time taken by the ball for these horizontal


Hmax= ....(iv) and vertical displacement. Then velocity along
2g
horizontal direction remains constant = u ; initial vertical
From (iii) & (iv), Hmax = d/2 velocity is zero
\ nb = ut ....(1)
u 2 sin 2q
23. (a) Range of projectile, R = nh = 0 + (1/2) gt2 ....(2)
g From (1) & (2) we get
The range is same for two angle q1 and q2 provided nh = (1/2) g (nb/u)2
q2 = 90º – q1
2hu 2
2
u sin 2q1 Þ n= (eleminating t)
gb 2
At an angle q1, range R1 =
g 26. (a) y = (1/2) gt2 (downward)
At an angle of projection q2, Þ 1000 = (1/2) × 10 × t2 Þ t = 14.15 sec

u 2 sin 2q 2 æ 144´10 3 ö
Range R2 = ç ÷
g x = ut = ç 60´60 ÷ × 14.15= 571.43 m
è ø
u 2 sin 2 ( 90 º - q1 ) u 2 sin 2q1 27. (b) Horizontal component of velocity
= = = 720 × 5/8 = 200 m/s
g g Let t be the time taken for a freely falling body from 490.
Þ R1 = R2 Then y = (1/2) gt2
\ other angle = 90º - q1 = 90º – 15º = 75º Þ 490 = (1/2) × 9.8 × t2 Þ t = 10 second
Now horizontal distance = Velocity × time
= 200 × 10 =2000m
2u sin q
24. (a) t1 = Hence the bomb missed the target by 2000 m
g 28. (a) Since W = D K implies that the final speed will be same.
2u sin(90º -q) 2u cos q 29. (a) The time of flight depends only on the vertical
t2 = = component of velocity which remains unchanged in
g g
collision with a vertical wall.
2 u 2 sin 2q 2 30. (a) In statement-2, if speed of both projectiles are same,
t1 t2 = = .R horizontal ranges will be same. Hence statement-2 is
g g g
correct explanation of statement-1.
where R is the range, Hence t1t2 µ R
t.me/Ebooks_Encyclopedia27. t.me/Magazines4all

EBD_7156
18 DPP/ P 07
DAILY PRACTICE
PROBLEMS
PHYSICS
SOLUTIONS 07
(1) (b) We have angular displacement (6) (d) We know that
linear displacement Circumference 2pr
= radius of path Time period = =
Criticalspeed gr
DS 2 ´ 22 ´ 4
Þ Dq = = = 4 sec
r 7 ´ 10 ´ 4
Here, DS = n (2pr) = 1.5 (2p × 2 × 10–2) = 6p × 10–2
Circumference 2 pr
6 p ´ 10 -2 (7) (b) Velocity = =
\ Dq = = 3p radian Time of revolution 60
2 ´ 10 -2
2p ´ 1 p
® Total angular displacement = = cm/s
60 30
(2) (a) We have w av =
Total time
2 2
For first one third part of circle, æ pö æ pö
Change in velocity Dv = çè 30 ÷ø + çè 30 ÷ø
S1 2 pr / 3
angular displacement, q1 = =
r r p
= 2 cm/s
For second one third part of circle, 30
2pr / 3 2 p (8) (a) Let the radius of the orbit be r and the number of
q2 = = rad revolutions per second be n. Then the velocity of electron
r 3
is given by v = 2pnr,
Total angular displacement,
q = q1 + q2 = 4p/3 rad v2 4p 2 r 2 n 2
Total time = 2 + 1 = 3 sec \ Acceleration a = = = 4 p2 r n2
r r
® 4p / 3 4p 2p Substituting the given values, we have
\ w av = rad/s = = rad/s a = 4 × (3.14)2 × (5.3 × 10–11) (6.6 × 1015)2
3 6 3
= 9.1 × 1022 m/s2 towards the nucleus.
(3) (c) Angular speed of hour hand,
The centripetal force is
Dq 2p FC = ma = (9.1 × 10–31) (9.1 × 1022)
w1 = = 12 ´ 60 rad/sec = 8.3 × 10–8 N towards the nucleus.
Dt
(9) (b) Given that radius of horizontal loop
Angular speed of minute hand, r = 1 km = 1000 m
2p w 12 9000 ´ 5
w2 = rad/sec Þ 2 = Speed v = 900 km/h = = 250 m/s
60 w1 1 18
1 2 dq v2 250 ´ 250
(4) (d) We have q = w0t + at Þ = w0 + at Centripetal acceleration ac= = = 62.5 m/s2
2 dt r 1000
This is angular velocity at time t.
Now angular velocity at t = 2 sec will be Centripetal acceleration ac 62.5
\ Gravitational acceleration = = = 6.38 : 1
g 9.8
æ dq ö
w = çè dt ÷ø = w0 + 2a = 1 + 2 × 1.5 = 4 rad/sec 2
t =2sec æ 5ö
2 ç 18 ´ ÷
(5) (d) The distance covered in completing the circle is v è 18 ø
(10) (b) We know that, tan q = =
2pr = 2p × 10 cm rg 100 ´ 10
2 pr 2p ´ 10
The linear speed is v = = = 5p cm/s 1 1
t 4 = Þ q = tan–1
40 40
The linear acceleration is,
v
v2 (5p )2 (11) (a) The angular velocity is w =
a= = = 2.5 p2 cm/s2 r
r 10 Hence, v = 10 m/s
This acceleration is directed towards the centre of the circle r = 20 cm = 0.2 m \ w = 50 rad/s
t.me/Ebooks_Encyclopedia27. t.me/Magazines4all

DPP/ P 07 19
(12) (b) Given that w = 1.5t – 3t2 + 2 (18) (a) Let W = Mg be the weight of the car. Friction
force = 0.4 W
dw
a= = 1.5 – 6t
dt M v2 W v2
Centripetal force = =
When, a = 0 r gr
Þ 1.5 – 6t = 0
1.5 W v2
Þt= = 0.25 sec 0.4 W =
6 gr
(13) (c) Given v = 1.5 t2 + 2t Þ v2 = 0.4 × g × r = 0.4 × 9.8 × 30 = 117.6
Linear acceleration a = dv/dt = 3t + 2 Þ v = 10.84 m/sec
This is the linear acceleration at time t (19) (c) Let v be the speed of earth's rotation.
Now angular acceleration at time t We know that W = mg
a 3t + 2 3 m v2
a= Þa= Hence W = mg –
r 2 ´ 10 -2 5 r
Angular acceleration at t = 2 sec
3 m v2
3 ´ 2+2 8 or mg = mg –
5 r
(a)at t = 2sec = = × 102
2 ´ 10 -2 2
2 m v2 2g r
= 4 × 102 = 400 rad/sec2 \ mg = or v2 =
3 r 5
(14) (a) Angular displacement after 4 sec is
1 2 1 2 1 2 ´ 9.8 ´ (6400 ´ 103 )
q = w0t + at = at = × 4 × 42 = 32 rad Now v2 =
2 2 2 5
Angular velocity after 4 sec Solving, we get v = 5 × 109 m/sec,
w = w0 + at = 0 + 4 × 4 = 16 rad/sec
(15) (a) Given a = 3t – t2 æ 2g ö
w= çè ÷ø = 7.8 × 104 radian/sec.
5r
dw
Þ = 3t – t2 Þ dw = (3t – t2)dt (20) (a) Let T be the tension produced in the stretched string.
dt
The centripetal force required for the mass m to move in a
3t 2 t 3 circle is provided by the tension T. The stretched length of
Þ w= - +c the spring is r (radius of the circle). Now,
2 3
Elongation produced in the spring = (r – l0)
At t = 0, w = 0 Tension produced in the spring,
3t 2 t 3 T = k (r – l0) ........ (1)
\ c = 0, w = - Where k is the force constant
2 3 Linear velocity of the motion v = 2p r n
Angular velocity at t = 2 sec, (w)t = 2 sec
mv2 m(2prn)2
3 8 10 \ Centripetal force = =
= (4) - = rad/sec r r
2 3 3
= 4p2 r n2 m ........ (2)
Since there is no angular acceleration after 2 sec
Equating equation. (1) and (2), we get
\ The angular velocity after 6 sec remains the same.
k (r – l0) = 4p2 r n2 m (\ T = mv2/r)
(16) (d) ˆix + yj
ˆ , x = r cos q, Þ kr – k ll0 = 4 p r n m
2 2

y = r sin q where q = wt r (k – 4p2 n2 m) = k l0


k l0
r = î ( r cos wt) + ĵ (r sin wt) Þ r= ........(3)
(k - 4 p 2 n 2 m)
v = dr/dt = – î (wr sin wt) – ĵ (w r cos wt) Substituting the value of r in eqn. (1) we have
a = d2 r/dt2 = – w2 r
(17) (c) Let R be the normal reaction exerted by the road on the é kl 0 ù 4p 2 n 2 ml 0 k
T=k ê 2 2
- l 0 ú or T =
car. At the highest point, we have ë (k - 4p n m) û (k - 4 p2 n 2 m)
m v2
= mg – R, R should not be negative. (21) (c) Two types of acceleration are experienced by the car
(r + h)
(i) Radial acceleration due to circular path,
Therefore v2 £ (r + a)g = (8.9 + 1.1) × 10
or v2 £ 10 × 10v £ 10 m/sec v2 (30)2
ar = = = 1.8 m/s2
\ vmax = 10 m/sec r 500
t.me/Ebooks_Encyclopedia27. t.me/Magazines4all

EBD_7156
20 DPP/ P 07
(ii) A tangential acceleration due to increase of tangential
speed given by
at = Dv/Dt = 2 m/s2
Radial and tangential acceleration are perpendicular to each
other.
Net acceleration of car
a= 2
a 2r + a 2t = (1.8) 2 + (2) 2 = 2.7 m/s
(22) (a) For A :

N sin q mg rg
Þ = Þ tan q = 2
N cos q 2
mv / r v
mv A2
Required centripetal force = r r rg
3l But tan q = \ = 2
h h v
(net force towards centre = T1)
This will provide required centripetal force Þ v= hg = 9.8 ´ 9.8 ´ 10 -2 = 0.98 m/s
mv A2 (24) (d) (1) Centripetal force is not a real force. It is only the
particle at A, \ T1 = requirement for circular motion. It is not a new kind of
3l force. Any of the forces found in nature such as gravitational
force, electric friction force, tension in string, reaction force
m(vB2 ) may act as centripetal force.
For B : Required centripetal force =
2l (3) Work done by centripetal force is always zero.
Remember w i.e. angular velocity, of all the particles is same
v2
vA vB vC (25) (a) We know, a =
\ w= = = r
3l 2l l
When a system of particles rotates about an axis, the (10)2
Hence v = 10 m/s, r = 5 m \ a = = 20 m/s2
angular velocity of all the particles will be same, but their 5
linear velocity will be different, because of different (26) (a) Given that the mass of the particle, m = 2 kg
distances from axis of rotation i.e. v = rw. Radius of circle = 3 m
Angular velocity = 60 rev/minute
2mv 2A 60 ´ 2p
Thus for B, centripetal force =
9l = rad/sec = 2p rad/sec
60
Because the angle described during 1 revolution is 2p radian
2mv 2A
Net force towards the centre T2 – T1 = The linear velocity v = rw = 2p × 3 m/s = 6p m/s
9l
v2 (6p )2
The centripetal acceleration = = m/s2
2mv 2A 5mv 2A r 3
Þ T2 = + T1 = (Putting value of T1)
9l 9l = 118.4 m/s2
For C :
mv2
(27) (a) F = = mrw2
mvC2 mvA2 r
Centripetal force. = Here m = 0.10 kg, r = 0.5 m
3l 9l
Net force towards centre = T3 – T2 2pn 2 ´ 3.14 ´ 10
and w = = = 2 rad/s
2 2 t 31.4
mvA mvA F = 0.10 × 0.5 × (2)2 = 0.2
\ T3 – T 2 = Þ T3 = + T2
9l 9l (28) (a) In non-uniform circular motion acceleration vector
makes some angle with radius hence it is not
6mvA2 perpendicular to velocity vector.
T3 = (on putting value of T2)
9l (29) (c) If speed is increasing there is a tangential acceleration.
Net acceleration is not pointing towards centre.
mv2 (30) (b) Both statements are true but statement-2 is not correct
(23) (b) N cos q = and N sin q = mg
r explanation for statement-1.
t.me/Ebooks_Encyclopedia27. t.me/Magazines4all

DPP/ P 08 21

DAILY PRACTICE
PROBLEMS
PHYSICS
SOLUTIONS 08
(1) (a). O centrifugal force acting upward at point D, should be equal
or greater than the force mg acting downward at point D
q l should be equal or greater than the force mg acting
T Tcosq downward. Therefore
T q m v2D
·A · Tsinq · r
³ mg or v 2D ³ r g
mg
mg From equation (2) v 2D = 2g (h – 2r),

5
Form figure \ 2g (h – 2r) ³ r g Þ h ³ r
2
T cos q = mg ...... (1)
(3) (d). See fig, Here v = 360 km/hr = 100 m/sec
2 2
mv mv
T sin q = = ...... (2)
r l sin q N

mg
mg
Form eq. (1), T = · N
cosq
When the string is horizontal, q must be 90º i.e.,cos 90º = 0
N
mg
\T= =¥
0
mg
Thus the tension must be infinite which is impossible, so
mv2
the string can not be in horizontal plane. At lower point, N – mg = ,
The maximum angle q is given by the breaking tension of R
the string in the equation T cos q = m.g
mv2
Here T (Maximum) = 8 N and m = 0.4 kg N = weight of the flyer = mg +
\ 8 cos q = 0.4 × g = 0.4 × 10 = 4 R

1 70 ´ (10000)
cos q = (4/8) = , q = 60º N = 70 × 10 + = 2100 N
2 500
The angle with horizontal = 90º – 60º = 30º
mv2
At upper point, N + mg = ,
0.4 ´ v 2 R
From equation (2), 8 sin 60º =
4 sin 60o mv2
N= – mg = 1400 – 700 = 700 N
R
32sin 2 60º
v2 = = 80 sin2 60º
0.4 mv2
At middle point, N = = 1400 N
Þ v = 80 sin 60º = 7.7 m/sec R
(2) (a). Let m be the mass of the ball. When the ball comes (4) (a). Given that U(r) = 10r3
down to B, its potential energy mgh which is converted So the force F acting on the particle is given by,
into kinetic energy. Let vB, be the velocity of the ball at B. ¶U ¶
F=– =– (10 r3) = –10 × 3 r2 = –30 r2
1 ¶r ¶r
Then, mgh = m vB2 For circular motion of the particle,
2
The ball now rises to a point D, where its potential energy
m v2
is mg(h – 2r). If vD be the velocity of the ball at D, then, F= = 30 r2
r
1 Substituting the given values, we have,
m g (h – 2 r) = m v 2D ......(2)
2
Now to complete the circular path, it is necessary that the 3 ´ v2
= 30 × (10)2 or v = 100 m/s
10
t.me/Ebooks_Encyclopedia27. t.me/Magazines4all

EBD_7156
22 DPP/ P 08
The total energy in circular motion (8) (b). Suppose v be the velocity of particle at the lowest
position B.
1
E = K.E. + P.E. = mv2 + U(r) According to conservation of energy
2 (K.E. + P.E.) at A = (K.E. + P.E.) at B
1
= × 3 × (100)2 + 10 + (10)3 = 2.5 × 104 joule A
2 l
O
Angular momentum
= mvr = 3 × 100 × 10 = 3000 kg–m2/sec l
2pr 2 ´ p ´ 10 p
Also time period T = = = sec
v 100 5
B
(5) (a). Let T be the tension, q the angle made by the string
with the vertical through the point of suspension.
h 1 mg
The time period t = 2p = = p/2 1
g frequency
Þ 0 + mgl = mv2 + 0 Þ v = 2gl
2
g h 1
Therefore w = =4Þ = mv2
h g 16 q (9) (a). Maximum tension T = + mg
T r
h g
cos q = = h mv2
l 16 \ = T – mg
= 0.6125 Þ q = 52º 14' r
Linear velocity · mv2
= (l sin q)w =1 × sin 52º 14' × 4 or = 163.6 – 4 × 9.8 Þ v = 6 m/s
mg r
= 3.16 m/s
(10) (c). The situation is shown in fig. Let v be the velocity of
v2 the bob at the lowest position. In this position the P.E. of
(6) (d). Centripetal acceleration, ac = = k2 rt2 bob is converted into K.E. hence -
r
\ Variable velocity v = k2r 2 t2 = k r t
The force causing the velocity to varies
dv
F= m =mkr
dt
The power delivered by the force is,
P = Fv = mkr × krt = mk2r2t
1
(7) (a). We know centripetal acceleration mgl = mv2 Þ v2 = 2gl ....(1)
2
(tangential velocity) 2 (200)2 If T be the tension in the string,
ac = = = 400 m/sec2
radius 100
mv 2
then T – mg = ....(2)
at l
From (1) & (2), T = 3 mg
r
(11) (b). The velocity of the swimmer w.r.t. water vSR = 4.0 km/
ac h in the direction perpendicular to the river. The velocity of
O r
river w.r.t. the ground is vRG = 3.0 km/h along the length
of river.
Y
Tangential acceleration
at = 100 m/sec2 (given) VSG
2 2 o VSR
\ anet = a c + a t + 2a c a t cos 90 = 2
ac + a t 2

X
= (400) 2 + (100) 2 = 100 17 m/s2
VRG
t.me/Ebooks_Encyclopedia27. t.me/Magazines4all

DPP/ P 08 23
r
The velocity of the swimmer w.r.t. the ground is vSG where
mv2
= T + mg cos a
® ® ® l
V SG = V SR + V RG
On leaving the circular path
2
VSG = VSR 2
+ VRG = 4 2 + 32 T=0

= 16 + 9 = 25 = 5 km / hr mv2
\ = mg cos a
(12) (b). The minimum speed at highest point of a vertical circle l
Þ v2 = g l cosa ....(1)
is given by vc = rg = 20 ´ 9.8 = 14 m/s According to law of conservation of energy
(13) (a). The speed at highest point must be (K.E. + P.E.) at A = (K.E. + P.E.) at B
2p 1
v> gr , v = rw = r Þ 0 + 2mgl = mv2 + mgh
T 2
2p Þ v2 = 2g (2l – h) ....(2)
\ r > rg
T 5
From (1) & (2) h = l
2pr 3
r 0.5
T< < 2p < 2p < 1.4 sec h-l
rg g 9.8 Also, cos a =
Maximum period of revolution = 1.4 sec l
(14) (a). Let the particles leaves the sphere at height h, S
(16) (d) T sin q = M w 2 R
A
q
N T sin q = Mw 2 L sin q
B From (i) and (ii) L
h q q T
mg v
T = Mw 2 L q
2
= M4 p 2 n 2 L mw R
R
2
mv2
æ 2ö
= M4p2 ç ÷ L
= mg cos q – N è pø
R
When the particle leaves the sphere i.e. N = 0 = 16 ML
2
mv 50
= mg cos q (17) (a). v = 60 km/hr = m/s
R 3
Þ v2 = gR cos q ....(1) r = 0.1 km = 100m
According to law of conservation of energy
(K.E. + P.E.) at A = (K.E. + P.E.) at B v2
\ tan q = = 0.283
1 rg
Þ 0 + mgR = mv2 + mgh \ q = tan–1 (0.283)
2
Þ v2 = 2g (R – h) ....(2)
v2
2 (18) (c). We know that tan q = ..... (1)
From (1) & (2), h = R rg
3 Let h be the relative raising of outer rail with respect to
2 inner rail. Then
Also cos q=
3 h
(15) (a). Let the body will have the circular path at height h tan q = ...... (2)
l
above the bottom of circle from figure
(l = separation between rails)
v
v2
B From (1) & (2) , h = ×l
rg

120
h
O Hence v = 48 km/hr = m/s, (r = 400 m, l = 1m),
9
mg
(120 / 9)2 ´ 1
\ h= = 0.045 m = 4.5 cm
A 400 ´ 9.8
t.me/Ebooks_Encyclopedia27. t.me/Magazines4all

EBD_7156
24 DPP/ P 08
(19) (a). The woman has two velocities simultaneously while
running on the deck, one velocity is equal to the velocity mv2B
of ship i.e. 12 m/s due east and other velocity is 5 m/s due TB L 4 v 2 + gL 4
\ = = or B =
north. TT mvT2 1 v 2T - gL 1
- mg
N L
/s
13 m or v 2B + gL = 4v 2T - 4gL but v 2B = v 2T + 4gL
5 m/s

12m/s E \ vT2 + 4gL + gL = 4v 2T - 4gL Þ 3v 2T = 9gL


The resultant velocity of woman 10
\ vT2 = 3 ´ g ´ L = 3 ´ 10 ´ or v T = 10m / sec
= (12)2 + (5)2 = 13 m/s 3
r r r
(20) (c). If we consider velocity of rain with respect to the man (22) (d). Use definition of relative velocity VPQ = VP - VQ
is V km/h. r r
VP = const. ; VQ = const.
r r r r
vrg = 4km/h Rain \ | VPQ | = | VQP | = const. ; | VP | > | VQ |
r r
\ VPQ ® + ve ; VQP = - ve i.e. towards origin.
vmg = 3km/h (23) (c). He can only reach the opposite point if he can cancel
up the velocity of river by his component of velocity.
(24) (a). v = Rw
Road v1 > v2
(25) (b), (26) (b), (27) (c).
Relative velocity of man w.r.t. ground The path of a projectile as observed by other projectile is a
® ® ® straight line.
v mg = vm - v g ........(1)
u0sinq
Velocity of rain w.r.t. ground
® ® ® q
u0cosq
v rg = vr - v g ........(2)
u0sinq

u0cosq q v ba
vmg = 3 km/hr u0cosq B
u0sinq

vrm vrm= 4 km/hr v A = u cos q iˆ + (u sin q - gt) ˆj.vAB = (2u cos q) ˆi

v B = -u cos q ˆi + (u sin q - gt) ˆj ; a BA = g - g = 0


-vmg The vertical component u0 sin q will get cancelled.
The relative velocity will only be horizontal which is equal
® ® ® to 2u0 cos q
Velocity of rain w.r.t. man v rm = vr - v m Hence B will travel horiozontally towards left w.r.t A with
On subtracting eqn. 1 from eqn. 2 constant speed 2u0 cos q and minimum distance will be h.
® ® ® Srel l
v rm = v rg - v mg =
Vrel 2u 0 cos q
(28) (a) When two bodies are moving in opposite direction,
| vrm |= vrg 2 + vmg 2 = 42 + 32 = 5 km / hr relative velocity between them is equal to sum of the
(21) (d) Since the maximum tension TB in the string moving in velocity of bodies. But if the bodies are moving in same
the vertical circle is at the bottom and minimum tension TT direction their relative velocity is equal to difference in
is at the top. velocity of the bodies.
(29) (b) Time taken is shortest when one aims perpendicular to
mv2B mv2T the flow.
\ TB = + mg and TT = - mg
L L
(30) (d) v r / m = v r2 + v m
2
t.me/Ebooks_Encyclopedia27. t.me/Magazines4all

DPP/ P 09 25

DAILY PRACTICE
PROBLEMS
PHYSICS
SOLUTIONS 09
(1) (a) Force causing the acceleration = 400 – 200 = 200N (8) (a) (a) The elevator having an initial upward speed of 8 m/
mass of the boy = 200/9.8 sec is brought to rest within a distance of 16 m
200 Hence, 0 = (8)2 + 2a (16) (Q v2 = u2 + 2as),
hence acceleration = F/m = × 9.8 = 9.8 m/s2
200 8´8
r a= - = – 2 m/sec2
F 6iˆ + 8jˆ 2 ´ 16
(2) (a) Acceleration = = in the direction of force Resultant upward force on elevator = T – mg. According to
m 10
and displacement Newton's law.
T – mg = ma
ˆ ˆ
r r 1r 1 æ 6i + 8j ö or T = mg + ma = m (g + a) = 1000 (9.8 – 2) = 7800 N
S = ut + at 2 = 0 + ç 10 ÷ 100 = 30 î + 40 ĵ (b) Let P be the upward force exerted on the man by the
2 2 è ø
elevator floor. If m' be the mass of the man, then, weight of
4 the man acting downward = m' g ,
So the displacement is 50 m along tan–1 with x-axis Upward force on the man = P – m'g
3
According to Newton's law. P–m' g = m' a or
(3) (a) From the law of conservation of momentum
P = m' (a + g) = (– 2 + 9.8) = 624 N
1000 × 5 + 0 = (1000 + 60) v
(9) (d) As P and Q move down, the length l decreases at the
1000 ´ 5 rate of U m/s
Þv= = 4.71 m/s
1060
A b b B
10
(4) (b) Weight of disc = kg , y
1000
q q
Let speed of the bullet = v
So rate of change of momentum of the bullets P Q
2 ´ 10 ´ 5
=
1000
v = applied force on the disc M
2 ´ 10 ´ 5 10 ´ g From figure, l2 = b2 + y2
Now × v=
1000 1000 Differentiating with respect to time
Þ v = 0.98 m/s2 = 98 cm/s2
dl dy
(5) (c) Total mass = 80 + 40 = 120 kg 2l = 2y (Q b is constant)
The rope cannot with stand this load so the fire man should dt dt
slide down the rope with some acceleration dy l dl 1 dl U
\ The maximum tension = 100 × 9.8 N \ = . = . =
dt y dt cos q dt cosq
m (g – a) = tension ,
120 (9.8 – a) = 100 × 9.8 Þ a = 1.63 m/s2 (10) (a) The engine, coach, coupling and resistance are, shown
(6) (b) Suppose the velocity of the body at the instant when it in figure.
reaches the pile of sand be v. Then
v2 = 0 + 2 (9.8) × (5 metre) = 98(Q v2 = u2 + 2as) COACH ENGINE
T
98 DRIVING
a=– = – 980 m/sec2 FORCE
2 ´ (0.05)
Driving force = 4500 N
Now, retarding force
F = mass × acceleration= 0.02 kg × (– 980 m/sec2) = –19.6 N (5 + 4)104
(7) (b) Impulse = F . t = Area under F-t curve from 4 ms to 16 Opposing force (Resistance) = = 900 N
100
ms = Area under BCDFB
= Area of trapizium BCEF + area of DCDE Resultant force = 4500 – 900 = 3600 N
Mass of engine and coach = 9 × 104 kg
1 1 According to Newton's law, F = ma
= (200+800) (2×10–6) + ×10 × 10–6 × 800
2 2 \ 3600 = 9 × 104 a
= 10 × 10–4 + 40 × 10–4 N–s = 50 × 10–4 or a = (3600) / (9 × 104) = 0.04 m/sec2
= 5.0 × 10–3 N-s So acceleration of the train = 0.04 m/sec2
t.me/Ebooks_Encyclopedia27. t.me/Magazines4all

EBD_7156
26 DPP/ P 09
Now considering the equilibrium of the coach only, we (17) (a) The change in momentum in the final direction is equal
have (T – R) = 4 × 104 × 0.04 (Q F = ma)
2.50 æ 250 ö
4 ´ 104
to the impulse = × 28 – çè - 1000 ´ 24÷ø = 13 Ns
1000
or T – = 4 × 104 × 0.04,
100
impulse 13
T = 4 × 104 × 0.04 + 4 × 102 = 1600 + 400 = 2000 N and force = = = 1300 N
time 1/ 100
® ®
(11) (d) Given that F1 = (8iˆ + 10j)ˆ and F = (4iˆ + 8j) ˆ in the direction of the ball.
2
r r dpr r r
Then the total force F = 12iˆ + 18jˆ (18) (b). We know F = Þ Fdt = dp
dt
r r r
r F 12iˆ + 18jˆ Þ 2 × 2 = dp Þ 4 = dp
So acceleration a = = = 2iˆ + 3jˆ m/sec2
m 6 Therefore change in momentum = 4 Ns
Net acceleration r dpr
®
(19) (a) We know = F
dt
| a | = 2 2 + 32 = 4+9 = 13 m/sec2 r r r r r r
(12) (c) From the relation Þ Fdt = dp = p2 - p1 = mv 2 - mv1
F 1000 r
F = ma Þ a = = = 1 m/s2 Þ 4 ĵ . 1 = 2 . v2 – 2(2 î )
m 1000
r ˆ = 2.vr - 2 (2i)ˆ = 4jˆ + 4iˆ
As the force is brake force, acceleration is – 1 m/s2 using Þ 2v2 4j.1 2
relation v2 = u2 + 2as, we obtain r
Þ v2 = 2iˆ + 2ˆj
2
æ 5ö r
2 çè18 ´ ÷ø Þ | v2 | = 2 2 m/s
u 18
2as = u2 Þ s = = = 12.5 m (20) (c) Initial momentum of the ball
2a 2
150
(13) (a) The water jet striking the block at the rate of 1 kg/s at a = × 12 = 1.8 kg.m/sec
speed of 5 m/s will exert a force on the block 1000

dm 150
F=v =5×1=5N Final momentum of the ball =– × 20= – 3.0 kg m/sec
dt 1000
Change in momentum = 4.8 kg m/sec
4.8
2 kg Average force exerted = Impulse/ time = = 480 N
a .01
(21) (b) Initial momentum of the body = mu = 20 × 3 = 60
And under the action of this force of 5 N, the block of mass and final momentum of the body = – mu = –20 × 3 = – 60
2 kg will move with an acceleration given by, The change in momentum of body in initial direction
F = ma Þ a = F/m = 5/2 = 2.5 m/s2 = – 60 – 60 = –120
(14) (a) Relative speed of the ball = (v + u) The change in momemtum imparted to the body in opposite
Speed after rebouncing = – (v + u) direction = 120
Dv m[(v + u) - {-(v + u)}] \ The impulse imparted to the body = 120 Ns
So, F = m = (22) (a) (1) Since the lift is moving down with an acceleration of
Dt t
3 m/sec2, then the inertial force F = ma, acts upwards on
2m(v + u) the body
=
t
2
R a=3 m/s
dp F=ma
(15) (b) F = Þ F dt = dp = p2 – p1
dt
Þ F × 1 = mnv – 0
Þ F = mnv
(Total mass of the bullets fired in 1 sec = mn) mg
(16) (a) The initial momentum = 15 × 10 = 150 kgm/s and Now, R + F= mg
change in momentum 0 - 150 or R = mg – F = mg – ma = m (g – a) = 60 (9.8 – 3) = 408 N
Force = = = –10 N (2) When the lift is moving down with constant velocity
time 15
A constant force of 10 N must be acting in opposite a = 0 and hence, R = mg = 60 × 9.8 = 588 N
direction to the motion of body. (3) The lift is now moving down with a retardation of
t.me/Ebooks_Encyclopedia27. t.me/Magazines4all

DPP/ P 09 27
3 m/sec2. i.e. W' = W + R = m (g + a)
The retardation is 3 m/sec2 in the downward direction is Hence, scale show a reading of
equivalent to an acceleration of 3 m/sec2 upwards.
Hence the direction of fictitious force is downwards. æ 50g ö
m (g + a) Newton = çè 50 +
a ÷ø
kg wt
Now, R = mg + ma = m (g + a) = 60 (12.8) = 768 N
(23) (b) When the lift is moving up m (g + a) = force (26) (a) Tension = m (g + a), when lift moving up, putting the
values, we get
æ gö 175 = 25 (9.8 + a) Þ a = 2.8 m/s2
10 ç g + ÷
m(g + a) è 3ø [negative sign shows that lift is moving downward]
The scale reading = = = 13.3 kg
g g (27) (b) Apparent tension, T = 2T0
When lift is moving down the scale reading æ a0 ö
So, T = 2T0 = T0 ç 1 + ÷
æ gö è gø
10 ç g - ÷
m(g - a) è 3ø
= = = 6.67 kg a
g g or 2 = 1 + 0 Þ a0 = g = 9.8 m/s2
g
(24) (a)
(1) A reference frame in which Newton’s first law is valid is (28) (b) Cloth can be pulled out without dislodging the dishes
called an inertial reference frame. from the table because of inertia. Therefore,
(2) Frame moving at constant velocity relative to a known statement- 1 is true.
inertial frame is also an inertial frame. This is Newton's third law and hence true. But
(3) Idealy, no inertial frame exists in the universe for statement 2 is not a correct explanation of statement 1.
practical purpose, a frame of reference may be considered (29) (d) According to Newton’s second law
as Inertial if its acceleration is negligible with respect to
the acceleration of the object to be observed. Force
Acceleration = i.e. if net external force on the
(4) To measure the acceleration of a falling apple, earth can Mass
be considered as an inertial frame. body is zero then acceleration will be zero.
(25) (a)
(i) In the case of constant velocity of lift, there is no reaction, dp
(30) (c) F= = Slope of momentum -time graph
therefore the apparent weight = actual weight. Hence the dt
reading of machine is 50 kg wt. i.e. Rate of change of meomentum = Slope of
(ii) In this case the acceleration is upward the reaction R = momentum-time graph = force.
ma acts downward, therefore apparent weight is more than
actual weight .
t.me/Ebooks_Encyclopedia27. t.me/Magazines4all

EBD_7156
28 DPP/ P 10
DAILY PRACTICE
PROBLEMS
PHYSICS
SOLUTIONS 10
(1) (c) Force on the block (5) (a) We have acceleration
= Mass of the block × acceleration of the system
Fcos q 50 3
a= = = 5 3 m /sec2
P m 10
=M×
M+m The velocity after 2 sec, v = u + at

1 Þ v = 0 + 5 3 × 2 = 10 3 m/sec
(2) (b) Mass of the rope = 8 × = 4 kg (6) (a) All the forces acting on the two blocks are shown in
2
fig. As the blocks are rigid under the action of a force F,
Total mass = 50 + 4 = 54 kg both will move together with same acceleration.
F 108 R1
\ a= = = 2 m/s2
m 54
Force utilised in pulling the rope =4×2 = 8 N R1 M
Force applied on mass = 108 – 8 = 100 N
(3) (b) Mass of the rope = 15 × 2 = 30 kg F R
A
m
F 25 5
acceleration = = = m/s2
m 30 6 mg Mg
At the point 7 m away from point of application the a = F/(m+M) = 3/(1+2) = 1 m/s2
mass of first part of rope = 14 kg Now as the mass of larger block is m and its acceleration
5 a so force of contact i.e. action on it.
\ Force used in pulling 14 kg = 14 × = 11.67 N MF 2´3
6
f = Ma = = = 2N
The remaining force = (25 –11.67)N = 13.33 N M + m 2 +1
(4) (b) The various forces acting are shown in fig. (7) (a) As the same force is applied to the combined
The force of 100N has mass, we have

(i) horizontal component of 100 cos 30º = 50 3 N 1 1 1 a1a 2 5 ´ 15


= + or a= = = 3.75 m/s2
and (ii) A vertical component = 100 sin 30º = 50N a a1 a 2 a1 + a 2 5 + 15
(8) (a) As net force on the rod = F1 – F2 and its mass is M so
R acceleration of the rod will be
a = (F1 – F2)/M ...(i)
100 Now considering the motion of part AB of the rod,
N 50 3N which has mass (M/L)y,
30° Acceleration a given by
(i) Assuming that tension at B is T
M
W=10×10N F1 – T = y×a (from F = ma)
L
50N
M F1 - F2
Since the block is always in contact with the table, the Þ F1 – T = y (using eq. (1))
L M
net vertical force
R = mg + F sin q = (10 × 10 + 50) N = 150N æ yö æ yö
Þ T = F1 ç 1 - ÷ + F2 ç ÷
When the block moves along the table, work is done è Lø è Lø
by the horizontal component of the force. Since the (9) (b) The net acceleration of the system is given by
distance moves is 10 m, the work done is 1 1 1
= +
50 3 × 10 = 500 3 Joule. a a1 a 2
If v is the speed acquired by the block, the work done 1 1 1 1
must be equal to the kinetic energy of the block. = + + ...... = 1 + 2 + 3 + ......... + n
a a1 a 2 an
Therefore, we have
1 n n 2
500 3 = × 10 × v2 Þ v2 = 100 3 Þ v = 13.17 m/sec = [2 + (n – 1) 1] = [n + 1] =
2 2 2 n(n + 1)
t.me/Ebooks_Encyclopedia27. t.me/Magazines4all

DPP/ P 10 29
(10) (a) As the mass of the system is 6 + 4 + 2 = 12 kg and (15) (b) The string is massless and inextensible the tension T is
applied force is 60 N, the acceleration of the system same. Let mass B move down the inclined plane.
F 60 For B the equation of motion m1g sin q – T = m1a
a= = = 5 m/s2 30 × 9.8 × sin 53º – T = 30a
m 12
Þ 235.2 – T = 30 a ...(1)
Now at point A as tension in pulling the rope of mass
and for A the equation of motion
2kg and block Q of mass 4kg.
TA = (2 + 4) × 5 = 30N T – 20 × 9.8 × sin 37º = 20a
Similarly for B and C, T – 117.6 = 20a ...(2)
TB= (1 + 4) × 5 = 25N From (1) & (2) T = 164.64 N
and TC = (0 + 4) × 5 = 20N (16) (c)
(11) (a) In case (a), the pulling force = 2mg – mg = mg
and the mass is 2m + m = 3m
so acceleration a = mg/3m = g/3
While in case (b), the pulling force = 2mg – mg = mg
T
but, the mass in motion = m + 0 = m
Acceleration, a = mg/m = g ma m a
(12) (c) It this problem as the pulling force is 2mg while
a
opposing force is mg, so net force
F= 2mg – mg = mg,
mg
and as the mass in motion = m + m + m = 3m (Force diagram in the frame of the car)
force mg g Applying Newton’s law perpendicular to string
So the acceleration = = =
mass 3m 3 mg sin q = ma cos q
Now as A is accelerated up while B and C down. so
tension T1, is such that mg < T1 < 2mg a
tan q =
Actually for the motion of A, g
4 Applying Newton’s law along string
T1 = m (g + a) = m(g + g/3) = mg
3 Þ T - m g 2 + a 2 = ma
Now to calculate tension in the string BC we consider
the downward motion of C,
i.e. T2 = m (g – a) = m (g – g/3) = (2/3) mg or T = m g 2 + a 2 + ma
(13) (a) As pulley Q is not fixed so if it moves a distance d the (17) (a) As A moves up and B moves down with acceleration a
length of string between P and Q will changes by for the motion of A ,
2d (d from above and d from below) i.e. M will move 2d. T – 11 g = 11 a ... (i)
This in turn implies that if a (®2d) is the acceleration of for the motion of B,
M, the acceleration of Q and so 2M will be of (a/2) 11.5 g – T = 11.5 a ...(ii)
Now if we consider the motion of mass M, it is From (i) & (ii) ,
accelerated down so T = M(g – a) ...(1)
And for the motion of Q, m1 - m 2
2T – T' = 0 × (a/2) = 0 Þ T' = 2T ...(2) a= m +m
1 2
And for the motion of mass 2M,
T' = 2M (a/2) Þ T' = Ma ...(3) (11.5 - 11)9.8
g= = 0.218 m/sec2
1 11.5 + 11
From equation (2) and (3) T = Ma, so eq. (1) reduces Assuming that the particles are initially at rest, their
2
velocity at the end of 4 sec will be
æ 1ö 2 v = u + at = 0 + 0.218 × 4 = 0.872 m/s
çè 2 ÷ø Ma = M (g – a) Þ a = g (18) (a) The height ascended by A in 4 sec
3
(14) (a) The tension is same in two segments 1 2 1
For B the equation is h = ut + at = 0 + (0.218) 42 = 1.744 m
2 2
(40 × 9.8 – T) = 40a ...(1)
This is also the height descended by B in that time.
For C the equation is
(19) (c) At the end of 4 sec the string is cut. Now A and B are no
1 longer connected bodies but become free ones, falling
(T – 50 × 9.8 × ) = 50a ...(2)
2 under gravity.
From equation (1) and (2) a = 1.63 m/s2 Velocity of A, when the string was cut
distance of fall = 0.872 m/s upwards.
1 2 1 Acceleration a = – g (acting downwards),
S= at = × 1.63 × 42 = 13.04 m displacement from this position in the subsequent 2 sec
2 2
t.me/Ebooks_Encyclopedia27. t.me/Magazines4all

EBD_7156
30 DPP/ P 10
(2) If the downward acceleration of the lift is a = g, then the
1 2 1
h = ut + at = (0.872) × 2 + (–9.8) 22 body will enjoy weightlessness.
2 2 (3) If the downward acceleration of the body is a > g, then
= 1.744 – 4.9 × 4 = –17.856m the body will rise up to the ceiling of lift
A descends down by a distance of 17.856 m from the (25) (d), (26) (d), (27) (a).
position it occupied at the end of 4 sec from its start. B
has a free fall. Its position is given by Initial elongation = 2R cos 30° = 3R
So the acceleration of mass M is (2/3)g
while tension in the string PQ will be N
T = M(g – (2/3)g) = (1/3)Mg F
The force exerted by clamp on the pulley
30° 60° mg
2 2 2
= T +T = Mg
3
(20) (a) Here the system behaves as a rigid system, therefore
every part of the system will move with same accelera-
tion. Thus applying newton’s law
mg – T = ma ......... (i) Extension in the spring is
2T – mg = ma ......... (ii) x = AB – R = 2R cos 30° – R = ( 3 - 1) R
Doubling the first equation and adding
\ Spring force
1
mg = 3ma or acceleration a = g ( 3 - 1) mg
3 F = kx = ( 3 - 1) R = 2mg
(21) (c) Tension in the string R
FBD of bead is
æ gö
T = m (g – a) = m çè g - ÷ø 3 3 3
3 N = F (mg cos 30°) = (2mg + mg) = mg
2 2
2 Tangential force F1 = F sin 30° – mg sin 30°
T= mg
3
mg
(22) (a) = (2mg – mg) sin 30° =
(1) Inertia µ mass 2
(2) 1 Newton = 105 dyne \ tangential acceleration = g/2
28. (d) Here the acceleration of both will be same, but their
r DM r r masses are different. Hence, the net force acting on
(3) Thrust on rocket F = v - Mg
Dt each of them will not be same.
(4) Apparent weight of a body in the lift accelerated up is 29. (c) The FBD of block A in Figure is
W = m (g + a).
(23) (b) N
(1) If a1, a2, ... an be the accelerations produced in n
different bodies on applying the same force, the mg
acceleration produced in their combination due to the
The force exerted by B on A is N (normal reaction). The
1 1 1 1
same force will be = + + ...... force acting on A are N (horizontal) and mg (weight
a a1 a 2 an downwards). Hence statement I is false.
(2) Newton's Ist and IIIrd law can be derived from second 30. (d) T – m1g = m1a – .... (1)
law therefore IInd law is the most fundamental law out m2g – T = m2a – .... (2)
of the three law.
(24) (a) æ 2m1m2 ö æ m 2 - m1 ö
Solving (1) and (2), T1 = çè m + m ÷ø g a = ç m + m ÷ g
(1) For equilibrium of a body under the action of concurrent 1 2 è 1 2ø
® ® ® ®
forces F1 + F2 + F3 + .....Fn = 0
t.me/Ebooks_Encyclopedia27. t.me/Magazines4all

DPP/ P 11 31

DAILY PRACTICE
PROBLEMS
PHYSICS
SOLUTIONS 11
(1) (a) Let the contact force on the block by the surface be F
minimum possible force F that can be applied, the friction
which makes an angle q with the vertical. The component
is limiting and hence
of F perpendicular to the contact surface is the normal f = mN, where N is normal force.
force N and the component F parallel to the surface is the in the vertical direction, there is no acceleration
friction f. As the surface is horizontal, N is vertically upward. \ N = mg
For vertical equilibrium in the horizontal direction,

F N let the acceleration be a, then

q mN = ma
mmg = ma

a = mg
f f = mN
Next consider the motion of M
The equation of motion is
N = Mg = (0.400) (10) = 4.0 N
F = mN = Ma
The frictional force is f = 3.0 N
F – mmg = Mmg
f 3 F = mg (M + m)
tan q = = Þ q = tan–1 (3/4) = 37º
N 4
N
(2) (c) The magnitude of the contact force is
F= 2 2
N 2 + f 2 = (4) + (3) = 5.0 N
(3) (c) The forces on the block are f = mN F

N f
Mg
(5) (b) When A moves with B the force opposing the motion is
mg the only force of friction between B and S the horizontal
q and velocity of the system is constant

(i) the weight mg downward by the earth R2


(ii) the normal contact force N by the incline, and
(iii) the friction f parallel to the incline up the plane, by the A
incline.
R1
As the block is at rest, these forces should add up to zero. F
Also since q is the maximum angle to prevent slipping, this B
is a case of limiting equilibrium and so f1
f = mSN
Taking component perpendicular to the lncline, F = f1 = mR1 = 0.25 (4 + 8) = 3N
N – mg cos q = 0 Þ N = mg cos q ....... (1) (6) (d) When A is held stationary the friction opposing the
Taking component parallel to the incline motion is between A and B and B and S. So
f – mg sin q = 0 Þ f = mg sin q ........ (2)
\ mSN = mg sin q R2 A
Dividing (2) by (1) ms = tanq
q = tan–1mS = tan–1 (0.3) f2
(4) (a) When the maximum force F is applied, both the blocks R1
F B
move together towards right. The only horizontal force on
the upper block of mass m is that due to the friction by the
S f1
lower block of mass M. Hence this force on m should be
towards right. The force of friction on M by m should be F = mR1 + mR2 = 3 + 0.25 (4)
towards left by Newton's third law. As we are talking of the F= 3+1= 4N
t.me/Ebooks_Encyclopedia27. t.me/Magazines4all

EBD_7156
32 DPP/ P 11
(7) (d) In this situation for dynamic equilibrium of B (10) (c) If y is the maximum length of chain which can be hang
out side the table without sliding, then for equilibrium of
R2 A the chain, the weight of hanging part must be balanced by
force of friction from the portion on the table
f2
f2 L–y
F R1 R

f1 f1

F = mR1 + mR2 + T ..... (1) W’


While for the uniform motion of A y
T = mR2 ....... (2)
Substituting T from Equation (2) in (1) we get
F = mR1 + 2mR2 = 3 + 2 x 1 = 5N
(8) (a) Figure shows the forces acting on the two blocks. As
we are looking for the maximum value of M/m, the W
equilibrium is limiting. Hence the frictional forces are equal
to m times corresponding normal force. W = fL ....... (1)
Equilibrium of the block m gives
M
N1 But from figure W = y g and
L

T M
m R = W' = (L – y)g
mN1 N2 L
mM
mg mN1 M So that fL = mR = (L – y) g
L
mg Substituting these values of W and fL in equation (1) we
q
get
T = mN1 and N1 = mg Þ T = mmg .... (1) M M
Next consider the equilibrium of the block M. Taking yg = m (L – y) g
L L
components parallel to the incline
T + mN2 = Mg sin q (11) (d) The insect will crawl up the bowl till the component of
Taking components normal to the Incline its weight along the bowl is balanced by limiting friction
N2 = Mg cos q so, resolving weight perpendicular to the bowl and along
These give T = Mg (sin q – m cos q) ...... (2) the bowl we get
From (1) and (2) mmg = Mg (sin q – m cos q)
M m R
=
m sin q - m cos q
(9) (a) The situation is shown in figure in the limiting
y
equilibriums the frictional force f will be equal to mN.
fL q R = mg sin
N
F q mg sinq mg cos
q

F q
fS = m N
fL = mg cos q
Mg y 1
R R 1 =
For horizontal equilibrium tanq = = 1 = ; 2 2 m
f L mR1 m R -y
F sin q = mN
For vertical equilibrium R1
F cos q + mg = N m2y2 = R2 – y2 ; y=
Eliminating N from these equations m2 + 1
F sin q = mF cos q + mmg
R é ù
m 1
F= So, h = R – y = R – = R ê1 - ú
(sin q - m cos q) m2 + 1 ê (m 2
+ 1) ú
ë û
t.me/Ebooks_Encyclopedia27. t.me/Magazines4all

DPP/ P 11 33

(12) (a) (18) (a) (i) In the force applied v/s friction graph : The graph is
a straight line of slope 45º for small F and a straight line
parallel to the F-axis for large F.
m (ii) There is small kink on the graph
(19) (a) (i) force of friction between two bodies may be equal to
L h zero
(ii) bodies may be rough
(20) (b) It is easier to pull a body than to push, because the
friction force is more in pushing than that in pulling
Loss in P.E. in reaching the bottom = mgh and gain in K.E. (21) (a)ma = µmg
a = µg
1 (22) (a)
reaching the bottom = mv2
2 (1) Kinetic friction is lesser than limiting friction.
where v is velocity gained by the body in reaching the (2) In rolling the surfaces at contact do not rub each other.
bottom (3) If a body is at rest and no pulling force is acting on it,
force of friction on it is zero.
1
\ Net loss in energy = mgh – mv2 (23) (a)
2 (1) Force of friction is partically independent of microscopic
work done against friction = FL area of surface in contact and relative velocity between
them. (if it is not high)
1 2
\ mgh – mv2 = FL ; v = (mgh - FL) (2) Normally with increase in smoothness friction
2 m decreases. But if the surface area are made too smooth by
(13) (a) Let R be the normal reaction on the block exerted by the polishing and cleaning the bonding force of adhesion will
floor. The limiting (maximum) force of static friction is increase and so the friction will increase resulting in 'Cold
fs = msR = msmg welding'
(3) Friction is a non conservative force, i.e. work done
= 0.4 × 2kg × 9.8 ms–2 = 7.84 N
against friction is path dependent.
The applied force F is 2.5 N, that is less than the limiting
(24) (c)
frictional force. Hence under the force F, the block does
(2) Friction may opposes the motion
not move. So long the block does not move, the (adjustable)
(4) If the applied force is increased the force of static friction
frictional force is always equal to the applied force. Thus
also increases upto limiting friction.
the frictional force is 2.5 N.
(25) (a), (26) (a), (27) (a).
(14) (b) When the block does not slip on the table surface, it
Fmax = kx + µ mg
performs simple harmonic motion along with the table.
Fmin = kx – µ mg
x = a sin wt
\ Fmax + Fmin = 2µ mg
The instantaneous acceleration of the block is
or 2 = 2 µ 10
d2 x \ µ = 0.1
= –w2a sin wt Fmax + Fmin = 2kx ......... (1)
dt 2
From graph, Fmax + Fmin = 5 and x = 0.1
Putting in eq. (1)
d2 x
The maximum acceleration is = w2a t = 2k (0.1) ; k = 25 N/m
dt 2 max
When x = 0.03
The maximum force on the block is fmax = mw2a kx = 25 × 0.03 = 0.75 N, which is less than µ mg
where m is its mass. The frictional force on the block is = 0.1 × 10 = 1N
mmg. since the block is at rest with respect to the table, we \ The block will be at rest, without applying force F.
have mw2a = mmg (28) (b) It is easier to pull a heavy object than to push it on a
(2pf)2a = mg level ground. Statement-1 is true. This is because the
mg 0.72 ´ 10 normal reaction in the case of pulling is less as compared
Þa= = = 0.02 m by pushing. (f = m N). Therefore the functional force is
4p 2f 2 4 ´ (3.14) 2 ´ 32
small in case of pulling.
(15) (c) Stopping distance is independent on mass. Statement-2 is true but is not the correct explanation of
(16) (a) (i) coefficient of static friction is always greater than the statement-1.
coefficient of kinetic friction
(29) (c) W = (force) × (displacement of point of application)
(ii) limiting friction is always greater than the kinetic friction
(30) (d) Statement – 2 is false because friction force may be
(iii) limiting friction is never less than static friction
more than applied force when body is retarding and
(17) (d) The system can not remain in equilibrium
external force is acting on body.
t.me/Ebooks_Encyclopedia27. t.me/Magazines4all

EBD_7156
34 DPP/ P 12
DAILY PRACTICE
PROBLEMS
PHYSICS
SOLUTIONS 12
r
(1) (a) Here F = -ˆi + 2jˆ + 3kˆ &
r v2 mu 2
Now from (1), m = kt +
d = (0 – 0) î + (0 – 0) ĵ + (4 – 0) k̂ = 4 k̂ 2 2
rr
\ W (Work done) = F.d = (-ˆi + 2jˆ + 3k) ˆ . 4 k̂ = 12 J 1
Þ m (v2 – u2) = kt .....(2)
(2) (a) The minimum force with a body is to be pulled up along 2
the inclined plane is mg (sin q + m cos q) mdv
rr Again v= k
Work done, W = F.d dt
= Fd cos q = mg (sin q + m cos q) × d dv
= 5 × 9.8 (sin 60º + 0.2 cos 60º) × 2 = 98.08 J Þ m.v v = k Þ mv2 dv = kdx
dx
5 5 Intergrating,
(d) W = ò F dx = (7 - 2x + 3x 2 ) dx
(3)
ò 1
0 0 m (v3 – u3) = kx .....(3)
3
5 5 From (2) and (3),
é 2x 2 ù é 3x3 ù
= [7 x ] – ê 2 ú + ê 3 ú = 135 Joule
5

3 æ v -u ö
2 2
0 êë úû 0 êë úû 0
t= ç ÷ (x)
(4) (d) Given that, power = Fv = P = constant 2 è v3 - u 3 ø
dv mdv (7) (b) Mass of the chain hanging = 4 × 3 = 12 kg
or m v=P [as F = ma = ] Shift in center of gravity = 4/2 = 2m
dt dt
Work done, W = mgh = 12 × 9.8 × 2 = 235 .2 J
P v2 P (8) (b) Mass of 2 litre, water = 2 kg
or ò v dv = ò m dt Þ 2
=
m
t + C1 Total mass to be lifted = 2 + 0.5 = 2.5 kg
Work done , W = mgh = 2.5 × 9.8 × 6 = 147 J
Now as initially, the body is at rest
(9) (b) The following two forces are acting on the body
i.e v = 0 at t = 0 so, C1 = 0
(i) Weight mg is acting vertically downward
2Pt (ii) The push of the air is acting upward.
\ v= As the body is accelerating downward, the resultant force
m
is (mg – F)
1/2 Workdone by the resultant force to fall through a vertical
ds ds æ 2Pt ö
(5) (b) By definition v = or =ç distance of 20 m = (mg – F) × 20 joule
dt dt è m ÷ø
1
1/2 1/2 Gain in the kinetic energy = mv2
æ 2Pt ö æ 2P ö 2 3/2 2
Þ ò ds = ò ç dt Þ s = ç ÷ t + C2
è m ÷ø è mø 3 Now the workdone by the resultant force is equal to the
change in kinetic energy i.e.
Now as t = 0, s = 0, so C2 = 0
1
1/2 (mg – F) 20 = mv2 (From work-energy theorem)
æ 8P ö 2
s= ç t3/2
è 9m ÷ø
F
mdv
(6) (c) The force acting on the particle =
dt

æ mdv ö
Power of the force = ç
è dt ÷ø
v = k (constant)
a

v2
Þm = kt + c .....(1)
2

mu 2
At t = 0, v = u \ c=
2 mg
t.me/Ebooks_Encyclopedia27. t.me/Magazines4all

DPP/ P 12 35

mg l mgl
U= × =
5 10 10
\ Work done = U = mg l/50
(12) (a) At maximum speed all the power is used to overcome
the resistance to motion. Hence if the maximum speed is v,
1 then 50000 = 1000 × v or v = 50 m/s
or (50 – F) 20 = × 5 × (10)2
2 At 25 m/s, let the pull of the engine be P, then the power
or 50 – F = 12.5 or F = 50 – 12.5
50, 000
\ F = 37.5 N or P = = 2000 N
Work done by the force = – 37.5 × 20 = – 750 joule 25
(The negative sign is used because the push of the air is Now resultant force = 2000 – 1000 = 1000 N
upwards while the displacement is downwards.) Applying Newton's law ; F = ma, we have
(10) (a) 1000 = 1000 a or a = 1.0 m/s2
(13) (a) 1 mole i.e.235 gm of uranium contains 6 × 1023 atoms, so
R 2 kg i.e. 2 × 103 gm of uranium will contain
F sin 45°
2 ´ 103 ´ 6 ´ 1023
= atoms = 5.106 × 1024 atoms
235
F cos 45° Now as in each fission only one uranium atom is con-
sumed i.e. Energy yield per uranium atom
= 185 MeV = 185 × 1.6 × 10 –13 J = 2.96 × 10–11 J
So Energy produced by 2 kg uranium
mR = (No. of atoms ) × (energy /atom)
mg = 5.106 × 1024 × 2.96 × 10–11 = 1.514 × 10–14 J
As 2 kg uranium is consumed in 30 days i.e. 1.51 × 10–14 J
The different forces acting on the block are shown in fig.
of energy is produced in the reactor in 30 days i.e.
Now we have
2.592 × 106 sec
R + F sin 45° = m g ............(1)
So, power output of reactor
F cos 45° = m R ............(2)
From equation (1) and (2) E 1.514 ´ 1014 J
= = = 58.4 MW
m mg t 2.592 ´ 106 S
\ F=
cos 45º +m sin 45º (14) (c) When the vehicle of mass m is moving with velocity v,
Substituting the given values, we have 1
the kinetic energy of the where K = mv2 and if S is the
0.20 ´ (5 ´ 9.78) 2
F= = 11.55 N
(0.707) + (0.20 ´ 0.707) stopping distance, work done by the friction
The block is pulled through a horizontal distance W = FS cos q = m MgS cos 180º = – m MgS
r = 20 metre So by Work-Energy theorem,
Hence, the work done W = D K = Kf – ki
W = F cos 45° × r = (11. 55 × 0.707) × 20 = 163. 32 Joule
(11) (c) 1 2
v2
Þ – m MgS = 0 – Mv Þ S =
2 2mg
(15) (a) As T = (2p/w),
so w = 2p/(3.15 × 107) = 1.99 × 10–7 rad/s
Now v = rw = 1.5 × 1011 × 1.99 × 10–7 » 3 × 104 m/s
Now by work - energy theorem ,
l/5
1
W = Kf – Ki = 0 – mv2
2
1
=– × 6 × 1024 (3 × 104)2 = – 2.7 × 1033 J
Mass of the hanging part of the chain = (m/5) The weight 2
mg/5 acts at the centre of gravity of the hanging chain, i.e., Negative sign means force is opposite to the motion.
at a distance = l/10 below the surface of a table. (16) (b) As the particle is moving in a circle, so
The gain in potential energy in pulling the hanging part on
the table. mv2 k 1 k
= 2 Now K.E = mv2 =
r r 2 2r
t.me/Ebooks_Encyclopedia27. t.me/Magazines4all

EBD_7156
36 DPP/ P 12
dU
(22) (a) W = 0
Now as F=– (23) (b)
dr
(1) There will be an increase in potential energy of the
r r
æ kö k system if work is done upon the system by a conservative
Þ P.E, U = – ò Fdr = ò + çè r 2 ÷ø dr = – r
force.
¥ ¥
(2) The work done by the external forces on a system equals
k k k the change in total energy
So total energy = U + K.E = – + =-
r 2r 2r (24) (a)
Negative energy means that particle is in bound state . (1) The work done by all forces equal to change in kinetic
(17) (c) Let the mass of the person is m energy
Work done, W = P.E at height h above the earth surface
(2) The work done by conservative forces equal to change
= (M + m) gh in potential energy
or 4900 = (M + 10) 9.8 × 10 or M = 40 kg
(18) (b) As the rod is kept in vertical position the shift in the (3) The work done by external and nonconservative forces
centre of gravity is equal to the half the length = l/2 equal to change in total energy
(25) (b), (26) (b), 27. (c)
l 4
Work done W = mgh = mg = 20 × 9.8 × = 392 J For vertical block
2 2
(19) (a) We know that the increase in the potential energy mg = kx + 2T ....... (1)
For horizontal block
é1 1 ù
DU = GmM ê - ú T = k (2x) ....... (2)
ë R R 'û
From eq. (1) and eq. (2)
According to question R' = R + R = 2R
mg
é1 1 ù GMm x= = 0.2m
DU = GMm ê - ú = 5k
ë R 2R û 2R
\ Extension of vertical spring = 0.2m
1 Extension of horizontal spring = 2x = 0.4m
(20) (c) In first case, W1 = m(v1)2 + mgh
2
From conservation of energy
1
m(12)2 + m × 10 × 12
= 1 2 1 1 1
2 mgx = kx + k(2x)2 + mv2 + m(2v)2
2 2 2 2
= 72 m + 120 m = 192 m
and in second case,W2 = mgh = 120 m 3 2 3
The percentage of energy saved mgx = kx + mv 2
2 2
192m - 120m
= × 100 = 38% 7 3
192m mgx = mv 2
10 2
a b
(21) (c) Given that, U (x) = -
12
x x6 v=
7
gx
15
du
We know F = – = (–12) a x–13 – (– 6b) x–7 = 0 Required speed = 2v = 1.9 m/s
dx
(28) (d) Statement – 1 is true but statement – 2 is false.
6b 12a
or
7
= (29) (a) Work done by action reaction force may be zero only if
x x13 displacement of both bodies are same.
1/6 (30) (b) Both statements are true and independent.
æ 2a ö
or x6 = 12a/6b = 2a/b or x = ç ÷
è bø
t.me/Ebooks_Encyclopedia27. t.me/Magazines4all

DPP/ P 13 37

DAILY PRACTICE
PROBLEMS
PHYSICS
SOLUTIONS 13
(1) (a) Let m1 and m2 be the masses of bullet and the rifleman
m1. Applying conservation of linear momentum for the
and v1 and v2 their respective velocities after the first shot.
collision of bullet with plate m2.
Initially the rifleman and bullet are at rest, therefore initial
i.e. mv1 = (m2 + m) v
momentum of system = 0.
0.02 v1 = (2.98 + 0.02) v
As external force is zero, momentum of system is constant
i.e. initial momentum = final momentum 3
i.e. v1 = v = 150 v .........(2)
= m1v1 + m2v2 .02
(10 ´ 10-3 kg)(800m / s) Required percentage loss in initial velocity of bullet
m1 v1
or v2 = =– = – 0.08 m/s u - v1 200v - 150v
m2 100 kg
× 100% = × 100 = 25%
Velocity acquired after 10 shots u 200 v
= 10 v2 = 10 × (–0.08) = – 0.8 m/s (4) (a) Part (I) - The horizontal component of the momentum of
i.e, the velocity of rifle man is 0.8 m/s in a direction opposite the bullet is equal to the momentum of the block with the
to that of bullet. bullet
(2) (c) Let the mass of block and bullet be M and m respectively mu cos a = (M + m) V ..........(1)
If v is the velocity of bullet and V is the velocity of block Where V is the velocity of the block plus bullet embedded
with bullet embedded in it, in it.
Now according to conservation of momentum , Part (II) - As the block can move as a pendulum, the block
mv = (M + m) V rises till its kinetic energy is converted into potential energy.
(10×10 –3)(300) = (290×10–3 + 10 ×10–3) V or V = 10 m/s So, if the block rises upto a height h,
1 1
The kinetic energy just after impact is(M + m) V2, which (M + m) V2 = (M + m) gh ..........(2)
2 2
is lost due to work done on it by the force of friction F. From (1) & (2)
Since force of friction F = m (M+m)g and the work done is 2
æ m ö u 2 cos 2 a
given by Fd, we have h= ç ÷ .
èM+mø 2g
1
(M + m) V2 = m (M + m) gd
2 æ 20 ´ 10-3 ö
2
cos2 30o
= çç ÷ .(200)2 .
÷ = 0.15 m
1 V
2
1 10 2 1 2 (2) (10)
è ø
or m= = × =
2 gd 2 (10)(15) 3 (5) (c) Initial velocity of bullet, u1 = 500 m/s
(3) (a) Let the in itial velocity of the bullet of mass Let v1 and v2 be the speeds of bullet and block after collision
m = 20 g = 0.020 kg be u and v the velocity with which each
plate moves.
The initial momentum of system (bullet + plate) = mu

v1
m u1
0.1m m1 m v2
2
m4 m2
Final momentum of system = m1v + (m2 + m) v 1
respectively then, m v 22 = mgh
(Since bullet remains in 2nd plate) 2
\ According to principle of conservation of momentum
Þ v2 = 2gh = 2 ´ 9.8 ´ 0.1 = 1.4 m/s
i.e. mu = m1v + (m2 + m) v,
i.e. 0.02u = 4v According to principle of conservation of linear momentum,
We have
4
or u= = 200 v ..........(1) m1u1 + 0 = m1v1 + m2 v2
.02 or 0.01 × 500 = 0.01 v1 + 2 × 1.4 Þ v1 = 220 m/s
Let v1 be the velocity of the bullet as it comes out of plate
t.me/Ebooks_Encyclopedia27. t.me/Magazines4all

EBD_7156
38 DPP/ P 13
(6) (a) The rate of change of momentum is equal to force \ (v1 – v2) = 0 ......(4)
Solving equations (3) and (4), we have
dp dm
F= =v (Here v is constant) v1 = 3 3 m/s and v2 = 3 3 m/s
dt dt
dm
Here v = 4 × 103 m/s & = 50 × 10–3 kg/s
dt
\ F = 4 × 103 × 50 × 10 –3 = 200 N
v1
(7) (a) Given that, Initial velocity = u
30°
u 30°
Final velocity =
4
v2
So by conservation of momentum, we have
u 3u
1×u+0=1× + m × v2 Þ mv2 = ....... (1) According to law of conservation of energy
4 4
and by conservation of energy, we have Energy before collision = Energy after collision
1 1 1 1
1 1 æ uö 1
2 m u12 + m u 22 = m v12 + m v 22
× 1 × u2 + 0 = × 1 ç ÷ + m v 22 2 2 2 2
2 2 è 4ø 2
1 1 1
m (9)2 + 0 = m (3 3 )2 + m (3 3 )2
15 2 2 2 2
or v 22
= u ....... (2)
16
81 m 54 m
From equation (1) and (3), =
2 2
(mv2 )2 (9 / 16)u 2 L.H.S. # R.H.S.
= or m = 0.6 kg i.e., energy is not conserved in this collision or this is a
mv 22 (15 / 16)u 2
case of inelastic collision.
(8) (a) Initial momentum of the balls (9) (a) The situation is shown in fig.
=m×9+m ×0= 9m ........(1) Let v1 and v2 be the velocities of two pieces after explosion.
where m is the mass of each ball. Applying the law of conservation of energy, we have
Let after collision their velocities are v1 and v2 respectively.
Final momentum of the balls after collision along the same m1 = 4kg v1
line = mv1 cos 30° + mv2 cos 30° m = 8kg
mv1 3 mv2 3 q
= + ........(2) 30°
2 2
According to law of conservation of momentum u = 50m/s
v2
mv1 3 mv2 3
9m= +
2 2 1 1 1
(8) (50)2 + 15000 = (D) v12 + (D) v 22
9´ 2 2 2 2
= v1 + v2 .......(3)
3 or 25000 = 2 ( v12 + v22 ) ......(1)
Applying the law of conservation of momentum along
x-axis and y-axis respectively, we get
8 (50) = 4 v1 cos q + v2 cos 30° ......(2)
and 0 = 4 v1 sin q
Stationary ball
= 4 v2 sin 30° = 2 v2 ......(3)
(a) Before collision
The initial momentum of the balls along perpendicular v2
or sin q = ......(4)
direction = 0 . 2v1
Final momentum of balls along the perpendicular direction From eq. (2)
m 100 = v1 cos q + v2 cos 30°
= mv1 sin 30° – mv2 sin 30° =
(v – v ) (10) (a) Let m be the mass of the rocket and vr the relative
2 1 2
velocity of the gas ejecting from the rocket. Suppose the
Again by the law of conservation of momentum
fuel is burnt at a rate (dm/dt) to provide the rocket an
(m/2) (v1 – v2) = 0
acceleration a.
t.me/Ebooks_Encyclopedia27. t.me/Magazines4all

DPP/ P 13 39
(13) (a) Let the speed of the body before explosion be u. After
vr æ dm ö
Then a = ç ÷ –g ......(1) explosion, if the two parts move with velocities u1 and u2
m è dt ø in the same direction, then according to conservation of
Here vr = 250 m/s, m = 500 kg, g = 10 m/s2 and a = 20 m/s2 momentum,
dm m a Mu1 + (1 – a) M u2 = Mu
Now from (1) = (a + g) The kinetic energy T liberated during explosion is given
dt vr
1 1 1
500 by T = a M u12 + (1 – a) M u 22 – Mu2
= (20 + 10) = 60 kg/s 2 2 2
250
(11) (a) Let m1 and m2 be the masses of electron and hydrogen 1 1 1
= a M u12 + (1 – a) M u 22 –
atom respectively. If u1 and v1 be the initial and final 2 2 2M
velocities of electron, then initial kinetic energy of electron [ a Mu1 + (1 – a) Mu2]2
æ 1ö 1
Ki = çè ÷ø m u12 = M a (1 – a) [ u12 + u 22 – 2 u1 u2]
2 2

æ 1ö 2T
Final kinetic energy of electron Kf = çè ÷ø m v12 (u1 – u2)2 =
2 a (1 - a )M
Fractional decrease in K.E.,
2T
Ki - K f v12 Þ (u1 – u2) =
a(1 - a)M
= 1 - .........(1)
Ki u12 (14) (a) The situation is shown in fig.
For such a collision, we have Let A and B be two pieces of equal mass (1/5 kg) which fly
off perpendicular to a each other with equal velocity
æ m1 - m 2 ö (30 m/sec)
v1 = ç m + m ÷ u1
è 1 2ø Momentum of A or B = (1/5 × 30)

v1 æ m1 - m 2 ö
\ =ç ÷ .........(2) A 1
u1 è m1 + m 2 ø m = kg
5
From eqs. (1) and (2) we have v = 30 m/s
2
Ki - K f æ m1 - m 2 ö 4m1m2
=1– ç =
Ki è m1 + m 2 ÷ø (m1 + m2 )2
B
Ki - K f 4(m 2 / m1 ) 4 ´ 1850
or = =
Ki (1 + m 2 / m1 )2 (1 + 1850) 2 1
m= kg
= 0.00217 = 0.217% 5
(12) (c) v = 30 m/s
3
C m= kg
5
m

\ Resultant momentum
m = {(1 / 5) ´ 30}2 + {(1 / 5) ´ 30}2 = 6 2 kg m/sec
along the bisector of Ð AOB
2m
(3/5) × v = 6 2 Þ v = 10 2 m/sec
(15) (c) The situation is shown in fig.
v/ 2 Equating the total initial and final momentum along each
axis, we get
Now the total energy released in the explosion mv1 + 0 = (M + m) v' cos q .........(A)
2 0 + Mv2 = (M + m) v' sin q ......... (B)
1 1 1 æ v ö 3
= mv2 + mv2 + 2m ç = mv2 Squaring and adding eq. (A) and (B), we get
2 2 2 è 2 ÷ø 2 (mv1)2 + (Mv2)2 = (M + m)2 v'2 .........(C)
t.me/Ebooks_Encyclopedia27. t.me/Magazines4all

EBD_7156
40 DPP/ P 13
(18) (a) A bullet is fired from the gun. The gun recoils, the kinetic
y energy of the recoil shall be less than the kinetic energy of
the bullet.
(M+m)v¢ (19) (a) Conservation of linear momentum is equivalent to
Newton's second law of motion
(20) (a) In an inelastic collision momentum is conserved but
A kinetic energy is not.
q x (21) (a) Inelastic collision is the collision of electron and
mv1 positron to an inhilate each other.
(22) (a) Total kinetic energy is not conserved in inelastic
collisions but momentum is conserved
Mv2 (23) (a) (1) when m1 = m2 and m2 is stationary, there is maximum
transfer of kinetic energy in head an collision
(2) when m1 = m2 and m2 is stationary, there is maximum
transfer of momentum in head on collision
(3) when m1 >> m2 and m2 is stationary, after head on
The final momentum collision m2 moves with twice the velocity of m1.
(24) (a) Momentum remains conserved
P = (M + m) v' = [(mv1 ) 2 + (Mv 2 ) 2 ]
(25) (a) Speed of particle after the collision
[form eqn. (3)]
2
Dividing eqn. (2) by eqn. (1), we have æ 15 ö
= ç ´ 3 ÷ + 25 = 5.036 m /s
è 43 ø
Mv2 æ Mv2 ö
tan q = or q = tan–1 ç mv ÷
mv1 è 1ø 30
(26) (b) Speed of the sphere just after collision = m/s
(16) (a) Let the angle of reflection be q' and the magnitude of 43
velocity after collision be v'. As there is no force parallel to (27) (a) Angular speed of sphere is zero as impulse due to
the wall, the component of velocity parallel to the surface collision passes through centre of sphere.
remains unchanged. (28) (c) When e = 0, velocity of separation along common normal
Therefore, v' sin q ' = v sin q ......(1) zero, but there may be relative velocity along common
As the coefficient of restitution is e, for perpendicular tangent.
component of velocity (29) (c) Statement – 1 is false but statement – 2 is true.
Velocity of separation = e x velocity of approach (30) (d) Momentum remains constant before, during and after
–(v' cos q ' – 0) = –e (v cos q – 0) ......(2) the collision but KE does not remain constant during
From (1) and (2) the collision as the energy gets converted into elastic
potential energy due to deformation.
v' = v sin 2 q + e2 cos 2 q
and tan q ' = tan q/e
(17) (a) The fraction of energy lost is given by,
DE mg(h - h ') h - h'
= =
E mgh h
given that, h = 2 meter and h' = 1.5 meter
DE 2 - 1.5 1
\ = =
E 2 4
t.me/Ebooks_Encyclopedia27. t.me/Magazines4all

DPP/ P 14 41

DAILY PRACTICE
PROBLEMS
PHYSICS
SOLUTIONS 14
r r
1. (c) m1 = 1, m2 = 35.51, r1 = 0, r2 = 1.27iˆ æ b hö
i.e. coordinates of centre of mass is çè , ÷ø
r r 3 3
r m r ´ m2 r2 r 35.5 ´ 1.27 ˆ
r = 11 r= r r
m1 + m2 Þ
i r m v + m2 v2
1 + 35.5 6. (b) vcm = 1 1
m1 + m2
r 35.5
r= ´ 1.27iˆ = 1.24iˆ
36.5 2 ´ 2 + 4 ´ 10
= = 7.3 m/s
2+4
y 7. (b) Let m1 = m, m2 = 2m, m3 = 3m, m4 = 4m

H Cl 2m 3m
x
m1 m2

a sin 60°
1.27Å a

r r
r m1v1 + m2 v2 60°
2. (d) vcm = x
m1 + m2
m 4m
a cos 60°
2 ´ 3 + 3 ´ 2 12
= = = 2.4 m/s
2+3 5 r
3. (c) m1 = 12, m2 = 16 y r1 = 0iˆ + 0 ˆj
r
r1 = 0iˆ + 0 ˆj, r2 = 1.1iˆ + 0 ˆj r a a 3 ˆ
r2 = a cos 60iˆ + a sin 60 ˆj = i + j
r r 2 2
r m r + m2 r2
r1 = 1 1
m1 + m2 r 3 a 3 ˆ
C O x r3 = (a + a cos 60)iˆ + a sin 60 ˆj = aiˆ + j
m1 m2 2 2
r 16 ´ 1.1 ˆ
r1 = i = 0.63iˆ r
28 r4 = aiˆ + 0 ˆj
i.e. 0.63Å from carbon atom. by substituting above value in the following formula
r r r r r r r
r m v + m2 v2 + m3 v3 r m1r1 + m2 r2 + m3r3 + m4 r4 3 ˆ
4. (a) vcm = 1 1 r= = 0.95ai + aj
m1 + m2 + m3 m1 + m2 + m3 + m4 4

20 ´ 10iˆ + 30 ´ 10 ˆj + 50 ´ 10kˆ é 3 ù
= So the location of centre of mass ê0.95a, aú
100 4 û
ë
\ vcm = 2iˆ + 3 ˆj + 5kˆ 8. (d)
5. (c) We can assume that three particles of equal mass m are r r
9. (d) m1 = 2kg, m2 = 4kg, v1 = 2m / s, v2 = -10m / s
placed at the corners of triangle.
ur ur r r
y r m v + m2 v2
r1 = 0iˆ + 0j,r
ˆ = biˆ + 0jˆ
2 vcm = 1 1
ur (0,h) m1 + m2
and r3 = 0iˆ + hjˆ
2 ´ 20 - 4 ´ 10
ur ur ur = = 0m / s
uuur m1 r1 + m2 r2 + m3 r3 2+ 4
\ rcm = 10. (a) As initially both the particles were at rest therefore
m1 + m2 + m3
velocity of centre of mass was zero and there is no
x
(0,0) (b,0) external force on the system so speed of centre of mass
b h
= ˆi + ˆj remains constant i.e. it should be equal to zero.
3 3
t.me/Ebooks_Encyclopedia27. t.me/Magazines4all

EBD_7156
42 DPP/ P 14
11. (a) For translatory motion the force should be applied on
80 80
the centre of mass of the body, so we have to calculate the 0+2´ + 4´ +0
location of centre of mass of 'T' shaped object. 2 2 30
= =
Let mass of rod AB is m so the mass of rod CD will be 2m. 16 2
Let y1 is the centre of mass of rod AB and y2 is the centre of
30
mass of rod CD. We can consider that whole mass of the Similarly y = so, r = x 2 + y2 = 30 cm
rod is placed at their respective centre of mass i.e., mass m 2
is placed at y1 and mass 2 m is placed at y2. 14. (b) Linear density of the rod varies with distance

y dm
= l (Given ) \ dm = ldx
dx
D
A B
y1
l
dx
y2
l
x x
C

Taking point 'C' at the origin, position vector of point y1


ur ur
and y2 can be written as r1 = 2l ˆj, r2 = l ˆj , and m1 = m and
Position of centre of mass
m2 = 2m
Position vector of centre of mass of the system
xcm =
ò dm ´ x
r r
r m r + m2 r2 m2l ˆj + 2ml ˆj ò dm
rcm = 1 1 =
m1 + m2 m + 2m
3
4ml ˆj 4 ˆ ò (l dx) ´ x
= = lj
3m 3 =
0
3

Hence the distance of centre of mass from C = l


4
ò l dx
3 0
12. (a) Initial acceleration is zero of the system. So it will
always remain zero because there is no external force on 3
3 é 2 x3 ù
the system.
ò (2 + x) ´ xdx ê
ëê
x + ú
3 úû
0 0
13. (b) According to figure let A is the origin and co-ordinates = =
3 3
of centre of mass be (x, y) then, é x3 ù
ò (2 + x)dx ê2x + ú
2 úû
y 0 êë 0

2 kg 4 kg
D C 9 + 9 36 12
= = = m.
9 21 7
6+
2
(x,y) 15. (c) Centre of mass lies always on the line that joins the
two particles.
8 kg r 2 kg For the combination cd and ab this line does not pass
x through the origin.
A B
For combination bd, initially it pass through the origin
but later on it moves toward negative x-axis.
m1 x1 + m 2 x 2 + m3 x 3 + m 4 x 4 But for combination ac it will always pass through
x=
m1 + m 2 + m3 + m 4 origin. So we can say that centre of mass of this
combination will remain at origin.
t.me/Ebooks_Encyclopedia27. t.me/Magazines4all

DPP/ P 14 43

1 ´ 0 + 1 ´ PQ + 1 ´ PR PQ + PR m1
16. (b) xcm = = By solving d' = - d
1 +1+ 1 3 m2
and ycm = 0 Negative sign shows that particle m2 should be
17. (a) displaced towards the centre of mass of the system.
R1
1
22. (a) We know m1r1 = m2r2 Þ m × r = constant \ r µ
m
23. (a) Depends on the distribution of mass in the body.
R2 r1 m2 1
24. (a) m1r1 = m2 r2 Þ = \r µ
mg r2 m1 m

m1a1 + m2 a2 m ´ 0 + m ´ 3g 3g
Due to net force in downward direction and towards 25. (b) acm = = =
m1 + m2 m1 + m2 2
left centre of mass will follow the path as shown in
figure. 26. (b)
18. (a) Initially both the particles were at rest so vcm = 0. As
external force on the system is zero therefore velocity
of centre of mass remains unaffected.
r r l0
19. (a) m1r1 + m2 r2 = 0 0 m/s v
m ˆ 3m r m m
Þ 15 j + r2 = 0
4 4
r
Þ r2 = -5 ˆj 2
1 æ 3mg ö 1
i.e. larger fragment is at y = – 5 cm. By COE, kç ÷ = mv 2
2 è k ø 2
20. (b) Centre of mass is closer to massive part of the body
therefore the bottom piece of bat have larger mass.
21. (b) Initial position of centre of mass 9mg 2 m
v= = 3g
k k
m1 x1 + m2 x2
rcm = ...(i)
m1 + m2 m ´ 0 + mv v 3g m
vcm = = =
m+m 2 2 k

1 2 1
27. (a) By COE in CM-frame, mvref = kx 2
x1 d 2 2
2
m1 m2 1m æ mö 1 2
2 2 ç 3g k ÷ = 2 kx
è ø
x2
9 2 m2 x=
3mg
g = kx 2 ;
If the particle of mass m1 is pushed towards the centre 2 k 2k
of mass of the system through distance d and to keep
the centre of mass at the original position let second 28. (b) Statement-1 is True, Statement-2 is True; Statement-2
particle displaced through distance d' away from the is NOT a correct explanation for Statement-1.
centre of mass. 29. (a) Initially the electron and proton were at rest so their
m1 ( x1 + d ) + m2 ( x2 + d ') centre of mass will be at rest. When they move towards
Now rcm = ...(ii) each other under mutual attraction then velocity of
m1 + m2
centre of mass remains unaffected because external
Equating (i) and (ii) force on the system is zero.
m1 x1 + + m2 x2 30. (d) The centre of mass of a system of particles depends
m1 + m2 only on the masses of particles and the position of the
particles relative to one another. The location of
m1 ( x1 + d ) + m2 ( x2 + d ') reference frame will not affect the location of centre of
= mass.
m1 + m2
t.me/Ebooks_Encyclopedia27. t.me/Magazines4all

EBD_7156
44 DPP/ P 15
DAILY PRACTICE
PROBLEMS
PHYSICS
SOLUTIONS 15
1. (a) Initial angular momentum of ring. L = Iw =Mr2w 10. (d) Angular momentum, of earth about its axis of rotation,
Final angular momentum of ring and four particles
2 2p 4pMR 2
L = lw = MR 2 ´ =
2 2
( 2 '
system Mr w = Mr + 4mr w = ) Mw
M + 4m
5 T 5T

2. (b) The angular momontum of a system of particles is con 2 2p 2


11. (d) Angular momentum, L = mvr = mwr = m ´ ´r
served when no external torque acts on the system. T

L2 ( )
2
3. (c) Rotational kinetic energy E \ L = 2EI 2 ´ 3.14 ´ 6 ´ 10 24 ´ 1.5 ´ 1011
2l = = 2.7 ´ 1040 kg - m 2 / s
7
3.14 ´ 10
L E A IA 1 2p ´ 1800
Þ A = ´ = 100 ´ = 5 12. (c) w = 2p n = = 60p rad/s
LB E B IB 4 60
4. (c) Angular momentum L = Iw constant P = t ´w
\ I increases and w decreases P 100 ´ 103
5. (c) Conservation of angular momentum Þ t = = = 531 N - m
w 60p
I1w1+I2w2 = (I1w1+I2)w ur
r dL L -L 4 A - A0 3 A0
I1w1 + I2 w 2 13. (a) t= = 2 1= 0 =
Angular velocity of system w = dt Dt 4 4
I1 + I 2
æ 60 ö
1 2p ç 0 - ÷
2p (n2 - n1 ) è 60 ø
\ Rotational kinetic energy = ( I1 + I2 ) w
2
2 14. (c) a = =
t 60

æI w +I w ö
2
( I1w1 + I2 w2 ) 2 -2p -p
1 = = rad / sec2
= ( I1 + I2 ) ç 1 1 2 2 ÷ =
2 ( I1 + I2 ) 60 30
2 è I1 + I2 ø
\t = I a
1 1 2´ p p
6. (d) Kinetic energy E = Lw = L ´ 2pn = = N -m
2 2 30 15
r r ur
L 2 E 2 n1 15. (a) t = r ´ f = (7$i + 3 $j + k$ ) ´ (-3i$ + $j + 5k$ )
\ E ¥L ´ n Þ = ´
L1 E1 n 2
$i $j k$
r
L 2 é E1 / 2 ù é n1 ù L2 L t = 7 3 1
=ê ú´ê ú Þ L2 = =
L1 ë E1 û ë 2n1 û 4 4 -3 1 5

L2 = $i (15 - 1) - $j (35 + 3) + k$ (7 + 9)
7. (b) E = . If boy stretches hgis arm then moment of
2I = 14$i - 38 $j + 16k$
inertia increases and accordingly kinetic energy of the system
t 1000
1 16. (a) a = = = 5 rad / sec 2
decreases because L = constant and E¥ I 200
I
From w = w 0 + a t = 0 + 5 ´ 3 = 15 rad/s
8. (c)According to conservation of angular momentum
t 30
I1w 17. (a) a = = = 15 rad/s 2
\ I1w1 = I2 w 2 Þ I1w = ( I1 + I2 ) w 2 Þ w 2 = I 2
I1 + I2
1 2
-7 -3 2 Q q = w 0t + at
9. (a) L = 2EI = 2 ´ 10 ´ 8 ´ 10 = 4 ´ 10 kg m / s 2
t.me/Ebooks_Encyclopedia27. t.me/Magazines4all

DPP/ P 15 45
22. (a) As mechanical contact is not made, total angular
1
= 0+ ´ (15) ´ (10)2 momentum remains constant.
2 \ Iw0= constant
= 750 rad Differentiating both sides,
18. (d) As the block remains stationary therefore D (Iw0) = 0
For translatory equilibrium Þ I Dw0 + w0 DI = 0
å Fx = 0 \ F = N Dw DI DI Dw 0
Þ + =0 Þ =-
and å Fy = 0 \ f = mg w I I w0

Dw 0 DI
f Also, =-
w0 I

2 DR æ DI 2DR ö
=- çQ = ÷ = -2 a DT
R è I R ø

L2 1
23. (a) E= = K (given) \ K µ (If L = constant)
F 2I I
O
When child stretches his arms the moment of inertia of
system get doubled so kinetic energy will becomes half i.e.
N K/2.
24. (c). Angular impulse = change in angular momentum : Frt =
L Þ L1 < L2
mg
L2
K= Þ K1 = K2
2I
For rotational equilibrium åt = 0 25. (b); 26. (a); 27. (c)
By taking the torque of different forces about point 0 Drawing the F.B. D of the plank and the cylinder.
uuur uur uuur uuuur
t F + t f + t N + t mg = 0 F sin q
As F and mg passing through point O N1
uur uuur
\ t f + tN = 0

As t f ¹ 0 \t N ¹ 0 F cos q
and torque by friction and normal reaction will be in f1
opposite direction.
19. (c) The velocity of the top point of the wheel is twice that
of centre of mass and the speed of centre of mass is mg
same for both the wheels (Angular speeds are different).

æ 4500 - 1200 ö f1
2p ç ÷ø
2p (n2 - n1 ) è 60 N1
20. (d) a = = rad/s2
t 10
Mg
3300
2 p
60 3 6 0 d e g ree N2
= ´
10 2 p s2 f2

a = 1980 degree/s 2 Equations of motion are


F cos q – f1 = ma ....(1)
1 F sinq + N1 = mg ....(2)
21. (b) q = w 0 t + a t 2 f1+ f2 = MA .....(3)
2
f1R – f2R = Ia .....(4)
Þ q = 100 rad A = Ra .....(5)
100
\ Number of revolution = = 16 (approx.)
4 ´ 55 ´
1
2p 4 F cos q 2
a= = = 10 m/s 2
3M + 8m [ ( 3 ´ 1) + (8 ´ 1)]
t.me/Ebooks_Encyclopedia27. t.me/Magazines4all

EBD_7156
46 DPP/ P 15
1 29. (b) t = rF sin q. If q = 90° then t max = rF
3´ 1 ´ 55 ´
3MF cos q 2 = 7.5 N Unit of torque is N-m.
f1 = =
3M + 8 m 3 ´ 1+ 8 ´1 30. (d) Torque = Force × perpendicular distance of the line of
action of force from the axis of rotation (d).
1 Hence for a given applied force, torque or true tendency
1 ´ 55 ´
and f = MF cos q = 2 = 2.5 N of rotation will be high for large value of d. If distance
2
3M + 8 m 3 ´ 1 + 8 ´ 1 d is smaller, then greater force is required to cause the
ur same torque, hence it is harder to open or shut down
r dL
28. (b) t = and L = I w the door by applying a force near the hinge.
dt
t.me/Ebooks_Encyclopedia27. t.me/Magazines4all

DPP/ P 16 47

DAILY PRACTICE
PROBLEMS
PHYSICS
SOLUTIONS 16
(1) (b) As the mass of disc is negligible therefore only moment
1 R 2.5
of inertia of five particles will be considered. (8) (b) MR 2 = MK 2 Þ K = = = 1.76 cm
2 2 2
I= å mr 2 = 5 mr2 = 5 × 2 × (0.1)2 = 0.1 kg-m2 (9) (c) I = 2MR2 = 2 × 3 × (1)2 = 6 gm-cm2

(2) (a) I=
1
2
1
2
(
MR 2 = ´ pR 2 t ´ p ´ R 2 ) (10) (a) I=
5
4
Mr 2
(11) (a)
Þ I µ R4 (As t and p are same)
4 y
4
I æR ö æ 0.2 ö 1 Q
\ 1 =ç 1÷ =ç ÷ =
I2 è R 2 ø è 0.6 ø 81
R/2
1 1
(3) (a) I= MR 2 = ´ 0.5 ´ (0.1)2 = 2.5 ´ 10 -3 kg - m 2
2 2

t 31.4
(4) (a) I== = 2.5kg m 2
a 4p 2R
(5) (d) Let the mass of loop P (radius = r) = m
So the mass of loop Q (radius = nr) = nm Moment of inertia of the system about yy¢
Iyy¢ = Moment of inertia of sphere P about yy¢
+ Moment of inertia of sphere Q about yy¢
Moment of inertia of sphere P about yy¢
2
r 2 æRö
= M ç ÷ + M ( x) 2 [Parallel axis theorem]
5 è2ø
P
Q 2
2 æRö 2
= = M ç ÷ + M (2R )2 = MR + 4MR 2
Moment of inertia of loop P, IP = mr2 5 è2ø 10
Moment of inertia of loop Q. IQ = nm (nr)2 = n3 mr2 2
2 æRö
IQ Moment of inertia of sphere Q about yy¢ is M çè ÷ø
\ = n3 = 8 Þ n = 2 5 2
IP
2
MR 2 2 æRö 21
(6) (c) Moment of inertia of sphere about its tangent Now I yy ¢ = + 4MR 2 + M ç ÷ = MR 2
10 5 è2ø 5
7 7
MR 2 = MK 2 Þ K = R (12) (a) M.I. of system about the axis which passing through
5 5 m1
(7) (a) Moment of inertia of system about point P
m m m1
l
2 a a
p

m2 m3
m m a/2 a/2

2
æ l ö 2 2
= 4m ç = 2ml 2 and 4mK2 = 2ml2 Isystem = m1 (0)2 + m2 æç ö÷ + m3 æç ö÷
a a
è 2 ÷ø è 2ø è2ø
l
\K = a2
2 Isystem = (m2 + m3 )
4
t.me/Ebooks_Encyclopedia27. t.me/Magazines4all

EBD_7156
48 DPP/ P 16
Ml 2 3
(13) (a) M.I. of rod (1) about Z – axis I1 = Moment of inertia of loop about given axis = MR 2
3 2
2
3 æ Lö 3rL3
= rL ç ÷ =
2 è 2p ø 8p 2
3
1 1 æ M ö 1 M2
(19) (b) M.I. of disc = MR 2 = M ç =
2 2 è pt r ÷ø 2 pt r
2 1
æ M M ö
çè As r = Therefore R 2 =
2
pR t pt r ÷ø

1
Ml 2 If mass and thickness are same then, I µ
M.I. of rod (2) about Z-axis, I 2 = r
3
M.I. of rod (3) about Z – axis, I3 = 0 I r 3
Because this rod lies on Z-axis \ 1 = 2 = .
I 2 r1 1
2Ml 2 (20) (c) According to problem disc is melted and recasted into
\ Isystem = I1 + I2 + I3 = a solid sphere so their volume will be same.
3
(14) (c) Distribution of mass about BC axis is more than that 2 4 3
VDisc = VSphere Þ pRDisc t= pRSphere
about AB axis i.e. radius of gyration about BC axis is 3
more than that about AB axis.
i.e. KBC > KAB \ IBC > IAB > ICA 3 æ RDisc ö 4 3 é RDisc ù
Þ pRDisc çè 6 ÷ø = 3 pRSphere êt = 6 , given ú
ë û
Ml 2 0.12 ´ 12
(15) (a) I= = = 0.01 kg - m 2 RDisc
12 12 3 3
Þ RDisc = 8RSphere Þ RSphere =
(16) (c) 2
Moment of inertia of disc
1 1 2
IDisc = MR Disc = I (given)
2
2
\ M ( R Disc ) = 2I
x 2

2 2
Moment of inertia of sphere Isphere = MRSphere
5
2
1 2 æ RDisc ö M 2I 1
I1 = M.I. of ring about its diameter = mR
2 = M = ( RDiscs )2 = =
2 5 çè 2 ÷ø 10 10 5
I2 = M.I. of ring about the axis normal to plane and (21) (d) Moment of inertia of system about YY'
passing through centre = mR2 I = I 1 + I2 + I3 Y
Two rings are placed according to figure. Then
1 3 3 1
1 3 = MR 2 + MR 2 + MR 2
I xx ¢ = I1 + I 2 = mR 2 + mR 2 = mR 2 2 2 2
2 2 2 3
(17) (a) Mass of the centre disc would be 4M and its moment 7
= MR 2
2
1 2
of inertia about the given axis would be (4 M ) R .
2 (22) (d) As C is the centre of mass, so, IC will be minimum.
For the given section the moment of inertia about the Also more mass is towards B so IA > IB.
(23) (a) Applying the theorem of perpendicular axis,
1
same axis would be one quarter of this i.e. MR 2 . I = I1 + I2 = I3 + I4
2 Because of symmetry, we have I1 = I2 and I3 = I4 Hence
(18) (d) Mass per unit length of the wire = r I = 2I1 = 2I2 = 2I3 = 2I4 or I1 = I2 = I3 = I4
Mass of L length, M = rL i.e. sum of two moment of inertia of square plate about
and since the wire of length L is bent in a or of circular any axis in a plane (Passing through centre) should be
L equal to moment of inertia about the axis passing
loop therefore 2pR = L Þ R = through the centre and perpendicular to the plane of
2p the plate.
t.me/Ebooks_Encyclopedia27. t.me/Magazines4all

DPP/ P 16 49
(24) (d) Moment of inertia depends on all the three factors given (28) (c) Radius of gyration of a body is not a constant quantity.
in (1), (2) & (3). Its value changes with the change in location of the
axis of rotation. Radius of gyration of a body about a

é2 ù r12 + r22 + ..... + rn2


(25) (d) I = 4 ê MR2 + M (R 2)2 ú given axis is given as K =
ë5 û n
(29) (c) The moment of inertia of a particle about an axis of
é2 ù rotation is given by the product of the mass of the
= 4MR 2 ê + 2ú
ë 5 û particle and the square of the perpendicular distance
of the particle from the axis of rotation. For different
4 MR 2 ´ 12 48 MR 2 axis, distance would be different, therefore moment of
= = . inertia of a particle changes with the change in axis of
5 5
rotation.
(26) (b) Let a be the acceleration of centre of mass (30) (a) When earth shrinks, it angular momentum remains
Mg – T = 0 ... (i)
2 2p
F.x = T.2x ... (ii) constant. i.e. L = I w = mR 2 ´ = constant.
5 T
\ T µ I µ R 2 . It means if size of the earth changes
F then its moment of inertia changes.
x In the problem radius becomes half so time period

M 1
(Length of the day) will becomes of the present value
4
(27) (c) remain the same 24
i.e. = 6 hr.
4
t.me/Ebooks_Encyclopedia27. t.me/Magazines4all

EBD_7156
50 DPP/ P 17
DAILY PRACTICE
PROBLEMS
PHYSICS
SOLUTIONS 17
1. (a) Since disc is rolling (without slipping) about point O.
Hence g sin q g sin q g / 2 5 g
w 6. (d) a= = = =
K 2 2 7 / 5 14
1+ 1+
Q R2 5

K2 2
C As q = 30o and =
2 5
R
P 2 gh
7. (b) We know v =
k2
1+
O r2
OQ > OC > OP Q v = rw
\ vQ > vC > vP v 2 gh
\w = =
2. (d) Applying the theorem of perpendicular axis, r r + k2
2

I = I1 + I2 = I3 + I4 2mgh 2mgh 2mgh


Because of symmetry, we have Þw= 2 2
= 2
=
mr + mk mr + I I + mr 2
I1 = I2 and I3 = I4
K2
Hence I = 2I1 = 2I2 = 2I3 = 2I4 8. (a) Because its M.I. (or value of ) is minimum for
R2
or I1 = I2 = I3 = I4
sphere.
i.e. sum of two moment of inertia of square plate about 9. (b) As body is moving on a frictionless surface. Its
any axis in a plane (Passing through centre) should be mechanical energy is conserved. When body climbes
equal to moment of inertia about the axis passing up the inclined plane it keeps on rotating with same
through the centre and perpendicular to the plane of angular speed, as no friction force is present to provide
the plate. retarding torque so
3. (a) By the conservation of energy
1 2 1 1
Iw + mv 2 ³ Iw2 + mgh Þ v ³ 2gh
2 2 2
1
10. (a) MR 2 = I Þ MR 2 = 2I
l/2 2
Moment of inertia of disc about a tangent in a plane
a
5 5 5
= MR2 = (2I ) = I
P.E. of rod = Rotational K.E. 4 4 2
l 1 11. (d) Moment of inertia of system about YY’
mg sin a = Iw 2
2 2 I = I1 + I2 + I3

l 1 ml2 2 3 g sin a 1 3 3 7
= MR 2 + MR 2 + MR 2 = MR 2
Þ mg sin a = w Þw=
2 2 3 l 2 2 2 2
But in the problem length of the rod 2L is given Y

3g sin a
\w =
2L
4. (c) Graph should be parabola symmetric to I- axis, but it 1
should not pass from origin because there is a constant
value Icm is present for x = 0 .
2 3
2 gh 2 gh 4
5. (b) v= = = gh
K2 1 3
1+ 1+
R2 2
t.me/Ebooks_Encyclopedia27. t.me/Magazines4all

DPP/ P 17 51
19. (a) M.I. of complete disc about ‘O’ point
I Ring MR 2
12. (b) = = 2 :1 1
I Disc 1 ITotal - (9M ) R 2
MR 2 2
2
13. (a)
14. (b) It follows from the theorem of parallel axes.

15. (a) l
A B 2R/3
O
P
R
O

Moment of inertia of Rod AB about point P and

MI2
perpendicular to the plane = R/3
12
M.I. of rod AB about point ‘O’
2
MI 2 æ Iö MI2 O
= + Mç ÷ =
12 è 2ø 3 R
(By using parallel axis theorem)
but the system consists of four rods of similar type so
by the symmetry
æ Ml2 ö
Isystem = 4 ç ÷
è 3 ø
Radius of removed disc = R
16. (a) 3
17. (d) l
9M
\ Mass of removed disc = =M
9
b [As M = pR 2 t \ M ¥R 2 ]
O M.I. of removed disc about its own axis
2
1 æRö MR 2
2
Mb = Mç ÷ =
M.I. of plate about O and parallel to length = 2 è3ø 18
12 Moment of inertia of removed disc about ‘O’
18. (d) Iz = Ix + I y Iz 2
MR2 æ 2R ö MR2
Iremoved disc = Icm + mx2 = +M ç ÷ =
18 è 3 ø 2
Ix
M. I. of complete disc can also be written as
ITotal = I Re moved disc + I Re maining disc

MR 2
ITotal = + I Re maining disc ..... (ii)
2
Equating (i) and (ii) we get
Iy
MR 2 9MR 2
+ I Re maining disc =
2 2
200 = I D + I D = 2 I d
9MR 2 MR 2 8MR 2
\ I D = 100 gm ´ cm 2 \ I Re maining disc = - = = 4MR 2
2 2 2
t.me/Ebooks_Encyclopedia27. t.me/Magazines4all

EBD_7156
52 DPP/ P 17
2 æ K2 ö
ML2 æLö 1 1 K2
20. (d) I = Icm + Mx 2 = +M ç ÷ 24. (a) KT = KR Þ mv2 = mv2 ç 2 ÷ Þ \ 2 = 1
12 è4ø 2 2 èR ø R
I I cm
This value of K 2 / R 2 match with hollow cylinder..
25. (b) 26. (c) 27. (d)
(i) Let acceleration of centre of mass of cylinder be a then
acceleration of block will be 2a.
For linear motion of cylinder T + f – 2mgsinq = 2m(a)
For rolling motion of cylinder
L/4 L/4 æ 2mR 2 ö æ a ö
ML2
ML 7ML2 2 (T – f) R = Ia = ç 2 ÷ çè R ÷ø Þ T – f = ma
= + = è ø
12 16 48
For linear motion of block
21. (c) w 2 = w 20 - 2aq Þ 0 = 4p 2 n2 - 2aq 2
mg – T = m (2a) Þ a = (1 - sin q) g
2 7
æ 1200 ö
4p2 ç ÷
q= è 60 ø = 200p2 rad T
2´ 4
2m T
\ 2pn - 200p 2 Þ n = 100 p = 314 revolution R f
1 2 m 2a
22. (b) Rotational K.E. = Iw & 2mgsinq
2 mg
q
1 2 1 ///////////////////////////////////////////
T.E. = Iw + MV 2
2 2
æ2 ö æ 3 + 4sin q ö mg
1 1 1 (ii) T = mg - 2m ç g (1 - sin q)÷ = ç ÷ø
= Iw2 + MR 2 w2 = w2 (I + MR 2 ) è7 ø è 7
2 2 2
For ring I = MR2 æ 1 + 6sin q ö
(iii) F = T – ma = ç
è 7 ÷ø mg
1 2 1
\ T.E. = w (MR 2 + MR 2 ) = w2 ´ 2MR 2 28. (c) The acceleration of a body rolling down an inclined
2 2
g sin q
1 plane is given by a =
Rotational K.E. = MR 2 w2 I
2 1+
MR 2
1
MR 2 w2 I MR 2
2 1 For hollow cylinder
\a = = = =1
1 2
w ´ 2MR 2
2 MR 2 MR 2
2
1
MR 2
2 2I 1
For a solid sphere I = MR 2 For solid cylinder = =
2 2 2
5 MR MR
1 æ2 Þ Acceleration of solid cylinder is more than hollow
ö 1 7
\ T.E. = w2 ç MR 2 + MR 2 ÷ = w2 MR 2 ´ cylinder and therefore solid cylinder will reach the
2 è5 ø 2 5 bottom of the inclined plane first.
\ Statement -1 is false
Rotational K.E. = 1 ´ 2 MR 2 w2 • Statement - 2
2 5
In the case of rolling there will be no heat losses.
1 2 Therefore total mechanical energy remains conserved.
´ MR 2 w2 The potential energy therefore gets converted into
2 5 2
b= = kinetic energy. In both the cases since the initial
1 2 7 7
w MR 2 ´ potential energy is same, the final kinetic energy will
2 5
also be same. Therefore statement -2 is correct.
K2 29. (b) Frictional force on an inclined plane
23. (a) Time of descent µ . Time of descent depends upon
R2 1
= g sin a ( for a disc ) .
the value of radius of gyration (K) or moment of inertia 3
(I). Actually radius of gyration is a measure of moment 30. (c) The moment of inertia about both the given axes shall
of inertia of the body. be same if they are parallel. Hence statement–1 is false.
t.me/Ebooks_Encyclopedia27. t.me/Magazines4all

DPP/ P 18 53

DAILY PRACTICE
PROBLEMS
PHYSICS
SOLUTIONS 18
(1) (c) If r is the distance between m and (M – m), the gravita- (7) (c) The P.E. of the object on the surface of earth is
tional force will be -
GMm
m(M - m) G U1 = –
R
F=G = 2 (mM – m2)
r2 r GMm
The P.E. of object at a height R, U2 = –
dF (R + R)
The force will be maximum if, =0
dm
GMm 1
d éG ù The gain in P E is U2 – U1 = = mgR
i.e, (mM - m 2 ) ú = 0 2R 2
dm êë r 2 û
é GM ù
m 1 êQ g = 2 on surface of earth ú
or = (as M and r are constants) ë R û
M 2 (8) (c) Resultant force on particle '1'
c Fr = 2 F + F'
(2) (c) mg = 3 kg , v = 2
Gm2 Gm2 mv2
m0 3 3 or Fr = 2 2 + 2 =
m= = = kg 2r 4r r
1 - (v 2 / c 2 ) c 2 3
1- 2 Gm æ 2 2 + 1ö
4xc 2
r çè 4 ÷ø
or v =
(3) (a) g' = g – Rew2 (at equator l = 0)
If a body is weightless, (9) (b) The resultant gravitational force on each particle pro-
g' = 0 , g – Rew2 = 0 vides it the necessary centripetal force
g 10 mv2
Þ w= = = 1.25 × 10-3 rad/sec. \ = F2 + F2 + 2F2 cos 60o = 3F ,
R 6400 ´ 103 r
(4) (b) The apparent weight of person on the equator 3 2 l
(latitude l = 0) is given by But r = l´ =
W' = W – m Rew2, 2 3 3
3 3 GM
W' = W = mg \ v=
5 5 l
3 3 (10) (b) The acceleration due to gravity on the surface of the
mg = mg – mRw2 or mRw2 = mg – mg earth, in terms of mass Me and radius Re of earth, is
5 5
GM e
2g 2 9.8 given by g =
w= = 5´ rad/ sec R e2
5R 6400 ´ 103
if Mm be the mass of the moon, Rm its radius, then the
= 7.826 × 10–4 rad/sec
acceleration due to gravity on the surface of the moon
(5) (c) According to question,
GM m
G ´ 4M p G Me will be given by g' =
g' = on the planet and g = on the earth R m2
R 2p R 2e
Dividing eq. (ii) by eq. (i), we get
Q Rp = Re and Mp = Me
2 2
g' g' Mm æ Re ö 1 æ 4ö 1
Now, = 4 Þ g' = 4g = 40 m/sec2 = M ç ÷ = ´ ç ÷ =
g g e è Rm ø 80 è 1 ø 5
Energy needed to lift 2 kg mass through 2m distance \ g' = g/5.
= mg'h = 2 × 40 × 2 = 160 J (11) (b) The value of g at the height h from the surface of earth
Gm1 Gm 2 æ 2h ö
(6) (d) Vg = Vg1 + Vg2 = - - g ' = g ç1 - ÷
r1 r2 è Rø
The value of g at depth x below the surface of earth
é102 103 ù
= – 6.67 × 10–11 ê + ú –7 æ xö
ëê 0.5 0.5 ûú = – 1.47 × 10 Joule/kg g ' = g ç1 - ÷
è Rø
t.me/Ebooks_Encyclopedia27. t.me/Magazines4all

EBD_7156
54 DPP/ P 18
(16) (d) In the position of solar eclipse, net force on earth
æ 2h ö æ xö
These two are given equal, hence çè1 - ÷ø = çè 1 - ÷ø FE = FM + FS
R R In the position of lunar eclipse, net force on earth
On solving, we get x = 2h F'E = FS – FM
(12) (a) If g be the acceleration due to gravity at the surface of
the earth, then its value at a height h above the earth's
surface will be -
g g' 1
g¢ = Here =
2 g 9
æ h ö
çè1 + R ÷ø
e

1 1 h
\ =
2
or 1 + =3
9 æ h ö R e
çè 1 + R ÷ø
e
\ Change in acceleration of earth,
or h = 2 Re = 2 × 6400 = 12800 km.
(13) (c) Consider the case of a body of mass m placed on the 2GM 2 ´ 6.67 ´ 10 -11 ´ 7.36 ´ 10 22
Df = =
earth's surface (mass of the earth M and radius R). If g R2 3.822 ´ 1016
is acceleration due to gravity, then = 6.73 × 10-5 m/s2
Me m GM e (17) (c) Let Me be the mass of the earth. The work required
mg = G 2 or g =
R R2 é 1 1 ù
where G is universal constant of gravitation. W = GMe m ê R - R + h ú
ë e e û
Now when the radius is reduced by 1%, i.e., radius
becomes 0.99 R, let acceleration due to gravity be g', GM e mh gR e2 mh
= = [\ GMe = gRe2]
GM e R e (R e + h) R e (R e + h)
then g' =
(0.99R) 2 mgh
From equation (A) and (B), we get =
æ h ö
çè1 + R ÷ø
g' R2 1 e
= =
g (0.99R) 2 (0.99) 2 (18) (a) The P.E of the mass at d/2 due to the earth and moon is

2 Earth
æ 1 ö Moon
\ g' = g × ç P
è 0.99 ÷ø
or g' > g R1 O2
O1 R2
m M 2
Thus , the value of g is increased. M 1
(14) (a) Force of gravity at surface of earth, d
F1 = Gm M/R2 .......... (1) GM1m GM 2 m
Force of gravity at height H is U=– 2 –2
d d
F2 = Gm M (R + H)2 .......... (2)
Dividing (A) by (B) and Rearranging 2Gm
or U = – (M1 + M2) (Numerically)
d
æ F1 ö
H = R ç F - 1÷ = 350 km where (F2 = .9F1) 1 G
è 2 ø m Ve2 = U ÞVe = 2 (M1 + M 2 )
2 d
(15) (a) The extension in the length of spring is
(19) (d) Let m be the mass of the body. The gravitational poten-
mg GMm tial energy of the body at the surface of the earth is
x= = ,
k r2 k GM e m
U=–
Re
x2 R2
\ xµ 1 , \ = The potential energy at a height 10 Re above the sur-
x1 (R + h) 2
r2 face of the earth will be
2 GM e m
æ 6400 ö U' = –
or x2 = 1 × ç
è 7200 ÷ø
= 0.79 cm . (R e +10R e )
t.me/Ebooks_Encyclopedia27. t.me/Magazines4all

DPP/ P 18 55
\ Increase in potential energy (23) (a) All statements except (4) are wrong.
(24) (a) Value of g decreases when we go from poles to equator.
GM e m æ GM e m ö 10 GMe m
U' – U = – + ç R ÷ = 11 R
11R e è e ø e
This increase will be obtained from the initial kinetic
energy given to the body. Hence if the body be thrown (25) (b) r2 r1
with a v velocity then
10 GMe m 20Gme
1
mv2 = 11 R Þ v = 11R Gravitational PE at perihelion < ,GMm / r1 as r1 is
2 e e minimum Therefore, PE is minimum.
Substituting the given values, we get (26) (c) Total energy = constant.
(27) (c) As Pluto moves away, displacement has component
æ 20 ´ (6.67 ´ 10 -11 ) ´ (6 ´ 10 24 ) ö opposite to air force, hence work done is –negative.
v= ç ÷
è 11 ´ (6.4 ´ 106 ) ø Fg
(28) (b) For two electron = 10 -43 i.e. gravitational force is
= 1.07 × 104 m/s. Fe
Gm ∋ M , m( negligible in comparison to electrostatic force of
(20) (b) F< attraction.
r2 (29) (c) The universal gravitational constant G is totally
dF different from g.
For maximum force <0
dm FR 2
G=
d æç GmM Gm2 ö÷÷ Mm
Þ ç , ÷< 0
dm èçç r 2 r 2 ø÷÷
The constant G is scalar and posses the dimensions
é M -1L3T -2 ù .
m 1 ë û
Þ M , 2m < 0 Þ <
M 2 GM
g=
(21) (b) 2 2
g ' = g - w R cos l R2
-2
For weightlessness at equator l = 0 and g ' = 0 g is a vector and has got the dimensions éë M LT ùû .
0

g 1 rad It is not a universal constant.


\ 0 = g - w2R Þ w = =
R 800 s (30) (a) As the rotation of earth takes place about polar axis
therefore body placed at poles will not feel any cen-
(22) (a) k represents gravitational constant which depends only
trifugal force and its weight or acceleration due to grav-
on the system of units.
ity remains unaffected.
t.me/Ebooks_Encyclopedia27. t.me/Magazines4all

EBD_7156
56 DPP/ P 19
DAILY PRACTICE
PROBLEMS
PHYSICS
SOLUTIONS 19
(1) (a) A body projected up with the escape velocity vewill go (4) (a) When S2 is closest to S1, the speed of S2 relative to S1
to infinity. Therefore, the velocity of the body falling is v2 – v1 = p × 104 km/hr. The angular speed of S2 as
on the earth from infinity will be ve. Now, the escape observed from S1 (when closest distance between them
velocity on the earth is is r2 – r1 = 3 × 104 km)

ve = gR e = 2 ´ (9.8m/s 2 ) ´ (6400 ´ 10 3 m) v2 - v1 p ´ 10 4 p
w= =– 4 = – 3 rad/hr ,
r2 - r1 3 ´ 10
= 1.2 × 10 104 m/s = 11.2 km/s.
The kinetic energy acquired by the body is p
| w|= rad/hr
1 1 3
K= m ve2 = × 100 kg × (11.2 × 103 m/s)2 (5) (c) Period of revolution of earth around sun
2 2
9
= 6.27 × 10 J. 4p 2 R e2
Te2 =
GMm GM GMs
(2) (d) We know that 2 = m w2r or = w2 r..
r r2 Period of revolutions of moon around earth

GM 4p2 R m
2
Tn2 =
\ r3 = GM e
w2
where w is the angular velocity of the satellite 2 3
æ Te ö æ Me ö æ R e ö
In the present case, w = 2w0, \ ç = çM ÷ ç
÷
è Tm ø è s ø è R m ÷ø
where w0 is the angular velocity of the earth.
\ w = 2 × 7.3 × 10–5 rad/ sec. 2 3
G = 6.673 × 10–11 n-m2/kg2 Ms æ Tm ö æ R e ö (393)3
and M = 6.00 × 1024kg. \ M =ç ÷ ç ÷ = = 3.56 × 105
e è Te ø è R m ø 132
Substituting these values in equation (A), we get
(6) (a) According to law of conservation of angular momen-
(6.673 ´ 10 -11
) (6.00 ´ 10 ) 24 tum, mvr = constant
r3 = Þ vr = constant
(2 ´ 7.3 ´ 10 -5 ) 2 vmax .rmin = vmin. rmax
Solving we get r = 2.66 × 107m. VB vmax rmax
(3) (a) Þ = = =x
VA v min rmin
v2 (7) (a) Angular momentum of satellite, J = mvr. But,
v1 ­ GMm mv2 GM
­ S2 2 = Þv=
r
r r
R 1 S1
\ J= m GMr
¬ R2 ®
GM
(8) (a) The orbital velocity of space ship, v0 =
r
From Kepler's Law, T2 µ r3
If space, ship is very near to earth's surface,
2 3 2 æ 104 ö
æ T1 ö æ r1 ö æ 1ö GM
\ ç ÷ =ç ÷ Þç ÷ =ç r ÷ r = Radius of earth = R \ v0 =
è T2 ø è r2 ø è 8ø è 2 ø R
Þ r2 = 4x 104 km = Rg = 6.4 ´ 106 ´ 9.8
2 pr = 7.9195 × 103 m/sec = 7.195 km/sec
v = wr = The escape velocity of space-ship
T
ve = 2Rg = 7.9195 2 = 11.2 km/sec
æ r1 r2 ö
\ | v2 – v1| = 2p ç T - T ÷ = p × 104 km/hr Additional velocity required = 11.2– 7.9195=3.2805 km/
è 1 2ø sec.
t.me/Ebooks_Encyclopedia27. t.me/Magazines4all

DPP/ P 19 57
(15) (c) Total energy of the particle at P
(9) (b) The escape velocity ve = 2gR
1 GM1m GM 2 m
Now, (Ve)moon = 2gR E = EkP + U = mv 2e – -
2 d/2 d/2
(Ve)earth = 2 ´ 6g ´ 10R , 1 2Gm
= mv 2e – (M1 + M2)
(Ve )earth 2 d
So (V ) =8 At infinite distance from M1 and M2, the total energy
e moon
of the particle is zero.
2GM 1 2Gm
(10) (b) Escape velocity = = 2gR \ mv 2e = (M1 + M2),
R 2 d

Vp gp Re 4G
´ \ ve = (M1 + M 2 )
\ = ge R p = 10 ´ 1 = 10 d
Ve

Vp = 10 Ve GM gR 2 9.8 ´ 6.42 ´ 1012


(11) (a) We know that T2 µ a3 (16) (d) v = = =
r r 8 ´ 106
Given that (12 T)2 µ a13 and T2 µ a23
= 7.08 km/sec.
a13 (12T)2 (17) (b) From conservation of energy,
\ = = 144 The energy at height h = Total energy at earth's sur-
a 23 T2
face
a1 GMm 1 GMm
or = (144)1/3 = 5.242 0– = mv2 – ,
a2 R+h 2 R
Hence the jupiter's distance is 5.242 times that of the
1 GMm GMm GMm GMm
earth from the sun. mv2 = – = –
2 R R+h R 2R
(12) (b) We know that T2 µ a3 Þ T µ (a)3/2
3 GM R 2g
T æa ö2 Þ v = = = Rg
\ mars = ç mars ÷ = (1.524)3/2 = 1.88 R R
Tearth è a earth ø
As the earth revolves round the sun in one year and = 6400 ´ 103 ´ 9.8 = 7.919 × 103 m/s
hence, Tearth = 1 year. = 7.919 km/sec
\ Tmars = Tearth × 1.88 = 1 × 1.88 = 1.88 earth-year. (18) (a) If a body is projected from the surface of earth with a
3/ 2 3/2
velocity v and reaches a height h, then using law of
Tmercury æ a mercury ö æ 0.387 ö
(13) (d) =ç ÷ = ç 1 mgh
Tmars è a mars ø è 1.5 24 ÷ø conservation of energy, mv2 = .
2 1+ h / R
3/2
æ 1.524 ö Given v = Kve = K 2gR and h = r – R
\ Tmars = Tmercury × ç
è 0.387 ÷ø
1 mg(r - R) R
= (0.241years) × (7.8) = 1.9 years. Hence, mK2 2gR = or r =
2 r-R 1 - K2
1+
2 R
æ 2 pr ö
T2 çè v ÷ø (2pr) 2 1 4p 2
(19) (a) Orbital speed,
0
(14) (a) = 3 = r =
r3 r r3 GM GM v0 = g R e = 9.8 ´ (6.4 ´ 10 6 )
= 7.2 × 103 m/s = 7.2 km/s.
mv02 GMm 2 GM Period of revolution,
[\ = 2 , v0 = ]
r r r
T = 2p R/g
2 2
T 4p
Slope of T2 – r3 curve = tan q = 3 = = 2 × 3.14 (6.4 ´ 10 6 )/9.8 = 5075 s = 84.6 minutes.
r GM
t.me/Ebooks_Encyclopedia27. t.me/Magazines4all

EBD_7156
58 DPP/ P 19
(20) (d) If the period of revolution of a satellite about the earth
be T, then GM
(24) (a) v0 =
r
4p 2 (R e + h)3
T2 = (25) (d) T 2 µ R3
GMe
where h is the height of the satellite from earth's sur- Te2 = KRe3 ; Tm2 = kRm
3
; T 2 = kR3
face.
Re + Rm
4p 2 (R e + h)3 R=
\ Me = 2
GT2
3
The satellite is revolving just above the earth, hence h 2
éT 2 / 3 T 2 / 3 1 ù
is negligible compared to Re. Þ T = k ê e1/ 3 + m1/ 3 ´ ú
êë k k 2 úû
4p 2 R e3
\ Me = 3/ 2
GT2 é T 2 / 3 + Tm2 / 3 ù
ÞT =ê e ú
4 ëê 2 ûú
But Me = p Re3 r where r is the density of the earth.
3
GM s M e GM s M
4 4p 2 R e3 (26) (a) Ee = - =–
Thus p Re3 r = 2 Re 2R
3 GT2
2 Re Ee M
= ´
3p Me æ Re + Rm ö
\ r T2 = . 2ç ÷ø
G è 2
which is universal constant. To determine its value,
2M æ Re ö
3p 3 ´ 3.14 = Ee
r T2 = = M e çè Re + Rm ÷ø
G 6.67 ´ 10 –11 m3 / kg-s2
(27) (c) Areal velocity of the artificial planet around the sun
E KQ 2
vQ will be more than that of earth.
(21) (a) = .
E KP v2P g
(28) (a) v0 = R e
Linear velocity of earth, Re + h

2pR e 6.28 ´ 6.4 ´ 10 6 For satellite revolving very near to earth R e + h = R e


Ve= = = 463 m/s
Te 24 ´ 3600
As ( h << R )
Orbital velocity, V0 = R e g = 7.9 × 103 m/s
v 0 = R e g ; 64 ´ 105 ´ 10 = 8 ´ 103 m / s = 8 kms -1
According to question,
VP = V0 + Ve = 7900 – 463 = 7437 m/s Which is independent of height of a satellite.
VQ = V0 + Ve = 7900 + 463 = 8363 m/s (29) (d) Due to resistance force of atmosphere, the satellite
2 revolving around the earth losses kinetic energy.
E KQ æ 8363 ö
\ =ç Therefore in a particular orbit the gravitational
E KP è 7437 ÷ø attraction of earth on satellite becomes greater than
that required for circular orbit there. Therefore satellite
2GM
(22) (b) ve = i.e. escape velocity depends upon the moves down to a lower orbit. In the lower orbit as the
R
potential energy ( U = -GMm / r ) becomes more
mass and radius of the planet.
negative, Hence kinetic energy ( Ek = GMm / 2r )
2GM 8
(23) (a) ve = = R pGr
R 3 increases, and hence speed of satellite increases.
(30) (a) Because gravitational force is always attractive in nature
If mean density is constant then ve µ R
and every body is bound by this gravitational force of
ve R e 1 vp attraction of earth.
= = Þ ve =
vp R p 2 2
t.me/Ebooks_Encyclopedia27. t.me/Magazines4all

DPP/ P 20 59

DAILY PRACTICE
PROBLEMS
PHYSICS
SOLUTIONS 20
8. (a). Limiting stress = 4.0 × 108 N/m2
MgL
1. (a). Y = F 400
pr 2 Dl = = 4.0 × 108
but Mg/pr2 = 20 × 108 & Dl = L then A A
Y = 20 × 108 N/m2 or A = 10–6 m2
2. (b). F = Y a DtA 1/2
A = 2 × 10–6 m2, Y = 2 × 11 N/m2 æ 4Aö
1/2 æ 4 ´ 10-6 ö
\ D = çè ÷ =ç ÷
a = 1.1 × 10–5, t = 50 – 30 = 20°C p ø è p ø
F = 2 × 1011 × 1.1 × 10–5 × 20 × 2 × 10–6 = 88N.
3. (d). Work done on the wire = 1.13 × 10–3 m = 1.13 mm
1 1 F 4.8 ´ 103 N
W= F × l = × stress x volume x strain 9. (c). Stress = = = 4.0 × 107 N/m2
2 2 A 1.2 ´ 10-4 m 2
1
W= × Y × strain2 × volume Dl 1 ´10-3
2 10. (c). Strain = = = 5 × 10–4, longitudinal
l 2
1 Dl 2 YADL2 11. (c). F = Y A a Dt
W = × Y × 2 × AL =
2 L 2L = 2 × 1011 × 3 × 10–6 × 10-5 × 20
F = 120 N.
2 ´ 1011 ´ 10-6 ´ 10-6 12. (c). Compressibility
W= = 0.1 J
2 ´1 1 DV
c= =- = 5 × 10–10
1 K VDp
4. (d). W = × load × elongation
2 \ Fractional decrease in volume
1 DV
W= × 5.4 × 106 × 3 =– = c D p = 5 × 10–10 × 15 × 106
2 V
W = 8.1 × 106 ergs = 7.5 × 10–3
5. (d). By Hook's law 13. (c). Increase in length on heating Dl = a L DT
F / A FL To annul this increase if pressure applied is p then
Y= =
l / L Al Dl
p=Y = Ya DT
L
16 ´ 1
Y= = 2 × 1011 N/m2 = 2 × 1011 × 1.1 × 10–5 × 100 = 2.2 × 108 N/m2
(4 ´ 10 ) (0.2 ´10 -2 )
-8
14. (c). y = 2h (1 + s)
DPV y = 2.4 × h
6. (a). B = – 2.4 h = 2h (1 + s)
DV
(1 + s) = 1.2
Given, DP = hdg = 200 x 103 x 10
DP = 2 × 105 N/m2 s = 0.2
15. (c). Stress = F/A = 10/(2 × 10-6) = 5 × 106 N/m2
DV 0.1
= = 10–3
V 100 Stress 5 ´ 106
Strain = = = 2.5 × 10–5
Y 2 ´ 1011
2 ´ 106
\B= = 2 × 109 N/m2 l = L × strain = 1 × 2.5 × 10–5
10-3 l = 2.5 × 10–5 m
stress F / A F L Dl Mg 1000 ´ 980 ´ 100
7. (b). Y = = =
strain l / L Al 16. (b). L = AY =
1012 ´ 0.01
FL 20 ´ 9.8 ´ 4 Dl = 0.0098 cm.
\l= =
A Y p ´ (10 -3 )2 ´ 1.96 ´ 1011 17. (a). Volume = Mass/density
= 1.27 × 10–3 m = 1.27 mm Area of cross-section = volume/length
t.me/Ebooks_Encyclopedia27. t.me/Magazines4all

EBD_7156
60 DPP/ P 20
21. (c) F = YAaDq \ F µ A
mass 15.6 ´ 10-3
= = = 8 × 10–7 m2 1
density ´ length 7800 ´ 2.5 22. (a) For twisting, angle of shear f µ
L
Fl 8 ´ 9.8 ´ 2.5 i.e. if L is more then f will be small.
Y= =
ADL (8 ´ 10 -7 ) ´ 1.25 ´ 10-3 23. (a) Y = 2h (1 + s )
Y = 1.96 × 1011 N/m2
0.5Y - h
18. (c). Potential energy per unit volume = u 24. (a) Y = 2h (1 + s ) Þ s =
h
1
= × stress × strain
2 Fcos q Fcos2 q
25. (a) Tensile stress = =
stress a / cos q a
But Y =
strain 26. (a) Tensile stress is maximum when cos 2 q is maximum,
\ stress = Y x strain = Y x S
i.e., q = 0°
\ Potential energy per unit volume = u
Fsin q Fsin q cos q
1 1 27. (b) Shearing stress = =
= × (YS)S = YS2 a / cos q a
2 2
Fsin 2q
l1 L1 r22 =
2a
19. (d). l =
2 L 2 r12 28. (a) Elasticity is a measure of tendency of the body to regain
L1 = L, L2 = 2L, r1 = 2R., r2 = R its original configuration. As steel is deformed less than
rubber therefore steel is more elastic than rubber.
l1 L R2 1 29. (a) Bulk modulus of elasticity measures how good the body
\ = . 2 =
l 2 2L 4R 8 is to regain its original volume on being compressed.
Therefore, it represents incompressibility of the material.
Force F - PV
20. (c). stress = = 2 K=
Area pr where P is increase in pressure, DV is change
DV
1 in volume.
\ stress S µ 2 30. (a) A bridge during its use undergoes alternating strains
r for a large number of times each day, depending upon the
2 movement of vehicles on it when a bridge is used for long
æ S1 ö æ r2 ö time, it losses its elastic strength. Due to which the amount
\ ç ÷ =ç ÷
è S2 ø è r1 ø of strain in the bridge for a given stress will become large
and ultimately, the bridge may collapse. This may not
r1 2 S1 1 happen, if the bridges are declared unsafe after long use.
Given r = 1 \ =
S2 4
2
t.me/Ebooks_Encyclopedia27. t.me/Magazines4all

DPP/ P 21 61

DAILY PRACTICE
PROBLEMS
PHYSICS
SOLUTIONS 21
1. (a) Force required to separate the plates Let the width of each plate is b and due to surface
tension liquid will rise upto height h then upward force
2TA 2 ´ 70 ´ 10-3 ´ 10 -2
F= = = 28 N due to Surface tension
t 0.05 ´ 10 -3 .......(i)
= 2Tb cos q
2. (c) T T Weight of the liquid rises in between the plates
q q = Vdg = (bxh)dg ........(ii)
Equating (i) and (ii) we get, 2T cos q = bxhdg

2T cos q
\h =
xdg
10. (b) Mass of liquid in capillary tube
æ 1ö
Weight of metal disc = total upward force M = pR 2 H ´ r\ M µ R 2 ´ ç ÷ (As H µ1/ R)
è Rø
= upthrust force + force due to surface tension
= weight of displaced water + T cos q (2pr ) \ M µ R . If radius becomes double then mass will
becomes twice.
= W + 2prT cos q
11. (d) In the satellite, the weight of the liquid column is zero.
æ D2 d 2 ö So the liquid will rise up to the top of the tube.
3. (d) W = T ´ 8p(r22 - r12 ) = T ´ 8p ç - ÷
ç 4 4 ÷ø 12. (b) Tc = T0 (1- µ t )
è
i.e. surface tension decreases with increase in
= 2p( D 2 - d 2 )T temperature.
4. (b) Increment in area of soap film = A2 - A1 13. (a)
14. (d) Tension in spring T = upthrust - weight of sphere
= 2 ´ [(10 ´ 0.6) - (10 ´ 0.5)] ´10 -4 = 2 ´10 -4 m 2 = V sg - V rg = V hrg - V rg (As s = hr)
Work done = T ´ DA
= (h - 1)V rg = (h - 1) mg.
= 7.2 ´ 10-2 ´ 2 ´ 10-4 = 1.44 ´10 -5 J 15. (b)
5. (a) 16. (c)
6. (c) Excess pressure inside soap bubble is inversely proportional 17. (c) A stream lined body has less resistance due to air.
1 18. (a) Weight of cylinder = upthrust due to both liquids
to the radius of bubble i.e. DP µ
r æA 3 ö æ A Lö
This means that bubbles A and C posses greater V ´ D ´ g = ç ´ L÷ ´ d ´ g + ç ´ ÷ ´ 2d ´ g
è5 4 ø è 5 4ø
pressure inside it than B. So the air will move from A
and C towards B. æA ö A´ L´d ´ g
Þ ç ´ L÷ ´ D ´ g =
rr 5´ 4 è5 ø 4
7. (b) r = 1 2 = = 20 cm
r1 - r2 5 - 4
D d 5
Þ = \D = d
8. (c) The radius of resultant bubble is given by R 2 = r12 + r2 2 . 5 4 4

9. (b) x 19. (b) Velocity of efflux when the hole is at depth h, v = 2 gh


b
Rate of flow of water from square hole
Q1 = a1v1 = L2 2 gy
Rate of flow of water from circular hole
h
Q2 = a2 v2 = pR 2 2 g (4 y )
According to problem Q1 = Q2
L
Þ L2 2 gy = pR 2 2 g (4 y) Þ R =
2p
t.me/Ebooks_Encyclopedia27. t.me/Magazines4all

EBD_7156
62 DPP/ P 21
( )
20. (a) Let A = cross-section of tank o
23. (d) At critical temperature Tc = 370 C = 643 K , the
A surface tension of water is zero.
24. (d)
25. (b)
PBottom > PSurface . So bubble rises upward.
3m B
v At constant temperature V µ 1 ( Boyle 's law )

52.5 cm
P
Since as the bubble rises upward, pressure decreases, then
from above law volume of bubble will increase i.e. its size
increases.
26. (a)
a = cross-section hole
27. (d).
V = velocity with which level decreases
F = Patm × Area = 105 × 1 × 10–6 = 0.1 N
v = velocity of efflux
av æ 2T ö
F = ç Patm + ´ A = 0.10023 N
From equation of continuity av = AV Þ V =
A è r ÷ø
By using Bernoulli’s theorem for energy per unit volume 28. (a) Since the excess pressure due to surface tension is
Energy per unit volume at point A inversely proportional to its radius, it follows that
= Energy per unit volume at point B smaller the bubble, greater is the excess pressure. Thus
when the larger and the smaller bubbles are put in
1 1
P + rgh + rV 2 = P + 0 + rv 2 communication, air starts passing from the smaller into
2 2 the large bubble because excess pressure inside the
former is greater than inside the latter. As a result, the
2 gh 2 ´ 10 ´ (3 - 0.525)
Þ v2 = = = 50( m / sec) 2 smaller bubble shrinks and the larger one swells.
2 2
æaö 1 - (0.1) 29. (b) Statement-1 is True, Statement-2 is True; Statement-2
1- ç ÷
è Aø is NOT a correct explanation for Statement-1.
30. (a) In a stream line flow of a liquid, according to equation
21. (c) If the liquid is incompressible then mass of liquid
of continuity,
entering through left end, should be equal to mass of liquid
av = constant
coming out from the right end.
Where a is the area of cross-section and v is the velocity
\ M = m1 + m 2 Þ Av1 = Av2 + 1.5A.v of liquid flow. When water flowing in a broader pipe
Þ A ´ 3 = A ´ 1.5 + 1.5A.v Þ v = 1 m / s enters a narrow pipe, the area of cross-section of water
decreases therefore the velocity of water increases.
F 6.28 ´ 10 -4
22. (b) T= = = 5 ´ 10 -2 N / m
2 pr 2 ´ 3.14 ´ 2 ´ 10 -3
t.me/Ebooks_Encyclopedia27. t.me/Magazines4all

DPP/ P 22 63

DAILY PRACTICE
PROBLEMS
PHYSICS
SOLUTIONS 22
1. (c) Due to volume expansion of both mercury and flask, é 75 45 ù
-6 -6
the change in volume of mercury relative to flask is = x ê ´ 10 ´ 100 + (20 - x) ´ ´ 10 ´ 100 ú
ë2 2 û
given by DV = V0 [ g L - g g ]Dq = V [ g m - 3a g ]Dq
On solving we get x = 10 cm.
= 50[180 × 10–6 – 3 × 9 × 10–6] (38 – 18) 5. (b) Due to volume expansion of both liquid and vessel,
= 0.153 cc the change in volume of liquid relative to container is
2. (a) greal = gapp. + gvessel given by DV = V0 [ g L - g g ]Dq
So (gapp. + gvessel)glass= (gapp. + gvessel)steel
Given V0 = 1000 cc, ag = 0.1× 10–4/°C
Þ 153 × 10–6 + (gvessel)glass= (144 × 10–6 + gvessel)steel
Further, (gvessel)steel = 3a = 3 × (12 × 10–6) = 36 × 10–6/°C \ g g = 3a g = 3 ´ 0.1 ´ 10-4 / °C = 0.3 ´ 10-4 / °C
Þ 153 × 10–6 + (gvessel)glass = 144 × 10–6 + 36 × 10–6 \ DV = 1000[1.82 ´ 10-4 - 0.3 ´ 10-4 ] ´ 100 = 15.2cc
Þ (gvessel)glass = 3a = 27 × 10–6/°C 6. (b) g r = g a + g v ; where g r = coefficient of real expansion,
Þ a = 9 × 10–6/°C g a = coefficient of apparent expansion and
3. (c) Initial diameter of tyre = (100 – 6) mm = 994 mm, g v = coefficient of expansion of vessel.
994 For copper g r = C + 3a Cu = C + 3A
= 497mm
So, initial radius of tyre R =
2 For silver g r = S + 3a Ag
and chan ge in diameter DD = 6 mm, so
C - S + 3A
6 Þ C + 3A = S + 3a Ag Þ a Ag =
DR = = 3 mm 3
2
After increasing temperature by Dq, tyre will fit onto 7. (d) V = V0 (1 + gDq) Þ Change in volume
wheel V – V0 = DV = A.Dl = V0gDq
Increment in the length (circumference) of the iron tyre
V0 .Dq 10-6 ´ 18 ´ 10-5 ´ (100 - 0)
g g Þ Dl = =
DL = L ´ a ´ Dq = L ´ ´ Dq [As a ´ ] A 0.004 ´ 10-4
3 3
= 45 × 10–3m = 4.5 cm
ægö 8. (b) Loss of weight at 27°C is
2pDR = 2pR ç ÷ Dq = 46 – 30 = 16 = V1 × 1.24 r1 × g ...(i)
è3ø
Loss of weight at 42°C is
3 DR 3´ 3 = 46 – 30.5 = 15.5 = V2 × 1.2 r1 × g ...(ii)
Dq Þ =
g R 3.6 ´ 10-5 ´ 497 Now dividing (i) by (ii), we get
Þ Dq = 500°C 16 V 1.24
= 1´
4. (b) DL = L0 aDq 15.5 V2 1.2
Rod A : 0.075 = 20 × aA × 100 V2 15.5 ´ 1.24
But = 1 + 3a(t2 - t1 ) = = 1.001042
75 V1 16 ´ 1.2
Þ aA = ´ 10-6 / °C
2 Þ 3a (42° – 27°) = 0.001042
Rod B : 0.045 = 20 × aB × 100 Þ a = 2.316 × 10–5/°C
45 mL
Þ aB = ´ 10-6 / °C 9. (b) Heat lost in t sec = mL or heat lost per sec = . This
2 t
For composite rod: x cm of A and (20 – x) cm of B we must be the heat supplied for keeping the substance in
have molten state per sec.
x (20 – x) mL Pt
\ = P or L =
t m
10. (b) Initially ice will absorb heat to raise it's temperature to
aA A B aB 0°C then it's melting takes place
20 cm If m1 = Initial mass of ice, m1' = Mass of ice that melts
0.060 = x aA × 100 + (20 – x) aB × 100 and mW = Initial mass of water
t.me/Ebooks_Encyclopedia27. t.me/Magazines4all

EBD_7156
64 DPP/ P 22
By Law of mixture 17. (b) Suppose m kg of ice melts then by using
Heat gained by ice = Heat lost by water Þ m1 × (20) + W = H
m1' × L = mWcW [20] ( Joules ) ( Joules )
Þ 2 × 0.5 (20) + m1' × 80 = 5 × 1 × 20 Þ Mgh = mL
Þ m1' = 1 kg Þ 3.5 × 10 × 2000 = m × 3.5 × 105
So final mass of water = Initial mass of water + Mass Þ m = 0.2 kg = 200 gm
of ice that melts = 5 + 1 = 6 kg. 18. (d) Coefficient of volume expansion
11. (a) If mass of the bullet is m gm,
Dr (r - r ) (10 - 9.7)
then total heat required for bullet to just melt down g= = 1 2 = = 3 ´ 10-4
Q1 = mcDq + mL = m × 0.03 (327 – 27) + m ×6 r.DT r.( Dq) 10 ´ (100 - 0)
= 15m cal = (15m × 4.2)J Hence, coefficient of linear expansion
Now when bullet is stopped by the obstacle, the loss in
g
1 a= = 10-4 / °C
-3 2 3
its mechanical energy = (m ´ 10 )v J
2 19. (b) As we know
(As m gm = m × 10–3kg) g real = g app. + g vessel
As 25% of this energy is absorbed by the obstacle,
Þ g app. = g glycerine – g glass
75 1 2 3
Q2 = ´ mv ´ 10-3 = mv 2 ´ 10-3 J = 0.000597 – 0.000027
100 2 8
= 0.00057/°C
Now the bullet will melt if Q2 ³ Q1
( Pt - P0 )
3 2 20. (a) t= ´ 100°C
i.e. mv ´ 10-3 ³ 15m ´ 4.2 Þ vmin = 410 m/s ( P100 - P0 )
8
12. (c) Heat gain = heat lost (60 - 50)
= ´ 100 = 25°C
CA 3 (90 - 50)
C A (16 - 12) = CB (19 - 16) Þ = 21. (c) Since specific heat = 0.6 kcal/gm × °C
CB 4
= 0.6 cal/gm × °C
CB 5 From graph it is clear that in a minute, the temperature
and CB (23 - 19) = CC (28 - 23) Þ = is raised from 0°C to 50°C.
CC 4
Þ Heat required for a minute
C A 15 = 50 × 0.6 × 50 = 1500 cal.
Þ = ....(i) Also from graph, boiling point of wax is 200°C.
CC 16
22. (b) The horizontal parts of the curve, where the system
If q is the temperature when A and C are mixed then, absorbs heat at constant temperature must depict
C A 28 - q changes of state. Here the latent heats are proportional
C A (q - 12) = CC (28 - q) Þ = ...(ii) to lengths of the horizontal parts. In the sloping parts,
CC q - 12
specific heat capacity is inversely proportional to the
On solving equation (i) and (ii) q = 20.2°C slopes.
13. (a) Same amount of heat is supplied to copper and water 23. (d) Let L0 be the initial length of each strip before heating.
so mcccDqc = mWcWDqW Length after heating will be
mc cc (Dq)c 50 ´ 10-3 ´ 420 ´ 10
Þ DqW = = = 5°C Brass Strip
mW cW 10 ´ 10 -3 ´ 4200
14. (b) Heat lost by hot water = Heat gained by cold water in
beaker + Heat absorbed by beaker
Þ 440 (92 – q) = 200 × (q – 20) + 20 × (q – 20)
d
Þ q = 68°C Copper Strip
15. (a)
16. (b) Firstly the temperature of bullet rises up to melting
point, then it melts. Hence according to W = JQ.
R q
1 2
Þ mv = J .[m.c.Dq + mL] = J [mS (475 - 25) + mL]
2

mv 2
Þ mS (475 - 25) + mL =
2J
LB = L0 (1 + α BΔT ) = ( R + d ) θ
t.me/Ebooks_Encyclopedia27. t.me/Magazines4all

DPP/ P 22 65

LC = L0 (1 + αC ΔT ) = Rθ r1 1500
r2 = = = 1350 kg/m3
(1 + gDT ) æ -3 1000 ö
R + d 1 + αB ΔT çè1 + 1 ´ 10 ´ ÷
Þ = 9 ø
R 1 + αC ΔT

d r 2 v 2 A2 = r1v1 A1
Þ 1+ = 1 + ( α B - α C ) ΔT
R Þ 1350 × v2 = 1500 × 0.1
v2 = 1/9 m/s
d
Þ R= \ Volume rate of flow at the end of tube
( α B - αC ) ΔT
1
= A2v2 = 4 × 10–4 ´
1 1 9
ÞRµ and R µ
ΔT ( B αC )
α -
4 40
24. (a) A bimetallic strip on being heated bends in the form of = ´ 10-4 m3 = ´ 10-5 m3
9 9
an arc with more expandable metal (A) outside (as
shown) correct. Volume rate of flow at the entrance = A1v1
= 0.1 × 4 × 10–4 = 4 × 10–5m3
Hence, difference of volume rate of flow at the two
ends
aA > aB æ 40 ö -5 4
B = ç - 4÷ ´ 10 = ´ 10-5 m3
B è 9 ø 9
A A aB
28. (d) Celsius scale was the first temperature scale and
Fahrenheit is the smallest unit measuring temperature.
aA aB aA 29. (a) Linear expansion for brass (19 × 10–4) > linear expansion
for steel (11 × 10–4). On cooling the disk shrinks to a
greater extent than the hole and hence it will get loose.
25. (a) 26. (c) 27. (c) 30. (b) The latent heat of fusion of ice is amount of heat
r1v1A1 = r2v2A2 required to convert unit mass of ice at 0°C into water
m = 1500 kg/m3 × 0.1 m/s × 4 (cm)2 at 0°C. For fusion of ice
msDT = 10000 L = 80 cal/gm = 80000 cal/gm = 8000 × 4.2 j/kg
1500 × 0.1 × 4 × 10–4 × 1500 × DT = 10000 = 336000 J/kg.

10000 1000
DT = = °C
90 9
t.me/Ebooks_Encyclopedia27. t.me/Magazines4all

EBD_7156
66 DPP/ P 23
DAILY PRACTICE
PROBLEMS
PHYSICS
SOLUTIONS 23
dQ KADq 0.2 ´ 1 ´ (20 - q)t
1. (a) = H=
dt l 0.002
For both rods K, A and Dq are same
(200)2 ´ t 0.2 ´ (20 - q)t
dQ 1 Hence =
Þ µ 20 ´ 4.2 0.002
dt l q = 15.24° C
Þ
( dQ / dt ) semi circular 4. (b) If suppose KNi = K Þ KAl = 3K and KCu = 6K
So, Since all metal bars are connected in series.
( dQ / dt ) straight
æQ ö æQ ö æQ ö æQö
So, ç ÷ =ç ÷ =ç ÷ =ç ÷
l straight è t øCombination è t øCu è t ø Al è t ø Ni
=
l semi circular
3 1 1 1
and = + +
2r 2 K eq KCu K Al K Ni
=
= .
pr p
1 1 1 9
2. (b) Suppose thickness of each wall is x then = + + =
6 K 3K K 6K
æQö æQ ö
çè ÷ø =ç ÷ Þ Keq = 2K
t combination è t ø A
25 cm 10 cm 15 cm
K s A(q1 - q2 ) 2KA(q1 - q) Q Cu Ni Al Q
Þ =
2x x 100° C q1 q2 0° C

2 ´ 2K ´ K 4 æQ ö æQö
Q Ks = = K Hence, if ç ÷ =
(2K + K ) 3 è t øCombination çè t ÷øCu
and (q1 - q2 ) = 36°
K eq A(100 - 0) KCu A(100 - q1 )
Þ =
l Combination l Cu

2 KA(100 - 0) 6KA(100 - q1 )
Þ =
(25 + 10 + 15) 25
A B Þ q1 = 83.33° C
æQö æQ ö
Similar if ç ÷ =
è t ø Combination çè t ÷ø Al
2K K
2 KA(100 - 0) 3KA(q2 - 0)
Þ =
q1 x q x q2 50 15
Þ q2 = 20° C
4 5. (b) Let the temperature of junction be q then according to
KA ´ 36 2KA(q - q) following figure.
Þ 3 = 1
2x x 50°C
Hence temperature difference across wall A is
2K
(q1 - q) = 12° C H1
100°C H
3. (a) Heat developed by the heater
3K q H2
V 2 t (200)2 ´ t
H= . = K
R J 20 ´ 4.2
Heat conducted by the glass 20°C
t.me/Ebooks_Encyclopedia27. t.me/Magazines4all

DPP/ P 23 67

H = H1 + H 2 1
On solving equation (i) and (ii) we get K = and q0
Þ 15
3K ´ A ´ (100 - q) 2 KA(q - 50) KA(q - 20) = 24° C. Putting these values in equation (iii) we get q
= + = 42.7° C.
l l l
Þ 300 – 3q = 3q – 120 Ql
8. (c) t=
Þ q = 70° C KA(q1 - q2 )
6. (d) Wein's displacement law is
mLl V rLl
l mT = b = =
KA(q1 - q 2 ) KA(q1 - q 2 )
b
Þ lm = 5 + 10
T 5 ´ A ´ 0.92 ´ 80 ´
= 2
2.88 ´ 106 0.004 ´ A ´ 10 ´ 3600
= = 1000 nm
2880 = 19.1 hours.
Energy distribution with wavelength will be as follows Q KADq
9. (c) =
t l
El mL K (pr 2 )Dq
Þ =
t l
2
U2 æ m ö Kr
Þ Rate of melting of ice ç ÷ µ
èt ø l
U1
U3 1
Since for second rod K becomes th, r becomes
4
500
499

900
1000

1499
1500

l(nm) double and length becomes half, so rate of melting will

be twice i.e. æç ö÷ = 2 æç ö÷ = 2 ´ 0.1 = 0.2 gm/sec.


m m
From the graph it is clear that è t ø2 è t ø1
U2 > U1
7. (a) According to Newton law of cooling dQ KA
10. (d) = dq
q1 - q2 éq + q ù dt l
= K ê 1 2 - q0 ú
t ë 2 û 0.01 ´ 1
= ´ 30 = 6 J/sec
0.05
5 min Heat transferred in one day (86400 sec)
80° C 64°C
1 Q = 6 × 86400 = 518400 J
10 min Now Q = mL
52°C
2 Q 518400
15 min Þ m= = = 1.552 kg = 1552 g.
L 334 ´ 103
q=?
3 dT sA 4
11. (b) = (T - T04 )
For first process : dt mcJ
(80 - 64) é 80 + 64 ù [In the given problem fall in temperature of body dT =
=Kê - q0 ú ....... (i) (200 – 100) = 100 K, temp. of surrounding T0 = 0 K,
5 ë 2 û
Initial temperature of body T = 200 K].
For second process :
(80 - 52) 100 s 4 pr 2
é 80 + 52 ù = (2004 - 0 4 )
=Kê - q0 ú ....... (ii) dt 4 3
10 ë 2 û pr rcJ
3
For third process :
rrcJ
(80 - q) é 80 + q ù Þ dt = ´ 10-6 s
=Kê - q0 ú ....... (iii) 48s
15 ë 2 û
t.me/Ebooks_Encyclopedia27. t.me/Magazines4all

EBD_7156
68 DPP/ P 23
rrc 4.2 50 - q æ 50 + q ö
= ´ ´ 10-6 =Kç - 25 ÷ .......... (ii)
s 48 10 è 2 ø
7 r rc 7 rrc On dividing, we get
= µs ; µs [As J = 4.2]
80 s 72 s 10 60
=
12. (c) Suppose temperature difference between A and B is 50 - q q
100° C and q A > qB Þ q = 42.85° C
d q sA 4
C
15. (d) = (T - T04 ). If the liquids put in exactly similar
dt mc
calorimeters and identical surrounding then we can
H/2 H/2 consider T0 and A constant then

A B d q (T 4 - T04 )
µ
H H dt mc
If we consider that equal masses of liquid (m) are taken
H/2 H/2 at the same temperature then
dq 1
D
µ
dt c
So for same rate of cooling c should be equal which is
Heat current will flow from A to B via path ACB and
not possible because liquids are of different nature.
ADB. Since all the rod are identical so
Again from equation (i),
(Dq) AC = (Dq) AD
d q (T 4 - T04 )
Dq µ
(Because heat current H = ; here R = same for all) dt mc
R
d q (T 4 - T04 )
Þ q A - qC = q A - q D Þ µ
dt V rc
Þ qC = q D Now if we consider that equal volume of liquid (V) are
i.e. temperature difference between C and D is zero. taken at the same temperature then
dq 1
C µ .
dt rc
So for same rate of cooling multiplication of r × c for
R R two liquids of different nature can be possible. So,
option (d) may be correct.
A B 16. (d) For cooking utensils, low specific heat is preferred for
2T T it's material as it should need less heat to raise it's
temperature and it should have high conductivity,
because, it should transfer heat quickly.
R R
æQö K1 A1 (q1 - q2 )
17. (b) çè ÷ø =
t 1 l
D
K A (q - q )
and æç ö÷ = 2 2 1 2
Q
13. (a) Initially at t = 0 è t ø2 l
Rateof cooling (R) µ Fall in temperature of body (q – q0)
æQö æQö
R1 q1 - q0 Given, ç ÷ = ç ÷
Þ è t ø1 è t ø 2
=
R2 q 2 - q0
Þ K1 A1 = K 2 A2
100 - 40 3 18. (a) Convection may be stopped
= =
80 - 40 2 19. (d) Heated fluid becomes less dense than the cold fluid
above it
60 - 50 æ 60 + 50 ö 20. (c) According to Kirchoff's law, the ratio of emissive power
14. (c) = Kç - 25 ÷ ........... (i)
10 è 2 ø to absorptive power is same for all bodies is equal to
t.me/Ebooks_Encyclopedia27. t.me/Magazines4all

DPP/ P 23 69
the emissive power of a perfectly black body i.e., Initial temperature of calorimeter, T1 = 15°C
æeö Temperature of ball, T2 = 100°C
çè ÷ø = EBlack body for a particular wave length
a body Final temperature of the liquid, T = 23°C
Specific heat of solid, c2 = 0.092 cal/g°C
æ el ö
çè a ÷ø = ( El ) Black body Heat gained by turpentine and calorimeter is
l body
mc (T – T1) + W (T – T1) = 200c (23 – 15) + 4 (23 – 15)
Þ el = al El = (200c + 4) 8
21. (a) As for a black body rate of absorption of heat is more.
Heat lost by the ball is
Hence thermometer A shows faster rise in temperature
but finally both will acquire the atmospheric Mc2 (T2 – T) = 100 (0.092) (100 – 23)
temperature. = 708.4 cal.
22. (b) According to the principle of calorimetry
According to Stefan's law Heat gained = Heat lost
\ (200c + 4) 8 = 708.4
E = eAσT 4 Þ E1 = e1AσT14 and E 2 = e 2 AσT24
1600c + 32 = 708.4
Q E1 = E 2 \ e1T14 = e 2 T2 4
708.4 - 32
or c = = 0.42 cal/g°C
1600
1 1
æe ö4 æ 1 4 ö4 28. (d) Equivalent thermal conductivity of two equally thick
Þ T2 = ç 1 T14 ÷ = ç ´ ( 5802 ) ÷ plates in series combination is given by
è e2 ø è 81 ø

Þ TB = 1934 K

And, from Wein's law λ A ´ TA = λ B ´ TB


K1 K2
λ T λ - λ A TA - TB
Þ A = B Þ B =
λ B TA λB TA

1 5802 - 1934 3968 2 1 1


Þ = = Þ λ B = 1.5 μm = +
λB 5802 5802 K K1 K 2

23. (a) According to Newton's law of cooling. If K1 < K2

24. (a) In forced convection rate of loss of heat then K1 < K < K 2
29. (b) Both statement-1 and statement-2 are true but
Q statement-2 is not correctly explaining the statement-2.
µ A(T , T0 )
T 30. (d) According to Wein's displacement law the
25(a), 26(c), 27(c) 1
lm µ
T
Let c be the specific heat of turpentine
Hence statement-1 is true but statement-2 is false.
Mass of the solid, M = 100g
Mass of turpentine m = 200g
Water equivalent of calorimeter, W = 4g
t.me/Ebooks_Encyclopedia27. t.me/Magazines4all

EBD_7156
70 DPP/ P 24
DAILY PRACTICE
PROBLEMS
PHYSICS
SOLUTIONS 24
1. (b) Differentiate PV = constant w.r.t. V
R (Ti - T f ) R(T - T f )
DP DV 10. (d) W= Þ 6R = Þ T f = (T - 4) K
Þ P DV + V DP = 0 Þ = g -1 æ5 ö
P V çè - 1÷ø
3
2. (a) In isothermal compression, there is always an increase
of heat which must flow out the gas. 11. (c) PV g constant : Differentiating both sides
DQ = DU + DW Þ DQ = DW (\DU = 0)
dP dV
P gV g -1dV + V g dP = 0 Þ = -g
1.5 ´ 104 P V
Þ DQ = -1.5 ´ 104 J = cal = -3.6 ´ 103 cal
4.18 12. (c) V µ T at constant pressure
3. (c) DQ = DU + DW V1 T1 VT 300 ´ 280
Þ = Þ V2 = 1 2 = = 280 ml
Þ DU = DQ - DW = 2240 - 168 = 2072 J . V2 T2 T1 300
V 20 13. (b) In thermodynamic process, work done is equal to the
4. (b) Wiso = mRT loge 2 = 1´ 8.31´ 300log e = 1728 J
V1 10 area covered by the PV curve with volume axis.
Hence, according to graph shown
Tg Wadiabatic < Wisothermal < Wisobaric
5. (d) For adiabatic process = constant
Pg - 1
1-g (1-1.4) P
T æPö g T
Þ 2 = æç ö÷
4 1.4
Þ 2 =ç 1÷ Isobaric
T1 è P2 ø 300 è 1 ø

0.4
Þ T2 = 300(4) 1.4 Isothermal

6. (d) PV g = constant Adiabatic


g
P2 æ V1 ö V1 V2 V
Þ = Þ P2 = (8)5 / 3 P1 = 32 P1
P1 çè V2 ÷ø

g -1 14. (d) W = PDV = 2.4 × 104 × 1 × 105 = 24J


T2 æ V1 ö
7. (b) =
T1 çè V2 ÷ø V2 T2 274
15. (c) For isobaric process V = T Þ V2 = V ´ 273
1 1
5 2
-1
æ 27 ö 3 æ 27 ö 3 274V V
Þ T2 = 300 ç ÷ = 300 ç ÷ Increase = -V =
è 8 ø è 8 ø 273 273
2 16. (c) W = PDV = nRDT = 0.1 × 2 × 300 = 60 cal
ìïæ 27 ö1/ 3 üï æ3ö
2
17. (d) Fraction of supplied energy which increases the
= 300 íç ÷ ý = 800 ç ÷ = 675 K
internal energy is given by
îïè 8 ø þï è2ø

Þ DT = 675 - 300 = 375 K DU (DQ)V mCV DT 1


f = = = =
(DQ ) P (DQ) P mC P DT g
8. (b) In adiabatic change Q = constant Þ DQ = 0
So DW = -DU (\ DQ = DU + DW ) 7 5
For diatomic gas, g = Þ f =
5 7
9. (d) TV g -1 = constant
18. (a) In isothermal change, temperature remains constant,
g -1
æV ö Hence DU = 0.
Þ T2 = T1 ç 1 ÷ = 927o C
èV ø 2 Also from DQ = DU + DW Þ DQ = DW
t.me/Ebooks_Encyclopedia27. t.me/Magazines4all

DPP/ P 24 71

19. (c) From graph it is clear that P3 > P1 .


C
P3
P1
C A
P3 P2 B

P1
A V
E(V1) D(V2)
B
P2 24. (d) Work done = Area of ABC with V-axis
= P0(2V0 –V0) + 0 = P0V0 = nRT0 = RT0
Change in internal energy = nCVDT
E (V1 ) D (V2 ) V 3 9
= 1× R × (4T0 – T0) = RT
T0
2 2
Since area under adiabatic process (BCED) is greater
than that of isothermal process (ABDE).Therefore net 9 11
Heat absorbed = RT
T0 + RT0 = RT
T0
work done 2 2
W = Wi + (-WA ) \WA > Wi Þ W < 0
25. (b) AB is an isothermal process then
20. (b) PV 2 = constant represents adiabatic equation. So P
during the expansion of ideal gas internal energy of P × 2V = PB × 6V Þ PB =
3
gas decreases and temperature falls.
21. (a) For adiabatic process P
T1Vbg-1 = Constant A C
For bc curve P
T1Vbg-1 = T2Vcg-1
g-1
T2 æ Vb ö
or =ç ÷ ....(i)
T1 è Vc ø
B
For ad curve
V
g -1 2V 6V
T1Vag -1 = T2Vd
Now BC is an isochoric process then
g -1
T2 æ Va ö PB PC P P
or = .....(ii) = ; =
T1 çè Vd ÷ø TB TC 3T0 TC ; TC = 3T0
From equation (i) and (ii) 26. (a) Heat absorbed during BC is given by
Vb Va 3R
= Q = nCv DT = n ´ (TC - TB )
Vc Vd 2
22. (d) There is a decrease in volume during melting on an ice 3R
= n´ (2T0 ) = 3nRT0.
slab at 273K. Therefore, negative work is done by 2
ice-water system on the atmosphere or positive work 27. (b) Heat capacity is given by
is done on the ice-water system by the atmosphere. 1 dQ 1 Q
Hence option (b) is correct. Secondly heat is absorbed C= ; C=
n dT n 2T0
during melting (i.e. DQ is positive) and as we have
seen, work done by ice-water system is negative (DW 28. (c) As isothermal processes are very slow and so the
is negative). Therefore, from first law of different isothermal curves have different slopes so
they cannot intersect each other.
thermodynamics ΔU = ΔQ - ΔW 29. (d) Adiabatic compression is a rapid action and both the
Change in internal energy of ice-water system, DU will internal energy and the temperature increases.
be positive or internal energy will increase.
Q
23. (a) From graph it is clear that P3 > P1. 30. (a) c= ; a gas may be heated by putting pressure,
m.Dq
Since area under adiabatic process (BCED) is greater so it can have values for 0 to ¥ .
than that of isothermal process (ABDE). Therefore net CP and CV are it’s two principle specific heats, out of
work done infinite possible values.
W = Wi + ( - WA ) Q WA > Wi Þ W < 0 In adiabatic process C = 0 and in isothermal process
C= ¥.
t.me/Ebooks_Encyclopedia27. t.me/Magazines4all

EBD_7156
72 DPP/ P 25
DAILY PRACTICE
PROBLEMS
PHYSICS
SOLUTIONS 25
1. (c) Processes A to B and C to D are parts of straight line 4. (d) In all given cases, process is cyclic and in cyclic process
graphs of the form y = mx DU = 0 .
5 2
P g-1
æ L A ö3
-1
æ L ö3
TC = 2200 K T æV ö
P2 TB = 800 K 5. (d) T1V1g-1 = T2V2g-1 Þ 1 =ç 2 ÷ =ç 2 ÷ =ç 2 ÷
P2 B C T2 è V1 ø è L1 A ø è L1 ø
VB VC 6. (d) Oxygen is diatomic gas, hence its energy of two moles
TA = 600K 5
P1
A VA D TD = 1200 K = 2 ´ RT = 5RT
2
VD
Argon is a monoatomic gas, hence its internal energy
3
of 4 moles = 4 ´ RT = 6 RT
2
T
Total internal energy = (6 + 5) RT = 11RT
mR T1 - T2 WA T -T W
Also P = T (m = 6) 7. (c) hA = = Þ hB = 2 3 = B
V T1 Q1 T2 Q2
Þ P µ T . So volume remains constant for the graphs
AB and CD. Q1 T1 T2 - T3 T1
\ = ´ = \W A = WB
So no work is done during processes for A to B and C Q2 T2 T1 - T2 T2
to D i.e., WAB = WCD = 0 and
T1 + T3 800 + 300
WBC = P2 (VC - VB ) = mR (TC - TB ) \ T2 = = = 550 K
2 2
= 6 R (2200 - 800) = 6 R ´1400 J T1 - T2 T - (273 + 7)
8. (d) Initially h = Þ 0.5 = 1
Also WDA = P1 (V A - VD ) = mR(TA - TB ) T1 T1
= 6 R (600 - 1200) = -6 R ´ 600 J 1 T1 - 280
Hence work done in complete cycle Þ = Þ T1 = 560 K
2 T1
W = WAB + WBC + WCD + WDA
= 0 + 6 R ´1400 + 0 - 6 R ´ 600 , T1¢ - T2
Finally h1 =
= 6 R ´ 900 = 6 ´ 8.3 ´ 800 = 40 kJ T1¢
2. (d) W = Area bonded by the indicator diagram with V-axis)
T ¢ - (273 + 7)
1 Þ 0.7 = 1 Þ T1¢ = 933K
= ( PA + PB )(VB - VA ) T1¢
2
3. (d) For path ab : (DU ) ab = 7000 J \ increase in temperature = 933 - 560 = 373K » 380 K
9. (d) In both cylinders A and B the gases are diatomic
By using DU = mCV DT
( g = 1.4) . Piston A is free to move i.e. it is isobaric
5
7000 = m ´ R ´ 700 Þ m = 0.48 process. Piston B is fixed i.e. it is isochoric process. If
2
same amount of heat DQ is given to both then
For path ca :
(DQ )ca = (DU )ca + (DW )ca .......(i) (DQ)isobaric = (DQ)isochoric

\ (DU ) ab + (DU )bc + (DU )ca = 0 Þ m CP (DT ) A = m Cu (DT ) B

\ 7000 + 0 + (DU )ca = 0 Þ (DU )ca = -7000 J .... (ii) CP


Þ ( DT ) B = (DT ) A = g (DT ) A = 1.4 ´ 30 = 42 K
Cu
Also (DW )ca = P1 (V1 - V2 ) = mR (T1 - T2 ) T1 - T2
10. (b) In first case, h1 =
= 0.48 ´ 8.31´ (300 - 1000) = -2792.16 J ......(iii) T1
on solving equations (i), (ii) and (iii) 2T1 - 2T2 T1 - T2
(DQ )ca = -7000 - 2792.16 = -9792.16 J = -9800 J In second case, h2 = = =h
2T1 T1
t.me/Ebooks_Encyclopedia27. t.me/Magazines4all

DPP/ P 25 P
73

T2 1 500 500 1 C
11. (b) h = 1- Þ = 1- Þ = ...(i)
T1 2 T1 T1 2
2T0

60 T2' T2' 2 B
= 1- Þ = ...(ii) D
100 T1 T1 5
T0
Dividing equation (i) by (ii), A
500 5
= Þ T2, = 400 K V0 2V0
V
T2, 4
æ V0 ö
T2 300 1 Here W < 0 Þ Q < 0 and | W | = RT0 ln ç 2V ÷ + 2RT
T0
è 0ø
12. (a) hmax = 1 - =1- = = 25%
T1 400 4
æ 2V0 ö
So 26% efficiency is impossible.
ln ç V ÷ = RT
T0 ln 2
T2 (273 + 123) 150 1 è 0 ø
13. (a) h =1- =1- =1- = = 50% (25) (c) (26) (b) (27) (b)
T1 (273 + 27) 300 2
T2 25 300 1 300 P
14. (c) h =1- Þ =1- Þ = 1-
T1 100 T1 4 T1
c
o
T1 = 400 K = 127 C
15. (b) DQ = DU + DW
b
Þ DU = DQ - DW = Q - W (using proper sign) d
16. (b) (i)
17. (a) J DQ = DU + DW , DU = J DQ - DW a
DU = 4.18 ´ 300 - 600 = 654 Joule
V
18. (b) DQ = DU + DW
Þ DW = DQ - DU = 110 - 40 = 70 J Anticlockwise cycle Þ W < 0
19. (a) (ii) Process ab : Wab = 0, DUab > 0 Þ Qab > 0
20. (a) From FLOT Process bc : Wbc < 0, DUbc = 0 Þ Qbc < 0
Process cd : Wcd = 0, DUcd < 0 Þ Qcd < 0
Þ dU = dQ - dW Þ dU = dQ(< 0) (\ dW = 0) Process da : Wda > 0, DUda = 0 Þ Qda > 0
Þ dU < 0 So, temperature will decrease. (iii) da and bc are isothermal process.
21. (b) From FLOT DQ = DU + DW 28. (a) Second law of thermodynamics can be explained with
Work done at constant pressure the help of example of refrigerator, as we know that
refrigerator, the working substance extracts heat from
(DW ) P = (DQ) P - DU
colder body and rejcts a large amount of heat to a hotter
(DQ ) P - (DQ)V (As we know (DQ )V = DU ) body with the help of an external agency i.e., the electric
Also (DQ ) P =
mcP DT and (DQ )V =
mcV DT supply of the refrigerator. No refrigerator can ever work
without external supply of electric energy to it.
Þ ( DW ) p =
m(cP - cV )DT 29. (d) When the door of refrigerator is kept open, heat rejected
by the refrigerator to the room will be more than the
Þ (DW )P = 1´ (3.4 ´103 - 2.4 ´103)10 =
104 Cal. heat taken by the refrigerator from the room (by an
22. (a) Slow isothermal expansion or compression of an ideal amount equal to work done by the compressor).
gas is reversible process, while the other given pro- Therefore, temperature of room will increase and so it
cess are irreversible in nature. will be warmed gradually. As according to second law
dQ of thermodynamics, heat cannot be transferred on its
23. (a) For a reversible process ò T
=0 own, from a body at lower temperature to another at
24. (d) higher temperature.
30. (c) As there is no change in internal energy of the system
during an isothermal change. Hence, the energy taken
by the gas is utilised by doing work against external
pressure. According to FLOT
DQ = DU + pDV
Hence, DQ = DU = pDV
Therefore, statement-2 is true and statement-1 is false.
t.me/Ebooks_Encyclopedia27. t.me/Magazines4all

EBD_7156
74 DPP/ P 26
DAILY PRACTICE
PROBLEMS
PHYSICS
SOLUTIONS 26
1. (a) Closed vessel i.e., volume is constant Þ< CH >
P T P T = 2 < CHe >
Þ 1 = 1Þ = Þ T = 250 K
P2 T2 æ 0.4 ö T +1
P+ç ÷ P 12. (b) vrms > vau > vmp
è 100 ø

V1 T1 V (273 + 27) 300 5 7


2. (c) V µT Þ = Þ = = 1´ 1´
V2 T2 2V T2 T2 3 + 5
m1g1 m2 g 2 æ 5 ö æ7 ö
+ -1 -1
Þ T2 = 600 K = 327o C g1 - 1 g 2 - 1 çè 3 ÷ø çè 5 ÷ø 3
13. (a) g mix = = = = 1.5
3. (c) At low pressure and high temperature real gases m1 m 1 1 2
+ 2 +
behaves like ideal gases. g1 - 1 g 2 - 1 æ 5 ö æ 7 ö
ç - 1÷ ç - 1 ÷
N A PAVA TB è3 ø è5 ø
4. (d) PV = NkT Þ = ´
N B PBVB TA 14. (c) We know that
N A P ´ V ´ (2T ) 4 R R
Þ = = CP - CV = ÞJ=
NB V 1 J C P - CV
2P ´ ´ T
4
cal
5. (d) PV = mrT CP - CV = 1.98 ,
gm - mol - K
Since P, V , r ® remains same
Hence J
R = 8.32
1 m T 13 (273 + 52) 325 gm - mol - K
m¥ Þ 1 = 2 Þ = =
T m2 T1 m2 (273 + 27) 300 8.32
\J = = 4.20 J / cal
Þ m2 = 12 gm 1.98
i.e., mass released =13gm - 12gm = 1gm 3P 3PV P
15. (b) vrms = = Þ vrms µ
PV PV r m m
6. (c) m1 = , m2 =
RT RT
v1 P1 m2
(m + m 2 ) RT 2 PV RT Þ = ´
P' = 1 = ´ = 2P v2 P2 m1
V RT V

3kT 1 v P0 m / 2
7. (d) vrms = = vrms µ Þ = ´ Þ P2 = 2 P0
m m 2v P2 m
8. (a) 2
16. (d) P= E
3RT 3RT 3 ´ 8.3 ´ 300 3
9. (a) vrms = Þ M = 2 \M =
M vrms (1920)2 3
17. (a) For one gm mole; average kinetic energy = RT
2
= 2 ´ 10-3 kg = 2 gm Þ Gas is hydrogen.
18. (c) Average kinetic energy µ Temperature
10. (d) r.m.s velocity does not depend upon pressure.
11. (c) Average velocity of gas molecule is E1 T1 E T
Þ = Þ = 1 Þ T2 = 2T1
E2 T2 2 E T2
8 RT 1
v au = Þ va u µ
pM M T2 = 2(273 + 20) = 586 K = 313o C

< CH > M He 4 f
Þ = = =2 19. (d) Kinetic energy per gm mole E = RT
< CHe > MH 1 2
If nothing is said about gas then we should calculate
t.me/Ebooks_Encyclopedia27. t.me/Magazines4all

DPP/ P 26 75
the translational kinetic energy i.e.,
VA μ A
= ¹ 1 i.e. VA ¹ VB
3 3 VB μ B
ETrans = RT = ´ 8.31´ (273 + 0) = 3.4 ´103 J
2 2
PA μ A
20. (b) ( D Q ) P = m CP D T Similarly if VA = VB then = ¹ 1 i.e. PA ¹ PB .
PB μ B
cal 25. (b) n1Cv (T – T0) + n2 Cv (T – 2T0) = 0
Þ 2 ´ CP ´ (35 - 30) Þ C P = 7
mole - K
3
Q CP - CV = R T = T0
2
cal PT 3
Þ CV = C P - R = 7 - 2 = 5 Pf =
i f
= P0
mole - kelvin Ti 2
\ (DQ)V = mCV DT
26. (c) DQ = n1Cv (T f – T0 )
= 2 ´ 5 ´ (35 - 30) = 50 cal
21. (a) Average kinetic energy per molecule per degree of P0V0 3 æ3 ö 3
= ´ R ´ ç T0 – T0 ÷ = P0V0
2 RT0 2 è 2 ø 8
freedom = 1 kT . Since both the gases are diatomic
2 27. (c) Let DV is change in volume in any compartment then
and at same temperature (300 K), both will have the
æV ö
same number of rotational degree of freedom i.e. two. Pf ç 0 – DV ÷
Therefore, both the gases will have the same average P0V0 è 2 ø and
n1 = =
rotational kinetic energy per molecule 2 RT0 RT f

æ 1 ö
ç = 2 ´ kT = kT ÷ . æV ö
Pf ç 0 + DV ÷
è 2 ø
2 P0V0 è 2 ø
n2 = = Þ DV = 0
E1 1 2 RT0 RT f
Thus =
E2 1
28. (b) Internal energy of an ideal gas does not depend upon
22. (a) volume of the gas, because there are no forces of
Coefficient of volume expansion at constant pressure is attration/repulsion amongest the molecular of an ideal
1 gas.
for all gases. The average transnational K.E. is same Also internal energy of an ideal gas depends on
273
temperature.
3 29. (b) Helium is a monoatomic gas, while oxygen is diatomic.
for molecule of all gases and for each molecules it is kT
2 Therefore, the heat given to helium will be totally used
up in increasing the translational kinetic energy of its
kT molecules; whereas the heat given to oxygen will be
Mean free path λ = (as P decreases, l increases)
2πd 2 P used up in increasing the translational kinetic energy
of the molecule and also in increasing the kinetic energy
3RT 2RT of rotation and vibration. Hence there will be a greater
23. (b) v rms = , vP = = 0.816 vrms
M M rise in the temperature of helium.
30. (d) Maxwell speed distribution graph is asymmetric graph,
8RT because it has a long “tail” that extends to infinity.
v= = 0.92 v rms Þ v P < v < v rms
πM Also vrms depends upon nature of the gas and it’s
temperature.
1 1 3 3
mv 2rms = m v 2P = mv2P
Further E av =
2 2 2 4 dN
24. (d) According to problem mass of gases are equal so dv

number of moles will not be equal i.e. μ A ¹ μ B

PA VA PB VB
From ideal gas equation PV = μRT Þ =
μA μB
[As temperature of the container are equal]
From this relation it is clear that if PA = PB then vmp vav vrms v
t.me/Ebooks_Encyclopedia27. t.me/Magazines4all

EBD_7156
76 DPP/ P 27
DAILY PRACTICE
PROBLEMS
PHYSICS
SOLUTIONS 27
1. (c) a = 10 2
1 æaö
mw2 y 2
2. (d) y = A sin wt =
A sin 2p A
t Þ = A sin
2pt
Þt =
T U 2 y 2 çè 2 ÷ø 1
T 2 T 12 13. (b) = = = =
E 1 2 a 4
mw2 a 2 a
2p a 2 pt 1 2 pt 2
3. (c) y = a sin t Þ = a sin Þ = sin
T 2 3 2 3 14. (c) In S.H.M., frequency of K.E. and P.E.
= 2 × (Frequency of oscillating particle)
2pt p 2p t p 1
Þ sin = sin Þ = Þ t = sec 1 2 1 2 2
3 6 3 6 4 15. (c) Kinetic energy K = mv = ma w cos 2 wt
2 2
æ pö æ pö
4. (a) x = a sin ç wt + ÷ and x ' = a cos wt = a sin ç wt + ÷ 1
è 6ø è 2ø = mw2 a 2 (1 + cos 2wt )
2
æ pö æ pö p hence kinetic energy varies periodically with double
\Df = ç wt + ÷ - ç wt + ÷ = the frequency of S.H.M. i.e. 2 w .
è 6ø è 6ø 3
16. (a) At mean position, the kinetic energy is maximum.
5. (c) v = w a 2 - y 2 Þ 10 = w a 2 - (4)2 and 1 2 2
ma w = 16
Hence
2
8 = w a2 - (5)2 On putting the values we get

2p 2p p
w = 10 Þ T = = sec
On solving, w = 2 Þ w = = 2 Þ T = p sec w 5
T
17. (d) From the given equation, w = 2pn = 4p Þ n = 2 Hz
2p 2p
6. (b) vmax = aw = a ´ = (50 ´ 10 -3 ) ´ = 0.15 m / s 18. (a) Using x = A sin wt
T 2
p
7. (d) vmax = aw and Amax = aw 2 For x = A / 2, sin wT1 = 1/ 2 Þ T1 =
6w
A 4 p
Þ w = max = = 2 rad/sec For x = A, sin w (T1 + T2 ) = 1 Þ T1 + T2 =
vmax 2 2w
8. (d) At mean position velocity is maximum p p p p
Þ T2 = - T1 = - = i.e., T1 < T2
vmax 16 2w 2w 6w 3w
i.e., vmax = wa Þ w = = =4 19. (a) Let the piston be displaced through distance x towards
a 4
left, then volume decreases, pressure increases. If DP is
\ u = w a 2 - y 2 Þ 8 3 = 4 42 - y 2 increased in pressure and DV is decreased in volume,
then considering the process to take place gradually
Þ 192 = 16(16 - y 2 ) Þ 12 = 16 - y 2 Þ y = 2cm (i.e isothermal)
h
9. (a) Maximum acceleration = aw2 = a ´ 4p 2 n 2
= 0.01´ 4 ´ (p) 2 ´ (60) 2 = 144 p2 m / sec
M
2 Amax 7.5 Gas
10. (d) Amax = a w Þ a = = = 0.61 m
w2 (3.5) 2
11. (b) Comparing given equation with standard equation, P A
p
y = a sin(wt + f), we get, a = 2cm, w = x
2
1 1 = P2V2 Þ PV = ( P + DP )(V - DV )
PV
2
æpö p2 Þ PV = PV + DPV - PDV - DPDV
\ Amax = w2 A = ç ÷ ´ 2 = cm / s 2
è2ø 2 Þ DP.V - P.DV = 0 (neglecting DP.DV )
1 P.x
12. (d) E= mw2 a 2 Þ E µ a 2 DP ( Ah ) = P ( Ax ) Þ DP =
2 h
t.me/Ebooks_Encyclopedia27. t.me/Magazines4all

DPP/ P 27 77
This excess pressure is responsible for providing the
æ pö
restoring force (F) to the piston of mass M. and y3 = a sin ç wt + ÷
è 4ø
PAx
Hence F = DP. A = On superimposing, resultant SHM will be
h
é æ pö æ pöù
PA y = a êsin ç wt - ÷ + sin wt + sin ç wt +
Comparing it with F = kx Þ k = M w = 2
ë è 4 ø è 4 ÷ø úû
h
é p ù
Þw=
PA
Þ T = 2p
Mh = a ê2 sin wt cos + sin wt ú = a [ 2 sin wt + sin wt]
ë 4 û
Mh PA
20. (b) Time taken by particle to move from x = 0 (mean = a (1 + 2) sin wt
position) to x = 4 (extreme position) Resultant amplitude = (1 + 2)a
T 1.2 Energy in SHM µ (Amplitude)2
= = = 0.3 s
4 4 E Resultant æ A ö
2
Let t be the time taken by the particle to move from \ E = ç ÷ = ( 2 + 1)2 = (3 + 2 2)
Single è aø
x = 0 to x = 2 cm
2p 1 2p Þ Eresultant = (3 + 2 2) Esingle
y = a sin wt Þ 2 = 4sin t Þ = sin t
T 2 1.2
a
p 2p
Þ = t Þ t = 0.1s . 45°
6 1.2 OR a a+a 2
º ¾¾¾¾
®
Hence time to move from x = 2 to x = 4 will be equal 45°
a
to 0.3 - 0.1 = 0.2 s
Hence total time to move from x = 2 to x = 4 and
25. (a) Acceleration µ - displacement, and direction of
back again = 2 ´ 0.2 = 0.4sec
21. (b) acceleration is always directed towards the equilibrium
position.
26. (d) 27. (b)
d2 x b
Compare given equation with 2
+ w2 x = 0 ; w2 =
dt a

a max w 2 A b
= =w =
vmax wA a
At t = 0, f = p/2
1
Force constant (k) µ b
Length of spring
x = A sin (wt + f) = A cos t
a
2 28. (b)
l
K l1 3 3
Þ = = Þ K1 = K . dx
K1 l l 2 29. (b) x = a sin ωt and v = = aω cos ωt
dt
d2 y p
22. (c) y = Kt 2 Þ 2
= a y = 2K = 2 ´ 1 = 2 m/s 2 (Q K = 1m/s 2 ) It is clear that phase difference between ‘x’ and ‘a’ is
.
dt 2
30. (a) The total energy of S.H.M. = Kinetic energy of particle
l l + potential energy of particle.
Now, T1 = 2p and T2 = 2p
g ( ay )
g + The variation of total energy of the particle in SHM
with time is shown in a graph.
T1 g + ay 6 T2 6 Energy Zero slope
Dividing, = Þ Þ 12 =
T2 g 5 T2 5 Total energy
A
23. (a) At x = 0, v = 0 and potential energy is minimum so Kinetic energy
particle will remain at rest.
24. (d) Let simple harmonic motions be represented by
Potential energy
æ pö
y1 = a sin ç wt - ÷ , y2 = a sin wt
è 4ø
T/4 2T/4 3T/4
t.me/Ebooks_Encyclopedia27. t.me/Magazines4all

EBD_7156
78 DPP/ P 28
DAILY PRACTICE
PROBLEMS
PHYSICS
SOLUTIONS 28
1. (b) When the particle of mass m at O is pushed by y in 2 p 2p
the direction of A . The spring A will compressed by Putting v = 2 gh and w = = =p
T 2
y while spring B and C will be stretched by
we get T = m( g + p 2 gh)
y ' = y cos 45o . So that the total restoring force on the
mass m along OA.

FC FB k
C B 5. (b)

O m
1
Force constant (k ) µ
FA Length of spring

2
l
k l1 3 3
Þ = = Þ k1 = k
A k1 l l 2
6. (b) Initially time period was
Fnet = FA + FB cos 45o + FC cos 45o
l
T = 2p
= ky + 2ky ' 45o = ky + 2k ( y cos 45o ) cos 45o = 2ky g
Also
When train accelerates, the effective value of g becomes
Fnet = k ' y Þ k ' y = 2ky Þ k ' = 2k
( g 2 + a 2 ) which is greater than g.
m m
T = 2p = 2p Hence, new time period, becomes less than the initial time
k' 2k period.
2. (b) When mass 700 gm is removed, the left out mass (500
+ 400) gm oscillates with a period of 3 sec
(500 + 400
\ 3 = t = 2p
k
......(i)
When 500 gm mass is also removed, the left out mass
is 400 gm.
400
\ t ' = 2p ...(ii)
k
a
3 900
Þ = Þ t ' = 2sec
t' 400
3. (a) Slope is irrelevant hence
geff g
1/ 2
æM ö
T = 2p ç ÷
è 2k ø
7. (b) In accelerated frame of reference, a fictitious force
4. (a) Tension in the string when bob passes through lowest
(pseudo force) ma acts on the bob of pendulum as
mv 2 shown in figure.
point T = mg + = mg + mvw (\ v = r w)
r
t.me/Ebooks_Encyclopedia27. t.me/Magazines4all

DPP/ P 28 79
13. (a) No momentum will be transferred because, at extreme
position the velocity of bob is zero.
14. (c) The effective acceleration in a lift descending with
g
acceleration is
3
g 2g
g eff = g - =
3 3

æ L ö æ L ö æ 3L ö
\ T = 2p ç ÷ = 2p ç ÷ = 2p ç ÷
ma a ç geff ÷ è 2g / 3 ø è 2g ø
Hence tan q = = è ø
mg g
k1k2
-1 æ
aö 15. (c) In series keq = k + k so time period
Þ q = tan ç ÷ in the backward direction. 1 2
ègø
m(k1 + k2 )
l T = 2p
8. (c) T = 2p (Independent of mass) k1k2
g
1
DT 1 Dl 1 16. (c) Spring constant (k ) µ
9. (b) T µ l Þ = = ´ 1% = 0.5% Length of the spring (l )
T 2 l 2
10. (c) If suppose bob rises up to a height h as shown then as length becomes half, k becomes twice i.e. 2k
after releasing potential energy at extreme position 17. (b) Standard equation for given condition
becomes kinetic energy of mean position 2p
x = a cos t Þ x = -0.16 cos(pt )
T
[As a = -0.16 meter, T = 2sec]
q h = l (1 – cos q )
m m
18. (d) t1 = 2p k and t2 = 2p k
l l 1 2
Equivalent spring constant for shown combination is
k1 + k 2 . So time period t is given by

m
t = 2p
k1 + k 2
1 2 By solving these equations we get
Þ mgh = mvmax Þ vmax = 2 gh
2 t -2 = t1-2 + t2-2
l -h
Also, from figure cos q = 19. (a) With mass m2 alone, the extension of the spring l is
l given as
Þ h = l (1 - cos q) m2 g = kl ........(i)
So, vmax = 2 gl (1 - cos q) With mass (m1 + m2 ) , the extension l ' is given by
11. (a) If initial length l1 = 100 then l2 = 121 (m1 + m2 ) g = k (l + Dl ) .......(ii)

l T l The increase in extension is Dl which is the amplitude


By using T = 2p Þ 1 = 1 of vibration. Subtracting (i) from (ii), we get
g T2 l2
m1 g = k Dl
T1 100 m1 g
Hence, = Þ T2 = 1.1T1 Dl =
T2 121 or
k
T2 - T1 20. (a) If y1 = a1 sin wt and a2 sin(wt + p)
% increase = ´100 = 10%
T1 y1 y1 a
12. (d) After standing centre of mass of the oscillating body Þ + Þ y2 = 2 y1
a1 a2 a1
will shift upward therefore effective length will
This is the equation of straight line.
decrease and by T µ l , time period will decrease.
t.me/Ebooks_Encyclopedia27. t.me/Magazines4all

EBD_7156
80 DPP/ P 28
21. (c) Energy of particle is maximum at resonant frequency T - Mg cos θ = Centripetal force
i.e., w2 = w0 . For amplitude resonance (amplitude
maximum) frequency of driver force Mv 2
Þ T - Mg cosθ =
L
w = w02 - b 2 2 m2 Þ w1 ¹ w0
Also tangential acceleration a T = g sin θ.
c 25. (a) Except (4) all statements are wrong.
22. (b) A = ; when b = 0, a = c, amplitude
a+b-c 26. (b) 27. (b).
A ® ¥. This corresponds to resonance. For minimum time period w2A = mg
23. (b) Let the velocity acquired by A and B be V, then
4p 2
v A = mg , T = 0.2 sec,
mv = mV + mV Þ V = T2
2 At t = 0.05 sec.
1 1 1 1 2p
Also mv 2 = mV 2 + mV 2 + kx 2 y = A sin wt = 1 sn × 0.05cm. = 1cm.
2 2 2 2 0.2
Where x is the maximum compression of the spring.
1
1/ 2 PE = mgy = 1 × 10 × = 0.1 Joule
æ mö 100
On solving the above equations, we get x = v ç ÷
è 2k ø 28. (c) Statement -1 is False, Statement-2 is True.
At maximum compression, kinetic energy of the 29. (a) The time period of a oscillating spring is given by,

1 1 mv2 m 1
A – B system = mV 2 + mV 2 = mV 2 = T = 2p
ÞT µ
2 2 4 k k
24. (d) Since the spring constant is large for hard spring,
therefore hard spring has a less periodic time as
compared to soft spring.
30. (d) Time period of simple pendulum of length l is,
L –f +f L
l
q T = 2p ÞT µ l
g
DT 1 Dl
mg sinq Þ =
T 2 l
mg mg cosq
DT 1
\ = ´ 3 = 1.5%
From following figure it is clear that T 2
t.me/Ebooks_Encyclopedia27. t.me/Magazines4all

DPP/ P 29 81

DAILY PRACTICE
PROBLEMS
PHYSICS
SOLUTIONS 29
v 330 w 2pf
1. (d) v = nl Þ l = = = 1.29 m (v ) wave = = = fl
n 256 k 2p / l
2. (a) Time required for a point to move from maximum pY0
displacement to zero displacement is \ (vmax ) particle = 4vwave = Y0 ´ 2pf = 4 f l Þ l =
2
T 1
t= = 9. (d) y = f ( x 2 - vt 2 ) doesn’t follows the standard wave
4 4n
equation.
1 1
Þn= = = 1.47 Hz æ 2px ö
4t 4 ´ 0.170 10. (a) y1 = a1 sin ç wt - ÷ and
è l ø
3. (b)
æ 2px ö æ 2px pö
y2 = a2 cos ç wt - + f ÷ = a2 sin ç wt - +f+ ÷
è l ø è l 2ø

p l æ pö
and D =
So phase difference = f + çf+ ÷
2 2 p è 2ø
11. (d) On comparing the given equation with standard
2p
2d1 + 2d 2 = v ´ t1 + v ´ t2 Þ 2(d1 + d2 ) = v (t1 + t2 ) equation y = a sin (vt - x ) . It is clear that wave
l
v (t1 + t2 ) 340 ´ (1.5 + 3.5) speed (v )wave = v and maximum particle velocity
d1 + d2 = = = 850 m
2 2
(vmax ) particle = aw = y0 ´ co-efficient of t = y0 ´ 2pv
4. (c) At given temperature and pressure l
1 v1 r2 4 a ´ 2 pv
vµ Þ = = = 2 :1 \ (vmax ) particle = 2(w ) wave Þ = 2v Þ l = py0
r v2 r1 1 l
5. (b) The distance between two points i.e. path difference 12. (a) Compare the given equation with y = a sin(wt + kx)
between them
50
l l p l v We get w = 2 pn = 100 Þ n = Hz
D= ´f= ´ = = (\ v = nl ) p
2p 2p 3 6 6n 13. (c) A wave travelling in positive x-direction may be
360 2p
ÞD= = 0.12m = 12cm represented as y = A sin (vt - x) . On putting values
6 ´ 500 l
6. (d) y1 = a sin(wt - kx ) and 2p æ x ö
y = 0.2 sin (360t - x) Þ y = 0.2 sin 2p ç 6t - ÷
æ pö 60 è 60 ø
y2 = a cos( wt - kx ) = a sin ç wt - kx + ÷
è 2ø 14. (a) Comparing the given equation with y = a sin(wt - kx)
p w
Hence phase difference between these two is . We get w = 3000p Þ n = = 1500 Hz
2 2p

I1 a12 æ 0.06 ö 4
2 2p 1
7. (c) = 2 =ç ÷ = and k = = 12p Þ l = m
I 2 a2 è 0.03 ø 1 l 6
1
8. (d) Comparing the given equation with y = a sin(wt - kx) , So, v = nl Þ v = 1500 ´ = 250 m / s
6
2p 15. (b) Given, y = 0.5sin(20 x - 400t )
We get a = Y0, w = 2pf , k = . Hence maximum
l
Comparing with y = a sin(wt - kx)
particle velocity (vmax ) particle = aw = Y0 ´ 2p f and
w 400
wave velocity Gives velocity of wave v = = = 20 m / s.
k 20
t.me/Ebooks_Encyclopedia27. t.me/Magazines4all

EBD_7156
82 DPP/ P 29
l 2π
16. (b) With path difference , waves are out of phase at the For the given wave ω = 2πn = 15π, k = = 10π
2 λ
point of observation. Coefficient of t ω 15π
Now v = = = = 1.5m / sec
2 2 2 2 1 2 2p Coefficient of x k 10π
17. (b) A = a = a + a + 2a cos q Þ cos q = - Þ q =
2 3
2π 2π
18. (c) For interference, two waves must have a constant phase and λ = = = 0.2 m.
relationship. Equation ‘1’ and ‘3’ and ‘2’ and ‘4’ have k 10π

p wBA 1 p2 v
a constant phase relationship of out of two choices. 24. (a) pm = , I = wp m A Þ I = m
2 v 2 2B
Only one S2 emitting ‘2’ and S4 emitting ‘4’ is given 25. (d), 26. (c), 27. (a).
so only (c) option is correct. 1
(i) I µ , dinitial = R, dfinal = 3R,
19. (a) The resultant amplitude is given by d2
Iinitial 9 I
AR = A2 + A2 + 2 AA cos q = 2 A2 (1 + cos q ) = Þ Ifinal = initial
Ifinal 1 9
= 2 A cos q / 2 (\1 + cos q = 2 cos 2 q / 2) (ii) During the first half time, wavelength first increases as
the component of velocity of source increases till it be-
1 1 æ pö comes equal to the velocity of source itself, then it de-
20. (d) y= sin wt ± sin ç wt + ÷
a b è 2ø creases till it becomes zeros.
p / 3 2p + p / 3 5p / 3 2p + 5p / 3
p (iii) t1 = , ....... , t 2 = ,
Here phase difference = w w w w
2
\ The resultant amplitude t2

2 2
æ 1 ö æ 1 ö 1 1 a+b
= ç ÷ +ç ÷ = + =
è aø è bø a b ab
30° t=0 60°
21. (a) In a wave equation, x and t must be related in the form
30° 60°
( x - vt )

1
We rewrite the given equations y = t1
1 + ( x - vt ) 2

1
For t = 0 , this becomes y = , as given
(1 + x 2 ) p
when w = rad / s , t1 = 1, 7, 13, ....... t2 = 5, 11, 17,
For t = 2 , this becomes 3
28. (c) The velocity of every oscillating particle of the medium
1 1 is different of its different positions in one oscillation
y= =
[1 + ( x - 2v ) 2 ] [1 + ( x - 1) 2 ] but the velocity of wave motion is always constant i.e.,
particle velocity vary with respect to time, while the
2v = 1or v = 0 m/s
wave velocity is independent of time.
22. (c)
Also for wave propagation medium must have the
y properties of elasticity and inertia.
l Distance travelled by wave (l)
29. (b) Velocity of wave =
P R Time period (T )
x Wavelength is also defined as the distance between two
Q nearest points in phase.
l/2 30. (b) Transverse waves travel in the form of crests and
through involving change in shape of the medium. As
liquids and gases do not possess the elasticity of shape,
23. (b) therefore, transverse waves cannot be produced in
Standard wave equation which travel in negative x-direc- liquid and gases. Also light wave is one example of
tion is y = A sin ( ωt + kx + f0 ) transverse wave.
t.me/Ebooks_Encyclopedia27. t.me/Magazines4all

DPP/ P 30 83

DAILY PRACTICE
PROBLEMS
PHYSICS
SOLUTIONS 30
1. (b) n1 = Frequency of the police car horn observer heard v n1 æ V ö
by motorcyclist 9. (d) By using n ' = n v - v Þ n = çè V - S ÷ø
s
n1 = Frequency of the siren heard by motorcyclist.
v - v0 æ 330 - 33 ö
n' = n =ç ´ 100 = 90 Hz
v2= Speed of motor cyclist 10. (a)
v è 330 ÷ø
330 - v 330 + v 11. (d) The apparent frequency heard by the observer is given
n1 = ´ 176; n2 = ´ 165
330 - 22 330 by
\ n1 - n2 = 0 Þ v = 22 m / s v 330 330
n' = x= ´ 450 = ´ 450 = 500 Hz
2. (c) Frequency of first over tone of closed pipe = Frequency v - vs 330 - 33 297
of first over tone of open pipe 12. (b) Observer is moving ayay form siren 1 and towards the
siren 2.
3v v 3 gP 1 gP é gP ù
Þ = Þ = ê\ v = ú v
4 L1 L2 4 L1 r1 L2 r2 ë r û

4L1 r1 4 L r1
Þ L2 = =
3 r2 3 r2
3. (b) For observer note of B will not change due to zero
relative motion.
Observed frequency of sound produced by A Stationay Moving Stationay
siren 1 observer siren 2
(330 - 30) Hearing frequency of sound emitted by siren 1
= 660 = 600 Hz
330
æ v - v0 ö æ 330 - 2 ö
\ No of beats = 600 - 596 = 4 n1 = n ç = 330 ç = 328Hz
è v ÷ø è 330 ÷ø
2v
4. (c) Open pipe resonanace frequency f1 = Hearing frequency of sound emitted by siren 2
2L
æ v - v0 ö æ 330 + 2 ö
n2 = n ç = 330 ç = 332 Hz
Closed pipe resonance frequency f 2 =
nv è v ø ÷ è 330 ÷ø
4L
Hence, beat frequency = n2 - n1 = 332 - 328 = 4
n
f 2 = f1 (where n is odd and f 2 > f1 ) \ n = 5 13. (b) At point A , source is moving away from observer so
4
apparent frequency n1 < n (actual frequency). At point
n(v - v0 )
5. (b) B source is coming towards observer so apparent
v - vs frequency n2 > n and point C source is moving

1 1 1æ v ö perpendicular to observer so n3 = n
6. (a) Wave number = but = l ' == l ç v - v ÷ and
l è sø Hence n2 > n3 > n1
14. (c) According the concept of sound image
v
vs = v + v person 345 + 5
3 n' = .272 = ´ 272 = 280 Hz
v - v person 345 - 5
, æ v ö v
\ (W .N .) = (W .N .) çè v - v / 3÷ø = 256 ´ 2u / 3 Dn = Number of beats = 280 – 272 = 8 Hz
15. (a) The observer will hear two sound, one directly from
3 source and other from reflected image of sound
= ´ 256 = 384
2
7. (d) Since there is no relative motion between observer and
source, therefore there is no apparent change in
frequency.
æ v ö æ 350 ö
n' = nç = 1200 ´ ç = 1400 cps
8. (c)
è v - vs ÷ø è 350 - 50 ÷ø
t.me/Ebooks_Encyclopedia27. t.me/Magazines4all

EBD_7156
84 DPP/ P 30
Hence number of beats heard per second OA i.e y = 0, 0 £ x £ L
æ v ö æ v ö AB i.e. y = L, 0 £ y £ L
=ç n-ç n
è v - vs ÷ø è v + vs ÷ø BC i.e. y = L, 0 £ x £ L

2nuus 2 ´ 256 ´ 330 ´ 5 OC i.e. x = 0, 0 £ y £ L


= = = 7.8 Hz The above conditions are satisfied only in alternatives
u2 - u2s 335 ´ 325
(b) and (c).
16. (a) In closed pipe only odd harmonics are present. Note that u ( x, y) = 0 for all four values e.g. in
17. (a) Maximum pressure at closed end will be atmospheric
pressure adding with acoustic wave pressure alternative (d ), u ( x, y) = 0 for y = 0, y = L But it is not
So Pmax = PA + P0 and Pmin = PA - P0 zero for x = 0 or x = L . Similarly in option (a)
u ( x, y) = 0 at x = L, y = L but it is not zero for
Pmax PA + P0
Thus P =
min PA - P0 x = 0 or y = 0 , while in options (b) and (c),
u ( x, y) = 0 for x = 0, y = 0, x = L and y = L
1 1
18. (b) nclosed = (nopen ) = ´ 320 = 160 Hz 26. (b), 27. (d).
2 2
l
19. (a) l = 1.21Å For fundamental force, = s Þ l = 2s
2
A A Y
N N Velocity of waves is, v =
N r where Y is Young's modu-
1.21 Å lus of quartz and r is its density.

1 l n æn ö 270 v 2.87 ´ 104 Y 1 2.87 ´ 10 4


20. (a) n µ Þ 2 = 1 Þ l2 = l1 ç 1 ÷ = 50 ´ = 13.5cm From f0 = = Þ ´ =
l l1 n2 è n2 ø 1000 l s r 2s s
21. (c) Loudness depends upon intensity while pitch depends Þ Y = 8.76 × 1012 N/m2
upon frequency.
3 ´ 2.87 ´ 104
22. (d) Using l = 2 ( l2 - l1 ) Þ v = 2n ( l2 - l1 ) For 3rd harmonic, f = 3f0 = 1.2 × 106 Hz Þ
s
Þ 2 ´ 215(63.2 - 30.7) = 33280cm / s = 1.2 × 106 Þ s = 0.07175 cm.
Actual speed of sound v0 = 332m / s = 33200cm / s
Hence error = 33280 – 33200 = 80cm / s 28. (d) As emission of light from atom is a random and rapid
23. (c) phenomenon. The phase at a point due to two
3 7 7 3 independent light source will change rapidly and
1 2 5 5 2 1 randomly. Therefore, instead of beats, we shall get
300 301 308 300 305 308 uniform intensity. However, if light sources are LASER
303 307
beams of nearly equal frequencies, it may possible to
8 8 observe the phenomenon of beats in light.
24. (a) Doppler shift doesn't depend upon the distance of listner 29. (d) The person will hear the loud sound at nodes than at
from the source. antinodes. We know that at anti-nodes the displacement
25. (b) Since the edges are clamped, displacement of the edges is maximum and pressure change is minimum while at
nodes the displacement is zero and pressure change is
u ( x, y) = 0 for line -
maximum. The sound is heared due to variation of
pressure.
y
Also in stationary waves particles in two different
C B segment vibrates in opposite phase.
(0,L) (L,L)
30. (a) Stationary wave
A

N N
N

O A A
x A node is a place of zero amplitude and an antinode is
(0, 0) a place of maximum amplitude.
t.me/Ebooks_Encyclopedia27. t.me/Magazines4all

DPP/ P 31 85

DAILY PRACTICE
PROBLEMS
PHYSICS
SOLUTIONS 31
1. (b)
20 division of vernier scale = 8 div. of main scale
A
æ 8ö æ 2ö
Þ 1 V.S.D. = çè ÷ø M.S.D. = çè ÷ø M.S.D.
c=
a+ b+ c b d
20 5 3

Least count B a C
æ 2ö If h is the depression or elevation then the radius of
= 1 M.S.D – 1 V.S.D. = 1 M.S.D. – çè ÷ø M.S.D
5 c2 h
curvature is given by r = +
6h 2
æ 2ö æ 3ö 3
= çè1 - ÷ø M.S.D. = çè ÷ø M.S.D. = ´ 0.1 cm. = 0.06 cm. Pitch
5 5 5 8. (b) L.C. =
No. of circular divisions
1 9. (b) The specific heat of a solid is determined by the method
(Q 1 M.S.D. = cm. = 0.1 cm.)
10 of mixture.
10. (a)
Directly we can use
0.5
æ b -aö 11. (d) Least count of screw gauge = mm = 0.01mm
L.C. = M - V = ç M 50
è b ÷ø
\ Reading = [Main scale reading + circular scale
æ 20 - 8 ö æ 1 ö 3 reading × L.C] – (zero error)
= çè ÷ø çè ÷ø cm. = cm. = 0.06 cm.
20 10 50 = [3 + 35 × 0.01] – (–0.03) = 3.38 mm
2. (c) Within elastic limit it obeys Hooke's Law i.e., stress µ 12. (d) 30 Divisions of vernier scale coincide with 29 divisions
strain. of main scales
29
Therefore 1 V.S.D = MSD
3. (c) Least count = 1 ´ 1 cm = 1 30
N 10 10N Least count = 1 MSD – 1VSD
4. (b) 5th division of vernier scale coincides with a main scale 29
= 1 MSD - MSD
1 30
division. L.C. = = 0.1mm
10 1
= MSD
\ Zero error = – 5 × 0.1 = – 0.5 mm 30
This error is to be subtracted from the reading taken for 1
= ´ 0.5° = 1 minute.
measurement. Also, zero correction = + 0.5 mm. 30
5. (b) If Y = Young's modulus of wire, M = mass of wire,
0.5
g = acceleration due to gravity, x = extension in the wire, 13. (c) Least count = = 0.01mm
50
A= Area of cross-section of the wire,
l = length of the wire. Zero error = 5 × L.C
= 5 × 0.01 mm
Mgx DY DM Dx DA Dl = 0.05 mm
Y= Þ = + + +
Al Y M x A l Diameter of ball = [Reading on main scale] + [Reading
on circular scale × L . C] – Zero error
DY 0.01 0.01 2 ´ 0.001 0.001
Þ = + + + = 0.065 = 0.5 × 2 + 25 × 0.01 – 0.05
Y 3.00 0.87 0.041 2.820
= 1.20 mm
DY Dg Dl DT
or ´ 100 = ± 6.5% 14. (d) = +2
Y g l T
6. (b) The instrument has negative zero error.
7. (c) If A, B and C be the points corresponding to the Dl and DT are least and number of readings are
maximum in option (d), therefore the measurement of
impressions made by the legs of a spherometer then
g is most accurate with data used in this option.
t.me/Ebooks_Encyclopedia27. t.me/Magazines4all

EBD_7156
86 DPP/ P 31
mg L 2 2
15. (b) We know that Y = ´ d y æ dy ö dy
Þ xy + xç ÷ - y =0
D 2 l dx 2 è dx ø dx
p
4 18. (a) The condition for terminal speed (vt) is
Weight = Buoyant force + Viscous force
4mgL 4 ´ 1 ´ 9.8 ´ 2
Þ Y= =
( ) ´ (0.8 ´ 10 )
2 2
pD l p 0.4 ´ 10 -3 -3
Fv B=Vr2 g

= 2.0 ´ 1011 N/m 2

Now DY = 2DD + Dl
Y D l
[Q the value of m, g and L are exact]
0.01 0.05 W=V r 1g
= 2´ + = 2 × 0.025 + 0.0625
0.4 0.8
= 0.05 + 0.0625 = 0.1125
Þ DY = 2 × 1011 × 0.1125 = 0.225 × 1011 Vg (r1 - r1 )
\ V r1 g = V r2 g + kvt2 \ vt =
k
= 0.2 ´ 1011 N/m 2
16. (b) The time period of a simple pendulum is given by 19. (d) From the figure it is clear that liquid 1 floats on liquid
2. The lighter liquid floats over heavier liquid.
l l l Therefore we can conclude that r1 < r2
T = 2p \ T 2 = 4p 2 Þ g = 4p 2 2
g g T Also r3 < r2 otherwise the ball would have sink to the
bottom of the jar.
Dg Dl DT Also r3 > r1 otherwise the ball would have floated in
Þ ´ 100 = ´ 100 + 2 ´ 100
g l T liquid 1. From the above discussion we conclude that
Case (i) r1 < r3 < r2.
Dl = 0.1 cm, l = 64cm, DT = 0.1s, T = 128s 20. (c) In case of water, the meniscus shape is concave
upwards. Also according to ascent formula
Dg
\ ´ 100 = 0.3125 2T cos q
g h=
rrg
Case (ii)
Dl = 0.1 cm, l = 64cm, DT = 0.1s, T = 64s The surface tension (T) of soap solution is less than
water. Therefore rise of soap solution in the capillary
Dg tube is less as compared to water. As in the case of
\ ´ 100 = 0.46875
g water, the meniscus shape of soap solution is also
Case (iii) concave upwards.
Dl = 0.1 cm, l = 20cm, DT = 0.1s, T = 36s
l
Dg 21. (c)
\ ´ 100 = 1.055 A
g Y

Dg Wire (1)
Clearly, the value of ´ 100 will be least in case (i)
g
2
2r ( d1 - d 2 ) g 3A
17. (c) Terminal velocity, vT = Y
9h

vT
2 (10.5 - 1.5) 9 l/3
= Þ vT = 0.2 ´ Wire (2)
0.2 (19.5 - 1.5) 2 18
\ vT = 0.1 m / s As shown in the figure, the wires will have the same
2
Young’s modulus (same material) and the length of the
ìï d2y æ dy ö üï
2
dy wire of area of cross-section 3A will be l/3 (same
x í - By 2 - B ç ÷ ý + By =0
ïî dx è ø
dx ïþ dx volume as wire 1).
t.me/Ebooks_Encyclopedia27. t.me/Magazines4all

DPP/ P 31 87
For wire 1, 25. (a) Screw gauge is used to measure the diameter (d) of the
F/A
wire so that the area of cross-section is calculated by the
Y= ...(i) formula
D x/l
For wire 2 , pd 2
A=
F '/ 3 A 4
Y= ...(ii) 26. (b) Both the statements (1) & (2)are precautions to be taken
Dx /( l / 3)
during the experiment.
F l F' l 27. (a) The liquid cools faster first and slowly later on when its
From (i) and (ii) , ´ = ´ Þ F ' = 9F
A Dx 3 A 3Dx temperature gets close to surrounding temperature.
22. (b) Lower the vernier constant, more accurate measurement 28. (a) Maximum percentage error in measurement of e, as given
is possible by it. by Reyleigh’s formula.
(Given error is measurement of radius is 0.1 cm)
d
23. (b) Effective length = MC = MN + NC = l + De = 0.6 DR = 0.6 × 0.1 = 0.06 cm.
2 Percentage error is
M De
´ 100 =
0.06
´ 100 = 3.33%
e 0.6 ´ 3
29. (b) Speed of sound at the room temperature.
l l1 = 4.6 cm, l2 = 14.0 cm.,
N l = 2 (l2 – l1) = 2 (14.0 – 4.6) = 18.8 cm.
d/2 18.8
C v = f l = 2000 ´
100
= 376 m / s

30. (c) End correction obtained in the experiment.


l 2 - 3l1 14.0 - 3 ´ 4.6
e= = = 0.1 cm.
24. (c) Here, original length (L) = y, 2 2
Extension (l) = x, Force applied (F) = p
Area of cross-section (A) = q
FL
Now, Young's modulus (Y) =
AL
yp
ÞY=
xq
t.me/Ebooks_Encyclopedia27. t.me/Magazines4all

EBD_7156
88 DPP/ P 32
DAILY PRACTICE
PROBLEMS
PHYSICS
SOLUTIONS 32
r
ur e2 $ e2 r æ\ r$ = r ö
1. (d) Q1 + Q2 = Q .....(i) 4. (c) F = - k 2 r = - k . 3 r çç ÷
r ÷ø
r r è
Q1Q2 5. (c) After following the guidelines mentioned above
and F =k ..... (ii)
r2
From (i) and (ii)
FC FD
kQ1 (Q - Q1 )
F=
r2 +Q +Q FAC
For F to be maximum A FA
B
dF Q
= 0 Þ Q1 = Q2 =
dQ1 2
2. (a) The position of the balls in the satellite will become as
shown below
180°
L L
+Q +Q C
D
+Q
Thus angle q = 180o

1 Q2
and force = . Fnet = FAC + FD = FA2 + FC2 + FD
4p Î0 (2 L )2

3. (b) FA = Force on C due to charge placed at A kq 2 kq 2


Since FA = FC = and FD =
a2 ( a 2)2
FA
2kq2 kq 2 kq 2 æ 1ö q2 æ1+ 2 2 ö
+2µC Fnet = + = 2 + =
2 èç ÷ø ç ÷
a2 2a 2
a 2 2
4p Î0 a è 2 ø
C 120°
6. (a)
FB
1 (+7 ´10-6 (-5 ´10-6 ) 1 35 ´10 -12
F= =- N
4pe 0 r 2 4pe0 r2

+ 1µC 1 (+5 ´ 10-6 ( -7 ´ 10 -6 ) 1 35 ´ 10 -12


- 1µC F¢ = = - N
4 pe 0 r2 4pe0 r2
A 10 cm B kQ
7. (c) Electric field outside of the sphere E out = ...(i)
r2
10-6 ´ 2 ´10-6 kQx
= 9 ´109 ´ = 1.8 N Electric field inside the dielectric shphere Ein = ...(ii)
(10 ´10-2 )2 R3
r2 x
FB = force on C due to charge placed at B From (i) and (ii), Ein = Eout ´
R
10-6 ´ 2 ´10-6 3 ( 20 )
2
= 9 ´109 ´ = 1.8 N Þ At 3 cm, E = 100 ´ = 120 V/m
(0.1) 2 103
Net force on C

Fnet = ( FA ) 2 + ( FB ) 2 + 2 FA FB cos120o = 1.8 N


t.me/Ebooks_Encyclopedia27. t.me/Magazines4all

DPP/ P 32 89
8. (c) Electric lines force due to negative charge are radially
inward. 2q 2q
2E
2E
E
q q Þ Enet = 0
2E E
2E
2q 2q

2q q
E
2E
q 2E E E
2q Þ
9. (a) In non-uniform electric field, intensity is more, where E 2E 2E
the lines are more denser. E
2E
10. (b) According to the question,
2q q
mg Enet = 0
eE = mg Þ E =
e
11. (b) Because E points along the tangent to the lines of force. 15. (b) D C
If initial velocity is zero, then due to the force, it always EA
moves in the direction of E. Hence will always move
on some lines of force. EB ED
O
12. (b) The field produced by charge - 3Q at A , this is E as EC
mentioned in the example.
A B
3Q
\ E = 2 (along AB directed towards negative
x
Þ D C
A B
charge) Q –3Q
x
Now field at location of -3Q i.e. field at B due to 2E 2E
Q E Enet
charge Q will be E ' = 2 = (along AB directed B
x 3 A
away from positive charge)
13. (c) The electric field is due to all charges present whether E A = E, E B = 2 E , EC = 3E, ED = 4 E
inside or outside the given surface.
14. (b) ur s $ 2s $ s $ 2s $
q 16. (b) E =- k- k- k =- k
q 2e 0 2e 0 2e0 e0
E 17. (b)
E
E
q q Þ Enet = 0
E E
E q1
q q
18. (c) According to Gauss law Ñò E.ds =
e0
q –q Ñò ds = 2prl; (E is constant)
E 2E q1 q 1
E 120° \ E. 2prl = ÞE= i.e. E µ
q q Þ e0 2pe0 r r
E E E 2E
E 2E
q –q æ 3Q ö
Enet = 2 E 19. (c) Let sphere has uniform chare density r ç ÷ and E
è 4pR 3 ø
is the electric field at distance x from the centre of the
sphere.
t.me/Ebooks_Encyclopedia27. t.me/Magazines4all

EBD_7156
90 DPP/ P 32
Applying Gauss law, 26. (a) According to Gauss law
uur uur
E.4p x2 =
q
e0
=
rV¢
e0
=
r 4 3
´ px
e0 3
Ñò s
E.dS = 4pr 2 E (r )
x
r
(V = Volume of dotted sphere)
ò 4pr r (r) dr
2
R
r = 0
\ E = 3e x Þ E = µ x Î0
0
r
20. (b) T sin q = qE
ò0 r
2
4pA ( R - r )dr
and T cos q = mg + Þ 2
4 p r E (r ) =
+ Î0
qE + q
Þ tan q = +
mg + T T cos q
+ 4pA æ r 3 R r 4 ö
= ç - ÷
q æ s ö +
T sin q
qE Î0 çè 3 4 ÷ø
= ç ÷
mg è 2e0 ø mg
Þ s µ tan q. A æ rR r 2 ö
Hence, E (r ) = ç - ÷ , for 0 < r < R
Î0 çè 3 4 ÷ø
Q
21. (d) Next flux through the cube fnet = ; so flux through
e0 3Q
But A =
q pR 4
one face fface =
6e 0
3Q é 1 æ r ö 1 æ r ö ù
2
22. (d). For A : Power consumed P = I2R \ We get, E (r ) = ê ç ÷ - ç ÷ ú
Î0 R 2 êë 3 è R ø 4 è R ø úû
q
But e = f , 27. (b) The electric field outside the sphere is given by :
0

dq kQ
q = a e0 t 2 Þ I = = 2a e 0 t Þ P = 4a 2 e 02 Rt 2 E (r ) = , for r ³ R
so r2
dt
For B : Assuming initial charge in reservoir be q0 then 28. (c) If electric lines of forces cross each other, then the
electric flux through a closed electric field at the point of intersection will have two
direction simultaneously which is not possible
q 0 - ae 0 t 2
Spherical surface around S2 will be fS2 = physically.
e0 29. (c) Electric field at the nearby point will be resultant of
dfS2 existing field and field due to the charge brought. It
For C : = -2at may increase or decrease if the charge is positive or
dt
negative depending on the position of the point with
23. (c). The time period will change only when the additional
respect to the charge brought.
electrostatic force has a component along the direction
of the displacement, which is always perpendicular 30. ()
to the string.
24. ()

25. (b) Net charge inside the sphere = òsphere r dV


Due to spherical symmetry, we get
R R
ò0 4pr r(r )dr ò0 r
2 2
Q= = 4pA ( R - r ) dr

æ R4 R4 ö
= 4pA ç - ÷
ç 3 4 ÷ø
è

3Q
\ A=
pR 4
t.me/Ebooks_Encyclopedia27. t.me/Magazines4all

DPP/ P 33 91

DAILY PRACTICE
PROBLEMS
PHYSICS
SOLUTIONS 33
1. (c) ABCDE is an equipotential surface, on equipotential 9. (b) Potential at A= Potential due to (+q) charge
surface no work is done in shifting a charge from one + Potential due to (– q) charge
place to another.
1 q 1 (-q)
2. (c) Potential at centre O of the square = . + =0
4pe0 a2 + b 2 4pe0 a 2 + b2
Q Q
10. (c) Point P will lie near the charge which is smaller in
a magnitude i.e. -6mC . Hence potential at P
a 2

O
P -6mC 12mC
Q Q
x 20 cm

æ Q ö 1 (-6 ´ 10-6 ) 1 (12 ´ 10-6 )


V0 = ç
ç 4pe (a / 2) ÷÷ V= + = 0 Þ x = 0.2 m
è 0 ø 4pe0 x 4pe0 (0.2 + x)
Work done in shifting (– Q ) charge from centre to
infinity
11. (a) Work done W = q -6 (V A - VB ); where
W = -Q(V¥ - V0 ) = QV0
q = 3 ´10-6 coulomb where
4 2Q 2Q 2
= Q. =
4pe0a pe 0a é (-5 ´10-6 ) 2 ´10-6 ù 1
3. (b) Using VA = 1010 ê + 6
ú = ´10 volt
-2 -2
êë 15 ´ 10 5 ´ 10 úû 15
2QV v QA q 1
v= Þvµ Q Þ A = = = -6 -6
é ù
M vB QB 4q 2 and V B = 1 0 1 0 ê ( 2 ´ 1 0 ) - 5 ´ 1 0 ú
-2
4. (a) Work done in moving a charge from P to L , P to M êë 1 5 ´ 1 0 5 ´ 10 -2 úû
and P to N is zero while it is q (VP – Vk) > 0 for motion
13
from P to k. =– ´ 106 volt
15
5. (a) KE = q(V1 - V2 ) = 2 ´ (70 - 50) W = 40 eV
é1 æ 13 öù
6. (a) The electric potential V ( x, y , z ) = 4 x 2 volt \ W = 3 ´ 10-6 ê ´ 106 - ç ´ 106 ÷ ú
ë15 è 15 øû
ur æ ¶V $ ¶V $ ¶V ö
Now E = - ç $i +j +k ÷ = 2.8 J
è ¶x ¶y ¶z ø
12. (c)
¶V ¶V A p
¶V
Now ¶x = 8 x, ¶y = 0 and =0 +q
¶z
ur
Hence E = -8$i, so at point (1m, 0, 2m) l l
p net
ur
E = -8 xi$ volt/meter or 8 along negative X - axis.
7. (b) Electric fields due to electrons on same line passing 60°
B C
through centre cancel each other while electric potential +q l – 2q
due to each electron is negative at centre C. Therefore, p
ur
at centre E = 0,V ± 0
ur r Pnet = p 2 + p 2 + 2 pp cos 60o = 3 p = 3 ql (\ p = ql )
8. (a) By using W = Q( E.Dr )

Þ W = Q[(e1$i + e2 $j + e3 k$ ).(ai$ + b $j )] = Q( e1a + e2b )


t.me/Ebooks_Encyclopedia27. t.me/Magazines4all

EBD_7156
92 DPP/ P 33
13. (d) According to figure, potential at A and C are both 18. (b) W = q(V02 - V01 )
equal to kQ. Hence work done in moving -q charge
Q1 Q2
from A to C = - q (VA - VC ) = 0 where V01 = +
4pe 0 R 4pe0 R 2

A Q2 Q1
and V02 = +
–q 4pe 0 R 4pe0 R 2

l l q(Q2 - Q1 ) ( 2 - 1)
Þ W = q(V02 - V01 ) =
4pe0 R 2

B C 19. (d) V = q é 1 1 ù q é1 1 1 ù
+Q l 4pe0 x0 ê1 + 3 + 5 + ...ú - 4pe x ê 2 + 4 + 6 + ...ú
ë û 0 0 ë û

q é 1 1 1 ù q
(+1.6 ´ 10-19 ) = 1 - + - + ...ú = loge 2
14. (c) Q
V = k ´ = 9 ´ 109 ´ = 27.2V 4pe0 x0 êë 2 3 4 û 4pe 0 x0
r 0.53 ´ 10-10
20. (b) Potential decreases in the direction of electric field.
15. (c) Potential will be zero at two points Dotted lines are equipotential surfaces
\V A = VC and VA > VB
q1 = 2mC q 2 = -1mC
M N y
O C
x=0 x=4 x=6 x = 12
l l' A B
6
At internal point (M) :
E
1 é 2 ´ 10-6 (-1´10-6 ) ù
´ê + ú =0
4pe 0 êë (6 - l ) l úû kp -3
21. (d) Eequatorial =
3
i.e.E µ p and E µ r
r
Þl =2
22. (a) Suppose neutral point N lies at a distance x from dipole
So distance of M from origin;
of moment p or at a distance x2 from dipole of 64 p.
x =6-2 = 4
At exterior point (N): ® ®
p 64 p
N
é 2 ´10-6 (-1´10-6 ) ù 1 2
1
´ê + ú =0 x1
4pe0 ëê (6 - l ') l¢ ûú p
25 cm
Þl'=6 At N |E.F. due to dipole 1 | = |E.F. due to dipole 2 |
So distance of N from origin, x = 6 + 6 = 12
1 2p 1 2 ( 64 p )
16. (a) V = VAB + VBC + VCD . .
Þ = 4 pe0 ( 25 - x )3
4pe0 x 3
k .5Q0 k .(-2Q0 ) k .(3Q0 )
= + +
R R R 1 64
Þ = Þ x = 5 cm.
=
6 kQ0 x 3 ( 25 - x )3
R
æ 120 ö
(a). BC = 2R sin ç = 3R
è 2 ÷ø
3Q0 23.
=
2p Î0 R
1 æ 2q / 3 ö q
Electric field at O = ç ÷ =
9 p 4pe 0 R è R ø 6pe 0 R 2
2
17. (a) V = 9 ´10 . 2
r along negative X-axis.
(1.6 ´10-19 ) ´1.28 ´ 10 -10
= 9 ´109 ´ = 0.13V
(12 ´ 10-10 )2
t.me/Ebooks_Encyclopedia27. t.me/Magazines4all

DPP/ P 33 93

B
q/3 F–
30° F
Net force F :
C 120° 60° x
–2q/3 O F+
60° 60°
60° 30°
Net torque immediately after it is released Þ clockwise
A A body cannot exert force on itself.
q/3 28. (d) When the bird perches on a single high power line, no
current passes through its body because its body is at
equipotential surface i.e., there is no potential
The potential energy of the system is non zero
difference. While when man touches the same line,
Force between B & C
standing bare foot on ground the electrical circuit is
1 (q / 3) ( -2q / 3) q2 completed through the ground. The hands of man are
= =
4 pe0 ( 3R) 2 54pe 0 R 2 at high potential and his feet’s are at low potential.
Hence large amount of current flows through the body
1 æ q q 2q ö of the man and person therefore gets a fatal shock.
+ - =0
4pe 0 çè 3 3 3 ÷ø
Potential at O = 29. (a) Electron has negative charge, in electric field negative
charge moves from lower potential to higher potential.
24. (d) The given graph is of charged conducting sphere of 30. (b) Potential is constant on the surface of a sphere so it
radius R0. The whole charge q distributes on the sur- behaves as an equipotential surface. Free charges
face of sphere (electrons) are available in conductor. The two
25 (b), 26 (b), 27 c statements are independent.
F+
+


F–

(F+ > F– as E+ > E–)


t.me/Ebooks_Encyclopedia27. t.me/Magazines4all

EBD_7156
94 DPP/ P 34
DAILY PRACTICE
PROBLEMS
PHYSICS
SOLUTIONS 34
1. (b) Net electrostatic energy
10 ´10-6 ´10 ´10-6 10 ´10-6 ´10 ´10-6 ù
kQq kq 2
kQq + + ú
U= + + =0 10/100 10/100 úû
a a a 2
100 ´10-12 ´100
kq æ Q ö 2q = 3 ´ 9 ´109 ´ = 27 J
Þ çQ + q + ÷=0ÞQ=- 10
a è 2ø 2+ 2
1 ( q )( -2q ) 1 (-2q )( q ) 1 ( q )( q )
2. (c) Electric field is perpendicular to the equipotential 7. (c) U System = 4 pe a
+
4pe0 a
+
4 pe0 2a
surface and is zero every where inside the metal. 0

kq q 7q2
3. (c) Þ ( Q + q + Q ) = 0 Þ Q = - net potential energy U System = -
8pe0a
l 2
ur ur
1 q2 8. (c) In the given condition angle between p and E is zero.
U net = 3´ .
4pe0 l Hence potential energy U = - pE cos 0 = - pE = min.
4. (d) Length of the diagonal of a cube having each side b is Also in uniform electric field Fnet = 0
3b. So distance of centre of cube from each vertex is 1 (-e)(-e)
9. (b) U = As r decreases then U
4p Î0 r2
3b
. increases and sign of U is '+ve' so, U increases.
2
é 1 1 1 1 ù
Hence potential energy of the given system of charge is 10. (c) U = 2kq 2 ê - + - + + .....ú
ë a 2a 3a 4a û
ì 1 ( -q )( q ) ü -4q2
U = 8´ í . ý= 2q 2 é 1 1 1 .....ù
î 4pe0 3b / 2 þ 3pe0b =- 1- + - + ú
4pe0a êë 2 3 4 û
5. (a) Change in potential energy ( D U ) = U f - U i
2q 2 loge 2
q3 =-
4pe0 a
11. (b) The initial energy of the system
q

40 cm
50 cm
q2
q1
D
q q
30 cm 10 cm a

40 cm 1 q2
Ui= =U
1 éæ q1q3 q2 q3 ö æ q1q3 q2 q3 ö ù 4pe 0 a
ÞVU = ê + - + ú The final energy of the system
4 pe0 ëçè 0.4 0.1 ÷ø çè 0.4 0.5 ÷ø û
1 é q2 q2 q2 ù
1 q Uf = ê + + ú = 3U
ÞVU = [8q2 q3 ] = 3 (8q2 ) 4pe0 êë a a a úû
4pe 0 4pe0 Thus work done, W = Uf– Ui = 3U – U = 2U
\ k = 8q2 kq1q2
12. (d) U =
1 q1q2 r
6. (c) For pair of charge U = . kQ
4pe0 r 13. (c) As potential at A and B is same, VA = VB = . So,
d
1 é 10 ´ 10-610 ´ 10-6 work done in both cases will be same.
U System = ê
4 pe0 êë 10 /100 kq q
14. (b) U = e 1 2 . There will be 6 pairs, 4 on a side of square
r
and 2 as diagonal.
t.me/Ebooks_Encyclopedia27. t.me/Magazines4all

DPP/ P 34 95
15. (c) Apply conservation of mechanical energy between 23. (b) Force = eE
point a and b : (K.E. + P.E.)a = (K.E. + P.E.)a Work done = force × distance
Force and distance are in opposite direction, so work
k (3 ´ 10 -9 )q0 k (3 ´ 10 -9 ) q0 is negative.
Þ 0+ -
0.01 0.02 W = – eE × d
Here, distance increases so, potential energy increases.
1 2 k (3 ´ 10-9 )q0 k (3 ´ 10-9 )q0 24. (d) Under electrostatic condition, all points lying on the
= mv + -
2 0.02 0.01 conductor are in same potential. Therefore, potential
at A = potential at B.
Put the values we get : v = 12 15 = 46 m/s
From Gauss's theorem, total flux through the surface
kqQ kqQ kq 2 of the cavity will be q / ε 0 .
16. (b) U = - - + = 0 Þ Q / q = 1/ 4
r r 2r Note : Instead of an elliptical cavity, if it would and
17. (b) Find potential at A and C due to charge at B, then been a spherical cavity then options (a) and (b) were
required work done is W = q (VA – VC)
also correct.
18. (d) It depends whether both charges are of same or opposite
25. (b) q1 + q2 = 0
sign. q2
19. (a) Because work is to be done by an external agent in
moving a positive charge from low potential to high Q
C
potential and this work gets stored in the form of q1
potential energy of the system. Hence, it increases. B
æ 3 3 1 ö A
20. (a) U = kq 2 ç - + - ÷
è d 2d 3d ø
æ 2 3 1 ö æ 2 2 1 ö
+ kq 2 ç - + - ÷ + kq 2 ç - + - ÷
è d 2d 3d ø è d 2d 3d ø
æ 1 2 1 ö æ 2 1 ö kq1 kQ kq2
+ kq 2 ç - + - + kq 2 ç - + VA = + +
÷ ÷ R 2 R 4R
è d 2d 3d ø è d 2d ø
kq kQ kq2
æ 1 2 ö 2æ 1 ö VC = 1 + +
+ kq 2 ç - + ÷ + kq ç - d ÷ 4 R 4 R 4R
è d 2 d ø è ø VA = VC
æ 12 12 4 ö \ q1 = – Q/3 and q2 = Q/3
U = kq 2 ç - + - ÷
26. (b) VA = k éê
d –Q Q Q ù Q
è 2 d 3d ø + + ú =
ë 3R 2R 12R û 16 pe0 R
12kq 2 æ 1 1 ö
=- ç 1- + ÷ é –Q Q Q ù 5Q
d è 2 3 3ø 27. (c) VB = k ê + + ú =
ë 6 R 2 R 12 R û 48pe0 R
kQ 1 1 e0 k 2Q 2 28. (a) Inside electric field is zero but not outside.
21. (c) V = Þ u = e0 E 2 = 29. (c) Earth also has some surface charge density due to which
r 2 2 r4
it produces electric field in the surrounding space.
V4 µu 30. (d) Net potential at centre
22. q 4 B kq kq kq kq
+ - + - =0
A a/ 2 a/ 2 a/ 2 a/ 2
and field is zero due to symmetry.
C +q –q
a

kq 9 ´ 109 ´ 1 ´ 10 -6
AC = 5m, V = = a a
AC 5
= 1.8 × 103 = 1.8 kV
VB = (VB)due to q + (VB)i, where (VB)i = Potential at B a
due to induced charge –q +q
If electric potential at a point is zero then the magnitude
kq of electric field at that point is not necessarily to be
\ 1.8 × 103 = + (VB )i
AB zero.
Þ 1.8 × 103 = 2.25 × 103 + (VB)i
Þ (VB)i = – 0.45 kV
t.me/Ebooks_Encyclopedia27. t.me/Magazines4all

EBD_7156
96 DPP/ P 35
DAILY PRACTICE
PROBLEMS
PHYSICS
SOLUTIONS 35
1. (a) By using
V
across A is V and potential difference across B is
V = V0 e -t / CR Þ 40 = 50e -1/ CR Þ e -1/ CR = 4 / 5 3
Potential difference after 2 sec hence energy of the system now is
2 2
æ4ö 1 1 æV ö 10
V ' = V0 e-2 / CR = 50(e-1/ CR )2 = 50 ç ÷ = 32V U 2 = (3C )V 2 + (3C ) ç ÷ = CV 2 ......(ii)
è5ø 2 2 è3ø 6
Fraction of energy after 1 sec
U1 3
So, =
1 U2 5
C (V f ) 2 2
æ 40 ö 16
= 2 =ç ÷ = 4. (c) Plane conducting surfaces facing each other must have
1 è 50 ø 25
C (Vi ) 2 equal and opposite charge densities. Here as the plate
2
areas are equal, Q2 = -Q3 .
2. (c) The given circuit can be redrawn as follows. All
The charge on a capacitor means the charge on the
capacitors are identical and each having capacitance
inner surface of the positive plate (here it is Q2 )
e A
C= 0 Potential difference between the plates
d
charge Q 2Q
= = 2 = 2
1 2 capacitance C 2C

3 2 Q - ( -Q2 ) Q2 - Q3
= 2 =
2C 2C
4
5. (b) While drawing the dielectric plate outside, the
5 4 capacitance decreases till the entire plate comes out
and then becomes constant. So, V increases and then
becomes constant.
6. (b) Given circuit can be reduced as follows
+ – 3C 3C
V
A B
| Charge on each capacitor | = | Charge on each plate | ( C = capacitance of each capacitor)
e A The capacitor 3C ,3C shown in figure can with stand
= 0 V
d maximum 200 V .
Plate 1 is connected with positive terminal of battery \ So maximum voltage that can be applied across
e0 A A and B equally shared. Hence maximum voltage applied
so charge on it will be + .V cross A and B be equally shared. Hence Maz. voltage
d
applied across A and B will be (200 + 200) = 400 volt.
Plate 4 comes twice and it is connected with negative 7. (c) Common potential
terminal of battery, so charge on plate 4 will be
C V + C2 ´ 0 C1
2e 0 A V'= 1 = .V
- V C1 + C2 C1 + C2
d
3. (c) Initially potential difference across both the capacitor C1V1 - C2V2 6 ´ 12 - 3 ´ 12
is same hence energy of the system is 8. (b) V = = = 4 volt
C1 + C2 3+ 6
1 1
U1 = CV 2 + CV 2 = CV 2 .....(i) A
2 2 K1e0
2 = K1e0 A
In the second case when key K is opened and dielectric 9. (d) C1 =
medium is filled between the plates, capacitance of both æd ö d
ç ÷
the capacitors becomes 3C, while potential difference è2ø
t.me/Ebooks_Encyclopedia27. t.me/Magazines4all

DPP/ P 35 97
13. (d) In the given system, no current will flow through the
A
K 2 e0 branch CD so it can be removed
C2 = 2 = K 2e0 A
æd ö d C
ç ÷ 5µF
è2ø
10µF 10µF
K 3e 0 A K 3 e 0 A
and C3 = =
2d 2d
A B
C1C2 æK KK ö e A
Now, Ceq = C3 + = ç 3 + 1 2 ÷. 0
C1 + C2 è 2 K1 + K 2 ø d 10µF 10µF
5µF
10. (b) Given circuit is a balanced Wheatstone bridge.
D

11. (a) P
Effective capacitance of the system = 5 + 5 = 10mF
2C 2C 14. (c) Volume of 8 small drops = Volume of big drop
4 4
2C 8 ´ pr 3 = pR 3 Þ R = 2r
3 3
C
C C As capacity is proportional to r , hence capacity
becomes 2 times.
Q 15. (c) Potential difference between the plates
C + C = 2C
V = Vair + Vmedium

P s s
= ´ (d - t ) + ´t
e0 K e0 + –
2C 2C + –
s t + –
ÞV = (d - t + ) + –
2C e0 K A K
Þ + –
C + –
2C Q t + –
= (d - t + )
Ae 0 K
Q
Hence capacitance t
2C/ 2 = C

Q Q
C= =
P V Q t
(d - t + )
Ae0 K
2C 2C

e0 A e0 A
Þ = =
t æ 1ö
C C (d - t + ) d - t ç1 - ÷
K è Kø

Q e0 A K e0 A
16. (b) C = = 1 pF and C ' = = 2 pF \ K = 4
C + C = 2C d 2d
17. (a) Potential difference across the condenser
P
s s
V = V1 + V2 = E1t1 + E2 t2 = t1 + t2
2C
K1e0 K 2e0

2C s æ t1 t ö Q æ t1 t ö
Þ Þ CPQ = 3C ÞV = ç + 2 ÷= ç + 2 ÷
2C
e0 è K1 K 2 ø Ae 0 è K1 K 2 ø
18. (d) When the battery is disconnected, the charge will
remain same in any case.
Q
Capacitance of a parallel plate capacitor is given by
12. (d) The two capacitors formed by the slabs may assumed e0 A
to be in series combination. C=
d
t.me/Ebooks_Encyclopedia27. t.me/Magazines4all

EBD_7156
98 DPP/ P 35
When d is increased, capacitance will decreases and 23. (a) Electric field between the plates of a parallel plate
because the charge remains the same, so according to s Q
capacitor E = = i.e E µ d 0
q = CV , the voltage will increase. Hence the e Ae0
electrostatics energy stored in the capacitor will
increase. e0 A
19. (c) 24. (a) Capacitance of parallel plate condenser =
s –s d

25. (a)
A. .B .C C1 C2+C3

s s V
EA = - =0
2e0 2e0
EC = 0,
s s s V (C2 + C3 )
EB = + = . VC1 =
2e0 2e0 e0 C1 + (C2 + C3 )

V V0 / k V0 Initially C3 = 0
20. (b) E = = =
d d kd VC2
So VC1 = =6 ........... (1)
( 3 + 3) ´ (1 + 1) C1 + C2
6´ 2 ö
21. (c) Ceq = + 1 = æç ÷ +1 =
5
mF
( ) ( )
3 + 3 + 1 + 1 è 6+ 2ø 2 Now, at VC1 = 10, C3 = ¥

5 V (C2 + C3 )
\ Q = C ´ V= ´ 100 = 250 mC Þ 10 =
2 C1 + (C2 + C3 )
æ 6´ 2 ö
Change in 6 mF branch – VC = ç ÷100 = 150 mC Þ 10 =
V
Þ 10 = V .......... (2)
è 6+ 2ø
æ C1 ö
B çè C + C + 1÷ø
2 3

Eq. (1) and (2),


6mF 2mF
10 æC ö
= 6 Þ 5 = 3 ç 1 + 1÷
æ C1 ö è C2 ø
1mF
çè C + 1÷ø
2

C1 5 2
Þ = -1 =
C2 3 3
A C
100 V 10 (C2 + C3 )
26. (b) Now, VC1 = =8
150 (C1 + C2 + C3 )
VAB = = 25 V and VBC = 100 – VAB = 75 V
6
æC C ö
22. (c) Capacitance will be increased when a dielectric is 10 ç 2 + 3 ÷
è C1 C1 ø
introduced in the capacitor but potential difference will Þ = 8 Þ C = 2.5C
remain the same because battery is still connected. So æ C2 C3 ö 3 1
çè1 + C + C ÷ø
according to q = CV, charge will increase i.e. Q > Q0 1 1
and 1 1 1
27. (c) + » (where C3 ® ¥ )
C3 + C2 C1 C1
1 1
U= QV0 , U0 = Q0 V0 Þ Q > Q0 so U > U 0
2 2 \ Total energy = energy in C1
\ Required ratio = 1
t.me/Ebooks_Encyclopedia27. t.me/Magazines4all

DPP/ P 35 99
28. (b) The electric field due to one charged plate at the 30. (b) By the formula capacitance of a capacitor
s KA K
location of the other is E = and the force per unit C1 = e0 ´ µ
2e 0 d d
C1 K1 d 2 K d /2 1
s2 Hence, = ´ = 1´ =
area is F = sE = . C2 d1 K 2 K 2 3K 6
2e0
or C2 = 6C1
29. (d) A charged capacitor, after removing the battery, does
not discharge itself. If this capacitor is touched by Q
Again for capacity of a capacitor C =
someone, he may feel shock due to large charge still V
present on the capacitor. Hence it should be handled Therefore, capacity of a capacitor does not depend
cautiously otherwise this may cause a severe shock. upon the nature of the material of the capacitor.
t.me/Ebooks_Encyclopedia27. t.me/Magazines4all

EBD_7156
100 DPP/ P 36
DAILY PRACTICE
PROBLEMS
PHYSICS
SOLUTIONS 36
6. (a). The circuit is equivalent to Fig. It is a balanced
1. (b). The current in 1W resistance is 3A. The current in 3W wheatstone bridge between abcd, and then in parallel (2R)
resistance is resistances. Thus ignoring resistance between bd arm. The
R2 6 circuit is equivalent to three (2R) resistances in parallel
I1 = I = × 3 = 2A. (abc, adc, aRRc).
R1 + R 2 3+ 6
b
2
Therefore the ratio is .
3 R
a c
(R1 + R 2 ) 1 · ·
2. (d). R = + [(R1 + R2)2 + 4 R3
2 2 R
(R1 + R2)]1/2 .....(a)
R1 = 1W, R2 = 0. R3 = 2W. ......(b) d
From eqs. (a) and (b)
1 1 R R
1
R= + [1 + 4 × 2 × 1]1/2 = [1 + 3] = 2W.
2 2 2
1 1 1 1 3
3. (b). Since Q is connected in parallel the net resistance i.e. = + + =
becomes R/2, so the current I = 2V/R, double the value. R eq 2R 2R 2R 2R
4. (b). Since there is no current in edcb part, the p.d. across
be should be 2V. Let current in 500 W is I, then same current 2
Þ Req = R
flows through X (think). Therefore, for loop abefa, 3
12 = I (500) + IX 7. (c).
or 12 = I (500) + 2 (\ IX = 2 volt) b
Thus I = (1/50) A or from IX = 2,
X = 2 × 50 = 100 W. r r
5. (b). Let l1 be the initial length of the wire. Then the new a c
length will be A · r · B

110 11 r r
l2 = l = l
100 1 10 1
Since, the volume remains constant d

11 Imagine, A being pulled on the left side, then abcd becomes


A1l1 = A2l2 or A1 /A2 = l2/l1 = a balanced wheatstone bridge Fig. The arm bd can be
10
(where A1 and A2 are initial and final area of cross–section ignored. Then resistance between A, B becomes = r.
of the wire). 1 1 1 1
If R1 and R2 are the initial and final resistances, then
i.e. = + = Þ Req = r
R eq 2r 2r r
2
R1 l1 A 2 10 10 æ 10 ö 8. (d) R = 91´ 102 » 9.1 kW.
= = ´ =ç ÷
R 2 l 2 A1 11 11 è 11 ø
9. (d) Spacific resistance doesn't depend upon length and area.
2
R 2 æ 11ö
or =ç ÷ 10. (b). The diagram can be redrawn as shown in fig.
R1 è 10 ø
Now, percentage change in resistance is
DR æ R 2 - R1 ö
R1 × 100 = çè R ÷ø × 100
1

éæ 11 ö 2 ù
= êçè 10 ÷ø - 1ú × 100 = 21%
êë úû
t.me/Ebooks_Encyclopedia27. t.me/Magazines4all

DPP/ P 36 101
3W
2´ R
12. (c). R= 2+2 + Þ 2R + R2 = 8 + 4R + 2R
2+R
2W 4W
X A C B Y 4 ± 16 + 32 2 ± 2 3
Þ R2 – 4R – 8 = 0 Þ R = =
2
4W 8W R cannot be negative, hence R = 2 ± 2 3 = 5.46 W

6W
V2
13. (c). P = . If resistance of heater coil is R, then resistance
+ – R
2V
3W R
The effective resistance RAC between A and C of parallel combination of two halves will be
4
1 1 1 3 3
= + = P1 P2
R AC 2 4 4 \RAC = 4 ohm So = =
R/4 1
=
P2 P1 R 4
The effective resistance RCB between C and B
1 1 3 8 60 ´ 8 ´ 301
RCB = + = \RCB = ohm. 14. (c). Total kWh consumed = = 14.4
4 8 8 3 1000
Hence cost = 14.4 ´ 1.25 = ` 18
4 8
Now, RACB = RAC + RCB = + = 4ohm.
3 3 15. (d). Since all bulbs are identical they have the same
Corresponding to points X and Y, the resistances 3 ohm, 4 resistances. The current I flowing through 1 branches at
ohm and 6 ohm are in parallel, hence effective resistance A. So current in 2 and 3, as well as in 4 will be less than I.
RXY is The current through 5 is also I. Thus 1 and 5 glow equally
brightly and more than 2, 3 or 4.
1 1 1 1 4+3+ 2 9
= + + = = 16. (b). Let R1 and R2 be the resistances of the coils, V the
R XY 3 4 6 12 12 supply voltage, Q the heat required to boil the water.
12 4 Heat produced by first coil of resistance R1 in time t1
\ RXY = = ohm.
9 3 V 2 t1 V 2 ´ 6 ´ 60
(= 6 min) = Q = = cal ......(a)
4 2 JR1 4.2R1
Total resistance R of the circuit = + = 2W.
3 3 Heat produced in second coil of resistance R2 in time
t2 (= 8 min)
2
Current in the circuit = =1A
2 V 2 t1 V 2 ´ 6 ´ 60
=Q= = .....(b)
Power dissipated in the circuit = i2 R = 1 × 2 = 2 watts JR 2 4.2R 2
Potential difference between X and
Equating (a) and (b), we get
4 4
Y = i × RXY = 1 × = V.. 6 8 R
3 3 = i.e. 2 = 8 = 4
R2 R2 R1 6 3
4
\ Potential difference across 3 ohm resistor = V.. 4
3 or R2 = R .....(c)
3 1
4/3 4 (i) When the two heating coils are in series, the effective
Current in 3 ohm resistor = = = 0.44 amp.
3 9 resistance is
( 30 + 30 ) 30 60 ´ 30 4 7
11. (c). Requivalent = 30 + 30 + 30 = = 20 W R' = R1 + R2 = R1 + .R = R .
( ) 90 3 1 3 1
with two coils in series, let the kettle take t' time to boil.
V 2 1
\ i= = = ampere V2 t ' V2 t '
R 20 10 Q= = .....(d)
JR ' æ7 ö
4.2 ´ ç R1 ÷
è3 ø
t.me/Ebooks_Encyclopedia27. t.me/Magazines4all

EBD_7156
102 DPP/ P 36
t'
Comparing (a) and (d), we get = 6 × 60
(7 / 3)
24V
7 21V
or t' = × 6 × 60 sec = 14 min.
3
(ii) When the two heating coils are in parallel, the effective
resistance is,
3V
æ4 ö
R1 ç R1 ÷
R1 R 2 è3 ø 4
R ¢¢ = = = R1
R1 + R 2 æ4 ö 7
R1 + ç R1 ÷
è3 ø 22
P1 v2 R L æv ö æ 9ö
= 1 Þç 1÷ =ç ÷ =9
In parallel arrangement of heating coils, let t" be the time P2 R L v 22 è v2 ø è 3ø
taken by kettle to boil, so
Sol. (18-20).
V2t " V 2 t '' 12V
Q= = .....(5) I= = 2A
JR " æ4 ö (1 + 5) W
4.2 ´ ç R1 ÷
è7 ø
1W 12 V
Comparing (a) and (5), we get

t" 4
= 6 × 60 or t" = × 6 × 60 sec = 3.43 min.
(4 / 7) 7
17. (c).

7.5amp
24V 5W
v 15V 2kW é 1.5 ´ 6 ù
I= R ê R p = 7.5 ú Þ Rate of chemical energy conversion = EI = 12 × 2 = 24 W
eq ë û and P (in battery) = I 2 r = 4 W
Also, P (in resistor) = I 2 r = 20 W
18. (a) 19. (c) 20. (a)
9V 1.2kW
21. (d) Resistivity of a semiconductor decreases with the
temperature. The atoms of a semiconductor vibrate with
larger amplitudes at higher temperatures thereby
increasing it's conductivity not resistivity.
240 60 22. (d) It is quite clear that in a battery circuit, the point of
I= Þ = 7.5mA lowest potential is the negative terminal of the battery
32 8
and the current flows from higher potential to lower
(1) Currrent I is 7.5mA potential.
(2) Voltage drop across RL is 9 volt 23. (b) The temperature co-efficient of resistance for metal is
positive and that for semiconductor is negative.
P1 v12 R 2 225 ´ 6 In metals free electrons (negative charge) are charge
(3) P = R 2
Þ
2 ´ 81
= 16.66
carriers while in p-type semiconductors, holes (positive
2 1 v2
charge) are majority charge carriers.
(4) After intercharging the two resistor R1 and R2

v 24
I= = ´ 7 = 3.5mA
R eq (48)
t.me/Ebooks_Encyclopedia27. t.me/Magazines4all

DPP/ P 37 105

DAILY PRACTICE
PROBLEMS
PHYSICS
SOLUTIONS 37
(1) (d). Let R is resistance of the voltmeter. The effective
V 25
resistance across points A, B is (7) (a). \ i = = A
R 1000
60 ´ R Let R' be the required resistance to be connected in
r= ..............(1)
60 + R series with voltmeter.
The current in the circuit is I = 12/(50 + r)
V'
The p.d. across AB points is V = Ir So i =
R+R'
12
or 6 = ×r or 50 + r = 2r 25
50 + r Here V' = 250, R = 1000 W and i = A
or r = 50 W ..............(2) 1000
using it in (1), 25 250
\ = Þ R' = 9000 W.
60 1000 1000 + R '
we get 50 =
60 + R (8) (d). The potential difference between A and B in the absence
300 + 5R = 6R of voltmeter = 2 volt.
K
or R = 300 W + – ()
VAB

G G G 4V
R1 R2
(2) (c). S = = = W B
n - 1 100 - 1 99 A 4kW
4kW
ig 10
(3) (b). S = G= × 99 = 111W
i - ig 100 - 10
4kW
80 ´ 80 V
(4) (c). R = 20 + = 60W RV
80 + 80
V 2 1
i= = = amp. Current flowing in the circuit
R 60 30
E E
1 I= =
\ V = iR' = × 40 = 1.33 volt. RR R2 + R '
30 R2 + 1 V
R1 + R V
(5) (a). According to Kirchhoff's first law
At junection A, iAB = 2 + 2 = 4A
At junection B, iAB = iBC – 1 = 3A 4 2
I= = ampere
4´4 3
2A 4+
1A 4+4
1.3 A Potential difference measured by voltmeter
A B
C 2 4
2A V'AB = IR' = ×2=
i 3 3
At junection C, i = iBC – 1.3 = 3 – 1.3 = 17 amp Error in the reading of voltmeter
(6) (b). The current required for a full-scale deflection of the
4 2
galvanometer is = VAB – V'AB = 2 – = volt
i = 4.0 x 10-4 x 25 = 10-2 A 3 3
Let a resistance R W is to be connected in series 2
The error in voltmeter reading for 2 volt p.d. = volt
V 3
Then by the ohm's law, we have i =
G+R The error in voltmeter reading for 1 volt p.d.
Here G = 50 W, V = 2.5 V and i = 10-2 A 2 1 1
´ = volt
=
V 2.5 3 2 3
\ G+R= = -2 = 250
i 10 the error in voltmeter reading for 100 volt p.d.
Þ R = 250 – G = 250 – 50 = 200W. 100
= = 33.3% volt
3
t.me/Ebooks_Encyclopedia27. t.me/Magazines4all

EBD_7156
106 DPP/ P 37
(9) (d). E = V + I r = I R + I r 13. (b) Cells area joined in parallel when internal resistance is
Þ E = 0.25 x 10 + 0.25 × r higher then a external resistance. (R << r)
In second stage
E
Þ E = 0.5 × 4 + 0.5 r i=
r
Subtracting eq. (b) from eq. (a) R+
n
2.5 + 0.25 r – 2.0 – 0.5 r = 0
0.5 = 0.25 r E
14. (b). Current in the ammeter I =
0.5 é R 'ù
R '+ r ê1 + ú
r= = 2W. ë Rû
0.25
(10) (d) Suppose current through different paths of the circuit On increasing the value of R, the denominator will
is as follows. decrease and consequently the value of I will increase.
15. (a)
28 W 54 W I/6
I/3
I I/6

1 6V 2 I/3
I3 I/6 I/3
I/3 I/3
I/3
8V 12 V
I/6 I/3
After applying KVL for loop (1) and loop (2) I
I/6
1
We get 28i1 = – 6 – 8 Þ i1 = - A E
2
Let ABCDEFGH be skeleton cube formed of twelve
1 equal wires each of resistance R. Let a battery of e.m.f.
and 54i2 = – 6 – 12 Þ i2 = - A
3 E be connected across A and G. Let the total current
entering at the corner A and leaving the diagonally
5
Hence i3 = i1 + i2 = - A opposite corner G be I. By symmetry the distribution
6 of currents in wires of cube, according to Kirchoff's I st
law is shown in fig. ApplyingKirchoff's IInd law to mesh
5X + 2 × 10
(11) (d) VAB = 4 = Þ = 20 W ADCGEA, we get
X + 10
1 1 1
12. (b) The circuit can be simplified as follows – R– R– R+E=0
3 6 3
B C
30 W 5
i1 or E = IR ......(a)
i3 i3 6
A D If RAB is equivalent resistance between comers A and B,
40 W 40 V then from Ohm’s law comparing (a) and (b), we get
i2
F E 5
IRAB = IR
40 W 80 V 6
16. (d).
Applying KCL at junction A (y – z)
H
i3 = i1 + i2 ...(i) G
Applying Kirchoff's voltage law for the loop ABCDA y
I = x + 2y z (y – z)
– 30i1 – 40i3 + 40 = 0 A 2 (y–z)
Þ – 30i1 – 40(i1 + i2) + 40 = 0 y D
Þ 7i + 4i2 = 0 ...(ii) x z
Applying Kirchoff's voltage law for the loop ADEFA. E
(y – z)
– 40i2 – 40i3 + 80 + 4 = 0 y E F
Þ – 40i2 – 40(i1 + i2) = – 120
Þ i2 + 2i2 = 3 ...(iii) (y – z)
On solving equation (ii) and (iii) i1 = – 0.4 A. x + 2y B y C
t.me/Ebooks_Encyclopedia27. t.me/Magazines4all

DPP/ P 37 107
Let I = x + 2y current enter at point A, when a battery of The distribution of currents, keeping in mind symmetry
e.m.f. E and no internal resistance is connected across edge condition, is shown in fig.
AB. The edges AD and AH are symmetrically connected Let R (= 2W) be the resistance of each wire.
to A, therefore they will carry equal currents. The Applying Kirchoff's II law to mesh DGFC, we get
distribution of currents according to Kirchoff's Ist law is æ1 ö
shown in fig. çè - I1÷ø R + (I – 2I1)
2
If RAB is equivalent resistance, then from Ohm's law,
E = RAB I = RAB (x + 2y) ....(a) æ1 ö
and from Kirchoff's law applied to mesh containing AB R + çè - I1÷ø R – I1 R = 0
2
and cell E is
Rx=E .....(b) æ1 ö
(since R is resistance of each wire) or 2 çè - I1÷ø + (I – 2I1) – I1 = 0
2
Applying Kirchoff's II law to mesh AHEB
yR + zR + yR – xR = 0 or x – 2y – z = 0 .....(c) 2
or 2I – 5I1 = 0 or I1 = I ...... (b)
Applying Kirchoff's II law to mesh DGFC 5
nd
Applying Kirchoff's II law to external circuit AHEBE',
(y – z) R + 2 (y – z) R – zR = 0
or 4(y – z) –z = 0 or 4y = 5 z .....(d) we get
i.e. z = (4/5) y 1 1
.....(E) R + I1 R + R = E
2 2
Substituting this value in (c), we get
2
4 IR + IR = E' [Using (b)]
x – 2y – y = 0 5
5
7
14 5 or IR = E ......(c)
or y = x i.e. y = x 5
5 14 Comparing (a) and (c), wet get
Substituting value of y in (a), we get
7 7 7
æ 10 ö RAB I = IR i.e. RAB = R = × 2 = 2.8 W
E = RAB çè x + x÷ø 5 5 5
14 18. (c). In the first case I = E/(r + R) and in the second case
24 I' = E/(r + R/2) = 2E/(2r + R)
E = RAB x= R. x Using E = I(r + R), we get
14
æ 2r + 2R ö æ R ö
24 7 I' = I çè ÷ø = I çè 1 + ÷
RAB = R \ RAB = R. 2r + R 2r + R ø
14 12
17. (a). Thus the term in bracket is greater than 1 but less than
H (I/2-I1) 2. Thus 2I > I' > I
G 19. (b). Let R be the combined resistance of galvanometer and
I/2 an unknown resistance and r the internal resistance of
I I1 I/2-I1 I-2I1
A
each battery. When the batteries, each of e.m.f. E are
y D connected in series, the net e.m.f. = 2E and net internal
I1
resistance = 2r
E
E
I/2-I1 2E 2 ´ 15
F \ Current i1 = or 1.0 =
R + 2r R + 2r
I/2 I/2-I1 \ R + 2r = 3.0. ...(i)
· When the batteries are connected in parallel, the e.m.f.
I B I/2 C
remains E and net internal resistance becomes r/2. therefore
Let a battery of e.m.f. E is applied between points A and B. E 2E
Current i2 = =
Let a current I, enter through point A. r 2R + r
R+
If RAB is equivalent resistance between points A and B, 2
then from Ohm's law
RAB I = E 2E 2 ´ 15
\ 2R + r = = = 5.0 ...(i)
i2 0.6
Solving (i) and (ii), we get r = 1/3 W.
t.me/Ebooks_Encyclopedia27. t.me/Magazines4all

EBD_7156
108 DPP/ P 37
20. (a). The circuit with current distribution is shown in fig. 21. (a). After full charging, the steady current in the condenser
is zero, hence no current will flow in 4W resistance.
C
E 6 6
I= = = =1.5 A
i R+R' æ 2 ´ 3 ö 28 + 12
28 + ç
F è 2 + 3 ÷ø
G
100W Let current flowing in 2W resistance is I1
H
E i1 300W \ 2W and 3W resistance are connected in parallel
200W \ 2I1 = (1.5 – I1) x 3
D 5I1 = 4.5
i - i1
i I1 = 0.9 amp.
D
A B 24. (d). Ig
i G
110V
Applying Kirchoff's second law to the loop DEFGHID,
R
we have i1 × 100 – (i – i1) × 200 = 0
I–Ig
300 i1 – 200 i = 0 ......(1)
Now applying Kirchoff's second law to loop ADIHGCBA, For Ammeter IgG = (I – Ig) R
50 × 10–6 × 100 = 5 × 10–3 × (R) Þ R » 1W
we have. (i – i1) 200 + i × 300 = 110 For voltmeter Ig (R + G) = V
500i – 200 i1 = 110 .....(2) 50 mA (R + G) = 10V Þ R + G = 200 kW Þ R » 200kW
Solving eqs. (1) and (2), we get 25. (a) Potential at A = 6V
VA – VC = 4
3 1 Þ VC = 2V
i= amp and i1 = amp.
10 5 VAD VAC AD 4 2
26. (d) = = = = ;
VAB VAB AB 6 3
1
Current in 100 ohm resistance i1 = amp.
5 200
AD = cm.
3
1 27. (a) D is balance point, hence no current
Current in 200 ohm resistance i – i1 =
10 28. (a) Voltameter measures current indirectly in terms of mass
of ions deposited and electrochemical equivalent of
3
Current in 300 ohm resistance i = amp. æ mö
10 the substance çè I = ÷ø . Since value of m and Z are
Zt
Potential difference between A and C
measured to 3rd decimal place and 5th decimal place
= Potential difference across 100 ohm respectively. The relative error in the emasurement of
resistance current by voltmeter will be very small as compared to
or potential difference across 200 ohm resistance that when measured by ammeter directly.
29. (c) The e.m.f. of a dry cell is dependent upon the electrode
\ VA – VC = current × resistance
potential of cathode and anode which in turn is
1 dependent upon the reaction involved as well as
= i1 × 100 = × 100 = 20 volt. concentration of the electrolyte. It has nothing to do
5
with size of the cell.
Potential difference between C and B is given by So, statement-1 is false and statement-2 is true.
30. (d) V = E – ir = 4 – 2 × 2 = 0, During charging V > E.
3
VC – VB = i × 300 = × 300 = 90 volt.
10
t.me/Ebooks_Encyclopedia27. t.me/Magazines4all

108 DPP/ P 38
DAILY PRACTICE
PROBLEMS
PHYSICS
SOLUTIONS 38
e R 9 ´ 18
1. (a) Potential gradient x = . RAB = = 6W
( R + Rh + r ) L 27

0.2 ´10-3 ir 0.1 ´ 10-7


(c) Potential gradient ( x) =
2 R = = 10-2 V / m
Þ = ´ Þ R = 4.9W. 10. -6
10-2 ( R + 490 + 0 ) 1 A 10
2. (c) Let S be larger and R be smaller resistance connected æ 100 - l ö
in two gaps of meter bridge. 11. (d) S = ç ÷ .R
è l ø
æ 100 - l ö 100 - 20
\S = ç ÷R = R = 4R ......(i) æ 100 - l ö
è l ø 20 Initially, 30 = ç ÷ ´10 Þ l = 25cm
è l ø
When 15W resistance is added to resistance R, then
æ 100 - l ö
æ 100 - 40 ö 6 Finally, 10 = ç ÷ ´ 30 Þ l = 75cm
S=ç ÷ ( R + 15) = ( R + 15 ) .......(ii) è l ø
è 40 ø 4
So, shift =50cm.
From equations (i) and (ii) R = 9W
E1 l1 + l2 (6 + 2) 2
æ l1 ö æ 240 ö 12. (d) = = =
E2 l1 - l2 (6 - 2) 1
3. (b) r = R ç l - 1÷ = 2 ç 120 - 1÷ = 2W
è 2 ø è ø
E1 l1 + l2 58 + 29 3
4. (a) Potential difference per unit length 13. (c) = = =
E2 l1 - l2 58 - 29 1
V 2
= = = 0.5V / m 14. (d)
L 4
e R 2 10
e R 5 5 E= ´l = ´ ´ 0.4 = 0.16 V
5. (d) E = . ´ l Þ 0.4 = ´ ´l ( R + Rh + r ) L (10 + 40 + 0) 1
( R + Rh + r ) L ( 5 + 45 + 0 ) 10
Þ l = 8m

ïì ( 25 + 5) ïü
6. (d) Current through 2W = 1.4 í 10 + 2 + 25 + 5 ý = 1A
ïî ( ) ( ) ïþ
15. (c)
7. (c) Post office box is based on the principle of
Wheatstone’s bridge.
æl ö æ 150 ö
8. (b) Using r = R ç 1 - 1÷ = 2 ç - 1 ÷ = 1W
è l2 ø è 100 ø
9. (a) Since the given bridge is balanced, hence there will be Resistance of the part AC
no current through 9W resistance. This resistance has
R AC = 0.1´ 40 = 4W and RCB = 0.1´ 60 = 6W
no effect and must be ignored in the calculations.
X 4
9W In balanced condition = Þ X = 4W
6 6
5W 4W Equivalent resistance Req = 5W

1.4 A 5
so current drawn from battery i = = 1A .
5

æ l1 - l2 ö æ 55 - 50 ö
10 W 8W 16. (a) r = ç l ÷ ´ R ¢ Þ r = çè 50 ÷ø ´ 10 = 1W
è 2 ø
14 W
t.me/Ebooks_Encyclopedia27. t.me/Magazines4all

EBD_7156
DPP/ P 38 109
17. (c) Potential gradient If P is slightly icnreased, potential of C will decrease.
V e R Hence current will from A to C.
x= =
L ( R + Rh + r ) L If Q is slightly increased, potential of C will increase.
Hence current will flow from C to A.
2.2
Þ 2.2 ´ 10-3 = ´1 Þ R ' = 990W 25-27
(10 + Rh )
We have

E 2.4 ´ 10-3 V 10.0V


18. (a) E = xl = irl Þ i = = = 4 ´ 10-4 A Rs = - Rc = - 20.0W = 9980W
rl 1.2 ´ 5 Ifs 0.00100A
19. ( b) Give circuit is a balanced Wheaststone bridge circuit, At full-scale deflection, Vab = 10.0V, voltage across the
hence it can be redrawn as follows meter is 0.0200 V, voltage across Rs is 9.98 V, and current
through the voltmeter is 0.00100 A. In this case most of the
voltage appears across the series resistor.
The equivalent meter resistance is Req = 20.0 W + 9980 W
= 10,000W . Such a meter is described as a "1,000 ohms-
per-volt meter" referring to the ratio of resistance to full-
scale deflection. In normal operation the current through
the circuit element being measured is much greater than
0.00100 A, and the resistance between points a and b in
the circuit is much less than 10,000W. So the voltmeter
draws off only a small fraction of the current and disturbs,
only slightly the circuit being measured.
12 ´ 6 25. (d), 26. (c), 27. (b)
RAB = (12 + 6 ) = 4 W.
28. (d) The resistance of the galvanometer is fixed. In meter
20. (a) Balancing length is independent of the cross sectional bridge experiments, to protect the galvanometer from
area of the wire. a high current, high resistance is connected to the
21. (a) In meter bridge experiment, it is assumed that the galvanometer in order to protect it from damage.
resistance of the L shaped plate is negligible, but 1
actually it is not so. The error created due to this is 29. (a) Sensitivity µ µ (Length of wire)
Potential gradiant
called, end error. To remove this the resistance box and
the unknown resisance must be interchanged and then 30. (a) If either the e.m.f. of the driver cell or potential
the mean reading must be taken. difference across the whole potentiometer wire is lesser
22. (a) Ammeter is always connected in series with circuit. than the e.m.f. of the experimental cell, then balance
point will not obtained.
23. (a) In balanced Wheastone bridge, the arms of
galvanometer and cell can be interchanged without
affecting the balance of the bridge.

24. (d)
W W

W W
t.me/Ebooks_Encyclopedia27. t.me/Magazines4all

110 DPP/ P 39
DAILY PRACTICE
PROBLEMS
PHYSICS
SOLUTIONS 39
(1) (a) We know magnetic field due to a long straight current Let T be the period of rotation of the rod. Then
carrying wire
q qw 1 colulomb ´ 104 p / sec
µ0 i 4p ´ 10 ´ 3 -7 i= = =
B= = T 2p 2p
2pr 2p ´ 50 ´ 10-3 3
= 5 × 10 amp.
(Note that m0 = 4p × 10–7 in SI system) Now, a = 0.6 m, x = 0.8 m and m0 = 4p x 10–7 V-s/A-m.
= 1.20 × 10–5 Tesla = 0.12 G. Substituting these values in eq. (i) we get
[As 1 Gauss = 10–4 Tesla]
(2) (c) The magnetic induction produced due to a current (4p ´ 10 -7 V - s / A - m)(5 ´ 103 A)(0.6m) 2
B=
carrying arc at its centre of curvature is 2(0.36 + 0.64)3/2 m3
m 0 ia = 0.36p × 10–3 = 1.13 × 10–3 tesla
B = 4p r .......... (a) In the second case the current remains the same because
the rotating charge and the angular frequency are the
(subtending angle a at the centre of curvature) same. However, the radius of the loop becomes half (a
µ0ip p m0 i = 0.3 m) and the distance x is now 0.4 m.
Þ B= ´ =
4pr 4 16r µ0 ia 2
\ B=
2(a 2 + x 2 )3/2
O Q i P
R y
(3) (a) r (4 p ´ 10-7 V - s / A - m)(5 ´ 103 A)(0.3m) 2
90° x =
2(0.09 + 0.16)3/2 m3
S i T z
ur ur ur 4p ´ 10-7 ´ 5 ´ 103 ´ 0.09
BO = BQRS + BST . = tesla
2 ´ 0.25 ´ 0.5
ur ur 3 µ i ˆ ur µ i = 0.72 × 10–3 p = 2.26 × 10–3 tesla.
BPQ = zero, BQRS = ´ 0 k, BST = 0 kˆ
4 2r 4pr (6) (a) The magnetic field at the centre of a current carrying
coil having n turns is given by
ur µ i 3 3µ0i ˆ µ0i é 3p ù ˆ
Þ BO = 0 kˆ + k= + 1ú k m 0 ni
4 pr 4 2r 4 pr ëê 2 û B= N/A.m
ur ur ur 2r
(4) (a). BO = BPSR + BPQR .... (a) where i, is the current in the coil and r is the radius of
ur the coil.
m i é 2p - 2f ù m 0 i
B PSR = 0 ê = ˆ
[p - f] (-k) .... (b) Here i = 0.1 A, n = 1000 and r = 0.1 m.
4p ë r úû 2p r
(4p ´ 10-7 ) ´ 1000 ´ 0.1
ur m i 2sin f ˆ m i 2sin f m0 i \ B= = 6.28 × 10-4 N/A.m
BPQR = 0 . (-k) = 0 . = tan f (-k)ˆ 2 ´ 0.1
4p OQ 4p r cos f 2pr
(7) (a). The two coils are perpendicular to each other. Coil 1
.... (c) produces field along X axis and coil 2 produces field
From eqs. (a), (b) and (c)
along Y axis. Thus the resultant field will be-
ur m0 i m i
B= ˆ + 0 tan f (-k)
[p - f] (-k) ˆ B= B12 + B22 making an angle
2p r 2p r
æB ö
m0 i
ˆ q = tan -1 ç 2 ÷ with x axis
= [p – f + tan f] (-k) è B1 ø
2p r
(5) (a). The rotating rod is a current-loop whose radius a = 0.6m. µ0 NI
As B1 = B2 =
The magnetic field due to this current-loop at a point 2a
on its axis at a distance x from its centre is given by
æ µ0 NI ö µ0 NI
µ0 ia 2 Þ B= 2 = çè ÷= and q = 45º.
B= ...(i) 2a ø 2a
2(a 2 + x 2 )3/2
t.me/Ebooks_Encyclopedia27. t.me/Magazines4all

EBD_7156
DPP/ P 39 111
(8) (d) Applying ampere's law at P, Q and R respectively, we (12) (a). The magnetic field inside (near centre) a current carrying
find that there is no current enclosed by the circle of P. solenoid having n turns per unit length is given by
So magnetic induction at P is zero while that at Q and B = m0 ni newton/(ampere-meter),
R is non- zero. where i (ampere) is the current in the solenoid and
(9) (a). For a current carrying coil m0 = 4p × 10–7 newton/ampere2.
µ0 i Here n = 500/0.40 = 1250 per meter, i = 1.0 amp.
B= at centre and force on a current carrying \ B = (4 × 3.14 × 10-7) × 1250 × 1.0
2R
= 15.7 × 10-4 newton/(ampere-meter) = 15.7 gauss.
conductor ie
m 0 ni
µ i2 l (13) (b) We know, Bend =
F=ilBÞF= 0 2
2R
500
[µ0 ][A 2 ][L] Here n = = 2500/metre,
0.2
Þ [MLT–2] =
[L]
Þ [m0] = [MLT–2 A–2] 2Bend 2 ´ 8.71 ´ 10-6
\ i= =
(10) (c) By Biat Savart Law, µ0 n 4p ´ 10-7 ´ 2500
m 0 idl sin q
dB = 17.42 ´10-3 0.01742
4p r 2 = = amp amp.
p p
When q = 90º, then sin 90º = 1 = maximum
3/ 2
m 0i dl Bcentre æ x2 ö 1
\ dB = = maximum (14) (b) = ç 1´ 2 ÷ , also Baxis = Bcentre
4pr 2 Baxis ç R ÷ 8
è ø
(11) (a) The magnitude of the magnetic field at the centroid O
of the triangle due to a side PQ (say) is æ x2 ö
3/ 2
æ x2 ö
1/ 2
8
µ0 i Þ = ç 1´ 2 ÷ Þ 2 = çç 1´ 2 ÷÷
(sin f1 + sin f2 ) 1 ç R ÷
è ø è R ø
4p r
Where r is the perpendicular distance of PQ from O, x2 x2
and f1 , f2 th e angles as shown. The field is Þ 4=1+ Þ3= Þ x2 = 3R2 Þ 3R2
perpendicular to the plane of paper and is directed into R2 R2
plane of paper. Since the magnetic field due to each of
Þx= 3R
the three sides is the same in magnitude and direction,
the magnitude of the resultant field at O is Ni
Q
(15) (a) B0 = m0
2 pR

4p ´ 10-7 ´ 500 ´ 0.5


i = = 5 × 10–4 tesla
f2 2p ´ 0.1
(

f1 ( O
(16) (c) At P : Bnet = B12 + B22
P R 2 2
µ0 i æ μ 0 2i1 ö æ μ 0 2i2 ö
B= 3 (sin f1 + sin f2 ) = ç ÷ +ç ÷
4p r è 4π a ø è 4π a ø
Here i = 1 ampere, f1 = f2 = 60º B1
l l 1 P B2
and r = cot 60º = ´
2 2 3
a
and l is the side of the triangle (= 4.5 × 10–2 meter). A
i2
3 ´10 -7 ´ 1.0 æ 3 3ö
çç + ÷ i1
\B= 2 ÷ø C
æ1 -2 ö æ 1 ö è 2 D
ç ´ 4.5 ´10 ÷ ´ ç ÷ B O
è 2 ø è 3ø

3 ´ 10-7 ´ 2 ´ 3
(
μ0 2 2
)
1/ 2
= -2 = 4.0 × 10–5 weber/m2. = i1 + i2
4.5 ´ 10 2πa
t.me/Ebooks_Encyclopedia27. t.me/Magazines4all

112 DPP/ P 39
(17) (d) (22) (b). The arrangement is shown in fig.
(18) (b) Current distribution in the network is as shown.
I1 I2
Now, consider the pair of wires AB and GH. As current
in these wires produce equal but opposite magnetic
fields at centre O of the cube, resultant field due to the X Y
pair is zero. I P A

B 6 C
–I R
–I 6
–I F 6 G
–I 6
3
–I
O –I I
3
A 3 D –I
3
I –I –I
–I 6 6 The magnetic field at a point P in between the two
3
E –I H wires is
We can see five such more pairs namely :
6 r r r
B = B1 + B2 . The field B1 (due to current I1) points
(i) AE, CG
(ii) AD, FI down ward while B2 (due to current I2) points upwards.
Thus field at point P is-
(iiii) BC, EH
(iv) EF, DC m 0 é I1 I ù
B= ê - 2 ú in to the plane of paper..
(v) BF, OH 2p ë x R - x û
Magnetic field due to each of these pairs is zero. At x = R/2,
Therefore, resultant magnetic field at centre O is zero.
(19) (a) Magnetic field inside a solid cylinder of current is m 0 (I1 - I2 )
B= into the plane of paper, (if I1 > I2)
pR
µ0ir
Binside = m 0 (I 2 - I1 )
2pR 2 or B= out of the plane of paper (if I2 > I1)
pR
R (23) (d) (i) Fields due to both coils are in the same direction
µ0 i
Þ B0 = 2 (as per given µ0 N1I1 µ0 N 2 I2
2 pR 2 Þ B= +
2R1 2R 2
information)
If I1 = I2 = I, N1 = N2 = N,
4B0 pR
Þ i= µ0 NI(R1 + R 2 )
µ0 B=
2R1R 2
Magnetic field outside a solid cylinder of current is
(ii) Fields due to the two coils are in opposite direction,
µ i
Boutside = 0 µ0 N1I1 µ0 N 2 I2
2 pr Þ B= -
2R1 2R 2
æ 4B pR ö
µ0 ç 0 ÷ If I1 = I2 = I, N1 = N2 = N,
è µ0 ø
Þ Boutside at a distance 2R = = B0 µ0 NI(R 2 - R1 )
2p(2R) B=
(20) (d) As per sense of transversal, 2R1R 2
icrossing = I1 – I2 – I3 m0 I
r r (24) (a). For circular coil B1 =
2r
By Ampere's law, Ñ ò B.de = µ0 icrossing Circumference of the coil = 2pr = L.
r r Thus B1 = pm0 I/L = 3.14 m0 I/L
Þ Ñò B.de = µ0 (I2 – I1 – I3)
For square loop B2 = 2 2 m0 I/L = 3.60 m0 I/L
l Thus B1 < B2.
(21) (a) l = (2pr) n or n= r r uur ur
2p r
25. (d) Since M P B \ Torque = M ´ B is zero.
m 0 ni m 0 il 26. (d) The field must be in + k$ direction.
B= =
2r 4p r 2 27. (a)
28. (b) The statements are independently correct.
4p´10-7 ´ 6.28 ´1 r uur ur r
or B= = 6.28 × 10-5 Tesla. 29. (d) t = m ´ B Þ t = 0 for q = 0°, 180°.
2 ´ 2 ´ p´ (0.10)2 30. (b)
t.me/Ebooks_Encyclopedia27. t.me/Magazines4all

EBD_7156
DPP/ P 40 113

DAILY PRACTICE
PROBLEMS
PHYSICS
SOLUTIONS 40
(1) (b) Ekp = eV, \ Ek = qV, region between PQ and RS. The particle moves in a
\ Ek µ q, \ V = constant circular path of radius r in the magnetic field. It can just
Ekp : Ekd : Eka : : 1 : 1 : 2. enter the region x > b for r ³ (b – a)

q 2 r 2 B2
(2) (c) EK = mv
2m Now r = ³ (b - a )
qB
q 2 qa2 mp
\ Ek µ =
2
´ q (b - a) B q (b - a) B
m mp E kp or v³ Þ vmin =
m m
4 1 (11) (a) From figure it is clear that
= ´ = E Ka = 8eV..
1 4
r b
KVe 10-7 ´ 3 ´1.66 ´10-19 q
(3) (a) B = = = 1.2 Tesla. q
r2 (2 ´10-10 )2
(4) (a) F = qvB sinq
q
æ 1ö
= 2 × 1.6 × 10–19 × 105 × 0.8 × çè ÷ø
2
d
= 1.28 × 10–14 N [Q charge on a particle = 2e]
r r r
(5) (b) The direction of F is along (V ´ B) which is towards
d p
the right. Thus the beam deflects to your right side. sin q = also r = qB
(6) (b) The particle is moving clockwise which shows that r
force on the particle is opposite to given by right hand Bqd
palm rule of fleming left hand rule. These two laws are \ sin q = p
used for positive charge. Here since laws are disobeyed,
we can say that charge is negative. (12) (a) For on wire Q due to wire P is
(7) (b) The point lies at the circumference hence it will come 2 ´ 30 ´ 10
-7
back after a time period T FP = 10 ´ ´ 0.1 = 6 ´ 10– 5 N (Towards left)
0.1
2p m Force on wire Q due to wire R is
T=
qB 2 ´ 20 ´ 10
-7
FR = 10 ´ ´ 0.1 = 20 ´ 10–5 (Towards right)
(8) (a) The magnetic force on a current carrying wire of length 0.02
L, placed in a magnetic field B at an angleq with the Hence Fnet = FR – FP = 14 ´ 10–5N = 14 ´ 10–4 N
field is given by (Towards right)
F = i l Bsinq. r r r
Here B = 5.0 × 10–4 N/A.m. i = 2.0 A, (13) (d) F = q (v ´ B)
l = 50 cm = 0.50 m,
q = 60º ˆi ˆj kˆ
F = 2.0 × 0.50 × (5.0 × 10–4) × sin 60º r r 3 2 0 = k̂ (–10 × 105) = (– k̂ 106)
v´B =
= 4.33 × 10–4 N 5
According to the flemings left - hand rule, this force 5 ´10 0 0
will act perpendicular to both the wire and the magnetic q = 2e = 2 × 1.6 x 10-19 = 3.2 × 10-19 Coulomb
field. r
F = 3.2 × 10–19 (– k̂ × 106)
m r
1 4 16 Þ F = – 3.2 × 10–13 k̂ .
(9) (a, c) rµÞ rH : rHe : ro = : : =1:2:2
q 1 1 2 \ |F| = 3.2 × 10–13 Coulomb.
Radius is smallest for H+, so it is deflected most. (14) (b)
r
(10) (b) In the figure, the z-axis points out of the paper, and the \ F = q ( vr × B ) = 2evB sin 90º
magnetic field is directed into the paper, existing in the or F = 2evB
t.me/Ebooks_Encyclopedia27. t.me/Magazines4all

114 DPP/ P 40
(15) (a) Force on side BC AND AD are equal but opposite so
their net will be zero. 14 ´ 106
\ V= = 7 × 106 Volt.
2
10 cm (18) (a) If electron beam passes undeflected in simultaneous
B C r r
15 cm electric and magnetic fields E and B velocity of beam
2A r
v much be mutually perpendicular and the required
FAB FCD speed v is given by-
A D E 1 ´ 104
2 cm v= = = 5 × 106 m/s.
B 2 ´ 10-3
-7 2 ´ 2 ´1 If electric field is removed, the electron traverses a
But FAB = 10 ´ -2
´ 15 ´10-2 = 3 ´ 10–6 N
2 ´10 mv2 mv
circular path of radius r given by = evB or r = .
2 ´ 2 ´1 r eB
and FCD = 10-7 ´ ´ 15 ´10 -2 = 0.5 ´ 10–6 N
( 2 ´10 )
-2 Here m = 9.1 × 10–31 kg, v = 5 × 106 m/s.
e = 1.6 × 10–19 coul and B = 2 × 10–3 weber/m2
Þ Fnet = FAB – FCD = 2.5 ´ 10–6 N
(9.1 ´ 10 -31 ) (5 ´ 106 )
= 25 ´ 10–7 N, towards the wire. \ r=
(16) (b) In order to make a proton circulate the earth along the (1.6 ´ 10-19 ) (2 ´10 -3 )
r = 1.43 × 10-2 m = 1.43 cm.
equator, the minimum magnetic field induction B
should be horizontal nad perpendicular to equator. (19) (a) For L length or wire, to balance,
The magnetic force provides the necessary centripetal Fmagnetic = mg Þ ILB = mg,
force. Therefore B = mg/IL = (m/L)g/I

mv2 mv 45 ´10-3 ´ 9.8


i.e. qv B = or B = = = 1.47 × 10-2 tesla.
r qr 30
Here m = 1.7 × 10 kg, v = 1.0 × 107 m/s
–27 = 147 Gauss.
q = e = 1.6 × 10-19 coulomb, r = 6.37 × 106 m (20) (b) According to Fleming's left hand rule, magnetic force
on electrons will be downward.
1.7 ´ 10-27 ´ 1.0 ´ 107
B= = 1.67 × 10–8 weber/m2. e– e – e – e – e– e – e – ×
1.6 ´ 10-19 ´ 6.37 ´ 106
e – e – e – e – e– e– e – ×
mv2 qBr
(21) (b) \ F = mg = Bil
(17) (c) We have F = qvB = or v =
r m or 1 × 9.8 = 0.98 × i × 1, Þ i = 10A.
(22) (b) When currents flow in two long, parallel wires in the
3.2 ´ 10-19 ´ 1.2 ´ 0.45
= = 2.6 × 107 m/s. same direction, the wires exert a force of attraction on
6.8 ´ 10-27 each other. The magnitude of this force acting per meter
length of the wires is given by
v
The frequency of rotation n = µ0 i1i 2 i1i 2
2 pr
F= = 2 × 10–7 N/m.
2p R R
2.6 ´ 107 Here i1 = 10 A, i2 = 15 A, R = 30 cm = 0.3 m
= = 9.2 × 106 sec–1.
2 ´ 3.14 ´ 0.45
10 ´ 15
Kinetic energy of a-particle, \ F = 2 × 10–7 = 1 × 10–4 N/m.
0.3
1
EK = × 6.8 × 10–27 × (2.6 × 107)2 \ Force on 5m length of the wire
2 = 5 × (1 × 10–4) = (5 × 10-4) = 5 × 10-4 N (attraction).
= 2.3 × 10–12 joule. (23) (d) The electron will pass undeviated if the electric force
and magnetic force are equal and opposite. Thus
2.3 ´ 10-12
= eVolt = 14 × 106 eV = 14 MeVolt. E.e. = Bev or B = E/v but E = V/d
1.6 ´ 10-19
If V is accelerating potential of a-particle, then Kinetic Therefore, B = V = 600
energy = qV v.d. 3 ´ 10-3 ´ 2 ´ 106
14 × 106 eVolt = 2eV (since charge on a-particle = 2e) \ B = 0.1 Wb/m2.
The direction of field is perpendicular to the plane of
paper vertically downward.
t.me/Ebooks_Encyclopedia27. t.me/Magazines4all

EBD_7156
DPP/ P 40 115
(24) (b) The component of velocity of the beam of protons, (25) (a), (26) (a), (27) (c).
parallel to the field direction r r r
F = q (v ´ B) = q (x 2 - y2 ) kˆ
= v cosq = 4 × 105 × cos 60º = 2 × 105 m/sec.
and the component of velocity of the proton beam at (28) (c) When two long parallel wires, are connected to a bat-
right angle to the direction of field tery in series. They carry currents in opposite direc-
tions, hence they repel each other.
= v sinq = 4 × 105 × sin 60º = 2 3 × 105 m/sec.
(29) (c) No net force will act on charged particle if
therefore,the radius of circular path = (mv sinq /Be) r r r r
F = q [ E + v ´ B] = 0
1.7 ´ 10-27 ´ 2 3 ´ 105 r r r
or r = = 12.26 × 10–3 metre Þ E = -v ´ B Þ v need not to be perpendicular to B
0.3 ´ 1.6 ´ 10-19
(30) (c) In this case we can not be sure about the absence of the
or r = 1.226 × 10–2 metre. magnetic field because if the electron moving parallel
Pitch of the Helix = v cosq x (2pm/Be) to the direction of magnetic field, the angle between
2 ´ 105 ´ 2 ´ 3.14 ´ 1.7 ´ 10-27 velocity and applied magnetic field is zero
\ Pitch = (F = 0). Then also electron passes without deflection.
0.3 ´ 1.6 ´ 10-19
Also F = evBsin q Þ F µ B .
= 44.5 × 10–3 m = 4.45 × 10–2 m.
t.me/Ebooks_Encyclopedia27. t.me/Magazines4all

DPP/ P 41 115

DAILY PRACTICE
PROBLEMS
PHYSICS
SOLUTIONS 41
22 t = BiNA = (4p × 10-7) × (500/0.4) × 3 × 0.4 × 10
1. (b) M = NiA = 20 ´ (4 ´ 10-2 ) 2 ´ 3 = 0.3 A - m2 × p (0.01)2
7
= 6p2 × 10-7 = 5.92 × 10-6 N.m.
2. (a) The magnetic moment of current carrying loop
14. (d) The equivalent magnetic moment is
M = niA = ni (pr2)
M = iA = ef (pr2)
Hence the work done in rotating it through 180°
W = MB(1 – cosq) = 2MB = 2(nipr2)B v
As f =
= 2 × (50 × 2 × 3.14 ×16 × 10–4) × 0.1 = 0.1J 2 pr

3. (b) NiAB ev 2 evr


q= Þ q µ N (Number of turns) \ M= pr =
C 2 pr 2
4. (d) t = MB sin q Þ t max = NiAB , (q = 90°) Cq
uur ur 15. (b) i = Þi µq
5. (c) In equilibrium angle between M and B is zero. It NAB
ur 16. (d) Initially for circular coil L = 2pr and M = i × pr2
happens, when plane of the coil is perpendicular to B

2
æ Lö iL2
B = i ´ pç ÷ = ...(i)
è 2p ø 4p
Finally for square coil
2
æ Lö iL2 ...(ii)
M '=i´ç ÷ =
è4ø 16

i L/4

6. (a) t = NBiA = 100 × 0.2 × 2 × (0.08 × 0.1) = 0.32 N × m pM


7. (c) t = NBiA = 100 × 0.5 × 1 × 400 × 10–4 = 2 N–m Solving equation (i) and (ii) M ¢ =
4
8. (a) t = NiAB sinq = 0 (Q q = 0°) 17. (b) M = iA = i ´ pR 2
9. (c) M = NiA Þ M µ A Þ M µ r2 (As l = 2pr Þ lµ r)
Þ M µ l2 Qw 1
also i = Þ M = QwR 2
10. (a) 2p 2
18. (a) t = NBiA sinq so the graph between t and q is a
11. (c) t max = NiAB = 1 ´ i ´ (pr 2 ) ´ B sinusoidal graph.
æ Lö 19. (d) Initial magnetic moment = m1 = iL2
çè 2pr = L, Þ r = ÷
2p ø I
i L M
2
æ L ö L2iB m2 = M 2
t max = pi ç ÷ B=
è 2p ø 4p
L/2
12. (b) L
13. (b) The magnetic field at the centre of the solenoid is M
m1 L L/2 L
B = m0 ni = (4p × 10-7) × (500/0.4) × 3 N/A.m. m1 = iL2
The torque acting on a current-carrying coil having N
After folding the loop, M = magnetic moment due to
turns (say), placed perpendicular to the axis at the centre
of the solenoid is- æLö iL2 μ1
each part = i i ç ÷ ´ L = =
è 2ø 2 2
t.me/Ebooks_Encyclopedia27. t.me/Magazines4all

EBD_7156
116 DPP/ P 41
μ μ1 r æ1 ö
Þ m2 = M 2 = 1 ´ 2 = A3 = ç ´ 10 ´ 10 ´ 10-4 ÷ iˆ m 2
2 2 è2 ø
20. (a) Magnetic moment vector,
21. (d) r r
m = iA = 10(0.01iˆ + 0.005jˆ + 0.005k)Am
ˆ 2
22. (a) Couple of force on loop S will be maximum because
for same perimeter the area of loop will be maximum = (0.1iˆ + 0.05ˆj + 0.05k)Am
ˆ 2
and magnetic moment of loop = i ×A. So, it will also
be maximum for loop S. c. Torque,
r
q nBA t = (0.1iˆ + 0.05jˆ + 0.05k)
ˆ ´ (2iˆ - 3jˆ + k)
ˆ
23. (b) Sensitivity S = =
i C ˆi ˆj kˆ
24. (b) t = mBsin q is zero for q = 0°, 180°.
= 0.1 0.05 0.05 = -0.1iˆ - 0.4kˆ Nm
25-27 =
2 -3 1

28. (a) Due to metallic frame the deflection is only due to


current in a coil and magnetic field, not due to vibration
in the strings. If string start oscillating, presence of
metallic frame in the field make these oscillations
damped.
29. (b) The torque on the coil in a magnetic field is given by t
= nIBA cos q
For radial field, the coil is set with its plane parallel to
the direction of the magnetic field B, then q = 0° and
cos q = 1 Þ Torque = nIBA (1) = nIBA (maximum).
25. (a) 26. (b) 27. (b) 30. (c) Loop will not oscillate if in unstable equilibrium
(a) The net force on a current carrying loop of any position.
arbitrary shape in a uniform magnetic field is zero.
r
Fnet = 0
(b) The given loop can be con sidered to be a
superposition of three loops as shown in figure. The
area vector of the three loops (1), (2) and (3) are
r æ1 ö
A1 = ç ´ 10 ´ 10 ´ 10-4 ÷ ˆjm 2
è2 ø
r æ1 ö
A 2 = ç ´ 10 ´ 10 ´ 10-4 ÷ kˆ m 2
è2 ø
t.me/Ebooks_Encyclopedia27. t.me/Magazines4all

DPP/ P 42 117

DAILY PRACTICE
PROBLEMS
PHYSICS
SOLUTIONS 42
1. (a). Force between magnetic poles in air is given by 8. (c)
m 0 m1 m 2 9. (d) Here, d = 10 cm = 0.1 m ,
F= ´ H = 0.4 gauss = 0.4 x 10-4 T, M = ?
4p r2
Neutral points in this case, lie on axial line of magnet,
Given that m1 = 50 Am, m2 = 100Am, such that
r = 10 cm = 0.1 m and
µ0 = permeability of air = 4 p × 10–7 Hm–1. m 0 2M
=H
4p d3
4 p ´ 10-7 50 ´ 100
\F= . = 50 × 10–3 N
4p 0.1 ´ 0.1 2M
\ 10-7 × = 0.4 × 10-4
2. (a) Strength of a magnetic field due to a pole of strength m (0.1)3
is given by
M = 0.2 A m2
1 m 10. (d). t = F × r = MB sin 30
H= .
4 p r2
l ´ mBsin 30
Given that m = 40 Am, r = 20 cm = 20 × 10–2 m. F=
r
1 40
\ H= ´ = 79.57 Am–1 25 ´ 10-2 ´ 24 ´ 0.25 1
4 p (20 ´ 10 -2 ) 2 = ´ = 6.25 N
-2 2
Now, magnetic induction at the same point : 12 ´ 10
B = µ0 H = 4p × 10–7 × 79.57 = 10–4 wb/m2 m 0 2M
3. (c) Couple acting on a bar magnet of dipole moment M 11. (b). B = ´ = constant
4p x3
when placed in a magnetic field, is given by
t = MB sin q 1/3 1/3
where q is the angle made by the axis of magnet with æ M2 ö æ 1 ö
\ x2 = x1 ç = 20 × ç = 10 cm
the direction of field. è M ÷ø 1 è 2 ´ 4 ÷ø
Given that m = 5 Am, 2l = 0.2 m, q = 30°
and B = 15 Wbm–2 m0 2M m 2M
12. (d). B = = = 0
\ t = MB sin q = (m × 2l) B sin q 4 p ( l2 + x 2 )3/2 4p x 3
1
= 5 × 0.2 × 15 × = 7.5 Nm. 3
2 B1 æ x 2 ö
\ = = 8 : 1 approximately..
m 0 2m ' l B2 çè x1 ÷ø
4. (a) F = mB = m
4p x 3 13. (c). According to tangent law
BA = BB tan q
10 -7 ´ 2 ´ 200 ´ 0.05 ´ 100
= m 0 2M m0 M
8 ´ 10 -3 or = tanq
4p d13 4p d13
= 2.5 × 10-2 N
5. (d). W = MB (cosq1 - cosq2)
d1
MB \ d = (2 cot q)1/3
\ W1 = MB (cos 0º – cos 60º) = 2
2

3 MB I m l2
W2 = MB (cos 30º – cos 90º) = 14. (a). T = 2p = 2p 12 ´ m p lB
= 4 sec
2 MB
\ W2 = 3 W1.
m 2
6. (b). Loss in P.E. = gain in K.E. l
\ Ek = Ui – Uj = – MB cos 90º – (– MB cos 0º) 2
T' = 2p = 4 sec
= 4 × 25 × 10-6 = 10-4 J mp
12 ´ lB
7. (a). t = MB sin q = ml B sin q 2
= 10-3 × 0.1 × 4p × 10-3 × 0.5
= 2p × 10-7 N-m
t.me/Ebooks_Encyclopedia27. t.me/Magazines4all

EBD_7156
118 DPP/ P 42
Given that µ0 = 4p × 10–1 T mA–1,
I 1
15. (c). T = 2p or T µ M = 13.4 Am2,
MB M r = 15 cm = 0.15 m and l = 5.0 cm = 0.05 m

T1 3M - 2M 1.34
= \ BH = 10–7 ×
or T2 = 5 5 s [(0.15) + (0.5) 2 ]3/2
2
T2 3M + 2M
1.34
I = 10–7 × = 0.34 × 10–4 T
16. (b). T = 2p 0.025 0.025
MB
20. (a). As the magnet is placed with its north pole pointing
2
m l / 12 south, the neutral points are obtained on the axial line.
= 2p m p lB
or T µ ml At the neutral points the magnetic field B due to the
magnet becomes equal and opposite to the horizontal
component of earth’s magnetic field i.e., BH.
1/2
æ m lö Hence, if M be the magnetic dipole moment of the
T' ç ÷ T magnet of length 2l and r the distance of neutral point
= n n or T' =
T ç ml ÷ n from the centre of the magnet, then we have
ç ÷
è ø µ0 2Mr
B= . = BH
17. (a). The volume of the bar magnet is 4p (r 2 - l 2 ) 2

mass Given that µ0 = 4p × 10–7 TmA–1 ,


V = r = 40 cm = 0.40 m, l = 15 cm = 0.15 m and
density
HH = 0.34 Gauss = 0.34 × 10–4 T
6.6 ´ 10 -3 kg 4p BH (r 2 - l 2 ) 2
= = 8.3 × 10-7 m3. \M= .
7.9 ´ 103 kg / m3 µ0 2r
The intensity of magnetization is
(0.34 ´ 10-4 ) - [(0.40)2 - (0.15)2 ]2
2 = 10–7×
M 2.5 A - m 2 ´ 0.40
I= =
V 8.3 ´ 10-7 m 2 = 8.0 Am2
The pole strength of the magnet is,
= 3.0 × 106 A/m
18. (d). The compass box will be on the axial line of the magnet, M 8.0
m= = = 26.7 Am
1 2M 1 2 ´ 2lm 2l 0.30
Hence, . = . = H tan q 21. (a). The situation is shown in figure. The horizontal
4p r 3 4p r3 component of earth’s magnetic field at the location of
Given that H = horizontal component of the earth’s the cable (angle of dip q = 0 is)
magnetic field = 30 Am–1, q = 45°,
r = 20 cm = 0.02 m,
M = 2 l m = 4 × 10–2 m Magnetic North
North
10° Geographic
-2
2 ´ 4 ´ 10 m
Hence, 3
= 30 × tan 45°=30 × 1; Magnetic Meridian
4 p (0.2)
Meridian I Cable
3 Geographic 10° Geographic
30 ´ 4p ´ (0.2) West
\m= = 37.7 Am East
2 ´ 4 ´ 10-2 Current
19. (c). As the magnet is placed with its south pole pointing
south, hence the neutral point lies on the equatorial
line. At the neutral point, the magnetic field B due to South Magnetic
the magnet becomes equal and opposite to horizontal South
component of earth’s magnetic field i.e., BH.
Hence, if M be magnetic dipole moment of the magnet
of length 2l and r the distance of the neutral point from BH = B cos q = B cos 0 = B = 0.33 Gauss
its centre, then = 0.33 × 10–4 Tesla
BH is directed horizontally in the magnetic meridian.
µ0 M The magnetic field produced by the cable at a distance
B= = BH
4p (r 2 + l 2 )3/2 of R meter is given by
t.me/Ebooks_Encyclopedia27. t.me/Magazines4all

DPP/ P 42 119

µ0 I Figure shows a point Q on the axis of a short bar magnet


B= lying at a distance r from the centre of the magnet. It is
2 pR given that resultant magnetic field at Q is inclined at
2.5 5.0 ´ 10 -7 an angle of 45° w.r.t. earth’s field B.
= 2 × 10–7 × = T From the figure, it is clear that
R R
Ba = B tan 45° = B
According to right-hand-palm rule no, 1, the field B is
Now for a short bar magnet in this position,
directed horizontally along BH at a point below the
cable, and opposite to BH at a point above the cable. m 0 2M
Therefore, neutral points will be obtained above the Ba = B = .
4p r 3
cable. At these points, will be equal and opposite to
BH. Thus m 0 2M
or r3 = . = 2 × 125 × 10–6
4p B
5.0 ´ 10-7
= BH = 0.33 × 10–4 . \ r = (2 × 125 × 10–6)1/3 = 6.3 cm
R 25-27
5.0 ´ 10-7 Given that earth’s magnetic field, B = 0.39 G and angle
or R = = 15 × 10–3 m = 1.5 cm of dip, q = 35°
0.33 ´ 10-4 Horizontal and vertical components of earth’s magnetic
Thus, the line of neutral points lies above and parallel field B at the location of the cable are
to the cable at a distance of 1.5 cm from it. BH = B cos q = 0.39 cos 35°
22. (a) The magnetic lines of force are in the form of closed = 0.39 × 0.82 = 0.32 Gauss and
curves whereas electric lines of force are open curves. BV = B sin q = 0.39 sin 35°
23. (a) Inside a magnet, magnetic lines of force move from
= 0.39 × 0.57 = 0.22 Gauss
south pole to north pole.
The magnetic field produced by four current carrying
24. (b) Given that pole strength, m = 5.25 × 10–2 JT–1, q =
straight cable wires at a distance R is
45°
and B = 0.42 G = 0.42 × 10–4 T. µ0 I 1.0
B¢ = × 4 = 2× 10–7 × ×4
B 2pR 0.04
45° = 0.2 × 10–4 T = 0.2 Gauss
Beq. P Resultant magnetic field below the cable
According to right - hand, palm rule no. 1, the direction
r of B¢ below the cable will be opposite to that of
BH.Therefore, at a point 4 cm below the cable, resultant
horizontal component of earth’s magnetic field
N S
RH = BH – B¢ = 0.32 – 0.2 = 0.12 Gauss.
Figure shows a point P on the normal bisector of a Resultant vertical component of earth’s magnetic field
short bar magnet lying at a distance r from the RV = BV = 0.22 Gauss (unchanged)
centre O of the magnet. It is given that resultant \ Resultant magnetic field below the cable is
magnetic field at P makes an angle of 45° w.r.t. earth’s
field B. R= [R 2 H + R 2 V ]
From the figure, it is clear that
Beq = B tan 45° = B. = [(0.12)2 + (0.22)2 ]
Now, for a short bar magnet in this position
= 0.25 Gauss
µ0 M The angle that R makes with the horizontal is given by
Beq = B = .
4p r3
RV
q = tan–1
µ0 M RH
or r3 = .
4p B
0.22
5.25 ´ 10-2 = tan–1 = tan–1(1.8) @ 62°
0.12
= 10–7 × = 125 × 10–7.
0.42 ´ 10-4 Resultant magnetic field above the cable
–2
\ r = 5 × 10 m = 5 cm. Again, according to right - hand - palm no. 1, the
direction of B¢ at a point above the cable is the same as
B
that of BH.
Therefore, at a point 4 cm below the cable, the
45°
horizontal component of resultant magnetic field will
O S Q be
N
r Ba RH = BH + B¢ = 0.32 + 0.20 = 0.52 Gauss
t.me/Ebooks_Encyclopedia27. t.me/Magazines4all

EBD_7156
120 DPP/ P 42
Vertical component of resultant magnetic field will be 28. (a)
RV = BV – 0.22 Gauss ( unchanged) 29. (d) The earth has only vertical component of its magnetic
Hence, magnitude of resultant magnetic field below field at the magnetic poles. Since compass needle is
the cable only free to rotate in horizontal plane. At north pole
the vertical component of earth's field will exert torque
R = [R 2 H + R 2 V ] on the magnetic needle so as to aligh it along its direc-
tion. As the compass needle can not rotate in vertical
= [(0.52)2 + (0.52)2 ] plane, it will rest horizontally, when placed ont he mag-
= 0.56 Gauss netic pole of the earth.
The angle that R makes with the horizontal is given by 30. (c) It is quite clear that magnetic poles always exists in
pairs. Since, one can imagine magnetic field configu-
æ RV ö æ 0.22 ö ration with three poles. When north poles or south poles
q = tan–1 ç R ÷ = tan–1 çè ÷
è Hø 0.52 ø of two magnets are glued together. They provide a three
pole field configuration. It is also known that a bar
= tan–1 (0.43) @ 23°
magnet does not exert a torque on itself due to own its
25. (a) 26. (a) 27. (b)
field.
t.me/Ebooks_Encyclopedia27. t.me/Magazines4all

DPP/ P 43 121

DAILY PRACTICE
PROBLEMS
PHYSICS
SOLUTIONS 43
1. (a) Susceptibility of ferromagnetic substance is greater 24. (c)
than 1. 25. (b) Given that : H = 1600 Am–1 , f =2.4 ×10–5 Wb,
2. (c) Susceptibility of diamagnetic substance is negative and A = 0.2 cm2 = 0.2 × 10–4 m2.
it does not change with temperature. B = magnetic flux per unit cross - sectional are
3. (c) m d = 0 and m p ¹ 0
f 2.4 ´ 10 -5
= = = 1.2 Wbm–2
B A 0.2 ´ 10 -4
4. (d) m r = =4
B0 Magnetic permeability :
5. (b) Dimagnetic B 1.2 Wbm -2 or T
6. (b) Paramagnetic m= = = 7.5 × 10–4 T A–1 m
H 1600 Am -1
7. (d) A super conductor exhibits perfect diamagnetism.
8. (a) Soft iron is highly ferromagnetic. As µ = µ0 (1 + xm)
9. (a) Diamagnetism is the universal property of all
µ 7.5 ´ 10-4
substances. \ xm = -1 =
10. (c) Diamagnetic substances are repelled by magnetic field. µ0 4 ´ 3.14 ´ 10 -7
11. (b) Coercive force = 597.1 – 1 = 596.1.
12. (b) Because, diamagnetic substance, moves from stronger 26. (a) The energy lost per unit volume of a substance in a
magnetic field to weaker field. complete cycle of magnetisation is equal to the area of
13. (d) m r > 1, c > 0 the hysteresis loop.
14. (b) Hysteresis curve for a given material estimates 27. (a) Statement (4) is the only true statement among the given
hysteresis loss. choices.
15. (a) Neon atom is diamagnetic, hence it’s net magnetic 28. (b) The susceptibility of ferromagnetic substance decreases
moment is zero. with the rise of temperature in a complicated manner.
16. (b) On heating, different domains have net magnetisation After Curie's point the susceptibility of ferromagnetic
in them which are randomly distributes. Thus the net substance varies inversely with its absolute
magnetisation of the substance due to various domains temperature. Ferromagnetic substance obey’s Curies
decreases to minimum. law only above its Curie point.
17. (b) Repelled due to induction of similar poles. 29. (d) A paramagnetic sample display greater magnetisation
when cooled, this is because at lower temperature, the
18. (d) From the characteristic of B - H curve.
tendency to disrupt the alignment of dipoles (due to
19. (c) Diamagnetic substances are feebly repelled by magnets. magnetising field) decreases on account of reduced
20. (d) Net magnetic induction B = B0 + Bm = µ0H + µ0M random thermal motion.
21. (d) c m = (mr - 1) Þ c m = (5500 - 1) = 5499 30. (d) The permeability of a ferromagnetic material dependent
ur ur
m B on magnetic field, B = K m B0 , where B0 is applied
22. (c) Q mr = =
m 0 Hm 0 ur
field. The total magnetic field B inside a ferromagnet
8p may be 103 or 104 times the applied field B0 The
or mr = = 104
2 ´ 10 ´ 4 p ´ 10-7
3 permeability Km of a ferromagnetic material is not
ur uur
m constant, neither the field B nor the magnetization M
23. (a) WH = VAft =
d
Aft ur
increases linearly with B even at small value of B0.
0.6 From the hysteresis curve, magnetic permeability is
or WH = × 0.722 × 50
7.8 ´ 103 greater for lower field.
= 277.7 × 10-5 Joule
t.me/Ebooks_Encyclopedia27. t.me/Magazines4all

EBD_7156
122 DPP/ P 4 4
DAILY PRACTICE
PROBLEMS
PHYSICS
SOLUTIONS 44
r r e = 4 × 10–5 × 3.14 × (.5)2 × 2 = 6.28 × 10–5 volt
(1) (a) f = B.A
(9) (d) Rate of decrease of area of the semicircular ring
= ( 0.02iˆ ) . ( 30iˆ + 16ˆj + 23kˆ ) × 10–4 dA
– = (2R) V
= 0.6 × 10-4 Wb = 60m Wb dt
(2) (c) The induced emf According to Faraday’s law of induction, induced emf
d df dA
E = – df/dt = – (3t2 + 2t + 3) × 10-3 e= - = –B = – B(2RV)
V)
dt dt dt
(because given flux is in mWb).
Thus E = ( – 6t – 2) × 10–3 × × × × ×
N ×
At t = 2 sec, × × ×
E = ( – 6 × 2 – 2) × 10–3 = –14 mV. × × × × ×
(3) (a) The direction of current in the solenoid is clockwise. × × × × ×
On displacing it towards the loop a current in the loop M Q vt
will be induced in opposite sense so as to oppose its 2R
approach. Therefore the direction of induced current The induced current in the ring must generate magnetic
as observed by the observer will be anticlockwise. field in the upward direction. Thus Q is at higher
(4) (b) When north pole of the magnet is moved away, then potential.
south pole is induced on the face of the loop in front of (10) (b) E = Blv
the magnet i.e. as seen from the magnet side, a clockwise
induced current flows in the loop. This makes free 360 ´ 1000
= 4 × 10–4 × 50 × =2V
electrons to move in opposite direction, to plate a. Thus 60 ´ 60
excess positive charge appear on plate b. (11) (c) The flux through the area is
(5) (d) If electron is moving from left to right, the flux linked f = BA cos 57º = 42 × 10-6 × 2.5 × 0.545
with the loop (which is into the page) will first increase = 57 × 10-6 Wb.
and then decrease as the eletron passes by. So the (12) (a) The magnetic flux linked with the loop at any instant of
induced current in the loop will be first anticlockwise time t is given by
and will change direction as the electron passes by.
f = BAN cos wt
df or f = 10Ba2 cos wt
(6) (c) E = – Here N = 10, A = a2
dt
or df = – Edt = (0 - f) (13) (b) According to Lenz’s Law
or f = 4 × 10–3 × 0.1 300 ´ 103
= 4 × 10–4 weber (14) (c) e = Bvl = 0.4 × 10–4 × ´ 3 = 10–2 V
60 ´ 60
df d (15) (b) Magnetic flux passing through the disc is f = BA
(7) (d) e = =– [10t2 + 5t + 1] × 10–3
dt dt weber
= – [10 × 10–3 (2t) + 5 × 10–3] = 0.01 × 3.14 × (15 × 10–2 meter)2
meter 2
at t = 5 second = 7.065 × 10–4 weber.
e = –[10 × 10–2 + 5 × 10–3] = [0.1 + 0.005] The line joining the centre and the circumference of the
|e| = 0.105V disc cuts 7.065 x 10-4 weber flux in one round. So, the
(8) (a) For each spoke, the induced emf between the centre O rate of cutting flux (i.e. induced emf)
and the rim will be the same = flux × number of revolutions per second

1 100
e= BwL2 = BpL2 f (Q w = 2pf) = 7.065 × 10–4 × = 3.9 × 10–4 volt.
2 60 ´ 3
Further for all spokes, centre O will be positive while (16) (c) The magnetic flux passing through each turn of a coil of area
rim will be negative. Thus all emf's are in parallel giving A, perpendicular to a magnetic field B is given by
total emf e = BpL2 f f1 = BA.
independent of the number of the spokes. The magnetic flux through it on rotating it through 180º
Substituting the values will be
f2 = – BA.(- sign is put because now the flux lines
enters the coils through the outer face)
t.me/Ebooks_Encyclopedia27. t.me/Magazines4all

DPP/ P 4 4 123
\ change in magnetic flux
d ( Nf )
D f = f1 – f2 = – BA – (BA) = – 2BA. Qe=–
Suppose this change takes in time Dt. According to dt
Faraday's law, the emf induced in the coil is given by d
= –N (BA cos wt)
Df 2NBA dt
e=–N = , = NBAw sin wt
Dt Dt
where N is number of turns in the coil. The current in \ emax = NABw
the coil will be = 60 × 2 × 10–2 × 0.5 × 2p × 1800/60 = 113 volt.
Hence, induced emf can be 111 V, 112 V and 113.04 V
e 1 2NBA (23) (b) The change is flux linked with the coil on rotating it
i= =
R R Dt through 180º is
where R is the resistance of the circuit. The current = nAB – (–nAB) = 2nAB
persists only during the change of flux i.e. for the time
df
interval Dt second. So, the charge passed through the \ induced e.m.f. = –
circuit is dt

2NBA 2 ´ 1 ´ 0.1
q = i × Dt = . = 2nAB/dt (numerically) = = 20 V
R 0.01
Here N = 500, B = 0.2 weber/meter2, The coil is closed and has a resistance of 2.0 W.
A = 4.0 cm2 = 4.0 × 10–4 meter2 and R = 50 ohm. Therefore i = 20/2 = 10A.
(24) (d) Initial magnetic flux f1 = 5.5 × 10–4 weber.
2 ´ 500 ´ 0.2 ´ 4.0 ´ 10-4 Final magnetic flux f2 = 5 × 10–5 weber.
\ q=
50 \ change in flux
= 1.6 × 10–3 coulomb. Df = f2 - f1 = (5 × 10-5) – (5.5 × 10-4) = – 50 × 10–5
(17) (a) T weber.
(18) (c) The induced emf between centre and rim of the rotating Time interval for this change, Dt = 0.1 sec.
disc is \ induced emf in the coil
1 1 Df (-50 ´ 10-5 )
E= BwR2 = × 0.1 × 2p × 10 × (0.1)2 e=–N = – 1000 × = 5 volt.
2 2 Dt 0.1
= 10p × 10–3 volt, Resistance of the coil, R = 10 ohm. Hence induced
(19) (a) The induced emf current in the coil is
E = Blv = 0.2 × 10–4 × 1 × 180 × 1000/3600
= 0.2 × 18/3600 = 1 × 10–3. V = 1mV e 5 volt
i= = = 0.5 ampere.
(20) (d) The induced emf is obtained by considering a strip on R 10ohm
the disc fig. Then, the linear speed of a small element dr (25) (a), (26) (c), (27) (b)
at a distance r from the centre is = wr. The induced emf (a) The current of the battery at any instant, I = E/R.
across the ends of the small element is- The magnetic force due to this current
de = B(dr)v = B wr dr
Thus the induced emf across the inner and outer sides EBl
FB = IBL =
of the disc is R
This magnetic force will accelerate the rod from its
b 1
e= ò a Bwr dr 2
= Bw (b2 – a2) position of rest. The motional e.m.f. developed in the
rod id B/v.
r r r The induced current,
(21) (c) The induced emf e = – (v ´ B).l
r r r r Blv
For the part PX, v ^ B , and the angle between (v ´ B) Iinduced =
R
r
direction (the dotted line in figure and l is (90 – q). The magnetic force due to the induced current,
Thus
B2l 2 v
eP –ex = vBl cos (90 – q/2) = vBl sin (q/2) Finduced = Iinduced =
Similarly ey – ep = vBl sin (q/2) R
Therefore induced emf From Fleming’s left hand rule, foce FB is to the right
between X and Y is eyx = 2 B v l sin (q/2) and Finduced is to the left.
(22) (a) Given, area = 10 × 20 cm2 = 200 × 10-4 m2 Net force on the rod = FB – Finduced.
B = 0.5 T, N = 60, w = 2p × 1800/60
t.me/Ebooks_Encyclopedia27. t.me/Magazines4all

EBD_7156
124 DPP/ P 4 4
From Newton’s law, (b) The rod will attain a terminal velocity at t ® ¥, i.e.,
dv when e–t/t = 0, the velocity is independent of time.
FB - Finduced = m
dt E
vT =
Bl
EBl B2l 2 v dv
- =m (d) The induced current Iinduced = Blv/R. When the rod
R R dt has attained terminal speed.
On separating variables and integrating speed from v0
to v and time from 0 to t, we have Bl æ E ö
Iinduced = ´ = E/ R
dv Bl
R çè Bl ÷ø
= dt The current of battery and the induced current are of
E - Bvl mR
same magnitude, hence net current through the circuit
v dv Bl t is zero.
ò0 E - Bvl = mR ò0 dt (28) (c) Since both the loops are identical (same area and number
of turns) and moving with a same speed in same
æ E - Bvl ö B l
2 2
- ln ç magnetic field. Therefore same emf is induced in both
÷= t
the coils. But the induced current will be more in the
è E ø mR
copper loop as its resistance will be lesser as compared
B2 l2 to that of the aluminium loop.
E - Bvl - t
=e mR (29) (c) As the aircraft flies, magnetic flux changes through its
E wings due to the vertical component of the earth’s
E magnetic field. Due to this, induced emf is produced
v= (1 - e- t / r ) across the wings of the aircraft. Therefore, the wings
Bl
of the aircraft will not be at the same potential.
mR (30) (c) Lenz’s Law is based on conservation of energy and
where t=
(Bl) 2 induced emf opposes the cause of it i.e., change in
magnetic flux.
t.me/Ebooks_Encyclopedia27. t.me/Magazines4all

DPP/ P 45 125

DAILY PRACTICE
PROBLEMS
PHYSICS
SOLUTIONS 45
dI 1.0 22 I
1. (a). e = – L = – 10 × 10–3 = –1Volt. = P Þ IP = 0.01 A.
dt 0.01 220 0.1
\ | e | = 1 volt. 9. (c). Vp = 220V, Ip = 5S, Vs = 2200V
dI d Pp
2. (a). e = – L =–L [3t2 + 2t] Ps = , Is = ?
dt dt 2
= – L [6t + 2] = –10 × 10–3 [6t + 2]
(e)at t = 2 = – 10 × 10–3 (6 × 2 + 2) Vp Ip
Q VsIs =
= – 10 × 10–3 (14) = – 0.14 Volt 2
|e| = 0.14 volt. After putting the given value you will find
3. (a). The mutual inductance between two coils depends on Is = 0.25 A.
their degree of flux linkage, i.e., the fraction of flux linked 10. (d). During decary of current
with one coil which is also linked to the other coil. Here,
the two coils in arrangement (a) are placed with their planes Rt Rt 100´10-3
- E 100 100´10-3 1
parallel. This will allow maximum flux linkage. i = i0 e L
= eL = e = A.
R 100 e
dI1 0 - 10
4. (b). e2 = – M =- = 3 × 104 volt = 30 kV.. 11. (c). In a generator e.m.f. is induced according as Lenz’s
dt 3 ´10-3 rule.
5. (d). L2 and L3 are in parallel. Thus their combination gives The minus sign indicates that the direction of the induced
L 2 L3 e.m.f. is such as to oppose the change in current.
L' = = 0.25 H 12. (a). 'Immediately' after pressing the switch S, the current in
L 2 + L3
the coil L, due to its self-induction will be zero, that is
The L' and L1 are in series, thus the equivalent inductance i2 = 0.
is L = L1 + L' = 0.75 + 0.25 = 1H The current will only be found in the resistance R1 and this
6. (a). We use e = – L DI/Dt to determine the value of induced will be the total current in the circuit.
emf. (i) DI = (7 – 0) = 7A,
E 10 volt
Dt = (2 – 0) ms = 2ms \ i = i1 = = = 2.0 ampere.
R 2 5.0 volt
7
Thus e = – 4.6 × = – 16.1 × 103 volt 13. (a). (i) In series the same current i will be induced in both
2 ´ 10-3 the inductors and the total magnetic-flux linked with them
(ii) DI = 5 – 7 = – 2A, will be equal to the sum of the fluxes linked with them
dt = (5 – 2) ms = 3 ms individualy, that is,
(-2) F = L1i + L2i.
Thus e = – 4.6 × = 3.07 × 103 V If the equivalent inductance be L, then F = Li.
3 ´ 10-3 \ Li = L1i + L2i or L = L1 + L2.
7. (b). I = I0 (1 – e– t/t) (ii) In parallel, let the induced currents in the two coils be i1
1 and i2. Then the total induced current is
Where I = I and t = L/R
2 0 di di1 di 2
i = i1 + i2 \ = +
1 dt dt dt
Thus I = I (1 – e – t/t ) In parallel, the induced e.m.f. across each coil will be the
2 0 0
same.
1 di1 di 2
or = e – t/t or 2 = e+ t/t or log 2 = t/t Hence e = – L1
2 = –L2 .
dt dt
50 ´10-3 di
Thus t = t loge 2 = × 0.693 = 1.385 If the equivalent inductance be L, then e = – L .
0.025 dt
8. (a). VS = IS ZS Þ 22 = IS × 220
e di æ di1 di 2 ö e e
\ IS = 0.1A \ =- = – çè + +
dt ÷ø
=
L dt dt L1 L2
VS IP
=
VP IS
t.me/Ebooks_Encyclopedia27. t.me/Magazines4all

EBD_7156
126 DPP/ P 45
1 1 1 L1L 2
or = + or L= .
L L1 L 2 L1 + L2 dx
14. (d). If we try to find field of the small coil and then calculate A B
flux through long solenoid, the problem becomes very
difficult. So we use the following fact about mutual I
inductance. x
f 2 f1 C
M21 = M12, = F
I1 I2
Thus if I current flows in long solenoid, then flux f through
small coil is the same as the flux f2 that is obtained when I l
current flows through the small coil. Therefore, E D
f2 = f1 = (Field at small coil) × (area) × (turns)
æ N2 ö m N N AI
= ç m0 l I÷ (AN ) = 0 1 2
è 2 ø 1 l2
15. (a). The induced e.m.f. is a +l 2a + l
m 0 Ia é dx dx ù m 0 Ia æ 2a + l ö
Di f= ê ò + ò ú= 1n ç
e= –M or M =–
e
2p ê x x ú 2p è l ÷ø
Dt Di / D t ë l a +l û
Here e = 1500 volt. f m0 a æ 2a + l ö
M= ÞM= 1n ç ÷
1500 1500 ´ 0.001 I(primary) 2p è l ø
\M=– = = 0.5 henry..
(0 - 3.0) / 0.001 3.0
dI
dI p d ( I0 sin wt ) e = -M
16. (c). ES = M =M dt
dt dt
= MI0w |cos wt| m 0I0a æ 2a + l ö
e = -MI0 = - 1n ç
Þ Crest value = MI0w 2p è l ÷ø
= 1.5 ´ 1 × 2p × 50
= 471 V 2
e
é m0 1n 2a +l I ù at
ê 2p 2 0ú ( )
t=ë û
1 1 l
2 Heat produced =
17. (a) U = Li2 = ´100 ´10 -3 ´ (10 ) = 5J
2 2 R 8l
Vs 5 Vs 28. (a) Hysteresis loss in the core of transformer is directly
18. (c) Transformation ratio k = V Þ 3 = 60 Þ Vs = 100 V proportional to the hysteresis loop area of the core
p
material. Since soft iron has narrow hysteresis loop
ip Ns ip 1 area, that is why soft iron core is used in the transformer.
19. (b) = Þ = Þ i p = 0.04 A 29. (d) Efficiency of electric motor is maximum when the back
is Np 4 100
emf set up in the armature is half the value of the applied
Output 80 20 ´ 120 battery emf.
20. (a) h= Þ = 30. (a) Transformer works on ac only, ac changes in magnitude
Input 100 1000 ´ il
as well as in direction.
20 ´ 120 ´ 100
Þ il = = 3A.
1000 ´ 80
21. ()
22. (b)
23. (a)
24. (b)
25. (a) 26. (a) 27. (b)
Consider a strip at a distance x from the wire of thickness dx.
Magnetic flux associated with this strip
m 0 Ia
D f = B ad x = dx
2px
t.me/Ebooks_Encyclopedia27. t.me/Magazines4all

DPP/ P 46 127

DAILY PRACTICE
PROBLEMS
PHYSICS
SOLUTIONS 46
1. (b) The coil has inductance L besides the resistance R . 12. (b)
V 120
Hence for ac it’s impedance resistance R 2 + X L2 will i= = = 0.016 A
2 2 2
R +w L 100 + 4p2 ´ 602 ´ 202
be larger than it’s impedance R for dc.
13. (a) The voltage across a L-R combination is given by
i 4
2. (b) ir.m.s. = 0 = = 2 2 amperee V 2 = VR2 + VL2
2 2
T VL = V 2 - VR2 = 400 - 144 = 256 = 16 volt.
3. (d) The current takes sec to reach the peak value.
4 14. (b) Reading of ammeter,
2p 1 Vrms Vo wC
In the given question = 200p Þ T = sec i rms = =
T 100 Xc 2

1 200 2 ´ 100 ´ (1 ´ 10-6 )


\ Time to reach the peak value = 400 sec =
2
4. (b) 50 c/s or Hz
= 2 × 10–2 A = 20 m A
2pt
5. (b) E = E0 cos wt = E0 cos R C
T
15. (a)
2 p ´ 50 ´ 1 p
= 10 cos = 10 cos = 5 3 volt. VR VR
600 6
~
i12 + i22 1 Let the applied voltage be V, volt.
6. (c) irms = = (i12 + i22 )1/ 2
22 Here, VR = 12 V, VC = 5V
7. (c) Hot wire ammeter reads rms value of current. Hence
V= VR2 + VC2 = (12 )2 + ( 5)2 = 13 V.
its peak value = irms ´ 2 = 14.14 amp
16. (c) Z = X L = 2p´ 60 ´ 0.7
V0 120
8. (b) Vrms = = = 84.8 V 120 120
2 1.414 \i = = = 0.455 amperee
Z 2p ´ 60 ´ 0.7
9. (d) Peak value to r. m. s. value means, current becomes
17. (d) Current will be max at first time when
1 100 pt + p/3 = p/2 Þ 100 pt = p/6 Þ t = 1/600 s.
times.
2 18. (d) Z = R 2 + X 2 = R 2 + (2p fL)2
If peak is at t = 0, current is of the form
2
1 æ 0.4 ö
i = i0 cos100pt Þ ´ i0 = i0 cos100pt = (30) 2 + ç 2 p ´ 50 ´ ÷ = 900 + 1600 = 50W
2 è p ø

p 1 V 200
Þ cos = cos100 pt Þ t = sec = 2.5 ´10 -3 sec. i= = = 4 ampere
4 400 Z 50
19. (d) In purely inductive circuit voltage leads the current by 90o .
10. (b) Z = R 2 + X L2 , X L = wL and w = 2pf
P 60
20. (a) Current through the bulb i = = = 6A
2 2 2 2 V 10
\ Z = R + 4p f L
60 W, 10 W
11. (b) The applied voltage is given by V = VR2 + VL2 i L

V = (200)2 + (150)2 = 250 volt 10 V VL


i

100 V, 50 Hz
t.me/Ebooks_Encyclopedia27. t.me/Magazines4all

EBD_7156
128 DPP/ P 46
V= VR2 + VL2 V = V0 sin ( wt - f)

(100)2 = (10)2 + VL2 Þ VL = 99.5 Volt


The peak value V0 = 220 2 = 311V
Also VL = iXL = i ´ (2pvL)
Þ 99.5 = 6 ´ 2 ´ 3.14 ´ 50 ´ L = Þ L = 0.052 H. and as in case of ac,
wL 2p ´ 200 1 4
21. (a) Phase angle tan f = = ´ = V0
R 300 p 3 Vrms = ;Vrms = 220 V
2
4
\ f = tan -1 26. (b) In case of ac,
3
22. (a) The root mean square voltage is effective voltage.
2 2 622
23. (c) E = 141 sin (628t), Vav = V0 = ´ 311 = V
p p p
E0 141
Erms = = = 100V and 2pf = 628 314
2 1.41 27. (a) As w = 2pf ,2pf = 314i.e., f = = 50Hz
2´ p
Þ f = 100 Hz 28. (d) When ac flows through an inductor current lags behind
the emf., by phase of p/2, inductive reactance,
24. (d) Time taken by the current to reach the maximum value X L = wL = p.2 f .L, so when frequency increases
T 1 1 correspondingly inductive reactance also increases.
t= = = = 5 ´ 10-3 sec 29. (c) Like direct current, an alternating current also produces
4 4v 4 ´ 50
magnetic field. But the magnitude and direction of the
and i0 = irms 2 = 10 2 = 14.14 amp field goes on changing continuously with time.
30. (b) We can use a capacitor of suitable capacitance as a
25. (a) As in case of ac, choke coil, because average power consumed per cycle
in an ideal capacitor is zero. Therefore, like a choke
coil, a condenser can reduce ac without power
dissipation.
t.me/Ebooks_Encyclopedia27. t.me/Magazines4all

DPP/ P 47 129

DAILY PRACTICE
PROBLEMS
PHYSICS
SOLUTIONS 47
13. (b) Resonance frequency
1. (a) For resonant frequency to remain same
LC should be const. LC = const 1 1
w= = = 2500 rad/sec
-3
L LC 8 ´10 ´ 20 ´10-6
Þ LC = L' × 2C Þ L ' =
2
V 220
2. (b) At resonance, LCR circuit behaves as purely resistive Resonance current = = = 5A
R 44
circuit, for purely resistive circuit power factor = 1
3. (a) If the current is wattless than power is zero. Hence 100 10 -3 p
14. (c) P = Vr.m.s. ´ ir.m.s. ´ cos f = ´ ´ cos
phase difference f = 90° 2 2 3
4. (c) VL = 46 volts, VC = 40 volts, VR = 8 volts
102 ´ 10-3 1 10-1
= ´ = = 0.025 watt
E.M.F. of source V = 82 + (46 - 40)2 = 10 volts 2 2 4
15. (b) R = XL = 2XC
1
5. (c) Resonant frequency = does not depend on Z = R 2 + (X L - X C ) 2 XL–XC
2p LC
resistance. = (2X C )2 + (2X C - X C ) 2 Z
6. (a) At resonance LCR series circuit behaves as pure
= 4X 2C + X 2C
resistive circuit. For resistive circuit f = 0o
f
5R R
R R = 5X C =
7. (b) cos f = Z = 2
R2 + w2 L2 X L - XC 2XC - XC
tan f = =
12 R 2XC
= Þ cos f = 0.30
(12) + 4 ´ p ´ (60) 2 ´ (0.1) 2 -1 æ 1 ö
2 2 1
tan f = ; f = tan çè ÷ø
2 2
1 1
8. (a) f = Þ f µ 16. (d) Phase angle f = 90o , so power P = Vrms I rms cos f = 0
2p LC C
2
9. (b) In non resonant circuits Vrms (30)2
17. (b) P = = = 90 W
R 10
1
impedance Z = , with rise in 18. (c) At A : X C > X L
2
1 æ 1 ö
+ wC -
2 ç ÷ At B : X C = X L
R è wL ø
At C : X C < X L
frequency Z decreases i.e. current increases so circuit
19. (d) The instantaneous values of emf and current in
behaves as capacitive circuit.
inductive circuit are given by E = E0 sin at and
R 10 1
10. (c) cos f = = = Þ f = 60o pö
Z 20 2 æ
i = i0 sin ç w - ÷ respectively..
11. (d) è 2ø

1 æ pö
- 2pfL So, Pinst = Ei = E0 sin wt ´ i0 sin ç wt - ÷
XC - X L 2pfC è 2ø
12. (a) tan f = Þ tan 45o =
R R = E0i0 sin wt cos wt
1 1
ÞC= = E0i0 sin 2wt (sin 2wt = 2sin wt cos wt )
2pf (2pfL + R) 2
Hence, angular frequency of instantaneous power is
2w .
t.me/Ebooks_Encyclopedia27. t.me/Magazines4all

EBD_7156
130 DPP/ P 47
20. (d) The voltage VL and VC are equal and opposite so (DVL + DVC )max = DVC - DVL = 7.4 - 2.6 = 4.8 volt
voltmeter reading will be zero. 2 2
26. (c) E m = (DVR )max + ( DVC - DVL )max
Also R = 30W, X L = X C = 25W
= (8.8)2 + (4.8) 2 = 10 volt
V V 240
So i = = = = 8A 27. (a) If f ­ then (DVL)max ­
R 2 + ( X L - X C )2 R 30
1
28. (a) Capacitive reactance XC = . When capacitance
1 L wC
21. (d) Since quality factor, Q = (C) increase, the capacitive reactance decreases. Due
R C
22. (d) to decrease in its values, the current in the circuit will
23. (a) æ ö
ç I= E ÷
24. (c) Reactance Z = ( X L - X C )2 + R 2 increase ç
è R 2
+ X 2 ÷ and hence brightness of
C ø
source (or electric lamp) will also increase.
= (80 - 50 )2 + 402 29. (b) The phase angle for the LCR circuit is given by
= 50 W
1
R 40 X L - X C wL - wC
Power factor = cos f = = = 0.8 tan f = =
Z 50 R R
Vrms 200V where XL, XC are inductive reactance and capacitive
Irms = = =4A reactance respectively when XL > XC then tan f is
Z 50W
Power current = Irms.cos f = 4 × 0.8 positive i.e. f is positive (between 0 and p/2). Hence
= 3.2 A emf leads the current.
Wattless current = Irms.sin f = 4 × 0.6 30. (a) If resistor is used in controlling ac supply, electrical
= 2.4 A energy will be wasted in the form of heat energy across
Sol. 25-27 the resistance wire. However, ac supply can be
DVL controlled with choke without any wastage of energy.
This is because, power factor (cos f) for resistance is
unity and is zero for an inductance. [P = EI cos f].
DVR
25. (d)

DVC
t.me/Ebooks_Encyclopedia27. t.me/Magazines4all

DPP/ P 48 131

DAILY PRACTICE
PROBLEMS
PHYSICS
SOLUTIONS 48
N – m2 E0 = 2 3 × 102 N/C
1. (d) m0 = 4p × 10–7, e0 = 8.85 × 10–12
C2 P 2 1
10. (b) I = or = W/m2
1 meter A 4 p ´ 4 8p
so c = 3 ´ 108
m 0 e0 = sec
.
1 2
2. (b) Wavelength of visible spectrum is 3900 Å – 7800 Å. I= Î0 E0 c
2
3. (a) lg–rays < lx-rays < la–rays < lb–rays.
4. (c) Electric field between the plates of the capacitor is given 2I 2 ´ 1´ 36p´109
E0 = Î c or = 30 N/C
by 0 8p´ 3 ´108
s q
E= or B02
Î0 A Î0
11. (a) UB =
4µ0
Flux through the area considered
q A q E0 E
f= AÎ ´ 4 = 4Î Also =c \B = 0
0 0 B0 0 c

df E E0
Displacement corrent id = Î0 Hence B0 =
dt 1
µ0 Î0
dæ q ö i
= Î0 × dt ç 4 Î ÷ = 4
è 0ø
E 20µ0 Î0 100 ´ 8.84 ´ 10-12
5. (a) Electric field between the plates is \ UB = or
4µ0 4
Q
E= \ UB = 2.21 × 10–10 J/m3
Î0 A 12. (a) The speed of electromagnetic waves and in a medium
is given by
Q
\ fE = E. A or ×A 1
Î0 A
n=
(µe )
df E d æ Qö
\ id = Î0 or Î0 dt ç Î ÷ Where µ and e are absolute permeability and absolute
dt è 0ø permittivity of the medium.
We know that, µ = µ0µr and e = e0er. Hence
dQ
\ id = = i (charging current) 1 1 1
dt v= = ´
Hence id = 1A ( µ0µr .e0 e r ) (µ0 e 0 ) ( µr e r )
6. (c) E × B
7. (b) Since the direction of propogation of EM wave is given
c2
)
c
by E × B ( ^ ^
\ j ´ i = -k ^ or v =
( µr e r )
or er =
n2 (µr )
1
8. (d) Speed of E.M. wave = in medium hence (3 ´ 108 )2
µ0 Î0 µr Îr \ er = = 2.25
(2 ´ 108 )2 ´ 1
it will travel with different speed in different medium.
13. (a) Given By = 2 × 10–7 sin (0.5 × 103x + 1.5 × 1011t)
1 2 Comparing it with a standard equation for a progresive
9. (b) I = Î cE
2 0 0 wave travelling along the negative direction of x-axis
is
2I 2 ´ 500 ´109 ´ 36 p 2p æ 2px 2pvt ö
E0 = Î0 c or y = r sin (x + vt) = r sin çè + ÷
p´ 3 ´ 108 l l l ø
t.me/Ebooks_Encyclopedia27. t.me/Magazines4all

EBD_7156
132 DPP/ P 48
\ Speed and wavelength of wave becomes half and
æ 2px ö
= r sin ç + 2pnt ÷ frequency remain unchanged.
è l ø
11 For 25 - 27
2pn = 1.5 × 10
h = 150 km = 150 × 103 m
1.5 ´ 1011 Nm = 9 × 1010 per m3
n= = 23.9 × 109 Hz = 23.9 Hz D = 250 km = 250 × 103 m
2p
25. (a) Critical frequency of layer
14. (c) The given equation
Ey = 0.5 cos[2p × 108 (t – x/c)] ..... (1) fc = 9 N m = 9 ´ 9 ´ 1010 = 2.7 ´ 106 Hz.
indicates that the electromagnetic waves are 26. (b) Maximum usuable frequency
propagating along the positive direction of X-axis.
The standard equation of electromagnetic wave is given 2
D2 6
æ 250 ´ 103 ö
by Ey = E0 cosw(t – x/c) ..... (2) f = fc 1 + = 2.7 ´ 10 ´ 1 + ç

Comparing the given eq. (1) with the standard eq. (2), 4h 2 è 4 ´ 150 ´ 10 ø
we get w = 2p × 108 = 3.17 × 106 Hz.
or 2pn = 2p × 108 27. (c) If angle of incidence at this layer
\ n = 108 per second is fi , from second law of f = f c sec
f i.
c 3 ´ 108
Now, l = = =3m
n 108 f 3.17 ´ 106
sec fi = = = 1.174
15. (a) The maximum value of magnetic field (B0) is given by fc 2.7 ´ 106
E0 E0 fi = sec -1 (1.174) = 31.6°
B0 = = = 10–6 tesla
c c 28. (d) The electromagnetic waves of shorter wavelength do
The magnetic field will be along Z-axis not suffer much diffraction from the obstacles of earth's
The maximum magnetic force on the electron is
atmosphere so they can travel long distance.
Fb = |q (v × B)| = q n B0
= (1.6 × 10–19) × (2.0 × 107) × (10–6) 29. (b) The wavelength of these waves ranges between 300 Å
= 3.2 × 10–18 N and 4000 Å that is smaller wavelength and higher
16. (c) b-rays are beams of fast electrons. frequency. They are absorbed by atmosphere and
17. (b) convert oxygen into ozone. They cause skin diseases
and they are harmful to eye and cause permanent
c 3 ´ 108
18. (d) v = = 1.8 ×108m/sec blindness.
mr er = 1.3 ´ 2.14 30. (b) Radio waves can be polarised because they are
19. (c) lm > lv > lx transverse in nature. Sound waves in air are longitudinal
r r in nature.
20. (d) Direction of wave propagation is given by E×B .

mr m
21 . (c) Wave impedance Z = ´ 0
er e0

50
= ×376.6=1883W
2

22. (a)
23. (a) b- rays are beams of fast electrons.

me
24. (a) Refractive index = m 0e 0

e
Þ Then refractive index = =2
e0
t.me/Ebooks_Encyclopedia27. t.me/Magazines4all

134 DPP/ P 49
DAILY PRACTICE
PROBLEMS
PHYSICS
SOLUTIONS 49
360 360 (5) (c) Small and erect image is formed only by convex mirror.
(1) (a) Here, n = = =4 Plane mirror from images equal to object and concave
q 90
mirror form images bigger than object.
\ n is an even number. (6) (b) The image will be formed by the plane mirror at a 30
M2 cm behind it, while the image by convex mirror will be
formed at 10 cm behind the convex mirror.
Since for convex mirror u = – 50 cm
I2 O v = 10 cm
1 1 1 -1 + 5 4
= + = =
f -50 10 50 50
c• M1
50
f= = 12.5 cm
I3 4
I1 Therefore the radius of curvature of convex mirror is
r = 2 f = 25 cm
Thus, number of images formed = n – 1 = 3. All these (7) (a) The image of object O from mirrror M1 is I1 and the
three images lie on a circle with centre at C (The point image of I1 (the vitual object) from mirror M2 is I3.
of intersection of mirrors M1 and M2 ) and whose radius The image of object O from mirror M2 is I2 and the
is equal to the distance between C and object. image of I2 (the virtual object) from mirror M1 is I4.
(2) (c) Here, [n] = 5 Þ n – 1 £ 5 £ n Notice that this interpretation, according to ray diagram
rules, is valid only for Fig. (A). All others are
360 360 inconsistent.
\ –1£5£
q q (8) (a) Angle betwen incident ray and mirror = 90º - 30º = 60º
360 360
or, q ³ or q £
6 5
\ 60º £ q £ 72º 60º 30° 30° X
(3) (d) For the given q = 50º, 30º
60º
360 360
n= = = 7.2
q 50
The integer value of (7.2) is 7.
Thus number of images formed is 7. By law of reflection Ð i = Ð r
(4) (a) The situation is illustrated in figure. So angle of reflection Ð r = 30º.
Hence angle between mirror and reflected ray = 60º
M2 (9) (b) As shown in figure, ray AB goes to mirror M1, gets
q reflected and travels along BC and then gets reflected
B
a C
by M2 and goes in CD direction. If the angle between
b a N2 M1 and M2 be a, then
N1 X
i M1
q 90º-i i
O A M1 E
a
B A
XA is the incident ray. BC is the final reflected ray. It a D
is given that BC is parallel to mirror M1. Look at the
a a a
assignment of the angles carefully. Now N2 is normal M2
O C
to mirror M2. Therefore b = q
Then from D OAB In D OBC , ÐOBC and ÐOCB are equal to a
q + b + 90º – i = 180º \ 3a = 180º
or q + q + 90º – i = 180º or i = 2q – 90º a = 60º
Thus if the angle of incidence is i = 2q - 90º, then the
final reflected ray will be parallel to the first mirror.
t.me/Ebooks_Encyclopedia27. t.me/Magazines4all

EBD_7156
DPP/ P 49 135
(10) (a) Here, Object distance , u = – 15cm (12) (d) u = –50 cm , f = 25cm
focal length , f = – 5 cm 1 1 1
Object height, h0 = 0.2 cm =- + ;
25 50 v
O 1 1 1 2 +1 3
= + = =
v 25 50 50 50
I 50
v= = 16.6 cm.
3
We know, mirror formula, 1 1 2
(13) (a) We know, + =
1 1 1
v u R
+ = uR
v u f Þv=
2u - R
uf dv (2u - R).R - uR.2 du
ÞV = = .
u-f dt (2u - R)2 dt
(-15)( -5) æ R ö du
2
= = – 7.5 cm = –ç .
-15 + 5 è 2u - R ÷ø dt
Again, magnification, 2
dv æ R ö
-7.5 \ Speed of image =ç
è 2u - R ÷ø
v 1 .
m =- =- =- dt
u -15 2
du æ R ö2
= V
Now, | m | =
hi dt çè 2u – R ÷ø 0
ho (14) (a) For shrot linear object du and dv represent size of
object and image respectively.
0.2
Þ hi = | m | h o = cm = 0.1 cm
2 \
\

Thus, the image is formed at 7.5 cm from the pole of


\

2
\ \

the mirror and its size is 0.1 cm. 1


(11) (a)
\ \ \ \ \

u b
P O u+ du
\ \
\\

C F I
1 1 1
We know, + =
v u f

v2
Þ dv = – .du
u2
Here, Object distance, u = – 30 cm
Focal length, f = + 20 cm v2
Þ |db| = |du|
1 1 1 u2
We know, mirror formula, + =
v u f 2
æ f ö
uf -30 ´ 20 =ç ÷ .b
Þv= = = + 12 cm èf–uø
u - f -30 - 20
Again, magnification , v 1 1 1 u u
15. (b) \ m = - .Also = + Þ = + 1
-v -12cm 2 u f v u f v
m= = =
u 30cm 5
u u -v f
Now, Image height = m × object height Þ- = 1- Þ =
v f u f -u
2
= ´ 0.5 cm = 0.2 cm f
5 so m =
Thus the image is formed behind the mirror at a distance f -u
of 12 cm from the pole. Image height is 0.2 cm.
t.me/Ebooks_Encyclopedia27. t.me/Magazines4all

136 DPP/ P 49
16. (b) Given u = – 15 cm, f = – 10 cm, O = 1 cm
-f
–n = Þ nf + nu = –f
1 1 1 1 1 1 1 1 -f - u
+ = , = - = -
v u f v f u -10 -15
-(n + 1)
\ v = – 30 cm nu = –f –nf Þu= f
n
I v -30 22. (d) Here image can be real or virtual. If the image is real
=- =- =–2
O u -15 f = –30, u = ?, m = –3
I = – 2 × 1 = – 2 cm f -30
Image is inverted and on the same side (real) of size 2 m= Þ –3 = ; u = – 40 cm.
f -u -30 - u
cm. If the image is virtual
17. (b) As shown in the figure, when the object (O) is placed
between F and C, the image (I) is formed beyond C. It f -30
m= Þ 3= Þ u = –20 cm.
is in this condition that when the student shifts his eyes f -u -30 - u
towards left, the image appears to the right of the object 23. (b) By keeping the incident ray is fixed, if plane mirror
pin. rotates through an angle q reflected ray rotates through
an angle 2q
Movement towards left

F
q
I C O
q
q

A
18. (a)
I
30
60º º P
30º

30º N
B
O Q 24. (d) Real image
O
Ð i = Ð r = 30º I Virtual object
\ Ð OIQ = 60º
\ IOQ = 90º – 60º = 30º
19. (d) The image formed by a convex mirror is always virtual.
20. (d) From the ray diagram.
M 25. (b,c) Convex mirror and concave lens form, virtual image
A 2L X N for all positions of object.
d/2 (26) (a); (27) (d)
D L B
d/2
X
O
In DANM and DADB
I
ÐADB = ÐANM = 90° 70 cm
ÐMAN = ÐBAN (laws of reflection)
Also ÐBAN = ÐABD
Þ ÐMAN = ÐABD 700 cm 100 cm 700 cm
\ DANM is similar to DADB
µ2 µ1 µ2 – µ1
– = , when x = 70 cm
x d /2 v u R
\ = or x = d
2L L
1.5 1.2 1.5 – 1.2
So, required distance = d + d + d = 3d. – =
v –70 20
f
21. (a) m = –n ; m = 20 ´ 70 ´ 1.5
f -u Þv=
–1.2 ´ 20 + 0.3 ´ 70
t.me/Ebooks_Encyclopedia27. t.me/Magazines4all

EBD_7156
DPP/ P 49 137
Þ v = – 700 cm Similarly, for x = 80 cm
µ2 µ1 µ2 – µ1
v = 80 cm
– = and for x = 90 cm
v u R
v = 70 cm
1.2 1.5 1.2 –1.5 28. (d)
– =
v 900 –20 29. (d) When an object is placed between two plane parallel
mirrors, then infinite number of images are formed.
900 ´ 200 ´ 1.2 Images are formed due to multiple reflections. At each
Þv=
1.5 ´ 200 – 900 ´ 3 reflection, a part of light energy is absorbed. Therefore,
Þ v = – 90 cm distant images get fainter.
30. (d) The size of the mirror does not affect the nature of the
image except that a bigger mirror forms a brighter
image.
t.me/Ebooks_Encyclopedia27. t.me/Magazines4all

138 DPP/ P 50
DAILY PRACTICE
PROBLEMS
PHYSICS
SOLUTIONS 50
(a) We know that d = A (m – 1)
( )
(1) 2 2
Þ sin2 a < m1 - m 2 sin a < m12 - m22
d
or m = 1 +
A amax = sin–1 m12 - m22
Here A = 6 °, d = 3° , therefore
3 B
m=1+ = 1.5
6
i
(2) (c) According to given problem
A = 30°, i1 = 60° and d = 30° and as in a prism
r
d = (i1 + i2) – A, 30° = (60 + i2) – 30 i.e., i2 = 0 a O A
So the emergent ray is perpendicular to the face from
which it emerges.
Now as i2 = 0, r2 = 0 (6) (b) From the information given, it is clear that the apparent
But as r1 + r2 = A , r1 = A = 30° depth is 2.58 mm and the real depth is 4mm. Therefore,
So at first face the refractive index will be
1× sin 60° = m sin 30° i.e., m = 3 R 4
m= = = 1.55
c c 3 A 2.58
(3) (a) As are know, m = Þ v = = × 3 × 108 (7) (d) The apparent shift of the bottom point upwards will be
v m 4
x = x1 + x2
= 2.25 × 108 m/s
As, c = n l0 and v = n l æ 1ö æ 1 ö
= t1 ç 1 - m ÷ + t2 çè 1 - m ÷ø
v 1 è 1ø 2
Þ l / l0 = =
c m
æ 1 ö æ 1 ö
3 = 4 çè 1 - (4 / 3) ÷ø + 2 çè 1 - (3 / 2) ÷ø
i.e., l = l0/m = × 6000Å = 4500Å
4
(4) (d) Herer + 90º + r’ = 180º æ 3ö æ 2ö
= 4 çè 1 - ÷ø + 2 çè 1 - ÷ø = 1.67 cm.
Þ r’ = 90º – r 4 3
or, r’ = (90º – i) as Ð i = Ðr
C
Now, according to Snell’s law : (8) (d) Since v =
sin i = m sin r’ = m sin (90º – i) n
or, tan i = m The time taken are
or, i = tan–1 m = tan–1 (1.5) 20 (1.63) 20 (1.47)
(5) (b) Here the requirement is that i > c t2 = , t1 =
C C
m2 Therefore , the difference is
Þ sin i > sin c Þ sin i > m
1 20(1.63 - 1.47) 20 ´ 0.16
t2 - t1 = = = 1.07 × 10–8 sec.
sin a C 3 ´ 108
From Snell's law m1 = (9) (b) As the beam just suffers TIR at interface of region III
sin r
Also in DOBA and IV
r + i = 90° Þ r = (90 – i) Region I Region II Region III
Hence from equation (ii)
sin a = m1 sin(90 – i)
q n0 n0 n0
sin a
Þ cos i = m n0 2 6 8
1
2 q 0.2 m 0.6 m
æ sin a ö
sin i = 1 - cos 2 i = 1 - ç ÷ ...(iii) n n
è m1 ø nosin q
n0
sin q1 = 0 sin q2 = 0 sin 90°,
From equation (i) and (ii) 2 6 8
t.me/Ebooks_Encyclopedia27. t.me/Magazines4all

EBD_7156
DPP/ P 50 139
If the net deviation for the mean ray is zero,
1 -1 1
sin q Þ q = sin (m – 1) A = (m' – 1) A'
8 8
(10) (b) The ray of light returns back from the polished face (m - 1) 1.517 - 1
or, A' = A= ´ 5° = 4.2°.
AC. (m '- 1) 1.620 - 1
The angular dispersion produced by the crown prism
A is
dv – dr = (mv – mr) A
E 30º and that by the flint prism is
i 60º d 'v – d 'r = (m'v – m'r) A'
D
30º The net angular dispersion is ,
F d = (mv – mr) A – (m'v – m'r) A'
= (1.523 –1.514) × 5° – (1.632–1.613) × 4.2°
B C = – 0.0348°.
The angular dispersion has magnitude 0.0348°.
\ ÐADE = 90º. From the figure it is clear that the angle (14) (a) mv = 1.5230, mr = 1.5145, w = ?
of refraction at face AB is 30º. Hence from Snell’s law Mean refractive index,
sin i m v + m r 1.5230 + 1.5145
m= =
sin r m=
2 2
sin i m = 1.5187
As m = 2 and r = 30º \ 2 = sin 30º m v - mr
w=
m -1
2 1
or sin i = = = sin 45º 1.5230 - 1.5145 0.0085
2 2 = = = 0.0163
1.5187 - 1 0.5187
\ i = 45º
(11) (a) A = r1 + r2 = 60 .......(1) (15) (c) Here, w = 0.021; m = 1.53; w ' = 0.045;
In minimum deviation position m' = 1.65; A' = 4.2°
r1 = r2 .......(2) For no dispersion, wd+w'd'=0
From eqs. (1) and (2) or w A (m – 1) + w ' A ' (m – 1) = 0
A = 2r1 = 60º w ' A ' ( m '- 1)
\ r1 = 30º or A = –
w ( m - 1)
sin i sin 50º 0.045 ´ 4.2 ´ (1.65 - 1)
\n= = = 1.532 =– = – 11.04°
sin r sin 30º 0.021 ´ (1.53 - 1)
A + dm 60 + d m Net deviation,
sin sin d + d ' = A (m – 1) + A' (m ' – 1)
n= 2 or 1.532 = 2
= – 11.04 (1.53 –1) + 4.2 (1.65 –1)
A sin 30 °
sin = – 11.04 × 0.53 + 4.2 × 0.65
2
= – 5.85 + 2.73 = 3.12°
60 + d m 1.532 (16) (d) At first face of the prism as i1 = 0,
sin = = 0.766 sin 0 = 1.5 sin r 1 i.e., , r1 = 0
2 2
And as for a prism
\ dm = 40º r1 + r2 = A so r2 = A ......(1)
(12) (b) d1 = d2 But at second face, as the ray just fails to emerge
(m1 – 1) A1 = (m2 – 1) A2
i.e., r2 = qC ......(2)
(1.5 – 1) 6 = (1.6 – 1) A2
So from Eqn,.(1) and (2)
A2 = 5º
A = r2 = q C
(13) (a) The deviation produced by the crown prism is
d = (m – 1) A é1ù é2ù
and by the flint prism is But as qC = sin –1 ê ú = sin –1 ê ú = 42°
ë ûm ë3û
d ' = (m' – 1) A' So A = 42°
The prisms are placed with their angles inverted with (17) (a) Here, mb= 1.532 and mr = 1.514 A = 8° .
respect to each other . The deviations are also in Angular dispersion
opposite directions. Thus, the net deviation is = (mb – mr) A = (1.532 – 1.514) × 8
D = d – d ' = (m – 1) A – (m' – 1) A' ...........(1) = 0.018 × 8 = 0.144° .
t.me/Ebooks_Encyclopedia27. t.me/Magazines4all

140 DPP/ P 50
(18) (c) Velocity of the ball when it reaches at the height of 12.8 (21) (c) dPrism = (µ - 1)A = (1.5 - 1)4° = 2°
m. above the surface is v = 2 ´10 ´ 7.2 = 12 m/s
\ dTotal = dPrism + dMirror
Height of the ball from surface as seen by fish
= (µ – 1)A + (180 – 2i) = 2° + (180 – 2 × 2) = 178°
dh ¢ dh (22) (a) From the following figure
h¢ = mh Þ =m
dt dt
u=0
r i
20 u = 12 m/s a
12.8 m n
r + i = 90° Þ i = 90° – r
For ray not to emerge from curved surface i > C
Þ sin i > sin C Þ sin (90° – r) > sin C Þ cos r > sin C
1 ì 1ü
4 Þ 1 - sin 2 r > í\ sin C = ý
Þ v¢ = mv = ´ 12 = 16 m/s. n î nþ
3
(19) (a) Suppose, the angle of the crown prism needed is A and
that of the flint prism is A’. We have
Þ 1-
n
sin 2 a
2
n
>
1
2 n
Þ 1>
1
2 (1 + sin 2 a )
m - mr Þ n2 > 1 + sin2 a Þ n > 2 {sin i ® 1}
w= v or,, mv – mr = (m – 1) w
m -1 Þ Least value = 2
The angular dispersion produced by the crown prism
is (23) (a) w depends only on nature of material.
(mv – mr) A = (m –1) w A (24) (a)
Similarly , the angular dispersion produced by the flint (25) (c);
prism is (m´ – 1) w´ A´ 26. (b), 27 (a)
For achromatic combination , the net dispersion should The normal shift produced by a glass slab is,
be zero. Thus ,
(m – 1) w A = (m´ – 1) w´ A´ æ 1ö æ 2ö
Dx = ç1 - ÷ t = ç1 - ÷ (6) = 2cm
è mø è 3ø
A ' (m - 1) w 0.517 ´ 0.03
or , = = = 0.50 ......(1)
A (m '- 1) w ' 0.621 ´ 0.05 i.e., for the mirror, the object is placed at a distance
(32 – Dx) = 30 cm from it.
The deviation in the yellow ray produced by the crown
Applying mirror formula,
prism is d = (m – 1) A and by the flint prism is d´ = (m´
– 1) A´. The net deviation produced by the combination 1 1 1
+ =
is v u f
d – d´ = (m – 1) A – (m´ – 1) A´
or 1° = 0.517 A – 0.621 A´ ......(2) 1 1 1
+ =-
Solving (1) and (2 ), A = 4.8° and A´ = 2.4°. Thus, the v 30 10
crown prism should have its refracting angle 4.8° and or, v = – 15 cm
that of the flint prism should be 2.4°. (a) When x = 5 cm: The light falls on the slab on its return
(20) (a) For TIR at AC journey as shown. But the slab will again shift it by a
q>C distance.
B A
Þ sin q ³ sin C q
6 cm
1 q
Þ sin q ³
w µg I
µw C Dx
Þ sin q ³
µg

8 15 cm
Þ sin q ³
9
Dx = 2 cm. Hence, the final real image is formed at a
distance (15 + 2) = 17 cm from the mirror.
t.me/Ebooks_Encyclopedia27. t.me/Magazines4all

EBD_7156
DPP/ P 50 141
(b) When x = 20 cm: This time also the final image is at a (29) (b) The velocity of light in a material medium depends upon
distance of 17 cm from the mirror but it is virtual as its colour (wavelength). If a ray of white light is incident
shown. on a prism, then on emerging the different colours are
deviated through different angles.

Also dispersive power w =


( mV - m R )
( mY - 1)
i.e. w depends upon only m
I
(30) (c) Apparent shift for different coloured letter is
Dx
æ 1ö
d = h ç1 - ÷
è mø
(28) (b) As rays of all colours emerge in the same direction (of Þ lR > lV so mR < mV
incidence of white light), hence there is no dispersion, Hence dR < dV i. e. red coloured letter raised least.
but only lateral displacement in a glass slab.
t.me/Ebooks_Encyclopedia27. t.me/Magazines4all

DPP/ P 51 141

DAILY PRACTICE
PROBLEMS
PHYSICS
SOLUTIONS 51
1. (a). According to sign convention, it is given that
2 - 60 40
u = – 0.6 m, R = 0.25 m, µ1 = 1 (air), µ2 = 1.5 v= ´ =- = – 3.63 cm
3 11 11
\ AI = 3.63 cm

3. (c).

u v
Therefore, using x1 f f x2
µ1 µ2 µ2 - µ1 , we get
- + = • • • • •
u v R O F1 F2 I
1.5 1 1.5 - 1 1 0.5
= + =– +
v (- 0.6) 0.25 0.6 0.25

5 1
=– +2= As in case of thin lens the distance of either foci from the
3 3
optical centre is f ,
Þ v = 4.5 m
| u | = ( f + x1 ) and | v | = ( f + x2 )
The image is formed on the other side of the object (i.e.
inside the refracting surface). Substituting thses values of u and v in lens formula with
2. (a). On viewing from the closer surface A (near to object) proper sign
: 1 1 1
- =
The final image is formed at I. ( f + x 2 ) - ( f + x1 ) f
x1 + x 2 + 2f 1
=
or ( f + x1 )( f + x 2 ) f
i.e., fx1 + fx2 + 2f 2 = f 2 + fx1+ fx2 + x1 x2
O C or, x1 x2 = f 2
A
A
1cm
· B
I O C Far 4. (b). According to Lens-maker’s formula :
Nearing surface
surface 1 é 1 1 ù mL
= (m – 1) ê - ú with m =
f R
ë 1 R 2û m M

From sign convention Here f = 30 cm and R1 =10 cm and R2 = ¥


u = OA = – 4 cm, v = ?
1 é1 1ù
R = AC = – 5cm So, =(m–1) ê - ú
30 ë10 ¥ û
µ2 2
µ= µ = 3 3m – 3 = 1 or, m = ( 4 / 3 )
1 5. (a). For combination of lenses

m 1 µ -1 2/3 1 2/3 -1 1 1 1 1 1 25 1
- = Þ v +4 = -5 = + = + = =
v u R f f1 f 2 10 15 150 6
Therefore, f = 6 cm.
2 1 1 4 - 15 - 11
Þ = - = =
3v 15 4 60 60
t.me/Ebooks_Encyclopedia27. t.me/Magazines4all

EBD_7156
142 DPP/ P 51
6. (c). Let f2 is the focal length of the diverging lens. Then , 9. (b). The focal length of lens in water is given by
1 1 1
= + a mg -1 1.2 - 1
f f1 f2 fl = fa = 1.2 fa
a mg -1
-1 1.33
It is given that f1 = +20 cm, f = 30 cm a ml
1 1 1
= + 0.2 ´ 1.33
30 20 f 2 fl = - fa
0.13
1 1 1 2-3 1 Hence f is negative and as such it behaves as a divergent
or f = 30 - 20 = 60 = - 60 lens.
2
10. (a). The focal length of an equiconvex lens is given by
Thus f2 = – 60 cm
7. (a). As radius of curvature of silvered surface is 22 cm, so, 1 2 (m - 1)
=
f R

1
It is given that = + 5 and m = 1.5
f

2 (1.5 - 1)
Therefore, 5 =
R

1
or R = metre = 20 cm
5
11. (b). The question is based on the conventional method of
R -22 measurement of focal length by displacement method.
fM = = = –11 cm = – 0.11 m According to this method where D is the distance between
2 2
object and the image, and x is the displacement given to
1 1 1 the object.
and hence, PM = – = = D
f M -0.11 0.11 From the data x = 25 cm and D = 75 cm .
Further as the focal length of lens is 20 cm, i.e. 0.20 m, its Thus
power will be given by : (75)2 - (25) 2 (75 - 25) (75 + 25)
f= =
1 1 4 ´ 75 4 ´ 75
PL = = D
f L 0.20
50 ´ 100 50
Now as in image formation, light after passing through the = = = 16.7 cm
4 ´ 75 3
lens will be reflected back by the curved mirror the lens
again P = PL + PM + PL = 2PL + PM A1 A2
12. (c). m1 = and m2
2 1 210 O O
i.e. P = + = D A1A 2
0.20 0.11 11 Þ m1m2 =
So the focal length of equivalent mirror O2
Also it can be proved that m1m2 = 1
1 11 110
F=– =- m=- cm So O = A1A 2
P 210 21
i.e. the silvered lens behaves as a concave mirror of focal 13. (c). Effective power P = P1 + P2 = 4 – 3 = 1D
length (110/21) cm. So for object at a distance 10 cm in 14. (d). One part of combination will behave as converging
front of it, lens and the other as diverging lens of same focal length.
As such total power will be zero.
1 1 21 15. (c). Let the image of an object at O is formed at the same
+ =- i.e. v = –11 cm
v -10 110 point as shown in figure. The distance of O from the plane
i.e. image will be 11 cm in front of the silvered lens surface is x. The rays suffer refraction at first surface
8. (b). On covering the lens half by a black paper will reduce (curved) as they reach lens. After wards become parallel
the intensity of image and not the part of image. So full and gets reflected form plane surface and so retrace the
image is formed. path and image is formed at O itself.
t.me/Ebooks_Encyclopedia27. t.me/Magazines4all

DPP/ P 51 143

20 ´ 100 cm 40 ´ 100
So – =
3.5 cm u

x i.e., u = – 7 cm
· i.e., film is at a distance of 7 cm in front of projection lens.
O
And from lens formula 1 - 1 = 1 , here we have
v u f

1 1 1
m 1 m -1 - = or f @ 7 cm = 70 mm
- = 4000 -7 f
v u R
[as ( 1/4000) << (1/7) ]
u = – x, v = ¥ i.e., focal length of projection lens is 70 mm.
m 1 m -1 20. (a). As final image is at 25 cm in front of eye piece
+ =
¥ x R 25
1 1 1
- = i.e., ue = –
R -25 u e 5 6
x=
m -1
ve -25
As such O behaves as equivalent to centre of curvature of And so, me = = =6 ...(1)
u e (-25 / 6)
equivalent concave mirror.
Now for objective,
R v = L – ue = 20 – (25/6) = (95/6)
\ Radius = x =
m -1 So if object is at a distance u from the objective,
16. (a). As here f = 10 cm and v = 5m = 500 cm
6 1 1 95
- = i.e., u = – cm
1 1 1 95 u 0.95 94
So from lens formula - = , we have
v u f i.e. object is at a distance (95/94) cm in front of field lens.

1 1 1 é 500 ù v (95 / 6) é 94 ù
- = i.e., u=– ê cm = = -ê ú
ë 49 úû
Also, m = ...(2)
500 u 10 u (-95 / 94) ë6û

v 500 So total magnification,


So that m= = = – 49
u -500 / 49 é 94 ù
M = m xme = – ê ú × (6) = – 94
Here negative sign means the image is inverted with respect ë6û
to object. Now as here object is (2 cm × 2cm ) so the size
i.e., final image is inverted, virtual and 94 times that of
of picture on the screen
object.
Ai = ( 2 × 49 cm) × ( 2 × 49cm) = ( 98 × 98) cm2
21. (a). As object is distant, i.e., u = – ¥ , so
17. (a). As power of a lens is reciprocal of focal length in m,
1 1 1
1 1 - = i.e. v = f0 = 12 cm
P= = diopter = 20 D v -¥ f 0
-2 0.05
5 ´ 10 m
i.e. objective will form the image IM at its focus which is
18. (b). For relaxed eye, MP is minimum and will be
at a distance of 12 cm from O. Now as eye- piece of focal
D 25 length – 4 cm forms image I at a distance of 24 cm from it,
MP = = = 5×
f 5 24
1 1 1
While for strained eye, MP is maximum and will be - = Þ ue = = 4.8 cm.
-24 u e -4 5
D i.e, the distance of IM from eye lens EA is 4.8 cm. So the
MP = 1 + = 1 + 5 = 6×
f length of tube L = OA – EA = 12 – 4.8 = 7.2 cm.

I v
19. (a). As in case of projector, m = =
O u
t.me/Ebooks_Encyclopedia27. t.me/Magazines4all

EBD_7156
144 DPP/ P 51
26. (a) For lens L2, image must form at centre of curvature of
the curved surface after refraction through plane part
I f0=12 cm
µ2 µ
+ w =0
> f0 fe – R2 x '
4.8 cm

q0
> q
ue Þ x¢ = 8 cm
>> O E A
(
)
>>
(
)q IM
27. (b) Length of tube = x + x¢ = 18 cm
q0
B 28. (a) Focal length of lens immersed in water is four times
L= 7.2 cm >> >
the focal length of lens in air. It means
24 cm
fw = 4fa = 4×10 =40 cm
29. (d) Focal length of the lens depends upon its refractive
22. (d) In case of astronomical telescope if object and final
image both are at infinity. 1
index as µ ( m - 1) .
MP = – (f0/fe) and L = f0 + fe f
So here –(f0/fe) = – 5 and f0 + fe = 36
Since mb > mr so fb < fr
Solving these for f0 and fe, we get
Therefore, the focal length of a lens decreases when red
f0 = 30 cm and fe = 6 cm
light is replaced by blue light.
1 30. (b) The light gathering power (or brightness) of a telescope
23. (a) Resolving power of microscope µ
l µ (diameter)2. So by increasing the objective diameter even
far off stars may produce images of optimum brightness.
24. (a) Light gathering power µ Area of lens aperture or d2
25. (b) For lens L1, ray must move parallel to the axis after
refraction
µ1 µw µ1 – µw
+ = Þ x = 10cm
¥ x R1
t.me/Ebooks_Encyclopedia27. t.me/Magazines4all

DPP/ P 52 145

DAILY PRACTICE
PROBLEMS
PHYSICS
SOLUTIONS 52
From eqs. (1) and (2)
I1 9
1. (d) =
I2 1 2 ´ 51 ´ 10-8
d= = 5.1 × 10-4 m
2 ´ 10-3
2
é I1 ù
ê + 1ú 2 Imax
-1
Imax

I2 ú = é 9 + 1ù Imax - I min I
ê ú ê ú 7. (b) V = = min .... (1)
Imin I1 ë 9 - 1û Imax + Imin Imax
ê - 1ú +1
ëê I2 ûú Imin

Imax 42 4 æ I1 ö
2
= =
Imin 22 1 ç +1 ÷
Imax ç I2 ÷
2. (a) a1 = 6 units, a2 = 8 units =
Imin ç I1 ÷ .... (2)
ç -1 ÷
2 ç I ÷
é a1 ù 2 è 2 ø
é6 ù
ê + 1ú +1
Imax ë a 2 û
= = ëê 8 ûú According to question
Imin é a 2 2
ù é6 ù I1 1
1
ê a - 1ú êë 8 - 1úû = .... (3)
ë 2 û I2 4
From eqs. (2) and (3)
Imax 49
= 2
Imin 1 æ 1 ö 9
ç +1÷
3. (b) The separation between the sucessive bright fringes Imax 4 ÷ = 4 =9 .... (4)

is- Imin ç 1 ÷ 1
ç -1 ÷
Dl 1´ 600 ´10 -9 è 4 ø 4
b= =
d 0.1 ´ 10 -3 From eqs. (1) and (4)
b = 6.0 mm.
4. (b) wa = l/d [9 - 1] 8
V= = = 0.8
[9 + 1] 10
(w a ) water l water
\ wa µ l Þ =
wa l (m - 1)t (1.6 - 1) ´ 7 ´ 10 -6
8. (c) l = =
n 7
(w a ) water l
Þ = l = 6 × 10–7 meter Þ l = 6000 Å
wa m water l
1
Þ (wa)water = 0.15° 9. (d) b µ \ d On increasing d three times
d
5. (b) I' = I1 + I2 + 2 I1 I2 cos f b will become 1/3 times.
10. (b) \ PR = d Þ PO = d secq and CO = PO cos 2q
I1 = I, I2 = 9I, f = p
= d sec q cos 2q is
I' = I + 9I + 2 9I2 cos p = 10I – 6I = 4I O
Q R
Dl qq
6. (a) d = .... (1)
b
C
According to quesion.
l = 5100 × 10-10m
q
2
b= × 10–2 m .... (2) A
10 P
D = 2m, d = ?. B
t.me/Ebooks_Encyclopedia27. t.me/Magazines4all

EBD_7156
146 DPP/ P 52
Path difference between the two rays
D = CO + PO = (d sec q + d secq cos 2q) P
Phase difference between the two rays is
f = p (One is reflected, while another is direct)
Therefore condition for constructive interference Y
S1
should be
q q
l 3l d
D = , .......
2 2
l S2
or d sec q(1 + cos 2q) = Dx
2
D
or
d
cos q
(2 cos 2 q =
l
2
)Þ cos q =
l
4d As Dx = d sin q
11. (b) n1l1 = n2l2
10 × 7000 = n2 × 5000 2p
Phase difference f = (Path difference)
n2 = 14 l
12. (a, c) Path difference between the rays reaching infront of
2p 2p
slit S1 is. = ( d sin q ) = (150 sin q) = p sin q
S1P – S2P = (b2 + d2)1/2 – d l 300
For distructive interference at P IR = I1 + I2 + 2 I1I2 cos q
S1 P Here I1 = I2 and f = p sin q
æ p sin q ö
\ IR = 2I1 [(p sin q)] = 4I1 cos2 ç ÷
è 2 ø
b
æ p sin q ö
S2 IR will be maximum when cos2 ç ÷ =1
d è 2 ø
\ (IR)max = 4I1 = Io
( 2n - 1) l
S1P – S2P = æ p sin q ö
2 Hence I = Io cos2 ç ÷
è 2 ø
( 2n - 1) l If q = 0, then I = Io cosq = Io
i.e., (b2 + d2)1/2 – d =
2 If q = 30°, then I = Io cos2 (p/4) = Io/2
1/ 2 If q = 90°, then I = Io cos2(p/2) = 0
æ b2 ö ( 2n - 1) l
Þ d ç1 + ÷ -d = 15. (b) I = R2 = a12 + a22 + 2a1 a2 cos d
ç d2 ÷ 2
è ø
p
= I + 4I + 4I cos = 5I
æ b2 ö ( 2n - 1) l 2
Þ d ç1 + + ...... ÷ - d = 16. (b) Suppose the amplitude of the light wave coming from
ç 2d 2 ÷ 2
è ø the narrower slit is A and that coming from the wider
(Binomial Expansion) slit is 2A. The maximum intensity occurs at a place
where constructive interference takes place. Then the
b ( 2n - 1) l b2 resultant amplitude is the sum of the individual
Þ = Þ = 2n - 1 d
2d 2 ( ) amplitudes.
Thus,
b2 b 2 Amax = 2A + A = 3A
For n = 1,2 ............, l =
,
d 3d The minimum intensity occurs at a place where
13. (b) Distance of mth bright fringe from central fringes is destructive interference takes place. The resultant
mDl amplitude is then difference of the individual
Xm = amplitudes.
d
Thus, Amin = 2A – A = A.
c 3×108 Imax (Amax )2 (3A)2
14. (a, b) For microwave l = = = 300 m \ = = =9
f 106 Imin (Amin )2 (A)2
t.me/Ebooks_Encyclopedia27. t.me/Magazines4all

DPP/ P 52 147

I1 2 lD
17. (b) = 23. (a) Fringe width b =
I2 1 d
a1 I 2 According to de Broglie,
= 1 =
a2 I2 1 h h
Wavelength l = =
At the point of constructive interference, the resultant p 2meV
amplitude becomes (a1 + a2) = 2 + 1 at the point of As V increases, l decreases, b decreases.
destructive interference, the resultant amplitude is 1
Also b µ and b µ D .
(a1 – a2) = 2 –1 d
Imax (a1 + a 2 ) 2 ( 2 + 1) 2 (m - 1)tD
\ I = = = 34 24. (a) n =
min (a1 - a 2 ) 2 ( 2 - 1) 2 d
18. (d) For destructive interference : lD D b
Path difference= S1 P – S2 P = (2n –1) l/2. but b = Þ =
d d l
For n = 1, S1 P – S2 P= (2 × 1 – 1) l/2 = l/2.
n = (m – 1) t b/l
n =2, S1 P– S2 P = (2 × 2 – 1) l/2 = 3l/2.
20b = (m – 1) 2.5 × 10–3 { b/5000 × 10–8}
n = 3, S1 P – S2 P = (2× 3–1) l/2 = 5l/2.
n = 4, S1P – S2 P = (2 × 4 – 1) l/2 = 7l/2. 20 ´ 5000 ´ 10-8
n = 5,S1 P – S2 P = (2 × 5 – 1) l/2= 9l/2. m–1= = 0.4
n =6,S1 P – S2 P = (2× 6– 1) l/2 = 11l/2. 2.5 ´ 10-3
So, destructive pattern is possible only for path m = 1.4.
difference = 11l/2. dy
19. (b) The distance of a bright fringe from zero order fringe is 25. (a) S1P – S2P =
D
given by-
For central maxima,
nlD
Xn = dy
d – d sin f = 0
Dx = (n 0 + kt)
D & d is constant D
n1 l1 = n2 l2
n1 = 16, l1 = 6000 Aº, l2 = 4800 Å P
n1l1 16 ´ 6000
n2 = = = 20
l2 4800 y
20. (c) n1 l1 = n2 l2 for bright fringe 2
n = (n0 + kt)
n (7.5 × 10–5) = (n + 1) (5 × 10–5)
d f O
5.0 ´ 10-5 l
n= = 2.
2.5 ´ 10-5 S1

nlD 3lD
21. (b) Xn = or X3 = D
d d

3 ´ (5000 ´10-8 cm) ´ 200cm


X3 = = 1.5 cm. D sin f
0.02cm \y = ( y -coordinates of central maximum).
n0 + kt
2 dy – kD sin f
Imax æa +a ö a +a 26. (b) = = velocity of central maximum.
22. (b, d) =9 Þ ç 1 2 ÷ =9 Þ 1 2 =3 dt (n0 + kt )2
Imax è a1 – a 2 ø a1 – a 2
27. (d) For central maxima to be formed at O
a1 3 + 1 a1 æn ö
Þ = Þ a = 5 There I1 : I2 = 4 : 1 n ' ç –1÷ b = d sin f
a2 3 – 1 2 è n' ø
t.me/Ebooks_Encyclopedia27. t.me/Magazines4all

EBD_7156
148 DPP/ P 52
Here n' = n0 + kt, n = refractive index of plate. when a white light as a source is used, the central fringe
d sin f is white around which few coloured fringes are
n = n0 + kt + observed on either side.
b
28. (d) When d is negligibly small, fringe width b which is 30. (d) When one of slits is covered with cellophane paper,
proportional to 1/d may become too large. Even a single the intensity of light emerging from the slit is decreased
fringe may occupy the whole screen. Hence the pattern (because this medium is translucent). Now the two
cannot be detected. interfering beam have different intensities or
29. (d) In Young's experiments fringe width for dark and white amplitudes. Hence intensity at minima will not be zero
fringes are same while in Young's double slit experiment and fringes will become indistinct.
t.me/Ebooks_Encyclopedia27. t.me/Magazines4all

148 DPP/ P 53
DAILY PRACTICE
PROBLEMS
PHYSICS
SOLUTIONS 53
1. (b). The distance of first diffraction minimum from the
central principal maximum x = lD/d 12.5 ´ 10-3
q = l/a =
2
x l l
\ sin q = = Þd=
D d sin q 12.5 ´ 10-3 12.5 ´ 10-3 ´ 10-3
l= ×a=
2 2
5000 ´ 10-8 l = 6.25 × 10–6 m = 6250 mm
Þd= = 2 × 5 × 10–5
sin 30º 6. (a). Slit width = a = 0.2mm,
Þ d = 1.0 × 10–4 cm.
2ld
2. (b). Here, l1 = 5890Å = 5890 × 10–10 m b=
l2 = 5896 Å = 5896 × 10–10 m a
a =2mm = 2 × 10–6 m, D = 2m b 2l
Angular width Wq = =
3l1 x1 D a
For first maxima, sin q = =
2a D 2 ´ 6328
q= = 0.36°
3l1D 3l 2 D 0.2
Þ x1 = and x2 = 7. (a). Here distance of the screen from the slit,
2a 2a
D = 2m, a = ?, x = 5 mm
\ spacing between the first maxima of two sodium lines
= 5 × 10–3 m,l = 5000Å
3D = 5000 × 10–10 m
= x2 – x1 = (l – l1) For the first minima, sin q = l/a = x/D,
2a 2

3 ´ 2(5896 - 5890) ´ 10-10 Dl 2 ´ 5000 ´ 10-10


a= = = 2 × 10–4 m
= =9× 10–4 m. x 5 ´ 10-3
2 ´ 2 ´ 10-6
8. (d). Here, l = 6500Å = 6.5 × 10–7 m, a = 0.5 mm =
ax 5 × 10–4 m,
3. (d). = nl
f D = 1.8 m
Angular separation of two dark bands on each side of central
ax 0.3 ´ 10-3 ´ 5 ´ 10-3 bright band 2q = 2l/a
l= =
n.f 3 ´1 Actual distance between them,
–7
l = 5 × 10 m 2x = 2l/a x D
l = 5000Å 2 ´ 6.5 ´ 10-7 ´ 18
2x =
æ lö 5 ´ 10-4
4. (b). q = sin–1 çè ÷ø .....(1)
a 2x = 4.68 × 10–3 m
According to question 2fl
l = 2 × 10–3 m 9. (c). Width of central maxima =
a
a = 4 × 10–3 m .....(2)
From equation (1) and (2) 2 ´ 2 ´ 6000 ´ 10-10
= = 12 mm
æ 1ö 0.2 ´ 10-3
q = sin–1 çè ÷ø
2 l
q = 30° 10. (a). q = .....(a)
a
5. (a). Here the width of principal maxima is 2.5 mm, therefore
its half width is x
q= .....(b)
f
b 2.5
= = 1.25 × 10–3m From eqs. (a) and (b)
2 2
l x
=
b / 2 12.5 ´ 10 -3 a f
Diffraction angle q = =
D 2 fl
\aq=l x= .....(c)
a
t.me/Ebooks_Encyclopedia27. t.me/Magazines4all

EBD_7156
DPP/ P 53 149
According to question x = ?, f = 40 cm Now, if I0 is the intensity of unpolarised light incident
l = 5896 × 10–8 cm on polaroid P1, the intensity of light transmitted through
a = 0.5 × 10–1 cm .....(d) it,
From eqs. (c) and (d)
1 1 W
-8 I1 = I0 = I0 = (32) = 16 2 ....(2)
40 ´ 5896 ´ 10 2 2 m
x= 96 = 0.047cm
5 ´ 10-2 Now as angle between transmission axes of polaroids
P1 and P2 is q, in accordance with Malus law, intensity
1.22l 1.22l of light transmitted through P2 will be
11. (a). dq = or a =
a dq I2 = I1 cos2q = 16 cos2q [from Eq. (2)] ....(3)
According to question And as angle between transmission axes of P2 and P3
is f, light transmitted through P3 will be
10-3 ´ p
dq = 10–3 degree = Radian, I3 = I2cos2f = 16 cos2q cos2f [from Eq.(3)]
180 Above equation in the light of (1) becomes,
l = 5 × 10–5 I3 = 16 cos2q cos2 (90º – q) = 4(sin2q)2 ....(4)
1.22 ´ 5 ´ 10-5 ´ 180 According to given problem, I3 = 3 W/m2
a=
10-3 ´ 3.14 So, 4(sin 2q)2 = 3 i.e., sin2q = ( 3 /2)
a = 3.5 cm or 2q = 60º i.e. q = 30º
12. (a). Since the reflected light is very highly polarised, it Further in accordance with Eq. (4), I3 will be max. when
implies that incident light falls at polarising angle of sin 2q = max., i.e.,
incidence qP. From Brewster's law, sin 2q = 1 or 2q = 90º, i.e., q = 45º
m = tanqp 16. (c) In double refraction light rays always splits into two
\ qP = tan–1 (m) = tan–1 (4/3) = 53.1º rays (O–ray & E–ray). O–ray has same velocity in all
Since qP is the angle which the rays from sun make direction but E– ray has different velocity in different
with the normal to the interface, angle with the interface direction.
will be 90º – 53.1º = 36.9º.
For calcite me < m0 Þ ve > v0
13. (a). Angle of incident light with the surface is 30º. Hence
angle of incidence = 90º – 30º = 60º. Since reflected For quartz me > m0 Þ v0 > ve
light is completely polarised, therefore, incidence takes 17. (c) At polarizing angle, the reflected and refracted rays
place at polarising angle of incidence qp. are mutually perpendicular to each other.
\ qp = 60º 18. (d) The amplitude will be A cos60° = A/2
Using Brewster's law
m = tan qp = tan 60º 2lD
19. (c) Width of central maxima =
\ m= 3 d
14. (d). If unpolarised light is passed through a polariod P1, its
2 ´ 2.1 ´ 5 ´10 -7
intensity will become half. = =1.4×10–3 m = 1.4 mm
0.15 ´ 10 -2
1
So I1 = I with vibrations parallel to the axis of P1. 20. (a) Using d sinq = nl, for n = 1
2 0
Now this light will pass through second polaroid P2
l 550 ´ 10-9
whose axis is inclined at ana angle of 30º to the axis of sin q = = =10–3 = 0.001 rad
P1 and hence, vibrations of I1. So in accordance with d 0.55 ´ 10 -3
Malus law, the intensity of light emerging from P2 will
be A 1
21. (b) A = np dl Þ nd = = constant Þ n µ
pl d
2
æ 1 ö æ 3ö 3 (n = number of blocked HPZ) on decreasing d, n
I2 = I1 cos2 30º = çè I0 ÷ø ç ÷ = I0
2 2
è ø 8 increases, hence intensity decreases.
I2 3 I0
= 22. (b) Intensity of polarized light =
I0 8 = 37.5 % 2
15. (a). If q is the angle between the transmission axes of first I0 I 0
polaroid P1 and second P2 while f between the Þ Intensity of untransmitted light = I 0 - =
2 2
transmission axes of second polaroids P2 and P3, then
23. (a)
according to given problem,
q + f = 90º or f = (90º – q) ....(1)
t.me/Ebooks_Encyclopedia27. t.me/Magazines4all

150 DPP/ P 53
24. (a) It magnitude of light vector varies periodically during 28. (a) When a polaroid is rotated in the path of unpolarised
it's rotation, the tip of vector traces an ellipse and light light, the intensity of light transmitted from polaroid
is said to be elliptically polarised. This is not in nicol remains undiminished (because unpolarised light
prism. contains waves vibrating in all possible planes with
rotated in path of plane polarised light, its intensity
rn2 will vary from maximum (when the vibrations of the
25. (a) Multiple focii of zone plate given by f p =
( 2 p - 1) l , plane polarised light are parallel to the axis of the
polaroid) to minimum (when the direction of the
where p = 1, 2, 3 ........ vibrations becomes perpendicular to the axis of the
26. (a) Angular width is the angle subtended by the distance crystal). Thus using polaroid we can easily verify that
between first minima on either side at the centre of the whether the light is polarised or not.
slit. It is given by f = 2 q , where q is the angle of 29. (d) The nicol prism is made of calcite crystal. When light
diffraction. is passed through calcite crystal, it breaks up into two
For first diffraction minimum, a sin q = 1 l rays
l (i) the ordinary ray which has its electric vector
or sin q = l/a or q=
a perpendicular to the principal section of the crystal and
(ii) the extra ordinary ray which has its electric vector
2l
\ Angular width f = 2 q = i.e. f µl parallel to the principal section. The nicol prism is made
a
in such a way that it eliminates one of the two rays by
f1 l1 f 70 total internal reflection, thus produces plane polarised
= ; \ l 2 = l1 2 = 6000 ´ = 4200 Å
f2 l 2 f1 100 light. It is generally found that the ordinary ray is
27. (b) On immersing in liquid, a wavelength l = 6000 Å must eliminated and only the extra ordinary ray is transmitted
be behaving as l' = 4200 Å to get the same decrease in through the prism. The nicol prism consists of two
angular width. Therefore, refractive index of medium calcite crystal cut at - 68° with its principal axis joined
l 6000 by a glue called Canada balsam.
m= = = 1.43.
l¢ 4200 30. (d) The clouds consists of dust particles and water droplets.
Their size is very large as compared to the wavelength
of the incident light from the sun. So there is very little
scattering of light. Hence the light which we receive
through the clouds has all the colours of light. As a
result of this, we receive almost white light. Therefore,
the cloud are generally white.
t.me/Ebooks_Encyclopedia27. t.me/Magazines4all

EBD_7156
DPP/ P 54 151

DAILY PRACTICE
PROBLEMS
PHYSICS
SOLUTIONS 54
0.101 8. (b).
0.101
1. (a). la = Å, V =
V 0.004 6.62 ´ 10-34
l=
V = 25.25 V , V = 637.5 V 2 ´ 9.1 ´ 10-31 ´ 13.6 ´ 1.6 ´ 10-19
Ea =qa × Va » 1275 eV Þ l = 3.3 × 10–10 m = 3.3 Å
h 12.27
2. (d). l = 9. (c). l = Å
mv V
c 3 ´ 108 Þ V = 40–20 = 20 Volt
Q v= = = 1.5 × 107 m/sec
20 20 12.27
h = 6.626 × 10–34 J-s, m = 1.67 × 10–27 kg Þ l= Å = 2.75 Å
20
6.626 ´10 -34 150
\ l= 10. (d). Wavelength of electrons is l = Å
1.67 ´10-27 ´ 1.5 ´ 107 V
Þ l = 2.64 × 10–14 m Now, electrons have energy of 40 KeV, therefore they
1 1 1 are accelerated through a potential difference of 40 ×
3. (a). Q l µ Þ le µ , lp µ 103 volt.
m me mp
150
mp l= = 0.061 Å
le 40 ´ 103
\ l = me
p \ Resolving limit of electron microscope = 0.061 Å
11. (d). The linear momentum of the photon
h h
4. (c). l = Þv= , h 6.63 ´ 10-34 kg - m
mv ml = = = 5.43 × 10–27
l 122 ´ 10 -9 s
6.6 ´ 10-34
v= = 7.2 × 105 m/s p
9.1 ´ 10-31 ´ 10 ´ 10-10 Q p = mv Þ v =
m
1
5. (b). l µ
m
, 5.43 ´ 10-27
Þv= = 3.25 m/s
1.67 ´ 10-27
le mp 1836
= = 150
lp me 1 12. (b). V = volt, to determine the p.d. through which it was
le2
h accelerated to achieve the given de-broglie wavelength.
6. (a). lp = Then the same p.d. will retard it to rest. Thus,
2m p ep V
150
h V= volt, V = 3765 Volt = 3.76 kV
Þ ld = 0.2 ´ 0.2
2md ed V
hc h
13. (b). lphoton = and lproton =
ld m p ep E 2me
\ = Q md = 2mp,
lp m d ed λ photon λ photon 1
2m
Þ =c Þ µ
λ electron E λ electron E
l mp ep 1
ed = ep Þ d = =
lp 2m p e p 2
7. (d). Kmax of photoelectrons doesn't depends upon intensity
of incident light.
t.me/Ebooks_Encyclopedia27. t.me/Magazines4all

152 DPP/ P 54
18. (b). P = 10 × 103 watt, n = ?, l = 300 m
1 2 hc hc 1 2
14. (d). hv – W0 = mvmax Þ l - l = 2 mvmax nhc
2 0 P=
lt
æ l0 - l ö 1 2 2hv æ l0 - l ö
Þ hc ç ll ÷ = 2 mvmax Þ vmax = ç ÷ 6.62 ´ 10-34 ´ 3 ´ 108 ´ n
è 0 ø m è ll 0 ø 104 =
300 ´ 1
When wavelength is l and velocity is v, then
300 ´ 104
n= = 1.5 × 1031
2hv æ l 0 - l ö 6.62 ´ 10-34 ´ 3 ´ 108
v= ç ÷ ....(i)
m è ll 0 ø
hc f0
19. (b). V0 = – = 3.74 – 1.07 = 2.67 V
3l el e
When wavelength is and velocity is 'v' then 20. (a). The stopping potential for curves a and b is same.
4
\ fa = fb
2hc é l 0 - ( 3l / 4 ) ù Also saturation current is proportional to intensity
v¢ = ê ú ...(ii) \ Ia < I b
m êë ( 3l / 4 ) ´ l 0 úû
21. (c). hn = hn0 + Ek
Divide equation (ii) by (i), we get 6.6 × 10-34 × 3 × 1015 = 4 × 1.6 × 10–19 + Ek
19.8 × 10-19 – 6.4 × 10-19 = Ek
v¢ él0 - ( 3l / 4 ) ùû ll 0 Ek = 13.4 × 10–19 J
= ë ´
v 3 l0 - l 1
ll0 Þ mv2max = 13.4 × 10–19
4 2

æ4ö
1/ 2 éël0 - ( 3l - 4 ) ùû 2 ´ 13.4 ´ 10-19
v¢ = v ç ÷ vmax =
è3ø l 0l m

æ4ö
1/ 2 2 ´ 13.4 ´ 10-19
i.e v¢ > v ç ÷ = = 1.73 × 106 m/s
è3ø 9 ´ 10-31
22. (b). The maximum kinetic energy is
h
15. (d). De-Broglie wavelength l = p hc 1242 eV - nm
Kmax = –f= – 2.5 eV
l 280 nm
1 = 4.4 eV – 2.5 eV = 1.9 eV
Þ lµ
p Stopping potential V is given by eV = Kmax
i.e. graph will be a rectangular hyperbola. K max 1.9
V= = eV = 1.9 V
16. (d). If the incident light be of threshold wavelength (l0), e e
then the stopping potential shall be zero. Thus 23. (a). Q 2 d sin f = nl

hc 6.6 ´ 10-34 ´ 3 ´ 108 (2d sin f)max 2d sin 90º


=
l0 = , l0 = lmax = n min 1 = 2 × 10 Å
f 4.2 ´ 1.6 ´ 10-19
l0 = 2.946 × 10–7 m = 2946 Å lmax = 20Å
\ Possible wavelengths are 5Å, 10Å and 20Å.
hn hn 0
17. (b). Relation between V0 – n., V0 = - 12400
e e 24. (c). lmin = Å = 1.24 Å
Put it in the form of y = mx – c, 10000

hn0 c 3 ´ 108
here V0 = y, n = x, =c nmax = = = 2.4×1018 Hz.
e l min 1.24 ´ 10-10
æ hö 12400
\ y = çè ÷ø x – c 25. (b) DE =
e 4500Å
h D = 2.75 eV
\m= For photoelectric effect, DE > W0 (work function).
e
t.me/Ebooks_Encyclopedia27. t.me/Magazines4all

EBD_7156
DPP/ P 54 153
26. (a) DE = W0 + E ; (Ek) = DE – W0
hv h
For maximum value of (Ek), W0 should be minimum 28. (c) Mass of moving photon m = 2 = and E = mc2
c cl
W0 for lithium = 2.3 eV
\ (Ek) = 2.75 – 2.3 = 0.45 eV 29. (c) Less work function means less energy is required for
ejecting out the electrons.
27. (c) The maximum magnitude of stopping potential will be
30. (a) de-Broglie wavelength associated with gas molecules
for metal of least work function.
\ required stopping potential is 1
varies as l µ
hv – f0 T
Vs = = 0.45 volt.
e
t.me/Ebooks_Encyclopedia27. t.me/Magazines4all

DPP/ P 55 153

DAILY PRACTICE
PROBLEMS
PHYSICS
SOLUTIONS 55
2 8. (a) K.E. = - (T.E.)
1. (a) For n = 1 , maximum number of states = 2n = 2 and
9. (a) For Lyman series
for n = 2, 3, 4 maximum number of states would be 8,
18, 32 respectively, Hence number of possible elements c é 1 1 ù 3RC
vLyman = = Rc ê - ú=
= 2 + 8 + 18 + 32 = 60 l max ëê (1)
2
(2)2 úû 4

1 æ1 1 ö For Balmer series


2. (d) = RZ 2 ç 2 - ÷
l ç1 2 2 ÷ø é 1
è c 1 ù 5RC
vBalmer = = Rc ê - ú=
For di-ionised lithium the value of Z is maximum l max ëê (2)
2
(3)2 úû 36
3. (c) Lyman series lies in the UV region.
vLyman 27
4. (c) Transition A (n = ¥ to 1): Series limit of Lyman series \ =
vBalmer 5
Transition B (n = 5 to n = 2) Third spectral line of
Balmer series 1 æ 1 1 ö
10 (c) = Rç - ÷
Transition C (n = 5 to n = 3 ) : Second spectral line of l ç n2 n 2 ÷
è 1 2 ø
Paschen series
5. (b) Let the energy in A, B and C state be EA, EB and EC For first line of Lymen series n1 = 1 and n2 = 2
then from the figure For first line of Balmer series n2 = 2 and n2 = 3

l Lymen 5
l1 So, =
l Balmer 27
l3
l2 æ h ö
11. (d) Angular momentum L = n ç ÷
è 2p ø
h h
For this case n = 2 , hence L = 2 ´ =
( EC - EB ) + ( E B - E A ) = ( EC - E A ) 2p p

hc hc hc mv 2 1 e2 e
or + = 12. (c) = Þv=
l1 l 2 l3 a0 4 pe0 a02 4 pe0 a0 m

l1l 2 13. (d) We have to find the frequency of emitted photons. For
Þ l3 = emission of photons the transition must take place from
l1 + l2
a higher energy level to a lower energy level which are
6. (a) In the revolution of electron, coulomb force provides given only in options (c) and (d).
the necessary centripetal force Frequency is given by

Ze2 mv 2 Ze2 æ 1 1ö
Þ 2
= Þ mv 2 = hn = -13.6 ç - ÷
r r r çè n 2 n 2 ÷ø
2 1

1 2 Ze2 For transition from n = 6 to n = 2,


\ K.E. = mv =
2 2r -13.6 æ 1 1 ö 2 æ 13.6 ö
n1 = ç - 2÷ = ´ç ÷
e h è6 2
2 ø 9 è h ø
+r For transition from n = 2 to n = 1,
-13.6 æ 1 1 ö 3 æ 13.6 ö
1 1 n2 = - = ´ç ÷.
7. (a) P.E. µ - and K.E. µ h çè 22 12 ÷ø 4 è h ø
r r
As r increases so K.E. decreases but P.E. increases. \ n2 > n1
t.me/Ebooks_Encyclopedia27. t.me/Magazines4all

EBD_7156
154 DPP/ P 55
14. (d) E = - Z 2 ´13.6 eV = -9 ´13.6 eV = -122.4 eV 21. (a) Here radius of electron orbit r µ 1/ m and energy
E µ m , where m is the mass of the electron.
So ionisation energy = +122.4 eV
Hence energy of hypothetical atom
15. (c) For third line of Balmer series n1 = 2, n2 = 5
a0
E0 = 2 ´ (-13.6 eV ) = -27.2 eV and radius r0 =
1 é1 1 ù n12 n22 2
\ = RZ 2 ê - ú gives Z 2 =
l 2 2
êë n1 n2 úû (n22 - n12 )lR 2prn
22. (a) Time period, Tn = (in nth state)
On putting values Z = 2 vn

13.6 Z 2 -13.6(2) 2 i.e. Tn µ


rn
But rn µ n 2 and vn µ
1
From E = - = = -54.4 eV
n 2
(1) 2 vn n

é1 Therefore, Tn µ n 3 .
1 1 ù 1 é 1 1 ù 5R
16. (b) l = R ê 2 - 2 ú Þ l = Rê
2
- ú= Given Tn = 8Tn , Hence n1 = 2n 2. Þ n1 is even
ëê n1 n2 úû 3® 2 ëê (2) (3)2 úû 36 1 1
23. (d) 2.55eV = E4 – E2.
1 é 1 1 ù 3R Therefore other photon will have energy
and = Rê - ú=
l 4® 2 êë (2) 2
(4)2 úû 16 = E2– E1 = 10.2 eV.
Energy given to H-atom excitation = E2 – E1 = 12.75 eV.
l 20 20 Consider perfectly inelastic collision for other answer.
\ 4® 2 = Þ l 4® 2 = l0
l3®2 27 27
24. (a) Balmer series lies in the visible region.
1 é 1 1 ù 4
17. (a) = Rê - ú Þ l max = » 1213 Å 25. (b), 26. (d), 27. (a)
l max êë (1) 2 2
(2) úû 3R
Since 6 different types of photons are emitted implies 4C2
i.e. highest excitation state is n = 4
1 é 1 1ù 1
= Rê - ú Þ l min = » 910 Å Since emission energies are equal, lesser and greater so initial
and l 2 ¥ R
min ëê (1) úû state
18. (a) Maximum energy is liberated for transition En ® 1 and 12420 é1 1 ù
e= = 13.6Z2 ê - ú
minimum energy for En ® En-1 l ë 4 16 û
Þ Z2 = 16 Þ Z = 4
E1
Hence - E1 = 52.224 eV .....(i) 1 1
n2 E 4®1 = 13.6 (16) - = 20.4eV
1 16
E1 E1
and - = 1.224 eV ......(ii)
n 2
( n - 1) 2 1 1
E 4®3 = 13.6 (16) - = 10.6 eV
Solving equations (i) and (ii) we get 9 16
E1 = -54.4 eV and n = 5
28. (b) Bohr postulated that electrons in stationary orbits
13.6 Z 2 around the nucleus do not radiate.
Now E1 = - = -54.4 eV . Hence Z = 2 This is the one of Bohr’s postulate. According to this
12
the moving electrons radiate only when they go from
e0 n 2 h 2 one orbit to the next lower orbit.
19. (d) Radius R = 29. (b) Rutherford confirmed the repulsive force on a-particle
pnZe2
due to nucleus varies with distance according to inverse
Ze 2 mZ 2 e4 square law and that the positive charges are
Velocity v = and energy E = - 2 2 2 concentrated at the centre and not distributed
2e0nh 8e0 n h throughout the atom.
Now, it is clear from above expressions R.v µ n 30. (b) When the atom gets appropriate energy from outside,
20. (c) At closest distance of approach then this electron rises to some higher energy level.
Kinetic energy = Potential energy Now it can return either directly to the lower energy
level or come to the lowest energy level after passing
1 ( Ze)(2e)
Þ 5 ´ 106 ´1.6 ´10-19 = ´ through other lower energy lends, hence all possible
4pe0 r transitions take place in the source and many lines are
seen in the spectrum.
For uranium Z = 92, so r = 5.3 ´ 10 -12 cm
t.me/Ebooks_Encyclopedia27. t.me/Magazines4all

DPP/ P 56 155

DAILY PRACTICE
PROBLEMS
PHYSICS
SOLUTIONS 56
1. (c). The energy produced per second is 9. (b). By the forumula N = N0e–lt
0.631×t
106 N 1 0.6931
= 1000 × 103 J= eV 6.25 × 1024 eV Given N = and l = Þ 20 = e 3.8
1.6 ´ 10-19 0 20 3.8
The number of fissions should be, thus Taking log of both sides
0.6931 ×t
6.25 ´ 1024 or log 20 = log10 e
number = = 3.125 × 1016 3.8
200 ´ 106
0.6931 ×t ´ 0.4343
2 or 1.3010 = Þ t = 16.5 days
2. (b). No. of atoms in 2kg 92 U235 = × NA 3.8
235
10. (b). A = 238 - 4 = 234, Z = 92 - 2 = 90
2 11. (a). Dm = 0.03 a.m.u., A = 4
= × (6.02 × 1026) = 5.12 × 1024
235 Dm ´ 931
Þ DE =
5.12 ´ 10 24 A
Fission rate = = 1.975 × 1018 per sec
30 ´ 24 ´ 60 ´ 60 0.03 ´ 931
Usable energy per fission = 185 MeV Þ DE = = 7 MeV
4
\ Power output 12. (a). Q DE = Dm × 931 MeV
= (185 × 106)(1.975 × 1018)(1.6 × 10–19) watt
= 58.4 × 106 watt = 58.46 MW DE 2.23
Þ Dm = = = 0.0024 a.m.u.
931 931
6 ´1023
3. (d). \ 6 gm of 6C12 contains atoms = and each
2 R Al (27)1/3 3 6
13. (a). R = = =
atom of 6C12 contains electron, protons and neutrons Te (125)1/3 5 10
= 6, 6, 6
14. (d). R = R0 A1/3 = 1.2 × 10-15 × (64)1/3
\ No. of electron, protons and neutron in 6 gm of
12 23 23 23 = 1.2 × 10–15 × 4 = 4.8 fm
6C = 18 × 10 , 18 × 10 , 18 × 10 15. (b). Number of protrons in nucleus = atomic number = 11
4. (c). Use r = Mass/volume
Number of electrons = number of protons = 11.
1.66 ´ 10-27 ´ 16 Number of neutrons = mass number A – atomic number Z
= = 2.35 × 1017 kg m-3 N = 24 – 11 = 13
(4 / 3)p(3 ´ 10 -15 )
16. (d). Q equivalent mass of each photon = 1/2000 amu
5. (a). Mass defect Dm = M (Ra 226) – M(Rn 222) – M (a) Q 1 amu = 931 MeV
= 226.0256 – 222.0175 – 4.00026 = 0.0053 u.
6. (a). E = mc2 = (1.66 × 10–27) (3 × 108)2 J 931
\ Energy of each photon = = 0.465 MeV
= 1.49 × 10–10 J 2000
17. (c). Deuterium, the isotope of hydrogen consits of one proton
1.49 ´ 10-10 and neutron. Therefore mass of nuclear constituents of
= MeV = 931.49 MeV
1.6 ´ 10-13 deuterium = mass of proton + mass of neutron
7. (b). E = mc2 = 1.00759 + 1.00898 = 2.01657 amu.
= (9.1 × 10–31) (3 × 108)2 J = 0.51 MeV mass of nucleus of deuterium = 2.01470 amu.
8. (c). DE = D mc2 Mass defect = 2.01657 – 2.01470 = 0.00187 amu.
Binding energy = DE = 0.00187 × 931 MeV = 1.741 MeV.
0.5
Dm = kg = 0.005 kg
100
c = 3 × 108 m/s DE Dm ´ 931
18. (a). E = = MeV
DE = 0.005 × (3 × 108)2 A A
DE = 4.5 × 1014 J or watt-sec Dm = (3mp + 4mn) – mass of Li7
Dm = (3 × 1.00759 + 4 × 1.00898) – 7.01653
4.5 ´ 1014 Dm = 0.04216 a.m.u.
DE = = 1.25 × 10111 watt hour
60 ´ 60
0.04216 ´ 931 39.25
DE = 1.25 × 108 kWH DE = = = 5.6 MeV
7 7
t.me/Ebooks_Encyclopedia27. t.me/Magazines4all

EBD_7156
156 DPP/ P 56
19. (d). The sun radiates energy in all directions in a sphere. At 25. (c) The number of 12C atoms in 1g of carbon,
a distance R, the energy received per unit area per
second is 1.4 KJ (given). Therefore the energy released NA 6.022 ´ 1023
N= ´mÞ N = ´1
in area 4pR2 per sec is = 1400 × 4pR2 Joule the energy 12 12
released per day = 1400 × 4pR2 × 86400J = 5.02 × 1022 atoms.
where R = 1.5 × 1011 m, Thus The ratio of 14C/12C atoms = 1.3 × 10–12 (Given)
DE = 1400×4 × 3.14 × (1.5 × 1011)2 × 86400
\ Number of 14C atoms = 5.02 × 1022 × 1.3 × 10–
The equivalent mass is 12
Dm = DE/c2
= 6.5 × 1010
11 2
1400 ´ 4 ´ 3.14 ´ (1.5 ´ 10 ) ´ 86400
Dm = 0.693
9 ´ 1016 \ Rate of decay R0 = lN0 = T N0
1/ 2
Dm = 3.8 × 1014 kg
0.693 ´ 6.5 ´ 1010
B.E. \ R0 =
20. (b) 5730 ´ 365 ´ 24 ´ 3600
A
+ = 0.25 Bq = 0.25 (decays/s)
Fusion Fission
+ 26. (c) For 10g sample, number of decays = 0.5 per second.
i.e. R = 0.05 and R0 = 0.25 for each gram of 14C
A R 1 ln ( R0 / R ) ln ( R0 / R )
t /T 10 / 5
= e -lt Þ t = =
æ1ö æ1ö R0 l 1 (0.693 / T1/ 2 )
21. (c) Nt = N0 ç ÷ = 50000 ç ÷ = 12500
è2ø è2ø
5730 years æ 0.25 ö
Þt= ´ ln ç = 13310 years
22. (d). Power received from the reactor, 0.693 è 0.05 ÷ø
P = 1000 KW = 1000 × 1000 W = 106 J/s 27. (d) If there are no other radioactive ingredients, the sample
is very recent. But the error of measurement must be
106 high unless the statistical error itself is large. In any
P= eV/sec.
1.6 ´ 10-19 case, for an old sample, the activity will not be higher
P = 6.25 × 1018 MeV/sec than that of a recent one.
\ number of atoms disintegrated per sec 28. (d) The penetrating power is maximum in case of gamma
rays because gamma rays are an electromagnetic
6.25 ´ 1018 radiation of very small wavelength.
= = 3.125 × 1016 29. (b) b-particles, being emitted with very high velocity (up
200
to 0.99 c). So, according to Einstein's theory of
Energy released per hour = 106 × 60 × 60 Joule relatively, the mass of a b-particle is much higher
DE compared to is its rest mass (m0). The velocity of
Mass decay per hour = Dm = electrons obtained by other means is very small
c2 compared to c (Velocity of light). So its mass remains
nearly m0. But b-particle and electron both are similar
106 ´ 60 ´ 60 particles.
Þ Dm =
(3 ´ 108 )2 30. (b) Electron capture occurs more often than positron
emission in heavy elements. This is because if positron
Þ Dm = 4 × 10–8 kg emission is energetically allowed, electron capture is
23. (a) necessarily allowed, but the reverse is not true i.e. when
24. (a) In fusion two lighter nuclei combines, it is not the electron caputre is energetically allowed, positron
radioactive decay. emission is not necessarily allowed.
t.me/Ebooks_Encyclopedia27. t.me/Magazines4all

DPP/ P 57 157

DAILY PRACTICE
PROBLEMS
PHYSICS
SOLUTIONS 57
1. (a) With temperature rise conductivity of semiconductors 19. (b) In forward biasing the diffusion current increases and
increases. drift current remains constant so not current is due to
2. (b) the diffusion.
3. (b) In insulators, the forbidden energy gap is very large, in In reverse biasing diffusion becomes more difficult so
case of semiconductor it is moderate and in conductors net current (very small) is due to the drift.
the energy gap is zero. 20. (a) In a triclinic crystal a ¹ b ¹ c and a ¹ b ¹ g ¹ 90º
4. (c) In intrinsic semiconductors, the creation or liberation 21. (a)
of one free electron by the thermal energy creates one 22. (a) In figure (1), (2) and (3). P-crystals are more positive
hole. Thus in intrinsic semiconductors ne = nh as compared to N-crystals.
23. (a) Wood is non-crystalline and others are crystalleine.
5. (b) Both P-type and N-type semiconductors are neutral 24. (a) Resistance of conductors (Cu) decreases with decrease
because neutral atoms are added during doping. in temperature while that of semi-conductors (Ge)
6. (d) Conductor has positive temperature coefficient of increases with decrease in temperature.
resistance but semiconductor has negative temperature
coefficient of resistance. DV p (180 - 120)
25. (b) rp = = = 1.2 ´ 104 ohm
7. (d) DI p -3
(15 - 10) ´10
8. (c) At zero Kelvin, there is no thermal agitation and
therefore no electrons from valence band are able to
DI p (15 - 7 ) ´ 10 -3
shift to conduction band. 26. (a) gm = = = 5.33 ´ 10 -3 ohm -1
9. (c) Antimony is a fifth group impurity and is therefore a DVg ( - 2 .0 ) - ( - 3 .5 )
donor of electrons.
10. (d) At 0K temperature semiconductor behaves as an 27. (a) m = rp ´ g m = (1.2 ´ 10 4 ) ´ ( 5.33 ´ 10 -3 ) = 64
insulator, because at very low temperature electrons
cannot jump from the valence band to conduction band.
11. (b) Formation of energy bands in solids are due to Pauli’s 28. (a) According to law of mass action, ni2 = ne nh . In
exclusion principle. intrinsic semiconductors ni = ne = nh and for P-type
12. () semiconductor ne would be less than ni, since nh is
13. (a) In conductors valence band and conduction band may necessarily more than ni.
overlaps. 29. (d) Resistivity of semiconductors decreases with
14. (b) With rise in temperature, conductivity of semiconductor temperature. The atoms of a semiconductor vibrate with
increases while resistance decreases. larger amplitudes at higher temperatures there by
i increasing its conductivity not resistivity.
15. (a) Because vd = 30. (c) We cannot measure the potential barrier of a PN-
(ne )eA
junction by connecting a sensitive voltmeter across its
16. (b) In reverse biasing, width of depletion layer increases. terminals because in the depletion region, there are no
17. (b) Because in case (1) N is connected with N. This is not free electrons and holes and in the absence of forward
a series combination of transistor. biasing, PN- junction offers infinite resistance.
18. (d) Resistance in forward biasing R fr » 10W and
resistance in reverse biasing
R fr 1
RRw » 105 W Þ =
RRw 104
t.me/Ebooks_Encyclopedia27. t.me/Magazines4all

EBD_7156
158 DPP/ P 58
DAILY PRACTICE
PROBLEMS
PHYSICS
SOLUTIONS 58
12. (a)
1. (b) N N
Forward reverse
biased P biased

2. (d) a is the ratio of collector current and emitter current


C = A.B = A + B = A + B (De morgan’s theorem)
while b is the ratio of collector current and base current.
Hence output C is equivalent to OR gate.
10 -19
Ne 10 ´1.6 ´10
3. (a) Emitter current Ie = = = 1.6 mA
t 10-6

2
Base current Ib =
100
× 1.6 = 0.032 mA C = AB. AB = A.B + A.B = AB + AB = AB
In this case output C is equivalent to AND gate.
But, Ie = Ic + Ib
\ Ic = Ie – Ib = 1.6 – 0.032 = 1.568 mA 13. (c) For ‘XNOR’ gate Y = A B + AB
Ic 1.568 Ic 1.568 i.e. 0.0 + 0.0 = 1.1 + 0.0 = 1 + 0 = 1
\ a= = = 0.98 and b = I = = 49
Ie 1.6 0.032
b 0.1 + 0.1 = 1.0 + 0.1 = 0 + 0 = 0
4. (d)
1.0 + 1.0 = 0.1 + 1.0 = 0 + 0 = 0
90
5. (d) iC = ´ iE Þ 10 = 0.9 ´ iE Þ I E = 11mA 1.1 + 1.1 = 0.0 + 1.1 = 0 + 1 = 1
100
14. (d) The output D for the given combination
Also i E = i B + i C Þ i B = 11 - 10 = 1mA D = ( A + B).C = ( A + B) + C
6. (d) b = 50, R = 1000W, Vi = 0.01V If A = B = C = 0 then
i V 0.01 D = (0 + 0) + 0 = 0 + 0 = 1 + 1 = 1
b = c and ib = i = = 10-5 A
ib Ri 103 If A = B = 1, C = 0 then

Hence ic = 50 ´ 10-5 A = 500 mA D = (1 + 1) + 0 = 1 + 0 = 0 + 1 = 1

0.8 15. (a) The Boolean expression for ‘NOR’ gate is Y = A + B


7. (c) a = 0.8 Þ b = =4
(1 - 0.8) i.e. if A = B = 0 (Low), Y = 0 + 0 = 0 = 1 (High)
16. (d) The Boolean expression for ‘AND’ gate is R = P.Q
Di
Also b = c Þ Dic = b ´ Dib = 4 ´ 6 = 24mA Þ 1.1 = 1,1.0 = 0, 0.1 = 0, 0.0 = 0
Dib
17. (a) The given Boolean expression can be written as
8. (b) ie = ib + ic Þ ic = ie - ib
Y = ( A + B).( A.B) = ( A.B).( A + B) = ( A A).B + A( B.B)
9. (b)
= A.B + A B = A B
10. (d) For CE configuration voltage gain = b´ RL / Ri

Power gain A B Y
Power gain = b2 ´ RL / Ri Þ =b 0 0 1
Voltage gain
1 0 0
11. (b) For ‘OR’ gate X = A + B 0 1 0
i.e. 0 + 0 = 0, 0 + 1 = 1,1 + 0 = 1, 1 + 1 = 1 1 1 0
t.me/Ebooks_Encyclopedia27. t.me/Magazines4all

DPP/ P 58 159
18. (c) The Boolean expression for the given combination is But I e = I c + I b = 0.96 I e + I b
output Y = ( A + B ).C
Þ I b = 0.04 I e
The truth table is
A B C Y = (A + B).C I c 0.96 I e
\ Current gain, b = = = 24
0 0 0 0 I b 0.04I e
1 0 0 0
23. (a)
0 1 0 0
24. (b)
0 0 1 0
25. (a)
1 1 0 0
26. (b) The probability that a state with energy E is occupied
0 1 1 1
is given by
1 0 1 1
1 1 1 1 1
P(E) = (E -E ) / kT , where EF is the Fermi energy,,
Hence A = 1, B = 0, C = 1 e F +1
19. (d) For ‘NAND’ gate C = A.B T is the temperature on the Kelvin scale, and k is the
Boltzmann constant. If energies are measured from the
i.e. 0.0 = 0 = 1, 0.1 = 0 = 1 top of the valence band, then the energy associated with
a state at the bottom of the conduction band is E = 1.11
1.0 = 0 = 1,1.1 = 1 = 0
eV. Furthermore, kT = (8.62 × 10–5 eV/K) (300K) =
0.02586 eV. For pure silicon, EF = 0.555 eV and
20. (d)
(E – EF)/kT = (0.555eV) / (0.02586eV) = 21.46. Thus,
1
P(E) = = 4.79 ´ 10 -10
21.46
e +1
For the doped semi-conductor,
(E – EF) / kT = (0.11 eV) / (0.02586 eV) = 4.254
1
Hence option (d) is true. and P(E) = 4.254 = 1.40 ´ 10 -2 .
e +1
27. (a) The energy of the donor state, relative to the top of the
21. (c)
valence bond, is 1.11 eV – 0.15 eV = 0.96 eV. The
Fermi energy is 1.11 eV – 0.11 eV = 1.00 eV. Hence,
(E - E F ) / kT = (0.96eV - 1.00eV)
/(0.02586eV) = -1.547

1
and P(E) = = 0.824
-1.547
e +1
True Table 28. (d) In diode the output is in same phase with the input
X Y therefore it cannot be used to built NOT gate.
X Y P = X + Y Q = X.Y R = P+Q
29. (a) This is Boolean expression for ‘OR’ gate.
0 1 1 0 1 1 0
1 1 0 0 1 1 0
1 0 0 1 0 0 1
0 0 1 1 1 1 0 30. (d) Statement -1 is true but statement -2 is false.
Hence X = 1, Y = 0 gives output R = 1
Ic
22. (c) I = 0.96
e

Þ I c = 0.96 I e If A = 1, B = 0, C = 1 then Y = 0
t.me/Ebooks_Encyclopedia27. t.me/Magazines4all

EBD_7156
160 DPP/ P 59
DAILY PRACTICE
PROBLEMS
PHYSICS
SOLUTIONS 59
20. (a) The critical frequency of a sky wave for relection from
1. (a) By using fc ; 9 ( N max )1/ 2 Þ f c = 2.84MHz
a layer of atmosphere is given by fc = 9( N max )11/ 2
2. (d) Carrier frequency > audio frequency
3. (a) A maximum frequency deviation of 75 kHz is permitted Þ 10 ´106 = 9( N max )11/ 2
for commercial FM broadcast stations in the 88 to 108
MHz VHF band. 2
æ 10 ´106 ö 12 -3
4. (c) Carrier + signal ® modulation. Þ N max = ç ÷ ; 1.2 ´10 m
ç 9 ÷
5. (c) 270W è ø
6. (a) m1 > m2
21. (b) Core of acceptance angle q = sin -1 n12 - n22
fc 60
7. (c) MUF = = = 175MHz 22 (a) Remote sensing is the technique to collect information
cos q cos 70o
about an object in respect of its size, colour, nature,
8. (d) coordinated waves of a particular wavelength location, temperature etc without physically touching
9. (d) Surgery needs sharply focused beam of light and laser it. There are some areas or location which are
can be sharply focused. inaccessible. So to explore sensing is used. Remote
10. (d) Laser beams are perfectly parallel. So that they are very sensing is done through a satellite.
narrow and can travel a long distance without 23. (b)
spreading. This is the feature of laser while they are 24. (a) Laser beams are perfectly parallel. They are
monochromatic and coherent, these are characteristics monochromatic and coherent. These are characteristics
only. only.
11. (b) The formula for modulating index is given by
25. (d) f c = 9 N m = 9 ´ 9 ´ 1010
d Frequency var iation 10 ´ 103
mf = = = =5 = 2.7 ´ 106 Hz = 2.7 MHz
vm Modulating frequency 2 ´ 103

12. (b) fa < f f ( 250 ´ 103 )2


D2
26. (b) f = f c 1 + = 2.7 ´ 10 6 ´ 1 +
13. (c) An antenna is a metallic structure used to radiate or 4h 2 4 ´ (150 ´ 103 )
2
receive EM waves.
14. (a) Varying the local oscillator frequency = 3.17 × 106Hz
15. (b) In the channel or in the transmission line 27. (c) f = fc = sec fi
Frequency deviation 50
16. (a) Carrier swing = = = 7.143
Modulating frequency 7 f 3.17×106
Secfi = = = 1.174
f c 2.7 ´106
17. (c) In optical fibre, light travels inside it, due to total
internal reflection.
fi = sec –1 (1.174) = 31.6° .
18. (b) The process of changing the frequency of a carrier wave
(modulated wave) in accordance with the audio 28. (d) TV signals (frequency greater than 30 MHz) cannot
frequency signal (modulating wave) is known as be propagated through sky wave propagation.
frequency modulation (FM). Above critical frequency, an electromagnetic wave
19. (d) Following are the problems which are faced while penetrates the ionosphere and is not reflected by it.
transmitting audio signals directly. 29. (d) The electromagnetic waves of shorter wavelength do
(i) These signals are relatively of short range. not suffer much diffraction from the obstacles of earth’s
(ii) If every body started transmitting these low frequency atmosphere so they can travel long distance.
signals directly, mutual interference will render all of Also, shorter the wavelength, shorter is the velocity of
them ineffective. wave propagation.
(iii) Size of antenna required for their efficient radiation 30. (b) A dish antenna is a directional antenna because it can
would be larger i.e. about 75 km. transmit or signals in all direction.
t.me/Ebooks_Encyclopedia27. t.me/Magazines4all

DPP/ P 60 161

DAILY PRACTICE
PROBLEMS
PHYSICS
SOLUTIONS 60
1. (b) Voltmeter measures voltage across two points so it is 15. (c) In reverse-bias mode, a reverse current flows.
connected in parallel and ammeter measures current Therefore, (c) represents the form.
so it is connected in series. 16. (c) Larger the length, lesser will be the potential gradient,
P R so more balancing length will be required.
2. (a) = so by changing the gap resistance of copper 17. (a) LED is a p-n junction diode which always operates on
Q S forward bias.
strip gets cancelled. 18. (b) Emitter current is the sum of base and collector current
3. (b) Potential gradient = Potential drop/unit length by Kirchhoff's 1st law.
rl Irl
V = IR = I 19. (b) V = IR = \ potential gradient = k
A A
when A is decreased, k will increase.
V Ir
\ = ] 20. (d) Positive terminal is at lower potential
l A (0V) and negative terminal is at higher potential + 5V.
4. (d) Refractive index is the property of the material, hence 21. (d)
it does not depend on angle of the prism.
V 30.0
5. (a) A meter-bridge is a device which is based on the 22. (d) V = 30.0, I = 0.020 A, R = = = 1.50 kW
principle of Wheatstone bridge. I 0.020
6. (a) A potentiometer is device which is used to compare V DR DV DI
e.m.f.'s of two cells as well as to determine the internal Error : As R = \ = +
I R V I
resistance of a cell. It is based on the principle that when
a current flows through a wire of uniform thickness and æ DV DI ö
Þ DR = R çè + ÷
material, potential difference between its two points is V Iø
directly proportional to the length of the wire between
the two points. æ 0.1 0.001ö
= 1.50 × 103 çè + ÷ = 0.080 kW
7. (c) When a p-n junction diode is connected in reverse 30.0 0.020 ø
biased mode, a reverse current flows. 23. (a) u = – 0.30 m, v = – 0.60m
8. (b) A Zener diode is a heavily doped p-n junction diode By mirror formula,
which operates on reverse bias beyond breakdown
voltage. 1 1 1
+ =
9. (c) In a transfer characteristics Vi is plotted along x-axis v u f
and V0 along y-axis. 1 -1 1 -3.0
Þ = - Þ f= Þ f = 0.20m
Χ IC f 0.30 0.60 0.60
10. (a) Current gain in CE configuration is b = .
ΧI B 1 1 1 -df -dv -du
Þ = + Þ 2 = 2 - 2
Ic >> IB, hence it is maximum. f v u f v u
rl V Ir é 0.01
11. (c) V = IR = I \ <k< , 0.01 ù
A l A Þ df = (0.20)2 ê 2
+ ú
increase in I, will increase k, so it will decrease ëê (0.60) (0.30)2 ûú
sensitivity. Þ df = 0.0055 » 0.01m
12. (c) Þ Focal length f = (0.20 ± 0.01) m
13. (c) According to the figure the voltmeter and the resistor 24. (d) As shown in the figure.
are connected in parallel.
14. (b) Here I = 4A P l l
= , Pµ
Q 100 - l 100 - l
c
æ 30 ´ p ö pc P(unknown)Q
q = 30° = ç ÷ =
è 180 ø 6

I 4 4´6´7 2´6´7
Now, k = = = =
q æpö 22 11 G
ç ÷
è6ø
B
84
= = 7.6 A / rad l 100l
11
t.me/Ebooks_Encyclopedia27. t.me/Magazines4all

EBD_7156
162 DPP/ P 60
27. (c) because balance point depends upon the value of
DP Dl D (100 - l)
= + unknown and known resistance only.
P l 100 - l 28. (c) Potentiometer is used to measure the potential
Dl Dl difference between the two points of a wire.
= + 29. (a) Let j be the current density.
l 100 - l
I
0.1 0.1 DP Then j ´ 2pr 2 = I Þ j =
= + Þ ´ 100 = 0.42% 2 pr 2
40.0 60.0 P
25. (b) As shown in the figure, when the object (O) is placed rI
\ E = rj =
between F and C, the image (I) is formed beyond C. It 2pr 2
is in this condition that movement towards left
a r uur a
rI
//// '
Now, DVBC =- ò E. dr = - ò 2
dr
a + b 2 pr
/ //

Movement towards left a+b


////

F
//////////////

a
| |||
rI é 1 ù rI rI
|||
|||

=- - = -
||||

I C O
2p ëê r ûú a + b 2pa 2p (a + b)
///

//
/// On applying superposition as mentioned we get
' rI rI
when the student shifts his eye towards left, the image DVBC = 2 ´ DVBC = -
pa p(a + b)
appears to the right of the object pin.
26. (c) For a spherical mirror, the graph plotted between (1/u) rI
30. (c) E=
and (1/v) will be a straight line with a negative slope 2 pr 2
of (– 1) and position intercept (1/f) on the (1/v) axis
1 1 1
=- +
v u f

1/ f
tan q = -1
q
(1/u)
O 1/ f

You might also like